Dentin NBDE Part 2 [2]


261 40 14MB

English Pages [1513] Year 2016

Report DMCA / Copyright

DOWNLOAD PDF FILE

Table of contents :
CHAPTER 1: ORAL PATHOLOGY
METABOLIC & GENETIC DISEASES
INFLAMMATORY JAW LESIONS
CONNECTIVE TISSUE LESIONS
BENIGN EPITHELIAL TUMORS
VERRUCAL PAPILLARY LESIONS
NEOPLASMS
ODONTOGENIC ABNORMALITIES
WHITE LESIONS
BLOOD DISEASES
NEUROLOGIC & MUSCLE DISORDERS
NON-ODONTOGENIC CYSTS
ODONTOGENIC CYSTS
NON-ODONTOGENIC TUMORS
ODONTOGENIC TUMORS
PIGMENTED LESIONS
RED-BLUE LESIONS
PSEUDOCYSTS (NO EPITHELIAL LINING)
SALIVARY GLAND TUMORS
MALIGNANT SALIVARY GLAND TUMORS
ULCERATIVE CONDITIONS
VESICULO-BULLOUS DISEASES
RADIOGRAPHIC PATHOLOGY
CHAPTER 2: RADIOLOGY
OSTEONECROSIS
DIGITAL RADIOGRAPHY
SECONDARY RADITION, COLLIMATION, FILTRATION
PANORAMIC & CONE BEAM
CEPHALOMETRICS & BWX
SUBMENTAL-VERTICAL
WATER’S VIEW
TOWNE’S VIEW
CHEMICALS, SOLUTION, & DEVELOPING ERRORS
RADIOGRAPHIC TECHNIQUES & ERRORS
CHAPTER 3: DENTAL EMERGENCY PROTOCOL & MEDICALLY COMPROMISED CONSIDERATIONS
DENTAL EMERGENCY PROTOCOL
MEDICALLY COMPROMISED CONSIDERATIONS
CHAPTER 4: DENTAL PHARMACOLOGY
ANTIBIOTIC PRE-MEDICATION GUIDELINES
CENTRAL NERVOUS SYSTEM
ADRENERGICS (SYMPATHETICS)
ADRENERGIC AGONISTS
ANTI-ADRENERGICS (SYMPATHOLYTICS)
CHOLINERGICS
ANTI-CHOLINERGICS
NICOTINIC RECEPTOR ANTAGONISTS
DRUG ADMINISTRATION ROUTES & METABOLISM
DEA DRUG SCHEDULE
ANESTHETICS
NITROUS OXIDE
ANTI-ANXIETY AGENTS
CARDIOVASCULAR AGENTS
ANTI-DEPRESSANTS
ANTI-PSYCHOTICS
ANTI-HISTAMINES
ANTI-CONVULSANTS
ANTIBIOTICS
ANTI-FUNGALS & PROTOZOALS
ANTI-VIRALS
NSAIDS
ACETAMINOPHEN & OPIODS
ANTI-NEOPLASTIC (CANCER) DRUGS
HYPOGLYCEMICS
CHAPTER 5: PROSTHODONTICS
REMOVABLE PARTIAL DENTURES
MANDIBULAR MAJOR CONNECTORS
MAXILLARY MAJOR CONNECTORS
INDIRECT RETAINERS
DIRECT RETAINERS
RPD STRESS BREAKERS
SURVEYING RPD ABUTMENTS
COMPLETE DENTURES
MANDIBULAR COMPLETE DENTURES
MAXILLARY COMPLETE DENTURES
ORAL PATHOLOGY & ILL-FITTING COMPLETE DENTURES
IMMEDIATE DENTURES
OVERDENTURES & OCCLUSION
DENTURE TEETH SELECTION
PHONETICS
FIXED PARTIAL DENTURES (CROWN & BRIDGE)
PORCELAIN SHADE SELECTION
PONTIC DESIGN
CANTILEVER BRIDGES & PIER ABUTMENTS
MARYLAND BRIDGE
BRIDGES & BRIDGE ABUTMENTS
POST & CORES
PORCELAIN VENEERS
IMPRESSION MATERIALS & CEMENT
DENTAL IMPLANTS & OVERDENTURES
CHAPTER 6: OPERATIVE DENTISTRY
DENTAL CEMENTS
PREPARATION BASES & CAVITY LINERS
CARIES
GOLD
COMPOSITE
AMALGAM
DENTAL BIOMATERIALS
RESTORATIVE HAND INSTRUMENTS & BURS
CHAPTER 7: PERIODONTICS
PERIODONTIUM & FIBERS
FLAPS, GRAFTS, & SURGERY
GINGIVITIS & GINGIVAL CONDITIONS
PERIODONTITIS
PLAQUE & CALCULUS
SCALING & ROOT PLANING (SRP)
HYGIENE INSTRUMENTATION & POWER DRIVEN SCALERS
OCCLUSAL TRAUMA
ABSCESSES
ORAL HYGIENE INSTRUCTION
CHAPTER 8: ORAL SURGERY
NERVE ANATOMY
ARTERIES & GLANDS
MUSCLES OF MASTICATION & TMJ
FRACTURES
GENERAL ANESTHESIA
LOCAL ANESTHESIA & COMPLICATIONS
NITROUS OXIDE
EXODONTIA
GRAFTS
BIOPSY
ORAL SURGERY CONDITIONS & TREATMENT CONSIDERATIONS
CPR GUIDELINES
HORMONES, CALCIUM, BLOOD GLUCOSE
CHAPTER 9: ENDODONTICS
TOOTH FRACTURES
FLAPS & SLOB RULE
PULPOTOMY & APEXIFICATION
DIRECT & INDIRECT PULP CAPPING
CANAL ACCESS & DEBRIDEMENT
OBTURATION
IRRIGANTS & CHELATING AGENTS
ZOE, MTA, APICOECTOMY
PERIRADICULAR SURGERY & CURETTAGE
ENDODONTIC INSTRUMENTATION
MANAGING AVULSION INJURIES
EXTERNAL & INTERNAL RESORPTION
PULP, DENTIN, CEMENTUM
MANDIBULAR & MAXILLARY ROOT ANATOMY
RCT ADJUNCTS, INDICATIONS & CONTRAINDICATIONS
PERIAPICAL & PERIODONTAL ABSCESSES
PROBING LESIONS
REVERSIBLE & IRREVERSIBLE PULPITIES, NECROSIS
RESTORING TEETH AFTER RCT
CHAPTER 10: PEDIATRIC DENTISTRY
ANTI-ANXIETY MEDICATIONS
PULP TREATMENT
PEDIATRIC DISEASES & CONDITIONS
TOOTH ABNORMALITIES
TOOTH DEVELOPMENT & ERUPTION
PRIMARY DENTITION
FLUORIDE
SEALANTS
TOOTH TRAUMA
BEHAVIOR MANAGEMENT
CHAPTER 11: ORTHODONTICS
OVERBITE & OVERJET
MALOCCLUSION
ANGLE MALOCCLUSION CLASSIFICATION
CROSSBITES
CEPHALOMETRICS
MIXED DENTITION & ANALYSIS
PRIMATE SPACES & SPACE MAINTENANCE
ORTHODONTIC APPLIANCES
CHAPTER 12: BEHAVIORAL SCIENCE, PUBLIC HEALTH, OSHA & INFECTION CONTROL
BEHAVORIAL SCIENCE
PUBLIC HEALTH AND RESEARCH METHODS
ORAL HEALTH INDICES
OSHA AND INFECTION CONTROL
DENTIN “1,001+ HIGH-SPEED DRILL NBDE II PREVIEW
Recommend Papers

Dentin NBDE Part 2 [2]

  • 0 0 0
  • Like this paper and download? You can publish your own PDF file online for free in a few minutes! Sign Up
File loading please wait...
Citation preview

COPYRIGHT ©2016. DENTIN, LLC. ALL RIGHTS RESERVED. No part of this book may be produced, stored in a retrieval system, or transmitted in any form, or by any means, electronic, mechanical, photocopying, recording or otherwise, without the prior written permission from the publisher. Produced by DENTIN, LLC dentin.co Printed in the United States

DENTINTM SUPERIOR DENTAL INFORMATION

TABLE OF CONTENTS CHAPTER 1 ORAL PATHOLOGY METABOLIC & GENETIC DISEASES INFLAMMATORY JAW LESIONS CONNECTIVE TISSUE LESIONS BENIGN EPITHELIAL TUMORS VERRUCAL PAPILLARY LESIONS NEOPLASMS ODONTOGENIC ABNORMALITIES WHITE LESIONS BLOOD DISEASES NEUROLOGIC & MUSCLE DISORDERS NON-ODONTOGENIC CYSTS ODONTOGENIC CYSTS NON-ODONTOGENIC TUMORS ODONTOGENIC TUMORS PIGMENTED LESIONS RED-BLUE LESIONS PSEUDOCYSTS (NO EPITHELIAL LINING)

SALIVARY GLAND TUMORS MALIGNANT SALIVARY GLAND TUMORS ULCERATIVE CONDITIONS VESICULO-BULLOUS DISEASES RADIOGRAPHIC PATHOLOGY

CHAPTER 2 RADIOLOGY OSTEONECROSIS DIGITAL RADIOGRAPHY SECONDARY RADITION, COLLIMATION, FILTRATION PANORAMIC & CONE BEAM CEPHALOMETRICS & BWX SUBMENTAL-VERTICAL WATER’S VIEW TOWNE’S VIEW CHEMICALS, SOLUTION, & DEVELOPING ERRORS RADIOGRAPHIC TECHNIQUES & ERRORS

CHAPTER 3 DENTAL EMERGENCY PROTOCOL &

MEDICALLY COMPROMISED CONSIDERATIONS DENTAL EMERGENCY PROTOCOL MEDICALLY COMPROMISED CONSIDERATIONS

CHAPTER 4 DENTAL PHARMACOLOGY ANTIBIOTIC PRE-MEDICATION GUIDELINES CENTRAL NERVOUS SYSTEM ADRENERGICS (SYMPATHETICS) ADRENERGIC AGONISTS ANTI-ADRENERGICS (SYMPATHOLYTICS) CHOLINERGICS ANTI-CHOLINERGICS NICOTINIC RECEPTOR ANTAGONISTS DRUG ADMINISTRATION ROUTES & METABOLISM DEA DRUG SCHEDULE ANESTHETICS NITROUS OXIDE ANTI-ANXIETY AGENTS

CARDIOVASCULAR AGENTS ANTI-DEPRESSANTS ANTI-PSYCHOTICS ANTI-HISTAMINES ANTI-CONVULSANTS ANTIBIOTICS ANTI-FUNGALS & PROTOZOALS ANTI-VIRALS NSAIDS ACETAMINOPHEN & OPIODS ANTI-NEOPLASTIC (CANCER) DRUGS HYPOGLYCEMICS

CHAPTER 5 PROSTHODONTICS REMOVABLE PARTIAL DENTURES MANDIBULAR MAJOR CONNECTORS MAXILLARY MAJOR CONNECTORS INDIRECT RETAINERS DIRECT RETAINERS RPD STRESS BREAKERS SURVEYING RPD ABUTMENTS COMPLETE DENTURES MANDIBULAR COMPLETE DENTURES

MAXILLARY COMPLETE DENTURES ORAL PATHOLOGY & ILL-FITTING COMPLETE DENTURES IMMEDIATE DENTURES OVERDENTURES & OCCLUSION DENTURE TEETH SELECTION PHONETICS FIXED PARTIAL DENTURES (CROWN & BRIDGE) PORCELAIN SHADE SELECTION PONTIC DESIGN CANTILEVER BRIDGES & PIER ABUTMENTS MARYLAND BRIDGE BRIDGES & BRIDGE ABUTMENTS POST & CORES PORCELAIN VENEERS IMPRESSION MATERIALS & CEMENT DENTAL IMPLANTS & OVERDENTURES

CHAPTER 6 OPERATIVE DENTISTRY DENTAL CEMENTS PREPARATION BASES & CAVITY LINERS CARIES GOLD

COMPOSITE AMALGAM DENTAL BIOMATERIALS RESTORATIVE HAND INSTRUMENTS & BURS

CHAPTER 7 PERIODONTICS PERIODONTIUM & FIBERS FLAPS, GRAFTS, & SURGERY GINGIVITIS & GINGIVAL CONDITIONS PERIODONTITIS PLAQUE & CALCULUS SCALING & ROOT PLANING (SRP) HYGIENE INSTRUMENTATION & POWER DRIVEN SCALERS OCCLUSAL TRAUMA ABSCESSES ORAL HYGIENE INSTRUCTION

CHAPTER 8 ORAL SURGERY

NERVE ANATOMY ARTERIES & GLANDS MUSCLES OF MASTICATION & TMJ FRACTURES GENERAL ANESTHESIA LOCAL ANESTHESIA & COMPLICATIONS NITROUS OXIDE EXODONTIA GRAFTS BIOPSY ORAL SURGERY CONDITIONS & TREATMENT CONSIDERATIONS CPR GUIDELINES HORMONES, CALCIUM, BLOOD GLUCOSE

CHAPTER 9 ENDODONTICS TOOTH FRACTURES FLAPS & SLOB RULE PULPOTOMY & APEXIFICATION DIRECT & INDIRECT PULP CAPPING CANAL ACCESS & DEBRIDEMENT OBTURATION IRRIGANTS & CHELATING AGENTS

ZOE, MTA, APICOECTOMY PERIRADICULAR SURGERY & CURETTAGE ENDODONTIC INSTRUMENTATION MANAGING AVULSION INJURIES EXTERNAL & INTERNAL RESORPTION PULP, DENTIN, CEMENTUM MANDIBULAR & MAXILLARY ROOT ANATOMY RCT ADJUNCTS, INDICATIONS & CONTRAINDICATIONS PERIAPICAL & PERIODONTAL ABSCESSES PROBING LESIONS REVERSIBLE & IRREVERSIBLE PULPITIES, NECROSIS RESTORING TEETH AFTER RCT

CHAPTER 10 PEDIATRIC DENTISTRY ANTI-ANXIETY MEDICATIONS PULP TREATMENT PEDIATRIC DISEASES & CONDITIONS TOOTH ABNORMALITIES TOOTH DEVELOPMENT & ERUPTION PRIMARY DENTITION FLUORIDE SEALANTS TOOTH TRAUMA

BEHAVIOR MANAGEMENT

CHAPTER 11 ORTHODONTICS OVERBITE & OVERJET MALOCCLUSION ANGLE MALOCCLUSION CLASSIFICATION CROSSBITES CEPHALOMETRICS MIXED DENTITION & ANALYSIS PRIMATE SPACES & SPACE MAINTENANCE ORTHODONTIC APPLIANCES

CHAPTER 12 BEHAVIORAL SCIENCE, PUBLIC HEALTH, OSHA & INFECTION CONTROL

BEHAVORIAL SCIENCE PUBLIC HEALTH AND RESEARCH METHODS ORAL HEALTH INDICES OSHA AND INFECTION CONTROL DENTIN “1,001+ HIGH-SPEED DRILL NBDE II PREVIEW

ORAL PATHOLOGY & DISEASES METABOLIC & GENETIC DISEASES HYPERTHYROIDISM-caused by excessive production of thyroid hormone (THYROXIN). Thyroxin’s stimulates cellular metabolism, growth, and differentiation of all tissues. In excess, it leads to high basal metabolism, fatigue, weight loss, excitability, elevated temperature (heat intolerance, sweating), generalized osteoporosis, fine hair, diarrhea, tremor (shakiness), tachycardia (rapid heart rate). Oral manifestations are not uncommon, but if the disturbance starts early in life, premature tooth eruption and loss of deciduous dentition are common. GRAVES DISEASE & EXOPTHALMOS (bulging eyes). Two types of Hyperthyroidism: 1. Graves Disease (Toxic Diffuse Goiter)-most common form that occurs mostly in WOMEN ages 20-40. Usually arises after an infection or physical or emotional stress. Typical signs of hyperthyroidism are present plus GOITER (bulging neck) & EXOPHTHALMOS (bulging eyes). 2. Plummer’s Disease (Toxic Multinodular Goiter)-

caused by the presence of many toxic thyroid nodules (adenomas) within the thyroid gland. Plummer’s is uncommon in adolescents and young adults, and increases with age. Exophthalmos (bulging eyes) is rare.

HYPOTHYROIDISM-a clinical feature is WEIGHT GAIN, cold intolerance, lowered pitch of voice, mental and physical slowness, constipation, dry skin, coarse hair, and puffiness of face, eyelids, and hands. • Myxedema-very severe hypothyroidism in adults, much more common in WOMEN than men. Characterized by puffiness of face and eyelids, swelling of tongue and larynx. Skin becomes dry, and rough, and hair sparse. Individual has a low basal-metabolic rate and low body temperature, poor muscle tone, low strength, tires easily, and are mentally sluggish. Administering thyroid hormones alleviates myxedema. • Hashimoto’s Disease (Thyroiditis)-autoimmune

disease where the immune system attacks the thyroid gland causing primary hypothyroidism. It is the most common cause of hypothyroidism in the U.S., affecting more women then men. Potential causes include genetics, hormones, excessive iodine, or radiation exposure. Most common symptoms: fatigue, weight gain, pale/puffy face, feeling cold, joint or muscle pain, constipation, thin or brittle hair, depression, slow heart rate. • Cretinism-severe hypothyroidism in a child, due to lack of thyroid hormone causing retardation of growth and abnormal bone development. Severe mental retardation is caused by improper CNS development. If recognized early, Cretinism can be improved with thyroid hormones. • Dental findings in a child with hypothyroidism are a LARGE TONGUE, under-developed mandible, overdeveloped maxilla, delayed eruption, and longer retained deciduous teeth. HYPERPARATHYROIDISM-a common complication is KIDNEY STONES (renal calculi). Kidney stones form due to an increase in urinary excretion of calcium and phosphate. Osteoporosis, GIANT CELL GRANULOMAS, and metastatic calcifications are manifestations of hyperparathyroidism.

• The main cause is an ADENOMA (benign tumor of the gallbladder epithelium). Laboratory findings include hypercalcemia, decreased serum phosphorus, and increased serum alkaline phosphatase and serum PTH. Clinical characteristics: cystic bone lesions (osteitis fibrosa cystica or von Recklinghausen’s Disease of bone), nephrocalcinosis, kidney stones (renal calculi), and peptic duodenal ulcers. May find welldefined cystic radiolucencies on a panorex or periapical radiograph. • EXCESS LOSS OF CALCIUM in urine stimulates the parathyroid glands to undergo hyperplasia because the feedback mechanism that detects low serum calcium elicits growth of the gland. The resulting metabolic effects are identical to primary hyperparathyroidism effects. HYPOPARATHYROIDISM-in rare instances, associated with congenital thymic hypoplasia (DiGeorge’s syndrome). Hypoparathyroidism is most commonly caused by accidental surgery excision during thyroidectomy. ACROMEGALY – a hormonal disorder that occurs when the PITUITARY GLAND produces EXCESS GROWTH HORMONE (HYPERPITUITARISM) due to a BENIGN TUMOR after adolescence (fusion of long bone epiphyses). Most commonly affects MIDDLE-AGED ADULTS and can cause serious illness and premature death.

• In > 90% of acromegaly patients, GH overproduction is caused by a BENIGN TUMOR of the pituitary gland (ADENOMA). Whether or not the epiphyses of the long bones have fused with the shaft is the main determinant of whether gigantism or acromegaly will occur when there is over-secretion of GH by the pituitary gland. • Clinical Signs: soft tissue swelling of the hands & feet (an early feature), with patients noticing a change in ring or shoe size. Gradually, bony changes alter the patient’s facial features (i.e. brow & lower jaw protrude, nasal bone enlarges, and teeth spacing increases). • Oral Manifestations of Acromegaly & Gigantism: enlarged tongue, mandibular prognathism, teeth are tipped to buccally or lingually due to an enlarged tongue, and roots may be longer than normal. • GIGANTISM-caused by a benign tumor BEFORE adolescence (non-fusion of epiphyses). DWARFISM (Pituitary Dwarfs) – characterized by arrested growth caused by undersecretion of GROWTH HORMONE. Dwarfs often have limbs and features not properly proportioned or formed. • Oral Manifestations: delayed eruption rate & shedding of teeth, clinical crowns & roots appear smaller, dental arch is smaller causing malocclusion, and an under-

developed mandible. • MOST COMMON type of Dwarfism is ACHONDROPLASIA. Child is very short (~50 inches), fingers are stubby, bowed legs, bulging forehead, bossing of frontal bones, saddle-like nose, and mandibular prognathism. OSTEOGENESIS IMPERFECTA (“BRITTLE BONES”) –a rare genetic defect/disorder that affects the COLLAGEN PRODUCTION (major protein of the body’s C.T.). Person either has less collagen than normal, or poorer quality of collagen than normal causing WEAK BONES THAT FRACTURE/BREAK EASILY often from little or no cause. • The main clinical characteristic is EXTREME FRAGILITY & POROUS BONES with a proneness to fracture due to the effects of inadequate osteoid production. • Additional Clinical Features: BLUE SCLERA, deafness due to osteosclerosis, loose joints, low muscle tone, triangular face, and a tendency toward spinal curvature. • Teeth have bulbous crowns with a cervical constriction, partially or completely obliterated pulps, and narrower & shorter roots. Deciduous (primary) teeth are more severely affected than permanent teeth. Teeth

are poor and abnormal due to dentin malformation (Type 1 Dentinogenesis Imperfecta); may be linked to DENTINOGENESIS IMPERFECTA. • Treatment: NO KNOWN CURE. Treatment is directed toward preventing/controlling the symptoms. HYPOPHOSPHATASIA –resembles OSTEOGENESIS IMPERFECTA. It’s an inherited metabolic (chemical) BONE DISEASE that results from LOW LEVELS of ALKALINE PHOSPHATASE (enzyme essential the calcification of bone tissue). Loosening, hypocalcification, and premature loss of deciduous teeth are characteristic. Radiographically, large pulp chambers and alveolar bone loss are present. Hypophosphatasia’s severity varies greatly. Some patient’s have blue sclera that resembles osteogenesis imperfecta. Others have deformity of the arms, legs, and chest and/or frequent bouts of pneumonia and recurrent fractures. PAGET’S DISEASE OF BONE (OSTEITIS DEFORMANS) – a common, chronic, non-metabolic bone disorder characterized by an INCREASE in serum ALKALINE PHOSPHATASE levels. Bones become enlarged & deformed, dense, but fragile due to excessive breakdown and formation of bone. Has potential to undergo “spontaneous” malignant transformation. There is excessive bone destruction and unorganized bone repair.

• Radiographic Features: “COTTON-WOOL” APPEARANCE ON PANOREX (skull and jaws). Hypercementosis of roots and loss of lamina dura around roots. • Effects males & females, but rarely people under 40yrs (affects middle-aged & elderly people). • Cause is hereditary. Patients are predisposed to developing OSTEOSARCOMAS. • Signs & Symptoms: PAIN in affected area, bone deformity & susceptibility to fractures in the affected area, headache, and hearing loss if the affected area is the skull. Symptoms develop SLOWLY. • Clinical Features: Patients may give a history of progressively INCREASE IN HAT SIZE OR NEED FOR NEW DENTURES being made more frequently due to bony changes. • Bones are warm to touch due to increased vascularity. • Lab tests show highly increased serum alkaline phosphatase, urinary calcium, & hydroxyproline; with normal levels of SERUM PHOSPHATE & CALCIUM. • Treatment: Administer anti-metabolites or CALCITONIN to decrease bone resorption, or treat with a high-protein & high calcium diet.

OSTEOMALACIA (Adult Rickets) – SOFTENING of bones in adults because osteoid tissue in bones failed to calcify due to LACK OF VITAMIN D. More common in women, and may be asymptomatic until a bone fracture occurs. • STEATORRHEA-one of the most common causes of Osteomalacia due to FAT MALABSORPTION where the body cannot absorb fats, so fats are passed directly out of the body in stool causing poor absorption of vitamin D (fat soluble) and calcium. Osteomalacia affects ALL BONES, specifically at their epiphyseal growth plates. • Signs & Symptoms: pain in bones of the arms, legs, spine, and pelvis. RICKETS (Child Osteomalacia) –causing skeletal deformities, and usually accompanied by irritability and generalized muscle weakness. Bowlegs, pigeon breast, and

protruding stomach are signs. Teeth are affected by delayed eruption, malocclusion, and developmental abnormalities of dentin and enamel, with a higher caries rate. CEREBRAL PALSY – a group of disorders affecting body movement and muscle coordination due to an insult or anomaly of the brain’s motor control centers. This damage interferes with messages from brain to the rest of the body. The effects vary greatly among people. • CP is mainly characterized by SPASTIC PARALYSIS or impairment of control or coordination over voluntary muscles. Often accompanied by mental retardation, seizures, & disorders of vision/communication. • NO ORAL PATHOLOGIC MANIFESTATIONS are present in people with cerebral palsy, but several conditions are more common, or more severe than in the normal population: Higher incidence of periodontal disease, caries, bruxism, and malocclusion. Prone to gingival hyperplasia if Dilantin is used to control seizures. More susceptible to trauma, especially maxillary anterior teeth. DOWN SYNDROME – a congenital defect caused by a

chromosomal abnormality (TRISOMY 21), marked by various degrees of mental retardation and characteristic physical features (short, flattened skull, slanting eyes, thickened tongue/fissured, broad hands/feet, etc.) • Oral Manifestations: mandibular prognathism, increased periodontal disease, thickened or fissured tongue, delayed teeth eruption, higher incidence of congenitally missing teeth, malocclusion, & enamel dysplasia. MUSCULAR DYSTROPHY – a group of genetic diseases marked by progressive weakness & degeneration of skeletal or voluntary muscles that control movement. • Oral Manifestations: increase in dental disease if oral hygiene is neglected, weakness in muscles of mastication causing decreased maxillary biting force, higher incidence of mouth breathing, and open bite. ECTODERMAL DYSPLASIA – hereditary condition characterized by abnormal development of the skin and associated structures (hairs, nails, teeth, & sweat glands). ED involves all structures derived from ECTODERM, affecting MALES more than females. Manifests orally as reduced/missing teeth. • Clinical Signs: hypothrichosis (decrease in hair (fine

sparse hair), anhidrosis (no sweat or sebaceous glands, causing heat intolerance), Anodontia (complete absence of teeth), Oligodontia (partial absence of teeth), no tooth buds of the primary or permanent dentition (edentulous), depressed nose bridge, lack of salivary glands, and child appears much older than their true age. • Affects tooth bud development causing congenitally missing teeth (lack of permanent teeth) and/or pegshaped or pointed teeth. Enamel may also be defective. • Treatment: Dentures can be fabricated for young children, but they will need to be replaced periodically to accommodate the patient’s jaw growth. Implants can be placed once the jaw if fully developed, or orthodontics to close spaces. Treatment is complex so use a multidisciplinary treatment approach.

CLEIDOCRANIAL DYSPLASIA (DYSOSTOSIS)-genetic disorder of bone development characterized by absent or incomplete formed COLLAR BONES, heavy protruding jaw, wide nasal bridge, and dental abnormalities (malaligned teeth, multiple supernumerary teeth, and unerupted teeth). Observing a panorex or FMX alone often suggests the diagnosis.

PIERRE-ROBIN SYNDROME – an inherited disorder with the following findings in the NEONATE: o Micrognathia-smallness of the jaws. o Glossoptosis-downward displacement or retracted tongue. o Breathing problems & cleft palate. LATERAL CLEFTING OF THE LIP – results from failure of the MAXILLARY & FRONTAL NASAL PROCESSES TO MERGE. Cleft lip occurs during the 5th-6th week of embryonic life. It can be bilateral or unilateral, more common in males, and involves the LEFT SIDE more than the right side. CLEFT PALATE – occurs in 6th-8th week of embryonic life. Isolated cleft palates are more common in females, characterized by a fissure in the midline of the palate due to

failure of the two sides to fuse during embryonic development. The most severe handicap caused by cleft palate is an impaired mechanism PREVENTING NORMAL SPEECH & SWALLOWING. Important: Speech problems associated with Cleft Lip & Cleft Palate are usually due to the inability of the soft palate to close airflow into the nasal area. CHERUBISM – a BENIGN genetic autosomal dominant disease of the maxilla & mandible, typically in children by age 5 (affects males 2:1). Most cases occur in the MANDIBLE. The jaws are firm and hard to palpation, and regional lymphadenopathy may be present. BILATERAL expansion of the jaws gives the child a very round face, reminding one of cherubs (cupids) in paintings. The tumors stop growing shortly after puberty. As the patient’s age and size increases, the deformity is less noticeable. • Histologically, cherubism lesions closely resemble Central Giant Cell Granulomas. Histology shows a giant cell lesion with some reactive bone formation. However, perivascular collagen cuffing is pathognomonic for cherubism. • Radiographically: lesions appear as multiple, welldefined, multi-locular radiolucencies of the jaw. • No associated systemic manifestations. However, the

deciduous dentition may spontaneously shed prematurely, starting as early as age 3. There is often delayed eruption of the permanent dentition, which is often defective with the absence of numerous teeth and displacement of those teeth present. • Treatment: cautious waiting as Cherubism tends to regress in early adulthood. Do not treat with radiation therapy. CYSTIC FIBROSIS – a congenital/genetic metabolic disorder that causes EXOCRINE GLANDS (glands that secrete fluids into a duct) to produce ABNORMAL SECRETIONS mainly affecting GI and respiratory systems. CF results in several symptoms (the most important symptom affects the digestive tract and lungs). In some glands (glands in the pancreas & intestines), the secretions are thick or solid excessively viscous mucous that can completely block a gland. Mucous-producing glands in the lung’s airways produce abnormal secretions that clog the airways allowing bacteria to multiply. CF is the MOST COMMON genetic disease causing death among white people in the U.S. and is equally common in boys & girls. • CF is usually characterized by COPD, exocrine pancreatic insufficiency, and abnormally high sweat electrolytes (sweat glands secrete fluids that have a high sodium & chloride content).

• Oral Manifestations: staining of teeth as patients with CF are usually subjected to large amounts of tetracycline during childhood. A high % of children have dark-colored teeth (yellowish gray to dark brown). There is a greatly reduced caries rate in CF patients, probably due to saliva alterations and long-term use of antibiotics. • CF Signs & Symptoms: poor growth despite good appetite, malabsorption, and foul, bulky stools (steatorrhea), chronic bronchitis (COPD) with cough, recurrent pneumonia with respiratory infections, clubbing of fingers/toes, & barrel-chested appearance.

INFLAMMATORY JAW LESIONS OSTEOMYELITIS –inflammation or infection of the bone marrow and adjacent bone, usually caused by bacteria (Staphylococci) due to trauma or surgery by a direct extension from a nearby infection, or via the bloodstream. • Signs & Symptoms: pain, redness, swelling in the infected area, fever, and general malaise. Radiographically, poorly circumscribed radiolucency with a central sclerotic nidus may be present. CONDENSING OSTEITIS (CHRONIC FOCAL SCLEROSING OSTEOMYELITIS) – an unusual bone reaction to an infection (most often associated with a longstanding periapical infection) that occurs during instances of extremely high tissue resistance or in cases of low-grade infection. There may be no signs or symptoms of the disease, other than mild pain associated with an infected pulp. Mandibular 1st molar is the tooth most commonly involved. Most often occurs in young patients. • Radiographic Findings: periapical radiographs show pathognomonic, well-circumscribed radiopaque mass of sclerotic bone surrounding and extending below the apex of one or both roots. The entire root outline is

always VISIBLE (important feature that radiographically distinguishes it from a benign cementoblastoma). • A tooth with a condensing osteitis lesion can be treated with RCT or extracted, since the pulp is infected, and the infection has spread past the immediate periapical area. The sclerosing bone constituting the osteomyelitis is NOT attached to the tooth, so it remains after the tooth is treated or removed. PERIAPICAL ABSCESS – usually arises from pulpal infection of a tooth due to carious involvement of the tooth. The cellular debris and/or infection that caused the tooth pulp to become necrotic, slowly filters out of the root tip, producing an inflammatory reaction around the root tip. A periapical abscess can also occur after traumatic injury to a tooth, causing pulpal necrosis, and in cases of irritation of the periapical tissues (either by mechanical manipulation or application of chemicals) in endodontic procedures. Clinical Features: • Acute Periapical Abscess: tooth is extremely painful to percussion (may feel slightly extruded from its socket), and is MOBILE. Radiograph presents only a slight thickening of the periodontal membrane (PDL). • Chronic Periapical Abscess: presents as a granuloma or cyst (radiolucent area at the root apex), but there are usually no clinical features or symptoms (asymptomatic).

• Treatment: establish DRAINAGE by opening the pulp chamber (RCT) or extracting the tooth. If not treated, it can cause serious complications (i.e. osteomyelitis, cellulitis, & bacteremia). OSTEONECROSIS–bone death or necrosis; a rare complication of cancer patients (radiation and chemotherapy), patients with tumors or infectious embolic events, or with osteoporosis taking IV or oral bisphosphonates. May be caused by a defect in bone remodeling or wound healing (defect in osteoclast function). BISPHOSPHONATE-OSTEONECROSIS (BON)-a dental phenomenon that may lead to surgical complications (bone necrosis) due to impaired wound healing after extractions, periodontal surgery, or RCT. • Caution with patients taking IV bisphosphonates (Zometa (zoledronic acid) & Aredia (pamidronate) for osteoporosis and cancer treatment respectively. • Caution with patients taking oral bisphosphonates (Fosamax, Actonel, & Boniva). Occurs more common in patients taking IV bisphosphonates (20%) and less than 1% taking oral bisphosphonates.

OSTEOPOROSIS – a reduction of total skeletal mass due to INCREASED BONE RESORPTION, causing predisposition to pathologic fractures caused by calcium or estrogen hormone deficiencies over a long time period. BONES BECOME LESS DENSE & BRITTLE. • Osteoporosis is most common in THIN, ELDERLY WHITE WOMEN. Treatment: estrogen therapy, calcium & vitamin D supplements, bisphosphonates. OSTEOPETROSIS (“Albers-Schonberg Disease” or “Marble Bone Disease”) – an uncommon genetic disorder that manifests in infancy characterized by an OVERGROWTH & DENSENESS OF BONES due to a DEFECT IN OSTEOCLASTS which are needed for bone marrow formation. The long bones become dense and hard to the extent that BONE MARROW IS OBLITERATED (prevents bone marrow formation). BONES BECOME HARD BUT BRITTLE & DENSE. • Clinical Signs: abnormal bone & dental development, fragile bones, stunted growth anemia, spleen & liver

enlargement, blindness, and progressive deafness.

CONNECTIVE TISSUE LESIONS VON RECKLINGHAUSEN’S DISEASE (NEUROFIBROMATOSIS) – the most common feature is NEUROFIBROMATOSIS (multiple tumors of nerve tissue origin). VRD is a relatively common inherited autosomal dominant trait characterized by multiple neurofibromas, cutaneous café-au-lait macules, bone abnormalities, & CNS changes. • Clinical Signs: 6 or more café-au-lait macules > 1.5cm in diameter indicates VRD. • Treatment: No satisfactory treatment. The lesions run a high-risk of becoming malignant. • A single neurofibroma presents at any age as a noninflamed, asymptomatic nodule on the tongue, buccal mucosa, & vestibule. This single nodule is removed by surgical excision, and rarely occurs. SCLERODERMA – a relatively RARE autoimmune disease affecting the blood vessels & C.T. characterized by hardness & rigidity of the skin and subcutaneous tissue. The continuous deposition of collagen in major organs can cause dysfunction and potential organ failure.

• Clinical Features: systemic scleroderma usually appears during middle-age (30-50yrs), mainly in females (4:1). The skin is usually affected first and becomes indurated. • Oral Radiographs: show ABNORMAL WIDENING OF THE PDL AROUND ROOTS (this is also found in osteosarcomas). The space is created by a thickening of the periodontal-membrane due to an increase in size & number of collagen fibers. The enlarged space is almost uniform in width, surrounds the entire tooth root, making the tooth appear as if it is being extruded rapidly from its socket. Other oral radiographic features may include bilateral resorption of the angle of the mandible’s ramus, or complete resorption of the mandibular condyles and/or coronoid process. • Treatment: No satisfactory treatment, other than palliative therapy.

ORAL TRAUMATIC NEUROMA- a SOFT TISSUE TUMOR due to trauma to a peripheral nerve, usually appearing as a very small nodule/swelling (< 0.5cm in diameter) of the mucosa near/over the mental foramen on the alveolar ridge in edentulous areas, lips, & tongue. MOST COMMON SITE IS OVER THE MENTAL FORAMEN IN EDENTULOUS PATIENTS, but they can occur wherever a tooth has been extracted. Extraction sites in the anterior maxilla & posterior mandible are common sites. • In the oral cavity, the traumatic neuroma may be due to trauma from a surgical procedure (i.e. tooth extraction, local anesthetic injection, or accident). A nodule or swelling PAINFUL WHEN PALPATED, as applied digital pressure elicits a response described as an “electric shock”.

• Multiple neuromas on the lips, tongue, or palate may indicate the patient may have MEN III (Multiple Endocrine Neoplasia Syndrome). • Treatment: surgical excision of the nodule with small proximal portion of the involved nerve. Recurrence is uncommon. NEURILEMOMA (SCHWANNOMA) – a benign SOFT TISSUE TUMOR (encapsulated mass) of Schwann cells around the nerve that presents as an asymptomatic lump most common on the TONGUE. Bony lesions may cause pain or paresthesia. Derived from a proliferation of Schwann cells of the neurolemma that surrounds peripheral nerves. Covered by normal mucosa, sessile & does not metastasize but still do biopsy. Treatment: conservative excision. Recurrence is rare. NEUROFIBROMA-may be derived from the Schwann cell or Perineural Fibroblast and can occur in two forms: 1. Solitary Neurofibroma-an asymptomatic nodule on the tongue, buccal mucosa, or vestibule treated by surgical excision. 2. Multiple lesions as part of the Neurofibromatosis syndrome. Removing the lesions is impractical, but monitor due to the high risk/rate of malignant transformation.

FIBROMA (“IRRITATION FIBROMA OR “TRAUMATIC FIBROMA”)-the most common intra-oral BENIGN neoplasm of CONNECTIVE TISSUE (soft tissue) origin, that occurs in people of all ages and with equal frequency in both sexes. It’s the most common tumor in the oral cavity and is reactive. Fibromas can arise from almost any soft tissue in the mouth, but are most common on the buccal mucosa, lateral border of tongue, and lower lip. NODBULAR PINK MORPHOLOGY (pink, fibrous nodule with smooth surface).

• Clinical Features: usually pink (same color or lighter than normal buccal mucosa, painless, smooth, elevated, well-demarcated masses). Smooth, sessile, soft-to-firm nodule on buccal mucosa, lips, and tongue. • Microscopic Features: bundles of collagen interspersed (interlacing) with fibroblasts and small blood vessels.

• Histogenesis: fibrous C.T. (C.T. origin). • In most cases, the tumor is reported present for months or years with a slow-growing behavior pattern. • Some feel “true fibromas” of the oral cavity are rare, and are merely localized hyperplasia due to longstanding irritation or trauma (“irritation fibroma or “traumatic fibroma”). These fibrous nodules are comparable to hyperplasias from denture irritation (“epulis fissuratum”). The only difference between a “true fibroma (a true neoplasm) and “irritation fibroma” (not a true neoplasm), is the hyperplastic tissue with an irritation fibroma can regress after removing the irritant, while a true fibroma will not regress. Treatment: conservative surgical excision. Recurrence is rare. PERIPHERAL FIBROMA – a well-demarcated focal mass of hyperplastic tissue with either a sessile or pedunculated base. Similar in color to surrounding C.T., and may be ulcerated. Treatment: local excision & recurrence is RARE. 3 Forms of Peripheral Fibroma: 1. Peripheral Ossifying Fibroma-a gingival mass with visible characteristic calcified islands of bone and an ulcerated surface. The gingiva anterior to the permanent molars is most often affected. Histologically, in its high degree of cellularity, it exhibits bone formation (in contrast to peripheral fibroma).

Vascularity is NOT a prominent feature as it is with a pyogenic granuloma. POF is a subtype or variant form of a peripheral fibroma, although both originate from an inter-dental papilla and both occur more frequently in young adult females (but can occur at any age and are more common in children & young adults). Usually presents as a well-demarcated focal mass of hyperplastic tissue on the gingiva with a sessile or pedunculated base. It is usually the same color as normal mucosa or slightly reddened. May demonstrate bone radiographically and often demonstrates bone formation histologically. Treatment: Local Excision. POF lesions may recur (peripheral fibroma lesions rarely recur). 2. Peripheral Odontogenic Fibroma-gingival mass composed of well-vascularized, non-encapsulated fibrous C.T. 3. Giant Cell Fibroma-a fibrous hyperplasia composed of multi-nucleated C.T. cells. GIANT CELL TUMOR – a bone tumor of multi-nucleated giant cells that resemble osteoclasts scattered in a matrix of spindle cells. May be benign or malignant, and can cause pain, functional disability, and sometimes pathologic bone fracture. PAPILLARY FIBROMA – a benign neoplasm of C.T. origin.

LIPOMA – a COMPETELY BENIGN soft tissue tumor derived from ADIPOSE (FAT) tissue (C.T. origin). It is smooth or lobulated, sessile or pedunculated (foot-shaped), soft, movable, painless, yellowish-white nodular mass. Vessels are visible on the surface. Locations: floor of mouth, buccal mucosa, and tongue. YELLOWISH MASS covered by normal mucosa. A biopsy specimen will FLOAT in the formalin. Easier to diagnose than other tumors because it is yellowish, soft, smooth, & movable, and FLOATS IN FORMALIN. • Microscopic Features: lobules of mature fat separated by delicate C.T. septae. • Treatment: conservative excision. Recurrence is rare. Remove surgically only if it becomes painful, tender, infected, or enlarges to where it becomes bothersome.

RHABDOMYOMA – a RARE BENIGN tumor of

SKELETAL MUSCLE. Tongue is the most common place in the head & neck. Sessile, non-painful, and covered by normal mucosa. LEIOMYOMA – an always benign tumor of SMOOTH MUSCLE (smooth muscle neoplasm). Non-painful, sessile, (covered by normal mucosa). Bundles of spindle-shaped smooth muscle cells. LYMPHANGIOMA – a BENIGN YELLOWISH-TAN TUMOR composed of a mass of DILATED LYMPH VESSELS. Most common site in the oral cavity is the TONGUE, but can appear on the lips & neck. Derived from endothelial cells, C.T. origin. Clinical Features: painless, nodular, vesicle-like swelling that equally affects both sexes. • Superficial lesions are manifested as grayish-red papillary lesions. On the tongue, considerable enlargement can occur (macroglossia). The papillary lesions may contain fluid, and are often present at birth or arise early in life, but are less common than Hemangiomas.

• Microscopic Features: can be simple, circumscriptum, or cystic (closely related to cystic hygroma, containing serous fluid). • Treatment: SURGERY OR CRYOSURGERY. May recur due to their lack of encapsulation.

BENIGN EPITHELIAL TUMORS

PAPILLOMA – the most common BENIGN neoplasm of EPITHELIAL TISSUE ORIGIN. It appears as a pedunculated (foot-shaped), or sessile WHITISH cauliflower-like mass on the tongue (posterior border), lips, gingiva, or soft palate. Papillomas are soft in the oral cavity, but on exposed areas of the lips, are usually rough & scaly. • Papilloma-a vital benign cauliflower-like, white lesion with a verrucuous & pedunculated (foot-shaped) surface. Easy to diagnose. VIRAL. Lesion is not covered by normal mucosa and is a disease of epithelium. Nonulcerated, small, slow growing, usually non-painful. Lateral border of tongue, hard or soft palate are common areas. MUST EXCISE SURGICALLY and recurrence is

rare. • Microscopic Features: finger-like projections of stratified squamous epithelium supported by thin cores of vascular fibrous C.T. Epithelium may show hyperkeratosis or parakeratosis. Histogenesis: squamous epithelium. Verruca (Warts)-similar to a papilloma, but is NOT pedunculated caused by a viral infection. Do excision & biopsy, especially children who have this on their finger and place their finger in their mouth and spread the infection orally. Keratoacanthoma-a non-painful crater-formed lesion (VIRAL) growing for 2-3 months in the SKIN that looks like squamous cell or basal cell carcinoma, and can heal by itself, but must still do biopsy. Can last up to 6 months. It is usually only in the skin and very rarely inside the mouth. MULTIPLE ENDOCRINE NEOPLASIA SYNDROMES (MEN SYNDROME) – groups of syndromes characterized by tumors of various endocrine glands that occur in association with a variety of other pathologic features. The most important aspect of MEN syndrome is medullary carcinoma of the THYROID due to its ability to metastasize and cause death. Thus, detecting mucosal neuromas may alert the clinician for early diagnosis and treatment. MEN is classified into 3 groups:

1. Men I Syndrome-tumors or hyperplasias of the pituitary, parathyroids, adrenal cortex, and pancreatic islets (pancreas). 2. Men II (Sipple’s Syndrome)- parathyroid hyperplasia or adenoma, but NO tumors of the pancreas. Patients have pheochromocytomas of the adrenal medulla, and medullary carcinoma of the thyroid gland. 3. Men III Syndrome-mucocutaneous neuromas, pheochromocytomas of the adrenal medulla, and medullary carcinoma of the thyroid gland. The most constant feature is neuromas (especially in the oral cavity), most commonly on the lips, tongue, and buccal mucosa. EPULIS GRANULOMATOSUM – soft, non-painful, bleed easily, most often caused by RETAINED FOREIGN MATERIAL (i.e. bone or tooth fragment) due to an iatrogenic error. Most commonly found in a postextraction socket, almost always within 10 days of the extraction. • TREATMENT: CURETTAGE. • Microscopic Features: granulation tissue in bone, dentin, cementum, or foreign material. CONGENITAL EPULIS OF NEWBORNS (CONGENITAL

GINGIVAL GRANULAR CELL TUMOR) – composed of cells identical to a granular cell myoblastoma (granular cell tumor). Usually on the ANTERIOR GINGIVA OF NEWBORNS as a PINK, non-inflammed, pedunculated or broad-based mass. Maxillary gingiva is involved more than the mandibular gingival. Affects females more than males. • Treatment: surgical excision with minimal recurrence.

GRANULAR CELL MYOBLASTOMA (TUMOR) – an uncommon neoplasm of unknown etiology, presenting as an uninflamed, asymptomatic mass. TONGUE is the most common location in the head & neck region. Can affect any age group, but affects females more than males. Congenital Epulis & Granular Cell Myoblastoma lesions are histologically identical as they both contain granular cells. However, Congenital Epulis of newborns does not exhibit pseudo-epitheliomatous hyperplasia of the

overlaying epithelium that is often seen in Granular Cell Myoblastoma.

PYOGENIC GRANULOMA (PREGNANCY TUMOR) – an elevated ulcerated mass that bleeds easily, more common in females (pregnant women), and may recur. Caused by minor trauma that provides a pathway for non-specific organisms & calculus. Most commonly found on the gingiva, but also the lips, tongue, & buccal mucosa. TREATMENT: EXCISION (but may recur). Has exuberant granulation tissue microscopically.

• PYOGENIC GRANULOMA is BENIGN. The MOST common site is the INTER-DENTAL GINGIVA, but may also occur on the lower lip, tongue, & buccal mucosa. Rarely occurs on other areas of the oral mucous membrane. Arises due to minor tissue trauma (i.e. cementation of a crown or calculus) that provides a pathway for the invasion of non-specific types of microorganisms. Pregnant patients are prone to these lesions (called a “Pregnancy Tumor” in a pregnant patient). May be caused secondarily by an altered endocrine state (hormone changes) during pregnancy in the 1st trimester. • Clinical Features: soft, pedunculated (foot-shaped) broad-based growths with a smooth red surface due to the presence of hyperplastic granulation tissue that contains many capillaries. They are often ulcerated, bleed easily, and may look RASBERRY-LIKE. • Treatment: Surgical Excision after pregnancy. May occasionally recur.

VERRUCAL PAPILLARY LESIONS VERRUCA VUGLARIS (SQUAMOUS PAPILLOMA) – the common WART of VIRAL etiology (caused by Papilloma virus) that is a common skin tumor analogous to the oral papilloma. It has an incubation period of 6 weeks to 1 year. Although it is a primary lesion of the skin, it may occur in the oral cavity (especially lips & palate). Clinically, it is a sessile, soft, CAULIFLOWER-LIKE LESION. • Microscopically: a papillomatous lesion where the epithelium is thrown into folds. The lesion shows alternating hyperkeratosis, parakeratosis, and long epithelial ridges. If excised, they usually do not recur, but autoinoculation is possible. Intra-orally, that is how most cases develop. INFLAMMATORY FIBROUS HYPERPLASIA (“EPULIS FISSURATUM” OR “PALATAL PAPILLOMATOSIS”) – found at the area of the denture borders, more common in the maxilla (hard palate) than mandible caused by ill-fitting dentures and poor oral hygiene. • Clinical Features: rolls of soft-tissue in the muco-labial fold, RED-PINK, elongated, firm, ulceration, soft lesion.

• Treatment: SURGICAL EXCISION and re-evaluate the prosthesis or remake or reline the dentures.

NEOPLASMS LYMPHOEPITHELIOMA – a poorly differentiated squamous cell carcinoma involving lymphoid tissue in the tonsils & nasopharynx regions. It has a high frequency in young adults of East Asian decent. The primary lesion is usually very small (often completely hidden). SWELLING OF THE LYMPH NODES is the most common symptom, followed by sore throat, nasal obstruction, bloody nose, & headache. • Lymphoepitheliomas are composed of squamous or undifferentiated cells, with a slight-to-moderate amount of fibrous stroma that contain numerous lymphocytes. Lymphoepithelioma shows METASTASIS AT AN EARLY STAGE TO THE CERVICAL LYMPH NODES. • Treatment of choice is RADIATON, but the complicating factor is the relative inability to treat the widespread metastases in the various organs. POOR PROGNOSIS (30% 5-year survival rate). METASTIC CARCINOMA–the most common malignancy affecting SKELETAL BONES. However, metastic disease of the mandible and maxilla is unusual (~1%). Most important, a jaw tumor may be the first evidence of dissemination of a known tumor from its primary site.

Metastases to the jaws most commonly originates from primary carcinomas of the BREAST, KIDNEY, LUNG, COLON, PROSTATE, & THYROID. Metastatic carcinoma of the jaws is LEAST likely to originate from the brain. • Clinical Features of Metastic Jaw Lesions: may be completely asymptomatic, but there is usually paresthesia or anesthesia of the lip or chin due to involvement of the mandibular nerve (patient is usually aware of slight discomfort or pain). Teeth in the area are loose or extruded (the molar region is mainly involved), there can be swelling or expansion of the jaw, and appears as an asymptomatic radiolucency. Although rare, metastic jaw tumors affect the mandible MUCH more than the maxilla. Most common osseous malignancies are OSTEOSARCOMAS, followed by Chondrosarcomas, Fibrosarcomas, & Ewing’s Sarcoma. OSTEOSARCOMA (OSTEOGENIC SARCOMA)-a MALIGNANT BONE TUMOR of anaplastic cells derived from mesechyme. MOST COMMON PRIMARY MALIGNANT TUMOR OF BONE, arising in LONG BONES which show the greatest longitudinal growth. Joint involvement is rare. Its PEAK incidence is BEFORE epiphyseal fusion (ages 10-25yrs), but a later peak is associated with Paget’s Disease, Chronic Osteomyelitis, and previous radiotherapy.

• Radiographic Features: most important EARLY radiographic feature of an osteosarcoma of the jaw is a SYMMETRICALLY WIDENED PDL SPACE around one or more teeth. Other radiographic features may be “SUNBURST” or “SUN-RAY” appearance due to excessive bone production. Most osteosarcomas have a MIXED appearance radiographically (radiolucent + radiopaque).

ERWING’S SARCOMA – a MALIGNANT TUMOR developing from BONE MARROW, usually in long bones or pelvis of adolescent boys (peak ages 10-20yrs). It is an uncommon HIGHLY LETHAL MALIGNAN NEOPLASM OF BONE of uncertain origin. PELVIS, THIGH, and BODY TRUNK, are the most common sites. • Radiographic Characteristics: MOTH-EATEN destructive radiolucencies of the medulla, with erosion of the cortex with expansion. A variable periosteal “ONION-

SKIN” reaction may also be seen.

• Intermittent pain and swelling of the involved bone are the earliest clinical signs/symptoms. Fever & leukocytosis are also present. • Histologically, ES is difficult to distinguish from a neuroblastoma or reticulum cell sarcoma, but cells of ES contain GLYCOGEN. • When the jaws are involved, there is predilection for the RAMUS OF THE MANDIBLE with pain followed by rapid swelling and loosening of the teeth. MULTIPLE MYELOMA (“PLASMA CELL MYELOMA”) – a FATAL malignant neoplasm/lesion of bone marrow & plasma cells. Characterized by elevated blood levels of Bence-Jones Protein & multiple radiolucent areas in the mandible & skull. The tumor consists mainly of PLASMA

CELLS that destroy osseous tissues (progressive bone marrow destruction occurs and is replaced with neoplastic plasma cells). • Most patients are older than 40yrs (40-70yrs) affecting males 2x more than females (2:1). The vertebrae, ribs, and skull are most often involved. PAIN in the lumbar or thoracic regions of the spine is a common early symptom. • Jaws are RARELY a PRIMARY site, but are involved in 70% of cases. Mandibular molar-ramus area is the most common intra-oral site. Symptoms: swelling, pain, loose teeth, and paresthesia. • Radiographic Features: variable, slight demineralization to extensive bone destruction. Characteristic finding is multiple, small, discreet “PUNCHED OUT” radiolucencies of involved bones. A LATERAL SKULL RADIOGRAPH best confirms a Multiple Myeloma diagnosis. • Treatment: Chemotherapy/Radiation. Poor prognosis, with a median survival time of 2-3 years.

ODONTOGENIC MYXOMA–a RARE slow growing, usually asymptomatic MANDIBULAR TUMOR. Patients are usually < 35yrs of age. Causes localized jaw expansion. Treatment: Curettage. Not fatal.

OSTEOCHONDROMA – a BENIGN tumor of bone &

cartilage. TNM–a method to clinically stage and assess the prognosis and therapy of MALIGNANT NEOPLASMS (TUMORS) based on the primary tumor’s size (T), presence of regional lymph node involvement (N), and presence of distant metastases (M). Ex: a carcinoma of the oral cavity may have a TNM assessment of T2, N1, M0. • T = SIZE of the primary tumor. • N = Presence of regional LYMPH NODE involvement. • M = Presence of DISTANT metastasis MELANOMA exhibits a “radial” (horizontal) or “vertical” growth phases in the skin: 1. Radial Growth Phase-the INITIAL growth phase of melanoma just above & below the dermo-epidermal junction in a horizontal plane. It is clinically macular or only slightly elevated. 2. Vertical Growth Phase-begins when neoplastic cells populate the underlying dermis. Characterized clinically by an increase in size, change in color, nodularity, & ulceration. METASTASIS is possible when the melanoma reaches this phase. MALIGNANT MELANOMA –the MOST SEVERE and potentially serious type of SKIN CANCER mainly due to excessive exposure to UV sun radiation causing the tanning

cells (melanocytes) in the skin that produce a dark-colored substance (melanin) to undergo uncontrolled growth. May suddenly appear without warning, but often develops from or near a MOLE (NEVUS). Common in fair-skin white people, occurring anywhere on the skin. • SKIN CANCER IS THE MOST COMMON MALIGNANCY IN THE U.S. 1 in 100 people in the U.S. develop this cancer in a lifetime. Without treatment, it can widely metastasize and cause death. Linked to excessive SUN EXPOSURE & PAINFULL SUNBURNS during childhood. • Malignant Melanoma is an uncommon neoplasm of the oral mucosa but exhibits a definite predilection for the HARD PALATE & MAXILLARY ALEVEOLAR RIDGES. Unfortunately, oral mucosa melanomas have a poor prognosis (5-year survival rate for oral melanoma tumors is ~7%). The most common intra-oral site for melanoma is the hard palate.

4 TYPES OF MELANOMA: 1. Superficial Spreading Melanoma-the MOST COMMON form of malignant melanoma (65%), & most common cutaneous melanoma in Caucasians. The lesion is TAN, BROWN, BLACK, or ADMIXED on sun-exposed skin (especially BLACK). The cancer begins at one focus in the skin at the dermoepidermal junction (DEJ). It initially grows in a horizontal plane, along and just above & below the dermo-epidermal junction (this is the “radial” growth phase of melanoma which predominates), and is clinically macular or only slightly elevated. The “vertical” growth phase is characterized by an increase in size, change in color, nodularity, and at times ulceration.

2. Nodular Melanoma-much less common (~13% of cutaneous melanomas). THERE IS NO “RADIAL” GROWTH PHASE (it exists only in the “vertical” growth phase). NM presents as a sharply defined nodule with degrees of pigmentation (may be pink (amelanotic melanoma) or black, and occurs more often on the back, head, and neck of men. 3. Lentigo Maligna Melanoma-even less common (~10% of cutaneous melanomas), and is most common in the ELDERY population. The lesion may grow for years in the “radial” growth phase before developing into the more aggressive “vertical” growth phase. This radial growth phase is known as lentigo maligna (melanotic freckle of Hutchinson), while the vertical growth phase is known as lentigo maligna melanoma. 4. Acrolentiginous Melanoma-occurs on the hands & feet with a reputation for being ignored by the patient, resulting in the development of metastic disease. NEVUS (MOLES)-nearly all moles are normal. Atypical (Dysplastic) nevi-unusual moles are generally larger than normal moles, and are flat or have a flat part, with irregular borders with variable shades of color (especially brown, but can be a Blue Nevus). The presence of dysplastic nevi may mark a greater risk of malignant melanoma developing on

apparently normal skin.

ACQUIRED NEVI (MOLES) – small, usually dark, skin growths that develop from pigment-producing cells (melanocytes) in the skin. Fairly common on the skin and intra-orally (much more common than congenital nevi both intra-orally and extra-orally). When present, they are usually on the HARD PALATE, or may be on the gingiva & lips. Acquired nevi are microscopically classified into 5 subtypes: 1. Intramucosal Nevus- MOST COMMON nevus in the oral cavity. Nevus cells are located in the C.T. or lamina propria of the oral mucosa. Under palpation, these nevi appear SOLID & SLIGHTLY RAISED over the mucosa surface. 2. Blue Nevus-the SECOND most common acquired nevus in the oral cavity. Congenital, painless, color is based on the deep cutaneous or subcutaneous/submucosal deposits of melanin.

3. Compound Nevus-rare in the oral cavity. Nevus cells are located at the epithelium-lamina propria interface deep in the dermis. They are raised and solid. 4. Junctional Nevus-rare in the oral cavity. Nevus cells are located at the interface between the epithelium and lamina propria. They are flat and not detected by palpation. Some regard as pre-malignant, and may undergo transformation into malignant melanoma. 5. Intradermal Nevus (common mole)-the most common lesion of skin. Nevus cells lie exclusively in the dermis. TREATMENT OF CHOICE FOR ORAL PIGMENTATIONS with unknown etiologies is CONSERVATIVE, EXICIONAL BIOPSY to rule out melanoma. Recurrence of oral nevi is very rare and malignant transformation has not been reported. BASAL CELL CARCINOMA – a MALIGNANT epithelial cell tumor that begins as a papule that enlarges peripherally, forming a central crater that erodes, crusts, & bleeds. Only found on the skin, and NEVER in the mouth due to EXCESSIVE SUN EXPOSURE (UV radiation) or to x-rays. Metastasis is rare, but the local invasion by direct extension destroys underlying and adjacent tissues. Frequently develops on exposed skin

surfaces, face (nose), & scalp in middle-aged or elderly people. Treatment: eradicate the lesion by electrodessication or cryotherapy. BASAL CELL CARCINOMA IS THE MOST COMMON SKIN CANCER, that usually appears as an ulcerated, crateriform lesion. It may look exactly like SCC, but RARELY produces metastasis. • BASAL CELL CARCINOMASA ARE NEVER FOUND IN THE MOUTH (INTRA-ORALLY). • Common in adult Caucasians with fair complexions due to sun exposure or in patients with basal cell syndrome. • Basal Cell Carcinoma has a much better prognosis than squamous cell carcinoma since it DOES NOT PRODUCE METASTASIS (it is invasive, so if you do not do surgery, the patient will die, but if treated, the patient will be cured). Ex: Frontal skin lesion in a 72year male with two years of evolution. SQUAMOUS CELL CARCINOMA (EPIDERMOID CARCINOMA-the MOST COMMON MALIGNANCY IN THE ORAL CAVITY (90%) & occurs more often in the oral cavity than any other type of cancer (90% of all malignant oral cavity neoplasms). SCC is a malignant EPITHELIAL TUMOR that is twice as prevalent in males (ages 40-65yrs).

• More common on the LOWER LIP than intra-orally. The most common intra-oral site is the lateral border & ventral surface of the TONGUE (from this site, it often metastasizes to cervical lymph nodes). Dorsal tongue surface is almost never affected. FLOOR OR MOUTH is the second most common intra-oral site, with the WORST prognosis. • Risk Factors: smoking & smokeless tobacco (main risk factor), alcohol, painful & ill-fitting dentures, and chronic inflammation. • INVASION is the most reliable histologic criterion for diagnosing oral SCC. • Treatment: surgery & radiation to remove lymph nodes in neck. • Squamous Cell Carcinoma can be RED, irregular, nonpainful lesion, or a WHITE lesion caused by sun exposure. Lasts > 1 month. • Histology: hyperchromatism, pleomorphism, atypical mitosis, dyskeratosis, alteration of nuclei-cytoplasm ratio, acanthosis, but NOT sub-epithelial cleft. • Found MAINLY in posterior LATERAL tongue border (can also be on the ventral surface), but can be on lower lip & floor of mouth. NOT common on dorsum of tongue.

Metastasis of SCC occurs via CERVICAL LYMPHATICS. Risk increases with age. Squamous Cell Carcinoma Occurs in these Head & Neck Locations: 1. Nasopharynx: caused by tobacco & alcohol. Represents < 2% of all cancers in U.S. (the least common SCC). Affects males ages 30-40yrs. Roof or lateral wall is the most common site. Signs: cervical mass, earache, sore throat, and nasal obstruction.

2. Palate: uncommon, but caused by tobacco, alcohol, and denture irritation. Represents 10% of all oral carcinomas (soft palate is more common than hard palate). Affects men > 60yrs. Signs: painful ulcer, leukoplakia, exophytic mass. 3. Oropharynx: caused by tobacco & alcohol. Represents 10% of all head & neck cancers, affecting men > 50yrs. Signs: sore throat, dysphagia, painful ulcer, cervical mass. 4. Maxillary Sinus: etiology unknown. Represents 30% of all head & neck cancers. Affects men > 40yrs. Signs: chronic sinusitis, bulging palate, teeth loosening, paresthesia in the cheek. 5. Tongue: most common site is the LATERAL BORDER, but also occurs on the ventral surface (under the tongue) (dorsum is the least likely involved). Tongue cancer causes more deaths than any other malignant lesion in other regions of the head & neck because the tongue is the MOST COMMON INTRAORAL SITE OF CANCER because it is a highly mobile organ richly endowed with lymphatics & blood vessels that facilitate metastasis (most commonly metastasizes to the CERVICAL LYMPH NODES). It very RARELY gives rise to skeletal metastasis. • Etiology: tobacco, alcohol, syphilis, Plummer-

Vinson Syndrome. • Mostly effects men > 60yrs. on the posterior lateral border and middle third of the tongue. Presents as a painless ulcer with leukoplakia and erythroplakia. 6. Lips: THE MOST COMMON SITE FOR SCC (more than intra-orally). 95% of all SCC are found on the LOWER LIP (vermillion of the lower lip) of which 90-98% of lower lip cancers occurs in MALES 60+ due to chronic sun exposure & pipe smoking. This location (lips) is etiologically related to race, complexion, pipe smoking, sunlight. It is a PAINLESS ulcer and keratotic plaque. 7. Floor of Mouth: SECOND MOST COMMON INTRAORAL SITE FOR CANCER, occuring mainly in the ANTERIOR SEGMENT on either side of the midline near salivary gland orifices. Caused by tobacco & alcohol. Pre-malignant lesions of squamous epithelium most often occur here. VERY POOR PROGNOSIS. Typically effects men 40-60yrs. It is a painless ulcer with leukoplakia and erythroplakia. 8. Buccal Mucosa: generally occurs along the PLANE OF OCCLUSION, midway anteroposteriorly. Represents 10% of all oral carcinomas, caused by tobacco, alcohol, and denture irritation. A painless ulcer and exophytic mass with leukoplakia.

9. Gingiva & Alveolar Mucosa: more common in the POSTERIOR MANDIBLE than maxilla, with posterior sites affected more than anterior sites. Represents 1015% of all oral carcinomas, caused by tobacco and alcohol. Mainly effects men 60+ years. Common on the mandibular mucosa as a painless ulcer and plaque-like or exophytic mass. Squamous Cell Carcinoma is most easily managed when found on the LOWER LIP. SCC is the most common malignant oral tumor, representing >90% of all oral malignancies. SCC is 9-10 times more common in males than females, and while it is seen in all ages, its highest incidence is after age 40yrs. SCC IS MORE COMMON ON THE LIPS than intra-orally. • 95% of lip carcinomas occur on the LOWER LIP, and are usually discovered early and only a small percentage show lymph node metastasis. The prognosis is very good. • SCC of the TONGUE is the most common INTRAORAL malignancy. The most common location is the posterior lateral border, then the posterior 1/3 (tongue base). SCC is uncommon on the dorsum or tongue tip. These lesions metastasize early, and the prognosis is not as good as lip lesions. • MOUTH FLOOR is the 2nd most common INTRA-

ORAL location of SCC. It occurs mainly in older men (especially alcoholics and smokers). These lesions metastasize early with a poor prognosis. • Treatment of choice for ORAL CANCER (SCC) IS SURGERY. SQUAMOUS CELL CARCINOMA (3 TYPES): 1. VERRUCOUS CARCINOMA- RARE form of squamous cell carcinoma (MALIGNANT) that DOES NOT METASTASIZE, but occurs in oral cavity soft tissues (mandibular mucobuccal fold, alveolar mucosa, & palate), or laryngeal cavity due to tobacco chewing, smoking, or snuff dipping. Mostly effects men 60+ years. The tumor mass has a characteristic WHITISHCAULIFLOWER or CORAL-LIKE papillary appearance. Typically, develops on the vocal cords of an elderly male who has been a heavy cigarette smoker. It is known for its slow growth pattern and welldeveloped hyperkeratotic epithelial boundaries.

Pathologically, it is a well-differentiated squamous cell neoplasm that may invade or infiltrate the borders of adjacent structures, but does not metastasize. It can transform into an invasive form of carcinoma, or co-exist with other squamous cell carcinomas. Often misdiagnosed histologically as a benign lesion. Treatment: SURGERY. Prognosis: 60-70% have 5-year survival rate. Verrucous Carcinoma is a non-aggressive, very welldifferentiated tumor that does not metastasize. COMMON IN THE ELDERLY. Can present as a diffuse, WHITE, well-demarcated, painless papillary surface mass on the UPPER (MAXILLARY) ALVEOLAR RIDGE, measuring 6x4 cm, and can be present 2-3 years before a definitive diagnosis is made. Better prognosis than carcinoma invasive. Can destroy the bone.

2. CARCINOMA IN SITU: located only inside epithelium (intra-epithelium), atypical mitosis, hyperchromatism, all epithelial layers are affected, BUT IT DOES NOT INVADE C.T. (basal membrane is intact). Better prognosis than carcinoma invasive, but must treat. Carcinoma in situ is MALIGNANT, BUT CANNOT metastasize due to lack of blood or lymphatic vessels in epithelium. Located ONLY in the epithelium which does not have blood or lymphatic vessels so it cannot produce metastasis. Ex: 60-year-old alcoholic female with red FLAT area in the mouth floor. Area is flat, asymptomatic, present for 4 months, increasing in size, but not painful. Her medical history is non-contributory. 3. CARCINOMA INVASIVE-Ex: 60-year-old alcoholic female with red area in the mouth floor. Area is flat, asymptomatic, present for 4 months, increasing in size, but not painful. Her medical history is non-contributory. Characteristics of Malignant Lesions: • Erythroplasia: lesion is totally red or speckled red and white. Red, non-ulcerated area on a mucous membrane. The texture can be normal or rough, and its size varies (some are so small and may go undetected, while large areas are conspicuous to casual inspection). There are usually no symptoms, being neither elevated nor depressed, presenting as quiet, unpretentious lesions The border may be

sharp, or blend imperceptibly into surrounding normal mucosa. Important: early carcinoma often appears as an area of erythroplasia. There are certain areas of the oral mucosa that are more prone to malignancy. Additionally, oral cancer is more often seen in people over age 40yrs. Because of this, an area of erythroplasia in a cancer prone area in a patient over 40yrs is highly suspicious of malignancy and should be biopsied on the day it is seen (especially lesions whose duration exceeds 2 weeks). • Rapid growth, ulcerated fixed lesion that bleeds on gentle manipulation. The lesion and surrounding tissue is firm to touch. On physical examination, PAINLESS induration of soft tissue suggests an invasive malignant lesion.

ODONTOGENIC ABNORMALITIES ABRASION – abnormal, PATHOLOGIC WEARING AWAY (LOSS) of tooth structure. 1. Toothbrush Abrasion-most often results in Vshaped wedges at the cervical margins in canines & premolars. Caused by using a hard bristle toothbrush and/or horizontal brushing strokes with a gritty dentifrice. 2. Occlusal Abrasion-results in flattened cusps on all posterior teeth & worn incisal edges due to chewing or biting on hard foods or objects, and chewing tobacco. ATTRITION –physiologic wearing away of enamel & dentin due to NORMAL function or mainly excessive GRINDING/GRITTING/CLENCHING teeth together (BRUXING). The most noticeable effects are POLISHED FACETS (flat incisal edges that usually develop on the linguo-incisal of maxillary canines & central incisors, and facioincisal of mandibular canines). Discolored tooth surfaces, and exposed dentin.

EROSION – CHEMICAL loss of tooth structure from NONMECHANICAL MEANS such as drinking acidic liquids (soda) or eating acidic foods. Common in BULIMCS due to regurgitated stomach acids. Affects smooth surfaces and occlusal surfaces of teeth.

INTRINSIC STAINING: can be caused by the following except DIABETES MELLITUS. • Dentinogenesis imperfecta-causes a translucent or opalescent hue, usually gray to bluish-brown. • Erythroblastosis fetalis- causes intrinsic stain that is bluish-black, greenish-blue, tan, or brown. • Porphyria-causes an intrinsic stain that is red or

brownish. • Fluorosis-causes white opacities, or light brown to brownish-black. • Pulpal injury- intrinsic stain starts pink, then becomes orange-brown to bluish-black. • Internal resorption-causes a PINKISH intrinsic stain. • Tetracyclines-intrinsic stain varies from light-gray, yellow, or tan to darker shades of gray. ANKYLOSIS –fusion of surrounding alveolar bone to a tooth root. May be initiated by an infection or trauma to the PDL. The ankylosed tooth has lost its PDL space, and is actually fused to the alveolar process of bone. There is change in the continuity of the occlusal plane caused by the continued eruption of non-ankylosed teeth & growth of the alveolar process. GEMINATION (TWINNING)–a division of a single tooth germ by invagination causing incomplete formation of two teeth (usually the incisors). Incomplete splitting of a tooth germ. GOMPHOSIS – a type of fibrous joint where a conical process is inserted into a socket-like portion (i.e. styloid process in the temporal bone, or the teeth inserted into the dental alveoli). FUSION-the joining of two normally separated tooth buds to form A SINGLE, LARGE, WIDE CROWN (results in one

less tooth in the arch). Most commonly affects ANTERIOR INCISORS and can occur in the primary or permanent dentition. CONCRESCENCE – a condition where only the CEMENTUM of two or more teeth are joined. DILACERATION-a sharp bend or curve in a root due to trauma during tooth development. TAURODONTISM (“BULL-LIKE”)-found usually in MOLARS; the tooth body and pulp chamber are enlarged vertically at the root’s expense causing an apical shift of the pulpal floor and tooth furcation down the tooth root (large pulp chambers and short roots) causing teeth to look “bull-like”. Caused by failure or late invagination of Hertwig’s epithelial root sheath that is responsible for root formation.

DENS-IN-DENTE (Dens Invaginatus)-”tooth within a

tooth”, caused by a deep invagination of the enamel organ Hertwig’s epithelial root sheath during formation. Most commonly associated with a MAXILLARY LATERAL INCISOR.

MESIODENS – the most common SUPERNUMERARY TOOTH (extra tooth) usually appearing singly, or in pairs as a small tooth with a cone-shaped crown and short root visible between vital PERMANENT MAXILLARY CENTRALS on a radiograph. It may be erupted, impacted, or inverted. Appears situated in the maxilla near the midline and almost always posterior to normal central incisors. Thus, many mesiodens are bypassed by the permanent incisors still erupt into their normal position in the arch regardless.

If the mesiodens does not interfere with the normal eruption of the permanent maxillary central incisors, you can wait until the child is 6-8 years old to extract the mesiodens nonsurgically if it erupts, or surgically if it does not erupt. HYPERCEMENTOSIS – excessive (abnormal) CEMENTUM FORMATION around or on root surface after tooth eruption (often confined to the apical half of the root, but may involve the entire root). THICKENING OF CEMENTUM (ROOT TISSUE). Caused by trauma, metabolic dysfunction, chronic periapical inflammation, or when a tooth has lost its antagonist. Hypercementosis is merely a dental anomaly, but can be seen in ACROMEGALY & PAGET’S DISEASE. • Mainly affects VITAL teeth (MAINLY PREMOLARS), then first & second molars.

• Produces no significant clinical signs or symptoms, but seen radiographically as a BULBOUS ENLARGEMENT with a surrounding continuous/unbroken periodontal membrane space and normal lamina dura. Radiographically, with Paget’s Disease, there is complete absence of the periodontal membrane space & lamina dura that surrounds the hyperplastic cementum.

ENAMEL HYPOPLASIA–enamel developmental defect due to INCOMPLETE FORMATION of the enamel matrix. ENAMEL IS HARD, BUT, THIN & DEFICIENT IN AMOUNT, resulting in incomplete formation of the enamel matrix with a deficiency in the cementing substance. EH affects deciduous (primary) and permanent teeth, and is usually caused by illness or injury during tooth

formation, or due to a genetic disorder (genetic forms of EH are considered types of amelogenesis imperfecta). If only one permanent tooth is affected, it is usually caused by physical damage to the replaced primary tooth. WHITE & BROWN DEFECTS ON TOOTH SURFACE. • Clinical Features: lack of contact between teeth, rapid breakdown of occlusal surfaces, yellowish-brown stain that appears due to EXPOSED DENTIN.

ENAMEL HYPOCALCIFICATION-a hereditary dental defect where the ENAMEL IS SOFT & UNDERCALCIFIED, yet normal in quantity due to defective maturation of ameloblasts (a defect in the mineralization of the formed matrix). Teeth are chalky, surfaces wear down rapidly, and a yellowish-brown stain appears due to underlying EXPOSED DENTIN. Affects deciduous & permanent teeth.

AMELOGENESIS IMPERFECTA – an inherited hereditary ECTODERMAL DEFECT transmitted as a dominant trait that affects the deciduous & permanent dentition, causing enamel to be soft, thin, and yellow due to EXPOSED DENTIN through the thin enamel layer. Teeth are easily damaged and susceptible to decay. Crowns may or may not show discoloration. If discoloration is present, it varies depending on the type of disorder, ranging from yellow to dark brown. • Open contacts between teeth and occlusal surfaces/incisal edges are often severely abraded. • Radiographic findings are often distinctive & pathognomonic. When enamel is totally absent, the radiographic appearance makes the diagnosis obvious. When some enamel is present, thin radiopaque coverings on the proximal surfaces are visible. When anatomic crown forms are normal or near normal, the softness of the defective enamel may not be easily distinguished from dentin.

• Dentin, pulp, & cementum are NOT affected by AI (unlike dentinogenesis imperfecta). Exception: AI will only show pulp obliteration if there is advanced abrasion with secondary dentin formation.

3 TYPES OF AMELOGENESIS IMPERFECTA: 1. (Type 1) Hypoplastic AI: enamel has not formed to full normal thickness, or may be completely absent on newly erupted developing teeth due to defective formation of the enamel matrix. 2. (Type 2) Hypomaturation AI: enamel can be pierced by an explorer tip under firm pressure and chipped away from normal-appearing dentin. Characterized by IMMATURE CRYSTALLITES. 3. (Type 3) Hypocalcified AI: quantity of enamel is normal, but so soft it can be removed during a prophylaxis due to the defective

MINERALIZATION of the enamel matrix. DETINOGENESIS IMPERFECTA (HEREDITARY OPALESCENT DENTIN)– RARE disorder found in only 1:7,000 children. An inherited/hereditary MESODERMAL DEFECT OF DENTIN that only affects deciduous & permanent teeth. Teeth have an OPALESCENT HUE. • Clinical Features: teeth have amber, gray, or purple opalescence/translucence or discoloration, pulp chambers may be completely obliterated due to continued deposition of dentin, crowns are short & bulbous, with narrow roots. Enamel can chip away within 2-4yrs after eruption, exposing the dentin which is soft and wears away rapidly. Enamel is structurally and chemically normal. DI is usually easily detected and identified, as teeth exhibit a translucent or opalescent appearance. Abnormal constriction at the CEJ is another clinical feature detected by exploration.

DENTINOGENESIS IMPERFECTA (3 TYPES): 1. Type 1: dentin abnormality occurs in patients with

Osteogenesis Imperfecta, characterized by blue sclera or history of bone fractures. 2. Type 2: the most common; only a dentin abnormality exists. NO BONE INVOLVEMENT. 3. Type 3 (Brandywine Type): like Type 2, only a dentin abnormally exists. There are clinical & radiographic variations that include multiple pulp exposures in the deciduous (primary) dentition. DENTIN DYSPLASIA (ROOTLESS TEETH) – a hereditary disease transmitted as an autosomal dominant trait. Clinical Features: normal enamel, atypical dentin, pulpal obliteration, defective root formation, tendency toward multiple periapical radiolucencies and early exfoliation of teeth. Not associated with any systemic C.T. disorder. 2 Types of Dentin Dysplasia: 1. Type I (Radicular)-the more common type involving both dentitions. Normal morphology and color (deciduous & permanent teeth). Mobile teeth, premature exfoliation, short roots (rootless teeth), obliterated pulp chambers (deciduous), crescentshaped pulpal remnant (permanent), periapical radiolucencies, coronal dentin is normal, but root dentin is disoriented. PULPAL OBLITERATION BY EXCESS DENTIN “chevron” shaped pulp chambers.

2. Type II (Coronal)-involves both dentitions (but coronal dentin is normal). Deciduous teeth exhibit bluish-gray opalescent appearance, obliterated pulp chambers, amorphous & atubular dentin in the radicular portion of the teeth. Permanent teeth exhibit a normal clinical appearance, thistle-tube pulp chambers & stones, and true denticles. Pulpal obliteration of primary teeth, and pulp stones in permanent teeth. ANODONTIA – a developmental abnormality characterized by TOTAL ABSCENSE of teeth. Two Forms: 1. Complete/True Anodontia-a rare condition where ALL TEETH are missing. May involve the primary & permanent dentitions, and is usually associated with hereditary Ectodermal Dysplasia. 2. Partial Anodontia (Congenitally Missing Teeth)-a common condition usually affecting maxillary & mandibular 3rd molars (affects mainly maxillary 3rd molars), maxillary laterals, & mandibular 2nd premolars. Rule: if only one or a few teeth are missing, the absent tooth is the MOST DISTAL tooth (if a molar, then the 3rd molar is missing; if a premolar, then the 2nd premolar). OLIGODONTIA – congenital absence of MANY (but not all) teeth.

HYPODONTIA – absence of only a FEW teeth

WHITE LESIONS ORAL CANDIDIASIS (“THRUSH” OR “MONILIASIS”) – a FUNGAL infection of the oral cavity or vagina caused by a Candida species (usually Candida albicans) causing an inflammatory, pruritic infection with a thick, white discharge. Appears diffuse, curly or velvety white mucosal plaques on the cheeks, palate, and tongue that CAN BE WIPED OFF, leaving a red, raw, or bleeding surface. The most common symptoms are discomfort and burning of the mouth & throat, and altered taste. • Candida is a yeast-like fungi and normal inhabitant of the oral cavity & vaginal tract, but is normally held in check by indigenous bacteria of these areas. Factors that stimulate Candida growth are extended use of antibiotics (antibiotics prescribed for a dental infection), steroids, diabetes, pregnancy, or vitamin deficiency (iron, folate, B12, zinc). • Very common in patients on long-term antibiotic or chemotherapy, and immunosuppressed patients (AIDS). • Treatment: topical with LOZENGES (Trouches) & mouth rinses (NYSTATIN is most widely used).

ACUTE PSEUDOMEMBRANOUS CANDIDIASIS – the most common oral candidiasis, usually found on the buccal mucosa, tongue, and soft palate. Oral cytology smears diagnose acute pseudomembranous candida by revealing budding organisms with branching pseudohyphae. ANGULAR CHEILITIS (PERLECHE) – any chronic inflammatory lesion that occurs at the labial commissure (corners of mouth) due to unknown cause. Generally associated with LOSS OF VERTICAL DIMENSION in elderly patients. Mouth corners are painful, irritated, red, cracked, and scaly. Candida albicans fungus (Thrush) may grow in the corners of the mouth, keeping them sore. • Predisposing Factors: Candida albicans infection, loss of inter-maxillary distance (decreased vertical dimension), trauma to the labial commissure due to prolonged dental treatment, & vitamin deficiencies (especially riboflavin or thiamine). • Treatment: NYSTATIN will eliminate the FUNGAL infection only.

ACTINIC CHEILITIS (SOLAR CHEILITIS/FARMER’S LIP) – a PRE-MALIGNANT condition caused by chronic and excessive exposure to the UV sunlight radiation. A counterpart of actinic keratosis of the skin, and can also develop into squamous cell carcinoma. There is thick, WHITISH discoloration of the lip at the border of the lip and skin, and loss of the usually sharp demarcation between the red of the lip and normal skin (vermillion border). May lead to SCC, so it must be treated.

LEUKOEDEMA – a condition that mimics leukoplakia as it appears to be a WHITE PATCH, but is a just a VARIANT OF NORMAL MUCOSA. Varies from a filmy opalescence of the mucosa in the early stages, to a more definite grayish-white cast with a coarsely wrinkled surface in later stages. Usually occurs BILATERALLY and along the occlusal line in the bicuspid and molar region. Diagnostically, one can stretch the tissue and the white disappears (Important: Leukoplakia DOES NOT DISAPPEAR WHEN STRETCHED). NO TREATMENT REQUIRED.

• Leukoedem’s white appearance is caused by water inside spinous cells, causing light to reflect back as whitish. • Differential Diagnosis: leukoplakia, white sponge nevus, & benign intraepithelial dyskeratosis.

WHITE SPONGE NEVUS (FAMILIAL WHITE FOLDED DYSPLASIA)–an often genetic and benign BUCCAL MUCOSAL ABNORMALITY (often mistaken for leukoplakia. Characterized by WHITE (pearly or opalescent), CORRUGATED THICK SOFT FOLDING OF THE BUCCAL MUCOSA (bilaterally). Can also occur on the labial mucosa, alveolar ridge, and floor of mouth. Almost NEVER found on the gingival margin & dorsal of tongue. NO TREATMENT NECESSARY.

LEUKOPLAKIA – a PREMALIGNANT LESION WHITE PATCH or plaque on the oral mucosa that DOES NOT RUB OFF (UNLIKE CANDIDA) and cannot be assigned as any specific disease. Possible etiologic factors are tobacco (PIPE SMOKING), alcohol, oral sepsis, and chronic

irritation. Most often due to TOBACCO and chronic irritation (ill-fitting denture, rough filling, cheek biting). May be present for many months in a heavy pipe-smoker. Pipe-smoking is the most important predisposing etiologic factor in leukoplakia. More common in elderly men, and DOES NOT DISAPPEAR WHEN STRETCHED. Mouth floor, tongue, and lower lip are the regions at greatest risk for carcinoma occurring in leukoplakia. Speckled Leukoplakia-has mixed red and white areas. • Leukoplakia is a slow developing change in a mucous membrane characterized by thickened, white, firmly attached patches, that are slightly raised & sharply circumscribed. Lesions on the mouth floor and base of the tongue are the most aggressive. Most display no dysplasia, but can be pre-malignant so MUST BIOPSY. • Treatment: BIOPSY. Due to chance of malignant transformation, ALL LEUKOPLAKIAS MUST BE BIOPSIED & COMPLETELY EXCISED (if untreated, some progress to carcinoma). Although less common than leukoplakias, erythroplakias have a much greater malignant potential. Incisional Biopsy is indicated for a 3cm area of leukoplakia of the soft palate. Biopsy are not required for papillary fibroma, exostosis, Fordyce’s granules, or Hemangiomas as these are benign. • Any white or red lesion that does not disappear itself in 2 weeks must be re-evaluated and considered for BIOPSY

to obtain a definitive diagnosis.

• Carcinoma in situ is a term applied to mucosal lesions that resemble leukoplakia in all respects except dysplasia is very pronounced and involves almost all epithelial layers. It shows no tendency to invade or metastasize to other tissues. Exhibits all of the histologic characteristics of malignancy (pleomorphism, hyperchromatism, abnormal mitoses, anaplasia), but

DOES NOT show invasiveness or extension into adjacent structures. • Clinical Differential Diagnosis of White Patch: leukoplakia, lupus erythematosus, leukoedema, white sponge nevus, chemical or thermal burn, candidiasis, lichen planus, & migratory glossitis/stomatitis. HAIRY LEUKOPLAKIA-an unusual BENIGN form of leukoplakia seen mainly in people with HIV/AIDS or the immunocompromised caused by EPSTEIN-BARR VIRUS (HSV-4). Fuzzy, hairy white patches mainly on the tongue (may resemble THRUSH caused by Candida). Treatment: systemic anti-viral therapy (Acyclovir) or topical therapy with retinoic acids (tretinoin), or ablative therapy.

HAIRY

TONGUE–HYPERTROPHY

of

FILIFORM

PAPILLAE (NO TASTE BUDS). A BENIGN condition of the tongue dorsum which is FURRY due to elongated filiform papillae. Color varies from yellowish-white to brown or black. This is not Hairy Leukoplakia, but is just DISCOLORATION of the DORSUM tongue surface, elongation & hyperkeratosis of FILIFORM PAPILLAE ON TONGUE’S DORSUM SURFACE. 4 Types of Tongue Papillae: taste buds are present only on fungiform, circumvallate, & foliate papillae. 1. Filiform papillae-the most numerous. Small cones in “v”-shaped rows paralleling the sulcus terminalis, characterized by no taste buds and increased keratinization. NO TASTE BUDS. 2. Fungiform papillae-scattered among filiform papaillae. Flattened, mushroom-shaped, and found mainly at the tongue tip and lateral margins. HAVE TASTE BUDS. 3. Circumvallate papillae-the LARGEST, LEAST NUMEROUS papillae. Circular-shaped arranged in an inverted “v”-shaped row toward the back of the tongue. Associated with ducts of Von Ebner’s glands. HAVE TASTE BUDS. 4. Foliate papillae-found on the lateral margins as 3-4 vertical folds. HAVE TASTE BUDS.

Hairy tongue can be WHITE, GREEN, BROWN, OR BLACK completely covering the tongue dorsum as filiform papillae are stained and discolored with DEBRIS the patient eats or smokes. Ex: dorsal surface of tongue of a smoker exhibits elongated, brownish filiform papillae. Can present as a bright green, flat area on the tongue dorsum, with very poor dental status & oral hygiene. The green area is probably DEBRIS. Etiology: overgrowth of fungal microorganisms due to smoking or poor oral hygiene (brush tongue).

BENIGN MIGRATORY GLOSSITIS (GEOGRAPHIC TONGUE) OR (ERYTHEMA MIGRANS) A HARMLESS, USUALLY PAINLESS (maybe slight burning), COMMON condition due to desquamation of FILIFORM papillae (no taste buds). One or more irregular-shaped patches on the tongue exist. The center area is redder then the rest of the tongue, and edges of the patch are whitish color. These patches appear and remain for a short time, heal, then reappear at another site. The patches usually do not respond to treatment, but disappear spontaneously. Geographic Tongue often occurs with Fissured Tongue.

FISSURED TONGUE (“SCROTAL TONGUE”) –a DEEP, usually asymptomatic (maybe painful if infected with Candida Albicans) MEDIAN FISSURE with laterally radiating grooves that vary in number, but are usually symmetrically arranged across the DORSUM (TOP) OF THE TONGUE. Rare in children, but incidence increases with age. Found in Melkersson-Rosenthal Syndrome (along with Cheilitis Granulomatosum & Facial Nerve Paralysis).

STOMATITIS NICOTINA (“PIPE-SMOKER’S PALATE” OR NICOTINIC STOMATITIS): the initial clinical response is generalized palatal erythroplakia then becomes a white hyperkeratotic area with small red dots. Related to pipe smoking (tobacco), occurs ONLY ON THE PALATE, and mainly affects males. The palate is initially red & inflamed, then develops a diffuse, grayish-white, thickened, multi-nodular popular appearance with a small red “spot” in the center of each tiny nodule. This “spot” corresponds to orifices of palatal salivary gland ducts. Treatment: None, except to stop smoking. Not usually premalignant. • Found ONLY in the palate (palate is leathery white and full of keratin (hyperkeratosis with RED DOTS (inflamed minor salivary glands). The only lesion produced by tobacco that is not cancerous. Usually a white, generalized area with red dots on the hard palate that are

PAINLESS & non-indurated. White areas with multiple red dots (inflamed salivary glands in the palate).

LICHEN PLANUS – oral lesion mainly on the BUCCAL MUCOSA appearing as white or grayish-white striae arranged in a lace-like pattern (Wickman’s Striae) often bilaterally & symmetrically distributed, and usually asymptomatic, but may sometimes cause a burning sensation. A fairly common inflammatory disease, of unknown cause, but may be autoimmune. It usually affects the skin, mouth, or both. May also be on the tongue, lips, hard palate, & gingiva, but MAINLY BUCCAL MUCOSA. • Microscopic Features: hyperparakeratosis with thickening of the granular cell layer, development of a “saw-tooth” appearance of rete pegs, degeneration of the basal cell layer, and infiltration of inflammatory cells into the sub-epithelial layer of C.T.

• Affects women slightly more than men, and occurs most often in middle-aged adults. • Treatment: intra-oral lesions respond to TOPICAL STEROIDS. TWO ADDITIONAL FORMS OF LICHEN PLANUS: 1. Bullous Lichen Planus-fluid-filled vesicles project from the buccal mucosa surface. 2. Erosive Lichen Planus-intensely red or raw-appearing lesions that resemble desquamative gingivitis when they involve the gingiva. FORDYCE’S GRANULES (ECTOPIC SEBACEOUS GLANDS)– found in the oral mucosa, present in > 75% of adults. Usually appear as yellow or yellow-white submucosal clusters that are normal. Rice-like or cauliflower-like whitish-yellow or white asymptomatic papules 1-3mm usually found BILATERALLY on the

buccal mucosa, upper lip vermillion, mandibular retromolar pad, and tonsillar area (but can be on any oral surface). Surrounding mucosa looks normal, but they can remain constant throughout life.

BLOOD DISEASES PURPURA –HEMORRHAGES in the skin & mucous membranes that cause the appearance of purplish spots or patches. Tooth extractions are contraindicated due to potential excessive bleeding. THROMBOCYTOPENIC PURPURA (Werlhof’s Disease)a bleeding disorder characterized by a deficiency in the number of platelets, resulting in multiple bruises, PETECHIAE, & hemorrhage into the tissues. May be caused by heparin (warfarin) therapy. Oral manifestations are severe/profuse gingival hemorrhage, & palatal petechiae. Petechie can also be caused by streptococcus.

THROMBOCYTOPENIA-dominated clinically by PETECHIAE cutaneous bleeding, intra-cranial bleeding, and oozing from mucosal surfaces. Characterized by decreased platelet count causing prolonged bleeding time. THE MOST COMMON CAUSE OF BLEEDING DISORDERS. IDIOPATHIC THROMBOCYTOPENIC PURPURA (ITP)-a bleeding disorder due to a deficiency in the number of platelets causing multiple bruises, petechiae, & hemorrhage into the tissues. ITP is a common complication of Leukemia, Aplastic Anemia, & Aggressive Cancer Chemotherapy. • Bleeding Time is abnormally prolonged in ITP. THROMBOTIC THROMBOCYTOPENIC PURPURA (TTP)a severe and frequently fatal form characterized by thrombocytopenia, hemolytic anemia, renal insufficiency, fever, and neurologic abnormalities. Low platelet count in the blood and thrombosis in the terminal arterioles and capillaries of many organs. • Purpura-a condition characterized by hemorrhages in the skin and mucous membranes that result in the appearance of purplish spots or patches. • Petechiae-small pinpoint hemorrhages flush with the skin surface. • Ecchymosis (bruise)-a discoloration of an area of

skin or mucous membranes due to the extravasation of blood into the subcutaneous tissues due to trauma or hemorrhage. PROLONGED BLEEDING TIME (Thrombocytopenia) CONDITIONS: • A patient taking Dicumarol-inhibits formation of prothrombin in the liver. • A patient taking Heparin-acts as an antithrombin by preventing platelet aggregation. • Idiopathic Thrombocytopenic Purpura (ITP)-often associated with leukemia; a decrease in the number of platelets. • Von Willebrand’s disease-deficiency of vWF (von Willebrand’s factor); results in impaired platelet adhesion. • Long-term treatment with aspirin; aspirin is a cyclooxygenase inhibitor; results in impaired production of thromboxanes, important in platelet aggregation. AGRANULOCYTOSIS – an abnormal blood condition due to a severe reduction in the number of granulocytes (NEUTROPHILS) caused by ingesting a drug. It is an acute toxic effect/condition characterized by pronounced

LEUKOPENIA with a severe reduction in the number of polymorphonuclear leukocytes (PMNs). It is a toxic effect of certain anti-thyroid drugs (propylthiouracil, methimazole, carbimazole). Can occur at any age, but is more common in adult females. • WBC count is < 2,000 with almost complete absence of PMN neutrophils (polymorphonuclear leukocytes). Normal WBC count is 4,000-10,000 and neutrophils are usually 50-70%. • Agranulocytosis begins with a high fever, chills, and sore throat. The patient suffers from malaise, weakness, and prostration. Skin appears pale and anemic. The most characteristic feature is the presence of an INFECTION IN THE ORAL CAVITY. Signs and symptoms develop very rapidly (within a few days), and death may occur soon after. • Oral lesions (necrotizing ulcerations) are an important phase of the clinical aspects, appearing as necrotizing ulcerations of the oral mucosa of the GINGIVA & HARD PALATE. These lesions are ragged necrotic ulcers covered by a gray membrane. One important aspect is that there is little or no apparent inflammatory cell infiltration around the lesions. Histologically, the ulcerated lesions do not exhibit polymorphonuclear reaction due to the bacteria in the tissues.

• Treatment: ELIMINATE THE CAUSATIVE DRUG. Administer antibiotics to control the infection. SICKLE-CELL ANEMIA (SICKLE-CELL DISEASE) – a chronic, usually fatal inherited form of anemia marked by crescent-shaped red blood cells, characterized by fever, leg ulcers, jaundice, and episodic pain in the joints due to the production of abnormal hemoglobin (Hemoglobin S) due to a genetic defect. N2O administration is CONTRAINDICATED. • Primarily affects African-Americans (especially females), and usually manifests before age 30yrs. • SCA Signs: (patient is weak, short of breath, easily fatigued, and muscle and joint pain are common). • Dental radiographs show ENLARGED BONE MARROW (MEDULLARY) SPACES because of loss of many bony trabeculae (but the trabeculae that are present are abnormally prominent). Occasionally, osteosclerotic areas are noted in the midst of large radiolucent marrow spaces. However, the lamina dura & teeth are unaffected.

LEUKEMIAS-cancers of mainly WHITE BLOOD CELLS with most of unknown cause. LEUKEMIA involves uncontrolled proliferation of leukocytes causing a diffuse and almost total replacement of the red bone marrow with leukemic cells. Leukemia is classified by the dominant cell type and by duration from onset to death. It can modify the inflammatory reaction. While the cause is unknown, these agents are closely associated with leukemia development: 1. Ionizing radiation: increased incidence of leukemia among atomic bomb survivors & radiologists, usually myelogenous. 2. Viruses: shown to cause leukemia in fowl and rodents. Herpes-like viral particles have been cultured from patients with various types of leukemia and leukemic patients have high antibody titer to the Epstein-Barr Virus. 3. Genetic Mutations: Philadelphia chromosome

(translocation of chromosome material between chromosomes 22 and 9) is present in 90% of patients with Chronic Myelogenous Leukemia (CML). Also higher incidence of acute leukemia in patients with Down Syndrome (Mongolism) in which there is Trisomy 21. 4. Other: chronic exposure to benzol, aniline dyes, and related chemicals. All LEUKEMIAS occur in an ACUTE or CHRONIC form, but 50% are ACUTE. Acute Leukemia is the most common malignancies of the pediatric age group (under 20 years). ORAL MANIFESTATIONS OF LEUKEMIA: • Acute and chronic leukemias all cause ORAL LESIONS. The most common is seen in acute monocytic leukemia (a subtype of acute myeloid leukemia), where 80% of patients exhibit GINGIVITIS, GINGIVAL HYPERPLASIA, PETECHIAE, & HEMORRHAGE. Spontaneous gingival bleeding in acute leukemia is due to THROMBOCYTOPENIA. CLASSES OF LEUKEMIA: All Leukemia occurs in an ACUTE or CHRONIC form: 1. Myelogenous Leukemia-involves granulocytes & megakaryocytes. Philadelphia Chromosome and low levels of leukocyte alkaline phosphatase are common

findings. Massive splenomegaly is characteristic. CML is one of a group of diseases called myeloproliferative disorders. Other diseases in this class are Polycythemia Vera, Myelofibrosis, & Essential Thrombocythemia. CML is characterized by uncontrolled proliferation of immature granulocytes (but is the least malignant leukemia). CML accounts for 20% of all adult leukemias (typically affects middle-aged individuals). Although uncommon, CML may occur in younger individuals. • CML Clinical Signs: spongy bleeding gums, fatigue, fever, weight loss, moderate splenomegaly, joint/bone pain, and repeated infections. • Leukemic cells in 95% of CML patients have a Philadelphia Chromosome which is the result of a reciprocal translocation between chromosomes 9 and 22 which result in a shortened chromosome 22. • Mean survival time with CML is 4 years with death due to hemorrhage or infection. 2. Lymphocytic Leukemia-involves lymphocytes. Chronic Lymphocytic Leukemia runs a variable course (older patients may survive years even without treatment). Lymph node enlargement is the main pathologic finding. May be complicated by autoimmune

hemolytic anemia. • Acute lymphocytic (lymphoblastic)-is largely confined to children (the most common leukemia in children). Lymph node enlargement is common. In 75% of cases, the lymphocytes are not B nor T cells, but are “null” cells. Bone and joint pain are common. 3. Monocytic Leukemia-involves monocytes. ORAL LESIONS ARE COMMON and may be the initial manifestations of the disease. Gingivitis, gingival hemorrhage, generalized gingival hyperplasia, petechiae, ecchymosis, and ulcerations. Chronic Monocytic Leukemia is VERY RARE. Chronic Leukemia (Clinical Features): • Insidious onset (slow) with weakness and weight loss. May be detected during an examination for another condition (i.e. anemia, unexplained hemorrhages, or recurrent intractable infections). • Organ involvement is similar to acute leukemia: skin is often involved, and may manifest as petechiae or ecchymosis, recurrent hemorrhages and bacterial infections are common (anemia). ACUTE LEUKEMIA – has an abrupt onset of a few

months (not insidious) with fever, weakness, malaise, severe anemia, and generalized lymphadenopathy. Untreated patients dies within 6 months (usually due to brain hemorrhage or superimposed bacterial infection). With intensive chemotherapy, radiation, and marrow transplants, remissions lasting up to 5 years may be obtained. • Clinical Features: severe anemia, hemorrhages, and slight enlargement of the lymph nodes or spleen. Primary organs involved are bone marrow, spleen (splenomegaly), & liver (hepatomegaly). Petechiae & ecchymosis in skin and mucous membranes, hemorrhage from various sites, and bacterial infections are common. • Lab Findings: leukocytosis 30,000-100,000/mm3 with immature forms (myeloblasts & lymphoblasts) predominating; anemia and thrombocytopenia, prolonged bleeding and coagulation times, tourniquet test is usually positive. ACUTE MYELOID/MYELOGENOUS LEUKEMIA (AML)malignant BONE MARROW disease where hematopoietic precursors are arrested in an early stage of development (has an abrupt onset). AML is distinguished from other related blood disorders by the presence of > 30% MYELOBLASTS in the blood and/or bone marrow that contain AUER RODS in their cytoplasm. THE MOST MALIGNANT LEUKEMIA. More common in adults.

POLYCYTHEMIA VERA (PRIMARY ERYTHEMIA): A CHRONIC myeloproliferative condition of TOO MANY ERYTHROCYTES (RBC) produced in the circulation due to tumorous abnormalities in tissues that make RBC, making BLOOD TOO THICK to pass easily through small blood vessels in the body. This leads to CLOT FORMATION & BLOCKAGE of vessels causing a STROKE (cerebrovascular accident). Usually accompanied by leukocytosis. Splenomegaly, due to vascular congestion, occurs in 75% of patients. Usually occurs within in ages 2080yrs., with age 60yrs being the mean age of onset. • PV Clinical Features: headache, weakness, weight loss, pruritus, hemorrhage and thrombosis. • PV Oral Manifestations: oral mucous membranes (especially gingiva & tongue) appear deep purplish-red, gingiva is swollen and bleeds easily, and submucosal petechiae (purplish spots), ecchymosis (petechiae, but bigger), and hematomas are common. • Secondary Polycythemia-an increase in the total number of erythrocytes (RBC) due to another condition (i.e. chronic tissue hypoxia of advanced pulmonary disease, high altitude (Osker’s Disease), or secretion of erythropoietins by certain tumors. PLUMMER-VINSON SYNDROME - a rare disorder associated with severe and chronic iron-deficiency

ANEMIA occurring mainly in women ages 30-40yrs. Due to the predisposition to develop carcinoma of the oral mucous membranes, it is essential to diagnose early, so treatment can be given ASAP (administer iron, vitamin B complex, and a high protein diet). • Systemic Symptoms: weakness, pallor, difficulty swallowing (dysphagia) due to esophageal stricture or web, and difficulty breathing (dyspnea). • Oral Symptoms: angular stomatitis, smooth, red, painful tongue with papillae atrophy. APLASTIC ANEMIA – a form of anemia where bone marrow’s capacity to produce RBCs is defective. THE MOST SERIOUS & LIFE-THREATENING blood dyscrasia associated with drug toxicity. 1. Primary Anemia-unknown cause, affects young adults and is usually fatal. Symptoms: pallor, weakness, malaise, dyspnea (difficulty breathing), headache, and vertigo. Oral symptoms: spontaneous bleeding, bruising (petechiae), and gingival infections. 2. Secondary Anemia-caused by exposure to toxic agents (i.e. radiation, chemicals, or drugs like Chloramphenicol). Occurs at any age, with the same symptoms as primary anemia. Prognosis is good after the cause is removed.

PERNICIOUS ANEMIA-a disease caused by an inability to absorb adequate amounts of vitamin B12 from the digestive tract. A relatively common, chronic, progressive megaloblastic anemia, caused by the lack of secretion of INTRINSIC FACTOR in normal gastric juice (intrinsic factor is necessary for adequate vitamin B12 absorption, which is required for erythrocyte maturation). As a result fewer than normal RBC are produced. Lack of intrinsic factor, causes poor B12 absorption and leads to low RBC production. Diagnosed by the Schilling Test to determine the body’s ability to absorb an oral dose of B12. • Characterized by a Triad of Symptoms: weakness, SORE PAINFUL TONGUE (ATROPHIC GLOSSITIS), and tingling of extremities.

THALASSEMIA MAJOR & MINOR –hemolytic anemias

caused by a genetic defect, characterized by a low level of erythrocytes & abnormal hemoglobin. Oral Manifestations: oral mucosa may exhibit anemic pallor, flaring of maxillary anterior teeth with malocclusion. ERYTHROBLASTOSIS FETALIS (HEMOLYTIC DISEASE OF NEWBORN) – a severe hemolytic disease of the fetus or newborn caused by the production of maternal antibodies for fetal RBC. It usually involves Rh factor incompatibility between the mother and fetus. Characterized by excessive destruction of erythrocytes due to an antigenantibody reaction in the infant’s bloodstream resulting from the placental transmission of maternally formed antibodies against the incompatible antigens of the fetal blood. In Rh factor incompatibility, the hemolytic reaction only occurs when the mother is Rh (-) and infant is Rh (+). • Oral Manifestations: teeth have a green, blue, or brown hue due to deposition of blood pigment in enamel and dentin. ENAMEL HYPOPLASIA may occur, affecting the incisal edges of anterior teeth and middle portion of the deciduous canine and first molar crown. ERYTHROCYTE SEDIMENTATION RATE (ESR)-a nonspecific test that only monitors the PROGRESSION OF DISEASE. ESR is the rate at which RBC settle out in a tube of unclotted blood, expressed in mm/hour. Blood is collected in an anticoagulant and allowed to sediment in a

calibrated glass column. At the end of one hour, the lab measures the distance the erythrocytes have fallen in the tube. The speed that the RBC fall to the bottom of the tube reflects the degree of inflammation • Elevated sedimentation rates are not specific for any disorder, but indicate the presence of inflammation. Inflammation causing an alteration of blood proteins that make RBC aggregate, becoming heavier than normal. ESR rises during inflammation, tissue degeneration, suppuration, and necrosis. Certain non-inflammatory conditions (i.e. pregnancy) are also categorized by high sedimentation rates. WISKOTT-ALDRICH SYNDROME – affects ONLY BOYS and causes eczema, low platelet count, and a combined deficiency of B and T lymphocytes that lead to repeated infections. Children who survive past age 10 usually develop cancers (lymphoma and leukemia). EMBOLUS-a mass (detached BLOOD CLOT = thrombus), an air bubble or a foreign body that moves within a blood vessel to lodge at a site distant from its place of origin. The danger from an embolus is it can lodge in vascular beds of vital organs, occluding blood flow and causing infarction. • The most common source of a Pulmonary embolism is thrombophlebitis (a thrombus formed within a vein). Femoral vein is the common source of

origin of the thrombus which then occludes a blood vessel in the lung. GINGIVAL HYPERPLASIA: diffuse soft tissue overgrowth affecting both jaws with a pink-to-red color and firm consistency from the mucogingival junction to the free gingival margin. • Local Factors: can be caused by poor oral hygiene, malocclusion, tooth malformations, caries, faulty restorations, allergens, chronic mouth breathing. • Systemic Factors: diabetes, hormone changes (puberty and pregnancy), immunoincompetence, gingival fibromatosis, Wegener Granulomatosis, aplastic anemia, leukemia, scurvy, and drugs PHENYTOIN (DILANTIN), VALPROIC ACID, CYCLOSPORINE, & CALCIUM CHANNEL BLOCKERS (CCBs) (ex: NIFEDIPINE for hypertension and cardiovascular diseases). Other CCB that can cause gingival hyperplasia (Amlodipine, Felodipine, Verapamil, and Diltiazem). • Develops 1-9 months after Calcium Channel Blocker administration. • A biopsy of constant hyperplasia can rule out a systemic disease like LEUKEMIA. • Gingivectomy can treat, but it will recur with

continued use of most CCBs. Discontinuing the CCB usually restores the gingiva to normal size with in one month. • Histologic Examination: epithelial hyperplasia with acanthosis, parakeratosis, and elongated, slender epithelial RETE PEGS and dense C.T. with foci of chronic inflammation.

NEUROLOGIC & MUSCLE DISORDERS Trigeminal Neuralgia (Tic Douloureux) – an excruciating, PAINFULL illness where the person feels sudden stab-like pains in the face that usually last only moments, but are among the most severe pain humans can feel. This pain is provoked by touching a “trigger zone” near the nose or mouth, caused by degeneration of the trigeminal nerve or by applying pressure to the nerve. Can affect any of the trigeminal’s three branches (V1, V2, or V3). The momentary bursts of pain recur in clusters, lasting many seconds. Paroxysmal episodes of the pains may last hours. • Drug Treatment: drug of choice is Carbamazepine (Tegretol), an analgesic and anticonvulsant that usually relieves the pain within 48hrs. Tegretol is also prescribed to treat certain seizure disorders. MULTIPLE SCLEROSIS (MS)-a chronic, often disabling disease that randomly attacks the CNS (brain and spinal cord) due to an autoimmune response where the immune system attacks a person’s own tissues. Women are affected 2x more than men, with the onset of symptoms

occurring usually between ages 20-40yrs (tingling, numbness, paralysis, and blindness). Patients with MS may have facial and jaw weakness, and Bell’s Palsy & Trigeminal Neuralgia may develop more frequently in MS patients. GLOSSOPHARYNGEAL NEURALGIA – pain similar to trigeminal neuralgia that arises from the glossopharyngeal nerve (CN 9). It is not as common as trigeminal neuralgia, but the pain may be as severe. Occurs in both sexes (middle-aged or elderly). Sharp, sudden, shooting almost always UNILATERAL pain in the ear, pharynx, nasopharynx, tonsils, or posterior tongue. POSTHERPETIC NEURALGIA-a persistent burning, aching, itching, and hyperesthesia along distribution of a cutaneous nerve after an attack of HERPES ZOSTER. May last a week or many months. Involves FACIAL NERVE (CN 7) & geniculate ganglion that produces Ramsey Hunt Syndrome (facial paralysis & otalgia/earache). MYASTHENIA GRAVIS – a chronic condition of EXTREME MUSCLE WEAKNESS due to an autoimmune disorder where the body creates antibodies against its own nicotinic ACh (acetylcholine) receptors in the neuromuscular junctions. The muscles are quickly fatigued with repetitive use. It is typical for the patient to have a flattened smile and DROOPY EYES with slow papillary light responses (double vision). XEROSTOMIA & RAMPANT

CARIES may be present because acetylcholine needed for proper transmission of nerve impulses is destroyed, so salivary glands do not receive adequate stimulation. • Immune system produces autoantibodies that attack Acetylcholine receptors that lie on the muscle side of the neuromuscular junction causing dysfunction of the myoneural junction (the neuromuscular junction functions abnormally causing muscle weakness). This decreases the responsiveness of the muscle fibers to acetylcholine released from motor neuron endings. Difficulty speaking and swallowing, and weakness of the arms and legs are common (muscles of the face, neck, arms, and legs are affected). • 10% of people develop a life-threatening weakness of the breathing muscles (a condition called myasthenic crisis). • Drugs that increase the level of acetylcholine (e.g. pyridostigmine or neostigmine) may be given for treatment. EATON-LAMBERT SYNDROME – similar to myasthenia gravis in that it is an autoimmune disease causing weakness, but due to the inadequate RELEASE of acetylcholine rather than by abnormal antibodies that attack acetylcholine receptors as in Myasthenia Gravis.

FREY’S SYNDROME (AURICULOTEMPORAL SYNDROME)–an uncommon phenomenon due to damage to the auriculotemporal nerve and subsequent reinnervation of the sweat glands by parasympathetic salivary fibers. Can occur after surgery (i.e. removal of a parotid tumor, ramus of the mandible, or infection of the parotid that has damaged the auriculotemporal nerve (branch of V3). Gustatory sweating is the chief complaint. Patient exhibits flushing and sweating of the involved side of the face during eating. BELL’S PALSY –facial paralysis from damage to the FACIAL NERVE (CN7). Can occur at any age, but disproportionately attacks pregnant women, diabetics, and people with influenza, cold, or other upper respiratory infection. • Clinical Signs: unilateral paralysis of all facial muscles with loss of eyebrow & forehead wrinkles, drooping of eyebrows, flattening of the nasiolabial furrow, sagging of the mouth corner, and inability to frown or raise the eyebrows. The upper and lower lips may also be paralyzed on the affected side. Drooping mouth on one side with a watering eye, loss of taste sensation on the anterior portion of the tongue may occur. • Sudden onset, but paralysis begins to subside in 2-3 weeks, and gradual, complete recovery occurs in > 85% of patients.

• Important: While giving an inferior alveolar block, if you inject anesthetic solution into the parotid gland capsule, you may cause a Bell’s Palsy-like feeling by anesthetizing the facial nerve.

NON-ODONTOGENIC CYSTS CONGENITAL CYSTS: 1. Branchiogenic Cyst-arises from the persistence of the second branchial arch cleft. This cyst is located along the anterior border of the sternocleidomastoid muscle at any level in the neck. The cyst is lined with ciliated and striated squamous epithelium, and contains a milky/ mucoid fluid. Treatment: complete surgical excision. 2. Dermoid Cyst-relatively uncommon cyst in the oral cavity. This cyst often contains hair, sebaceous and sweat glands, and tooth structures. The most common site is the MOUTH FLOOR. Treatment: complete surgical excision. 3. Thyroglossal Duct Cyst-may arise from any part of the thyroglossal duct. Found in a midline position and is usually dark colored, and may be vascular resembling a hemangioma. Hemorrhage into the mouth is a common and important symptom caused by rupturing of the overlying veins. Treatment: complete excision of the tract to the base of the tongue (often including part of the hyoid bone).

NON-ODONTOGENIC FISSURAL CYSTS (DEVELOPMENTAL CYSTS): (Nasopalatine, Nasoalveolar, Median Palatal, and Globulomaxillary cysts). NASOPALATINE DUCT CYST (Incisive Canal Cyst)OVAL or “HEART-SHAPED” radiolucency in the midline of the hard palate. Most common nonodontogenic/developmental/fissural cyst. Usually asymptomatic (but patient may complain of tender swelling of the palate), or may produce an elevation in the anterior part of the palate. Teeth are vital. Treatment: surgical excision/enucleation. Prognosis is excellent. Occurs in bone (intra-osseous). • Radiographic Features: a circular (round) welldemarcated oval or heart-shaped radiolucency between and above the MAXILLARY CENTRAL INCISORS (rarely just lateral to the midline) on a radiograph clinically seen as a marked swelling in the region of the palatine papilla, situated distal to the roots of the central incisors. The pulps of the anterior teeth are vital. The lesion crosses the midline. Do not confuse with an enlarged palatine foramen. • Cysts that arise from epithelial remnants in the incisive canal are the most common type of maxillary developmental cyst. Histologically, its walls are lined with vessels, nerves, and mucous glands (remnants of nasopalatine ducts within bone).

They most often remain limited in size and are asymptomatic. Some however, become infected or have a tendency to grow extensively. When this occurs, surgical intervention is indicated.

Palatine Papilla Cyst-the soft tissue (and much less common) variant of the Nasopalatine Duct Cyst. NASOLABIAL CYST (Nasoalveolar Cyst)-a soft tissue cyst of the UPPER LIP (extra-osseous cyst) superficially located in soft tissue of the upper lip that histologically develops from epithelial remnants from the inferior and anterior portion of the nasolacrimal duct. • Clinical Features: swelling below or inside the NOSTRIL that may present in the canine region. CANNOT SEE THIS CYST ON A RADIOGRAPH, but may produce “cupping” of underlying bone. NOT WITHIN BONE (extra-osseous) so not visible on a radiograph. Treatment: Enucleation (surgical excision). Excellent

prognosis. GLOBULOMAXILLARY CYST-an inverted “PEARSHAPED” radiolucency in bone between the roots of the maxillary lateral & canine (often causes roots of the involved teeth to DIVERGE). • Clinical Features: usually asymptomatic, but occasionally produces swelling with or without pain. All regional teeth are vital. Occurs within bone (intra-osseous). • Histologic Features: consists of epithelial remnants where the globular & maxillary processes are fused. • Radiographic Features: inverted PEAR-SHAPED radiolucency between the maxillary lateral & canine roots. Teeth are vital, but roots may be divergent. Do not confuse with a Lateral Periodontal Cyst.

• Treatment: Enucleation without disturbing the teeth. Excellent prognosis. MEDIAN PALATAL CYST-rare, but may occur anywhere along the MEDIAN PALATAL RAPHE, usually in the HARD PALATE MIDLINE, posterior to the pre-maxilla (occurs in bone; intra-osseous). May produce swelling on the palate. Clinically, this lesion presents as a firm, painless swelling. This cyst may represent a more posterior version of a Nasopalatine Duct Cyst, rather than a separate cystic degeneration of epithelial rests at the line of fusion of the palatine shelves. • Histologic Features: epithelial remnants in the line of fusion between the palatine processes. Appears as a soft, fluctuant or crepitant swelling in the hard

palate midline. • Radiographic Features: well-demarcated radiolucency in the midline of the hard palate. • Treatment: Enucleation with an excellent prognosis. MEDIAN ALVEOLAR CYST-rare, but occurs in the bony alveolus (intraosseous) between the central incisors. Distinguished from a periapical cyst by the fact that the adjacent teeth are vital. Treatment: enucleation.

ODONTOGENIC CYSTS LATERAL PERIODONTAL CYST-a painless unilocular, well-defined TEAR-DROP SHAPE radiolucency (with opaque margins) that occurs 95% of the time between and along the lateral surfaces OF VITAL MANDIBULAR CANINE & PREMOLAR ROOTS. DO NOT PROBE THE AREA. Affects middle aged males > than females. Developmental cyst treated by surgical enucleation. Histologic Features: thin lining of non-keratinized epithelium.

RADICULAR CYST (APICAL PERIODONTAL CYST OR

PERIAPICAL CYST)-the MOST COMMON ODONTOGENIC CYST, mainly found at the ROOT APEX. It develops within a pre-existing periapical dental granuloma. Increased osmotic pressure in the cyst lumen is important in its pathogenesis. Clinical Features: asymptomatic, tooth is NECROTIC; can be sensitive to percussion. • Radiographic Features: well-circumscribed radiolucency at the tooth apex. • Histologic Features: exhibits a lumen (true cyst) invariably lined by stratified squamous epithelium. The cyst wall is condensed C.T. with plasma cells, lymphocytes, and PMN leukocytes. • Treatment: RCT with apicoectomy, or extraction with socket curettage.

RESIDUAL CYST-occurs when a tooth with a radicular (periapical) cyst is extracted, but the radicular cyst is left undisturbed and persists within the jaw now as a residual cyst. To prevent a residual cyst, you must CURETTE the radicular cyst out of the tooth socket after extraction. Equally affects males and female at any age. Usually asymptomatic, and is found in EDENTULOUS areas. It is a radicular cyst left in the jaw after a tooth extraction. • Radiographic Features: well-defined radiolucency not associated with a tooth. Usually solitary. • Histological Features: same as a radicular cyst (apical periodontal cyst), has stratified squamous epithelium lining the lumen.

DENTAL GRANULOMA –the MOST COMMON SEQUELAE OF PULPITIS AT THE ROOT APEX. Only distinguished from a radicular cyst histologically. • Clinical Features: asymptomatic, NECROTIC TOOTH, but may be percussion sensitive. • Radiographic Features: circumscribed radiolucency at the tooth apex. • Histologic Features: lined by stratified squamous epithelium. Cyst wall is fibrous C.T. with macrophages, lymphocytes, cells, and capillaries. • Treatment: RCT or extraction of the involved tooth. DENTIGEROUS CYST (FOLLICULAR CYST) – an odontogenic cysts always associated with the crown of an UNERUPTED or DEVELOPING tooth or dental anomaly (ex: odontoma). Most commonly found with a developing 3rd molar. Can cause marked displacement of teeth due to pressure of accumulated fluid that usually displaces the tooth apically. If a tooth with a dentigerous cysts begins to erupt, the bulging the cyst produces on the ridge is an eruption cyst. • Clinical Features: found in children & teenagers in mandibular 3rd molar & maxillary canine area (70% in the mandible). Associated with impacted or unerupted teeth.

The 2nd most common odontogenic cyst. • Radiographic Features: well-defined, unilocular radiolucency. • Histologic Features: lined by non-keratinized, stratified squamous epithelium with NO rete-pegs. • An ameloblastoma is most likely to develop in the wall of a dentigerous cyst.

ERUPTION CYST – a soft-tissue variant of the dentigerous cyst, invariably associated with an erupting tooth (usually primary, but occasionally permanent teeth). Mainly effects gingival tissues, not bone. • Clinical Features: usually a smooth-surface lesion that is REDDISH-PINK or BLUISH-PINK-BLACK, fluctuant, localized swelling of the ALVEOLAR

RIDGE over the crown of an erupting primary or permanent molar. The intense bluish color is due to accumulation of blood. Due to this appearance, it can be mistaken for a hemangioma or hematoma.

• Treatment: usually NO treatment is necessary. In rare cases, incision or removal of the overlying tissue may be required due to pain or tenderness associated with the cyst. PRIMORDIAL CYST (FOLLICULAR CYST) –well-defined, oval radiolucent lesion that differs from periodontal and dentigerous cysts since it contains no calcified structures. Located in the MANDIBULAR 3rd molar space. It may be locular, multilocular, or multiple. This cyst is found in place of a tooth rather than directly

associated with a tooth. Equally affects males and females under age 25yrs. • Histologic Features: lined by stratified squamous epithelium; no rete pegs (arises from epithelium of the enamel organ). ODONTOGENIC KERATOCYST – follicular & dentigerous cysts that contain keratinizing material, and differs from other odontogenic cysts due to their microscopic appearance & clinical behavior. Keratocysts may resemble periodontal, primordial, or follicular cysts, and usually CANNOT be distinguished radiographically. The most remarkable feature is their GREAT TENDENCY TO REOCCUR (over 30% reoccur). Usually occurs between ages 10-30yrs. Often associated with an impacted tooth. 50% are found in the mandibular 3rd molar area. • Keratocysts increase in size mainly by a process of epithelial cell multiplication. • Radiographic Features: well-circumscribed radiolucency with smooth margins, and thin radiopaque borders. • Histologic Features: thin layer of corrugated parakeratin. Uniform thin stratified squamous lining. Distinct cuboidal to columnar basal layer with varying amounts of keratin debris in the lumen.

• Treatment: excision of the overlying mucosa in the area where the cyst wall is adhered. TRAUMATIC BONE CYST – may be completely devoid of solid or liquid material (but may contain blood, fluid, debris, or be completely empty). It is most commonly found in younger people with no sex predilection, in the MANDIBLE BETWEEN THE CANINE & RAMUS. Regional teeth are vital. • Clinical Case: a large radiolucent area on the mandible apical to the premolars and molars on a panorex. No clinical symptoms. Teeth are not carious and respond normally to vitality tests. Medical history is unremarkable. No fluid or tissue is evident. GINGIVAL CYST – a rare, circumscribed swelling of the gingiva usually found in the canine & premolar areas on the mandible. Usually limited to the gingiva, but larger cysts may erode the bone. Gingival cysts are easily excised.

NON-ODONTOGENIC TUMORS FIBROUS DYSPLASIA – a rare, GENETIC abnormal condition that affects young people (15-30 years) characterized by the fibrous replacement of the osseous tissues in affected bones. The cause is unknown. Demonstrates typical “GROUND GLASS” appearance of bone, thus if this appearance is seen, additional tests like a skull radiograph & blood chemistries are performed to help diagnose. Characterized by normal bone replaced by fibrous tissue. Three types exist depending on the extensiveness of bone involvement: 1. Monostotic-involves one bone. 2. Polyostotic-involves more than one bone. 3. Albright’s Syndrome-involves more than one bone with endocrine disturbances. Pathologic fractures are the chief complaint.

Fibrous Dysplasia is a symptomatic alteration of bone (SWELLING MASS OF BONE OCCURS). Normal bone is replaced by fibrous tissue & non-functional bony trabeculae (teeth may look like they are moving or displaced due to bone growing tissue). Panorex shows typical “GROUND-GLASS” BONY APPEARANCE. Thus, ground-glass is seen, perform additional diagnostic tests (skull radiograph & blood chemistries) to confirm the diagnosis. • Fibrous Dysplasia Treatment: SURGERY to remove the area when the lesion stops growing. Benign osseous lesions were treated with radiotherapy, but this may produce osteogenic sarcoma (bone cancer), so do not

want to radiate benign bone. • Radiographic: when mature it has a radiopaque “GROUND-GLASS” appearance (diffuse/not welldefined, mixed radiopaque-radiolucent areas). MAIN CHARACTERISTIC is you never see the lesion’s borders (it blends with the bone). THREE TYPES OF FIBROUS DYSPLASIA: 1. Monostotic Fibrous Dysplasia-the most common form of fibrous dysplasia (80%) that affects children & young adults (both sexes equally). This form affects one bone (ribs & femur are common sites). The jaws are also commonly affected (mainly the maxilla, presenting as a painless swelling/bulge). A panorex reveals a radiopaque mass/lesion with irregular borders (poorly defined margins) with “ground/frosted glass” appearance. When several adjacent bones are affected, it is called “Craniofacial Fibrous Dysplasia”. • A differential diagnosis of fibrous dysplasia of the jaws is Ossifying Fibroma. However, radiographically, an ossifying fibroma has a well-circumscribed appearance. • Often causes expansion and deformity of the jawbone and tooth displacement. There is radiographic characteristic thickening at the skull base. The lesion is usually diffuse

and radiopaque with “ground-glass” appearance radiographically. Malignant transformation is possible if treated with radiation therapy, so treatment usually consists of SURGICAL removal when possible. However, since these lesions are not well-circumscribed, surgical recontouring is done to remove the portion of the lesion causing the facial deformity. 2. Polyostotic Fibrous Dysplasia-usually displays a segmental distribution of the involved bones (multiple bones). Occurs during childhood (mainly females who reach puberty prematurely). Affects long bones, face, clavicles, and pelvic bones. Initial signs may be a limp, pain, or fracture on the affected side. 3. Albright’s Syndrome (McCune-Albright Syndrome)-a disease of unknown cause affecting the bones (bone disease), skin pigmentation (irregular brown spots/skin pigmentation), and causing premature sexual development (endocrine problems). The extent of these problems varies depending on the individual. Hallmark sign is FEMALE PREMATURE PUBERTY (early sexual development in the male is less common). Affects young people (males & females equally). Albright’s is THE MOST SEVERE form of polyostotic fibrous dysplasia (involves multiple bones) with endocrine involvement.

• Triad of Symptoms: polyostotic fibrous dysplasia, Café-au-lait brown skin spots, & endocrine abnormalities (most common is precocious sexual development in females), and pathologic bone fractures. Malignant transformation potential of polyostotic (mainly) & monostotic fibrous dysplasias into osteosarcomas is an additional complication. • Treatment: No treatment. Drugs that inhibit estrogen production (Testolactone) have been used with some success. GARDNER’S SYNDROME – a polyposis syndrome inherited in a dominant manner. The most serious complication of is multiple (thousands) of POLYPS that affect the large intestine, duodenum, colon, and stomach. The polyps usually appear around age 15yrs. and eventually become malignant eventually causing COLON CANCER. • Oral Findings: MULTIPLE ODONTOMAS, multiple impacted & supernumerary teeth (like Cleidocranial Dysostosis), and multiple jaw osteomas giving a “COTTON-WOOL” (LIKE PAGET’S DISEASE) appearance to the jaws by appearing as dense, wellcircumscribed radiopacities. Associated with bony tumors in the jaw and skull. When GS is suspected

based on oral findings, refer the patient to a Gastroenterologist. • Multiple desmoid tumors (fibromatosis), and epidermoid skin cysts may also occur.

CENTRAL GIANT CELL GRANULOMA – a BENIGN tumor (accounts for < 7% of all benign jaw tumors). Occurs almost exclusively in the JAWBONES after TRAUMA (fall, blow, or tooth extraction). Occurs mainly in children or young adult ages 0-20yrs (more common in females), and either jaw may be involved (but affects the mandible more). • Most common in ANTERIOR SEGMENT (SYMPHYSIS OF MANDIBLE) of females during 0-20 years of age. May sometimes cross the midline. • Pain is not a main feature of this lesion. Slight to moderate bulging of the jaw due to expansion of the cortical plates occurs in the involved area depending on the extent of bone involvement.

• Radiographic Findings: unilocular or multi-locular radiolucencies of bone with well-defined margins (similar to ameloblastoma & odontogenic keratocyst). • Histologically, has loose fibrillar C.T. Multi-nucleated giant cells are prominent throughout the C.T. • Treatment: Curettage or surgical excision. These lesions fill in with new bone after excision.

MANDIBULAR TORI (TORUS MANDBULARIS) –bony exophytic growths that occur along the lingual surface of the mandible, superior to the mylohyoid ridge. Mandibular tori most often appear on the LINGUAL SURFACE of the mandible, usually in the PREMOLAR REGION. • Mandibular tori may occur singly, but there is a marked tendency toward bilateral occurrence, and the lesion is not always confined to the premolar region. Unlike palatal tori, mandibular tori are more readily demonstrated radiographically. • Maxillary (palatal) & mandibular tori are NOT

pathologic, and are rarely of clinical significance while teeth are still present. However, if a complete denture must be made, tori should be carefully removed. EXOSTOSIS (TORI) – slow-growing, BENIGN BONY KNOTS on the hard palate or lingual aspect of the mandible (usually symmetrical in the mandible). Exostosis are the most common exophytic lesions.

CONDYLAR HYPERPLASIA – a rare, UNILATERAL enlargement of the condyle of unknown cause. May be due to mild, chronic inflammation that stimulates the growth of the condyle or adjacent tissues. Afflicted patients usually exhibit a unilateral, slowly progressive elongation of the face with deviation of the chin AWAY from the affected side. The affected TMJ may or may not be painful, and there is usually severe malocclusion.

CONDYLAR HYPOPLASIA & APLASIA – can occur unilaterally or bilaterally. If unilateral, there is obvious facial asymmetry, which may alter occlusion and mastication. If unilateral, the mandible shifts TOWARD the affected side during opening. In bilateral cases, there is no shift. CENTRAL OSSIFYING FIBROMA –SLOW-GROWING, PAINLESS, BENIGN, asymptomatic neoplasm (tumor)/lesion that may occur in the MAXILLA or MANDIBLE. Due to its slow growth, the cortical plates of bone and overlying mucosa or skin are almost always invariably intact. Most commonly involves only ONE bone (Paget’s Disease, however, involves multiple bones). • May occur at any age, but is far more common in YOUNG ADULTS. It has an extremely variable radiographic appearance depending on its stage of development. However, despite the stage of development, the lesion is always WELLCIRCUMSCRIBED and demarcated from surrounding bone (in contrast to fibrous dysplasia). In its early stage, it is a RADIOLUCENT area, but as it matures, it becomes a uniform radiopaque mass. • Early Clinical Feature: TEETH DISPLACEMENT. • Amazing similarity clinically exists between a Central

Ossifying Fibroma & Central Cementifying Fibroma (a tumor of odontogenic origin). These are two separate benign tumors, identical in nature except for the cell undergoing proliferation (the osteoblast with bone formation in COF, and cementoblast with cementum formation in the CCF). Treatment: conservative excision. Recurrence is rare.

HISTIOCYTOSIS X–disorders where abnormal scavenger immune system cells histiocytes (macrophages), and eosinophils proliferate in the bone and lungs, CAUSING SCARS TO FORM. Three diseases are grouped under the generic term Histiocytosis, and occur due to metabolic defects in the reticuloendothelial system, and are characterized by proliferation of histiocytes (macrophages) of loose C.T. 1. EOSINOPHILLIC GRANULOMA- the MOST BENIGN

FORM of Histiocytosis X more common in males ages 20-40yrs. It may be totally asymptomatic, but there may be local pain or swelling, especially if a bone fracture occurs. In the mouth, the mandible is most likely affected with teeth on the affected side being loose and gingivitis. When the LUNGS are affected, the symptoms may include coughing, shortness of breath, & weight loss. Pneumothorax is a common complication. Radiographic Features: lesion appears as irregular radiolucent areas involving superficial alveolar bone. Jaw lesions usually appear as single or multiple radiolucencies that may be so wellcircumscribed, as to resemble cysts or periapical granulomas. o Treatment: bone lesions often resolve spontaneously, and do not require treatment unless they cause symptoms. Curettage provides diagnostic biopsy material and is curative. 2. LETTERER-SIWE DISEASE (Acute Disseminated Form)-starts before age 3yrs and is usually FATAL without treatment. Histiocytes damage the lungs (pneumothorax may occur), skin, lymph glands, bone, liver, and spleen. Oral lesions are uncommon. 3. Hand-Schuller-Cristian Disease (Chronic Disseminated Form)-usually begins in early childhood

(more common in boys). Triad of Symptoms: exophthalmos, diabetes insipidus, & bone destruction (skull and jaws). Oral signs: halitosis, sore mouth, mobile teeth. Treatment: people with Hand-Schuller-Christian Disease or Eosinophilic Granuloma may recover spontaneously. All three disorders may be treated with corticosteroids & cytotoxic drugs (ex: Cyclophosphamide). Use radiation therapy if there is bone involvement. Death usually results from respiratory or heart failure. VERRUCIFORM XANTHOMA (“HISTIOCYTOSIS Y”) – a benign soft tissue tumor that presents as a normal or white colored verrucous lesion. Its cause is UNKNOWN, as it is not associated with any systemic condition. In adults, the alveolar and palatal mucosa are the common sites. • Histologic Features: verrucous, hyperparakeratotic surface with parakeratotic plugging. Large “foam” cells in C.T. papillae between elongated rete ridges. • Treatment: simple excision; no recurrence.

ODONTOGENIC TUMORS AMELOBLASTOMA-tumors of odontogenic epithelial origin and MOST COMMON EPITHELIAL (ECTODERMAL) ODONTOGENIC TUMOR! (its occurrence equals the frequency of all other odontogenic tumors combined). Consists entirely of odontogenic epithelium that shows the differentiation of the familiar, histologic layers of the enamel organ at sites. Enlargement of the tumor may expand the buccal, lingua cortical plates of bone, or palatal bone plates. Root resorption of teeth adjacent to the tumor is common, and in many cases an UNERUPTED, MANDIBULAR 3rd MOLAR is associated with the radiolucent defect. Ameloblastomas are slow-growing, locally invasive tumors that usually run a benign course (do not infiltrate). Often asymptomatic, a painless swelling or expansion of the jaw is the usual clinical presentation. Ameloblastomas occur in three different clinical-radiographic situations with different treatments & prognosis:

AMELOBLASTOMA-often associated with unerupted teeth mainly in the posterior body and angle of the mandible (but can be in the maxilla). Looks like a multiloculated “SOAP-BUBBLE” appearance on a panorex. BENIGN TUMOR of ODONTOGENIC ORIGIN that is usually painless. Can cause severe facial/jaw abnormalities due to its growth potential which destroys surrounding bone. Treatment: SURGICAL EXICISION. • Histogenesis: may arise from rests of the dental lamina, epithelial lining of a dentigerous cyst, basal cells of the oral epithelium (mucosa), developing enamel organ, and possibly remnants of Hertwig’s sheath. Consists entirely of odontogenic epithelium that shows the differentiation of the histologic layers of the enamel organ at sites. • Clinical Features: most often seen in adolescents in the mandibular (retro) molar area. THE MOST

AGGRESSIVE ODONTOGENIC TUMOR that is usually benign, but often shows a highly expansive and locally invasive mode of growth. • Ameloblastoma Radiographic Features: multi-locular or uni-locular RADIOLUCENT lesion on vital teeth with a “soap bubble” appearance when the radiolucent loculations are large & honeycombed when the loculations are small. Has irregular-scalloped margins. Appears similar to a Central Giant Cell Granuloma in the mandible. • Microscopic Features: various microscopic patterns of the tumor include the follicular (most common pattern of multiple islands with reverse polarity) & plexiform (also the most common pattern of large anastomosing cords), cystic, acanthomatous (extensive squamous metaplasia, keratin formation in central portions of epithelial islands), granular cell (cells with prominent granular cytoplasm), desmoplastic (thin cords), & basal cell (islands of hyperchromatic basaloid cells) patterns. All are non-encapsulated. • Treatment: varies depending on the subtype. Recurrence is common (50-90%) if inadequately treated. Very rarely metastasizes. ADENOMATOID ODONTOGENIC TUMOR (ADENOAMELOBLASTOMA) – a BENIGN tumor of

ectodermal origin (purely epithelial) representing 3-7% of all odontogenic tumors. Limited to CHILDREN & TEENAGERS (10-19yrs), and uncommon in patients older than 30yrs. Not a variant of ameoloblastoma (best classified as a hamartoma; benign, not a true neoplasm). Occurs mainly in the ANTERIOR MAXILLA (2x more common in maxilla than mandible) and affects females 2x more than males. Affects 2nd decade (teenagers). Asymptomatic or painless swelling. • Clinical Features: most are small (< 3cm diameter). Clinically looks like a gingival fibrous lesion. Often asymptomatic, circumscribed, unilocular radiolucency associated with the crown of an UNERUPTED tooth (mostly canines). Radiolucency sometimes extends apically along the root past the CEJ (helps distinguish it from a dentigerous cyst). May be completely radiolucent, but often contains fine snowflake calcifications (tiny radiopaque foci). • Histology: well-defined lesion surrounded by a thick, fibrous capsule. Enamel organ, lining of dentigerous cyst, reduced enamel epithelium, Rests of Malessez. Derived from ectoderm (epithelial) enamel organ and remanants of dental lamina. Tumor is composed of spindle-shaped epithelial cells that form sheets, strands, or whorled masses of cells in a scant fibrous stroma. • Treatment: completely benign, and its capsule allows it

to enucleate easily from bone (enucleation). Recurrence is rare, and is not aggressive. CALCIFYING EPITHELIAL ODONTOGENIC TUMOR (“PINDBORG TUMOR”) – a rare lesion/tumor (< 1% of all odontogenic tumors) derived purely from ECTODERM (epithelial). Found in patients mainly 30-50 years old, no sex predilection (30% are 4th decade). Uncommon in children and adolescents. 2/3 of cases occur in MANDIBLE (molar-premolar area). Painless, slow-growing swelling is the most common clinical sign. Rarely extra-osseous. Radiolucent-radiopaque areas associated with an unerupted tooth and amyloid production. • Radiographic Features: unilocular or more often multilocular radiolucent defect. Scalloped margin. Tumor is often associated with an impacted tooth (mandibular 3rd molar). Calcifications within the tumor are often most prominent around the crown of an impacted tooth. • Histology: reduced enamel epithelium, has discrete islands, strands, or sheets of polyhedral epithelial cells in a fibrous stroma. • Treatment: conservative local resection to include a narrow rim of surrounding bone. 15% recurrence rate (tumors treated by curettage have highest recurrence rate if inadequately treated). Good prognosis.

SQUAMOUS ODONTOGENIC TUMOR – rare benign odontogenic tumor/neoplasm derived purely from ECTODERM (epithelial). Found in patients ages 8-75yrs (average 38yrs). Randomly distributed throughout the alveolar processes of maxilla and mandible with no site or sex predilection. Most common complaint is painless or mild painful gingival swelling associated with tooth mobility. • Clinical Features: may be asymptomatic, painless swelling associated with mobile teeth. • Radiographic Features: triangular or circumscribed radiolucency LATERAL to the roots of an unerupted or erupted tooth. May be an ill or well defined area with sclerotic margins. Most are small lesions that rarely exceed 1.5cm diameter. Histogenesis: Rests of Malassez. • Treatment: conservative local excision or curettage, and close follow-up. Recurrence is rare. CEMENTOMA (PERIAPICAL CEMENTAL DYSPLASIA) – BENIGN odontogenic tumor that occurs most frequently in the ANTERIOR MANDIBLE (periapical region), and often affects multiple VITAL teeth. Cementoma is an unusual response of the periapical bone to some local factor (i.e. traumatic occlusion or infection). While it appears to arise from teeth, the lesions arise within bone. Age,

gender, location, radiographic appearance, and tooth vitality are important diagnostic criteria. Histogenesis: PDL. Do not mistake this as a need to do RCT (it is not a periapical abscess). Do a pulp vitality test to diagnose. If the pulp is vital, then NO TREATMENT IS REQUIRED. • Clinical Features: occurs at the apex of vital anterior teeth, affecting women over age 30yrs (especially BLACK women) more than men. Asymptomatic, usually multiple, small periapical radiolucent areas in the mandibular incisor area. Depending on its stage, a cementoma may appear radiolucent, mixed-radiolucent, or completely radiopaque. Three Cementoma Stages: • Stage 1: lesion is a periapical RADIOLUCENCY. • Stage 2: lesion begins to calcify and become more radiopaque (MIXED). • Stage 3: well-defined RADIOPACITY bordered by a thin radiolucent line. Radiographic Features: small sharply circumscribed radiopacity attached to, or adjacent to apices of teeth. Early lesions are radiolucent, then have a central opacity, and are densely radiopaque when mature. The opacities are not cementum, but are BONE.

BENIGN CEMENTOBLASTOMA (TRUE CEMENTOMA)– usually affects males under 25yrs. Mandibular premolars or molars. Usually solitary, and may cause expansion of the cortical plates. Tooth is vital. Histogenesis: PDL. • Radiographic Features: well-demarcated, mottled or densely radiopaque mass with radiolucent periphery

attached to the root, causing the root resorption. • Microscopic Features: cementum-like tissue with conspicuous reversal lines, variable amounts of fibrous C.T. with sheets of uncalcified “cementoid” especially at the periphery. • Treatment: EXTRACT THE INVOLVED TOOTH. GIGANTIFORM CEMENTOMA (FAMILIAL MULTIPLE CEMENTOMAS) –affects middle-aged black women. Multiple, often symmetrical, and may cause jaw expansion. Histogenesis: PDL. • Radiographic Features: large, dense lobulated radiopaque masses. • Microscopic Features: large sheets of tissue that resemble secondary cementum. • Treatment: conservative excision. ODONTOGENIC MYXOMA – an AGGRESSIVE tumor derived from the papilla, dental sac, or PDL. Occurs in adults 30-40yrs as a painless swelling in the mandible. • Radiographic Features: poorly-defined, multi-locular radiolucency that may be associated with an unerupted or displaced teeth.

• Treatment: curettage with cautery. High rate of recurrence if inadequately treated. ODONTOGENIC FIBROMA – derived from dental papilla, dental sac, or PDL. Occurs as a painless swelling in the mandible of children & young adults. Radiographically, it is a multilocular or unilocular radiolucency that may be associated with unerupted or displaced teeth. Treatment: Enucleation. Recurrence is rare. CEMENTIFYING FIBROMA – a well-defined radiolucency with scattered radiopaque foci. Occurs in the mandible of adults as a painless swelling. Histogenesis: PDL. Treatment: curettage. Recurrence is rare. ODONTOMA (Hamartoma)-ANY odontogenic tumor often associated with an unerupted tooth. The average age of people found with an odontoma is 14-years old. Remove with oral surgery.

COMPLEX ODONTOMA- amorphous RADIOPAQUE mass with a thin, radiolucent rim at the junction of surrounding bone (has varying densities). Most common in the POSTERIOR mandible. Derived from ectodermal & mesenchymal components of tooth germ. Affects ages 2030yrs, and found in the mandibular premolar-molar area. Asymptomatic, but may delay eruption of permanent teeth. • Radiographic Features: well-defined, radiopaque mass surrounded by a narrow radiolucent zone. May or may not be associated with an erupted tooth. • Histologic Features: conglomerate mass of dental tissues (dentin, enamel, cementum). Characterized by

the formation of calcified enamel and dentin in an abnormal arrangement because of lack of morphodifferentiation. Treatment: Surgery. DOES NOT RECUR.

COMPOUND ODONTOMA – a tumor of enamel and dentin common in the ANTERIOR MAXILLA (incisorcanine area) derived from ectodermal & mesenchymal components of tooth germ. It is arranged in the form of anomalous MINIATURE TEETH (several small abnormal teeth). Affects ages 20-30yrs. May also appear in the mandible canine-premolar area, and can cause delayed eruption or prevent eruption of permanent teeth. • Radiographic Features: groups of small radiopacities

(multiple small tooth-like structures with a thin radiolucent rim at the junction with surrounding bone.) Common between the maxillary premolar and central incisor on a panorex. • Microscopic Features: multiple, small malformed teeth of dentin, enamel, and cementum. • Treatment: Enucleation (Surgery), and does not recur.

PIGMENTED LESIONS PEUTZ-JEGHERS SYNDROME (HEREDITARY INTESTINAL POLYPOSIS SYNDROME) – an autosomal dominant inherited disorder, characterized by MULTIPLE INTESTINAL POLYPS distributed through the entire intestine (especially the jejunum) and intra-oral MELANIN pigmentations of the lips and oral mucosa at birth or early age (often during the first decade of life), and at this time is restricted to the oral region. • Intra-orally, the pigmentations can be found anywhere on the mucosa, but are most common on the BUCCAL MUCOSA, GINGIVA, and HARD PALATE. The mucosal surface of the LOWER LIP is almost always involved. These spots or macules vary in intensity, and range in shades of BROWN, BLUE, and BLACK. The tongue seldom shows this melanin pigmentation. The oral pigmentations are harmless, but their presence is important to determine if multiple polyposis exist in the intestines and colon which may be harmful. There is a strong tendency for the multiple polyps of the colon to undergo malignant change. • During succeeding decades of a patient’s life, pigmentations may arise on the skin’s extremities. Pigmentations of Peutz-Jeghers Syndrome may

occur without polyps, and multiple polyps may be present without any pigmentations. • When Peutz-Jegher Syndrome is suspected based on oral pigmentations, other conditions to consider in the differential diagnosis are Addison’s Disease & Albright’s Syndrome. ADDISON’S DISEASE (“Chronic Adrenocortical Insufficiency” or “Hypocorticolism)-a rare endocrine disorder caused by ADRNEAL CORTEX HYPOFUNCTION of CORTISOL (GLUCOCORTICOID)—not enough cortisol is produced. The hyposecretion is either due to an adrenal gland disorder (primary adrenal insufficiency), or inadequate secretion of ACTH by the pituitary gland (secondary adrenal insufficiency). CAUSES BRONZING OF THE ENTIRE SKIN. Cortisol’s most important function is to help the body respond to STRESS. Occurs in all age groups, affecting men and women equally. MAIN DENTAL CONCERN during dental procedures is this patient’s adrenal cortex has no capacity to produce extra cortisol in response to stress, which may result in Addison’s Crisis. • Clinical Signs: usually start gradually and include weight loss, loss of appetite, muscle weakness, low BP, darkening (hyperpigmentation) of the skin in both exposed & unexposed parts of the body (most visible on scars, skin folds, pressure points like elbows, knees,

knuckles, toes, and oral mucous membranes). Also nausea, vomiting, diarrhea. • Oral Signs: diffuse pigmentation of the gingiva, tongue, hard palate, & buccal mucosa. Although cutaneous pigmentation will usually disappear, after therapy, pigmentation of the oral tissues tends to persist. • Lab Tests: show low blood concentrations of Na+ and glucose, increased serum K+, and decreased urinary output of certain steroids. ALBRIGHT’S SYNDROME (McCune-Albright Syndrome)-a severe form of polyostotic fibrous dysplasia that causes lesions of nearly all skeletal bones, brown patches of cutaneous pigmentation (café-au-lait spots), and endocrine dysfunction (especially precocious puberty in girls). There is also an increased incidence of osteosarcoma. AMALGAM TATTOO – a common finding sometimes mistaken for a melanin-pigmented lesion. Most common on the gingiva, buccal mucosa, and alveolar mucosa.

ASPIRIN BURN-occurs when patients place an aspirin tablet against an aching tooth, allowing the cheek or lip to hold it in position, while it dissolves slowly. Within a few minutes, a burning sensation of the mucosa is noted, and the surface becomes blanched or white (ORAL MUCOSA NECROSIS), with subsequent sloughing of the necrotic epithelium.

• Heavy Metal Bismuth Ingestion-used to treat some dermatologic disorders and various other diseases. Bismuth pigmentation appears as a “bismuth line” (thin,

blue-black line in the marginal gingiva that is sometimes confined to the gingival papilla). Most pigmented skin tumors are composed of NEVUS cells due to a developmental anomaly of MELANOCYTES, and are RARE in the oral cavity. The initial flat, raised lesion can become nodular, with an increase in consistency. Spontaneous involution may occur, and malignant transformation is a rare complication. When found intra-orally, pigmented skin tumors are most often on the HARD PALATE, but may appear on the gingiva and lips. If a pigmented lesion shows ulceration, increase in size, color darkening, a biopsy is performed to determine if malignant transformation is occurring. CONGENITAL NEVI (BIRTHMARK) – are usually larger (> 10cm) but as time passes, it can change from a flat, pale tan macules into elevated, verrucous, hairy lesions. ~15% occur on the head & neck skin. Have a higher incidence of malignant transformation than acquired nevi. Most common intra-oral location is the HARD PALATE. FOCAL MELANOSIS – may occur at any age and presents as a single or multiple small, FLAT BROWN asymptomatic lesion found mainly on the LOWER LIP. Focal melanosis describes two similar lesions that differ in their location and size: 1. Labial Melanotic Macule-a lesion on the lips (MAINLY

LOWER LIP), and almost always near the midline. Most lesions are 5mm or less in diameter.

2. Oral Melanotic Macule intra-oral lesion found on the gingiva, buccal mucosa, and palate. Most lesions are under 1.0 cm in diameter.

Treatment: melanotic macules with a short history are EXCISED and BIOPSIED to rule out the possibility of malignant melanoma. Lesions present > 5 years without a change in size or color, are followed unless the patient requests removal.

RED-BLUE LESIONS MEDIAN RHOMBOID GLOSSITIS –often affects MIDDLEAGED ADULTS. Once thought to be a congenital abnormality related to the persistence of the tuberculum impar, but is now believed to be a permanent end result of a CHROINC CANDIDA ALBICANS INFECTION. Diabetics, immunosuppressed patients, and patients on long-term antibiotic therapy are most susceptible. • Clinical Features: smooth, denuded, BEEFY-RED LESION devoid of filiform papillae. Mainly found on the MIDLINE of TONGUE DORSUM, just anterior to circumvallate papillae. Generally asymptomatic, and NO TREATMENT is usually necessary.

ERYTHROPLAKIA – a persistent, VELVITY-RED PATCH that cannot be characterized clinically as any other

condition. Like “leukoplakia”, it has no histologic connotation, but most erythroplakias are histologically diagnosed as severe epithelial dysplasia, carcinoma insitu, or invasive squamous cell carcinoma. May be located anywhere in the mouth, but are MOST likely found in the mandibular mucobuccal fold, oropharynx, and floor of the mouth. Equally affects males and females, especially over 60yrs.

BURNING TONGUE SYNDROME – patients usually do not exhibit clinically detectable lesions, but symptoms of intense pain and burning. BTS is frustrating for the patient and clinician because there is usually no clear-cut cause and no uniformly successful treatment. Typically affects MIDDLE-AGED FEMALES (it affects men too, but usually at an older age). Rare in children & teenagers. • Possible Etiologic Factors: anemias (pernicious anemia & iron deficiency anemia), diabetes mellitus,

gastric disturbances (i.e. hyperacidity or hypoacidity), psychogenic factors (emotional conflict, cancerophobia), trigeminal neuralgia, microorganisms (especially Candida Albicans & Streptococci), xerostomia (dry mouth) associated with Sjogrens Syndrome, anxiety, & drugs. Local irritation (tobacco, spices), and vitamin deficiency (especially B complex). PERIPHERAL GIANT CELL GRANULOM-relatively uncommon, pedunculated broad-based growths with a smooth surface (usually). Always on GINGIVA (between 1st permanent molar & incisors) or ALVEOLAR PROCESS. Mandibular gingiva is affected more than maxillary gingiva. It represents an unusual hyperplastic C.T. response to injury of gingival tissues. REDDISH-BLUE in color, sometimes lobulated, and bleed easily. Most patients are older than age 20yrs (Central Giant Cell Granuloma occurs mainly before age 20yrs). Affects females 2x more than males. Radiographs are usually negative. Clinically, it may resemble a fibroma or pyogenic granuloma. • Histologically: identical to Central Giant Cell Granuloma. Consists of a non-encapsulated tissue mass composed of a delicate reticular & fibrillar C.T. stroma with multi-nucleated giant cells. • Treatment: COMPLETE SURGICAL EXCISION.

HEMANGIOMA – BENIGN tumor consisting of a mass of BLOOD VESSELS. It can range from a small mass, to very large sacs of unsightly red, purple, or blue blood vessels (they can enlarge to very alarming sizes). Present at birth, childhood, or arise later in life (enlarges as the child grows). In certain locations, large hemangiomas can interfere with proper organ development and function. • Clinical Features: a common BENIGN tumor of a PROLIFERATION OF BLOOD VESSELS, affecting females 2x more than males (2:1). It is a soft, smooth, blue, red, purple, or purplish-red mass that most commonly affects the TONGUE, BUCCAL MUCOSA, LIPS, PALATE. • Microscopic Features: capillary, cavernous, and a hemangioendothelioma of stratified squamous epithelium covering of loose, fibrous C.T. that contains many thinwalled engorged vascular spaces.

• Treatment: laser therapy or surgery. May regress spontaneously. DO NOT INCISIONAL BIOPSY.

PSEUDOCYSTS (“NOT TRUE CYSTS”) NO EPITHELIAL LINING TRAUMATIC BONE CYSTS (PSEUDOCYSTS)–non-cysts found in the MANDIBLE (between canine & ramus) of mainly TEENAGERS due to trauma. PAINLESS, WELLDEFINED SCALLOPED RADIOLUCENCY between teeth. Although sometimes asymptomatic, it may produce jaw enlargement. Regional teeth are vital. • AKA: simple bone cyst, hemorrhagic bone cyst, unicameral bone cyst, extravasation bone cyst, idiopathic bone cyst, and solitary bone cyst. • Treatment: open the lesion, curettage, and suture. It may contain blood, serosanguineous fluid, debris composed mainly of a blood clot, or may be completely devoid of solid material. ANEURYSMAL BONE CYST– a BENIGN bone lesion generally regarded as a “reactive process” (not a neoplastic or cystic process). A RARE expansile, osteolytic bone lesion consisting of a proliferation of vascular tissue

that forms a lining around blood-filled cystic lesions. Most occur equally in males and females under age 20yrs, and is UNCOMMON AFTER AGE 30yrs. • Involves the proximal humerus, femur, tibia, and pelvis (uncommon in jaws, but usually appears in the mandible). Lesions are usually tender or PAINFULL on motion of the affected bone. Upon entering the lesion surgically, excessive bleeding occurs, and the tissue often resembles a “bloodsoaked” sponge. • Histology: has no epithelial lining (thus is a “pseudocyst”). Consists of fibrous C.T. stroma with many cavernous or sinusoidal blood-filled spaces. Fibroblasts & macrophages (histiocytes) line the sinusoids. Multi-nucleated giant cells (similar to cells of a giant cell granuloma) are dispersed throughout. • Radiographic Features: bone is expanded and appears CYSTIC with a “HONEYCOMB or SOAPBUBBLE” appearance. • Treatment: surgical curettage or excision, with little chance of recurrence.

SALIVARY GLAND TUMORS BENIGN SALIVARY GLAND TUMORS CLINICAL FEATURES–normal mucosa, painless, nodular, localized, movable, firm, slow-growing, well-differentiated, & encapsulated/well-circumscribed. • Radiographically, a benign neoplasm in bone may be distinguished from a malignant neoplasm in because with a benign lesion, the CORTEX REMAINS IN TACT, but may be thinned and the part involved may be expanded. Also, benign margins are usually defined and demarcated from surrounding bone. PLEOMORPHIC ADENOMA (Mixed Tumor)- MOST COMMON BENIGN salivary gland tumor. MONOMORPHIC ADENOMAS: Basal Cell, Canalicular, Myoepithelioma, Sebaceous, Papillary Cystadenoma Lymphomatosum (Warthin’s Tumor), Onocytoma (oxyphilic/acidophilic adenoma). The most common site of intra-oral MINOR salivary gland neoplasms/tumors is the PALATE. The most common site of intra-oral MAJOR salivary gland neoplasms is the PAROTID GLAND.

NECROTIZING SIALOMETAPLASIA – a benign lesion of MINOR salivary glands, characterized by necrosis of the glandular parenchyma with associated squamous metaplasia & hyperplasia of the ductal epithelium. Its etiology is unknown, but may be related to vascular insufficiency and infarction of the glands. HARD PALATE is the most common site. • NS benign lesion shows no racial or sex predilection, with most patients > 40yrs of age. Clinically, it presents as a tender deep ulcer with sharply demarcated margins. Histologically, there is lobular necrosis of the glandular parenchyma, with squamous metaplasia & hyperplasia of the ductal epithelium. Clinically and histologically, the lesion may stimulate a malignancy. In the past, the lesion was misdiagnosed as a squamous cell carcinoma or mucoepidermoid carcinoma. • After performing a BIOPSY and establishing a diagnosis, additional treatment is usually not recommended since healing usually occurs within 6-12 weeks. MUMPS -most common VIRAL DISEASE of the SALIVARY GLANDS caused by RNA-Paramyxovirus. Clinically, 90% of cases occur before age 14yrs. A major sign is sudden salivary gland swelling without purulent discharge from the duct. PAROITID GLAND is involved 90% of the time, and is bilaterally involved in 2/3 of cases. Patient presents with mild fever, malaise, and anorexia.

Most cases are self-limiting. • Complications: orchitis (inflammation of the testis) and epididymitis can occur in post-pubertal males, and may cause sterility. CNS disturbances causing meningitis & encephalitis. Deafness, myocarditis, pancreatitis, oophoritis, and pyelonephritis. • Serum amylase may be elevated during the acute phase. Prevent by administering a live, attenuated vaccine which is 95% effective for a least 5 years. However, in non-inoculated patients, it can still cause acute nonsuppurative salivary adenitis. MEASELES (RUBEOLA)-an RNA Paramyxovirus spread by nasopharyngeal secretions. Characterized by KOPLIK SPOTS (small, irregular red dots with bluish-white centers on the soft palate or buccal mucosa opposite the 1st and 2nd molars, near each STENSON’S DUCT). Measles can cross the placenta.

MUCOCELES (MUCOUS RETENTION CYST)- fluid-filled sac under the mucosa usually on the LOWER LIP (rarely on the upper lip) usually due to trauma (biting lip or tongue). It involves the MINOR salivary glands and their ducts (ex: trauma to the salivary duct by lip biting or pinching). A COMMON LESION that may also appear on the palate, cheek, tongue, and mouth floor. Treatment: Surgical Excision. • Clinical Features: may lie deep in tissue or be very superficial. Depending on its location, its clinical appearance varies: Superficial mucocele- a raised, circumscribed vesicle, several millimeters to 1cm in diameter with a bluish-translucent cast. Deeper mucocele-appears as a fluctuant swelling also, but the tissue is normal in color.

RANULA (“TRUE RETENTION CYST”)- a fluctuant & painless cyst (swelling) that presents as a translucent, bluish, well-rounded, smooth-surfaced bulge that protrudes from ONE SIDE of the MOUTH FLOOR. It characteristically occurs UNILATERALLY in the floor of the mouth. Caused by an obstruction of either the SUBMANDIBULAR or SUBLINGUAL GLANDS, due to a salivary stone or soft organic substance. A history of increased size just before or during a meal, and decrease in size between meals is of diagnostic significance. • Treatment: COMPLETE SURGICAL EXCISION of the cyst roof or gland.

MIKULICZ’S DISEASE (“BENIGN LYMPHOEPITHELIAL LESION”) –rare salivary gland lesion, closely related to Sjogren’s Syndrome. A progressive, asymptomatic enlargement of the PAROTID & SUBMANDIBULAR glands. Appears initially unilateral, but over time becomes BILATERAL. Unknown etiology, but evidence suggests that both Mikulicz’s & Sjogren’s Syndrome are autoimmune diseases where the patient’s own salivary gland tissue becomes antigenic. Occurs most often in MIDDLE-AGED WOMEN. • Histologic Features: replacement of gland parenchyma by lymphocytic infiltrate that contains scattered epimyoepithelial islands within (this is the histologic

cornerstone for the diagnosis). • Most are BENIGN, but malignant transformation of the epimyoepithelial islands may occur.

MALIGNANT SALIVARY GLAND TUMORS Clinical Features of MALIGNANT Salivary Gland Tumors: PAINFUL, ulcerated mucosa, nodular, firm, fixed, rapid growth, invasion, immovable, metastasis, and not well-differentiated (anaplastic). Metastasis is the most important characteristic that distinguishes a malignant tumor from a benign tumor. PARASTHESIA suggests metasatic disease. • Histologic Features of Malignancy: anaplasia, abnormal mitosis, pleomorphism, hyperchromatism, increased nuclear-cytoplasmic ratio. • Host response to a malignancy is best reflected by lymphocytic infiltration at the edge of a tumor. The most characteristic feature of a malignancy rather than an inflammatory lesion is the malignancy will grow after the causative agent is removed. The most important characteristic of malignant neoplasms that distinguishes them from benign neoplasms is their ABILITY TO METASTASIZE. • Histologic Grading of Malignant Neoplasms: attempts to estimate the aggressiveness or degree of malignancy

of a malignant neoplasm based on the degree of differentiation of the component cells and number of mitoses. Grading mainly applies to squamous cell carcinomas, and is of limited clinical use. Most pathologist use three grades, and designate SCC as well-differentiated, moderately well-differentiated, or poorly differentiated. • Grade 1 = well-differentiated. • Grade 2 = moderately, well-differentiated. • Grade 3 = poorly undifferentiated. • Grade 4 = undifferentiated. ADENOCARCINOMA-affects MAJOR salivary glands (50%) & MINOR glands (50%). Its frequency is 25% minor glands, 5% submandibular glands, and 3% of parotid tumors. Usually presents as an asymptomatic mass. 80% survival rate for low-grade carcinoma, and 40% survival rate for high-grade carcinoma. ADENOID CYSTIC CARCINOMA-a malignant salivary gland tumor that usually affects MINOR salivary glands of the PALATE (70%), parotid gland (15%), & submandibular gland (14%). Represents 31% of minor salivary gland tumors, 14% of submandibular gland tumors, and 2% of parotid gland tumors. Patient presents with pain and/or nerve dysfunction in 25%-33% of patients. Facial weakness or paralysis is common. A slow, but relentless tumor progression, with a 20% twenty-year survival rate.

ACINIC CELL CARCINOMA-a malignant salivary gland tumor most likely associated with the parotid gland (96%), submandibular gland (2-3%), and minor salivary glands of the palate (1-2%). 2-4% of parotid tumors are acinic cell carcinoma. Patient presents with swelling, pain, or tenderness, and may have facial weakness or paralysis. 90% cure rate. MUCOEPIDERMOID CARCINOMA-usually occurs in the parotid gland (60%), palate (13%), & submandibular gland (6%). Represents 10% of minor salivary gland tumors, 6% of parotid tumors, and 5% of submandibular tumors. Patient usually has asymptomatic swelling, with a peak incidence in the 3rd decade of life (30s). Patient may have facial weakness or paralysis. Low-grade carcinomas have an 85-100% 5-year cure rate, while high-grade

carcinomas have a 20-45% 5-year cure rate. Metabolic Conditions & Chronic Salivary Gland Enlargement: Diabetes Mellitus, chronic alcoholism, malnutrition (anorexia & bulimia), obesity, hypertension, & hyperlipidema. PAROTID GLAND is most frequently enlarged (either unilaterally or bilaterally). Other Conditions Associated with PAROTID GLAND Enlargement: 1. Sjogren’s Syndrome & Sarcoidosis 2. Warthin’s Tumor (Papillary Cystadenoma Lymphomatosum) 3. Infections (mumps, actinomycosis, tuberculosis) 4. Benign Lymphoepithelial Lesion (Mikulicz’s Disease) 5. Acute Epidemic Parotitis 6. Malnutrition SJOGREN’S SYNDROME – a salivary gland disorder of unknown cause, but is AUTOIMMUNE. Marked mainly by chronic inflammation of the salivary and lacrimal glands, that usually progresses to fibrosis and atrophy of these glands. Mainly affects post-menopausal women who present with dry eyes, dry mouth. ~50% of cases, have BILATERAL enlargement of the parotid & submandibular glands. • Symptoms: rheumatoid arthritis, xerostomia (dry mouth), & keratoconjunctiva sicca (dryness of the

eyes). However, all 3 symptoms rarely occur in one patient. The decrease in salivation may cause rampant caries reminiscent of radiation caries. • Histologic Features: histological features of the salivary gland lesions in Sjogren’s Syndrome & Benign Lymphoepithelial Lesion (Mikulicz’s Disease) are identical. • Treatment: treat the symptoms. Keratoconjunctivitis is treated with ocular lubricants, and xerostomia is treated with saliva substitutes (artificial saliva). Biopsy of the labial or palatal salivary glands, sialograms, salivary flow rate tests, and blood work may help establish a diagnosis. However, radiation, surgical excision, and chemotherapy are NOT used as treatment. • Malignant lymphomas & “Pseudolymphomas” (“Atypical Benign Lympohiod Hyperplasia”) develop in some patients diagnosed with Sjogren’s Syndrome. Thus, close follow-up of patients is required. XEROSTOMIA (DRY MOUTH) –not a disease, but a SYMPTOM of certain diseases and medications. Xerostomia is caused by sialadenitis (insidious inflammatory disease of the major salivary glands), Sjogren’s syndrome, medications (anti-cholinergic drugs (Atropine & Scopolamine) & anti-psychotics (Phenothiazines: Chlorpromazine & Prochlorperazine),

cancer therapy, nerve damage, Alzheimer’s, stroke, bone marrow transplants, endocrine disorders, stress, anxiety, depression, and nutritional deficiencies. Xerostomia is often caused by failure of the salivary glands to function normally, but it may also occur in people with normal salivary glands. Xerostomia can cause health problems by affecting nutrition, and psychological health. Extreme cases can cause rampant tooth decay and periodontal disease. SIALOLITHS (salivary calculus or stones) in Wharton’s Duct (Submandibular Duct) are often found with OCCLUSAL RADIOGRAPHS. Calculus deposition in the salivary ducts and glands is more common in middle-age. The most common symptoms of duct obstruction are an increase and decrease in swelling of the gland (especially at mealtime). The swelling may or may not be painful, and may also occur in children. Transillumination of the soft tissue is useful to detect sialolithiasis in children. • Rate of occurrence in the SUBMANDIBULAR GLAND & DUCT is much higher than in the parotid or sublingual glands due to the submandibular saliva’s tenacity, and long, irregular duct shape. • Treatment: surgical extirpation of the sialolith. Salivary stones located in the glandular parenchyma also usually require surgical removal of the gland.

• OCCLUSAL RADIOGRAPHS are NOT useful for identifying sialoliths in STENSON’S DUCT, the mental foramen, or hyoid bone.

ONCOCYTOMAS (“OXYPHILIC/ACIDOPHILIC ADENOMA”) – a small, benign RARE glandular tumor composed of large cells with a granular & eosinophilic cytoplasm due to the presence of many mitochondria. Can occur in the kidney, salivary glands, & endocrine glands. Its development may be related to the AGING PROCESS. Treatment: surgical excision; recurrence is rare. • Clinical Features: most commonly seen in the PAROTID GLANDS of patients over age 50 (slightly more common in women). Slow growth, and they rarely reach any significant size. • Histologic Features: the tumor is an encapsulated mass composed of relatively large cells (arranged in sheets, cords, or form tubular or acinar structures). Cells have

BRIGHT PINK CYTOPLASM and small, round nuclei. SIALOSCINTIGRAPHY-a simple, non-invasive procedure that SEPERATES BENIGN TUMORS (ex: Warthin’s tumor & Oncocytoma) of the salivary glands from MALIGNANT TUMORS, and greatly affects the course of treatment. WARTHIN’S TUMOR (“PAPILLARY CYSTADENOMA LYMPHOMATOSUM”)-BENIGN PAROTID tumor (almost exclusively in the PAROTID GLAND), arising from heterotopic ductal epithelium within lymph nodes or near the parotid gland. Strong association with cigarette smoking. • Clinical Features: most patients are older 60-70 years (affects males 5:1). 95% are unilateral. The tumor most often arises in the tail of the parotid gland as a painless, non-tender, slow growing, firm-to-fluctuant nodule over the ANGLE or RAMUS OF THE MANDIBLE. • Histologic Features: tumor is encapsulated and composed of CYSTIC SPACES surrounded by two uniform rows of cells with centrally placed pyknotic nuclei. The lining epithelium (papillary infoldings) of the cystic spaces protrude back into the spaces. The papillary foldings have lymphoid stroma aggregates with a geminal center formation that are interspersed.

• Treatment: SURGERY. Recurrence is uncommon and it is highly unlikely to become malignant. PLEOMORPHIC ADENOMA (“BENIGN MIXED TUMOR”) –the MOST COMMON tumor of the major & minor salivary glands. The term “mixed tumor” is used because it is of both ectodermal & mesenchymal origin. • Clinical Features: MOST COMMON SALIVARY GLAND NEOPLASM. Affects more women than men (most patients between 40-60yrs). ~93% arise in MAJOR salivary glands (these are almost exclusively PAROTID tumors (84%), but can also affect the submandibular gland. Present as firm, painless lumps below and anterior to the ear. ~7% arise in the oral cavity with the PALATE being by far the most common intra-oral site (firm, painless swelling). In most cases, it does not cause ulceration of the overlying mucosa.

• Histologic Features: epithelial (ectodermal) component consists of round, polyhedral, elongated, or stellate cells that are relatively small and stain uniformly. The mesenchymal component varies from areas of myxomatoid tissue, to areas of dense, hyalinized C.T., pseudocartilage, or bone. • Treatment: SURGICAL EXCISION with a generous margin of normal tissue is the treatment of choice. Inadequate initial removal of the mixed tumor in major salivary glands may cause recurrence. ~25% of benign mixed tumors undergo malignant transformation untreated for an extended period.

ULCERATIVE CONDITIONS STEVENS-JOHNSON SYNDROME –a SEVERE bullous form of ERYTHEMA MULTIFORME where systemic symptoms are severe, and lesions are extensive, involving multiple body areas (especially mucous membranes). SJS is characterized by the acute onset of fever, and eruptive, ulcerative lesion on the skin, oral mucosa, and eyes. Frequently, genitalia, lungs, and joints are affected, and it can be fatal. Blindness can occur due to a secondary infection. • Clinical Features: typical “BULL’S-EYE-SHAPED” LESIONS with the classical triad of eye lesions, genital lesions, & stomatitis. The lesions are severe and often vesicular or bullous, with hemorrhage after denudation. • Treatment: IV fluids, systemic steroids, palliative rinses and antibiotics. ERYTHEMA MULTIFORME-allergic hypersensitivity reaction in response to MEDICATIONS (sulfonamides, penicillins, barbiturates, & phenytoin), infections, or illness. Associated infections include herpes simplex & mycoplasma infections. The exact cause is unknown, but is believed to involve DAMAGED SKIN BLOOD VESSELS

with subsequent damage to skin tissues. EM occurs mainly in children and young adults, and its diagnosis is based on the classic target OR “BULL’S-EYESHAPED” SKIN LESION that appears as a central lesion surrounded by concentric rings of pallor and redness over the dorsal (top) of the hands and forearms.

• Low-grade fever, general malaise, and headache usually precede the lesions by 4-7 days. • Oral lesions appear as red macules, papules, or vesicles that may become eroded and painful, covered by a yellowish-white membrane after rupturing. • Treatment: topical palliative rinses and sometimes low-dose systemic steroids.

RECURRENT APHTHOUS ULCERS (ULCERATIVE/APHTHOUS STOMATITIS “CANKER SORES”) –may be associated with STRESS, a bacterial infection, trauma (self-inflicted, oral surgery procedures, routine dental procedures), endocrine conditions (menstrual cycle), allergic factors (certain foods or drugs), immunologic abnormalities, iron, vitamin B, or folic acid deficiencies. Cause is unknown, but they may be caused by an autoimmune reaction. 1. Recurrent Aphthous Minor (Minor Aphthous Ulcer)the MOST COMMON form (“canker sore”). Occurs more often in women, starting as a single or multiple superficial erosion covered by a gray membrane (1-5 lesions). Looks like an erythematous halo with yellowgrayish color with a red border). The lesion can be VERY PAINFUL, and varies from 2-10mm in diameter. Generally last for 7-14 days, and heal gradually with LITTLE OR NO SCARRING. Occurs on cheeks, lips, tongue, roof of mouth. 2. Recurrent Aphthous Major (Major Aphthous Ulcer) large, usually > 10mm, VERY PAINFUL ulcers that occur frequently. Patients usually get more than one ulcer at a time. Unlike minor aphthous ulcers, major aphthous ulcers can last up to 6 weeks and LEAVE A SCARE UPON HEALING. Occurs on non-keratinizing tissue (LIPS, TONGUE, CHEEKS), but their border can extend onto keratinized tissue.

3. Recurrent Herpetiform Ulcerations-the MOST SEVERE aphthous ulcer, characterized by crops of multiple, small, shallow ulcers, often in cluster of up to 100 (1-3mm lesions), and may occur in any area of the oral cavity. These lesions are present almost continuously for 1-3 years, with relatively short remissions. More common in adult FEMALES, and usually heal within a month and DO NOT LEAVE SCARRING.

*Exact cause is unknown, but citrus, stress, allergies, immune reactions, and B12, iron, zinc, and folic acid deficiencies may contribute (manifestation of Behcet’s Disease & Crohn’s Disease) Some Treatments: Non-alcoholic mouthwash, steroids.

Important: vesicular lesions (vesicles) DO NOT PRECEDE the formation of these ulcers. This is a distinctive diagnostic feature. When an ulcer heals, the epithelium that will eventually cover the defect is derived from intact epithelium at the ulcer margin. ACTINOMYCOSIS – a subacute-to-chronic bacteria infection with Actinomyces (A. israelii) which are gram (+) filamentous bacteria that are normal inhabitants of the oral cavity and GI tract, characterized by contiguous spread, suppurative & granulomatous inflammatory reaction, and formation of multiple abscesses and sinus tracts that discharge SULFUR GRANULES. This infection is most likely to cause CHRONIC SUPPURATIVE LESION about the jaws. The most common clinical forms are cervicofacial actinomycosis (LUMPY JAW), thoracic, and abdominal actinomycosis. CERVICOFACIAL ACTINOMYCOSIS (“LUMPY JAW”) – the most common manifestation of Actinomycosis. Infection typically occurs in patients with poor dental hygiene or after surgery. In the initial stages, there is soft tissue swelling of the perimandibular area. Direct extension into the adjacent tissues occurs over time, along with the development of fistulas that discharge purulent material containing yellow sulfur granules (these granules are actually colonies of bacteria). HISTOPLASMOSIS – disease caused by the FUNGUS

Histoplasma Capsulatum. The symptoms vary greatly, but this disease primary affects the LUNGS. Sometimes affects other organs (Disseminated Histoplasmosis). Histoplasmosis infection is usually asymptomatic, but may produce a benign, mild pulmonary illness (the primary form of the disease). • Oral Manifestations: nodular, ulcerative, or vegetative lesions on the buccal mucosa, gingiva, tongue, palate, or lips usually covered by a non-specific indurated gray membrane. SYPHILIS – a sexually transmitted disease caused by a spirochete Treponema Pallidum that occurs in 3 stages (primary, secondary, & tertiary). Syphilis is usually treated with a PENICILLIN injection. 1. Primary Syphilis-the first symptom is a non-painful ulcer (“canker”) that appears 2-6 weeks after exposure/infection. Found on the body part exposed to the partner’s ulcer (i.e. penis, vulva, or vagina). It can also develop on the cervix, tongue, lips, or other parts of the body. The chancre disappears within a few weeks even without treatment. If not treated during the primary stage, ~33% of people will progress to chronic stages.

2. Secondary Syphilis-a highly infectious stage that occurs 6 weeks after non-treatment of primary syphilis. Often marked by a skin rash characterized by brown “penny-sized” sores. Widely disseminated spirochetes cause mucous membranes to exhibit a reddish-brown maculopapular cutaneous rash and ulcers covered with a mucoid exudates (mucous patches). Condylomata lata (elevated broad-based plaques) are also seen on the skin and mucosal surfaces. 3. Tertiary Syphilis-occurs in infected people many years after non-treatment of secondary syphilis. A GUMMA (a focal nodular mass) typifies this stage. Most commonly occurs on the PALATE & TONGUE. Bacteria damages the heart, eyes, brain, nervous system, bones, joints, or almost any other part of the body. In this late stage, untreated syphilis, although not contagious, can cause serious heart abnormalities, mental disorders,

blindness, other neurologic problems, and death. Headache, stiff neck, and fever are symptoms of neurosyphilis. CONGENITAL SYPHILIS – caused by an infection by the spirochete bacteria Treponema Pallidum during the fetal period. Expectant mothers with syphilis can transmit the disease through the placenta to the unborn infant. Nearly 50% of all infants infected during gestation die shortly before or after birth. The severity of congenital syphilis depends on the time the organisms pass the placental barrier (protected up to the 16th week), the mother’s stage of syphilis, and the fetus’s immunologic response. If treated by the 4th or 5th month of pregnancy, 95% show no manifestations. However, if untreated, fetal sepsis may result in stillbirth or visceral & mucocutaneous manifestations.

• Newborn Symptoms: irritability, blood discharge from nose, early rash (small blisters or flat or bumpy rash on the face, palms, and soles), failure to thrive, later rash (copper-colored vesicles on the palms and soles), saddle nose, frontal bossing, short and high maxilla. • Infant/Child Symptoms: bone pain, joint swelling, abnormal teeth (HUTCHINSON’S INCISORS) pegshaped WIDELY SPACED permanent incisors, and notched at the end with a centrally placed crescentshaped deformity. Also gray, mucous-like patches on the anus and vulva (Condyloma lata), saber shins (bone abnormality of the lower leg), visual loss, CN VIII nerve deafness & intestinal keratitis, and scarring of the skin around earlier lesions of the mouth, genitalia, and anus (“rhagades”).

• Manifestations of congenital syphilis: large frontal bone, Saddle nose, Hutchinson’s teeth (very wide incisor crowns), & MULBERRY MOLARS (look like a mulberry). Not all people with mulberry molars have syphilis. There is NO chancre in congenital syphilis.

VESICULO-BULLOUS DISEASES HERPES SIMPLEX VIRUS – always a vesicle (small virus filled with fluid) that breaks and the crust forms due to dried fibrin. Related to viruses that cause Mononucleosis, Chickenpox, and Shingles (not Mumps). HSV is one of the most common viral diseases of man. HERPES SIMPLEX VIRUS TYPE 1 – oral herpes (affects outside of lips, face, skin, & oral mucosa) TRANSMITTED BY DIRECT CONTACT. HSV-1 includes herpetic gingivostomatitis (primary & secondary, and herpes labialis). It is labial & intra-oral herpes that are groups of small ulcerations in the hard palate, outside of lips, gingiva, or hands and fingers. • HSV Treatment: “SUPPORTIVE” focused on relieving the acute symptoms so that fluid and nutritional intake can be maintained. Treatment involves analgesics, topical anesthetics before eating, maintaining electrolyte balance, and anti-viral agents. NO CORTICOSTEROIDS (contraindicated)! PRIMARY HERPETIC GINGIVOSTOMATITIS (Acute Herpetic Gingivostomatitis)-the primary herpes infection (HSV-1) that mainly affects YOUNG CHILDREN (under age 5), but may also affect young adults (15-

25yrs). It usually occurs in a child who has not had any contact with HSV-1, and who thus has no neutralizing antibodies. Nearly all primary infections are of the subclinical type that may only have flu-like symptoms, with 12 mild sores in the mouth that go unnoticed by parents. In other children, the primary infection may be manifested by acute symptoms (acute herpetic gingivostomatitis) these prodromal symptoms are (fever, malaise, irritability, headache, dysphagia, vomiting, cervical lymphadenopathy) 1-2 days prior to local lesions. Then, fiery red gingival tissues and small yellowish vesicles form that rupture quickly, causing painful shallow, round, discrete ulcers with an erythematous (red) halo on the FREE & ATTACHED MUCOSA. Thus, the primary HSV-1 infection ranges from sub-clinical (asymptomatic) to severe systemic infections. Dehydration is the most serious potential problem due to the child not wanting to eat or drink because of the pain. A generalized marginal gingivitis may precede the ulcers.

PRIMARY HERPETIC GINGIVOSTOMATITIS most commonly occurs in CHILDREN & YOUNG ADULTS. Patients develop FEVER, irritability, regional lymphadenopathy, and headache. Within days, the gingiva is intensely inflamed. Any part of the oral mucosa and lips may become involved. Vesicles then form and rupture shortly later to leave shallow VERY PAINFUL ulcers covered with a GRAY MEMBRANE and surrounded by a RED HALO. Ulcers heal on their own in 7-14 days. • Treatment: fluid intake, good oral hygiene, and gentle debridement of the mouth. In healthy individuals, the lesions heal in 7-14 days, and the ulcers heal WITHOUT SCARRING. • After recovery from primary HSV, the virus is not

cleared from the body, but lies dormant in a nonreplicating state in the sensory nervous system (trigeminal ganglion). Periodically, latency reactivates and allows the virus to return to the skin or mucous membranes, where it causes a recurrent infection. • Primary herpes occurs only in YOUNG PATIENTS (baby, children, adolescents). Fever, malaise, pain, or could be CHICKENPOX (varicella zoster virus) in a 6yr old child. Multiple vesicles, sick for one week. • AFTER the initial primary attack during early childhood, HSV-1 remains inactive in the TRIGEMINAL GANGLION (sensory nerve ganglia), but often reappears as the familiar “cold sore” on the OUTSIDE of the lips (“Recurrent Herpes Labialis”). Emotional stress, trauma, and excessive sun exposure may cause recurrent herpetic lip lesions. Acyclovir 5% ointment (Zovirax) can successfully reduce the duration and severity of these lip sores. SECONDARY (RECURRENT) HERPETIC STOMATITISgenerally occurs in ADULTS, triggered by trauma, fatigue, RTI (respiratory tract infection), stress, allergy, or UV exposure that causes the release/reactivation of the latent HSV-1 virus. This reactivation causes a recurrent infection (i.e. cold sores) on the lips (that is bound to

periosteum), hard palate, attached gingiva, and alveolar ridge. Site-specificity is a characteristic manifestation. • Recurrent Herpes Simplex Virus-COLD SORES on the lips are the most common manifestation of infection. Some factors often associated with a recurrent outbreak are sunburn, fatigue, emotional upset, trauma, upper respiratory tract infection, or menstruation. Often one day before the vesicles form, tingling or itching of the skin or mucosa occurs. Vesicles ulcerate and resolve just like in primary herpes. • Remains LOCALIZED on the lower lip or inside the mouth. Lasts about 2 weeks then heals by itself, but recurrent herpes virus always lays dormant in the trigeminal nerve ganglion. May or may not have fever, malaise. May present with vesicles in hard palate. HERPES LABIALIS (Fever Blisters/Cold Sores)extremely common HSV-1 disease. Characterized by an eruption of small, usually PAINFULL blisters on the skin of the lips, mouth, gingiva, or skin around the mouth. The reason most patients suffering from Recurrent Herpes Labialis rarely give a history of having had acute herpetic gingivostomatitis is the primary infection was subclinical. HSV-1 lesions (cold sores) are found more forward in the mouth (tongue, gingiva, buccal mucosa) appearing as

vesicles (small, clear, ulcerated, and crusty blisters) around the mouth and lips.

Histological Features: cytopathic effect (CPE) takes the form of BALLOONING DEGENERATION of epithelial cells with loss of cohesion to adjacent cells. The nuclei are often multiple, with margination of the chromatin around the intranuclear inclusions called Lipschultz Bodies. These changes are seen in Tzanck smear scrapings taken from an unroofed vesicle. HERPES SIMPLEX VIRUS TYPE 2 (“HERPES GENITALIS” OR “GENITAL HERPES”)– transmitted via SEXUAL CONTACT, and affects the mucosa of the genitalia and anal regions, but may also occur in mouth. Not as common as HSV1. “Genital Herpes” may have serious consequences in pregnant women because the virus can be transmitted to the infant during vaginal

delivery, and can damage the infant’s CNS and/or eyes. HERPES SIMPLEX VIRUS TYPE 3 (HSV-3) –caused by Herpes VARICELLA-ZOSTER virus (DOES NOT CROSS THE MIDLINE). Produces recurrent herpes (adult shingles/herpes zoster), & chickenpox (primary herpes). Occurs after activation of varicella virus, the affected skin has VERY PAINFULL ulcerated redicule vesicles, and the lesion follows the path of the trigeminal nerve. May occur in an elderly patient with a history of leukemia (compromised immune system). VARICELLA VIRUS – can cause Herpes Zoster lesions along sensory nerve roots in later life. It is a member of the herpes virus group that causes chickenpox (varicella) & shingles (herpes zoster). Varicella virus is highly contagious and may be spread by direct contact or droplets. The histology of chickenpox and shingles shows the same cytopathic effect as herpes simplex. • Chickenpox-primarily a disease of CHILDREN that peaks at school-age in winter and spring, characterized by the appearance on the skin and mucous membranes of successive crops of typical pruritic vesicular lesions that break easily and scab. Usually accompanied by mild constitutional symptoms (fever, malaise), and is very contagious 1 day before the rash’s onset and until all of the vesicles have crusted. It is relatively benign in

children, but adult infection may be complicated by pneumonia and encephalitis. ZIG (Zoster Immune Globulin) reaches morbidity in high-risk children. • Shingles (Herpes Zoster)-caused by the reactivation of a LATENT varicella-zoster virus that may have remained in the body from a childhood chickenpox. The virus reaches sensory ganglia of the spinal and cranial nerves, producing an inflammatory response. Characterized by painful vesicles on the skin or mucosal surfaces along the distribution of the sensory nerve. • Shingles (Herpes Zoster) Example: 16-yr old male with fever & malaise, DOES NOT CROSS THE MIDLINE, and is very painful. Can also be in a 65year old male with a history of leukemia (weak immune system) who had chickenpox as a child. The affected skin shows vesicles and lesions follow the path of the nerve due to activation of varicella virus. HERPES SIMPLEX VIRUS TYPE 4 –caused by EPSTEINBARR VIRUS (EBV) a member of the herpes virus group that causes Infectious Mononucleosis (Kissing Disease), Hairy leukoplakia, Chronic Fatigue Syndrome, and two cancers (Burkitt’s Lymphoma & Nasopharyngeal Carcinoma). 1. HAIRY LEUKOPLAKIA-this is NOT hairy tongue.

Appears 99% of time in HIV+ patients. ALWAYS white furry lesions on LATERAL tongue borders (bilateral) caused by EPSTEIN-BARR VIRUS. Can usually diagnose without biopsy. Anti-viral medication removes the lesion, but it reappears when patients stops taking medications. Only treat if the patient requests with anti-viral medication. Presents as a WHITE extensive area in the LATERAL border of the tongue, present for 4 months, asymptomatic. The surface lesion is irregular, bluish areas on the gingiva, candida, and recurrent herpes infection may coexist. 99% occurs in HIV+ patients, so patient should get an HIV test. 2. INFECTIOUS MONONUCLEOSIS (HSV-4)-no specific oral manifestations, but secondary lesions occur and neck swellings, which are also characteristic of Hodgkin’s Disease and Tuberculosis. BURKITT’S LYMPHOMA (HSV-4)-cancer caused by EPSTEIN-BARR VIRUS. High-grade Non-Hodgkin’s Lymphoma (cancer)/neoplasm with a viral etiology endemic in AFRICA, but occurs sporadically in North America. There are significant differences between the African & Non-African forms, but BOTH are histologically identical. One manifests most often as a large osteolytic jaw lesion (African Form), the other as an abdominal mass (Non-African Form). • Jaw lesions usually present as expanding intra-oral

masses with mobility of the involved teeth. Radiographically: “MOTH-EATEN” appearance with poorly marginated bone destruction. • Non-African Burkitt’s Lymphoma patients are generally older (ages 9-12yrs), with no gender predilection. Presents most often as an ABDOMINAL MASS involving mesenteric lymph nodes or Peyer Patches in the ileocecal region, often with intestinal obstruction. Involvement of the gonads, retroperitoneum, and other viscera are less common. • Burkitt’s lymphoma is the FIRST HUMAN CANCER WITH STRONG EVIDENCE OF VIRAL ETIOLOGY. Epstein-Barr virus (herpes-type virus) as these patients have high titers of antibodies against EBV. Important: Epstein-Barr is also associated with infectious mononucleosis, & orally hairy leukoplakia.

African Burkitt’s Lymphoma (African Jaw Lymphoma)affects younger patients (age 3-8yrs) than the non-African form. Affects males more than females (2:1), and typically involves the mandible, maxilla, & abdomen. Extra-nodal involvement of the retroperitoneum, kidneys, liver, ovaries, and endocrine glands may be affected. HERPES SIMPLEX VIRUS TYPE 5 – cytomegalovirus that affects salivary glands. HERPES SIMPLEX VIRUS TYPE 8–associated with Kaposi’s sarcoma-AIDS/HIV. Kaposi Sarcoma: 34-year old male with AIDS, common on palate. Can have Kaposi Sarcoma without AIDS, but in U.S. it is usually with AIDS. It’s a superficial cancer on the skin, multiple, made of blood vessels, & is NEGATIVE to pressure test (so don’t confuse with Hemangioma which is pressure test positive +). Kaposi’s Sarcoma is very common on the PALATE. Mostly associated with HIV, but can have Kaposi’s sarcoma even without AIDS. (-) to pressure test. RED LESIONS on the hard palate. KAPOSI’S SARCOMA–an oral manifestation most commonly associated with AIDS. It is a MALIGNANT NEOPLASM originating in the skin, characterized by abnormal vascular proliferation (cancer of the lining of the blood vessels). It occurs on multiple sites, especially lower extremities. The initial lesions are small, red papules that enlarge and fuse to form purple-to-brown spongy

nodules. It spreads to lymph nodes and internal organs. • HARD PALATE is the most common intra-oral location of KAPOSI’S SARCOMA, followed by the gingiva and buccal mucosa. • Important: AIDS is caused by an RNA retrovirus (HIV = HTLV-III) acquired by sexual contact (homosexual & heterosexual), or contaminated blood products.

TESTS TO DIAGNOSE HERPETIC LESIONS: 1. Tzanck Smear-a cytologic examination of fluid harvested from an unopened vesicle stained with giemsa, and viewed by the light microscope. Pathologist looks for LIPSHULTZ BODIES (epithelial cells with intra-nuclear inclusions). 2. Fluorescent Staining-cells show (+) fluorescence when stained with fluorescent labeled HSV immune serum and globulin. Used to distinguish between herpes zoster & herpes simplex.

3. Isolation in Tissue Culture 4. Antibody Titers (Anti-HSV Ab Titers)-a test for complement fixing or neutralizing antibody in acute and convalescent sera, and on tissue sections (begins in 1 week, and peaks at 3 weeks). 5. Biopsy-shows intra-epithelial cleft covered by an exudates of fibrin & PMN leukocyte. The epithelium exhibits degenerative cells that include bizarre giant cells, and cells with displaced chromatin with perinuclear halos and inclusions. *Arthrogram is NOT a test to diagnose herpetic lesions. HERPANGINA –an acute infectious disease (stomatitis/mouth inflammation) affecting YOUNG CHILDREN caused by a Group A Coxsackie Virus. It is differentiated clinically from HSV-1 (cold sore virus) as herpangina oral ulcerations/vesicles usually occur in the BACK OF THE THROAT around the tonsils and posterior palate. Herpangina may also appear on the tongue. • Clinical Features: are mild and of short duration compared to HSV-1. Herpangina begins with a sore throat, fever, headache, and sometimes vomiting & abdominal pain. Papules or vesicles soon form in the pharynx, and evolve into shallow ulcers that heal spontaneously. Herpangina usually runs its course in

less than 1 week. Treatment: PALLIATIVE.

HAND, FOOT & MOUTH DISEASE (HFMD)- VIRAL INFECTION (usually Coxsackie A) that usually affects infants and children. Produces red-erythematous lesions/macules/vesicles areas in the mouth, foot, & hands. Oral lesions/vesicles appear on the buccal mucosa, tongue, gingiva, & lips. • Patient has fever, malaise, vomiting, fatigue (may be confused with a cold). HFMD is self-limiting, disappearing by itself in < 15 days, & does not return. Erythematous macules develop central vesicles. A very mild disease that is often not diagnosed. No treatment, but symptoms may be eased with analgesics. BENIGN MUCOUS MEMBRANE PEMPIGOID (BMMP) – a chronic self-limiting mucocutaneous autoimmune disease, usually limited to oral and ocular mucous membranes

(conjunctival involvement may cause blindness). Usually effects women over age 50yrs. More common in the oral cavity, but much less severe (better prognosis) than Pemphigus. Occurs more often than realized due to improper diagnosis. • A vesiculobullous disease where auto-antibodies act against basement membrane components (below epithelium = sub epithelial vesicle), between epithelium & C.T. so destruction is below the epithelium. Usually shows nice epithelium since the epithelium is complete & C.T. is separated (not seen in Pemphigus). • Oral lesions usually present as a “desquamative gingivitis” in which vesicles form, rupture, and leave gingival erosions. Found in all mucous membranes, but is less severe than Pemphigus. • Treatment: Biopsy and systemic STEROID therapy. PAINFUL ORAL ULCERS. Important: MAJOR histological difference between BMMP & Pemphigus Vulgaris is BMMP vesicles are sub epidermal with no acantholysis, while with Pemphigus Vulgaris, there is acantholysis and supra-basilar vesicle. PEMPHIGUS VULGARIS – a chronic RARE SKIN DISEASE characterized by the formation of vesicles &

bullae produced by dyhesion (acantholysis) of epidermal cells due to an autoimmune mechanism where antibodies attack the intracellular junction of epithelium. Usually occurs after age 30yrs (usually ages 30-50yrs), and occurs more frequently in JEWISH people. • Oral lesions are often the FIRST MANIFESTATION. Intact bullae are rarely seen in the oral cavity, rather, large areas of ulceration and erosions covered by a white or blood-tinged exudates. Sometimes, areas of epithelium will slide off simply by rubbing of an apparently unaffected area (Nikolsky’s Sign is an indication of Pemphigus vulgaris that may also be found in BMMP). This sign occurs when apparently normal epithelium may be separated at the basal layer and RUBBED OFF when pressed with a sliding motion. Pemphigus is often fatal without treatment, which includes high-dose systemic steroids or chemotherapy (Methotrexate).

• Histologic Features: vesicles and bullae are formed entirely intra-epithelially, just ABOVE the basal layer of cells (suprabasilar vesicles). There is intercellular edema & loss of intercellular bridges with loss of cohesiveness (acantholysis or dyhesion). Clumps of cells are often found floating free in the vesicle space (Tzanck cells). • Pemphigus Vulgaris is the most common form, and is more severe than Pemphigoid (BMMP). With Pemphigus, the patient has auto-antibodies produced against epidermal cell surface glycoprotein (a component of desmosomes that attaches epithelial cells together)

causing painful oral ulcers. More common in older females (> 50yrs). Intraepithelial vesicle. A very bad disease whose only treatment is STEROIDS. Patient has vesicles on the skin. GINGIVA is red, inflamed and is the most common location in mouth. • MUST DO BIOPSY TO DIAGNOSE using 2 small portions of tissue (one in a vile, the other goes into a vile of Michel solution (the fixative to do the immunofluorescence). If bullae is intraepithelial then its Pemphigus Vulgaris. If bullae are between epithelial & C.T. it is Pemphigoid. Produce Ab against desmosomes (attach one epithelial cell to another), so desmosomes are destroyed and the epithelium is broken causing gingival ulcerations. Tissue in the “Michel solution” shows yellowish fluorescence areas show where the antibodies are against desmosomes. RUBELLA VIRUS–causes German Measles (Rubella). Characteristic RASH (flat, pink spots on the face that spreads to other body parts). Oral manifestations may include swollen and congested tonsils, and red macules. HEPATITIS A (VIRAL OR INFECTIOUS HEPATITIS) – caused by an RNA Enterovirus usually transmitted by the FECAL-ORAL ROUTE. An infectious disease of the LIVER that most often occurs in young adults and is prevalent in areas with inadequate sewerage. SHELLFISH from

contaminated waters is also a prime source. Initial symptoms (fever, abdominal pain, nausea, then jaundice) appear after an incubation period of 3-6 weeks. Vaccination exists. • Damage to liver cells causes increased serum levels of enzymes like transaminases, normally active in liver cells. The detection of the increased serum levels of transaminases helps diagnose the disease. In most cases of Hepatitis A, the infection is self-limiting, and recovery occurs within 4 months. HEPATITIS B (“SERUM HEPATITIS”) – an infectious LIVER DISEASE caused by a DNA virus that produces liver inflammation and necrosis. Main method of transmission is exposure to contaminated blood or serum (but can be transmitted sexually or via blood transfusions). There is a high rate of transmission among drug addicts who may use contaminated needles. Vaccination exists. • Signs & Symptoms: similar to Hepatitis A (fever, abdominal pain, nausea), but it has a longer incubation period of 2-3 months. Symptoms develop slower, but last longer. Most patients fully recover, but some develop chronic liver disease. • Hepatitis B transmission is a MAJOR CONCERN to the dental profession, which have at least 3x higher risk than the general population to acquire this virus. Thus,

ALWAYS USE UNIVERSAL PRECAUTIONS. Hepatitis D is found ONLY in patients with acute or chronic episodes of Hepatitis B. • Hepatitis B vaccine is recommended for all health care personnel. A series of 3 doses is required (the 2nd and 3rd doses are given at 1 & 6 months after the 1st dose. The injection is given IM (in the deltoid muscle). • The presence of SURFACE ANTIGEN (A or B) in a patient’s serum indicates the patient is potentially infected with Hepatitis (“carrier state”). Hepatitis viruses are HIGHLY HEAT RESISTANT (even more than AIDS virus). However, autoclaving properly kills them. HEPATITIS C-an infectious liver disease that can cause liver scarring and cirrhosis. Spread mainly via blood-toblood contact associated with IV drug use, poorly sterilized medical equipment, and transfusions. Hepatitis C virus persists in the liver and can be treated with medication (Peginterferon and Ribavirin). There is no vaccine, and liver transplantation may be required for severely damaged livers. IMPORTANT DIAGNOSTIC TERMS: • BIOPSY – the MOST RELIABLE technique to diagnose soft tissue lesions. Scalpel is the instrument of choice since it cleanly removes tissues without dehydrating them as cautery or a high-frequency cutting

knife may. FORMALIN (10%) is the fixative of choice. The rationale for surgical removal and biopsy of a large periapical lesion suspected to be of inflammatory origin is that a clinical diagnosis can be confirmed microscopically. Biopsy is the only way to distinguish between a granuloma and a cyst. • EXCISIONAL BIOPSY –involves TOTAL EXCISION of a small lesion for microscopic study. This is preferred if the lesion size is such that it may be removed along with a margin of normal tissue, and the wound can be closed primarily (ex: a 1cm exophytic mass-a lesion that grows outward from an epithelial surface on the cheek). • INCISIONAL BIOPSY (DIAGNOSTIC BIOPSY) – removes only a SMALL SECTION of tissue for examination. Done when lesions are too large to excise initially without having established a diagnosis, or are of such a nature that excision would be inadvisable.

RADIOGRAPHIC PATHOLOGY MOST TO LEAST COMMON LESIONS 1. PERIAPICAL ABSCESS—well-defined ovoid-shaped radiolucenty at root apex.

2. DENTIGEROUS CYST—large, well-defined unilocular radiolucency associated with unerupted molar clinical crown.

3. Incisive Canal Cyst (nasopalatine duct cyst)-heartshaped well-circumscribed radiolucency of midline of hard palate:

4. Lateral Periodontal Cyst-well circumscribed unilocular radiolucency lateral to the roots of the premolars (between roots of lower premolars).

5. Residual Cyst-large, round radiolucency of mandible in the area of a previously extracted molar.

6. Traumatic Bone Cyst (Simple Bone Cyst)-welldeliniated radiolucent lesion with projections between the roots of the vital premolar and molar teeth producing a SCALLOPED appearance.

7. AMELOBLASTOMA-large, multilocular radiolucent lesion of the posterior body and ramus of the mandible.

8. ODONTOGENIC KERATOCYST-large well-delineated radiolucent leion of the body of the mandible.

9. CENTRAL GIANT CELL GRANULOMA-large radiolucent lesion with scalloped borders of the body of the mandible—root resorption often accompanies.

10. OSTEOMYELITIS-poor defined radiolucenies of the body and ramus of mandible.

11. PERIAPICAL CEMNTO-OSSEOUS DYSPLASIA-well circumscribed mixed radiolucent and radiopaque lesions at the apical area of the mandibular anteriors.

12. TORI-bilateral mandibular tori superimposed on the canine roots appearing as round, smooth-surfaced radiopaque structures on mandibular lingual alveolar ridge bilaterally in premolar areas:

13. ROOT TIP: radiopaque area of the alveolar ridge with the structural appearance of the portion of a tooth root. A thin periodontal ligament may be visible:

14. CONDENSING OSTEITIS-localized area of increased radiodensity of alveolar bone from the apical portion of the root into the underlying bone forming a bulbous appearing radiodense mass. Faint outline of the root visible within the bone showing the increased radiodensidty.

15. OSTEOSCLEROSIS: localized irregular-shaped area of bone with increased radiodensity in the area of a missing tooth.

16. COMPOUND ODONTOMA-dome shaped radio dense mass above the crown of an unerupted tooth composed

of numerous small tooth-like structures, surrounded by a radiolucent zone.

17. COMPLEX ODONTOMA-large, irregular shaped radiodense mass preventing tooth eruption. The radiodensity of the mass resembles the adjacent teeth.

RADIOLOGY Radiographs show shading from black to white (most radiolucent to most radiopaque). From least to most radiopaque: (PDL space, dentin, enamel, ZOE, and amalgam). Radiopaque Structures & Dense Materials: metals, enamel, dentin, and bone that INHIBIT the passage of xrays and appear WHITE on the processed film. Less radiation penetrates the structure and reaches the film. Radiolucent Structures and Materials: less dense materials like soft tissue and air space that appear gray to black on processed film) ALLOWING RADIATION to pass through by absorbing very little radiation. More radiation penetrates the structure and reaches the film. • Radiolucent Lesions-appear every time bone is DESTROYED. Unilocular & Multilocular are terms only used to describe radiolucent lesions. 1. well-defined unilocular (one cavity): border is well-defined. Most benign lesions are unilocular, well-defined. 2. well-defined multilocular: border is welldefined with several cavities. 3. well-defined honeycomb or soap bubble

(multilocular): 4. diffuse-cannot follow the border of the radiolucency: 90% of the time it is cancer. If loss of cortical plates, the first diagnosis is cancer. OSTEORADIONECROSIS-the necrosis of bone produced by ionizing radiation that is more common in the MANDIBLE than maxilla due to the richer vascular supply to the maxilla, and because the mandible is more often irradiated. The most common factors precipitating osteoradionecrosis are pre-irradiation & post-irradiation extractions, and periodontal disease. Damage to blood vessels (not nerves or muscles) predisposes a patient to developing osteoradionecrosis. A complications that can occur with patients taking IV Bisphosphonates or oral bisphosphonates for more than three years orally (i.e. Fosamax). Osteoradionecrosis is more common IN THE MANDIBLE probably because of the richer vascular supply to the maxilla and the fact that the mandible is more frequently irradiated. Caution with patients on IV bisphosphonates or if taking oral bisphosphonates (i.e. Fosamax). • May occur after dental extractions. • The most common factors precipitating osteoradionecrosis are pre and post irradiation extractions and periodontal disease.

HAMULAR PROCESS-the bony projection that arises from the SPHENOID BONE and extends downward and slightly posteriorly. On the radiograph, it is seen in proximity to the posterior surface of the maxillary tuberosity. It varies greatly in length, width, and shape among patients. It usually exhibits a bulbous point, but sometimes the point is tapered.

CORONOID PROCESS-the mandible anatomical landmark that often appears in the periapical radiographs of the MOLAR REGION of the MAXILLA. As the mouth is opened, the coronoid process moves forward, and thus comes into view most often when the mouth is opened to its fullest extent at the time the radiograph is taken. It appears as a tapered or triangular radiopacity seen below, or sometimes superimposed on the molar teeth and maxilla.

Dental Radiographs should be retained INDEFINITELY. The dental record must include documentation of informed consent and the exposure of radiographs (i.e. number and type of films, rationale for exposure and interpretation). Legally, dental radiographs are the PROPERTY OF THE DENTIST. However, patients have a right to reasonable access to dental radiographs, which includes having a copy forwarded to another dentist. • Patients may refuse dental radiographs, but the dentist must decide whether an accurate diagnosis can be provided, and whether treatment can be provided. • No document can be signed by the patient that releases the dentist from liability. DIGITAL RADIOGRAPHY-requires nearly 90% LESS

radiation than traditional radiography because the sensor is more sensitive to x-rays. Also, radiation exposure to the patient is reduced 50-80% with digital radiography. The sensor is used in place of film in digital radiography. Intra-oral, panoramic, & cephalometric images can all be obtained digitally. Advantages: ♦ Superior gray-scale resolution. The human eye can only appreciate 32 shades of gray. The traditional radiograph differentiates 16-25 shades of gray, while the digital image uses up to 256 shades of gray. ♦ Less patient radiation and increased speed of image viewing. ♦ Decreased cost of equipment and film, image enhancement (this may also be a disadvantage in legal issues since the image can be manipulated), superior patient education tool. Although initial set-up costs for digital radiography can be significant, the elimination of film, darkroom chemicals, and equipment reduces maintenance costs. ♦ Each digital sensor costs between $7,000-$9,000 (initially expensive, but over the long-term, the office saves money due to savings in time, no film required, developing chemicals, etc.). Digital radiography is the absolute future in dentistry.

STORAGE PHOSPHOR IMAGING -a digital imaging system that uses a REVERSIBLE IMAGING PLATE rather than a sensor to record the image. Plates are more flexible, thus more comfortable for the patient.

Direct Digital Imaging System-uses an intraoral sensor attached to a fiberoptic cable. Indirect Digital Imaging System-scans an existing radiograph and digitizes the image. Charge-Coupled Device (CCD)-currently the most common digital image receptor. It is a solid state detector with a silicon chip embedded in it (it is the electronic circuit in the silicon chip that is sensitive to x-rays). CCD technology is also used in home video cameras, fax machines, and telescopes. Primary Radiation-the radiation generated at the ANODE of the x-ray tube that is attenuated by the filter and object. The amount of primary radiation follows the inverse square law measured from the focal spot. The attenuation of primary radiation is measured with a narrow beam geometry to include all secondary (i.e. scattered radiation). SECONDARY RADIATION (“Scattered Radiation”)-arises from interactions of the primary radiation beam with atoms in the object being imaged. Since scattered radiation deviates from the straight line path between the x-ray focus and image receptor, scattered radiation is a major source of image degradation in both x-ray and nuclear medicine imaging techniques. When x-ray radiation passes through a patient, 3 interactions can occur (coherent scatter, photo

electric absorption, and Compton scattering). Of these, most scattered x-rays in diagnostic x-ray imaging arise from Compton scattering. A LEADED, RETANGULAR cone (PID) best reduces the amount of scatter radiation to the patient as this greatly reduces the size of the beam. In performing normal dental diagnostic procedures, the operator receives the greatest hazard from secondary (scatter) radiation. COLLIMATION-the control of the SIZE & SHAPE of the x-ray beam using metal plates, slots, or bars to confine and direct radiation (i.e. x-rays or gamma rays) to a specific region and/or to discriminate against radiation from unwanted directions (i.e. scattered radiation). In x-ray imaging systems, a collimator mounted to the x-ray tube is used to define the dimensions of the beam which is to be incident on the subject and detector. To minimize radiation dose and to comply with government regulations, a certain level of precision must be maintained. • It is a basic rule of radiation hygiene that the radiation beam be as small as practical. For intraoral radiography, by state law, the diameter of a circular beam of radiation at the patient’s skin cannot be larger than 2.75 inches. One can use a diaphragm or metal cylinders, cones, or tubes to collimate the beam. • These devices reduce the amount of radiation received by the exposed tissues and the radiation to

surrounding tissues due to x-ray beam divergence. X-ray beam is composed of rays of different wavelengths and penetrating power (polychromatic) because the potential across the x-ray tube constantly changes as the kilovoltage changes: o Short wavelength (high energy) x-rays: have great penetrating power (are useful). Short wavelength xrays are produced at higher kilovoltages and penetrate objects more readily (these form the image on the film). o Long wavelength (low energy) x-rays: are produced at lower kilovoltages (kVp) thus have low penetrating power (are easily absorbed), and do not reach the film in reasonable quantities because they are attenuated by the soft tissues. Low energy rays only add to the total amount of radiation the patient receives. Aluminum discs are used to “filter” out these USELESS long wave rays to increase the overall quality of the x-ray beam. FILTRATION-the removal of parts of the x-ray spectrum using absorbing materials in the x-ray beam. The x-ray spectrum reaching the patient is filtered by attenuating material in its path. Filtering the beam is used to modify the spectral or spatial distribution of x-ray (or both). Filtration reduces patient dose, contrast, & film density.

1. Inherent filtration-the filtration of an x-ray beam by any parts of the x-ray tube or tube shield through which the beam must pass. The parts include the glass envelope of the x-ray tube and OIL surrounds the x-ray tube to cool the tube to dissipate heat, and the exit window in the tube housing. The inherent filtration corresponds to ~0.51mm of aluminum. 2. Added filtration-obtained by placing thin sheets of aluminum in the cone to filter the useful beam further. 3. Total filtration of the x-ray beam before it reaches the patient consists of the inherent filtration + added filtration. The recommended total is the equivalent of 0.5mm (below 50 kVp) and 2.5mm (over 70 kVp) of aluminum. When taking radiographs, the operator should STAND AT LEAST 6 FEET AWAY from the patient to reduce radiation exposure to the operator and should also stand behind a lead shield if possible. The operator should NEVER remain in the room holding the x-ray packet in place for the patient. If a film must be held in place by someone else (i.e. for a child), drape the parent and have him or her hold the film. All dental personnel should wear film badges that monitor exposure dosages. 1. The operator must avoid the primary beam by positioning themselves at a 90°-135° angle to the

beam. 2. Always maintain proper infection control when taking and processing dental radiographs! Ekta-Speed Film-MOST EFFECTIVE way to REDUCE exposure time, amount of radiation reaching the patient, and amount of scatter radiation (secondary radiation) to the dentist. FACTORS TO REDUCE PATIENT RADIATION: • Digital X-rays • Lead apron & thyroid (cervical) collar-the most effective at stopping x-rays, so patient should always wear both. Cervical collar protects the thyroid gland. • Increasing filtration using an aluminum disk. • Lead diaphragms placed within the cone of an x-ray tubehead. • Collimating an x-ray beam. • Ekta-Speed Film • Increasing the source-film distance. • Intensifying screens-used for all extra-oral radiography (panoramics, cephalometrics). Committee on Radiation Protection of National Bureau of Standards: recommends a person who works near radiation be exposed in 1 year to a maximum dose of 5 REM (0.1 REM per week). Secondary (scatter radiation)

is the greatest hazard to the dental team. • MPD (Maximum Permissible Dose of Radiation Exposure). The yearly MPD for a non-occupationally exposed person is 0.5 REM and 5 REM for people who work near radiation. Sequence of Radiation Injury: 1. Latent Period-period of time between radiation exposure and onset of symptoms. It may be short or long, depending on the total dose of radiation received and amount of time it took to receive the dose. 2. Period of Cell Injury-comes after the latent period. Cellular injury may cause cell death, changes in cell function, or abnormal mitosis of cells. 3. Recovery Period-the last event in the sequence of radiation injury. Some cells recover from the radiation injury, especially if the radiation is “low level”. Effects of radiation exposure are ADDITIVE, and the damage that remains non-repaired accumulates in tissues. The cumulative effects of repeated radiation exposure can lead to various serous health problems (i.e. carcinogenesis, which leads to various carcinomas, genetic mutations that cause birth defects, different leukemias, and cataracts).

Carcinogenesis & Genetic Mutations are important and serious effects of repeated exposure to low doses of xradiation. The mechanisms involved are due to frame-shift mutations, synergism with chemical carcinogens, and altered DNA repair enzyme functions. Cells in the body have different sensitivities to radiation. In general, the greater the rate of potential for mitosis and the more immature the cells and tissues, the more susceptible or sensitive these cells are to radiation. • Radiosensitive Cells: small lymphocytes (immature blood cells), bone marrow, reproductive cells (sperm/ova), and immature bone cells. Prostate gland is very sensitive to radiation. Hemopoietic tissue is the most sensitive to radiation • Radioresistant Cells: mature bone, muscle, and nerve (pulp). Muscle cells are the most radio-resistant. Radiation Absorbed Dose (“RAD”)-a measure of the energy imparted by any type of ionizing radiation to a mass of any type of matter. The traditional unit of absorbed dose is the “rad”. Equivalent Dose (“Dose Equivalent”)-the correct unit of measurement used by the dentist to compare the

biologic-risk effects/estimates of different types of radiation damage to a tissue or organ. Effective Dose-used to estimate the risk in humans. Exposure-a measure of radiation quantity, the capacity of the radiation to ionize air. Roentgen (R) is the traditional unit of radiation exposure measured in air. Roentgen only applies to x-rays and gamma rays. X-rays have more energy than light. ~1% of the energy released in the x-ray tube is released as x-rays. Electromagnetic Radiations: includes microwaves, xradiation, visible light, and gamma radiation. X-rays & gamma rays are types of non-particulate radiation energy. PANORAMIC RADIOGRAPH: an extra-oral radiograph where a bite piece is held between the patient’s front teeth. This bite piece should either be sterilized after each use, or covered with a disposable plastic slipcover. • Indications: diagnose oral pathology not visible on periapical radiographs, treating planning (especially orthodontic cases), evaluation of anomalies, as one part of the follow-up evaluation in surgical and trauma cases, edentulous patients prior to fabrication of full dentures, and in patients unable to tolerate intra-oral radiographs.

• Advantages: shows areas that may not be visible on a full-mouth series (and shows both arches on the same film). Greater patient comfort (eliminates gagging), and requires less time than a full-mouth series. Panoramic radiograph is the screening x-ray for pathology of the jaws. It is excellent for third molar pathology and to observe the TMJ, sinuses, and in sialography (a technique used in radiology that films the salivary gland after an opaque substance is injected into its duct). However, a sialolith that is located in Wharton’s duct is best viewed with a cross-sectional occlusal radiograph. • Disadvantages: provides less image detail & definition than periapical radiographs due to intensifying screens, movement of the x-ray tube and film, and increased object-film distance. ♦ loss of image detail (making it difficult to diagnose early carious lesions). Bite-wing x-rays are required to diagnose carious lesions. ♦ Image distortion due to increased object-film distance. ♦ Inadequate for interproximal caries detection or detecting periodontal breakdown (bone loss). ♦ Proximal overlapping (especially in posterior areas). ♦ Added exposure to a large area of body tissue in addition to the oral tissues. ♦ Poor definition of interproximal caries. However, by supplementing a panoramic with

posterior or anterior bitewings, a more complete radiographic survey of the patient is obtained. The panoramic is NOT a substitute for intra-oral films. Panoramic Positioning Errors: 1. Chin Tilted too Far Upward: a positioning error most likely to cause a REVERSE OCCLUSAL PLANE CURVE on a panoramic radiograph. A “reverse occlusal plane curve” is where mandibular structures look narrower and maxillary structures look wider (looks like a “frown”). 2. Chin Tilted too Far Downward: occlusal plane shows an excessive upward curve (looks like a “big smile”), there is severe interproximal overlapping, and the anterior teeth appear highly distorted. CONE BEAM VOLUMETRIC TOMOGRAPHY: diagnostic imaging technology using radiation in a similar manner to conventional imaging, but converts images into 3-D view that can be manipulated by software for IMPLANTS, ORTHODONTICS, ORTHOGNATHIC TMJ, and diagnostic purposes. • Hundreds of “slices” of information are gathered and assembled by software into 3-D views of any aspect (coronal, axial, sagittal planes). Data can be imported into software (ex: Romexis by Planmeca) for diagnostic

analysis and pre-surgical treatment planning (ex: implant placement) and stereolithographic biomodels.

CEPHALOMETRICS (“Lateral Head Radiograph)-a technique employing oriented radiographs for the purpose of making head measurements used to study craniofacial growth, diagnosis, planning orthodontic treatment, and evaluation of treated cases. Cephalometrics are useful to assess tooth-to-tooth, bone-to-bone, and tooth-to-bone relationships. Serial cephalometric films can show the amount and direction of growth. • Lateral head radiograph (cephalometric x-ray) must be

compared with “normal” lateral radiographs from an accepted norm. Linear and angular measurements are obtained using known anatomical landmarks in the lateral head radiography of the patient. These measurements are then compared with those considered WNL, thus enabling the orthodontist to assess aberrations in the dentition and jaw structures that cause malocclusion. • Cephalometric radiographic analysis includes hard tissue structures (bone & teeth), and measurements of soft tissue structures (nose, lips, soft tissue chin). • Superimposition of longitudinal cephalometric studies is generally on a reference plane and registration point to best demonstrate the growth of structures farthest from the plane and point. The most stable area from which to evaluate craniofacial growth is the anterior cranial base due to its early cessation (stopping) of growth. BITEWING RADIOGRAPHS-an intraoral radiograph MOST useful to detect INTERPROXIMAL CARIES and demonstrate alveolar bone resorption (periodontitis). Bitewings show the crowns of both maxillary & mandibular teeth, but NOT the root apices. The main reason to take bitewings is to detect interproximal caries. • Bitewings are most useful to monitor the

progression of periodontal disease as they show the crestal bone levels and interproximal areas of both arches. For the film to be of diagnostic use, the quality of the dimensional accuracy, open contacts, and optimum contrast and image clarity must be excellent. When taking bitewings, the film is placed in either a horizontal or vertical position. • Vertical bitewings-provide more periodontal information (i.e. bony defects and furcation involvement). A fuzzy or indistinct image of crestal bone is often associated with early periodontitis. The vertical bitewing angulation should be between +8° and +10°. Vertical bitewings show more alveolar bone than traditional horizontal bitewings. • Adjust horizontal angulation to direct the central ray toward the center of the film. • Two bitewings are usually taken on a child (one on each side). If the child only has the primary dentition, then #0 film is used. If the child has a mixed dentition, #1 film is used. Once the child has 2nd molars, two to four #2 films are used (if using 4 films, one film images the premolar area, while the other images the molar area). Sometimes two, long #3 films are used (one per side) rather then two #2 films on each side. This practice is not recommended due to the curvature of the arch, making it difficult to open all contacts on one film.

SUBMENTAL-VERTICAL (Submentovertex)-an x-ray designed to diagnose BASILAR SKULL FRACTURES, and provides some diagnostic information about the zygoma, zygomatic arches, and mandible. This film is taken with the source below the mandible and the film above the head. The submentovertex radiographic view is used when you suspect a FRACTURE OF THE ZYGOMATIC ARCH.

WATER’S VIEW-the standard radiograph of choice for showing an ANTERIOR VIEW of the PARANASAL SINUSES and of the mid-face and orbits. This is a posterior-anterior projection with patient’s face lying against the film and the x-ray source behind the patient’s head. Water’s is one of the best films for radiographic diagnosis of MID-FACIAL FRACTURES, sinus infections, and its view best demonstrates lesions of the maxillary sinus.

TOWNE’S VIEW-the best film to visualize the CONDYLES & NECK OF MANDIBLE from an AP projection. The patient lies on his back with the film under his head. The x-ray source is from the front, but is rotated 30° from the Frankfort plane and is directed right at the condyles. • Towne’s view is often of value to assess the status of the condyles, condylar neck, and rami because superimposition of the mastoid and zygoma over the condylar neck region in the straight postero-anterior projection often makes interpretation difficult. Towne’s view eliminates this superimposition to give good visualization of the condylar area and rami. • Reverse Towne’s View-used to identify fractures of the condylar neck and ramus area.

Conventional TMJ Radiographs: show the condyles position in the glenoid fossa, range of the condyles’ antero-

posterior movement, and areas of bone destruction on the condylar heads. Developer Solution-a chemical solution that converts the invisible image on a film into a visible image composed of minute masses of black metallic silver. Its function is to reduce silver halide crystals to black metallic silver. X-ray Developing Solution Contains 4 Chemicals: 1. Developing agent (hydroquinone)-a chemical compound capable of changing the exposed silver halide crystals to black metallic silver, while producing no appreciable effect on the unexposed silver halide crystals in the immulsion. It gives details to the x-ray image. Elon, also quickly generates gray tones on the xray image. 2. Antioxidant preservative (sodium sulfite)-prevents developer solution from oxidizing in the presence of air. 3. Accelerator (sodium carbonate)-an alkali that activates the developing agents and maintains the alkalinity of the developer at the correct value. It softens gelatin of emulsion. 4. Restrainer (potassium bromide)-is added to developers to control the action of the developing agent so it does not develop the unexposed silver halide

crystals to produce fog. When taking panoramic radiographs, if the films keep getting lighter & lighter after each development, to correct this problem simply replenish the developing solution. As the developing solution gets weaker, the films get lighter. Both the developing and fixing solutions should be replenished daily. These solutions need to be changed regularly and the tanks scrubbed and cleaned. • Factors that affect developing solution life: cleanliness of tanks, size & number of films processed, and solution temperature. • Yellowish-brown film is caused by insufficient fixing or rinsing. • Fogged film may result from improper film storage or outdated films. • Low solution levels will appear as developer cut-off (straight clear border) or fixer cut-off (straight black border). X-RAY FIXING SOLUTION (“Fixer”) Solution:-a chemical solution whose function of is to STOP development and remove remaining unexposed crystals. Fixing time is always at least twice as long as the developing time. Fixer contains 4 chemicals:

1. Clearing agent (sodium or ammonium thiosulfate)commonly called “hypo” dissolves and removes underdeveloped silver halide crystals from the emulsion (one of the main functions of fixing solutions). The chemical clears the film so the black silver image produced by the developer is distinctly perceptible. When the film is improperly cleaned, the remaining unexposed silver halide crystals darken upon exposure to light and obscure the image. 2. Antioxidant preservative (sodium sulfite)-prevents the decomposition of the fixer chemical. 3. Acidifier (acetic acid)-necessary for the correct action of the other chemicals and also neutralizes any alkaline developer that may be carried over by the film or hanger. 4. Hardener (potassium alum)-shrinks and hardens the gelatin in the emulsion. It shortens drying time and protects the emulsion from abrasion. If a dried radiograph were processed a second time, there would be no change in contrast or density. After processing a film, if it appears the color BROWN, the most likely cause is the fixing time was not long enough. A film appears brown when it is not completely fixed.

RADIOGRAPHIC TECHNIQUES & ERRORS VERTICAL ANGULATION-directing x-rays so they pass vertically through the part being examined. This is accomplished by positioning the tubehead and direction of the central ray in an up-and-down (vertical) plane. Foreshortening & Elongation are produced by INCORRECT VERTICAL ANGULATION. • Foreshortening-a shortened image is caused by excessive vertical angulation. The teeth APPEAR TO SHORT due to either too much vertical angulation, or poor chair position. • Elongation-an elongated image is caused by insufficient (too little) vertical angulation. Elongation is the MOST COMMON error when taking dental radiographs where the teeth APPEAR TOO LONG due to either too little vertical angulation, or the film not being parallel to the long axis of the teeth or the occlusal plane not being parallel to the floor.

HORIZONTAL ANGULATION-maintaining the central ray at 0° as the tube is moved around the head. This is accomplished by positioning the tubehead and direction of the central ray in a side-to-side (horizontal) plane. The general rule for horizontal angulation is the central ray should be perpendicular to the mean antero-posterior plane of the teeth being x-rayed. • Overlapping-interproximal areas are overlapped due to incorrect horizontal tube angulation (central x-ray was not directed perpendicular to the curvature of the arch and through the contacts). Overlapping reduces the diagnostic quality of film to detect interproximal caries since teeth images are superimposed on each other).

The central ray is at 0° when the x-ray tube is adjusted so the central ray is parallel to the floor. If the tubehead is directed at the floor, it is positive angulation. If the x-ray tubehead is directed toward the ceiling, it is negative angulation. BISECTING ANGLE TECHNIQUE-a technique based on the geometric rule of isometry which states two triangles are equal if they have two equal angles and share a common side. The image on the film is equal to the length of the tooth when the central ray is directed at 90° to the imaginary bisector. A tooth and the radiographic image are equal in length when two equal triangles are formed that share a common side (imaginary bisector). The bisecting technique works as follows to produce a tooth image that is accurate if done correctly. Advantage: decreased exposure time. ♦ X-ray film is placed along the lingual/palatal surface of the tooth. At the point where the film contacts the tooth, an angle is formed by the plane of the film, and the long axis of the tooth. ♦ The person taking the x-ray must visualize a plane that bisects this angle (this plane is the imaginary bisector which creates two equal angles and provides a common side for the two imaginary equal triangles. The central ray is positioned perpendicular to the imaginary bisector.

♦ Disadvantages: x-ray film image may be dimensionally distorted (amount may vary); due to the use of a short cone (which causes divergent rays) the image is not a true reproduction of the object; may not be able to judge the correct alveolar bone height. PARALLELING TECHNIQUE: based on the concept of parallelism since the film is placed parallel to the long axis of the tooth being x-rayed, and the central x-ray beam is directed perpendicular or at right angles to the long axis of the teeth & plane of the film. A film holder (XCP) must be used to keep the film parallel to the long axis of the tooth. • Advantages: little or no root superimposition on a maxillary molar view, accurate diagnosis of periodontal bone height (given minimal distortion), and the image formed is dimensionally accurate. • Disadvantages: film placement may be difficult in some areas (i.e. low palatal vaults), increased exposure time is required due to the use of a LONG CONE, and XCP holders/rings can be cumbersome to work with and may cause patient discomfort. The object-film distance must be increased to keep the film parallel, which results in image magnification and loss of definition. Also, the source-film distance must be increased to compensate for the image magnification, and to ensure that only the most parallel x-rays will be aimed at the tooth and film. Using a long cone to increase the target-

film distance (16 inch target-film distance) is required to provide greater definition and less image magnification. • AKA: XCP (Extension Cone Paralleling Technique), Right-Angle Technique, Long-Cone Technique. BUCCAL OBJECT RULE (“Tube Shift Technique” or “SLOB” rule)-used to determine an object’s special position within the jaws. This technique uses two radiographs of an object exposed with slightly different tube angulations, then compares the object’s position on the radiograph with respect to a reference point (i.e. the tooth root). • SLOB (Same Lingual, Opposite Buccal): if the object in question appears to move in the SAME direction as the x-ray tube, then it is on the LINGUAL aspect. If it appears to move in the OPPOSITE direction as the x-ray tube, then it is on the BUCCAL aspect. • If the x-ray tube is shifted and directed from a mesial direction/angulation and the object in question moves mesially from the reference point, then the object lies lingual to that reference point. The x-ray tube was directed from a mesial angulation, the special position of the object lies LINGUAL. However, if the x-ray tube is shifted mesially and the object moves distally, it lies on the buccal aspect of the reference point. CERVICAL BURNOUT-a phenomenon caused

by

relatively low x-ray absorption on the mesial or distal surfaces of teeth, between the edges of the enamel and adjacent crest of the alveolar ridge. Because of this diminished x-ray absorption, these areas appear relatively radiolucent will ill-defined margins. It is caused by the normal configuration of the affected teeth (CEJ) which results in decreased x-ray absorption in those areas. These radiolucencies should be anticipated when viewing xrays of almost any tooth, and must not be mistaken for a carious lesion.

Important: for a given beam of radiation, the intensity is INVERSELY proportional to the square of the distance from the radiation source. The intensity of an x-ray beam at a given point is dependent on the distance of the measuring device from the FOCAL SPOT. The reason for this decrease in intensity (the reason it is inversely proportional) is because the x-ray beam spreads out as it moves away from the source. The “spread out” beam is less intense. Focal Spot-the small area of tungsten on the anode (target) from which x-rays emanates and that receives the impact of the speeding electrons. Focal spot is 1 of 3 factors that influences image sharpness. The size of the x-ray tube focal spot influences radiographic DEFINITION. • Target (tungsten target)-a tungsten wafer embedded in the anode face at the point of electron bombardment. • Target Film Distance (source-to-film distance)distance from the x-ray source (the focal spot on the tungsten target) to the film. It is determined by the length of the cone (position-indicating device = PID). Two standard target-film distances are used in intra-oral radiography. 1. 20cm (8 inches): is the short cone that exposes more tissue by producing a more divergent beam. 2. 41cm (16 inches): is the long cone that reduces the amount of exposed tissue by producing a less

divergent beam and a sharper image. Half-Value Layer (HVL)-determines the QUALITY OF THE PENETRATING X-RAY BEAM. HVL the amount of aluminum or copper thickness required to reduce the intensity of an x-ray beam by 50%. To reduce x-ray beam intensity, aluminum filters are placed in the beam’s path inside the x-ray tubehead. When placed in the x-ray beam’s path, the aluminum thickness that reduces the beams intensity by 50% is called the “half-value layer”. • Due to the spectral nature of x-rays, HVL is not constant. When measuring multiple half-value layers, the 2nd HVL is greater than the 1st HVL since the mean energy of the x-ray spectrum is increased following the passage of the 1st HVL that results in x-rays becoming more penetrating. • In oral diagnostic radiography, the HVL of the radiation beam is ~2mm of aluminum (this means 50% of the xrays exiting the vacuum tube are absorbed by 2mm of aluminum. Doubling the thickness of aluminum will not absorb all of the x-rays, but one HALF of the remaining x-rays. Intensifying Screens-devices used in extra-oral radiography that convert x-ray energy into visible light. The visible light then exposes the screen film. Thus, the radiation a patient receives is decreased. Used for all

extra-oral radiography (panoramics, cephalometrics) films to decrease the amount of radiation exposure to patients. Cassette Holder-a light-tight device used in extra-oral radiography to hold film and intensifying screens. Radiograph Operator Controls 3 Factors: 1. KILOVOLTAGE (kVp)-the quality or penetrating power of the x-ray beam that controls the speed of ELECTRONS. The speed that electrons travel from the filament of the cathode to the anode’s target depends on the potential difference between the two electrodes (kilovoltage). Thus, this has a very important effect on the x-rays produced at the focal spot. Kilovoltage has nothing to do with the number of electrons that compose the stream flowing from cathode→anode. The number of electrons (which determines the quantity of x-rays produced), is controlled by the temperature of the tungsten filament (milliamperage setting). The hotter the filament, the more electrodes are emitted and available to form the electron stream (x-ray tube current). In the x-ray tube, the number of electrons flowing/sec is measured in millamperes. The intensity of x-rays produced at a particular kilovoltage depends on that number. Suitable ranges of dental x-rays are 65-100 kVp Important: kilovoltage influences the x-ray beam

and radiograph by altering contrast quality (for patients with thick jaws, increase kilovoltage), determining the quality of the x-rays produced, and determining the velocity of the electrons to the anode. kVp most directly affects radiographic CONTRAST and predominantly determines the penetrating ability of the x-ray beam. • Setting the x-ray machine for a specific milliamperage actually means adjusting the filament temperature to yield the current flow indicated. • To increase film density: ↑ mA, kVp, & time, and ↓ source-object distance. • One effect of a change in kVp is a change in the xray’s penetrating power. Increasing kilovoltage (kVp): reduces subject contrast (and the longer scale of contrast) and produces new, more penetrating xrays, while emitting less penetrating x-rays that were also produced at the lower kilovoltage. Increasing kVp causes the resultant x-ray to have A LONGER SCALE OF CONTRAST. Decreasing kilovoltage: increases subject contrast (and the shorter the scale of contrast). 2. MILLIAMPERAGE (mA)-controls the QUANTITY of x-

rays produced (it’s the quantity or number of x-rays produced based on the number of electrons) and the temperature of the tungsten filament (a hotter filament produces more x-rays). Suitable ranges for dental x-rays are 7-15mA. • An increase in mA produces a DENSER & DARKER image and INCREASES the quantity of x-ray photons. • Adjusting mA on the dental x-ray unit affects the QUANTITY of x-ray photons. 3. EXPOSURE TIME-the length of time x-rays are produced and the time the patient is exposed to them. Using high speed E or F film or digital x-rays reduces exposure time. DENSITY-an image’s overall darkness or lightness is affected by kVp. ↑kVp = ↑density and darker image (helps identify alveolar bone changes) while ↓kVP = ↓density and lighter image (helps reveal caries). • To increase film density: ↑ mA, kVp, & time, and ↓ source-object distance. CONTRAST-how sharply dark and light areas are distinguished on an image. The difference in the degree of blackness (density) between adjacent areas on a

radiograph. Image contrast and quality are affected by mainly kVp and filtration. • Low kVp setting (65-70) produces a HIGH CONTRAST image of many black and white areas with minimal shades of gray (not 50 shades of gray ☺). Short-scale of contrast. • High kVp setting (> 90) produces a LOW CONTRAST image with many shades of gray and minimal black and white areas. High kVp is preferred in dentistry. Longscale of contrast.

5 Rules to Create Accurate Images when taking x-rays: 1. Use the smallest practical focal spot. The size of the focal spot influences radiographic definition or SHARPNESS (they are inversely proportional; the SMALLER FOCAL SPOT = SHARPER IMAGE). The operator cannot control the size of the focal spot. 2. Use the longest source-film distance practical in the situation. 3. Place the film as close as possible to the structure being radiographed. 4. Direct the central ray at as close to a right angle to the film as anatomical structures allow. 5. Keep the film parallel to the structure being

radiographed. Image Magnification is minimized by USING a LONG CONE. DENTAL X-RAY TUBE: the protective high vacuum leaded glass housing surrounded by refined oil (to help absorb excess heat) with high insulating powers. X-Ray Tube Components: 1. Filament-a coiled tungsten wire inside the cathode (+) that when heated to incandescence, emits/produces the stream of electrons. 2. Molybdenum cup-houses the tungsten filament. 3. Electron stream-travels from the filament in the cathode to the tungsten target. 4. Tungsten target-located in the anode (-) to stop the stream of electrons. 5. Focal Spot-the portion of the tungsten target (made of tungsten) struck by the electron beam. It contains a small area “focal spot” that the electrons strike to produce x-rays. The focal spot’s size directly influences x-ray definition (the larger the focal spot, the greater loss of image definition and sharpness). 6. Copper sleeve-located in the cathode. 7. Vacuum 8. X-ray beam-produced when the electron stream bounces off focal spot on the tungsten target 9. Leaded glass housing-houses the entire x-ray tube.

X-rays are generated when a stream of electrons (produced by the filament) travels from the CATHODE→ANODE and is suddenly stopped by its impact on the tungsten target. The filament is located in the cathode and is made of tungsten wire. The small area on the target that the electrons strike is the focal spot (the xray source).

Prescribing Dental Radiographs: • Dental radiographs are taken ONLY when needed as

judged by each patient’s needs. Decisions about the number, type, and frequency of dental x-rays are DETERMINED ONLY BY THE DENTIST. Every patient has a different dental condition, thus the frequency of xrays differs as well. There are guidelines published by the ADA that assist a dentist in prescribing the number, type, and frequency of dental radiographs. • Patients with tooth decay, periodontal disease, tooth mobility, pain in one or more teeth, or possible impacted teeth need more frequent radiographic examinations than patients without such problems. A pediatric patient who is caries free and asymptomatic, the child’s first bitewing radiographs should not be taken until the spaces between the posterior teeth have closed. • Occult Diseases-includes small carious lesions, cysts, & tumors that present no clinical signs or symptoms. Because occult disease in the perioral tissues is rare (except for caries), a radiographic examination of the jaws should NOT be done solely to look for it in an individual with teeth when there are no clinical signs or symptoms. However, every xray taken must be evaluated for these lesions. Remember: caries is an exception to the above rule due to its much higher prevalence compared to occult cysts or tumors.

DENTAL EMERGENCY PROTOCOL ANAPHYLACTIC REACTION (allergic reaction develops in seconds or minutes) after local anesthetic, nitrous, or dental material exposure. LIFE-THREATENING causing BRONCHOSPAM & DROP IN B.P. (EPI pen bronchodilates and raises patient’s BP): 1. Call 911 and position the conscious patient in a comfortable position. 2. Get the preloaded EPI syringe in the emergency kit and inject EPI pen into patient’s DELTOID, TONGUE, or LATERAL THIGH. 3. Re-administer EPI pen in 5 minutes ONLY IF SYMPTOMS PERSIST. CHEST PAIN (ANGINA PECTORIS)—patient has tight, heavy, or constricted chest pain and may clench their fist against their chest. 1. Call 911 and position patient so he/she is comfortable and ask if they have their nitroglycerin tablets or spray on them. If not, give 2 sprays of NITROGLYCERIN (vasodilator) onto the patient’s

tongue. 2. Dental treatment can continue if patient and doctor are comfortable. Important: do not give spray/tablets if the patient has chest pain and feels faint or dizzy (means BP is dropping), or if the patient took VIAGRA within 24hrs. HEART ATTACK: after chest pain, patient says their pain is getting worse, patient has taken 3 doses of nitroglycerin at five minute intervals and pain continues, or the chest pain went away but comes back, or the patient has had no history of heart disease or chest pain. CRUSHING, INTENSE, RADIATING PAIN from the chest to the stomach or to the left side of the neck, jaw, left arm, and/or pinkie finger (tingles). SKIN TURNS ASHEN GRAY & PATIENT MAY SWEAT PROFUSELY. 1. Call 911 and position the patient so they are comfortable. 2. Administer 50% nitrous oxide & 50% oxygen (has same effect as IV Morphine) for pain and delivers more oxygen to the muscles/brain. 3. Give two sprays of nitroglycerin on patient’s tongue and have the patient CHEW 1 tablet of adult-dose ASPRIN (325mg) EXCEPT if patient is allergic to aspirin, has a bleeding disorder, or

gastric/peptic ulcer. ASPIRIN prevents clot from getting bigger. CARDIAC ARREST (UNCONSCIOUS PATIENT): 1. Call 911 while dentist lays the patient flat in dental chair with their feet ELEVATED. 2. (CAB): Dentist starts chest compressions, check airway and open with the HEAD LIFT/CHIN TILT and then check for breathing and carotid pulse (in the groove under and to the side of the Adam’s Apple). Follow CAB (compressions, airway, and breathing)-new AHA guideline. 3. Dentist gives CPR (30 chest compressions for every 2 breaths). 30:2 ratio. DIABETIC SHOCK (HYPOGLYCEMIA = LOW BLOOD SUGAR). MENTAL CONFUSION, patient feels cold, sweaty, and shaky. • Before treatment ask patient when did they take their insulin and eat last. If not recent, give patient third cup of ORANGE JUICE or NON-DIET SODA wait 5 minutes, then give another third cup. After 5 minutes, give last third cup. WITHIN 15 minutes signs should subside. • Unconscious Diabetic: 1. CALL 911 and lay patient flat with feet

raised, check airway (head tilt/chin lift), check breathing (look, listen, feel) and check carotid pulse. DO CPR if not breathing/no pulse (15 compressions for every 2 breaths). 15:2. 2. DO NOT ADMINSTER DRUGS! SYNCOPE (FAINTING): can occur due to EMOTIONAL STRESS (nervousness). Occurs when there is a temporary decrease in blood flow to the brain due to sudden drop in BP, HR or blood volume change. Can happen to anyone, but the patient usually has an underlying medical condition like anemia or heart disease. Signs a patient may faint: light-headed, nausea, heart palpitations. 1. Lay patient in dental chair with legs ELEVATED. 2. Place a cold, wet towel over their forehead 3. Administer oxygen and AROMATIC AMMONIA held under patient’s nose to stimulate blood flow to brain via movement. SIEZURE (EPILEPSY): caused by signals in the brain are disrupted. GRAND-MAL is the most-common type (last 2-3 minutes; the body becomes rigid and relaxes). A seizure lasting more than 5 minutes is called Status Epilepticus and is life-threatening). Signs: patient may have a visual, sound, or smell aura immediately before seizure starts (so ask the patient with seizure history if they have a

common aura or if they have taken their anti-epileptic medication. 1. Call 911. 2. Remove any sharp objects away from the patient and have one person gently hold the arms and the other the legs to protect patient from injury. Note: if a minor has a seizure, call the parent into the room to help.

DENTAL CONSIDERATIONS FOR MEDICALLY COMPROMISED PATIENTS CHRONIC OBSTRUCTIVE PULMONARY DISEASE (Asthma, Bronchitis, Emphysema): • Sit patient upright in dental chair. • No rubber dam in severe cases. • No N2O if severe emphysema. • Avoid barbiturates, narcotics, anti-histamines. • Avoid erythromycin, clarithromycin if patient takes theophylline. ASTHMA: • Have patient bring their bronchodilator inhaler to each appointment. • Avoid Aspirin, NSAIDs, narcotics/barbiturates. • Avoid Erythromycin if patient takes theophylline. • Avoid sulfite-containing local anesthetics. • Can use N2O or Diazepam for anxious patient. • Use pulse oximeter if necessary. TUBERCULOSIS: • Active TB: consult with physician before treatment

and only treat emergencies. • Prior TB (Non-Active): Use caution, get good medical history, periodic chest-x-rays to rule-out reactivation. If patient is TB-free confirmed by physician, treat as a normal patient. May be taking ISONIAZID (INH) for 6 months to 1 year prophylactically. VIRAL HEPATITIS (B,C,D,E): • Active Hepatitis: Consult with physician and treat on emergency basis only. • Non-Active Hepatitis: consult with physician, treat as normal. DIABETES MELLITUS: • In uncontrolled diabetics, they are prone to infection and poor-wound healing. • Insulin patients: eat normal meal before appointment (schedule morning or mid-morning). • Have sugar (OJ) available in case of diabetic shock. • With well-controlled diabetics, no alteration of treatment plan is required unless. Treat as usual. HYPERTHYROIDISM (Thyrotoxicosis): overactive thyroid produces too much T3 and/or T4 most commonly caused by Grave’s disease. • Avoid EPI and other vasoconstrictors in untreated patients.

HEMODIALYSIS: renal replacement therapy to remove creatine, urea, and free water from the blood during kidney failure. • Delay treatment until off dialysis machine for at least 4 hours (because of heparin). Best to schedule dental appointment the day after hemodialysis. • Pre-treatment screen for bleeding disorders. • Avoid drugs metabolized by the kidneys. • Consider antibiotic prophylaxis to minimize effects of bacteremia. • Avoid BP cuff on arm containing the shunt. • No routine dental care contraindications. END-STAGE RENAL DISEASE: • Check with physician for any hematologic disorders. • Monitor BP during before and during treatment. • Avoid drugs excreted by the kidney or nephrotoxic drugs. • Extensive reconstructive crown and bridge not recommended. PREGNANCY & LACTATION: • Do not placed patient in supine position in late pregnancy for long periods. • Avoid ASPRIN & NSAIDs. • Pregnancy gingivitis common • Consider use of fluoride or chlorhexidine gluconate. • Major procedures (crown and bridge) are best until after delivery (unless emergency).

• Avoid elective care in first trimester. 2nd and 3rd trimesters are best for elective treatment. SEIZURE (EPILEPTIC) PATIENTS: • Get bleeding time for patients taking Valproic Acid. • No aspirin and NSAIDs. • No propoxyphene and erythromycin for patients taking Carbamazepine. • Seizure is managed using a ligated mouth prop (bite block) at start of appointment. • Use metal instead of porcelain when possible. • PROTECT THE PATIENT FROM HARMING THEMSELVES AND THE DENTIST. RADIATION PATIENTS: • Patients may develop mucositis, xerostomia, taste loss, trismus, candidiasis and other secondary infections, cervical caries, osteoradionecrosis. • Before Radiation: extract teeth that cannot be repaired, restore large carious lesions, establish good oral hygiene, start daily fluoride treatment, treat RCT or extract non-vital teeth. • After Radiation: do not extract teeth and treat diseased teeth with RCT if indicated. Most other dental procedures can be performed. • Have patient come for frequent recall appointments (every 3-4 months) and continue daily fluoride treatment; chlorhexidine rinses for plaque and candidiasis control.

CHEMOTHERAPY PATIENTS: • Eliminate all oral infections and treat advanced caries. • Use topical fluoride for caries control and chlorhexidine for plaque/candidiasis. • Rx: Rinses for mucositis and xerostomia. HYPERTENSION: (normal 100). • Send for medical evaluation if BP is > 180/110 before any dental treatment. • Avoid orthostatic hypotension (change chair position slowly; don’t raise chair fast). • Use minimal. EPI (aspirate before injecting) no cord with EPI. • Caution using EPI with patients taking non-selective beta blockers. • Reduce dosage of barbiturates/sedatives whose action can be enhanced by anti-hypertensive agents. STROKE HISTORY (CVA): poor blood flow to the brain causes cell death. High BP is main risk for CVA. • No elective care for current transient ischemic attacks (delay treatment for 6-months). • Consider panorex periodically to assess carotid artery patency. • Short, morning appointments. • No EPI in retraction cord.

PACEMAKERS & DEFIBRILLATORS: • No antibiotic prophylaxis needed. • Avoid ultrasonic scalers or electrosurgery. THROMBOCYTOPENIA (prolonged bleeding due to decrease thrombocytes/platelets in blood): • Do not use aspirin-containing drugs or NSAIDs. • Use Acetaminophen/Tylenol and codeine if needed. HEMOPHILIA (Congenital Coagulation Disorders)excessive bleeding. • No dental procedures until cleared with physician. • Avoid aspirin and NSAIDs. Use acetaminophen (Tylenol) with or without codeine. VON WILLEBRAND’S DISEASE: coagulation abnormality due to protein deficiency required for platelet adhesion. • Avoid aspirin and NSAIDs. • Use Acetaminophen (Tylenol) with or without codeine or COX-2 inhibitors (Celecoxib, Rofecoxib). • Screen prolonged bleeding time, PFA-100, or prolonged partial thromboplastin time. ANTI-COAGULATION: (patients taking Heparin, Warfarin, Coumadin for clotting problems): • No dental procedures until medical consult done with patient’s physician (must achieve anti-coagulation levels). Check prothrombin time (PT). • Can do most surgical procedures if PT ratio is 2.5 or

less or INR is 3.5 or less. • May have to delay procedure 2-3 days if anticoagulation dose has to be reduced.

ALWAYS PRACTICE UNIVERSAL PRECAUTIONS!

DENTAL PHARMACOLOGY ANTIBIOTIC PROPHYLAXIS GUIDELINES FOR DENTAL PROCEDURES TO PREVENT INFECTIVE ENDOCARDITIS 2007 AMERICAN HEART ASSOCIATION GUIDELINES

Bacterial Endocarditis Prophylaxis (Antibiotic Prophylaxis) is Recommended: 1. Prosthetic/Artificial Cardiac Valves (i.e. biosynthetic (mechanical) & homograft (pig) valves). 2. Previous Bacterial Endocarditis (infection of the heart lining or heart valves). 3. Congenital Heart Disease of: (unrepaired cyanotic congenital heart disease, including those with palliative shunts and conduits); Completely repaired congenital heart disease with prosthetic material or device placed surgically or via catheter during the first 6 months after the procedure; repaired congenital heart disease with residual defects at the site or adjacent to the site of a prosthetic patch or device (which inhibits endothelialization); cardiac transplantation recipients with cardiac valvular disease. Complex Cyanotic Congenital Heart Disease (i.e. single ventricle states, transposition of great arteries, Tetrology of Fallot). 4. Surgically constructed synthetic PULMONARY shunts or conduits. 5. Hypertrophic Cardiomyopathy. 3. DENTAL PROCEDURES: extractions, periodontal

procedures (surgery, SRP, probing, and recall perio maintenance), implant placement, endodontic (RCT) instrumentation of surgery ONLY past the apex, subgingival placement of antibiotic fibers/strips, initial placement of orthodontic bands (not brackets), intraligamentary local anesthetic injections, prophylactic cleaning of teeth or implants where bleeding is anticipated. ALL DENTAL PROCEDURES THAT INVOLVE MANIPULATION OF GINGIVAL TISSUE OR PERIAPICAL REGION OF TEETH OR PERFORATION OF ORAL MUCOSA NEED ENDOCARDITIS PROPHYLAXIS. Bacterial Endocarditis Prophylactic Antibiotic is NOT required: • Isolated atrial or ventricular septal defects, or patent ductus arteriosus. • Coronary Artery Bypass Graft Surgery (CABG). • Mitral Valve Prolapse (with or without regurgitation). • Rheumatic Heart Disease, Bicuspid Valve Disease, Calcified Aortic Stenosis. • Congenital Heart Conditions (Hypertrophic Cardiomyopathy, Ventricular Septal Defect, or Atrial Septal Defect). • Heart murmurs, Kawasaki Disease without valvular dysfunction. • Cardiac Pacemakers (intravascular & epicardial) and implanted defibrillators. • Restorative Dentistry: local anesthetic injections

(non-intraligamentary), post & core placement, placing rubber dams, RCT not past the apex, postoperative suture removal, placement/adjustment of RPD and orthodontic appliances, impressions, fluoride treatments, radiographs, or shedding of primary teeth, or bleeding from trauma to the lips or oral mucosa.. NO antibiotic prophylaxis is required for patients who had a NON-VALVULAR devices placed unless it has been 2-3 weeks after surgery and healing is still occurring. Non-Valvular Devices: PACEMAKERS, implantable cardioverter defibrillators, left ventricular assist devices (LVAD), total artificial hearts, ventriculoatrial SHUNTS, peripheral vascular STENTS, hemodialysis prosthetic vascular grafts, intra-aortic balloon counterpulsation CATHETERS, coronary angiography and percutaneous coronary artery intervention, CORONARY ARTERY STENTS, vascular closure devices, and vena cava filters. ANTIBIOTIC PROPHYLAXIS & TOTAL JOINT REPLACEMENTS: • No scientific evidence currently supports antibiotic prophylaxis to prevent HEMATOGENOUS INFECTIONS before dental treatment in patients with total joint prosthesis. 2014 panel of experts convened by the ADA Council

of Scientific Affairs developed an evidence-based clinical practice guideline (CPG) to clarify the 2012 panel of the American Academy of Orthopaedic Surgeons (AAOS) and American Dental Association (ADA). The final recommended is in general, for patients with prosthetic joint implants, prophylactic antibiotics are NOT recommended before dental procedures to prevent prosthetic joint infection. The reasoning behind the 2014 recommendation is evidence shows dental procedures are not associated with prosthetic joint infections, antibiotics provided before oral care do not prevent joint implant infections, the potential harms of antibiotics like anaphylaxis, antibiotic resistance, opportunistic infections like Clostridium difficile, and the benefits or antibiotic prophylaxis may not exceed the risks for most patients. • Antibiotic prophylaxis is NOT indicated for dental patients with PINS, PLATES, & SCREWS, nor is it routinely indicated for most dental patients with total joint replacements. PREMEDICATION IS NO LONGER REQUIRED for patients after artificial joint replacement surgery. However, the dentist can consult with the patient’s physician if the patient has a compromised immune system (diabetes, RA, cancer, chemotherapy, and chronic steroid use) which may increase the risk of orthopedic implant infection.

• If unanticipated bleeding occurs, administer an antimicrobial prophylaxis within 2 hours after the procedure! • If a series of dental procedures is required: observe an interval of 9-14 days between procedures to reduce the potential for the emergence of resistant organisms, and allow the mouth to repopulate with antibiotic susceptible flora. • If a patient is taking an antibiotic normally used for endocarditis prophylaxis then SELECT a drug from a different class rather than increase the dose of the patient’s current regimen, or delay the procedure for 9-14 days after the patient completes the antibiotic. Oral Contraceptive Precautions: the risk of cardiovascular side effects increases in women who smoke cigarettes, especially women over age 35. The risk of a thromboembolism also increases. Women with hypertension are encouraged to use a non-hormonal form of contraception. The highest risk associated with use of oral contraceptives is thromboembolic disorders. Antibiotics have the potential to reduce the effectiveness of oral contraceptives. Thus, advise patients to use additional methods of birth control when taking antibiotics and oral contraceptives concurrently.

CENTRAL NERVOUS SYSTEM Central Nervous System (CNS) Neurotransmitters: Catecholamines pass the blood-brain barrier very poorly. 1. Acetylcholine-a neurotransmitter substance whose CNS effects are generated by the interaction with a mixture of nicotinic and muscarinic receptors. Secreted by cholinergic fibers. After a threshold stimulus, the liberation of acetylcholine alters the cell membrane’s permeability. 2. Epinephrine-a direct-acting catecholamine & adrenergic agonist. CNS neurons containing EPI have been identified, but their physiological properties are unknown. 3. Norepinephrine-a direct-acting catecholamine that works through α1, α2, & β adrenergic receptors in the CNS. 4. Isoproterenol-a direct-acting catecholamine. 5. Dopamine (Intropin)-a catecholamine (immediate precursor of NE) that acts through at least two subtype receptors, D1 (activates adenylyl cyclase)

and D2 (inhibits adenylyl cyclase). 6. Dobutamine 7. Serotonin (5-hydroxytryptamine)-works through at least 14 subreceptors “tryptominergic” type neurons. 8. GABA (gamma-aminobutyric acid)-the major INHIBITORY neurotransmitter in the CNS. 9. Opioid peptides: beta-endorphin, enkephalins, & dynorphin. 10. Glutamate & Aspartate-CNS amino acids that elicit POWERFUL EXCITATORY effects on neurons within every region of the CNS. Catecholamines-any one of a group of sympathomimetic compounds composed of a catechol molecule and the alipathic portion of an amine. Some catecholamines are produced naturally by the body (endogenous), and function as key neurological chemicals (i.e. EPI, NE, DA).

CNS STIMULANTS-a heterogenous group of compounds that produce various degrees of stimulation. Historically, CNS stimulants were widely used therapeutically, but today they have limited clinical use. Their use to treat respiratory depression caused by an overdose of CNS depressants is not safe or recommended. • Analeptic-a CNS stimulant with the ability to overcome drug-induced respiratory depression and hypnosis. Analeptics and respiratory stimulants include: Pentylenetetrazol (Metrazol), Nikethamide (Coramine), Doxapram (Dopram), Picrotoxin, & Strychnine. Analeptics have limited use, but a few are

occasionally indicated to stimulate respiration when a patient has pulmonary disease or to hasten recovery from a general anesthetic. • Xanthines-stimulants like caffeine, theophylline, and theobromine used to improve mental alertness, reduce the urge to sleep, and elevate mood. Caffeine is the only approved OTC stimulant. Theophylline & Theobromine are weaker CNS stimulants than caffeine. Theophylline is the only xanthine important in asthma treatment, by stimulating medulla respiratory centers to cause bronchial dilation. • Sympathomimetic Amines-potent CNS stimulants like amphetamines and related agents (i.e. Methylphenidate & Phenmetrazine). Sympathomimetic amines are used to treat narcolepsy, obesity, and ADHD. Caffeinism-a term for people dependent upon caffeine (CNS stimulant) (i.e. suffer side effects from having too much caffeine, take larger amounts, and need to keep drinking caffeine to function properly). It is thought to occur if you intake > 600-750mg of caffeine per day (more than 10 cups of coffee). Drinking > 1000mg per day is well into the toxic range. Caffeine stimulates the CNS unequally, with the CORTEX being the most excited, and the spinal cord being the least excited. Symptoms of Chronic

Caffeine Consumption: feelings of anxiety/nervousness, sleep disruption, irritability, diuresis, stomach complaints, palpations, and arrhythmias.

ADRENERGICS (SYMPATHETICS) Autonomic Nervous System (ANS): an efferent motor system that functions independent of consciousness (unconscious); controls automatic visceral functions required for life. Drugs block or mimic ANS transmitters to clinically modify autonomic function (cardiac & smooth muscle, vascular endothelium, exocrine glands and presynaptic nerve terminals). Other therapeutic drugs can produce autonomic side effects. 1. Sympathetic Nervous System Function: During rest, there is some sympathetic tone, but parasympathetic tone dominates. SNS adjusts body function in response to stress (trauma, fear, hypoglycemia, cold, exercise) “fight or flight” response is the activation of the sympathetic nervous system during emergencies. All sympathetic effector organs and tissues are activated due to its anatomic wiring and release of EPI from the adrenal medulla:↑ HR, CO, & BP, blood flows from skin and internal organs into skeletal muscle. Energy stores are mobilized, and pupils and bronchioles dilate. A FEAR REACTION activates the ANS sympathetic division to cause HYPERTENSION. Activating the sympathetic

portion of the ANS causes alpha1 adrenergic receptor activation which causes arteriolar vasoconstriction with an associated elevation of BP, leading to hypertension. Sympathetic activation of the eye causes MYDRIASIS (dilation). Sympathetic activation of the heart causes tachycardia. Sympathetic activation of salivary glands causes a thick, ropey-type salivary flow (decreased salivation). 2. Parasympathetic Nervous System Function: required for life to maintain essential DIGESTION & EXCRETION body functions. Parasympathetic actions oppose the sympathetic nervous system and dominate during “rest & digest”. Operates in discrete units based on specific body needs (specific parasympathetic effectors are activated as needed). Does not discharge as a complete system as it would produce massive, undesirable symptoms (organophosphate poisoning). Cranial nerves with parasympathetic activity: III, VII, IX, X. Miosis (pupillary constriction), bradycardia,

and increased salivation are physiological effects from activation of the parasympathetic nervous system of the ANS. 4 Types of Drug-Receptor Binding: implicit in the interaction of a drug with its receptor is the CHEMICAL BONDING of that drug to one or more specific sites on the receptor molecule. 1. Ionic Bonds-result from the electrostatic attraction between ions of opposite charge. 2. Hydrogen Bonds-a special type of interaction between polar (water) molecules. 3. Van der Waals forces-collectively describe the weak interactions that develop when two atoms are placed in close proximity. 4. Hydrophobic interactions-occur between the drug, its receptor, and the aqueous environment that can play a major role in stabilizing drug-receptor binding. Covalent Bonds-arise from the sharing of electrons by a pair of atoms, and are required for the structural integrity of molecules. Covalent bonds are generally NOT involved in drug-receptor interactions. Ex: Nitrogen Bonding. Implicit in the interaction of a drug with its physiologic receptor is that the drug does not create effects, but rather

modulates intrinsic physiological cell functions. Drugs can bind to 4 major families of physiological receptors to produce effects: 1. Receptors as Enzymes (cell surface protein kinases)-kinases exert their regulatory effects by phosphorylating proteins within the cell which alters the cell’s biochemical activities. By binding to kinases, drugs can also alter biochemical activities resulting in a drug effect. 2. Ion Channels-drugs can bind to ion channels in cell membranes to cause channel opening or closing. This alters the cell’s membrane potential to result in a drug effect. 3. G-protein Coupled Receptors-when drugs bind to G-protein receptors, second messengers like cyclic AMP are produced to cause an effect within the cell, resulting in a drug effect. 4. Receptors in the Cell Nucleus-receptors for steroid hormones are soluble DNA-transcription factors within the nucleus that regulate the transcription of specific genes. Modifying transcription of these genes produces a drug effect. ALPHA (α) RECEPTORS: are mainly EXCITATORY in nature (responsible for most excitatory effects like vasoconstriction and contraction of the uterus and spleen.

Alpha adrenergic receptors are located on vascular smooth muscle, pre-synaptic nerve terminals, blood platelets, fat cells, and CNS neurons. Exception: some alpha receptors mediate relaxation of GI smooth muscle. 2 types of alpha (α) receptors: 1. Post-junctional α1 adrenergic receptors: found in radial smooth muscle of the iris, arteries, arterioles, and veins, and in the GI tract. The most common alpha receptor. Causes CONTRACTION & VASOCONSTRICTION. NE & EPI combine with α1 receptors. Found in arterioles in skin, mucosa, viscera, and kidney (resistance vessels). 2. Pre-junctional α2 adrenergic receptors: inhibits the release of norepinephrine (NE). Less common alpha receptor. NE & EPI combine with α2 receptors. Found on presynaptic nerve endings to inhibit NE release. Found on post-synaptic endings in the CNS to decrease sympathetic tone. BETA (β) RECEPTORS: MAINLY responsible for INHIBITORY effects like vasodilation & relaxation of respiratory smooth muscle. Exception: some beta receptors mediate excitatory increases in the heart’s force & rate of contraction. 2 types of Beta (β) receptors: 1. post-junctional β1 adrenergic receptors: found in

HEART myocardium cells (heart mainly contains β1 receptors), intestinal tract smooth muscle, and adipose (fat) tissue. These are the less common beta receptors). NE & EPI bind to β1 to ↑ HR, CO, BP, and the force of contraction. 2. post-junctional β2 adrenergic receptors: the most common beta receptor found in BRONCHIOLAR & VASCULAR smooth muscle. These are found in arterioles and arteries in skeletal muscle to cause vasodilation and in bronchial and uterine smooth muscle to cause relaxation. Only EPI combines with β2 receptors to vasodilate bronchioles (treats asthma), skeletal muscle, uterus (relaxation), ↑ blood glucose (gluconeogenesis, glycogenelysis). NE usually combines weakly or not at all to β2 receptors. Pharmacologic Agonist-a drug that binds to physiologic receptors to result in specific cellular effects producing a predictable pharmacological response. Drugs that bind to physiologic receptors and mimic the regulatory effects of endogenous signaling compounds produce a pharmacologic effect due to the binding to the receptor. A drug that elicits a full response through this process is a pharmacologic agonist. • Partial Agonist-a drug that acts on the physiologic receptor, but elicits an effect that is only partly as effective as an agonist drug.

Pharmacological Antagonist-a drug that binds to the physiologic receptor, but does not trigger an effect. When an antagonist drug is present, the agonist drug cannot reach the receptor site to produce an effect. 1. Competitive Antagonism-occurs when a response can be achieved by increasing the agonist dose in the presence of an antagonist. 2. Non-Competitive Antagonism-occurs when a response cannot be achieved with increasing doses of agonist in the presence of an antagonist.

ADRENERGIC AGONISTS (SYMPATHOMIMETICS AGENTS) SYMPATHOMIMETIC AGENTS (ADRENERGIC AGONISTS): both directly & indirectly mimic effects of STIMULATION of organs and structures of the sympathetic nervous system. These agents bring about tissue responses resembling those produced by stimulation of the sympathetic nervous system. Adrenergic receptors: α1, α2, β1, & β2. Adrenergic agonists therapeutic uses: • Control superficial hemorrhage-α1 adrenergic agonist causes vasoconstriction with EPI (Adrenalin). • Allergic shock (anaphylaxis)-α1 adrenergic agonist causes vasoconstriction with EPI (Adrenalin), and β2 adrenergic agonists causes relaxation of bronchial smooth muscle (airway dilation). Adrenergic agonists are used to reverse an anaphylactic reaction. EPI is the prototypical adrenergic agonist. o EPI stimulates both beta-adrenergic receptors and alpha-adrenergic receptors in the sympathetic division of the autonomic nervous system (ANS). o Anaphylaxis is characterized by rapid, extreme reduction in BP and

bronchospasms. When injected, EPI rapidly reverses hypotension by causing vasoconstriction via the α1 receptor stimulation (EPI dilates bronchial tubes via β2 receptor stimulation, and increases cardiac output via β1 receptor stimulation on the cardiac muscle). • Nasal decongestant-α1 adrenergic agonist causes vasoconstrictor with Phenylephrine (Neo-Synephrine). • Bronchial relaxation/dilation & airway dilation (Asthma)-β2 adrenergic agonist o Asthma-a respiratory disorder characterized by recurring episodes of paroxysmal dyspnea, wheezing on expiration, coughing, and viscous mucoid bronchial secretions. Asthma episodes may be precipitated by inhalation of allergens or pollutants, infection, vigorous exercise, or emotional stress. o Bronchodilators (β2 adrenergic agonists that treat an acute asthma attack): EPI, Albuterol (Proventil), Salmeterol (Serevent), & Metaproterenol (Alupent). These drugs stimulate beta receptors in the airway to cause bronchodilation, thus are used to help reverse an acute asthmatic attack. They are taken via aerosol, inhalers, and nebulizer. o Aminophylline-a THEOPHYLLINE

compound administered orally as bronchodilators in reversible airway obstruction due to asthma or COPD (chronic obstructive pulmonary disease). Theophylline compounds RELAX bronchial smooth muscle to improve airway function. A CNS stimulant that treats asthma. • Cardiac stimulation-β1 adrenergic agonist like Isoproterenol. • Epinephrine (Adrenalin), Phenylephrine (NeoSynephrine), Albuterol (Proventil; Ventolin), & Isoproternol are all adrenergic agonists the bind to adrenergic receptors. 2 Types of Adrenergic Agonists (Sympathomimetic Agents): 1. Direct-Acting Agonist: those drugs that interact with α or β receptors. Direct-acting adrenergic agonists can be receptor selective or receptor non-selective: • Phenylephrine (Neo-Synephrine): α1 selective agonist. A nasal decongestant, and mydriatic in ophthalmic preparations to treat chronic orthostatic hypotension, and in combination with local anesthetics to prevent anesthetic diffusion away from the injection site. 100x less potent than EPI, • Clonidine (Catapres): α2 selective agonist. Used

primarily as an anti-hypertensive agent. • Dobutamine: β1 selective agonist. • Terbutaline: β2 selective agonist administered orally, subcutaneously, or by inhalation. Primarily used in long-term treatment of obstructive airway diseases or emergency treatment of bronchospasm. Given parenterally in emergency treatment of status asthmaticus and to delay premature delivery. • Albuterol: β2 selective agonist. Administered orally or by inhalation. Primarily used in long-term treatment of obstructive airway diseases, emergency treatment of bronchospasm, or to delay premature delivery. • Epinephrine (Adrenaline): a vasoconstrictor and α1,2 & β1,2 direct acting agonist (stimulates 4 receptors). o Physiologic actions produced by EPI: • Constricts arteriolar blood vessels (vasoconstriction) via binding to α1 receptors. Alpha receptor stimulation causes a vasopressor response (↑BP). • Relaxes bronchial smooth muscle (bronchodilation) by binding to β2 receptors. Beta-receptor stimulation causes airway dilation and increased cardiac output. In this way, EPI counteracts the vascular effects of histamine-related anaphylaxis. • Decreases blood volume in nasal tissues.

• Causes a hypertensive response. • Produces physiologic actions that are opposite (antagonize) the effects of HISTAMINE. EPI (Vasoconstrictor) Therapeutic Indications: Alleviates symptoms of an acute asthma attack via its bronchodilator properties. EPI treats bronchospasm associated with hypotension (i.e. anaphylaxis). EPI is the agent of choice to treat/reverse anaphylactic reactions (given sublingually or subcutaneously) because it has desirable vasopressor activity, bronchodilator properties, and has a rapid onset of action. EPI has stimulatory effects on α & β adrenergic receptors. EPI treats hypersensitivity reactions. The major reason that EPI vasoconstrictor is added to local anesthetics is to prolong the activity/duration of the local anesthetic (anesthesia) by decreasing the rate of diffusion and absorption from the injection site. EPI also reduces systemic toxicity by reducing the rate of vascular absorption into the systemic circulation, & provides hemostasis by reducing/controlling local bleeding at the injection site. EPI also enhances the onset of action, decreases

bleeding, and allergic reactions. EPI is used to restore cardiac activity in cardiac arrest relieves congestion of the nose, sinuses, and throat. Treats glaucoma by reducing internal eye pressure. Controls superficial hemorrhage/bleeding. EPI is administered IV, sublingually, subcutaneously, or intramuscularly. It has a very rapid onset of action when given via these routes. If NE or EPI stimulate or combine with eye αreceptors, they cause MYDRIASIS (pupil dilation). EPI is ineffective in treating hypotension because of its alpha receptor stimulatory actions on the vasculature which could cause an even further elevation of BP. EPI does not reduce anxiety, but increases anxiety because it has CNS stimulatory effects. EPI is a sympathomimetic agent used in dentistry as the vasoconstrictor for anesthetic solutions to prolong the duration of local anesthesia, and is the vasoconstrictor component used in gingival retraction cords.

Contraindications: patients with ANGINA conditions because EPI’s cardiostimulatory effects aggravate this condition. Common EPI side effects: headaches, agitation (anxiety), and tachycardia. EPI is used with caution in patients with high BP and hyperthyroidism. These patients may have an increased sensitivity to EPI. Norepinephrine (NE): α1,2 & β1 agonist. Isoproterenol: β1,2 agonist (a synthetic catecholamine) and the MOST POTENT BRONCHODILATOR. 2. Indirect-Acting Agonists (Sympathomimetic): drugs that cause release of stored NE at post-ganglionic nerve endings to produce their effects (Tyramine, Amphetamine, Methamphetamine, Hydroxyamphetamine, Methoxamine, & Ephadrine). Sympathomimetics are pharmaceutical agents that bring about tissue responses resembling those produced by stimulation of the sympathetic nervous system (SNS). Sympathomimetic, sympathomimetic amines, and adrenergic agents are all synonymous terms. In medicine, sympathomimetics (DA, EPI,

NE, isoproterenol, and phenylephrine) are used as pressor agents to maintain BP in vascular shock. They are used as bronchodilators for asthma attacks and for allergic states like anaphylactic shock. • Amphetamines-sympathomimetic amines (indirect-acting agonists) that stimulate CNS (central nervous system) & PNS (peripheral nervous system), increase systolic & diastolic blood pressures, and act as weak bronchodilators and respiratory stimulants. Amphetamines have a high abuse potential resulting in tolerance, psychological dependence, and severe social disability. Abuse causes extreme violence and paranoid psychotic behavior. They pass readily into the CNS and cause a rapid release of NE in the brain. • Amphetamine Therapeutic Uses: * Attention Deficit Hyperactivity Disorder (ADHD; Hyperkinesis)-ADHD Drugs: o Methylphenidate (Ritalin)-a mild CNS stimulant that increases attention span, reduces hyperactivity, and improves behavior in children with ADHD.

o Focalin (Dexmethylphenidate)-a new form of methylphenidate. o Concerta (Extended-Release Methylphenidate)-a long-acting form of Ritalin. o Adderal (Mixed Amphetamine Salts)-acts the same as Ritalin to treat ADHD. Adderal is a mixture of Dextroamphetamine and Amphetamine (Adderall). Adderall XR is sometimes used instead of Methylphenidate (Ritalin). o Strattera (Atemoxetine)-first non-stimulant approved for treating ADHD in children &adults. o Metadate CR (Controlled-Delivery Methylphenidate)-another long-term acting ADHD drug. o Dextroamphetamine (Dexedrine) * Narcolepsy-Dextroamphetamine (Dexedrine) is used to prevent daytime sleepiness. * Weight loss-Phentermine (Lonamin) is structurally similar to Dextroamphetamine.

SYMPATHOLYTIC AGENTS (ANTIADRENERGICS) SYMPATHOLYTIC AGENTS (ANTI-ADRENERGIC)-drugs that acts in a way opposite to the sympathetic nervous system. 4 types that all treat hypertension: 1. Beta-Adrenergic Receptor Blockers (β Blockers): with all “selective” beta-blockers, selectivity for the β1 is lost at high doses. As the dose is increased, they also block β2 receptors, thus having effects on bronchial smooth muscle. The most common adverse side effects of beta blockers are WEAKNESS & DROWSINESS. Beta blockers treat hypertension, angina, cardiac arrhythmias, MI, glaucoma, and prophylaxis of migraine. • Propranolol (Inderal), Timolol, & Nadolol (longestacting)-lipid soluble drugs that blocks both β1 & β2 receptors (thus are “non-selective” beta-blockers). Widely used to treat hypertension (↓BP by ↓CO). Non-selective beta blockers are absolutely contraindicated in patients with asthma or other chronic obstructive airway disease as they cause fatal bronchospasm. Selective β1 blockers are contraindicated in the patients. Also, contraindicated

in patients with insulin-dependent diabetes as they block hypoglycemia recovery. Propranolol exerts its major anti-anginal effect by BLOCKING beta-adrenergic heart receptors. Propranolol is the drug of choice for adrenergically induced arrythmias. • Acebutolol (Sectral)-a β1 cardioselective antagonist that treats hypertension and controls ventricular arrhythmias. It has low lipid solubility which reduces is likelihood of producing adverse CNS effects, and has mild intrinsic sympathomimetic activity (partial agonist activity at β2 receptors) similar to Pindolol. • Metoprolol (Lopressor)-competitive β1 cardioselective antagonist that blocks β1 receptors to treat hypertension, acute angina pectoris, and may be helpful after a heart attack (very similar to Atenolol). • Atenolol (Tenormin)-competitive β1 cardioselective antagonist that blocks β1 receptors to treat hypertension, chronic angina pectoris, or after a heart attack (MI recovery). Has a long plasma ½ life (long duration of action). Due to its low lipid solubility, it is excreted by the kidneys, minimally metabolized, and has a low potential for causing CNS

side effects compared to lipid-soluble beta-blockers like Propranolol. • Metoprolol & Atenolol are longer-acting and more predictable than Propranolol in producing therapeutic plasma levels. Since they are β1 selective, they are safer to use in patients with asthma or bronchitis. 2. Alpha-Adrenergic Receptor Blockers (α Blockers)pharmacological agents that can cause tachycardia, lower BP, vasodilation, and orthostatic hypotension (postural hypotension)-a fainting spell that occurs due to a rapid fall in BP when moving from the supine to the upright position when getting out of the dental chair. The symptoms are similar to simple fainting, but the condition is related to positioning. Any α antagonist can cause EPI reversal. The major pharmacological effect of αantagonists (blockers) is to ↓BP, eliciting reflex tachycardia. Alpha blockers are medications that act by competitively inhibiting catecholamine actions at the alpha receptor site to CAUSE BLOOD VESSELS TO RELAX (DILATE) and are used to reduce high BP and treat an enlarged prostate. While HYPOTENSION is the major adverse effect of alpha blockers, they cause relatively few adverse effects. Orthostatic Hypotension (Postural Hypotension)abnormally low BP occurring when an individual assumes

the standing posture. After vasovagal syncope, orthostatic hypotension is the 2nd most likely cause of transient unconsciousness in the dental office. • Factors that can cause orthostatic hypotension: administration and ingestion of drugs, prolonged recumbency and convalescence, inadequate postural reflex, pregnancy, various defects in the legs, Addison’s Disease, physical exhaustion, starvation, and chronic orthostatic hypotension (Shy-Drager Syndrome). The incidence of orthostatic hypotension increases with age. NSAIDs are not known to produce orthostatic hypotension as an adverse effect. • Drugs that can cause orthostatic hypotension: 1. Antihypertensives: (Guanethidine/Ismelin). 2. Phenothiazines: (Chlorpromazine/Thorazine & Thioridazine/Mellaril). 3. Tricyclic Antidepressants: (Doxepin/Sinequan, Amitriptyline/Elavil, & Imipramine/Tofranil). 4. Narcotics: (Meperidine/Demerol & Morphine). 5. Antiparkinson Drugs: (Levodopa/Larodopa/Dopar & Carbidopa + Levodopa (Sinemet). 2 TYPES OF ALPHA BLOCKERS (α1 & α2).

1. Selective Alpha Antagonists: block α1 receptors to treat cardiac conditions (hypertension) & Benign Prostatic Hyperplasia (BPH). ♦ Doxazosin (Cardura)-selective to block α1 receptors to treat hypertension. The preferred agent for hypertension due to its longer duration of action. ♦ Prazosin (Minipress)-selectively blocks α1 receptors and RARELY used to treat hypertension due to unwanted adverse effects. ♦ Terazosin (Hytrin)-selective to block α1 receptors to manage mild-to-moderate hypertension, and treat benign prostate hyperplasia (BPH). 2. Non-Selective Alpha antagonists: blocks α1 & α2 receptors and usually DO NOT TREAT CARDIAC CONDITIONS as blocking both alpha receptors can cause tachycardia (rapid heart beat) & palpitations (pounding heart beat). Used in pre-surgical management of Pheochromocytoma & Raynaud’s Phenomenon. ♦ Tolazoline (Priscoline)-selectively blocks α2 receptors (has moderate α2 adrenergic blocking activity). A parenteral anti-hypertensive agent whose actions are caused by a direct peripheral vasodilation (Tolazoline is not a centrally acting

anti-hypertensive). Treats persistent pulmonary hypertension of the NEWBORN. ♦ Phentolamine hydrochloride (Regitine) & Phenoxybenzamine hydrochloride (Dibenzyline)-a non-selective blocker of both α1 & α2 receptors. Their major clinical use is in pre-surgical management of Pheochromocytoma (an adrenal medulla tumor that releases excessive EPI & NE causing hypertension, tachycardia, & arrhythmias). Epinephrine Reversal-a predictable result of using EPI with a patient who has received an α-blocker. One of the best known effects of the alpha-receptor blocking agents (anti-adrenergic) is their ability to reverse the “pressor” action of adrenaline (EPI). In the absence of blocking agents, EPI & NE both cause BP to increase. After the alpha-receptors are blocked by an alpha blocker, the pressor effect of NE is reduced or abolished while EPI causes a fall in BP. This is because EPI stimulates α & β receptors in the cardiovascular system, but NE only stimulates alpha receptors (NE lacks β2 effects). After blocking the α receptors, only beta-receptors can be stimulated. “Pressor” Response-produces an increase in BP and is mediated by α-receptors. A “depressor” response produces a decrease in BP, and is mediated by β2

receptors. 3. Centrally-Acting Anti-hypertensive Agents-α2 selective agonists that inhibit adrenergic nerve transmission through actions within the CNS. They reduce BP by reducing cardiac output, vascular resistance (or both). • Clonidine (Catapres)-an α2 selective agonists used in combination with a thiazide diuretic & hydralazine that works by controlling nerve impulses along certain nerve pathways to relax blood vessels so blood flows through them easier to help lower BP (reduces HR, CO, and total peripheral resistance). • Guanfacine (Tenex) & Guanabenz (Wystensin)stimulate α2 adrenergic receptors to inhibit sympathetic nervous system outflow, reducing peripheral vascular resistance, and are used either alone, or with a thiazide diuretic. • Methyldopa (Aldomet)-most effective when combined with a diuretic to produce a FALSE transmitter (alpha-methylnorepinephrine) that replaces NE in the vesicular storage sites and is released by the nerve impulse. Most beneficial for treating hypertension in patients with RENAL DAMAGE. • Methyldopa Adverse Effects: cardiovascular

(orthostatic hypotension, bradycardia), CNS (sedation & fever), GI (colitis), and possible hepatitis and cirrhosis. 4. Neuronal Depleting Agents-deplete catecholamine (NE, EPI, & serotonin) stores from adrenergic terminals and in the brain. Reserpine (blocks uptake of NE, EPI, and serotonin into storage vesicles). & Guanethidine (blocks release of NE). Alpha and Beta adrenergic blocking agents act by COMPETITIVE INHIBITION of post-junctional adrenergic receptors.

CHOLINERGICS Autonomic nervous system (ANS) has CHOLINERGIC FIBERS that secrete Acetylcholine, and ADRENERGIC FIBERS that secrete Norepinephrine, EPI, or Dopamine (catecholamines). CHOLINERGIC FIBERS (Neurons)-release/secrete ACh. Cholinergic is a nerve ending that releases acetylcholine (ACh) as the primary neurotransmitter. It is also a synapse in which ACh is the primary neurotransmitter. 1. Preganglionic sympathetic & Preganglionic parasympathetic fibers 2. Postganglionic parasympathetic fibers. The action of ACh at postganglionic parasympathetic sites is called a “muscarinic response” Postganglionic sympathetic fibers are NOT cholinergic fibers, but ADRENERGIC fibers. Adrenergic blocking agents block the effect of impulses transmitted by adrenergic postganglionic sympathetic fibers via competitive inhibition. Postganglionic sympathetic neurons (fibers) that innervate sweat glands secrete acetylcholine. Acetylcholine Receptors-cholinergic receptors that are

subdivided as either: 1. Muscarinic receptors-are located mainly in autonomic effector cells (heart, vascular endothelium, smooth muscle, presynaptic nerve terminals, and exocrine glands) in the CNS. Muscarinic receptors respond to muscarine and ACh. The action of ACh at postganglionic parasympathetic sites is called a “muscarinic response”. 2. Nicotinic receptors-are located in ganglia, skeletal muscle end plates, and CNS. Nicotinic receptors respond to nicotine and ACh, but not to muscarine. A “nicotinic response” describes the stimulating action of ACh on the ganglia and its action at the neuromuscular junction of skeletal muscle. There are 2 major nicotinic receptors: 1. Nicotinic receptors at neuromuscular junctions of the somatic nervous system (SNS). Neuromuscular blockers act here. 2. Nicotinic receptors at autonomic ganglia in both sympathetic & parasympathetic systems. Ganglionic blockers act here. Drugs that chemically resemble ACh bind to muscarinic and nicotinic receptors and imitate the effects of parasympathetic postganglionic activity. Acetylcholine is the chemical mediator at all autonomic ganglia and

parasympathetic postganglionic synapses, and transmitter substance at the neuromuscular junction in skeletal muscle and sweat glands. Local anesthetics prevent or reduce the liberation of ACh at the NMJ. ACh causes an alteration in cell membrane permeability to produce the following cholinergic actions: Cholinergic Drug Actions: slowing of the heart (bradycardia), pupil constriction (miosis), stimulation of smooth muscles of the bronchi, GI tract, gallbladder, bile duct, bladder, and ureters (urination). Stimulation of sweat, salivary, tear, and bronchial glands. Cholinergic drugs are very useful to induce salivation and stimulate Acetylcholine (ACh) cholinergic receptors to cause salivation, miosis (papillary constriction), excessive sweating, flushing, increased GI motility (peristalsis), and bradycardia. Cholinergic drugs increase secretions, a cholinesterase inhibitor also increases secretions because it reduces acetylcholine metabolism. An overdose of a cholinergic drug causes: sweating, urination, bradycardia, copious serous saliva, but does NOT cause mydriasis (pupil dilation), since this is an adrenergic response.

All “M” receptors are CHOLINERGIC that bind to Acetylcholine (ACh) and are antagonized by Atropine. There are 3 classes of CHOLINERGIC AGONISTS (Choline Esters, Cholinergic Alkaloids, & Cholinesterase Inhibitors). These drugs stimulate muscarinic sites by mimicking the actions of Acetylcholine. However, if any of the cholinergic agents are administered before Acetylcholine, the action of ACh is enhanced and prolonged. These drugs are referred to as muscarinics, cholinergics, cholinomimetics, or parasympathomimetics: Indirect-Acting Cholinergic Agonists (Cholinesterase Inhibitors): their action increases the effects of Acetylcholine within the autonomic nervous system (ANS) and at neuromuscular junctions. By inhibiting cholinesterase, acetylcholine produced in the body is NOT broken down into by-products, but is allowed to accumulate into substantial levels to stimulate the cholinergic portions of the ANS and to stimulate skeletal muscle contractions at neuromuscular junctions. Thus, the actions of cholinesterase inhibitors causes a CHOLINERGIC EFFECT (indirect cholinergic effect) since these drugs indirectly cause a cholinergic response by allowing acetylcholine to build up in the body. The stimulation of skeletal muscle by EXCESS acetylcholine eventually causes muscle paralysis. Cholinesterase inhibitors inhibit acetylcholinesterase as

both muscarinic and nicotinic sites. 1. Edrophonium-an indirect-acting cholinergic agonist (cholinomimetic). Drug of choice to “diagnose” myasthenia gravis because of its rapid onset of action and reversibility. However, it does not treat myasthenia gravis due to its very short duration of action. It is also useful in differentiating between a myasthenic crisis and cholinergic crisis. It is a rapid, short-duration, parental cholinesterase inhibitor. 2. Neostigmine & Pyridostigmine: prescribed to treat myasthenia gravis and can reverse the blockade caused by non-depolarizing neuromuscular blocking drugs during general anesthesia. 3. Malathion & Parathion-insecticides. 4. Physostigmine-a reversible cholinesterase inhibitor. Treats glaucoma, Alzheimer’s, delayed gastric emptying, and orthostatic hypotension. Pralidoxime (Protopam)-a cholinesterase reactivator used as an antidote to reverse muscle paralysis due to organophosphate anticholinesterase pesticide poisoning. It also reverses the effects of an overdose of anti-cholinesterase agents used in the treatment of myasthenia gravis (i.e. Neostigmine, Pyridostigmine, and Ambenomium). Pralidoxime treats poisoning with

an organophosphate cholinesterase inhibitor. • Organophosphate Poisoning Symptoms: excessive salivation, bronchoconstriction, diarrhea, and skeletal muscle fasciculations (twitching). • Organophosphates-esters of phosphoric acid and an organic alcohol that inhibits the cholinesterase enzyme. 1. Isoflurophate & Echothiophate-treats glaucoma. 2. Malathion-a widely used insecticide. 3. Parathion-an insecticide. 4. Tabun-one of the most potent and toxic nerve gases. 5. Metrifonate-an anthelmintic agent that destroys intestinal worms. Direct-Acting Cholinergic Drugs: drugs that produce a cholinergic effect via direct stimulation of the cholinergic receptors. Esters and Alkaloids are used to stimulate smooth muscle activity and are direct-acting cholinomimetic agents whose effects resemble Acetylcholine. Two groups of drugs directly stimulate cholinergic receptors to cause a “direct cholinergic effect”: 1. Choline Esters: the most noticeable effects of choline esters are decreased BP due to generalized vasodilation, flushing of the skin, slowing of HR, and

increased tone and activity of the GI and urinary tracts. Topical application of these drugs to the eye causes miosis and decreases intraocular pressure. • Methacholine (Provocholine)-not used much anymore. • Carbachol (Isopto-Carbachol)-used in ophthalmology to produce miosis. • Acetylcholine Chloride-used in ophthalmology to produce miosis. • Bethanecol (Urecholine)-used for post-operative abdominal distension and urinary retention. 2. Cholinergic Alkaloids: • Pilocarpine (Pilocar or Salagen)-the most useful alkaloid used as a MIOTIC and to treat open-angle glaucoma and xerostomia. • Muscarine, Nicotine, & Lobeline. XEROSTOMIA-caused by medications (i.e. antihypertensives & antidepressants), cancer therapy (chemotherapeutic drugs & radiation treatment), Sjogren’s Syndrome, head & neck trauma causing nerve damage, and conditions like bone marrow transplants, endocrine disorders, stress, anxiety, depression, and nutritional deficiencies. • While xerostomia is not a disease, it can be a symptom of certain diseases, and can cause health problems by affecting nutrition and psychological health. Xerostomia can contribute to and increase the

chances of having tooth decay and mouth infections. • Temporary relief of xerostomia can come from saliva substitutes, sugarless hard candies, glycerinebased cough drops and lemon flavored glycerine mouthwash. Additionally, medications can be added, changed, or dosages altered to provide increased salivary flow. In dentistry, cholinergics drug treat dry mouth (Xerostomia) by inducing salivation. Cholinergic drugs used are: 1. Pilocarpine (Salagen)-a cholinergic agonist and alkaloid indicated to treat xerostomia caused by salivary gland hypofunction caused by radiotherapy for head and neck cancer by stimulating salivary flow. Common side effects: excess sweating, nausea, heartburn, and diarrhea due to the drug’s cholinergic nature. 2. Cevimeline (Evoxac)-a cholinergic agonist indicated to treat xerostomia in patients with Sjogren’s Syndrome. Common side effects: increased sweating, nausea, heartburn, diarrhea due to the drug’s cholinergic nature. Specific for the M3 receptor on the salivary glands. Saliva Functions: 1. washes away food debris and plaque from teeth to

prevent decay. 2. limits bacterial growth that cause decay and other mouth infections. 3. bathes the teeth and supplies minerals that allow remineralization of early cavities. 4. lubricates foods so they can be swallowed easier. 5. provides enzymes that aid in digestion. 6. helps us “enjoy” food by aiding in the “tasting” process. 7. moistens skin inside the mouth to make chewing and speaking easier. Cholinergic Crisis Symptoms: bradycardia (decreased HR), lacrimation, extreme salivation, vasodilation, and muscle weakness. Because a cholinergic crisis can causes muscle weakness like that of a myasthenic crisis, distinguishing the two conditions is difficult. Administering a short-acting cholinomimetic like edrophonium improves a myasthenic crisis, but worsens a cholinergic crisis. Typical cholinergic effects caused by stimulation of acetylcholine receptors (cholinergic receptors) are salivation, miosis, excessive sweating, flushing, increased GI motility and bradycardia. Mecamylamine (Inversine)-a nicotinic ganglion-blocking drug.

ANTI-CHOLINERGIC (ANTIMUSCARINIC) DRUGS ANTI-CHOLINERGIC (ANTI-MUSCARINIC) DRUGS: produce the opposite effects of cholinergic agents like dry mouth (xerostomia), mydriasis (papillary dilation), antispasmodic actions, decreased GI motility, reduction in gastric and salivary secretions, tachycardia, and dry skin. Anti-cholinergic drugs are contraindicated in GLAUCOMA PATIENTS. Adrenergic, Cholinergic, and Adrenergic blocking drugs are not contraindicated with glaucoma patients. Anti-cholinergic Actions: • Inhibits secretions of all nasal glands, mouth (dry mouth/xerostomia), pharynx, and respiratory tract. • An inhibitory effect on GI motility, thus can cause constipation and urinary retention. • Increases HR (tachycardia) and body temperature, and dilates the pupils (mydriasis). • Anti-cholinergic agents cause xerostomia by blocking postganglionic cholinergic fibers. • Anticholinergic drugs block receptor sites for

acetylcholine and decrease salivary flow and respiratory secretions during surgery. Anti-cholinergic drugs do not have their own intrinsic activity, but simply occupy the receptor site and prevent acetylcholine from occupying the same receptor. Accepted Therapeutic Indications: Parkinson Disease, motion sickness, post-operative bladder syndrome, and traveler’s diarrhea. Anti-cholinergic Contraindications: glaucoma, cardiovascular problems, GI or GU tract obstruction, and asthma. Anticholinergic/Anti-muscarinic Drugs: • Glycopyrrolate (Robinul)-treats traveler’s diarrhea and post-operative bladder syndrome. • Methantheline-an antimuscarinic to relieve cramps or spasms, ulcers. • Propantheline Bromide (Pro-Banthine)treats traveler’s diarrhea and is an antisecretory. • Benzotropine Mesylate (Cogentin)-treats Parkinsonism (an anti-parkinsonism). • Trihexyphenidyl HCL (Artane)-treats Parkinsonism (an anti-parkinsonism). • Atropine Sulfate-produces mydriasis and

cycloplegia. • Scopolamine-prevents or reduces motion (sea) sickness. NICOTINIC RECEPTOR ANTAGONIST (Nicotinic Blockers)-are divided into ganglionic-blocking drugs and neuromuscular blocking drugs: 1. Ganglionic Blockers-among the most potent agents available, but are SELDOM USED due to the annoying and sometimes disabling parasympathetic blockade which causes pronounced xerostomia, constipation, blurred vision, and postural hypotension. Ganglionic blockers have very limited clinical use. ♦ Mecamylamine (Inversine) & Trimethaphan (Arfonad)-treat severe or malignant hypertension, and during an emergency hypertensive crisis. These ganglionic blockers cause a rapid and reversible FALL IN BP that enables them to immediately reverse an emergency hypertensive crisis. Also used to create a “bloodless surgical field”. ♦ Hexamethonium & Tetraethylammonium-are no longer available in the U.S. for clinical use. 2. Neuromuscular Blockers-important for producing complete skeletal muscle relaxation and facilitating ENDOTRACHEAL INTUBATION as an adjunct to surgical anesthesia. Neuromuscular blocking drugs

interact with nicotinic receptors at the skeletal neuromuscular junction. Major danger of these drugs is they can lead to excessive paralysis (patient cannot breath and subsequent death). 2 classes of neuromuscular blockers. 1. Non-Depolarizing (Competitive) NMJ Blockers-competitively bind and compete with ACh at cholinergic nicotinic receptors to prevent acetylcholine from stimulating motor nerves, to cause muscle paralysis. Tubocurare (Curare)-the prototype nondepolarizing NMJ blocker. ♦ Other NMJ blockers: Gallamine (Flaxedil), Mivacurium (Mivacron), Vecuronium (Norcuron), Doxacurium (Nuromax), Pancuronium (Pavulon), Atracurium (Tracrium), Cisatracuronium (Nimbex), and Rocuronium (Zemuron). ♦ Neostigmine or Pyridostigmine-are cholinesterase inhibitors used to reverse the NMJ blockade (paralysis) caused by NMJ blockers. 2. Depolarizing NMJ Blockers (Non-Competitive)Succinylcholine (Anectine)-the prototype depolarizing NMJ blocking agent, and only depolarizing NMJ blocker used in the U.S.

Succinylcholine acts as a nicotinic agonist and depolarizes (desensitizes) the neuromuscular endplate. It binds to the ACh receptor and stimulates depolarization causing initial excitation, followed by a block of neurotransmission and muscle paralysis. Succinylcholine is used with caution in patients with low levels of pseudocholinesterase (enzyme that breaks down succinylcholine) which can result in respiratory failure. Succinylcholine can also cause muscarinic responses like bradycardia and increased glandular secretions. Administration of succinylcholine to a patient deficient in serum cholinesterase would most likely result in prolonged apnea. SKELETAL MUSCLE SPASMOLYTIC DRUGS (Skeletal Muscle Relaxants)-agents that relieve muscle spasms without paralysis by acting in the CNS or in skeletal muscle, but not at the neuromuscular end plate. Skeletal muscle relaxant drugs are used in certain chronic CNS diseases (i.e. multiple sclerosis, cerebral palsy, and cerebrovascular accidents/strokes) associated with painful muscle spasms. By reducing the muscle spasms, pain is reduced and patient mobility is improved. 1. Baclofen (Lioresal)-a derivative of GABA to treat chronic muscle spasm. Its site of action in reducing muscle spasms is the spinal cord to treat multiple

sclerosis and other spinal cord diseases. (Note: Diazepam (Valium) & Tizanidine (Zanaflex) also act in the spinal cord and are effective muscle relaxants). 2. Carisoprodol (Soma)-used to treat chronic muscle spasms and pain associated with acute TMJ pain. Its exact mechanism of action is not clear, but many effects have been attributed to its central depressive action. 3. Cyclobenzaprine (Flexeril)-used to treat acute muscle spasm through a central action, possibly at the brain stem level. Used to relive acute, painful musculoskeletal conditions. It is not effective for muscle spasm secondary to cerebral or spinal cord disease. 4. Methocarbamol (Robaxin)-a centrally acting muscle relaxant used to relive acute, painful musculoskeletal conditions and to manage tetanus. 5. Quinine-widely used to effectively relieve nocturnal leg cramps. Bioavailability-measures the rate & amount of therapeutically active drug that reaches the systemic circulation. A drug’s bioavailability is affected by the dissolution of a drug in the GI tract and destruction of a drug by the liver. • IV injection route provides complete 100%

bioavailability. • Bioavailability = amount of drug absorbed/amount of drug administered.

DRUG ADMINISTRATION ROUTES DRUG ADMINISTRATION ROUTES: drug’s onset of action is PRIMARILY determined by the RATE of ABSORPTION. The major effect of a drug is a factor (is determined by) how much of the drug is FREE IN PLASMA. ENTERAL ROUTE OF ADMINISTRATION (ORAL, RECTAL, SUBLINGUAL): 1. Oral Route (P.O.)-takes ~30min for the onset of a drug’s effect after swallowed. The oral route allows the use of many different dosage forms like tablets, capsules, and liquids. Oral route is an example of an ENTERAL ROUTE of administration. It is the most common route where the drug is swallowed. It is the most convenient for safe drug administration. It is safe, painless, and economical. The oral route is the safest and easiest route for drug administration, but it is also the most unpredictable and least effective route available. Oral route is most known for its significant hepatic “FIRST PASS” metabolism. The oral

administration of a drug is the one most accepted by patients. It is convenient because drugs can be given via tablets or capsules that contain an exact dose, making it easy for the patient to take the drug without assistance. After oral administration, drugs are generally absorbed best from the DUODENUM which has a large surface area due the presence of villi and microvilli. Drugs taken by mouth have to be absorbed (usually from the small intestine) before they can be transported to their site of action. Absorption may be slow, unpredictable and irregular due to the presence of variable amounts of food in different stages of digestion and to the varying degrees of acidity and alkalinity of the digestive juices. Moreover, blood from the intestinal tract passes first to the liver (some drugs are metabolized in the liver, while others may be stored there to be released only slowly). These considerations make it clear that oral administration is usually NOT USED IN MEDICAL EMERGENCIES or other occasions when a rapid effect is needed. Oral Route Disadvantage: drugs must be absorbed (usually from the small intestine) before

they can be transported to their site of action. Blood from the intestinal tract passes first to the liver (some drugs are metabolized in the liver “first-pass effect”, while others may be stored there to be released slowly). This consideration makes it clear that oral administration is not suitable in emergencies or other occasions when a rapid effect is needed. Emotional stress decreases the rate of absorption of a drug when given orally. 2. RECTAL ROUTE: administered as creams, enemas, or suppositories if the patient is VOMITING or UNCONSCIOUS. Not often used for systemic effects because drugs are poorly and irregularly absorbed rectally and there is POOR PATIENT ACCEPTANCE. 3. Buccal or Sublingual Route-a tablet is placed under the tongue or in the cheek (ex: Nitroglycerin for angina pectoris). Parenteral Administration (not via the intestine or GI tract): 4. Intramuscular Injection (IM)-the onset of action of drugs injected into muscle occurs rapidly (~5min) because of high blood flow through the muscles. IM injection is an injection made into a large skeletal muscle. Absorption from an IM injection is often faster

and yields a higher bioavailability than oral administration. Advantages: results in uniform absorption and can be used for solutions too irritant for subcutaneous injection. The speed of absorption of drugs given IM depends on the vehicle in which they are dissolved (absorption is rapid from aqueous solutions and slow from oily solutions). Other factors affecting absorption are vasoconstriction and congestive heart failure (slow absorption). IM injections Sites: buttocks, deltoid muscle, and anterior thigh (usually the site for young children). The bicep muscle is NOT an acceptable site for an IM injection. Proper needle depth in muscle is 1 inch into big adult muscle and ¼ inch in children. NEVER go deeper than 2/3 of the needle length. 5. Subcutaneous Injection-the onset of drugs injected under the skin takes ~15 minutes. Injected beneath the skin. Absorption may be less rapid. 6. Intra-arterial Injection-injected into a specific artery. Use caution because it burns. 7. Intravenous Injection-drug administration route that

produces the MOST RAPID ONSET of pharmacological effect by injecting directly into the bloodstream. When a drug is given IV, it is placed directly into the systemic circulation and delivered rapidly to all tissues and the drug receptor sites. For all other drug administration routes (except intra-arterial injection), the drug must be systemically absorbed prior to distribution to the drug’s receptor sites. Thus, the onset of pharmacological effects is slower. • When a drug is given by IV injection there is complete 100% bioavailability (the entire dose is placed into systemic circulation). With other administration routes, the drug can be lost before reaching the systemic circulation (i.e. with “first pass” effects a portion of an orally administered drug is eliminated through degradation by liver enzymes before the drug reaches its receptor sites). • A major advantage of IV drug administration is it allows for TITRATION of the drug. Other advantages: rapid onset, drugs that cause irritation when administered subcutaneously can be given IV without irritation. In case of an emergency, there is an open line through which emergency drugs can be injected. • IV Disadvantages: OVERDOSE (due to rapid onset),

phlebitis (due to local irritation), drug irretrievability, allergy, side effects of high plasma drug concentrations. 8. Inhalation-gases like nitrous oxide are rapidly absorbed through the lungs and gain access to the general circulation within 5 minutes. Inhalation (using nitrous oxide) is the MOST frequently utilized route of administration to sedate pediatric patients. The drug is given as an aerosol into the respiratory tract. 9. Topical Administration-includes ointments & creams applied to the skin and mucous membranes. The drug is placed on the skin to produce a LOCAL DRUG EFFECT. Not intended for systemic drug administration. 10. Patch Delivery-skin patches release drug into the bloodstream over 12-24hrs. Not intended for rapid drug administration. Transdermal route allows the drug to be placed on the skin to produce a SYSTEMIC EFFECT. Initial distribution of a drug into the tissues is mainly determined by the RATE OF BLOOD FLOW TO THE TISSUE, while a drug’s affinity for the tissue determines if the drug will concentrate at that site. *Gastric emptying time and degree of plasma protein binding (albumin) also effect drug distribution, but are less important that the RATE of blood flow to the tissues.

MOST DRUGS travel through the bloodstream by binding to ALBUMIN protein which is abundant is plasma which enables the drug to be carried to all tissue and organs. A drug bound to plasma albumin always has some portion of the drug that does not bind. This portion is free to leave the blood compartment to be taken up by tissues where the drug will elicit its pharmacological effect. The remaining “bound” portion of the drug then continuously releases more free drug to be taken up by tissues. Eventually, all of the drug in the blood compartment will be absorbed by this process. Interactions between two or more drugs can occur if they compete for binding on the plasma albumin. If drug “A” is bound to albumin before the patient takes drug “B”, and drug “B” has a greater binding affinity to albumin than drug “A”, then when drug “B” is binds, it will displace drug “A” from albumin, resulting in large amounts of unbound drug “A”. This may cause adverse reactions due to the sudden large amounts gaining access to the tissues and organs. Physiochemical properties of drugs that influence their passage across biologic membranes are lipid solubility, degree of ionization, molecular size, and molecular shape. The mechanism of drug transfer across biological membranes is by: 1. Passive Transfer-is essential to various processes of metabolism.

• Simple Diffusion: a process where lipid-soluble substances move across the lipoprotein membrane. Most drugs penetrate biomembranes by simple diffusion through membrane phospholipids. The amount of drug dissolving in the membrane at any time is directly proportional to the concentration gradient and its degree of lipid solubility (Note: only non-ionized drugs are soluble in lipid). • Filtration: water-soluble molecules small enough to pass through membrane channels may be carried through the pores by the bulk flow of water. Drugs of molecular weights of 60,000 or less can “filter” through capillary membranes. • Osmosis: movement of a pure solvent like water through a semi-permeable membrane from a solution that has a lower solute concentration, to a solution with a higher concentration. The membrane is impermeable to the solute, but is permeable to the solvent. 2. Specialized Transport: • Active Transport: involves lipid-insoluble substances (i.e. glucose) which are “shuttled” across plasma membranes by forming complexes with specific membrane constituents or “carrier molecules” that are within the cell to provide energy

for transporting drugs to regions of high concentration. • Facilitated Diffusion: term given to carrier-based transfer when the driving force is simply the concentration difference of the drug across the membrane. Most drugs are absorbed by facilitated diffusion. Factors Influencing Hepatic Drug Metabolism: 1. Microsomal enzyme inhibition: many drugs and environmental agents can inhibit many of the CYP isoforms of the P450 microsomal drug metabolizing system. Thus, many drugs usually metabolized by the particular CYP inhibited, will not be effectively metabolized and will achieve higher than expected blood levels. 2. Microsomal enzyme induction: agents that induce higher levels of the microsomal drug metabolizing enzymes may cause a more rapid metabolism of other drugs, thus resulting in lower than expected blood levels of a drug. 3. Plasma protein binding: drugs highly bound to plasma proteins will not enter the liver to be metabolized. This causes a longer plasma half-life of the drug.

4. Genetic factors: there is individual variance through genetic factors that contribute to different rates of drug metabolism in the hepatic microsomal enzyme system. 5. Pathology: hepatic impairment and liver disease usually will impair the microsomal drug metabolizing system. This causes elevated levels of unmetabolized drug. Factors that Control Urinary Elimination of Drugs: 1. Glomerular Filtration: all drugs are filtered through the glomerulus to enter the renal tubules. The amount of drug varies based on the degree of plasma protein binding, and bound drugs are not subjected to filtration. 2. Tubular Reabsorption: once the drugs enter the renal tubules, they may be reabsorbed back into the bloodstream through the renal tubular cells. Reabsorption favors the highly lipid soluble agents; the converse is that highly polar compounds are not effectively reabsorbed and are effectively excreted from the renal tubules. 3. Active Transport: rate of renal elimination also depends on if active transport into or out of the tubular fluid occurs. Other Excretory Drug Pathways:

♦ GI tract excretes some drugs through feces. This is not as common as urinary excretion. ♦ Most drugs can be detected in saliva after administration, but the salivary glands are not considered a route of drug excretion since the drug is re-swallowed along with saliva. ♦ Lungs excrete volatile compounds that were inhaled into the respiratory system. Nitrous oxide and volatile general anesthetics are excreted by the lungs. ♦ Some drugs are excreted through sweat glands, but this route accounts for only a small percentage of drug excretion. The most important enzyme systems for the biotransformation of drug molecules are found in the LIVER. Hepatic metabolism of drugs occurs in Phase I reactions catalyzed by a microsomal mixed-function oxidase system (P450 system) and in Phase II reactions is known as conjugation reactions. 1. Phase I Reactions: occur in the liver microsomal enzyme system (mixed-function oxidase system or P450 system). In this system, drug metabolism occurs in 3 basic patterns. First, the active parent drug can be converted into the inactive metabolite. Second, an active parent drug may be converted into a second active

compound that is subsequently converted into an inactive compound. Third, an inactive parent drug may be transformed into an active compound. • The most common reaction in drug metabolism is an oxidation reaction in which oxygen in the form of a hydroxyl group is attached to the drug molecule. • There are at least 5 distinct groups of microsomal drug metabolizing enzymes. These enzyme “families” are identified as a cytochrome (CYP prefix) followed by their numerical digestion (i.e. 1A2). Thus, the enzyme CYP 1A2 is a distinct drug metabolizing enzyme that converts a variety of drugs into the oxidized product. 2. Phase II Reactions: conjugation reactions involve coupling the drug with an acid present in cells (usually glucuronic acid). When coupled to glucuronic acid, the process is known as glucuronide conjugation with the resulting metabolite referred to as the “glucuronide”. Conjugations occur in the liver, kidney, and to a lesser extent in other tissues. • Conjugation of drugs results in polar, water-soluble compounds that are rapidly excreted in urine. Thus, the parent drug is effectively rendered inactive and transported out of the body by this process.

DEA DRUG SCHEDULE Controlled Substance Act of 1970-uses 4 criteria for including a drug into one of five DEA drug schedules: Potential for abuse (most important criteria); Medical usefulness, and degree to which the drug produce a physiological and/or physical dependence. DEA Drug Schedule: prescriber must have a Drug Enforcement Agency authorization number (DEA#) to prescribe scheduled drugs. A restrictive new rule change from the DEA makes it more difficult for doctors to prescribe opioids (hydrocodone combination products) moving them from Schedule III to Schedule II effective 10/6/14, to help prevent drug diversion and overdose associated with controlled substances. This could also negatively impact patients who truly need access to analgesics for dental pain. I. Schedule I: not legitimate for medical use. Among the substances classified by the DEA are Mescaline, LSD, Heroin, & Marijuana. Special licensing procedures must be followed to use these and other Schedule I substances. These drugs CANNOT BE PRESCRIBED. Only available for specific approved research projects.

II. Schedule II: considered to have a strong potential for ABUSE or ADDICTION, but have legitimate medical use. Substances classified by the DEA like Amphetamines, Morphine, Cocaine, Pentobarbital, Oxycodone, Methadone, and straight Codeine and various combinations of hydrocodone (ex: hydrocodone and acetaminophen = Vicodin; Lorcet). Must have a written prescription signed by a health professional (laws vary by state). These drugs can be written prescribed, but cannot be refilled. A new prescription must be written for refills. Prescriptions CANNOT BE CALLED INTO THE PHARMACY. III. Schedule III: have less potential for abuse or addiction than Schedule I or II drugs. These include various analgesic combination with < 90mg of codeine per dosage unit codeine (i.e. Acetaminophen with codeine = Tylenol 3 and Suboxone). Must have a written prescription signed by a health professional (laws vary by state). These drugs may be called into the pharmacy. The prescriber can authorize refills without writing a new prescription. IV. Schedule IV: a category of drugs with less abuse or addiction potential then Schedules I-III. These substances include Diazepam (Valium), Lorazepam (Ativan), Triazolam (Halcion), Alprazolam (Xanax), and chloral hydrate.

V. Schedule V: a category of drugs that have only a small potential for abuse and addiction. These substances include many commonly prescribed medications that contain only a small amount of Codeine. FDA determines which drugs are to be sold by prescription only. The prescription must have the address of the patient and dentist, and the dentist’s DEA number/license. Sample Prescription:

• Superscription: patient’s name, address, age, date. • Inscription: drug name and strength (i.e. 500mg tablets).

• Subscription: directions to the pharmacist (dosage form and amount to be dispensed). • Transcription (signa): directions to the patient (sig). • Signature: signature of person prescribing the medication MUST appear. • Always document prescriptions given to a patient in the patient’s chart, along with the date they were written and any specific instructions for patient use.

ANESTHETICS Local Anesthetics: adverse side effects of local anesthetics either cause TOXICITY or ALLERGY. 1. Toxicities: caused by too much anesthetic in the bloodstream, affecting the CNS and cardiovascular system. • CNS toxicity effects: restlessness, stimulation, tremors, convulsive seizures followed by CNS depression, slowed respiration, and even coma. • Cardiovascular effects: bradycardia and reduced cardiac output. 2. Allergies: • hypersensitivities and allergic reactions to local anesthetics, especially from amides are rare. These reactions are manifested as dermatologic reactions and edema at the injection site. • Asthmatic wheezing syndromes have occurred in response to local anesthetic injections. • Allergic reactions are more prevalent with ester local anesthetics (nasolabial swelling, itching, and oral mucosal swelling), not amide local anesthetics. BISULFITES: the component of a local anesthetic

solution that causes an ALLERGY. Patients may exhibit hypersensitivity to sulfites contained in some anesthetics. Sodium metabisulfite prevents the oxidation (deterioration) of the EPI vasoconstrictor in commercial preparations containing EPI. Most patients who react to bisulfites have a history of asthma, and the airway is hyperactive to the sulfites. Allergic reaction usually results in an asthmatic syndrome of wheezing and bronchial constriction. Bisulfites are present in only those commercial preparations containing EPI (vasoconstrictor). Preparations without EPI like Mepivacaine 3% (Carbocaine) do not contain bisulfites. Hypersensitivity or allergic reactions to local anesthetics (especially amides) are much more rare than allergic reactions to bisulfites. Local anesthetics REVERSIBLY BLOCK nerve impulse conduction and produce the reversible loss of sensation at their administration site. They are incorporated within the nerve membrane or bind to specific Na+ ion channels, restricting sodium permeability in response to partial depolarization. Local anesthetics do not produce loss of consciousness • Mechanism of Action: on the nerve axon, local anesthetics DECREASE Na+ UPTAKE through the axon’s sodium channels. When a local anesthetic is injected near the nerve, the solution interferes with the uptake of sodium from outside→inside the nerve by blocking the specific sodium channels,

thus blocking Na+ uptake. This decreases the nerve’s excitability below a critical level, and nerve impulses fail to propagate along the axon. Since axons carry pain sensations, these sensations will not be carried, and a blockage of pain results. Local anesthetics have NO EFFECT ON POTASSIUM (K+) at the nerve axon. • Local Anesthetic Mechanism: decrease pain sensation by blocking propagation of nerve impulses (“nerve block”). At therapeutic doses, locals block voltage-gated sodium (Na+) channels to inhibit generation and conduction of action potentials. • Local anesthetics help reduce saliva flow during operative procedures by reducing sensitivity and anxiety during tooth preparation. Local anesthetics DEPRESS small, unmyelinated nerve fibers that conduct pain & temperature sensations FIRST, and depress large, myelinated fibers LAST. Small nerves have a greater surface-volume ratio that accounts for the rapid onset of action. Clinically, the order of loss of function caused by local anesthetics is: • PAIN (first)→temperature→touch→proprioception→skeletal muscle tone (last)

• pain threshold-the lowest level of pain a patient will detect. Factors Influencing Absorption & Effects: • Site of injection–pH in area (acidic ↓ absorption; alkaline ↑ absorption), extent of tissue vascularity & perfusion, effects of local inflammation, or tissue damage (often a problem after injury). • Inflammation-lowers pH of surrounding tissue, making it more difficult for local anesthetic to permeate the membrane. An inflamed, acidic environment ↓ penetration of local anesthetic into tissue. Thus, is more difficult to achieve nerve block and satisfactory anesthesia in inflamed injured tissue. Local anesthetics in theory are LESS effective in acutely inflamed tissue than in normal tissue because inflamed tissue’s pH decreases (becomes more acidic) which decreases the available free base. At body pH (7.4), a local anesthetic when infiltrated, will chemically exist as a portion which is ionized (has a proton attached which is merely a hydrogen H+ atom) and as a non-ionized portion (no proton attached). The ionize portion has difficulty penetrating the nerve and will not be effective. The nonionized portion penetrates the nerve to cause anesthesia (this non-ionized portion is the “free base”). The more

proportion of anesthetic in the “free base” form, the more effective the anesthetic. When tissue conditions are normal (pH = 7.4), ~10-20% portion of an infiltrated local anesthetic is in the free base form (non-ionized form), which is enough to penetrate the nerve to cause anesthesia. When tissues are acidic (i.e. tissue infection), less free base portion exists and more ionized (H+) portion exists. Thus, there is not enough free base form to penetrate the nerve to cause anesthesia, and the local anesthetic when infiltrated into the tissue site is ineffective at the normal anesthetic doses. Non-ionized (free base form) is the form that readily penetrates tissue membranes. Local anesthetic free bases are fat-soluble (lipophilic) drugs that are converted into their water-soluble (hydrophilic) hydrochloride salts to allow preparation of an injectable solution. In solution, an equilibrium is established between the ionized & nonionized forms of the local anesthetic. The amount of drug in the ionized form depends on the pKa of the drug, and pH of the solution. At the usual solution pH of 6.0 or less, most locals are almost completely in the ionized form. Local anesthesia is obtained only if sufficient free base form is available. The lower the drug’s pKa, and higher the pH of the solution or injected tissues, the MORE free base available. Once the local anesthetic is injected, the buffering capacity

and tissue’s pH (normally 7.4), shifts the equilibrium in favor of the free-base form. At physiologic 7.4 pH, approximately 5-20% of the local anesthetic is in the free-base form, which is enough to penetrate and produce anesthesia. If infection or inflammation exists, the tissue’s pH may be acidic which greatly reduces the concentration of the freebase form, rendering the local ineffective. Main Point: the potential action of all local anesthetics depends on the ability of the anesthetic SALT to LIBERATE THE FREE-BASE. If you inject Lidocaine (pKa = 7.8) into tissues with a 7.8 pH, the Lidocaine will exist in an equal mixture of ionized & nonionized forms which will be more than enough to produce anesthesia. ESTER LOCAL ANESTHETICS: are metabolized in PLASMA. All esters have an “ester” grouping them within their chemical structure. An ester grouping is a bridge or link containing the –COOCH2- configuration. Drugs with 1 letter “i” in the name. • Ester local anesthetics are mainly available as TOPICAL anesthetics (Benzocaine, Tetracaine, & Dibucaine). Esters are also available as medical anesthetic preparations like Propoxycaine (Ravocaine). Ester-type local anesthetics are no longer available as dental anesthetic injectable

preparations due to their relatively high allergy incidence. • Esters are metabolized by the plasma enzyme “plasma cholinesterase” or “pseudocholinesterase” which splits the ester linkage within the chemical structure rendering the anesthetic ineffective. • Procaine (Novocaine)-one of the original estertype local anesthetics. When Procaine was metabolized by plasma cholinesterase, a highly allergic compound “Paraaminobenzoic Acid” (PABA) was formed. Many patients developed an allergy to PABA. Hydrolysis of procaine occurs mainly in the plasma. • Cocaine-the only local anesthetic that increases the pressor activity of EPI & NE. However, cocaine has no place in the routine practice of dentistry. Cocaine is a naturally occurring ESTER of benzoic acid and was the first local anesthetic used in dentistry and medicine. Cocaine is potent and highly toxic, and is the only local anesthetic that causes DEFINITE VASOCONSTRICTION. Cocaine is commercially available in various forms, and is applied to mucous membranes of the oral, laryngeal, and nasal cavities for use as a topical anesthetic. Cocaine causes euphoria, and abuse can

lead to a physical dependence. Although it is an excellent local anesthetic, the risk of abuse and intense local vasoconstriction prevents cocaine from being more widely used clinically. Cocaine’s pharmacology is unique among local anesthetics because it inhibits catecholamine uptake (NE, EPI, Dopamine, Serotonin) by adrenergic nerve terminals. This increases (potentiates) the action of endogenously released and exogenously administered sympathomimetic amines like Dobutamine, Dopamine, or EPI. Cocaine increases the risk of developing cardiac arrhythmias and hypertension (cocaine increases the pressor activity of these sympathomimetic amines). • ESTERS have a rapid onset & short duration of activity (except tetracaine which has a longer duration). Esters hydrolyzed in blood by butylcholinesterase have short t1/2. Allergic reactions & precipitation of acute asthmatic attacks are concerns with ester-type generally used topically AMIDE LOCAL ANESTHETICS: all are metabolized in the LIVER (except Articaine), and the metabolites are then renally excreted. All amides have an “amide” grouping within their chemical structure. An amide grouping is a bridge or link containing the –CONHCH2configuration. Amides are present in urine as the parent

compound in a greater percentage due to their more complex process of biotransformation. Amides are the only local anesthetics presently available as dental injectables (Lidocaine (Xylocaine), Prilocaine (Citanest), Bupivacaine (Marcaine), & Mepivacaine (Carbocaine), Etidocaine, & Articaine). All drugs with two letter “i” in the name. Amides are metabolized by the “hepatic microsomal enzyme system”, and the products formed to not have anesthetic actions and are excreted from the body by the KIDNEY. Amides are used with caution, or not at all in patients with compromised liver function. AMIDES-have a longer duration of action, and are metabolized by P450 enzymes in the liver so toxicity is more likely if amides are given to individuals with liver dysfunction or if given with other drugs that may alter hepatic metabolism. Amides less likely to produce allergic reactions. • Articaine (Septocaine)-the only amide-type local anesthetic metabolized in the BLOODSTREAM. It is chemically unique because it has an ester group attached to its molecule that can be acted upon by plasma cholinesterase to render it ineffective. Thus, it is the only amide metabolized in the bloodstream, and not the liver. Articaine is supplied as articaine HCL 4% solution with EPI 1:100,000. It is indicated for local, infiltrative, or conductive anesthesia in simple and complex dental and periodontal procedures. The onset of anesthesia after administration is 1-6min after injection. Complete anesthesia lasts ~1hr.

Articaine is contraindicated in patients with hypersensitivity to local anesthetics of the amide type or to sodium bisulfite. • Maximum dose of Articaine (Septocaine) recommended in one appointment is expressed as mg/kg body weight (not as total mg). 7mg/kg is the maximum recommended dose of Articaine in children and adults. In a typical kg adult male, the 7mg/kg dose = 490mg. Thus, the maximum recommended amount of Articaine that could be given to a 70kg adult in one appointment is 490mg. • The table below shows the number of dental cartridges containing 1.7ml volume of solution to provide the indicated amounts of Articaine Hydrochloride 4% and EPI 1:100K. The carpule fluid volume of 1.7ml is unique for Articaine, and is not the standard carpule volume of 1.8ml as with other dental anesthetics.

• Prilocaine (Citanest)-a local anesthetic amide used for nerve block, epidurals, and regional anesthesia. It

has an intermediate duration of action and is longer acting than Lidocaine, produces less vasodilation than equal amounts of Lidocaine, and is somewhat less potent than Lidocaine. Prilocaine is available as a 4% solution with or without EPI, which prolongs the anesthetic effect. While Prilocaine is 50% as toxic as Lidocaine, since methemoglobinemia is a possible reaction, Prilocaine is not used for patient with hypoxic conditions or patients with Hepatic (liver) disease. Prilocaine is metabolized into orthotoluidine (a product than can produce methemoglobinemia, a condition characterized by increased levels of methemoglobin in the blood which is less effective then hemoglobin in carrying oxygen in the blood. • Bupivacaine (Marcaine)-has the longest duration of action of any dental local anesthetic available. May be used with EPI. Appropriate for extended procedures although long duration of effect increases risk of systemic absorption & toxicity. Radiotoxic in some patients and used with caution if cardiovascular disease, elderly, or pediatric population. Often used in labor & procedures where motor control is essential because exhibits strong preference for sensory fibers & is long-acting. • Lidocaine (Xylocaine): an ANTI-ARRHYTHMIC

AGENT effective ONLY on the ventricle, often administered intravenously to treat life-threatening ventricular arrhythmias. When given IV to treat ventricular arrhythmias, it acts on the fibrillating ventricles to decrease cardiac excitability and spares the atria. It can effectively reverse a lifethreatening situation. Lidocaine and Mepivacaine are most likely to show cross-allergy. Lidocaine is a local anesthetic drug used topically in dentistry. • Mepivacaine (Carbocaine)-EQUAL TO LIDOCAINE in efficacy and used without EPI. Ineffective for topical application. Levonordeferin can be used as a vasoconstrictor. Duration of anesthesia in soft tissue is shorter than Lidocaine, thus less useful for procedures lasting > 25 minutes. TOXIC TO NEONATES, so avoid for labor and infants. Not best for dental procedures more than 30 minutes (short effects).

Local Anesthesia in Children: WEIGHT determines the maximum dose of a local anesthetic that can be administered in children. For Lidocaine (2%), a dosage of 4.4mg/kg is not exceeded (maximum adult dose of 300mg).

A dental local anesthetic carpule contains 1.8ml of a 2% Lidocaine solution with 1:100,000 EPI. Thus, the carpule contains 36mg of Lidocaine and 0.018mg of EPI. • Important: 1mL of a 2% solution of Lidocaine with 1:100K EPI contains 20mg of Lidocaine and 0.010mg of EPI. • Important: 1 dental carpule contains 1.8ml solution. Thus, 1.8ml of 2% Lidocaine solution with 1:100K EPI contains 36mg of Lidocaine and 0.018mg EPI.

Maximum recommended adult dose of Lidocaine is 300mg. To reach this max level, 15ml of 2% Lidocaine is needed. Important: there are 20mg of Lidocaine in every 1ml of 2% Lidocaine. o 300mg/20mg = 15ml. o 20mg x 1.8ml (in every carpule) = 36mg per carpule. o 300mg/36mg = 8.3 carpules (8 carpules of 2% Lidocaine can be used).

CHLORAL HYDRATE-only non-barbiturate sedativehypnotic agent indicated in the practice of dentistry. Traditionally used ORALLY in pre-operative management of the ANXIOUS PEDIATRIC DENTAL PATIENT. It has an unpleasant odor and bitter, caustic taste that can be partially masked in a flavored syrup. • Rapidly absorbed after oral administration with an onset of action of 15-30min. Its duration of action is ~4hrs. • Chloral hydrate is a sedative and hypnotic widely used for pediatric sedation.

• For children, chloral hydrate is available as a 500mg/5ml solution. The usual child dose is 50mg/kg up to a max of 1g. Children will often enter a period of excitement and irritability before sedation, as it affects brain centers that control wakefulness and alertness. Chloral hydrate DOES NOT RELIEVE PAIN. • Chloral hydrate is a prodrug that is metabolized to the active metabolite (Trichloroethanol) which may displace Warfarin from its protein binding sites causing an increase in the hypoprothrombinemic response to warfarin. • Sedative effects and/or respiratory depression with Chloral hydrate may be additive with other CNS depressants, so monitor for increased effects (includes ethanol, anti-depressants, narcotic analgesics, and benzodiazepines.

NITROUS OXIDE Nitrous Oxide (N2O)-a slight sweet smelling, colorless, inert gas at room temperature and pressure that cannot produce general anesthesia EXCEPT if administered at concentrations > 80% (thus, it cannot be used as a single agent to produce general anesthesia). At these concentrations, the lack of oxygen causes hypoxia. Inhalant anesthetics like halothane & isoflurane can produce general anesthesia at concentrations of 3-5%, thus are very useful in anesthesia. Nitrous oxide is stored under pressure in steel cylinders painted blue. Oxygen is stored in green tanks. • Advantages of Nitrous Oxide Analgesia: rapid onset of action, elevates pain threshold, produces euphoria, pleasant induction, titratable, rapid and complete recovery, virtually no adverse effects in absence of hypoxia, therapeutic for many medically compromised patients, and is suitable for all ages. • Nitrous oxide is used to produce SEDATION & MILD ANALGESIA. Nitrous oxide’s main therapeutic effect is relaxation/sedation (mild analgesia is a secondary effect). N2O is usually used in 30-50% concentrations along with pure oxygen. It is a colorless, non-irritating gas at root

temperature and pressure, and is non-flammable and non-explosive. Nitrous oxide delivery machines are pre-equipped with a failsafe mechanism that will not allow less than 20% oxygen to be delivered to the patient (nitrous oxide MUST be coupled with AT LEAST 20% oxygen). Nitrous oxide does not have local anesthetic (analgesic) properties. Thus, local anesthesia must be used in conjunction with nitrous oxide any procedure where pain is anticipated. • Onset of sedation occurs within 5 min and the recovery is just as rapid. The FIRST SYMPTOM of nitrous oxide onset is TINGLING OF THE HANDS. It is excreted unchanged by the lungs. Most common complaint from patients taking N2O is mild nausea. Always give the patient 100% oxygen after the procedure to prevent diffusion hypoxia. • N2O is quickly absorbed from the lungs and is physically dissolved in the blood. There is no biotransformation, and the gas is rapidly excreted by the lungs when the concentration gradient is reversed. It is recommended that the patient be maintained on oxygen for 5-10 minutes AFTER the sedation period. Nitrous oxide has a rapid onset (5min) and rapid recovery (5min). • Contraindications: patients with upper respiratory

infections, emphysema, bronchitis, 1st trimester of pregnancy (long-term exposure to low nitrous oxide doses can increase the incidence of spontaneous abortions), and in patients where communication is difficult (i.e. autistic patients). Nitrous is never used on patients with a contagious disease as it is difficult to sterilize the entire tubing. Environmental contamination by nitrous oxide is kept to a minimum by employing a scavenger system. • Important: Nitrous oxide is a SEDATIVE, NOT a general anesthetic because hypoxic levels are required to produce anesthesia. It is used alone to produce sedation or in combination with inhalation agents to supplement the anesthetic response.

Inhaled Ammonia-drug of choice for acting against SYNCOPE. Inhaled ammonia irritates trigeminal nerve sensory endings to cause a reflex stimulation of the medullary respiratory and vasomotor centers. An aromatic ammonia vaporole is crushed between the fingers and held near the patient’s nose. Administration of oxygen aids in combating tissue anoxia. Syncope Symptoms: beads of sweat on the

upper lip, weak thready pulse, cold clammy skin, pallor and a dizzy feeling. Loss of normal vasomotor tonus produces pooling of blood peripherally so that normal blood volume becomes insufficient. Placing the patient in a SUPINE POSITION & ELEVATING THE FEET gives the patient a transfusion of whole blood by utilizing forces of gravity. The head should not be more than 10° lower than the rest of the body. Types of Syncope: • Vasovagal, Neurogenic, & Orthostatic (treat all with high-flowing 100% oxygen). • Hyperventilation Syndrome-oxygen is not indicated. • 100% oxygen is contraindicated in patients with Chronic Obstructive Pulmonary Disease (COPD). 4 Stages of General Anesthesia that apply to INHALANTS (not IV general anesthesia): 1. Stage 1 (Amnesia/Analgesia)-patient experiences analgesia, eventually amnesia. 2. Stage 2 (Excitement/Delerium)-patient experiences excitation, struggles, and is possibly delirious. Begins with unconsciousness, ending with loss of eyelid reflex, purposeless movements, hyper-reaction, dilated pupils, reflex vomiting, tachycardia, and hypertension.

3. Stage 3 (Surgical Anesthesia)-patient begins stage with regular breathing, loss of reflexes, loss of pupil eyelash reflexes, complete loss of pain. Has 4 planes. 4. Stage 4 (Medullary Paralysis)-patient begins stage with loss of all spontaneous breathing and severe depression of vasomotor and respiratory centers in the Medulla. Cessation of respiration, ending in death without proper treatment. GENERAL ANESTHESIA AGENTS: 1. Inhalation Agents/Anesthetics: volatile liquids (i.e. Desflurane, Sevoflurane, Halthane, Isoflurane, & Enflurane) that decrease arterial pressure. • Isoflurane-increases coronary blood flow and is the safer drug to use if the patient has ischemic heart disease. Used for elderly. • Halothane & Enflurane-decreases cardiac output so NOT used for patients with history of cardiac arrhythmias. Halothane sensitizes the heart to catecholamines, increasing risk of ventricular arrhythmias in susceptible patients (cardiac disease, patients on sympathomimetic drugs or high levels of catecholamines). Enflurane-has CNS irritant effect at high doses, abnormal spike patterns, so avoid in patients with seizure disorders.

• Ether (diethyl ether), Halothane, Enflurane, Isoflurane, Sevoflurane, Methoxyflurane, & Desflurane are inhalation anesthetics. Inhalation anesthetics are drugs that are vaporized from the liquid form and inhaled to produce general anesthesia. These diverse drugs are simple lipophilic molecules ranging from ethers (diethyl ether), halogenated hydrocarbons (halothane), and halogenated ethers (isoflurane, enflurane, sevoflurane, methoxyflurane, and desflurane). 2. INTRAVENOUS AGENTS: a primary advantage of IV sedation is the ability to TITRATE individualized dosage. 1. Barbiturates: Thiopental/Pentothal, Methohexital/Brevital. • Thiopental (Pentothal)-most commonly used ultrashort-acting barbiturate. • Methohexital (Brevital)-used in outpatient procedures due to more rapid recovery. High lipid solubility so enters brain very rapidly and is redistributed to other tissues, so very shortacting unless prolonged procedure (can accumulate, especially if liver dysfunction). Respiratory depression with decreased CO & arterial BP. 2. Benzodiazepines-IV preparations of longer-acting

Lorazepam (Ativan) & Diazepam (Valium) provide preoperative sedation (benzodiazepines do not provide anesthesia) but are more irritating. Provide anterograde amnesia to procedure. • Midazolam (Versed)-short-acting (very short ½ life) and more common for pre-anesthesia as part of “balanced anesthesia” program because it only makes the patient groggy, and does not induce loss of consciousness. Preferred to diazepam for IV injection because it is more water soluble. • Diazepam (Valium) & Lorazepam (Ativan)anti-anxiety drugs. Contraindicated in patients with Narrow Angle Glaucoma. Diazepam is used for muscle spasticity in patients with cerebral palsy. IV diazepam is useful in status epilepticus. • Flumazenil-benzodiazepine antagonist that accelerates post-operative recovery from benzodiazepine sedation. • Chlordiazepoxide (Librium), Clonazepam (Rivotril), Flurazepam (Dalmane), Temazepam (Restoril), Triazolam (Halcion) • Alprazolam (Xanax)-has selective anxiolytic

effects in patients who suffer from Agoraphobia. • Benzodiazepines are frequently prescribed for symptomatic treatment of ANXIETY & SLEEP DISORDERS. They produce their effects via specific receptors involving GABA. Benzodiazepines are the most effective ORAL SEDATIVE DRUGS USED IN DENTISTRY. • Benzodiazepines are NOT useful to manage obsessive-compulsive disorders. • Benzodiazepines (especially Diazepam & Chlordiazepoxide) are often prescribed for pre-operative sedation. These drugs are tranquilizers used to produce conscious sedation in anxious patients. Diazepam is more potent than Chlordiazepoxide. These drugs do not produce hangovers like barbiturates and other sedative drugs. *Do not used these drugs during pregnancy. 3. NEUROROLEPTANALGESICS-neuroleptic-opioid combinations that combine Fentanyl & Droperidol. Opioids–all provide analgesia and anesthesia. IV morphine & Fentanyl.

• Fentanyl-highly potent opioid used as premedication or adjunct to inhalational agents. Used with haloperidol derivative Droperidal & NO to provide balanced anesthesia “Neuroleptanesthesia.” Often used with gaseous anesthesia. Fentanyl decreases pulmonary ventilation & halothane’s induction rate. Adverse: impaired ventilation & respiratory depression, increase intracranial pressure, and may precipitate acute attacks in asthmatics. Opioid adverse effects are reversed & recovery hastened by administration of Naloxone (Narcan) an opioid antagonist. • Fentanyl is available as a trans-mucosal preparation (Actiq), a transdermal patch formulation (Duragesic), and as an IV preparation (Sublimaze). 4. OTHERS: • Propofol (Diprivan)-IV anesthetic with rapid onset/recovery (more rapid than barbiturates) and better tolerated than other IV agents (popular “day surgery”). Metabolized in liver via conjugation with very short t1/2. A respiratory depressant like thiopental, but produces significant ↓ in arterial BP & CO. • Benefits: antiemetic agent unlikely to

produce vomiting and nausea, used for induction or anesthesia maintenance. Does not increase intracranial pressure so used in concerned patients. Safer for pregnant women and less likely to produce bronchospasm. • Concerns: contraindicated in children because has produced severe acidosis if respiratory infection present. Use with caution if patient has hypotension. Serious infections in some patients after use. • Etimodate (Amidate)-major advantage over other IV drugs with minimal respiratory & cardiovascular depressant effects. Rapid induction & recovery, but no analgesic effects. Often used with opioids, does not produce hypotension or have significant HR effects. Used if coronary artery disease or hypovolemia is present, and maintains cardiovascular stability. Adverse: high incidence of vomiting, and serious adrenocorticosuppression. • Ketamine-drug of choice for “Dissociative Anesthesia”. Ketamine causes catatonia, amnesia, & analgesia without loss of

consciousness by acting as an antagonist at N-methyl-D-Asparate (NMDA) receptor, blocking excitatory effects. Ketamine is highly lipophilic and undergoes redistribution & hepatic metabolism. Ketamine is the ONLY anesthetic that acts as cardiovascular stimulant. Significantly increases cerebral blood flow & intracranial pressure, so avoid use of brain tumor or head injury. Adverse: abnormal sensory perception, strange dreams, disorientation (“emergence phenomena”). Give Diazepam to minimize. Limited use, but sometimes used in children or patients in shock due to its cardio-stimulatory properties. Does not produce bronchospasms so used in asthmatics. Not good for adults, children tolerate it better.

ANTI-ANXIETY AGENTS Benzodiazepines, Barbiturates, & Narcotic Analgesics ALL produce SEDATION and can cause physiologic dependence. • Flumazenil (Mazicon)-a benzodiazepine ANTAGONIST used to reverse residual effects of benzodiazepines in the event of an overdose. Tranquilizer-a drug that promotes tranquility by calming, soothing, quieting, or pacifying without sedation or depressant effects. • Major tranquilizers: anti-psychotic agents. • Minor tranquilizers: anti-anxiety agents (Benzodiazepines). BENZODIAZEPINES (Minor Tranquilizers)-used as oral preparations to alleviate anxiety (anti-anxiety), induce sleep (sedative-hypnotic), anti-convulsant, and skeletal muscle relaxant. Used for IV conscious sedation during outpatient surgery. Benzodiazepines produce their calming effects by DEPRESSING THE LIMBIC SYSTEM & RETICULAR FORMATION through potentiation of the central inhibitor neurotransmitter (neurons) gamma-amino-butyric acid (GABA). Tolerance and physical dependence can occur

with prolonged high dosage, but they are much safer than barbiturates. • Adverse Effects: CNS depression (fatigue, slurred speech, drowsiness, sleepiness/sedation, & confusion, disorientation), GI disturbances (dry mouth, vomiting, nausea, diarrhea), other effects are hypotension, ataxia, and muscle relaxation. Never take benzodiazepines with alcohol as serious potentiation of the sedative effects of each can occur causing unexpected inebriation & respiratory depression. However, with normal doses, benzodiazepines rarely effect the respiratory system of healthy individuals. • Benzodiazepines taken orally as Tranquilizers: Chlordiazepoxide (Librium), Lorazepam (Ativan), Alprazolam (Xanax), & Diazepam (Valium). • Benzodiazepines prescribed as HYPNOTICS for Insomnia: Flurazepam (Dalmane), Temazepam, & Triazolam (Halcion). o Triazolam (Halcion)-a pre-operative sedative in dentistry that is metabolized in the liver by P450 isoform CYP 3A4 enzyme. Drugs that inhibit the actions of CYP 3A4 affect triazolam by causing an increase in serum levels of triazolam. Triazolam interacts with drugs that inhibit its

metabolism via the CYP 3A4 enzyme. Drugs that inhibit the metabolic pathway may have a profound effect on the clearance of triazolam. The resultant effects would be an increase in serum concentrations with an associated unexpected increase in the actions of triazolam. Thus, triazolam is avoided in patients receiving potent inhibitors of CYP 3A4. o Antifungal agents (traconazole, Ketoconazole, Fluconazole, Miconazole, Voriconazole) can significantly elevate triazolam serum levels causing toxicity with therapeutic doses. These antifungals inhibit CYP 3A4 isoform responsible for hepatic metabolism of triazolam, thus inhibiting the normal metabolism of triazolam. Do not administer triazolam to patients taking these antifungals. • Midazolam (Versed)-comes in liquid form for preoperative sedation in children, and as an injectable for IV conscious sedation. An important adjunct in the practice of anesthesiology, used as pre-operative sedatives and induction agents, and to maintain anesthesia. Diazepam (Valium)-treats anxiety, nervous tension,

muscle spasm, & seizures/convulsions. Diazepam is preferred over a barbiturate as an anti-anxiety medication since it has less addiction potential, and produces less profound CNS depression. Diazepam affects the limbic system of the brain (controls emotion). ♦ Indications: agent of choice to reverse status epilepticus induced by a local anesthetic overdose. ♦ If given IV for status epilepticus, use a LARGE VEIN to decrease the risk of thrombophlebitis (propylene glycol is the substance in the IV mixture that is the main cause of thrombophlebitis after IV administration of Diazepam). An important adjunct in the practice of anesthesiology, used as pre-operative sedatives and induction agents, and to maintain anesthesia. ♦ Contraindications: acute narrow-angle glaucoma & psychoses. ♦ Adverse Effects: most common side effects are drowsiness and fatigue. At a high therapeutic index, produces some amnesia, can be locally irritating to tissue, and may produce local thrombophlebitis. Serious adverse reactions are withdrawal symptoms caused by discontinuation of treatment. Diazepam is the drug of choice to treat lidocaine-

induced seizures. Buspirone (BuSpar)-a minor tranquilizer orally administered anxiolytic (anti-anxiety agent) whose mechanism works by DIMINISHING SEROTONERGIC ACTIVITY. Buspar is structurally and pharmacologically distinct from all other anxiolytics (including benzodiazepines & barbiturates) and distinct as it does not have anticonvulsant or muscle relaxant properties, does not impair psychomotor function or cause sedation (lacks CNS depressant activity) or physical dependence. Buspirone’s special characteristics: ♦ Slow onset of action (may take up to two weeks). ♦ Different mechanism of action than barbiturates or benzodiazepines. Most likely acts as a partial agonist on certain 5-hydroxytryptamine (serotonin) receptors, and diminishes serotonergic action. ♦ Fewer side effects and less sedation than benzodiazepines. ♦ Low abuse potential (benzodiazepines & barbiturates have abuse potential and may cause dependence). ♦ Does not possess hypnotic or anti-convulsant properties (benzodiazepines & barbiturates do). ♦ Side effects: limited to restlessness, dizziness,

headache, nausea, diarrhea, & paresthesia. Tardive dyskinesia is possible with long-term Buspirone therapy. Ethyl Alcohol (Ethanol)-causes a well-marked DIURESIS by inhibiting the production of ANTIDIURETIC HORMONE (ADH) or “vasopressin”. ADH decreases urine production by increasing water reabsorption by the renal tubules. Without ADH, there would be extreme water loss into the urine. • Ethyl alcohol DILATES blood vessels in the skin, depresses the CNS, and in blood levels > 400mg% usually causes coma and death. Alcohol euphoria results from removal of inhibitory activity of the cortex. • Recent evidence shows that frequent ingestion of moderate amounts of alcohol in any form (beer, wine, distilled spirits) reduces the risk of heart disease, especially in men. • Ethanol is a sedative; a hypnotic drug and the most important alcohol of pharmacologic interest. Ethanol abuse is responsible for many socioeconomic problems. Methanol and Ethylene Glycol are other alcohols. • Drugs that are synergistic with Ethanol: Diazepam, Meperidine, Pentobarbital, &

Chlorpromazine. When combined with alcohol, these drugs can cause fetal over sedation. *Synergism-the combined action of two or more drugs that is GREATER than the sum of their individual actions or that achieved with a single drug. Disulfiram (Antabuse)-used to manage ETHANOL ABUSE. Disulfiram is not a cure for alcoholism, but deters ethanol consumption. It is an antioxidant that interferes with the hepatic oxidation of the acetaldehyde metabolized from alcohol by inhibiting ALDEHYDE DEHYDROGENASE, a mitochondrial liver enzyme. • Even the ingestion of small amounts of ethanol results in high concentrations of acetaldehyde in the body. The unpleasant reaction that occurs “Disulfiram-Ethanol Reaction (DER)” consists of a throbbing headache, dyspnea, throbbing in the neck, nausea, copious vomiting, thirst, tachycardia, and hypotension. • Metronidazole also inhibits aldehyde dehydrogenase. BARBITURATES-are HYPNOTICS/SEDATIVES that depress neuronal activity by increasing membrane ion conductance (mainly chloride), reducing glutamateinduced depolarizations, & potentiating (prolonging)

the inhibitory effects of GABA. The primary pharmacologic effect of barbiturates is to DEPRESS NERVOUS TISSUE. They are metabolized in the liver, and possess serious drug dependence potential (can produce dependence). They do not possess analgesic properties. Barbiturates all have abuse and addiction potential, thus are controlled substances that require a DEA number. Barbiturates depress neuronal activity in the midbrain reticular formation, facilitating and prolonging the inhibitory effects of GABA and Glycine. Barbiturates use is determined by their DURATION OF ACTION: 1. Ultra-short acting Barbiturates: used IV to induce general anesthesia. The brief duration of general anesthetic action is due to the rapid rate of redistribution from the brain (CNS) to peripheral tissues. Ultra-short barbs will only maintain anesthesia while in the brain. Due to their high lipid solubility, they rapidly exit the brain to other tissues, causing the patient to awake within a few minutes after administration. For extensive procedures, ultra-short barbs are used to induce Stage III surgical anesthesia. For very brief procedures, they can be used alone. Length of hypnotic action: 5-20 minutes. These are the MOST LIPID SOLUBLE (so shortest duration of action). Thiopental (Pentothal), Thiamylal (Surital) &

Methohexital (Brevital)-ultra-short barbs only given by IV injection. Thiopental is the barbiturate that MOST readily penetrates the blood-brain barrier. Contraindications: porphyria, liver dysfunction (since they are metabolized in the liver), emphysema, & previous addiction to sedative hypnotic drugs. 2. Short-acting Barbiturates: used orally for their hypnotic, calming effect to treat INSOMNIA. Given preoperatively before a dental appointment to relieve anxiety. Length of hypnotic action: 1-3 hours. • Secobarbital (Seconal) & Pentobarbital (Nembutal). 3. Intermediate-acting Barbiturates: may also be prescribed to relieve anxiety before a dental appointment, although their side effects last longer than short-acting barbs. Also used for daytime sedation and to treat insomnia by suppressing REM sleep. Length of hypnotic action: 3-6 hours. • Amobarbital (Amytal) & Butabarbital (Butisol). 4. Long-acting Barbiturates: treats mainly daytime sedation & epilepsy (anti-convulsant). Duration of action is related to the lipid solubility, thus long-acting

barbs are the LEAST LIPID SOLUBLE (longest duration of action). Metabolized in the liver, these drugs produce serious drug dependence, but do not possess significant analgesic properties. Length of hypnotic action: 6-10 hours. • Phenobarbital (Luminal)-a barbiturate used as a sedative-hypnotic (CNS depressant). Not used in oral surgery. Largely replaced today by benzodiazepines because of less overdose potential. However, barbiturates are still used as anticonvulsants, as para-operative sedatives (ex. sodium thiopental), and analgesics for cluster headaches, migraines (ex. Fioricet). • Mephobarbital (Mebaral), & Primidone (Mysoline) • Barbiturates may decrease the half-lives of those drugs metabolized by the liver by inducing formation of the liver’s microsomal enzymes that metabolize drugs, which leads to an increased clearance of the affected drugs and a decrease in the drug’s effectiveness. Barbiturates exhibit a steeper dose-response relationship than benzodiazepines, and may precipitate acute porphyria in susceptible patients.

• Cause of death from ACUTE barbiturate poisoning or overdose is RESPIRATORY FAILURE. Other adverse reactions are CNS depression, euphoria, and habituation. The most important therapeutic measure taken in the event of barbiturate poisoning is to ASSURE ADEQUATE RESPIRATION. • Barbiturates are contraindicated in patients with respiratory disease or pregnant patients. • Physical dependence usually develops if barbiturates are abused. The dependence has a strong psychological and physical basis. Sudden withdrawal from high doses can be fatal.

CARDIOVASCULAR AGENTS ANTI-ARRHYTHMICS AGENTS-are classified according to Vaughan-Williams Classification System which places available anti-arrhythmic agents into 4 groups denoted by Roman numerals (I, II, III, IV). This system is loosely based on the channel or receptor that is involved. Most drugs that treat cardiac arrhythmias act primarily by INCREASING the refractory period of cardiac muscle. I. Sodium Channel Blockers: are further classified based on their effects on “action potential duration”: • Group IA: Quinidine, Procainamide, Amiodarone, & Disopyramide PROLONG action potential. • Procainamide-a potent Group IA antiarrhythmic agent used to treat severe cardiac arrhythmias (atrial fibrillation, atrial flutter, paroxysmal atrial tachycardia, and ventricular tachycardia). It is a derivative of the ester local anesthetic procaine (novocaine). Its properties are similar to other Group IA agents which decrease myocardial conduction velocity, excitability, and contractility by inhibiting Na+ influx through “fast” channels of the myocardial cell membrane, thus increasing the recovery

period after repolarization. • Quinidine-primary used to treat supraventricular tachyarrhythmias, but is contraindicated in some arrhythmias, especially those associated with heart block. • Group IB: Lidocaine, Mexiletine, & Tocainide SHORTEN action potential. • Group IC: Flecainide, Moricizine, & Propafenone do not effect action potential duration. • Lidocaine (Xylocaine): a sodium-channel blocker ANTI-ARRHYTHMIC AGENT effective ONLY on the ventricle, often administered intravenously to treat life-threatening ventricular arrhythmias. When given IV to treat ventricular arrhythmias, it acts on the fibrillating ventricles to decrease cardiac excitability and spares the atria. It can effectively reverse a life-threatening situation. • Quinidine-a sodium-channel blocker and PROTOTYPE anti-arrhythmic agent used mainly to treat ATRIAL FIBRILLATION. It is not effective for treating life-threatening ventricular fibrillation. • Disopyramide (Norpace)-a newer anti-arrhythmic agent (sodium-channel blocker) used to convert atrial arrhythmias into normal sinus rhythm.

• Propafenone (Rythmol)-another newer sodiumchannel blocker to treat ventricular arrhythmias & supraventricular tachycardias. II. Beta-Adrengeric Blockers: Propranolol & Esmolol are prototype anti-arrhythmic beta-blockers. Beta-blockers (Group II agents) as anti-arrhythmics are reserved for patients who only require control of ventricular rate during atrial tachyarrhythmias, or who have mildly symptomatic ventricular arrhythmias. Side effects: bradycardia & hypotension. III. Potassium Channel Blockers: Amiodarone (Cordarone)-the prototype Group III agent that is considered a Group III agent even though it also blocks Na+ channels (Group I action). Amiodarone is unique because it is the most potent and “broad-spectrum” anti-arrhythmic compound available. It blocks Na+, Ca2+, K+ channels, and beta receptors. It has impressive efficacy in suppressing supraventricular and ventricular arrhythmias. IV. Calcium Channel Blockers: Verapamil is the prototype Diltiazem & Nifedipine are also calcium channel blockers useful as anti-anginal agents to treat chronic stable angina pectoris by blocking calcium entry through the membranous calcium ion channels of cardiac and vascular smooth muscle to cause two effects: 1. peripheral arterioles dilate (vasodilators), and total

peripheral resistance (TPR) decreases, reducing after-load and reducing myocardial oxygen requirements. 2. increases oxygen delivery to the myocardium. • Verapamil-prototypical Group IV (calcium channel blocker) anti-arrhythmic agent that inhibits intracellular entry of calcium through the slow channels of calcium-dependent myocardium tissues that are concentrated in the heart’s SA & AV nodes. Of the calcium ion channel antagonists, only Verapamil and Diltiazem possess significant antiarrhythmic activity. Drug of choice to suppress paroxysmal supraventricular tachycardias stemming from the AV node, characterized by rapid cardiac rate of 160-190 beats/minute. Useful to treat angina, hypertension, and supraventricular tachyarrhythmias. • Calcium Channel Blockers have been associated with causing GINGIVAL HYPERPLASIA. ANTI-HYPERTENSIVE AGENTS-lower BP by reducing total peripheral resistance (TPR) & cardiac output (CO) via various mechanisms. 4 anti-hypertensives classified by their mechanisms of action: 1. Diuretics-inhibit Na+ reabsorption in the renal tubular cells in the kidney to cause excess Na+ and urinary

excretion, causing reduced blood volume. 2. Beta-Adrenergic Receptor Blockers (Beta Blockers)reduce the volume of cardiac output into the circulation, causing reduced peripheral pressure. There are 2 types of Beta Blockers: • Cardioselective Beta Blockers-block beta 1 receptors in the heart muscle, such as Atenolol (Tenormin) and Metoprolol (Lopressor & Toprol XL). • Non-Cardioselective Beta Blockers-include Nadolol (Corgard) and Propranolol (Inderal). 3. Angiotensin-Converting Enzyme Inhibitors (ACE Inhibitors)-include Lisinopril (Prinivil; Zestril), and Enalapril (Vasotec). ACE inhibitors, “inhibit” the conversion of inactive Angiotensin I→Angiotensin II (a vasoconstrictor). This causes peripheral vasodilation and secondarily increases urinary volume excretion. Both actions cause reduced BP. 4. Calcium Channel Blockers-include Amlodipine (Norvasc), Diltiazem (Cardizem), and Nifedipine (Procardia). These drugs inhibit calcium from entering into vascular smooth muscle, to cause vasodilation of the coronary and peripheral blood vessels, causing lower BP.

Miscellaneous Anti-Arrhythmic Drugs: Adenosine & Digoxin (Lanoxin/Digitalis): DIGOXIN (LANOXIN)-the most versatile and widely used cardiac glycoside (anti-arrhythmic) to treat ATRIAL FIBRILLATION & FLUTTER, by causing a positive inotropic effect (stronger heart beat) to directly increase the force of myocardial contractions (increased contractile strength). • Digoxin Indications: supraventricular arrhythmias, atrial fibrillation and flutter, cardiogenic shock, and congestive heart failure (CHF). Digoxin is no longer the first choice for CHF because it may increase the risk of death, but it is still sometimes used for certain patients. BETA BLOCKERS ARE THE FIRST CHOICE FOR CONGESTIVE HEART FAILURE (CHF) BEFORE USING DIGOXIN. • Cardiac Glycosides Mechanism: INHIBITS the Na+/K+ ATPase membrane pump in the heart to increase intracellular sodium levels to reverse the sodium-calcium exchanger. This ion reversal increases intracellular calcium causing decreased heart rate and increased heart contractility without increasing the energy the heart expends. • When combined with sympathomimetics, Digoxin can cause cardiac arrhythmias so anesthetics

containing EPI (vasoconstrictor) should be used cautiously. • Digoxin Adverse Effects: appetite loss, nausea, vomiting, and diarrhea. Increased gag reflex and salivation. Tetracycline and erythromycin increase digoxin levels in approximately 10% of patients. Does NOT cause gingival hyperplasia. • Contraindications: ventricular fibrillation & ventricular tachycardia. ANGINA PECTORIS-pain in the heart & chest that occurs during occlusion of the coronary arteries, triggered by physical exertion, increased BP, & vasoconstriction. ANTIANGINAL DRUGS: Prevent or Provide Relief of Angina Pectoris: 1. Nitroglycerin/Nitrostat (Nitrates)-a coronary artery vasodilator that relaxes blood vessels to provide increased blood flow and oxygenation to heart muscle. If administered SUBLINGUALLY, it is effective within 24min. The skin patch releases the drug over a 12-hour period to provide sustained blood levels to prevent angina. It is the single most effective anti-anginal agent available to manage acute angina episodes. It is fast-acting and relieves ACUTE ANGINAL ATTACKS in the dental office. For angina, Nitroglycerin & Nifedipine are usually used BEFORE Verapamil.

2. Amyl Nitrite (Nitrites)-a vasodilator and highly volatile substance only administered by INHALATION. Used in the emergency treatment of CYANIDE poisoning because it OXIDIZES HEMOGLOBIN into methemoglobin which binds cyanide tightly to keep it in the peripheral circulation and prevent it from accessing tissues. It is the most rapidly acting anti-anginal drug that produces effects within 10 seconds, but its duration of action is only 3-5min. Due to its extreme potency, there are uncomfortable side effects that invariably occur with its use (fainting and pounding headache). Amyl nitrite is rarely prescribed, and is NOT the drug of choice to treat angina. It is abused to produce euphoria and as a sexual stimulant. Adverse Effects: orthostatic hypotension & headache. 3. Dipyridamole (Persantine)-a non-nitrate vasodilator that treats angina (anti-anginal). 4. Verampamil (Isoptin), Nifedipine (Procardia) & Diltiazem (Cardizem)-calcium channel blockers that prevent angina attacks by dilating coronary blood vessels to improve blood flow to the heart muscle. Ca2+ channel blockers are “indirect vasodilators” given ORALLY to treat mild-to-moderate hypertension. 5. Propranolol (Inderal), Nadolol (Corgard), & Atenolol (Tenormin)-beta adrenergic blockers that prevents angina pectoris attacks by decreasing the heart’s work

load so that less oxygen is required. Direct Vasodilators-exert anti-hypertensive effects (treat hypertension) by causing a direct vasodilator action on the SMOOTH MUSCLE of arterioles, to produce a decrease in peripheral resistance & BP. Compensatory responses may be marked such as salt retention and tachycardia. Adverse Effects: GI upset, headache, dizziness, and tachycardia. 1. Hydralazine (Apresoline)-a direct vasodilator that treats hypertension. 2. Minoxidil (Loniten)-extremely efficacious direct vasodilator, thus is reserved for SEVERE HYPERTENSION. 3. Diazoxide (Hyperstat)-parenteral vasodilator used in hypertensive emergencies. 4. Sodium Nitroprusside (Nipride)-parenteral vasodilator used in hypertensive emergencies. 5. Nitroglycerin (Nitrostat)-a direct vasodilator that treats angina pectoris and hypertension. ACE Inhibitors & Angiotensin II Receptor Blockers: indirectly inhibit fluid volume increases when interfering with Angiotensin II because Angiotensin II stimulates the release of ALDOSTERONE which promotes Na+ and H2O retention. • Angiotensin-Converting Enzyme (ACE) Inhibitors: interfere with the conversion of Angiotensin I

(weak vasoconstrictor) into Angiotensin II (highly effective vasoconstrictor that simulates the release of Aldosterone) by inhibiting the ACE enzyme. ACE inhibitors drugs treat hypertension and congestive heart failure (CHF). Adverse Effects: cough, hypotension, neutropenia, anorexia, and polyuria. • Captopril (Capoten)-an angiotensin-converting enzyme inhibitor (ACE inhibitor). • Benazepril (Lotensin), Enalapril (Vasotec), Fosinopril (Monopril), Lisinopril (Zestril), Perindopril (Aceon), Quinapril (Accupril), Ramipril (Altace) • Angiotensin II Receptor Blockers-a new class of BP medications that INHIBIT Angiotensin II at its receptor site to prevent ANG-2 from constricting blood vessels that raise BP. Losartan, Valsartan, Candesartan, & Irbesartan. RENIN-a proteolytic enzyme produced by and stored in the juxtaglomerular apparatus in the kidney that surrounds each arteriole as it enters the glomerulus. Renin is released into the bloodstream where it has an important role in ANGIOTENSIN FORMATION. Renin acts on the precursor substance angiotensinogen which is manufactured by the liver and is present in the blood. Renin

converts Angiotensinogen→Angiotensin I. Then Angiotensin I is converted into Angiotensin II by Angiotensin Converting Enzyme (ACE) which is associated with the capillary walls in the lungs. Angiotensin II-a potent vasopressor that increases total peripheral resistance (TPR), and stimulates aldosterone release to cause an increase in plasma volume, venous return, stroke volume, and ultimately an INCREASE IN CARDIAC OUTPUT. DIURETICS-antihypertensive agents used to treat congestive heart failure by relieving edema and symptoms of dyspnea arising from pulmonary congestion. Diuretics also treat hypertension and manage edema associated with hepatic or renal disease. There are 4 categories of widely used diuretics: 1. Potassium (K+)-Sparing Diuretics-cause ↑Na+ & ↓K+ concentrations at the end of the distal convoluted tubules. Potassium sparing diuretics act in the collecting tubule to inhibit Na+ reabsorption, K+ secretion, & H+ secretion. HYPERKALEMIA is the most important toxic effect of potassium-sparing diuretics. Potassium-sparing diuretics conserve K+ while causing diuresis. Thus, no potassium is lost from the body as with other diuretics (i.e. Thiazides & Loops). 2 Types of Potassium-Sparing Diuretics:

1. Aldosterone Antagonists in Collecting Tubules: • Spironolactone (Aldactone)-a pharmacologic antagonist of aldosterone in the collecting tubule. Treats primary aldosteronism & heart failure since hyperaldosteronism is common in heart failure patients. Spironolactone competes with aldosterone receptor sites in the renal tubules causing increased secretion of Na+, Cl, and H2O, while conserving K+. 2. Sodium Channel Blockers in Collecting Tubules: • Amiloride (Midamor)-blocks Na2+ channels in luminal membrane of principal cells in the late distal tubule & collecting duct which decreases K+ excretion. It inhibits Na+ transport through ion channels in the luminal membrane causing a decrease in Na+K+ exchange. • Triamterene (Dyrenium)-K+ sparing diuretic that promotes Na+ & H2O excretion, but maintains K+. 2. Thiazides Diuretics-inhibit sodium reabsorption in the distal portion of the renal tubule within the kidney causing increased excretion of Na+ & H2O (affects salt transport in the distal convoluted tubule). Thiazide diuretics are the most widely used diuretic drugs

used to treat hypertension, but may require supplemental administration of K+ (potassium). Inhibits Na+/Cl- symport. • Dyazide-brand name for the combination of Triamterene + Hydrochlorothiazide (HCTZ). Dyazide combines the K+-sparing diuretics with HCTZ for greater efficacy than either drug individually. • Metolazone-oral quinazoline & sulfonamide diuretic to manage edema & hypertension; not a true thiazide. Produces diuresis in patients with impaired renal function. More potent than thiazides causing Na+ excretion in advanced renal failure unlike thiazides. • Indapamide-an oral antihypertensive & diuretic and first of a new class of antihypertensive/diuretics (indolines). Fat-soluble, non-thiazide diuretic with 1022hr ½ life. Used in advanced renal failure to stimulate additional diuresis in conjunction w/ loop diuretics. Metabolized & excreted by GI tract and kidneys, and is less likely to accumulate in patients with renal failure, and may be useful in their treatment. • Other thiazide diuretics: Chlorothiazide, Cyclothiazide, Polythiazide, Benzthiazide. • Sulfonamide derivatives structurally related to carbonic anhydrase inhibitors.

• All are secreted by the organic acid secretory system in the kidney. • Major Actions: • Inhibits sodium chloride (NaCl) resorption from luminal side of epithelial cells by inhibiting Na+/Cl co-transporter/symporter on the membrane. • Inhibits NaCl transport in early segment of distal convoluted tubule. • Increases concentration of Na+ & Cl- in tubular fluid. • Thiazides Treat: 1. Hypertension (anti-hypertensive drugs) by decreasing systolic & diastolic BP. 2. Edema of CHF decreasing extracellular volume. 3. Renal Edema (nephrotic syndrome) if loop diuretics fail. 4. Hypercalciuria (increase uric acid in urine-inhibit urinary Ca2+ excretion). Treats patients w/ calcium oxalate stones in urinary tract (calcium nephrolithiasis). 5. Nephrogenic Diabetes Insipidus (lack of responsiveness to ADH)-thiazides produce hyperosmolar urine due to volume contraction causing increased reabsorption at proximal tubule. Thiazides treat this by substituting ADH.

• Thiazide Adverse Effects (CNS, hematological, sexual, GI, dermatological disorders):☻ 1. Hypokalemia (K+ deficiency) is the most common problem with thiazide diuretics & can predispose patients on digitalis to ventricular arrhythmias. Treat with Spironolactone to maintain K+ levels. 2. Hyperuricemia (increase serum uric acid) by decreasing acid excreted. Can cause gout (uric acid deposits in joints). 3. Hypercalcemia (inhibits Ca2+ secretion causing increased Ca2+ in blood). 3. Loop (High-Ceiling) Diuretics-inhibit Na+K+2Cl- symport or cotransport. Inhibit reabsorption of Na+ and chloride in the ascending Loop of Henle to cause increased secretion of H2O, Na+, & Cl. Furosemide (Lasix) is the prototype loop diuretic. Other loop diuretics: Bumetanide, Torsemide, & Ethacrynic acid • Mechanism of Action: increase Ca2+ content of urine, while thiazide diuretics decrease Ca2+ concentration of urine. o Cause decreased renal vascular resistance and increased renal blood flow. o Acutely loop diuretics increase excretion of uric acid, where as chronic admin causes

reduced uric acid excretion, and increases renal blood flow. • Therapeutic uses: acute pulmonary edema of CHF, loop diuretics are DRUG OF CHOICE. Useful in emergencies due to their rapid onset of action. Treats hypercalcemia by stimulating tubular Ca2+ secretion, and treat hypertension when other diuretics and antihypertensives do not produce a satisfactory response.☻ • Adverse Effects: OTOTOXICITY as tinnitus, hearing impairment, deafness, vertigo, & sense of ear fullness. o Loop diuretics are toxic to the ear especially when used with aminoglycosides (antibiotics). o ETHACRYNIC is the most ototoxic. o Irreversible damage may result with continued treatment, and vestibular function is less likely to be distributed, but it too may be affected by prolonged combined treatment. o Hyperuricemia, acute hypovolemia (reduced blood volume), increase plasma LDL cholesterol and triglycerides, and decrease HDL.

o Skin rashes, photosensitivity, paresthesias, K+ depletion, & bone marrow depression. 4. Osmotic Diuretics-highly filtered by the glomerulus and exerts a solute-induced diuresis in the proximal tubule. Osmotic diuretics are used to reduce excess edema associated with neurosurgery or trauma to the CNS. Mannitol (Osmitrol), Glycerin (Glyrol), Isosorbide (Ismotic), & Urea (Ureaphil). Must be given via injection. • Mannitol-treats cerebral edema, increases delivery of Na2+ & water out of loop of Henle. Mannitol is NOT absorbed orally so it MUST be given intravenously. • Osmotic diuretics are not useful in treating conditions where Na+ retention occurs, but are used to maintain urine flow after acute toxic ingestion of substances capable of producing acute renal failure. • Osmotic diuretics treat patients with increased intracranial pressure, or acute renal failure due to shock, drug toxicities, and trauma. CORTICOSTEROIDS (STEROIDS)-steroid hormones produced by the ADRENAL CORTEX that DO NOT

CURE any disease (they represent replacement only Addison’s Disease). Corticosteroids treat asthma, arthritis (if patient has peptic ulcer disease, do not use corticosteroids for arthritis), allergic reactions, Addison’s Disease, Lupus Erythematosus, Aphthous Stomatitis, and TMJ pain. Corticosteroids consist of two major groups: 1. Mineralcorticoids-regulate Na+ and K+ reabsorption (metabolism) in the kidney’s collecting tubules. • Aldosterone-the MAJOR natural mineralcorticoid in humans, secreted by cells in zona glomerulosa of the adrenal cortex. Aldosterone secretion is regulated by ACTH and the renin-angiotensin system, and is very important in regulating blood volume & blood pressure, and promotes Na+ reabsorption into the blood from the glomerular filtrate. K+ is lost in urine because of the electronegativity created by the reabsorption of Na+ in the kidney tubules. Increased blood aldosterone levels result in high sodium and low potassium levels in the plasma. Decreased Na+ concentration causes the juxtaglomerular cells of the kidneys to

secrete RENIN which converts angiotensinogen→angiotensin I. Angiotensin I is converted into Angtiotensin II which then stimulates the adrenal cortex to release aldosterone. Addison’s Disease-caused by HYPOSECRETION of aldosterone & cortisol. For adrenal crisis, treat with 2ml of cortisol (hydrocortisone). Corticosteroids represent replacement only in Addison’s Disease. ADH (vasopressin)-decreases urine production by increasing water reabsorption by the renal tubules (it increases the permeability of the collecting ducts & distal tubules). Without ADH, there would be extreme water loss into the urine. At high concentrations, ADH causes arterioles to constrict (increases BP). Other Mineralocorticoids: Deoxycorticosterone & Fludrocortisone. Pharmacological Effects of Mineralocorticoids: increase in Na+ retention, and decrease K+ that can cause edema and hypertension. If excessive, it may lead to dehydration. If insufficient, may lead to hypotension.

2. Glucocorticoids-have important effects on carbohydrate, lipid, and protein metabolism, catabolism, immune responses, and inflammation (used as anti-inflammatory agents). The majority of anti-inflammatory and immunosuppressive actions of the glucocorticoids are the result of their action on ARACHIDONIC ACID METABOLISM. They induce the synthesis of a protein that inhibits phospholipase A2, which decreases production of prostaglandins and leukotrienes. • Cortisol (Hydrocortisone)-the MAJOR glucocorticoid that treats Addison’s Disease. • Synthetic Glucocorticoids: Prednisone (Deltasone) & Prednisolone (Prelone), Methylprednisolone, Dexamethasone (Decardon) & Triamcinolone (Atolone). • Beclomethasone, Budesonide, & Flunisolide: special glucocorticoids (INHALERS) developed to treat chronic asthma and bronchial disease by readily penetrating the airway mucosa, but have very short half-lives after they enter the blood so systemic effects and toxicity are greatly reduced. • Fluticasone (Flonase or Flovent)-a corticosteroid administered by inhalation to treat ASTHMA by decreasing inflammation in the airway of

asthmatics. This reduced inflammation enhances bronchodilating effects of the β2 adrenergic agonists. ♦ Other inhaled (aerosol) corticosteroids that treat asthma: Triamcinolone (Azmacort), Beclomethasone (Beconase), & Budesonide (Pulmicort). ♦ Inhaled steroids very often cause fungal infections of the mouth & throat. Localized infections with Candida albicans occur frequently in the mouth and pharynx with repetitive use of inhalant corticosteroids. ♦ Inhaled corticosteroids used for asthma, and nasal spray corticosteroids used for seasonal allergies do not achieve significant blood levels to cause the adverse effects caused by oral corticosteroids, so are used for their localized effects. Popular nasal spray corticosteroid products are Triamcinolone (Nasocort), Fluticasone (Flonase), and Budesonide (Rhinocort). Pharmacological Effects of Glucocorticoids: • Decrease in glucose utilization and inhibits protein synthesis. • Increases protein catabolism and gluconeogenesis (by breaking down endogenous proteins to amino acids, that are then converted to glucose for storage in the liver or for use by the body.

• Impaired wound healing and decreased resistance to infection. • Anti-inflammatory action, immunosuppression, and anti-allergenic action. These effects occur in target cells after the interaction of the steroid with a specific glucocorticoid receptor. Corticosteroid Contraindications: latent TB, viral (herpes), or fungal infections, AIDS, and patients with peptic ulcer disease (specifically gastric ulcers), and congestive heart failure. Corticosteroids may cause PEPTIC ULCERS. Corticosteroid Toxic Effects: growth inhibition, hyperglycemia, osteoporosis, psychosis, and salt retention. Corticosteroid Adverse Reactions: Cushing’s Syndrome (obesity and weakening of muscles), hyperglycemia, osteoporosis, peptic ulcers, and increased risk of infection. HMG-CoA Reductase Inhibitors (“Statin” Drugs)-a class of drugs that LOWERS BLOOD CHOLESTEROL levels by inhibiting a key enzyme in the cholesterol synthesis pathway in the liver. HMG-CoA Reductase is an enzyme called “hydroxyl methylglutaryl coenzyme A reductase which is necessary in the key step to synthesize cholesterol. When the “statin” drugs inhibit this enzyme, cholesterol is not produced in the liver, which decreases blood cholesterol levels.

“Statin” Drugs: Atorvastatin (Lipitor), Simvastatin (Zocor), Fluvastatin (Lescol), Lovastatin (Mevacor), & Pravastatin (Pravachol). Coronary Artery Disease (CAD)-a condition of narrowing of the heart’s blood vessels, restricting oxygen flow to the heart muscle. CAD has been correlated with the levels of blood cholesterol and triglycerides. If not treated, CAD can lead to myocardial infarction (heart attack). “Statin” drugs which lower blood cholesterol, are also effective in minimizing the treat of CAD. “Statin” drugs have the capability to increase the breakdown of skeletal muscle, thus releasing muscle protein. If skeletal muscle protein overloads the kidneys, renal failure can result. The erythromycin drugs (Macrolides) enhance the capabilities of the “statins” to cause this effect. It is advised that patients medicated with a “statin” drug, do not take erythromycin (macrolide) products.

ANTI-DEPRESSANTS 5 Major Categories of Antidepressant Drugs: 1. Tricyclic Antidepressants 2. Monoamine Oxidase Inhibitors 3. Selective Serotonin Reuptake Inhibitors 4. Serotonin & Norepinephrine Reuptake Inhibitors 5. Second Generation Miscellaneous Amitriptyline (Elavil)-the most widely used TRICYCLIC ANTI-DEPRESSANT to treat unipolar disorder (depression). Elavil has the greatest anti-cholinergic effects, while Desipramine (Norpramin) has the least anticholinergic effects. • Tricyclic antidepressants are the drugs of FIRST choice to treat UNIPOLAR DISORDER (depression). Tricyclics inhibit neuronal reuptake of NE & Serotonin (5-HT) in the brain, which results in potentiation of their neurotransmitter actions at postsynaptic receptors. • The most common CNS adverse reaction is DROWSINESS. Anti-cholinergic adverse effects are dry mouth (xerostomia), constipation, blurred vision, and tachycardia.

• Tricyclic antidepressants (i.e. Elavil), Serotonin & NE reuptake inhibitors (i.e. Effexor) are antidepressants with the highest incidence of dry mouth (xerostomia). These drugs induce significant xerostomia in nearly 75% of patients taking these medications due to their secondary anti-cholinergic nature. Drug-induced xerostomia must be treated palliatively with artificial salivary substitutes. • EPI (vasoconstrictor) in local anesthetic injections must be used cautiously in patients taking tricyclic antidepressants (i.e. Elavil), Serotonin & NE reuptake inhibitors (i.e. Effexor) to avoid transient and significant increases in blood pressure. These antidepressants greatly increase NE levels in tissues. In the presence of a vasoconstrictor administered via a local anesthetic injection, the patient can experience a significant elevation of blood pressure due to the vasopressor actions of the combination. • Other Tricyclic Antidepressants: Doxepin (Sinequan) & Imipramine (Tofranil). Monoamine Oxidase (MAO) Inhibitors-treat depression (but rarely used today due their dangerous interaction with some analgesics, SSRIs, and foods containing tyrosine). MAOs antagonize the action of monoamine oxidase (MAO) which is responsible for degrading the

naturally occurring monoamines (EPI, NE, Dopamine, & Serotonin). Increased levels of monoamines in the brain is responsible for the anti-depressant effect of MAO inhibitors. • MAO inhibitors: Isocarboxazid (Marplan), Tranylcypromine (Parnate), & Phenelzine (Nardil). • Local anesthetics containing EPI are contraindicated in patients taking MAO inhibitors. • The major limitation for the widespread use of MAO inhibitors to treat depression is the potential for serious side effects. MAO inhibitors interact with many drugs and foods containing large amounts of the amino acid TYRAMINE. Drugs that interact with MAO inhibitors are Meperidine (Demerol), EPI, and Ephedrine. LITHIUM-current drug of choice to treat the MANIC PHASE of bipolar disorder (“manic-depressive syndrome”). Bipolar disorder (“manic-depressive syndrome) is characterized by cyclical changes in affective state between the manic & depressive phases of behavior. Bipolar patients cycle between the two affected states. • Lithium is used mainly to suppress recurrences of the manic phase of bipolar disorder. It is NOT useful to treat acute manic episodes. Lithium can

prevent the occurrence of the depressive and manic episodes in some, but not all patients. Antidepressants are often administered with Lithium to manage the depressive phase of bipolar disorder, if lithium alone is insufficient. Neuroleptic Agents (Anti-Psychotic Agents or Major Tranquilizers)-treat ACUTE manic episodes. Chlorpromazine (a phenothiazine), & Haloperidol (acts similar to a phenothiazine) are effective in quelling extreme mania and psychotic behavior. Selective Serotonin Reuptake Inhibitors (SSRIs): have revolutionized the treatment of depression. The most important clinical distinction of SSRIs from all other antidepressants is SSRI’s very high specificity for blocking the reuptake of serotonin. These drugs increase the activity of serotonergic systems in the brain: 1. Fluoxetine (Prozac)- prototype SSRI with the LONGEST half-life. Treats obsessive-compulsive disorders (OCD) and depression. 2. Paroxetine (Paxil), Sertraline (Zoloft), & Fluvoxamine (Luvox)-have short half-lives, and can be given once daily. These drugs are also effective for treating PANIC ATTACKS, depression, and obsessive-compulsive disorders.

3. Citalopram (Celexa) & Escitalopram (Lexapro)-treats depression and anxiety. Adverse Effects: nausea, headache, anxiety, agitation, insomnia, and sexual dysfunction. SSRIs do not have secondary anti-cholinergic effects, thus do not cause any significant dry mouth. Selective Serotonin Reuptake Inhibitors (i.e. Prozac) do not have an effect on NE in tissues and interaction with a vasoconstrictor like EPI is not a problem. Serotonin & NE Reuptake Inhibitors: Venlafaxine (Effexor), Nortriptyline (Pamelor), & Desipramine (Norpramin). Second Generation Miscellaneous: Bupropion (Wellbutrin), Trazodone (Desyrel), Nefazodone (Serzone), Mirtazipine (Remeron).

ANTI-PSYCHOTICS ANTIPSYCHOTIC DRUGS-primarily indicated to treat “PSYCHOSIS” associated with SCHIZOPHRENIA, PARANOIA, & MANIC symptoms of manic-depressive illness. 1. Phenothiazines-the most widely used and most effective group of antipsychotic agents in medicine that have antipsychotic effects (improve mood & behavior) and neuroleptic effects (emotional quieting and development of extrapyramidal symptoms). These effects are due the phenothiazine’s ability to BLOCK DOPAMINERGIC SITES in the brain. Phenothiazines are the most effective antiemetic agents that depress the chemoreceptor trigger zone (CTZ) to reduce nausea and/or vomiting. • Phenothiazines: Chlorpromazine (Thorazine), Thioridazine (Mellaril), Promazine (Sparine), Fluphenazine HCL (Prolixin), Mesoridazine (Serentil), Triflupromazine (Vesprin), Acetophenazine (Tindal), Trifluoperazine (Stelazine). These act by blocking dopaminergic sites in the brain. • Phenothiazines pharmacologic actions are

sedation, anti-emetic activity, alpha adrenergic receptor blockage, and potentiate the action of narcotics. It is NOT an anti-convulsant. • Phenothiazine Contraindications: patients with severe CNS depression or epilepsy, and given with caution in patients with liver disease. • Phenothiazine Adverse effects: hypotension, liver toxicity (jaundice), dry mouth (xerostomia), & extrapyramidal reactions (tardive dyskinesia). Tardive dyskinesia-an extrapyramidal disease & irreversible side effect of long-term phenothiazine therapy due to the effect of the basal ganglia. Characterized by involuntary, repetitious movements of the face, limbs, and trunk. • Tardive Dyskinesia-a serious, irreversible neurological disorder that can appear at any age. It is a side effect of antipsychotic/neuroleptic drugs (i.e. phenothiazine). Symptoms can be barely noticeable or profound. Symptoms are uncontrollable movement of various body parts (body trunk, legs, arms, fingers, mouth, lips, or tongue). Once any TD symptoms appear, the antipsychotic/neuroleptic must be discontinued. 20% of people taking antipsychotics/neuroleptic drugs for more than 1 year are affected.

• Muscle spasms of oral-facial region is an extrapyramidal syndrome (EPS) caused by Phenothiazine-type antipsychotics. EPS is a variety of signs and symptoms caused by blockade of dopamine receptors in specific brain regions. EPS symptoms include Parkinson-like movements (shuffled gait, pill-rolling effect of fingers), muscle rigidity, spasms of the neck and facial muscles, tremors, and loss of muscle movement. • Use caution when administering any sedative drugs to a patient taking Phenothiazines (i.e. Chorpromazine, Fluphenazine, Prochlorperazine). These drugs will potentiate the action of sedative drugs. 2. Thioxanthenes-Chlorprothixene & Thiothixene are a less potent group of antipsychotics that treat Schizophrenia. 3. Butyrophenones-Haloperidol & Droperidol are highly effective antipsychotic drugs (potent dopamine antagonists) that treat Schizophrenia and Tourette’s Syndrome. 4. Diverse Heterocyclic Antipsychotics: newer agents that antagonize dopamine and serotonin in the brain. Molindone (Moban), Clozapine (Clozaril), Loxapine (Loxitane), Olanzapine (Zyprexa), Resperidone

(Resperdal), & Quetiapine (Seroquel). These drugs effectively treat Schizophrenia and are more effective & less toxic than the older drugs (i.e. Phenothiazines, Butyrophenones, & Thioxanthenes), but are much more expensive.

ANTI-HISTAMINES Antihistamines-antagonizing agents that compete with natural histamine for H1 & H2 receptor sites. Histamine is found in all tissues. It is stored in the cytoplasmic granules of tissue mast cells and blood basophils. Histamine is released in response to IgE-mediated (immediate) allergic and inflammatory reactions to produce a variety of physiologic actions in many tissues. Histamine plays an important role in hay fever, urticaria, angioneurotic edema, and in controlling stomach acid secretion (HCL). There are 2 types of histamine antagonists/receptors: 1. H1 receptor antagonists: competitively block H1 receptors, blocking the effects of histamine at these receptors. H1 antagonists block vasodilation, block bronchi constriction, and block capillary permeability that histamine usually causes. Blocking histamine’s effects overcomes the symptoms of seasonal allergies. H1 plays an extremely important role in allergic reactions. Antihistamines located at the H1 histamine receptor site: • 1st-Generation (Classic) H1 Blockers: Diphenhydramine (Benadryl is the most common), Chlorpheniramine (Chlor-Trimeton), & Tripelennamine

(Pelamine). These agents have a “broad spectrum of action” that includes antihistaminic, anticholinergic, antiserotonergic, antibradykinin, and sedative properties. • 2nd Generation H1 Blockers: Cetirizine (Zyrtec), Fexofenadine (Allegra), Loratidine (Claratin), & Desloratidine (Clarinex). Most of these agents due to their poor CNS penetration, cause LESS sedation and drowsiness than 1st generation agents. • All H1 blockers do not prevent histamine release, but COMPETE with free histamine from binding at H1 receptor sites. Binding is usually competitive, but some 2nd-generation agents bind non-competitively at higher doses. • Common Adverse Effects: drowsiness, dizziness, anticholinergic effects (dry mouth, nose & throat), nausea. Both H1 & H2 antagonists can stimulate and depress the CNS. 2. H2 receptor antagonists: important in gastric acid (HCL) secretion by competitively blocking H2 receptors to block histamine’s effects at H2 receptors (these antagonists compete with histamine ONLY at H2 receptors in the GI tract (however, H2 receptors are also located in vascular & bronchial smooth muscle). H2 antihistamines block stomach acid secretions and

treat duodenal ulcers by inhibiting histamine at parietal cells to interfere with one of several mediators for signaling parietal cells to secrete acid. H2 blockers reversible competitive antagonists of the actions of histamine on H2 receptors. They are orally active, and relatively non-toxic, thus can be given in large doses so the duration of action of a single dose may last 12-24hrs. These agents treat acid-peptic disease (especially duodenal ulcers and gastric peptic ulcers). H2 antagonists also treat Zollinger-Ellison Syndrome (hypersecretory disease), and gastroesophageal reflux disease (GERD). However, for both of these disorders, Omeprazole (Prilosec) which a “protonpump” inhibitor is more effective. • Cimetidine (Tagamet)-an antihistamine H2 receptor blocker that is more likely to provoke interactions with hepatically metabolized drugs (a potent inhibitor of hepatic drug-metabolizing enzymes). May cause gynecomastia (male breasts), however, Zantac, Pepcid, & Axid do not produce these adverse effect. • Ranitidine (Zantac)-an H2 receptor blocker that treats GERD (heartburn) by blocking the effects of histamine at H2 receptors. • Famotidine (Pepcid) & Nizatidine (Axid)-an H2 receptor blocker that blocks the effects of histamine at H2 receptors.

Omeprazole (Prilosec) & Lansoprazole (Prevacid)gastrointestinal drugs and proton-pump inhibitors (H+/K+ ATPase) that reduce stomach acid formation by inhibiting the proton-pump of the stomach’s parietal cells. These drugs inhibit the pump so that no H+ protons are pumped into the stomach contents, thus no HCL is produced. • Hydrochloric acid (HCL) is produced by the stomach’s parietal cells through a pump within each cell that pumps H+ protons into the stomach contents. This pump is the H+/K+ ATPase pump. HCL is used for food digestion, but an abundance can cause heart burn and acid indigestion. • Stomach acid is also reduced by inhibiting the effects of histamine in the stomach at the histamine Type-2 receptors (H2 receptors). Ordinarily, histamine stimulates gastric parietal cells to produce HCL. • Proton-pump inhibitors and H2 blockers are used to treat heartburn, indigestion, sour stomach, active duodenal ulcer disease, and gastroesophageal reflux disease (GERD). Drugs that Reduce GI Motility to treat DIARRHEA (Antidiarrheals): 1. Loperamide (Imodium)-an anti-diarrheal that acts on intestinal muscles to inhibit peristalsis. It is an

OPIOID, that does not penetrate the CNS like other opioids, thus is sold over-the-counter (OTC). Imodium has no evidence of drug abuse or dependence (unlike other opioids like Codeine, Morphine, & Meperidine). 2. Diphenoxylate with Atropine (Lomotil)-an antidiarrheal that inhibits excessive GI tract motility and propulsion. Commercial preparations contain a subtherapeutic amount of Atropine to discourage abuse. Lomotil requires a prescription, thus is not sold OTC. Important: Laxatives act in the REVERSE manner of anti-diarrheals by increasing GI tract motility to treat constipation. Laxatives include Magnesium Hydroxide (Milk of Magnesia), Castor oil, Metamucil, & Methylcellulose. ANTACIDS-neutralize excess stomach acid by a chemical reaction. Aluminum Hydroxide-the MOST POTENT ANTACID, but has less neutralizing capacity than calcium carbonate or sodium bicarbonate (the only systemic antacid). Gastric antacids are drugs that directly neutralize the gastric acid (HCL) secreted in the stomach. Antacid therapy is directed at decreasing the concentration and total load of gastric acid. Common OTC Antacids: o Sodium bicarbonate product: Alka-Seltzer

o Calcium carbonate products: Amitone & Tums o Aluminum hydroxide products: Alterna GEL & Amphogel o Magnesium hydroxide products: Milk of Magnesia (Maalox) o Bismuth subsalicylate salt products: PeptoBismol o Magnesium & Aluminum products: Maalox & Mylanta Aluminum Salts used as antacids: hydroxide, carbonate, phosphate, and aminoacetate. Dyspepsia-an impairment of the power or function of digestion.

ANTI-CONVULSANTS (ANTIEPILEPTICS) Epilepsy-a neurological disorder characterized by sudden, recurring attacks of motor, sensory, or psychic malfunction with or without loss of consciousness or convulsive seizures. The goal of anti-convulsant therapy is to reduce or eliminate these seizures. • Carbamazepine (Tegretol)-used a prophylaxis for partial seizures with complex symptomatology (psychomotor and temporal lobe seizures). Also treats tonic-clonic seizures (grand mal), and pain associated with TRIGEMINAL NEURALGIA by blocking Na+ channels. Adverse effects: diplopia, ataxia, enzyme induction, and blood dyscrasias, but rarely causes aplastic anemia. • Diazepam (Valium)-treats STATUS EPILEPTICUS and emergency treatment of seizures. May cause drowsiness, dizziness, and ataxia. • Ethosuximide (Zarontin)-the preferred drug for effectively treating absence seizures because it causes minimal sedation by BLOCKING CALCIUM CHANNELS. Adverse Effects: GI distress, lethargy, headache.

• Gabapentin (Neurontin)-used as an adjunct to treat partial seizures. • Phenytoin (Dilantin)-treats tonic-clonic (grand mal) seizures. The rate of gingival hyperplasia is diminished by proper oral hygiene. Produces Na+ channel blockade. Phenytoin is the most extensively used of all anti-epileptics, used IV for status epilepticus. Phenytoin-induced gingival hyperplasia is a common adverse effect, that may partially or completely obscure teeth crowns. • Valproic Acid (Depokote or Depakene)-also a preferred drug for effectively treating complex partial seizures, and adjunctively in patients with multiple seizure types, including absence seizures because it causes minimal sedation. It functions by causing neuronal membrane hyperpolarization. Adverse Effects: GI distress, lethargy, headache, blood dyscrasias, hepatotoxicity & liver failure. • Most of the commonly used anti-convulsants are CNS depressants. Thus, respiratory depression may occur with overdosage of anti-convulsants. PARKINSON’S DISEASE-a slowly progressing, degenerative disorder of the nervous system with several distinguishing features: tremor (shaking) when at rest, sluggish initiation of movements, and muscle rigidity.

In Parkinson’s disease, nerve cells in the BASAL GANGLIA DEGENERATE, causing decreased dopamine production. It may be treated, but not cured with various drugs: 1. Carbidopa-a drug used to treat PARKINSON’S DISEASE, but only works when combined with LEVODOPA (treats Parkinson’s Disease to replenish the brain’s supply of dopamine, which is the deficient neurotransmitter in Parkinson’s. • Administering Carbidopa + Levodopa = Sinemet, which reduces the required dose of levodopa by ~75%. When levodopa is given alone, most of the dose is metabolized before it reaches the brain. Thus, large doses are required and can cause unwanted side-effects. • Carbidopa inhibits the peripheral decarboxylation of levodopa to simultaneously reduce peripheral side-effects and to allow more levodopa to reach the brain. Since carbidopa does not cross the blood-brain barrier (BBB), levodopa is converted into dopamine in the brain. Thus, co-administration of carbidopa + levodopa in the form of SINEMET allows a significant reduction of levodopa dosage without reducing the desired effects. 2. Levodopa (L-dopa, Dopar)-in combination with

carbidopa, is the precursor of dopamine, and MAIN TREATMENT for Parkinson’s Disease. Administered with carbidopa to increase its effectiveness and reduce adverse effects. 3. Bromocriptine (Parlodel) & Pergolide-dopamine agonists often given in addition to levodopa early in the treatment process to enhance levodopa’s action, or may be given later in treatment when levodopa’s side effects become problematic. 4. Selegiline (Eldepryl)-selective inhibitor of MAO Type B (the enzyme that causes the oxidative deamination of dopamine in the brain). Selegiline is used as an adjunct to levodopa. 5. Amantadine (Symmetrel)-anti-viral agent that enters the CNS to treat Parkinson’s disease by potentiating dopaminergic responses. Anti-parkinson actions are unrelated to its antiviral effects. Used in the early stages for mild disease. 6. Anticholinergic drugs: includes Benztropine & Trihexyphenidyl, certain anti-depressants, & antihistamines like diphenhydramine. These may be given without L-dopa in early stages of the disease, with levodopa in later stages.

ANTIBIOTICS & ANTIFUNGALS Sulfonamides (“Sulfa drugs”)-BACTERIOSTATIC agents structurally similar to Para-aminobenzoic Acid (PABA) which is the basis for their antibacterial actions. Bacteria require PABA to synthesize folic acid which is needed to synthesize bacteria cellular components for bacterial cell growth. Due to the structural similarities between sulfonamides and PABA, the sulfonamides COMPETE with PABA to inhibit PABAs actions, which prevents bacterial folic acid synthesis to inhibit cellular growth. Sulfonamides are “sulfa drugs” because their molecules contain sulfur atoms and their unique structure gives them their unique antibacterial mechanism of inhibiting folic acid synthesis and cell growth. • Sulfonamides DO NOT TREAT dental infections because of a low degree of effectiveness against oral pathogens. However, they are used primarily in medicine to treat URINARY TRACT INFECTIONS (UTIs). • Bactrim-the brand name of combining Trimethoprim + Sulfamethoxazole. Bactrim is the drug of choice for many UTIs. Note: Trimethoprim component is an antimicrobial, while Sulfamethoxazole is a sulfonamide.

3 Allergic Reactions to Penicillin: a RASH is the most common sign/manifestation of an allergy to PENICILLIN. The most common adverse effect of penicillin therapy is an allergic reaction. These reactions occur in up to 10% of patients receiving penicillin. 1. Acute/Immediate Onset Reactions (Anaphylactic Shock)-occurs within 30min and are IgE mediated. Characterized by urticaria, angioedema, bronchoconstriction, GI disturbances, and shock. Death can result quickly if treatment is not given immediately (parenteral administration of EPI). This is rare, but can occur especially with oral dosing. • Cephalosporins: are definitely contraindicated for penicillin-allergic patients who exhibit immediatetype reactions. • Ex: a patient was given penicillin 15min ago and develops laryngeal edema, urticaria (welts that itch), severe hypotension, GI disturbances, bronchoconstriction (airway constriction), and shock. This patient is having an anaphylactic reaction (anaphylactic shock). • An anaphylactic reaction can be fatal if countermeasures like injecting EPI are not taken promptly. EPI prevents the release of substances from mast cells and antagonizes the actions of

histamine and leukotrienes of smooth muscle. 2. Accelerated Allergic Reaction-occurs 30-48hrs after administration of penicillin. Manifestations are urticaria, pruritus, wheezing, mild laryngeal edema, and local inflammatory reactions. Not life-threatening. 3. Delayed Allergic Reaction (Skin Rash)-occurs after 23 days. Approximately 80-90% of all allergic reactions occurring with penicillin are “delayed”, manifested by mild skin rashes (the most prevalent allergic manifestation). Hypersensitivity reactions-occur in up to 10% of patients receiving penicillin. Manifestations range from a mild rash to anaphylaxis. The rash may be urticarial, vesicular, bullous, or maculopapular. Rarely thrombopenic purpura develops. PROBENECID (Benemid)-co-administered with antibiotics to DELAY RENAL CLEARANCE of the antibiotic to elevate and prolong the antibiotic’s serum concentrations when high tissue concentrations are necessary. In some cases, Probenecid administration can more than double the drug’s serum concentration. The drug’s half-life is also prolonged. Given concurrently with natural penicillins to increase their blood levels. • Most penicillins are handled by the kidneys as organic acids and excreted by renal tubular secretion.

Probenecid interferes with tubular handling of organic acids inside the nephron. Probenecid affects most cephalosporins, natural penicillins, and other beta-lactam related antibiotics like Aztreonam and Imipenem. • Probenecid INHIBITS renal tubular cell secretion that raises penicillin blood levels by diminishing their tubular secretion. Probenecid is sometimes given simultaneously with penicillin to raise penicillin blood levels for increased activity, and is the DRUG OF CHOICE TO TREAT GOUT. • Nafcillin, Oxacillin, Cloxacillin, & Dicloxacillin are lipophilic and excreted by BILIARY MEANS (LIVER). Thus, a combination of Probenecid or dosage adjustment for renal dysfunction is NOT necessary for these penicillins. • Imipenem-beta-lactam antibiotic derived from Thienamycin, and first drug classified as a Carbapenem antibiotic. Currently the drug of choice for infections due to Enterobacter, and is usually combined with Cilastin and treats severe or resistant infections, especially nosocomial infections. • Aztreonam-a parenteral synthetic beta-lactam antibiotic (classified as a monobactam). The spectrum

is limited to aerobic gram (-) rods like Klebsiella, Pseudomonas, & Serratia. It has no gram (+) or anaerobic activity, and is synergistic with aminoglycosides. Therapeutic Management of GOUT involves three different aspects of the disease: 1. Colchicine-reduces the inflammation during acute attacks by impairing leukocytic migration to inflamed areas, and disrupts urate deposition and subsequent inflammatory response. Colchicine is most effective when given 12-36hrs after the symptoms begin. It is NEVER given IM or subcutaneously because it causes tissue irritation. • Adverse Effects: severe liver & kidney damage, and long-term therapy may cause bone marrow depression. • Indomethacin (NSAIDs) are important for treating acute gouty arthritis, but may cause renal damage or bone marrow depression. 2. Allopurinol (Zyloprim)-decreases uric acid production. Drug of choice for managing CHRONIC GOUT by inhibiting xanthine oxidase (enzyme that converts hypoxanthine→xanthine→uric acid).

• Adverse Effect: GI disturbances. 3. Probenicid (Benemid) & Sulfinpyrazone (Anturane)uricosuric agents that enhance uric acid clearance by acting in the kidney’s proximal convoluted tubules, and inhibit secretion of other weak acids (i.e. penicillin) in addition to inhibiting uric acid reabsorption. Important: normal penicillins and cephalosporins must be given in high and frequent doses due to their high rate of elimination by the kidneys. Probenicid, is given simultaneously to SLOW THEIR EXCRETION. PENICILLINS: are BACTERICIDAL by inhibiting cell-wall synthesis. A MAJOR disadvantage of penicillin is their high incidence of ALLERGIC reactions. ~10% of the population is allergic to penicillins. This incidence probably holds for any of the specific penicillins because there is a cross allergy from one to another. In non-allergic individuals, penicillin at normal therapeutic doses virtually have no side effects. Penicillins are BACTERICIDAL by causing death of invading bacteria. Penicillins are NOT active against viruses, fungi, Rickettsiae, or other nonbacterial organisms. All penicillins are DERIVATIVES of 6-aminopenicillanic acid and contain a beta-lactam ring structure connected to a thiazolidine ring (this beta-lactam ring is essential for

its antibacterial activity). This basic structure is synthesized by the penicillin mold from 2 amino acids (L-cysteine & Lvaline). Antibiotics containing this beta-lactam ring are “beta-lactam antibiotics” which includes penicillins, cephalosporins, and two newer groups of agents (Carbepenems & Monobactams). Most penicillins are directly EXCRETED INTO URINE via renal tubular cell secretion. Probenecid (Benemid)-an inhibitor of renal tubular cell secretion that raises the blood levels of the penicillins by diminishing their tubular secretion. Probenecid is sometimes given simultaneously with penicillin to raise the blood levels for increased activity, and is the DRUG OF CHOICE TO TREAT GOUT. 1. Penicillin VK-the antibiotic/antimicrobial associated with the highest incidence of drug allergy. However, it the antibiotic of choice for treating non-penicillinase producing gram (+) staphylococcal infections. It is prudent to use an antibiotic with narrow spectrum of action and one that is bactericidal to minimize the development of bacterial resistance. Penicillin VK has these properties. Has a relative limited spectrum of action against aerobic gram (+) cocci and anaerobes. Penicillin VK is a naturally occurring penicillin preferred for treating oral infections because it is more acid stable (more reliable oral absorption).

• Can be used in pregnant patients as it is not harmful to the fetus. It is not incorporated into bony tissue or in the teeth of children unlike tetracyclines. • Adverse effects: hypersensitivity that causes a skin rash and rare anaphylaxis. 2. Penicillin G (Benzylpenicillin)-the PROTOTYPE penicillin for comparison. By side-chain substitutions (substituting other groups at “R” position of the penicillin molecule) of the basic 6-aminopenicillanic acid molecule, the semi-synthetic penicillins are produced (a more acid stable, broader spectrum, or penicillanse resistant). • Penicillin G Procaine (Cysticillin)-always given intramuscular (IM) route. • Penicillin G Benzathine (Bicillin L-A)-always given intramuscular (IM) route to treat SYPHILIS & PREVENT RHEUMATIC FEVER. It has a longer duration of action than Penicillin G Procaine. 3. Carbenicillin-has the WIDEST (BROADEST) spectrum of antibacterial activity. Other penicillins with very broad spectrums of action are Ticarcillin & Piperacillin. In addition to being active against gram (+) cocci (streptococci, staphylococci, and pneumococci), and gram (+) rods (bacillus and others), these agents are

active against Pseudomonas, Proteus, Klebsiella, and Bacteroides. These antibiotics are used to treat URINARY TRACT INFECTIONS (UTIs) and other infections caused by susceptible gram (-) Pseudomonas species and Proteus species. Degraded in the stomach acid, so must be given parenterally (IV). Aminopenicillins (Ampicillin, Amoxicillin, Becampicillin)-like other penicillins, aminopenicillins inhibit bacterial cell wall synthesis. Aminopenicillins are characterized by the amino substitution of Penicillin G, allowing them to penetrate/work against many gram (-) bacteria more readily than the natural penicillins or the penicillinase-resistant penicillins. Gram (-) bacteria susceptible to aminopenicillins are Haemophilus influenzae, some Escherichia coli, & Proteus mirabilis. However, aminopenicillins are not stable to betalactamases (penicillinase) of either gram (+) or gram (-) bacteria. They are NOT penicillinase resistant. Aminopenicilins are useful for treating upper respiratory infections. 1. Ampicillin (Polycillin, Onipen)-an aminopenicillin with a broader spectrum of action than penicillin VK. Given orally or IV to primarily treat infections like otitis media, bronchitis, sinusitis, and acute bacterial cystitis caused by suspectible organisms. The preferred agent to treat URINARY TRACT INFECTIONS caused by

enterococci (streptococci that inhabit the intestine). • “Parenteral” ampicillin is the drug of choice for patients unable to take oral medications and who are not allergic to penicillin for prophylaxis for bacterial endocarditis. 2. Amoxicillin (Amoxil, Larotid)-an aminopenicillin with an extended spectrum of action which includes aerobic gram (+) cocci and anaerobes, and some gram (-) bacilli (Hemophilus, Proteus, Salmonella). Only given orally to primarily treat infections like otitis media, bronchitis, sinusitis, and acute bacterial cystitis caused by suspectible organisms. Compared to ampicillin, amoxicillin has a higher oral absorption, higher serum levels, longer half-life, and is less likely to cause adverse GI effects (diarrhea). The preferred agent to treat URINARY TRACT INFECTIONS caused by enterococci (streptococci that inhabit the intestine). • Amoxicillin (along with Cephalexin & Cephradine) is a drug of first choice for standard general prophylaxis medication in a patient NOT allergic to penicillin with a TOTAL JOINT REPLACEMENT (prosthetic implants). Prophylaxis standard dose/regimen is 2gm orally 1hr prior to dental treatment. • “Oral” amoxicillin is the drug of choice for

standard general prophylaxis for bacterial endocarditis in patients undergoing invasive dental procedures. It is the antibiotic of choice for standard prophylactic regimen of antibiotic coverage to prevent bacterial endocarditis. • Amoxicillin has a higher oral absorption, higher serum levels, and longer half-life than ampicillin. Amoxicillin is given orally, while ampicillin can be given orally or IV. • Methotrexate-has a clinically significant drug interaction with Amoxicillin. In large doses, amoxicillin inhibits renal tubular secretion of methotrexate, thus causing higher, prolonged serum levels of Methotrexate. Methotrexate may cause ulceration of the oral tissues. 3. Bacampicillin (Spectrobid)-an aminopenicillin used to treat upper and lower respiratory tract infections, urinary tract infections, and skin & skin structure infections. Bacampicillin is hydrolyzed to Ampicillin during absorption from the GI tract where it has a better absorption than ampicillin, and less GI side effects. A functional part of the chemical molecule of all the penicillins is the BETA-LACTAM RING (4-membered imbedded ring structure consisting of 3 carbons and 1 nitrogen atom) that is responsible for the antibacterial

activity of penicillin. Any alteration to the beta-lactam ring alters the penicillin’s antibacterial activity.

Beta-Lactamases-enzymes produced and secreted by a wide range of gram (+) & gram (-) bacteria as a defense weapon against cephalosporin and penicillin antibiotics. Beta-lactamase enzymes destroy the beta-lactam nucleus within these antibiotics by splitting open the beta-lactam ring structure to render the antibiotic ineffective. Bacterial enzymes that belong to the family of beta-lactamases are: • Cephalosporinases-beta-lactamases that work against cephalosporins.

• Penicillinases-an enzyme secreted by bacteria that splits open the beta-lactam ring to render the penicillin molecule ineffective against penicillinase secreting bacteria. They are beta-lactamases that work against penicillins. Penicillinase Resistant Penicillins: Beta-Lactamase Inhibitors: by combining an acid with the penicillin, the beta-lactamase enzyme is permanently inhibited by the acid, which maintains the penicillin’s antibacterial activity. These penicillins resist the actions of penicillinase because they have a protected beta-lactam ring that prevents the actions of the enzymes. These antibiotics are highly effective against penicillinase-producing Staphylococcal aureus, but much less effective against other gram (+) bacteria, and have almost no activity against gram (-) bacteria. 1. Methicillin-a structurally modified penicillin that allows the molecule to remain stable in the presence of the beta-lactamase enzyme, but may also narrow its spectrum of action, limiting the use of such antibiotics only to gram (+) cocci like Staphylococcus bacteria. Degraded in the stomach acid, so must be given parenterally (IV). Methicillin is prescribed primarily to treat severe penicillinase-producing staphylococcal infections. It is not frequently

used due to the incidence of intestinal nephritis and the availability of equally efficacious alternatives (Nafcillin & Oxacillin). Methicillin is given IV in severe penicillinase-producing staphylococcal infections. Methicillin-Resistant Staphylococcal Aureus (MRSA)-a group of resistant staphyloccoal bacteria that can be life-threatening and resist ALL penicillinase-resistant penicillins & cephalosporins. Such strains are also usually resistant to aminoglycosides, tetracyclines, erythromycins, and clindamycin. In the past, vancomycin has been used against MRSA bacteria. However, microorganisms resistant to vancomycin have been reported, and its use has been curtailed. 2. Augmentin-it is Amoxicillin + Clavulanate potassium which blocks the actions of penicillinase from reaching the beta-lactam ring. It is a popular commercial beta-lactamase inhibitor preparation used orally as a pill or liquid form. 3. Unasyn-it is Ampicillin + Sulbactam which blocks the actions of penicillinase from reaching the betalactam ring. A beta-lactamase inhibitor available for IV & IM use.

4. Dicloxacillin-similar spectrum of action as penicillin VK, but is active against penicillinase producing staphylococcus. Other Penicillinase-Resistant Penicillins: Nafcillin, Oxacillin, & Cloxacillin.

Important Penicillin Facts: Penicillins degraded in stomach acid, thus must be administered PARENTERALLY (by injection): Methicillin, Carbenicillin, & Penicillin G. Acid Stable Penicillins (given orally): • Penicillin VK, Amoxicillin, Ampicillin, Nafcillin, Oxacillin, Cloxacillin, & Dicloxacillin. Extended Spectrum Penicillins: Aminopenicillins (Ampicillin & Amoxicillin). Broad Spectrum Penicillins: Carbenicillin, Piperacillin, Ticarcillin have the WIDEST spectrum of

all penicillins. Penicillinase-Resistant Penicillins: Methicillin, Nafcillin, Oxacillin, Cloxacillin, & Dicloxacillin. CLINDAMYCIN-a BACTERIOSTATIC antibiotic effective against most STAPHYLOCOCCI, AEROBIC & ANEROBIC STREPTOCOCCI, but is MOST EFFECTIVE in treating infections due to BACTEROIDES SPECIES. • Clindamycin binds to the 50S ribosomal subunit, blocking bacterial protein synthesis (translocation). It used is restricted by its adverse side effects (severe diarrhea & abdominal pain/GI upset due to Pseudomembranous colitis) caused by the overgrowth of the bacteria Clostridium difficile. Thus, used cautiously because Clostridium difficile always resists Clindamycin. • Clindamycin is bacteriostatic, and active against most gram (+) and many anaerobic organisms (including anaerobic gram (-) bacteria Bacteroides fragilis). • In Dentistry, Clindamycin is an alternate antibiotic when amoxicillin cannot be used to prevent bacterial endocarditis in patients undergoing dental procedures due to an allergy to penicillins (Clindamycin is given to penicillin-allergic patients allergic since there is no cross allergenicity between penicillins &

clindamycin), to treat common oral-facial infections caused by aerobic gram (+) cocci and susceptible anaerobes, and for prophylaxis for dental patients with total joint replacement. • Used for prophylaxis of endocarditis in penicillin allergic patients with valvular disease. Treats oral infections & penetrates into bone (not CSF). Bacterial Endocarditis Prophylaxis (Antibiotic) is Recommended: 1. High-Risk Category: • Prosthetic cardiac valves (i.e. biosynthetic (mechanical) & homograft (pig) valves) • Previous bacterial endocarditis. • Complex Cyanotic Congenital Heart Disease (i.e. single ventricle states, transposition of great arteries, Tetrology of Fallot). • Surgically constructed synthetic pulmonary shunts or conduits. 2. Moderate-Risk Category: • Most other congenital cardiac malformations • Acquired valvular dysfunction (i.e. Rheumatic heart disease). • Hypertrophic cardiomyopathy. 4. Dental Procedures: extractions, periodontal procedures

(surgery, SRP, probing, and recall maintenance), implant placements of avulsed teeth, endodontic (RCT) instrumentation of surgery ONLY past the apex, subgingival placement of antibiotic fibers/strips, initial placement of orthodontic bands (not brackets), intraligamentary local anesthetic injections, prophylactic cleaning of teeth or implants where bleeding is anticipated. Bacterial Endocarditis Prophylactic Antibiotic is NOT required: • Isolated atrial or ventricular septal defects, or patent ductus arteriosus • Coronary Artery Bypass Graft Surgery (CABG). • Rheumatic Heart Disease • Mitral valve prolapse • Heart murmurs • Kawasaki Disease without valvular dysfunction and bicuspid valve disease • Cardiac Pacemakers (intravascular & epicardial) and implanted defibrillators • Calcified aortic stenosis • Restorative Dentistry: with or without a retraction cord, local anesthetic injections (nonintraligamentary), intra-canal RCT, post and core placement, placing rubber dams, post-operative suture removal, placement of RPD and orthodontic appliances, impressions, fluoride treatments, radiographs, or shedding of primary teeth.

NO Antibiotic prophylaxis for patients with non-valvular devices UNLESS 2-3 weeks after surgery and healing is still occurring. AHA does NOT recommend dental antibiotic prophylaxis AFTER these non-valvular devices are placed: • Non-Valvular Devices: PACEMAKERS, implantable cardioverter defibrillators, left ventricular assist devices (LVAD), total artificial hearts, ventriculoatrial SHUNTS, peripheral vascular STENTS, hemodialysis prosthetic vascular grafts, intra-aortic balloon counterpulsation CATHETERS, coronary angiography and percutaneous coronary artery intervention, CORONARY ARTERY STENTS, vascular closure devices, and vena cava filters. Antibiotic prophylaxis for dental patients with TOTAL JOINT REPLACEMENTS per ADA & AAOS: • Currently no scientific evidence supports antibiotic prophylaxis to prevent HEMATOGENOUS INFECTIONS PRIOR to dental treatment in patients with total joint prosthesis. • Antibiotic prophylaxis is NOT indicated for dental patients with PINS, PLATES, & SCREWS, nor is it routinely indicated for most dental patients with total joint replacements. • High-Risk Patients with prosthetic joints for Hematogenous Total Joint Infection: ALL patients

during the first 2 years after joint replacement require antibiotic premedication. If more than 2 years after prosthetic joint replacement pass with no complications, then premedication is NOT required. Feb 2009: AAOS recommends clinicians consider prophylaxis for all total joint replacement patients before any invasive procedures that may cause bacteremia with Cephalexin, Cephradine, or Amoxicillin 2gm PO 1 hour prior to procedure. If unanticipated bleeding occurs, administer an antimicrobial prophylaxis within 2 hours after the procedure! If a series of dental procedures is required: observe an interval of 9-14 days between procedures to reduce the potential for the emergence of resistant organisms, and allow the mouth to repopulate with antibiotic susceptible flora. If a patient is taking an antibiotic normally used for endocarditis prophylaxis then SELECT a drug from a different class rather than increase the dose of the patient’s current regimen, or delay the procedure for 9-14 days after the patient completes the antibiotic. Bacterial Endocarditis Prophylaxis: • Amoxicillin: 4 capsules (each 500mg = 2000mg = 2g) 1 hour before the procedure. Dispense 16

capsules. • Clindamycin-take if allergic to penicillin. 2 capsules (each 300mg = 600mg) 1 hour before appointment. • Cephalexin-4 capsules (each 500mg = 2000mg = 2g) 1 hour before appointment. Dispense 16 capsules. • Cefadroxil-4 capsules (each 500mg = 2000mg = 2g) 1 hour before appointment. Dispense 20 capsules. TETRACYCLINES-bacteriostatic antibiotics that exert its antibacterial activity by INHIBITING BACTERIAL PROTEIN SYNTHESIS in the bacterial cell and the FIRST CHOICE to treat mycoplasma pneumonia, chlamydia infections, rickettsial infections, & vibrio infections. Tetracyclines treat acne, gonorrhea, syphilis in patients allergic to penicillin, and exacerbations of chronic bronchitis. They have limited use for treating oral infections, but are good alternatives to penicillin for patient with ANUG (acute necrotizing ulcerative gingivitis) who require antibiotics. May be useful in periodontal diseases (juvenile periodontitis caused by actinobacillus infections). 1. Minocycline (Minocin)-treats acne, anthrax, and meningococcal prophylaxis. Ex: Arestin used to treat periodontal pockets causing the pocket to shrink. 2. Doxycycline (Vibramycin)-treats syphilis, rickettsia infections, Chlamydia, & mycoplasma infections, and is an alternative to mefloquine for malaria prophylaxis.

3. Demeclocycline (Declomycin) Tetracyclines Indications/Treatments: Medical infections causes by susceptible gram (+) & gram (-) bacteria. Infections caused by Mycoplasma, Chlamydia, or Rickettsia. Exacerbations of Chronic Bronchitis. Acne, gonorrhea, and syphilis in patients allergic to penicillin. Periodontitis associated with Actinobacillus Actinomycetemcomitans (AA). Tetracyclines ARREST RAPID BONE LOSS associated with juvenile periodontitis via tissue regeneration & enhanced repair due to their collagenase inhibiting effect. Tetracyclines have been used as alternatives to penicillin in ANUG patients (Acute Necrotizing Ulcerative Gingivitis) who require antibiotics. • Absorption of tetracyclines from the GI tract is inhibited by divalent & trivalent cations (Ca2+, Mg2+, Fe2+, Al3+. These cations form CHELATION PRODUCTS with the tetracyclines to prevent their

absorption. Thus, tetracyclines are NOT GIVEN with milk or dairy products that contain Ca2+, ironcontaining vitamins (contain Fe2+), mineral supplements containing these ions, or antacids that contain Mg2+. • Tetracycline absorption from GI tract is inhibited by divalent & trivalent cations (Ca2+, Mg2+, Fe2+, Al3+) thus, are not given with milk and dairy products, iron supplements, or antacids. The therapeutic effectiveness of tetracyclines is most affected (reduced) by concomitant ingestion of antacids. Tetracycline chelates with calcium. • Common Tetracyclines Adverse Effects: photosensitivity, nausea, diarrhea, fungal superinfections, teeth discoloration & enamel hypoplasia in young children. o LIGHT exposure causes oxidation of tetracyclines into a brown material that permanently discolors teeth. Tetracyclines are contraindicated in children up to age 8 and in pregnant women, since they can chelate Ca2+ ions and become incorporated in bony tissues. A greenish-brown discoloration effect is sometimes seen in newly erupted teeth of infants whose mothers have received

tetracycline during pregnancy. o Occurrence of opportunistic (superinfections) caused by Candida albicans due to the wide spectrum of antibacterial actions which alters normal flora (i.e. vaginal and oral candidiasis/vaginal oral yeast infections). Thus, tetracyclines use to treat odontogenic infections is limited because they cause “yeast” infections very easily. o PHOTOSENSITIVITY. The photosensitivity reaction caused by tetracyclines causes the appearance of red rashes or blotches over the skin in the presence of sunlight. • Tetracyclines are a group of broad-spectrum, bacteriostatic antibiotics that INHIBIT PROTEIN SYNTHESIS in the bacterial cell wall by binding to the 30S-subunit of the bacterial ribosome. Inhibiting ribosomal function interferes with the attachment of the growing amino acid chain to prevent complete formation of peptides from the ribosome. Since no peptides are formed, no proteins are formed. Since proteins are required for the bacterial cell to metabolically function, the lack of proteins causes a static state where the bacterium becomes vulnerable to phagocytosis by the body’s immune system.

MACROLIDES-inhibit protein synthesis. 1. Erythromycin-the prototype macrolide, and 2nd choice bacteriostatic agent to penicillins to treat oro-dental infections caused by aerobic gram (+) bacteria and against many oral anaerobes (not very effective against gram (-) bacteria). However, resistance may develop quickly. Adverse Effects: GI EFFECTS in ~21% of patients, and OTOTOXICITY (a less common, but significant adverse reaction to erythromycin (especially after IV admin) manifested as tinnitus and/or deafness. • Oral bioavailability of erythromycin is poor. It is readily inactivated by stomach acid, thus several salts have been developed to overcome this drawback: Erythromycin Stearate (Erythrocin) & Erythromycin Estolate (Llosone) has the best oral absorption, but can cause liver toxicity. • To prevent their destruction by stomach acid, erythromycins are usually ENTERIC COATED (a special coating applied to tablets or capsules that prevents their release and absorption of their contents until they reach the intestines. Erythromycin is metabolized in the liver, and excretion is mainly via bile. • In penicillin allergic patients, erythromycins are important alternatives, and may be given

prophylactically before dental procedures to prevent BACTERIAL ENDOCARDITIS. It is second only to penicillins to treat dental infections, since it is effective against most aerobic gram (+) bacteria that cause orodental infections. • Erythromycin was previously used as an alternate agent for the penicillin-allergic patient, but is NO LONGER USED due to GI tract upset (the most common side effect, so is always taken with food) & complicated pharmacokinetics. However, practitioners who successfully use erythromycin for prophylaxis, may choose to continue using it. • Erythromycin is a BACTERIOSTATIC antibiotic that binds to the 50S ribosomal subunit of susceptible bacteria, causing INHIBITION OF PROTEIN SYNTHESIS. • Erythromycin Derivatives: Erythromycin base (Emycin; Eryc), Erythromycin ethylsuccinate (EES), Erythromycin stearate (Erythrocin), and Erythromycin estolate (Llosone). 2. Clarithromycin (Biaxin) & Azithromycin (Z-Pak, Zithromax)-alternative antibiotics for prophylaxis against bacterial endocarditis. Their bacterial spectrums of activity are similar to Erythromycin, but they possess greater intrinsic activity against H. influenzae. These

macrolides concentrate within macrophages like Mycobacterium avium intracellulare. Both cause fewer GI tract adverse reactions and fewer drug interactions than erythromycin. • Both agents have significant tissue penetration ability, and a prolonged elimination half-life of Azithromycin (11-14hrs), allows for one daily dose, and twice-daily dosing for Clarithromycin. • 10% of patients receiving Clarithromycin and 5% of patients taking Azithromycin have adverse GI effects. CEPHALOSPORINS-a BROAD-SPECTRUM & BACTERICIDAL antibiotics that inhibits bacteria cell wall synthesis during cell division so wall closure does not occur (causing cell lyses/death) (just like penicillin’s mechanism). Cephalosporins act against a wide-range of gram (+) & gram (-) bacteria, and are used in penicillin allergic patients with Staphylococcal infections. There are 4 generations of cephalosporins. Progression from the 1st to the 4th generation is associated with a broadening of action against more gram (-) bacteria and a decreased activity against gram (+) bacteria. 1. 1st Generation Cephalosporins: Cephalexin (Keflex), Cephradine (Velosef), Cefadroxil (Duricef), Cefazolin (Ancef). Used to prevent bacterial endocarditis in patients with a history of non-immediate allergic

reactions to penicillin. ~10% of patients with penicillin allergies have cross allergenicity to cephalosporins. Cephalosporins are contraindicated in patients with immediate-type hypersensitivity reactions to penicillins (i.e. urticaria, angioedema, & anaphylaxis). o Cephalexin & Cephradine are drugs of first choice for standard prophylaxis medication in a patient NOT allergic to penicillin, with a TOTAL JOINT REPLACEMENT (prosthetic implants). Prophylaxis dose is 2 grams orally 1hr prior to dental treatment. This is according to the “Advisory Statement, Antibiotic Prophylaxis for Dental Patients with Total Joint Replacement” published by the ADA and American Academy of Orthopaedic Surgeons in JADA. 2. 2nd Generation Cephalosporins: Cefaclor (Ceclor), Cefuroxime (Ceftin), Cefoxitin (Mefoxin), & Cefprozil). Treats oro-dental infections (acute cellulitis) caused by gram (+) & (-) aerobic bacteria, and against anaerobic bacteria causing periapical abscesses. 3. 3rd Generation Cephalosporins: Cefixime (Suprax) & Cefoperazone (Cefobid). 4. 4th Generation Cephalosporins: Cefepime (Maxipime). AMINOGLYCOSIDES-bactericidal antibiotics with a broad

antibacterial spectrum, used primarily to treat AEROBICGRAM (-) infections. They have little action against gram (+) anaerobic or facultative bacteria. They irreversibly bind to the 30S ribosomal subunit of bacteria to inhibit protein synthesis. • Aminoglycosides are reserved to treat more serious bacterial infections because of their 2 well-known adverse effects ototoxicity & nephrotoxicity. Aminoglycosides may cause severe neuromuscular weakness lasting hours-to-days because of their potential curare-like effect. Aminoglycosides may aggravate muscle weakness in patients with muscular disorders like myasthenia gravis, infant botulism, or parkinsonism, thus are contraindicated or used with great precaution in patients with these conditions. • Aminoglycosides are poorly absorbed after oral administration, thus must be administered IM or IV to produce a systemic effect. They are rapidly excreted by the normal kidney. • Gentamicin (Garamycin), Amikacin (Amiken), Tobramycin (Nebcin), & Netilmicin (Netromycin). These aminoglycosides are effective against serous infections caused by aerobic-gram (-) bacteria like E. Coli, Enterobacter, Klebsiella, Proteus, Pseudomonas aeruginosa, and Serratia.

• Streptomycin-the first aminoglycoside effective to treat tuberculosis, but is rarely used today. • Neomycin (Mycifradin) & Kanamycin (Kantrex)due to its toxicity potential, neomycin is used only topically or locally (i.e. in the GI tract). Kanamycin is rarely used because of its marked tendency to cause ototoxicity. • Spectinomycin-sometimes used to treat gonorrhea. Bacitracin-inhibits bacterial cell wall synthesis, gram (+) activity. • Potentially nephrotoxic if systemic, so only administered topically. • Sold as a triple antibiotic ointment/Neosporin with polymyxin & neomycin to treat dermatological & ophthalmic infections. Polymyxin B-cationic basic peptides that (cationic detergents) that SCRUB BACTERIA CELL MEMBRANES. o Administered topically due to potential nephrotoxicity (its major adverse effect). o Only active against gram (-) rods = PSEUDOMONAS (treat corneal ulcers & external otitis). o Sold as a triple antibiotic ointment (containing bacitracin or neomycin) to treat SUPERFICIAL SKIN LACERATIONS (dermatological & ophthalmic

infections). Chloramphenicol (Chloromycetin)-a broad-spectrum antibiotic that can cause BONE MARROW DISTURBANCES (aplastic anemia) thus, has LIMITED USE due to its side effects. Effective against gram (+) & gram (-) bacteria and against anaerobes, used as a 2nd or 3rd line drug in medicine to treat SERIOUS INFECTIONS due to organisms resistant to less toxic antibiotics. It is a bacteriostatic antibiotic that reversibly binds to the 50S ribosomal subunits of susceptible organisms, to prevent amino acids transfer to growing peptide chains, thus INHIBITING bacterial PROTEIN SYNTHESIS. Chloramphenicol causes 3 Major Toxicities: 1. Aplastic Anemia2. Bone Marrow Suppression (anemia & neutropenia may occur during the first week of antibiotic therapy). 3. Gray Syndrome-characterized by circulatory collapse, cyanosis, acidosis, coma, and death. Tuberculosis-a bacterial infection caused by MYCOBACTERIUM TUBERCULOSIS. Anti-tubercular drugs either inhibit bacterial growth or kill the bacteria. Since Mycobacterium tends to develop resistance to any single anti-tubercular drug, combination drug therapy is the standard in treating tuberculosis. TB is ALWAYS treated by MULTI-DRUG THERAPY (minimum of two drugs): Anti-tubercular drugs: 1. Rifampin-given in combination with other agents.

Prevents transcription (RNA synthesis) by inhibiting bacterial DNA dependent RNA polymerase. Treats Mycobacterium tuberculosis (TB), Mycobacterium leprae (Leprosy). THE MOST POTENT ANTI-LEPROSY AGENT. 2. Isoniazid-often given in a 4 drug regimen along with Rifampin, Pyrazinamide, & Ethambutol. Adverse Effects: PERIPHREAL NEURITIS (paresthesia) caused by pyridoxine (vitamin B6) deficiency. Corrected with vitamin B6 supplements, and FATAL HEPATITIS. 3. Pyrazinamide-popular in combination with Rifampin. Enters CSF to treat TUBERCUOSIS MENINGITIS 4. Ethambutol-given in combination with other agents. Adverse Effects: Optic neuritis, hyperuricemia, & COLOR VISION DISTURBANCES 5. Streptomycin-given in combination with Isoniazid. 6. Rifabutin-active against MAI complex. ANTI-MALARIA AGENTS: malaria is a disease due to infection by protozoa genus plasmodium. Plasmodium falciparum is the most dangerous. 1. Mefloquine (Lariam)-a member of the blood schizonticide anti-malarial drugs that treats MALARIA caused by Plasmodium falciparum (the most

dangerous Protozoan), P. vivax malariae, and P. ovale. It is active alone against multi-drug resistant Plasmodium falciparum. 2. Chloroquin (Aralen)-eradicates erythrocytic (RBC) forms by inhibiting plasmodial heme polymerase, and damages parasitic membranes by lysing the RBC and parasite. Also inhibits parasitic DNA synthesis. Treats erythrocytic forms of Plasmodium falciparum & Plasmodium vivax. A systemic amebicide that treats amebic liver abscess and extraintestinal amebiasis. 3. Quinine-back-up agent for chloroquine used in combination with FANSIDAR chloroquine-resistant malarial strains. Adverse effect: CINCHONISM (nausea, vomiting, vertigo, tinnitus). 4. Atovaquone + Proguanil (Malarone) 5. Sulfadoxine + Pyrimethamine (Fansidar) 6. Halofantrine 7. Pyrimethamine-folate antagonist (antifolate) active against P. falciparum, P. malariae & Toxoplasma gondii.

ANTI-FUNGALS Mycoses-chronic fungal infections. Mycotic infections are superficial (involving skin caused by dermatophytes), or can infiltrate the skin to cause subcutaneous infections. Antifungal agents are drugs to treat systemic and subcutaneous fungal infections. Candidiasis-a fungal infection usually found in the oral cavity or vagina (C. Albicans) that causes an inflammatory, pruritic infection characterized by a thick, white discharge. Common in patients with an immune deficiency in Tlymphocytes, receiving chemotherapy, & immunosuppressed/AIDS patients. However, his yeastlike fungi is a normal inhabitant of the oral cavity and vaginal tract, but is usually held in check by the indigenous bacteria of these areas. • Angular Cheilitis-bilateral ulcers at the corners of the mouth linked to C. Albicans. • Nystatin (Mycostatin)-drug of choice (anti-fungal polyene antibiotic) for treating oral cavity Candidiasis (fungal infection). An anti-fungal used as “swish & swallow” to treat oral cavity Candida infections. Taken as an oral suspension to be swished around the mouth and swallowed or may also be prescribed

in an ointment form that alters the fungal cell membrane to treat cutaneous and mucocutaneous Candidiasis. Nystatin is an oral suspension that alters the fungal cell membrane. Nystatin’s mechanism & structure is similar to Amphotericin B. Drug of choice for treating Candidiasis in an HIV-infected patient (i.e. Hairy Leukoplakia). • Clotrimazole (Mycelex Troche)-anti-fungal taken as a troche (lozenge) that is slowly dissolved in the mouth and swallowed. Treats oropharyngeal Candidiasis by altering the fungal cell membrane. • Nystatin & Clotrimazole work by binding to sterols in the fungal cell membrane, increasing permeability, and permitting the leakage of intracellular components. This leads to death of the affected fungal cell. TOPICAL Anti-Fungals (Creams/Ointments): 1. Amphotericin-B (Fungizone): anti-fungal given IV or orally to treat severe systemic fungal infections caused by fungi like Candida species. Treats cutaneous & mucocutaneous Candidiasis by altering the fungal cell’s membrane. 2. Ketoconazole (Nizoral)-inhibits Ergosterol synthesis

to disrupt the fungal membrane. Can antagonize/inhibit amphotericin B’s antifungal effect. Given orally to treat Histoplasmosis, Nonmmeningeal coccidioidomycosis, Blastomycosis, Dermatomycosis. Can be given in a cream form to treat cutaneous & mucocutaneous candidiasis by altering the fungal cell membrane. Adverse Effects: toxicity causes ENDOCRINE EFFECTS by inhibiting cortisol & testosterone synthesis causing (gynecomastia, decreased libido, and menstrual irregularities). SYSTEMIC Anti-Fungal Agents: 1. Fluconazole (Diflucan)-inhibits ergosterol synthesis. Given orally or IV, crosses BBB (enters CSF). Drug of Choice: Mucosal candidiasis (oropharyngeal & esophageal), & Coccidiomycosis. Treats blastomycosis, histoplasmosis, & Cryptococcal meningitis in AIDS patients. 2. Ketoconazole (Nizoral)-used to treat oral, esophageal, and oral esophageal Candida infections by inhibiting ergosterol synthesis to disrupt the fungal membrane. Adverse Effects: toxicity causes ENDOCRINE EFFECTS by inhibiting cortisol & testosterone synthesis causing (gynecomastia, decreased libido, & menstrual irregularities). 3. Amphotericin-B (Fungizone): an anti-fungal of choice

(wide spectrum) given IV injection or orally to treat SEVERE SYSTEMIC FUNGAL INFECTIONS (MYCOSES) caused by fungi like Candida by binding to ergosterol in the fungal cell membrane. Not is CSF Also treats cutaneous & mucocutaneous Candidiasis by altering the fungal cell membrane. Adverse Effects: associated with high incidence of KIDNEY TOXICITY. 4. Itraconazole-inhibits ergosterol synthesis. BROAD spectrum anti-fungal given ORALLY. Drug of choice for Blastomycosis & Paracoccidioidomycosis. Well distributed in BONE, SPUTUM, & ADIPOSE TISSUE (Not CSF). 5. Flucytosine-a PRODRUG that inhibits fungal DNA & RNA synthesis and cell division. Given orally to treat systemic mycosis by Chromoblastomycosis, Candidiasis, & Cryptococcus. Enters CSF, excreted by Glomerular Filtration. Adverse Effects: GI, Reversible Bone Marrow Suppression.

ANTI-PROTOZOALS 1. Nitazoxanide (Alinia)-an oral ANTIPROTOZOAL that treats DIARRHEA caused by GIARDIA LAMBLIA and protozoal infections caused by Cryptosporidium Parvum. This type of diarrhea is an intestinal infection called Giardiasis, and is the most common protozoan infection in the U.S. It mechanism is interfering with the electron transfer reaction within the protozoa that is essential to anaerobic metabolism. 2. Atovaquone (Mepron)-an anti-protozoal agent that treats Pneumocystitis Carinii Pneumonia (PCP) in patients intolerant to Co-trimoxazole (formed by the combination of Trimethoprim + Sulfamethoxazole). Cotrimoxazole is the Drug of choice for Pneumocystis carinii pneumonia by inhibiting folic acid synthesis. 3. Eflornithine (Vaniqa)-has orphan drug status to treat the meningoencephalitic stage of Trypanosoma Brucei Gambiense infection (SLEEPING SICKNESS). Also indicated as a facial cream to reduce unwanted facial hair and areas under the chin. 4. Furazolidone (Furoxone)-antiprotozoal agent that treats DIARRHEA caused by susceptible Giardias Lamblia and Vibrio Cholerae.

5. Metronidazole (Flagyl)-a synthetic antibacterial & antiprotozoal effective against Trichomonas Vaginalis that causes Trichomoniasis. It is one of the most effective drugs available against anaerobic bacterial infections. Metronidazole is not a true antibiotic since it is not found in natural organisms. It is a synthetic substance produced in a chemical laboratory. Adverse Effects: dizziness, headache, nausea.

ANTI-VIRALS VIRUSES-double or single stranded DNA or RNA enclosed in a protein coat (CAPSID). Viruses lack a cell membrane, wall, and metabolic machinery, thus are OBLIGATE INTRACELLULAR PARASITES. Anti-Viral Medications: • Penciclovir (Denavir)-an antiviral agent that is active against Herpes Simplex Virus Type 1 (HSV-1). It is a cream formulation indicated to treat recurrent herpes labialis (cold sores) in adults. This condition is caused by HSV-1. Penciclovir is not available for systemic dosing. Penciclovir inhibits viral action by selectively inhibiting herpes viral DNA synthesis which inhibits viral replication. • Acyclovir-inhibits viral DNA polymerase/viral DNA synthesis. Available in tablets and creams to treat HSV-1, HSV-2, & varicella zoster (chicken pox/shingles). Drug of Choice for HSV Encephalitis, genital herpes, herpes labialis (cold sores) & varicella-zoster virus. Enters CSF, and accumulates during renal failure. o Valacyclovir (VALTREX)-PRODRUG of acyclovir given orally that is converted by 1st

pass metabolism into acyclovir. Treats HSV1&2, genital herpes, cold sores (herpes labialis), & herpes varicella-zoster virus (shingles). • Ganciclovir-inhibits viral DNA polymerase/viral DNA synthesis. Treats Cytomegalic Retinitis (CMV retinitis) & CMV prophylaxis in transplant patients (crosses BBB). • Docosanol cream (Abreva) & Lysine tablets-antiviral agents that treat Herpes Labialis (cold sores). AIDS (Acquired Immunodeficiency Syndrome)recognized since 1981 as a unique clinical syndrome caused by an infection with the human immunodeficiency virus 1 (HIV-1) or virus 2 (HIV-2). The major cellular defect caused by HIV infection is DEPLETION OF T-CELLS (subtype T-helper cells CD4 cells) which causes a compromised immune system that is susceptible to opportunistic infections (i.e. Hairy Leukoplakia, Candidiasis). • HIV is a RETROVIRUS responsible for the fatal illness from AIDS. A “retrovirus” has RNA as its nucleic acid and uses the reverse transcriptase enzyme to copy its genome in the DNA of the host’s cell chromosomes. This DNA segment is then permanently incorporated into the host cell’s DNA

within the nucleus. The integrated DNA segment can produce new RNA in the host cell’s cytoplasm, and then synthesizes viral proteins that are passed onto other host cells like the immune system macrophages. Ultimately, enough of the human immune cells are compromised so that immune function is lost. • HIV drugs: Didanosine (Videx), Zidovudine (Retrovir), Ritonavir (Norvir), Indinavir (Crixivan) all work by inhibiting steps in the HIV infection process within the target cells to stop immune system destruction by the HIV retrovirus. • HIV Protease Inhibitors: Ritonavir (Norvir), Amprenavir, Indinavir (Crixivan), Nelfinavir (Viracept), Suquinavir (Invirase). All inhibit HIV aspartate proteases required to produce the structural proteins & enzymes necessary for viral replication. Protease inhibitors suppress viral replication by inhibiting protease (enzyme responsible for cleaving viral precursor peptides into infective virions). Contraindicated with patients taking Rifampin. • Nucleoside Reverse Transcriptase Inhibitors: Zidovudine (Retrovir, AZT), Zalcitabine (Hivid, ddC), Didanosine (Videx), Stavudine & Lamivudine. All are converted into AZTtriphosphate analogs in cells to inhibit viral DNA

synthesis & replication by inhibiting reverse transcriptase (RT). These agents are nucleosides that inhibit the viral enzyme “reverse” transcriptase” to inhibit HIV viral RNA from being made into a DNA segment (prevents the HIV virus genome of being copied from RNA). Adverse: myelosuppression of bone marrow. o Stavudine (d4T, Zerit)-anti-retroviral drug that treats adults with HIV in combination with other anti-retroviral agents. • Non-Nucleoside Reverse Transcriptase Inhibitors: Nevirapine (Viramune), Adefovir (Hepsera), Efacirenz, & Delavirdine (Rescriptor). Noncompetitive inhibitors of HIV reverse transcriptase that inhibit the catalytic reaction of reverse transcriptase that is independent of nucleotide binding. Does not cause myelosuppresion. • Interferon-natural glycoproteins synthesized by recombinant DNA technology to activate host enzymes to block viral RNA translation, and interfere with ability of viruses to infect cells. • IFN-a (intron A)-treats Chronic Hepatitis B&C, Genital Papilloma, & Kaposi’s Sarcoma in HIV patients. Treatment of Viral Respiratory Infections (Influenza A&B, Respiratory Syncytial Virus (RSV):

1. Amantadine (Symmetrel) & Rimantadine-anti-viral agents that inhibit/block viral membrane matrix protein “M2” ion channel. Used for prophylaxis or early treatment of Influenza “A” virus. Amantadine enters the CNS to treat Parkinson’s disease. 2. Ribavirin-inhibits viral mRNA synthesis. Treats SERIOUS RSV infections in infants/young children, influenza A&B, Acute Hepatitis C, & SARS. Active orally, IV and as an AEROSOL to treat respiratory viral infections.

NSAID’s (NON-STEROIDAL ANTIINFLAMMATORY DRUGS) NSAID’s-have anti-inflammatory effects due to their ability to inactivate the enzyme “prostaglandin endoperoxide synthase” (cyclooxygenase). Enzyme inactivation inhibits the cyclooxygenase step of the arachidonic acid cascade, thus reducing local prostaglandin synthesis. NSAID’s have anti-inflammatory, analgesic, & antipyretic actions. Prostaglandins are derived from the unsaturated fatty acids in cell membranes. NSAIDs decrease production of inflammatory mediators possess anti-inflammatory, analgesic, & anti-pyretic effects. A “ceiling” effect exists for the analgesic properties. Most NSAIDs work best for mild to moderate pain, although efficacy varies between NSAIDs. Safer than corticosteroids for long-term use, but some NSAIDs have strong adverse effects, thus contraindicated in specific patient populations. Traditional NSAIDs “reversibly” reduce platelet aggregation (normal platelet function returns when the drugs leave the system). Aspirin, Cortisol, Ibuprofen (Motrin, Advil, Nuprin,

Rufen), & Indomethacin have significant antiinflammatory properties. Cyclooxygenase (COX)-the enzyme that produces prostaglandins which comes in two forms (COX-1 & COX2). COX-1 enzyme-produces prostaglandins in the GI tract. The prostaglandins formed act as a protective substance against the formation of GI ulcers. Traditional NSAIDs (Ibuprofen, Naproxen, Aspirin) inhibit the COX-1 & COX-2 enzymes to diminish the formation of the protective prostaglandins. Thus, they effectively reduce pain and inflammation, but can induce GI ulcers are a potential adverse effect. Since traditional NSAIDs (ibuprofen, naproxen, and aspirin) inhibit both COX-1 & COX-2 enzymes they are “Non-Selective COX Inhibitors: • Propionic Acid Derivatives: Ibuprofen (Motrin, Advil, Nuprin, Rufen)-has anti-inflammatory properties by inhibiting prostaglandin synthesis/production in peripheral tissues at sites of pain and inflammation. Inhibiting prostaglandin production decreases the inflammatory response at sites of surgery, injury, or infection which results in a reduction of perceived pain. Ibuprofen is the MAIN INGREDIENT in OTC Advil, Nuprin, & Medipren (which contains 200mg of ibuprofen). Ibuprofen is also the main ingredient in Motrin (only sold via

prescription) and contains at least 400mg of ibuprofen. More efficacious analgesics & antiinflammatory than aspirin. Better for moderate pain. One of the stronger NSAID analgesics. Patients intolerant to aspirin may not be able to tolerate ibuprofen. GI ulceration & upset, and drug interactions are common adverse effects. Fenoprofen, Suprofen, Naproxen, Naproxen Sodium, Ketoprofen, & Benoxaprofen are also NSAIDs propionic acid derivatives. Ibuprofen (Motrin, Advil) is a non-narcotic analgesic that may interact with Warfarin (Coumadin) to cause unnecessary bleeding because it inhibits platelet aggregation. Ibuprofen can be given to patients with a history of drug abuse since it has no addictive properties. Ibuprofen is an NSAID and nonnarcotic analgesic which have no liability for abuse or addiction. They are not controlled substances. • Naproxen/Anaprox/Naprosyn (Aleve)-potent antiinflammatory & analgesic. Longer-acting than ibuprofen, better compliance, relieves pain longer. Does not interact with Warfarin or oral hypoglycemics, so better for Type II Diabetes Mellitus patients. Inhibit platelet aggregation.

• Flurbiprofen (Ansaid)-inhibit platelet aggregation. • Ketorolac (Toradol)-a newer NSAID and more efficacious analgesic than aspirin. Sometimes used for moderate-to-severe pain after minor dental surgery or painful dental procedure. Suggested use for no > 5 days. Not for longer-term pain relief, as may be toxic with long-term use. • Acetic Acid Derivatives (NSAIDs): Indomethacin (Indocin), Sulindac (Clinoril), & Tolmetin (Tolectin). These NSAIDs can cause GI bleeding, ulcers, and possible stomach perforation. • Fenamic Acid Derivatives: Meclofenemate (Meclomen) & Mefenamic acid (Ponstel). Drugs that Increase Bleeding Times: aspirin, nonselective NSAIDs (Ibuprofen, Naproxen, Ketoprofen, Flurbiprofen), anti-platelet drugs (Clopidogrel (Plavix) & Ticlopidine (Ticlid), and anti-coagulants (Warfarin (Coumadin) & Heparin). Ibuprofen & other “non-selective” NSAIDs (inhibitors of COX1 & COX2 enzymes) inhibit platelet aggregation. This inhibition enhance/potentiates the anti-coagulant effects of Warfarin (Coumadin) to increase bleeding risk.

Clopidogrel (Plavix)-inhibits blood clotting (increases bleeding time) by irreversibly inhibiting platelet aggregation. Thus, the effects on blood clotting are the same as with aspirin. Clopidogrel does not cause gastric ulcers like aspirin, and is the ANTIPLATELET AGENT OF CHOICE FOR PATIENTS WITH A HISTORY OF ULCERS. SALICYLATES: interfere with blood clotting mechanisms by irreversibly reducing platelet adhesion (stickiness or aggregation). Bleeding time is prolonged until new platelets are formed. Aspirin (Acetylsalicylic Acid)-prototypical nonselective COX inhibitors (NSAID) that inhibits COX-1 & COX-2 (cyclooxygenase) to inhibit prostaglandin production (prostaglandins are potent inflammatory mediators). Aspirin’s analgesic effects are better if given to prevent prostaglandin production. Prevents worsening of pain. However, cannot reduce pain already caused by prostaglandin build-up. Aspirin INACTIVATES the cyclooxygenase enzyme that makes prostaglandins, thus inhibits prostaglandin synthesis. As a result, aspirin is an analgesic, antipyretic (reduces fever), and anti-inflammatory. Aspirin is an irreversible platelet inhibitor and can reduce blood clotting, causing prolonged bleeding. Aspirin may interact with Warfarin

(Coumadin) to cause unnecessary bleeding. Antipyretic action of salicylates (aspirin) is explained in part by cutaneous vasodilation leading to increased heat loss. Aspirin is an anti-inflammatory, antipyretic, and analgesic used to relieve headaches, toothaches, minor aches and pains, and to reduce fever. The GI tract rapidly absorbs it. • Low doses of aspirin taken regularly can have a cardioprotective effect. These low doses reduce thromboxane production of platelets to result in the inhibition of platelet aggregation. In this way, aspirin has the ability to inhibit the formation of lifethreatening thrombi (blood clots). • If a patient is taking ibuprofen, ASPIRIN is the analgesic that if given while the patient is taking ibuprofen will DIMINISH the analgesic effectiveness of the ibuprofen. When aspirin and ibuprofen are given together, the analgesic efficacy of both is less than that of aspirin or ibuprofen alone. Aspirin displaces ibuprofen from plasma protein binding sites, thus hastening its disappearance from the blood stream. It is then quickly eliminated from the body by the kidneys through urine.

• Aspirin inhibits blood clotting by inhibiting platelet aggregation in an irreversible manner. Inhibiting platelet aggregation prevents the activation of the coagulation pathway, thus no fibrin (clot) is formed. Aspirin does not affect the coagulation pathway. Discontinuation of aspirin for 5-7 days allows for normal clotting time to reappear due to the synthesis of new platelets. • ASPIRIN (NSAIDs) CONTRAINDICATIONS: o Bleeding disorders (aspirin increases BT) & Asthmatics. o Children with viral infections with or without fever due to a potential association with Reye’s Syndrome (a serious neurological defect). o Pregnancy, especially the 3rd trimester. o Peptic ulcers as aspirin may cause GI tract bleeding/ulcerations. o Adverse effects: GI upset, overdose causes hepatic & nephrotoxic. Death if overdose by respiratory acidosis. o Salicylism-describes all of the symptoms caused from ingesting extremely large doses of ASPIRIN (tinnitus/ringing in ears, vertigo/dizziness, nausea, sweating, vomiting, headache, and mental confusion).

COX-2 ENZYME-produces prostaglandins at sites of surgery, infection, and inflammation. When COX-2 enzyme is inhibited, less prostaglandins are produced, and there is less pain and inflammation. COX-2 selective inhibitors reduce pain and inflammation without any risk of GI ulcers. COX-2 Selective Inhibitors-newer NSAIDs that do not affect blood clotting (do not affect platelet function/aggregation) thus can be given to patients concomitantly taking “blood thinners” like aspirin, warfarin (Coumadin), & heparin. • Rofecoxib (Vioxx) & Celecoxib (Celebrex) & Valdecoxib (Bextra)-more efficacious analgesic than aspirin and ibuprofen with less GI side effects, and does not inhibit platelet aggregation, so SAFER for bleeding disorders. COX-2 selective inhibitors treat signs & symptoms or RHEUMATOID OSTEOARTHRITIS, acute pain, and pain from dysmenorrhea. • COX-2 selective inhibitors are not salicylates because they are not aspirin drugs and are not opiates because they do not work like morphine, and are not steroidal anti-inflammatories because they are not corticosteroids like hydrocortisone. HEPARIN-a high MW heteropolysaccharide found especially in the LUNGS and inactivates thrombin and other coagulation factors to prevent blood clotting. Heparin is

contained inside mast cells & basophils found in C.T. and in extracellular spaces near blood vessels, especially in the lungs. *Heparin neutralizes tissue thromboplastin and blocks thromboplastin generation. Heparin inhibits blood clotting by affecting the coagulation pathway to PREVENT FIBRIN FORMATION. • Administration of heparin causes increased bleeding time due to a potentiation of Antithrombin III, thus inactivating thrombin. This prevents the conversion of fibrinogen→fibrin. • Heparin acts as an anticoagulant by enhancing the inhibition rate of clotting proteases by Antithrombin III impairing normal hemostasis and inhibiting factor Xa. Low MW heparins have a small effect on partial thromboplastin time (PTT), but STRONGLY inhibit factor Xa. • Heparin inactivates thrombin and prevents the conversion of fibrinogen to fibrin (blood clot). • Heparin is used for prophylaxis and treatment of thromboembolic disorders. • Standard heparin consists of components with MW ranging from 4000-30,000 daltons (average is 16,000 daltons). Low MW heparins range from 2,000-8,000 daltons.

• Low MW heparin anticoagulant agents: (Enoxaprin/Lovenox, Dalteparin/Fragmin, & Tinzaparin/Innohep) are used to treat acute symptomatic deep vein thrombosis (DVT) and to prevent deep vein thrombosis after knee or hip surgery. These are administered subcutaneously since they cannot be absorbed from the GI tract. • Thrombin-Inhibitor Type Anticoagulants: administered IV to prevent post-operative deep vein thrombosis (DVT) after elective hip replacement surgery, and for prophylaxis or treatment of thrombosis in adults with heparin-induced thrombocytopenia. Their mechanism of action is directly inhibiting thrombin within the coagulation pathway, thus inhibiting fibrin formation. Lepirudin (Refludan), Argatroban, Danaparoid (Orgaran). WARFARIN (COUMADIN) & DICUMAROL-anticoagulants that ANTAGONIZE VITAMIN K to prolong blood clotting time, causing decreased liver synthesis of vitamin-K dependent factors (II, VII, IX, & X). Warfarin inhibits blood clotting by affecting the coagulation pathway to PREVENT FIBRIN FORMATION. • Used after a myocardial infarction to prevent coronary occlusion, treat pulmonary embolism, and venous thrombosis.

• Enhanced anticoagulant effects are seen when Warfarin or Dicumarol are combined with aspirin. Thus, use acetaminophen (Tylenol) for pain control rather than aspirin for patients taking anticoagulants. • Warfarin interferes with the hepatic synthesis of vitamin-K dependent coagulation factors (II, VII, IX, X) resulting in the inability of the coagulation pathway to form fibrin (blood clot). • Vitamin K-a group of fat-soluble vitamins essential for the synthesis of coagulation factors II, VII, IX, X, & prothrombin in the liver. Vitamin K enhances (improves) blood clotting. Prothrombin Time (PT)-the most valuable test used to evaluate if a patient taking anticoagulants is a surgical risk. PT test is a one-stage test to detect certain plasma coagulation defects owing to a deficiency of Factors V, VII, or X. Thromboplastin and calcium are added to a sample of the patient’s plasma and simultaneously to a sample from a normal control. The length of time required for clot formation in both samples is observed. Thrombin is formed from prothrombin in the presence of adequate calcium, thromboplastin, and the essential tissue coagulation factors. A prolonged PT indicates a deficiency in one of the factors (as in liver disease, vitamin K deficiency, or anticoagulation therapy with the drug Coumarin).

After prothrombin times (PT) are determined, they are expressed as an INR value (International Normalized Ratio). INR is the ratio of the prothrombin time measured in the patient divided by a standard prothrombin time value, and multiplied by a constant. Patients on anticoagulant therapy may have excess bleeding after dental treatment. Always check the patient’s medical history. If the patient is on anticoagulants, have their physician provide documentation of their INR (International Normalized Ratio) values to assess anticoagulant effects. INR is the ratio of the prothrombin time measured in the patient divided by a standard prothrombin time value multiplied by a constant. The higher INR, the greater the anticoagulant effect (greater clotting). • INR value of 1: means normal prothrombin times of ~12sec; normal blood clotting would be present. • INR > 1: indicates an anticoagulant effect exists. • Many patients taking anticoagulants have INR values of 2, 3, 4, 5, and even 6. • For surgical procedures, an INR of 1-1.5 indicates a normal prothrombin time (~12-18 seconds). This is within a safe range.

Glycoprotein IIb/IIIa Inhibitor Antiplatelet Agents: these intravenous agents are reversible anti-platelet agents used to prevent acute cardiac ischemic complications and used in patients with acute coronary syndrome. These agents block the platelet glycoprotein IIb/IIIa receptor (the binding site for fibrinogen, von Willebrand factor, and other ligands). Inhibiting binding at this final common receptor REVERSIBLY BLOCKS PLATELET AGGREGATION to prevent thrombosis. Platelet aggregation inhibition is reversible after cessation of the IV administration of the drugs. • Abciximab (Reopro), Eptifibatide (Integrilin), Tirofiban (Aggrastat). Conditions Managed by Anticoagulants (Warfarin/Coumadin) & Anti-platelet Agents (Aspirin & Clopidogrel (Plavix): 1. Coronary Artery Disease (CAD) by helping prevent threat of myocardial infarction in these patients. 2. Angina Pectoris (Unstable Angina) by preventing a thrombus from forming in the coronary arteries. 3. Myocardial Infarction (MI): drugs that prevent blood clotting prevent the threat of future infarcts. 4. Stroke: helps prevent a thrombus from forming, thus

preventing the threat of a cerebral embolism. Hypertension IS NOT MANAGED with anticoagulants or anti-platelet drugs, unless it is accompanied by the above cardiovascular problems, because these drugs DO NOT LOWER BLOOD PRESSURE. Rheumatoid Arthritis (RA)-a chronic inflammatory disease of the joints causing joint pain, swelling, and destruction. RA is characterized by chronic inflammation of the synovium that lines the joints. With disease progression, there is an accumulation of prostaglandins, leukotrienes, and other mediators in the inflammatory changes and tissue destruction in the synovial lining. These RA drugs (except gold injections) also treat Osteoarthritis (OA), characterized by progressive loss of articular cartilage. This may be the result of excessive loads on the joint or other factors. Agents that treat OA provide an analgesic and anti-inflammatory action to reduce pain within the joint. Anti-Rheumatic Agents: 1. Etanercept (Enbrel)-used to reduce signs and symptoms of active rheumatoid arthritis in patients who have had an inadequate response to one or more disease-modifying anti-rheumatic drugs (DMARDs). It is a recombinant DNA-derived protein that binds to tumor necrosis factor (TNF). TNF plays an important role in the inflammatory processes or rheumatoid arthritis (RA) and

the resulting joint pathology. 2. Infliximab (Remicade)-a chimeric monoclonal antibody that binds to TNF-α to reduce the inflammatory actions of this endogenous compound. Used to treat Crohn’s Disease & Rheumatoid Arthritis. 3. Prednisone-a corticosteroid that decreases the inflammatory response due to its anti-inflammatory actions. Treats a wide variety of inflammatory diseases, including rheumatoid arthritis and osteoarthritis. ♦ Short-term side effects: insomnia, indigestion, and arthralgia. ♦ Long-term side effects: edema (abdominal distension), psychological disturbances, peptic ulcer, osteoporosis, and muscle weakness. 4. Piroxicam (Feldene)-NSAIDs drug that inhibits prostaglandin synthesis. Used to manage inflammatory disorders and for the symptomatic treatment of acute and chronic rheumatoid arthritis and osteoarthritis. Common Adverse Effects: gastric irritation, heart burn, and nausea. 5. Gold Injections-decreases prostaglandin production. 6. Methotrexate-may affect immune function. 7. Nabumetone (Relafen)

Aspirin & NSAIDs (Ibuprofen, Naproxen) inactivate “cyclooxygenase” enzyme. Since COX synthesizes prostaglandins, inhibiting this enzyme inhibits prostaglandin synthesis. NSAIDs are also called COX inhibitors. Thus, NSAIDs have analgesic, antipyretic, and antiinflammatory actions (can treat acute inflammation). NSAIDs must be used cautiously in patients with PEPTIC ULCER DISEASE. • NSAIDs reduce the production of PROSTAGLANDINS associated with pain and inflammation. The mechanism of action of NSAIDs is they inhibit the cyclooxygenase step of the arachidonic acid cascade, thus inhibit the activity of prostaglandin synthetase. Prostaglandins-a group of hormone-like substances that mediate a range of physiological function like metabolism and nerve transmission. NSAIDs are used for pain control, arthritis, and painful menstruation. • NSAID Adverse Effects: GI upset (possible ulcers), prolonged bleeding time (due to reduced platelet aggregation). • NSAIDs Contraindications: patients with impaired renal function, pregnancy, and GI disease (ulcers).

ACETAMINOPHEN & OPIODS Drugs Without Anti-Inflammatory Properties: 1. Acetaminophen (Tylenol)-a WEAK inhibitor of prostaglandin synthesis in peripheral tissues, thus does not greatly affect the inflammatory response. Tylenol reduces pain by non-inflammatory mechanisms. It is unclear exactly how it works to reduce pain. Lacks antiinflammatory effects of aspirin, but is a good analgesic when aspirin or ibuprofen are contraindicated. Better to use in GI, bleeding disorders, asthma, young children, and pregnancy. Less drug interactions than aspirin, but overdose can be hepatotoxic, if mixed with alcohol. Acetaminophen is NOT an NSAID, but an alternative used for patients who cannot tolerate NSAIDS. Disadvantage: no peripheral anti-inflammatory effects since lacks antiinflammatory properties. • Pregnant or nursing females: for mild-to-moderate pain, acetaminophen is the accepted choice for short-term use (long-term effects unknown in pregnant women), and safe for breast feeding, but acetaminophen lacks anti-inflammatory properties).

• Acetaminophen does not hasten the elimination of ibuprofen. The analgesic efficacy of combining acetaminophen and ibuprofen is greater than either acetaminophen or ibuprofen alone. The combination provides effective pain relief. • Acetaminophen is a non-narcotic analgesic that DOES NOT AFFECT PLATELET AGGREGATION or the coagulation pathway. Thus, it does not affect the anticoagulant nature of Warfarin (Coumadin), so is safe in warfarin patients. • Acetaminophen has no effect on platelets nor the coagulation pathways, and does not affect bleeding times, even with high doses. Thus, acetaminophen is a non-narcotic analgesic that is the best choice to relive mild-to-moderate pain in a patient taking anti-coagulant medication (warfarin, heparin) because it does not effect blood clotting. • Acetaminophen’s 2 Major Pharmacological Actions: analgesic & antipyretic (fever reducer) effect. While it is not effective enough to reduce severe pain, it is effective in reducing mild-tomoderate pain. Acetaminophen is a weak inhibitor of prostaglandin formation. Thus, for mild-to-moderate pain, use acetaminophen (never NSAID’s aspirin or ibuprofen).

• Acetaminophen has little value in treating acute inflammation. Acetaminophen inhibits CENTRAL prostaglandin synthesis (it is an analgesic for low intensity pain and antipyretic). Because it is less effective than salicylates (aspirin) in blocking peripheral prostaglandin synthesis, it does not have anti-inflammatory activity and does not affect platelet function, thus does not affect clotting time. In large doses, acetaminophen can cause hepatic necrosis. • Large doses of acetaminophen can cause liver toxicity. Alcohol can seriously increase the hepatotoxic potential of acetaminophen. There are ~100 deaths annually due to liver toxicity caused by ingesting large continuous doses of acetaminophen. However, it rarely causes drug sensitivities, and can be given to patients will an allergy to aspirin. • Acetaminophen (Tylenol) is the only OTC noninflammatory analgesic commonly available in the U.S. It is a weak cyclooxygenase inhibitor in peripheral tissues, thus accounting for its lack of antiinflammatory effect. It may be a more effective inhibitor of prostaglandin synthesis in the CNS, resulting in analgesic and antipyretic action. Acetaminophen is a relatively safe ANTIPYRETIC drug with NO INFLAMMATORY ACTION.

• The combination of acetaminophen and propoxyphene (Darvocet-N or Wygesic) treats moderate-to-severe pain due to dental procedures. *Propoxyphene (Darvon) is an oral synthetic opioid analgesic structurally similar to Methadone. Darvon compound-65 is a combination of aspirin, caffeine, and propoxyphene. • Acetaminophen does NOT effect clotting time (unlike aspirin). It does not have significant anti-platelet effects. It is effective for the same indications as an intermediate-dose aspirin, thus is a useful aspirin substitute, especially in children with viral infections who are at risk for Reye’s Syndrome if they take aspirin.

2. Combination Analgesics: narcotic analgesics (pain killers) that effectively REDUCE PAIN (not inflammation) by working in the brain to block ascending pain impulses that travel from the periphery (PNS) into the brain (CNS). Codeine & hydrocodone are narcotic analgesics that can be given in combination with ibuprofen and other NSAIDs to provide effective analgesia. The narcotics work in the brain (CNS), while

ibuprofen and other NSAIDs work in peripheral tissues (PNS). These two different mechanisms complement each other to provide effective pain reduction. Narcotic analgesics DO NOT AFFECT BLOOD CLOTTING, thus do not enhance the anticoagulant effects of Warfarin (Coumadin). Narcotics with acetaminophen can be given safely to Warfarin patients. Codeine-codeine preparation is a LESS efficacious opioid analgesic with moderate abuse liability. Codeine is a narcotic analgesic and antitussive (cough suppressant), that is weaker than morphine, less addictive, and less constipating. Codeine is usually combined with other drugs (Empirin, Tylenol #2, #3, #4) • Codeine + Aspirin (Empirin)-an analgesic/antiinflammatory. Avoid in asthmatics and patients who cannot take aspirin as codeine precipitate acute asthma attacks. • Codeine + Acetaminophen/Tylenol (Tylenol 3)poor anti-inflammatory agent, but better than Empirin if the patient has GI problems, pregnant, or patients who cannot take aspirin. Hydrocodone (Vicodin, Lorcet, Norcet, Lortab)-a combination analgesic more efficacious than codeine, but potentially greater abuse liability. Avoid in asthmatics. Has poor anti-inflammatory, but good

analgesic (stronger than Tylenol). • Hydrocodone + Ibuprofen-a strong analgesic for moderate-to-moderately severe pain, with good anti-inflammatory properties. Ibuprofen has similar contraindications to aspirin, with GI problems being the most common side effect (take with milk or food to limit). Better for patients who cannot take aspirin or ibuprofen. • All combine a narcotic analgesic and cough reliever (hydrocodone) with a non-narcotic analgesic (acetaminophen) to relieve moderate-to-severe pain. Oxycodone-a combination analgesic that is more efficacious than codeine, but avoided in asthmatics. Treat moderately-to-severe pain. Oxycodone has the HIGHEST DEPENDENCY LIABILITY when compared to drugs like codeine, propoxyphene, and pentazocine. Oxycodone is contained in Percodan, Percocet. • Oxycodone + Aspirin (Percodan)-good analgesic with anti-inflammatory properties. The strongest pain medication you can prescribe on an outpatient (ambulatory) basis. Do not take it on an empty stomach. • Oxycodone + Acetaminophen (Percocet)-better if

the patient cannot take aspirin or ibuprofen, but lacks strong anti-inflammatory effects. The body contains 3 naturally occurring endogenous opioids peptides that produce morphine-like effects to reduce pain. Opioid receptors are activated by endogenous chemicals under physiologic conditions. 1. Beta-endorphins-bind to opioid receptors in the brain and have potent analgesic activity. 2. Enkephalins-bind to opioid DELTA receptors in the brain and are more widely distributed in the brain than beta-endorphins. Play a role in pain perception, movement, mood, and behavior. 3. Dynorphins-the MOST POWERFUL opioids found throughout the CNS & PNS that bind to KAPPA receptors. Dynorphins may regulate pain at the spinal cord level, influence behavior at the hypothalamic level, and function with other endogenous opioids to regulate the cardiovascular system. OPIOID RECEPTORS: 1. Mu (μ)-the prototypical opioid agonist for this receptor is MORPHINE (its analgesic activity depends on its binding to the mu receptor). The supraspinal analgesic activity of morphine is mediated primarily through

its influence on the Mu opiod receptor. 2. Delta (δ)-enkephalins are the typical agonist for the delta receptor. 3. Kappa (κ)-dynorphins are the typical agonist for the kappa receptor. OPIOIDS (NARCOTICS)-drugs WITHOUT ANTIINFLAMMAOTRY PROPERTIES used as very effective analgesics to relieve moderate-to-severe pain, antitussives, antidiarrheals, pre-anesthetic medications, and as analgesic adjuncts during anesthesia. • Opioids suppress the cough reflex (antitussive), cause constipation (antidiarrheal), and when used as pre-anesthetic medications, opiates reduce the amount of general anesthetic required for surgical anesthesia. Opiates are administered with caution to patients with HEAD INJURY or with a history of drug abuse and dependency. • Opioids are the most powerful drugs available for pain relief. Strong agonists (those with the highest analgesic efficacy) are Morphine, Meperidine, Fentanyl, and Methadone. • Codeine, Hydrocodone, and Oxycodone are mild-to-

moderate agonists (analgesic efficacy). • The most appropriate time to administer the initial dose of an analgesic to control post-operative pain is BEFORE the effect of the local anesthetic wears off. • Opioid Analgesics (i.e. morphine, codeine, meperidine, propoxyphene) MIMIC the body’s endogenous opioids at CNS opiate receptors to raise the pain threshold and increase pain tolerance. Opioids also cause chemoreceptor trigger zone stimulation and decrease alpha-adrenergic receptor responsiveness. • Common Side Effects: sedation and drowsiness (by depressing the conscious centers of the brain), dizziness, & nausea. The MOST common side effect of the narcotic (opiate) analgesics is NAUSEA. Narcotic analgesics DO NOT cause peptic ulcers or insomnia. • Less Common Adverse Effects: vomiting, hypotension, irregular/labored breathing (dyspnea), lightheadedness, nightmares, and insomnia. *Respiratory depression is dose related and causes death in narcotic drug overdose. It can occur with any of the narcotics. Respiratory depression is the MAJOR disadvantage of using opioids and is

the most significant and well-known adverse reaction. Death secondary to opioid overdose is usually always due to respiratory depression. When opioids are used correctly, the risk of severe respiratory depression is small as tolerance rapidly develops to this effect. The MOST SERIOUS side effect is respiratory depression. The cause of death from overdose of narcotics is respiratory depression and shut down of the respiratory system. • Abuse can and does occur with all narcotics. Narcotics are controlled substances under DEA Schedules II or III. A DEA number/license is required to prescribe narcotics/opioids like Hydrocodone (Vicodin), Meperidine (Demerol), and Codeine. Opiates are contraindicated in patients with severe head injuries, but not for patients with bronchial asthma, renal dysfunction, or acute myocardial infarctions. Morphine (opiates)-the standard drug to which all opiates (narcotics) are compared. Morphine influences the MU OPIOID receptor subtype. • Morphine Pharmacological Effects: analgesia, drowsiness, euphoria, mental clouding, miosis (pupillary constriction), constipation, nausea, vomiting, and respiratory depression. Morphine is an opium alkaloid.

• Length of Effectiveness: IV or IM (2-3hrs), oral (34hrs). Sustained release is 8-12hrs. • Morphine starts to work quickly. The oral form of morphine can be very effective for cancer pain. • Synthetic Morphine Derivatives: Hydromorphone (Dilaudid), Oxymorphone (Numorphan), & Nalbuphine (Nubain). • MORPHINE IS NOT USED IN DENTISTRY DUE TO ITS HIGH ADDICTIVE LIABILITY. Pentazocine (Talwin)-equally as strong as codeine. Chemically related to morphine, but has weaker analgesic properties. Given orally it lasts up to 4hrs. Not used intravenously to produce conscious sedation. Can block painkilling action of other opiods. Can cause confusion and anxiety, especially in elderly patients. Has abuse liability. Talwin compound combines the strong analgesic properties of Pentazocine and the analgesic, anti-inflammatory, and fever-reducing properties of aspirin. It is used to relieve MODERATE PAIN, and does not produce euphoria. Codeine-an opium alkaloid weaker (less potent) & less addictive opioid than morphine (a mild-to-moderate agonist). When combined with acetaminophen it is (Tylenol #3). Length of Effectiveness: by mouth (3-4hrs) taken with

aspirin or acetaminophen. Synthetic Codeine Derivatives: ♦ Oxycodone (OxyContin)-an opiate analgesic with similar potency to morphine. When combined with acetaminophen (Roxicet, Percocet, Percodan, & Tylox). Given orally it lasts 3-4hrs. Usually combined with aspirin or acetaminophen. Adverse Effects: nausea & constipation. It is a mild-to-moderate agonist. ♦ Hydrocodone-similar potency as morphine. When combined with acetaminophen it is called (Vicodin, Lorcet, Lortab, Maxidone, & Zydone). Given orally it lasts 4-6hrs. Usually combined with acetaminophen. When combined with ibuprofen it is (Vicoprofen). Among the opiates available for dentistry, hydrocodone products are the drugs of choice. It is a mild-to-moderate agonist. Meperidine (Demerol)-a SYNTHETIC narcotic (opioid) agonist that is weaker than morphine, but equally addictive. It’s duration of action is shorter than morphine, and it is the only narcotic agent that does not cause miosis (pupillary constriction). Demerol is used as an IV supplement during conscious sedation procedures, but is less potent than morphine and much less potent than fentanyl. Demerol is also used as an oral medication to control pain after dental surgery. Demerol treats moderate-to-severe pain (acute dental pain) and may be used as a preoperative

medication to relieve pain and allay anxiety. • Length of Effectiveness: IV or IM (3hrs) and not very effective given orally. • When combined with promethazine it is know as Mepergan Fortis. • Meperidine can cause seizures, tremors, and muscle spasms. • Synthetic Meperidine Derivatives: ♦ Fentanyl (Sublimaze)-a synthetic potent opioid analgesic used primarily as an IV sedative during conscious sedation procedures or procedures requiring general anesthesia. It is 80-100x more potent than morphine. In a fixed combination with the neuroleptic drug Droperidol, fentanyl is available as (“Innovar”). • Innovar-produces a syndrome calleda “neurolepanalgesia”. The neuroleptic action (major tranquilizing action) is produced by the Droperidol component, and the analgesic action by the Fentanyl component. • Fentanyl is also available as a lollipop-type lozenge (“Actiq”) for transmucosal absorption and as a transdermal patch (“Duragesic”) for delivery through a patch applied to the skin. ♦ Alphaprodine (Nisentil) & Alfentanil (Alfenta) ♦ Sufentanil (Sufenta) ♦ Diphenoxylate (in Lomotil) & Loperamide (in Imodium)

Methadone (Dolophine)-length of effectiveness given orally is 4-6hrs or longer. Also treats HEROIN WITHDRAWL. Propoxyphene (Darvon)-synthetic opioid derivate of Methadone, that in the form of propoxyphene napsylate with acetaminophen is called “Darvocet-N 100” (low abuse liability). Used for MILD pain control after dental surgery. Taken orally (not IV). Length of effectiveness given orally is 3-4hrs. Taken with aspirin or acetaminophen to treat mild pain. Propoxyphene is a very weak agonist drug. Levorphanol-lasts 4 hours given by IV, IM or orally. The oral form is strong and can be used instead of morphine. Narcotics (Opiates) Used in Dentistry for pain relief after dental surgery: Hydrocodone (Vicodin), Oxycodone (Percocet, Tylox), Meperidine (Demerol), Codeine, & Fentanyl. Codeine, Morphine, & Meperidine (Demerol) are analgesics that can produce drug-dependence and addiction. These are narcotic analgesics since the actions of this opioid is to produce DROWSINESS & SLEEP as a side effect. Opioid (Narcotic Analgesic) Antagonists: these agents reverse respiratory depressive effects caused by an overdose of Codeine, Morphine, Hydrocodone, Oxycodone, Meperidine (Demerol), & Fentanyl.

1. Naloxone (Narcan)-the prototype/pure opioid/narcotic competitive antagonist given IV, IM, or subcutaneously in medical emergencies to reverse narcotic overdose which results in respiratory depression and death due to respiratory shut down. Naloxone reverses the respiratory depressant effects of narcotics to counteract their lethal effects. Naloxone is the antagonist of choice to treat opioid overdose. 2. Nalmefene (Revex)-narcotic reversal agents that reverses respiratory depressive effects of narcotic analgesics. 3. Naltrexone (ReVia)-narcotic reversal agent that reverses respiratory depressive effects of narcotic analgesics and is used to treat alcohol dependence. Opiates produce drug dependence leading to addiction. Psychic dependence, physical dependence, and tolerance can develop upon repeated administration. Addiction-a compulsive, uncontrollable dependence on a substance, habit, or practice to such a degree that cessation causes severe emotional, mental, or physiological reactions. Habituation-an acquired tolerance from repeated exposure to a particular stimulus. Psychological and

emotional dependence on a drug, tobacco, or alcohol result from the repeated use of the substance, but without the addictive, physiological need to increase dosage. Tolerance-the phenomenon of decreased responsiveness to a drug after chronic administration. As a patient becomes tolerant to a drug, the dosage required to produce the usual effect is increased. Occurs when increasingly large doses of opiate are required to produce the same degree of analgesia. Psychic dependence-unlikely if an opiate is taken for a short period for pain relief. Physiological dependence is common to all forms of drug dependence and abuse. These drugs of abuse all have the ability to change a person’s mood and sensory perception. Physical dependence-a condition in which continued drug administration is required to prevent unpleasant withdrawal symptoms. Toxicity-a condition that results from exposure to a toxin or toxic amounts of a substance that does not cause adverse effects in smaller amounts. ♦ Toxicity is both dose-dependent and timedependent. ♦ Toxicity is often an extension of the desired drug

effect. ♦ Toxicity can range from nausea to death, and can be caused by even minimal concentrations of a drug. A drug with a high LD50 and low ED50 has a HIGH therapeutic index, thus is relatively SAFE. The purpose of an acute toxicity test is to determine the nature and extent of the untoward reactions that might follow the administration of a single dose (or an overdose) of a drug. A quantitative aspect of acute toxicity testing is to determine a drug’s lethal dose (LD) which is expressed as LD50. Standing alone, it conveys less information than the ratio of the lethal to the effective doses (LD50: ED50), a quantity that is called the “Therapeutic Index”. The greater a drug’s therapeutic index, the LESS likely fatalities will follow an accidental overdose. In the ideal situation, the therapeutic index would be 100, but this never happens. The ratio LD50: ED50 gives a drug’s SAFETY. • Therapeutic Dose-the amount of a drug that usually lies between the minimal and maximal doses of the drug. • Effective Dose (ED)-the effective dose at which 50% of people will respond. • Lethal Dose (LD)-the dose that kills 50% of the people who receive the drug, as determined by mice

experimentation. • Fatal Dose-a drug dose that kills. Drug Efficacy (“intrinsic ability”, “maximal” or “ceiling effect”)-the ability of a drug to produce a desired therapeutic effect regardless of dosage. Efficacy is the maximum effect a drug can cause, regardless of the dose. Drug Potency-the relative concentration of two or more drugs that produce the same drug effect. The effect usually chosen is 50% of the maximal effect and the dose causing this effect is the EC50. Potency is mainly determined by the affinity of the receptor for the drug. The smaller the EC50, the greater drug potency. Potency is a comparative term (one drug is more potent than another drug). ♦ Ex: Drug A in a dose of 10mg produces the same magnitude of response as Drug B in a dose of 50mg. The following is true: Drug A is 5x as potent as Drug B, but Drug A is not potent in and of itself. Also, if Drug A has a greater efficacy than Drug B, then Drug A is capable of producing a greater maximum effect than Drug B. ♦ When comparing drugs with respect to intensity of response, the drug that produces the greatest maximum effect is the drug with the highest efficacy.

Additive Effect-occurs when two drugs with similar effects are administered in combination, and the resulting response is the sum of the individual actions of each drug when given alone. An additive effect occurs when additive drugs are administered. The response is no greater than that which would be expected had the drugs been given one at a time. There is no enhancement of potential of the individual drugs as a result of being used in combination. Antagonistic Effect-occurs when the combined actions of two drugs with similar pharmacological effects are LESS than the sum of the drug’s individual effects. The effect of aspirin and ibuprofen (Motrin) in combination is less than the sum of their individual actions. Cumulative Action-an excessive accumulation effect that occurs if a drug is administered repeatedly and a higher concentration of the drug than is desired may be achieved. Idiosyncrasy-a response to a drug that is unusual or abnormal, or one that grossly deviates from the routine reaction. Synergistic Response-occurs when the combined action of two drugs with similar pharmacological effects is GREATER than the sum of the individual actions. Alcohol is synergistic with the valium-family (valium/diazepam, xanax, halcion), narcotics, and

barbiturates. Alcohol must be avoided when taking these medications. 4 Criteria to Select an Analgesic Agent for a Patient: 1. Type of Pain: severe, mild, moderate 2. Patient’s Age: • Infant & child: when calculating dosage, height, weight, body surface area, and renal hepatic function must be considered. • Adult • Elderly: drug response is affected by age-related changes in physiology and pharmacokinetics. 3. Concurrent Medications: consider unwanted drug interactions (especially with elderly). 4. Pregnancy: because virtually any drug a pregnant women takes can cross the placenta and enter fetal circulation, drug use in pregnant patients is a source of special concern. Thus, check with the patient’s OB-GYN prior to prescribing an analgesic. MERCURY-can cause prominent toxic effects like irritability, excessive saliva, loose teeth, gum disorders, slurred speech, and tremors. These symptoms are all CHRONIC, caused by inhalation of dust vapors of mercurial compounds, or from repeated ingestions of very small

amounts. Mercury absorbed into the circulatory system may be deposited into any tissue. Higher-than-average accumulations occur in the brain, liver, and kidney. Mercury does not collect irreversibly in human tissues. It has an average half-life of 55 days for transport through the body to the point of excretion. Thus, mercury that came into the body years ago, is no longer present in the body. The presence of mercury in the body is determined by a URINE TEST. • Treatment: gastric lavage with milk and egg white or sodium bicarbonate, fluid therapy, & chelation with Dimercaprol (British Anti-Lewisite/BAL) which forms a stable complex with mercury, allowing it to be excreted as an inactive compound. GROWTH HORMONE (GH) (Somatotropin)-produced by and secreted from the ANTERIOR PITUITARY GLAND. Somatotropin’s basic metabolic effects include: • Increased rate of protein synthesis in all body cells and increased mobilization of fats and the use of fat for energy. • Decreased rate of carbohydrate utilization throughout the body. Human growth hormone (HGH) is prepared commercially for use as replacement therapy in patients with growth

hormone deficiency. This commercial preparation is prepared as the purified polypeptide hormone of recombinant DNA origin with the same amino acid sequence that is produced by the pituitary gland. • HGH is indicated for children to treat growth failure due to lack of adequate endogenous growth hormone secretion. It has been used in adults who have a growth hormone deficiency due to pituitary disease. • HGH is administered as a subcutaneous injection or intramuscular (IM) injection 3x per week.

Oral Contraceptives (Birth Control)-block ovulation by inhibiting the anterior pituitary hormones FSH (Follicle Stimulating Hormone) & LH (Luteinizing Hormone). Oral contraceptives also produce alterations in the genital tract, including changes in cervical mucus, rendering it unfavorable for sperm penetration even if ovulation occurs. Changes in the endometrium may also occur, rendering it unfavorable for nidation (implantation of the fertilized ovum). • Oral contraceptives usually contain BOTH an estrogenic agent (ethinyl estradiol or mestranol) and progestational agent (levonorgestrel, norethindrone,

norgestimate, and norgestrel). • Oral Contraceptive Precautions: the risk of cardiovascular side effects increases in women who smoke cigarettes, especially women over age 35. The risk of a thromboembolism also increases. Women with hypertension are encouraged to use a non-hormonal form of contraception. The highest risk associated with use of oral contraceptives is thromboembolic disorders. • Antibiotics have the potential to reduce the effectiveness of oral contraceptives. Thus, advise patients to use additional methods of birth control when taking antibiotics and oral contraceptives concurrently. PAIN: 1. Pain Threshold-the LOWEST level of pain a patient will detect. 2. Phantom Pain-pain that has no organic basis and is fixed upon some portion of the anatomy (i.e. the sensation of pain felt in a limb, although the limb has been amputated). 3. Psychogenic Pain-pain produced or caused by psychic or mental factors, not organic factors.

4. Intractable Pain-pain that is resistant or refractory to ordinary analgesic agents. 5. Referred Pain-pain felt in an area other than the site of origin, like pain near the shoulder associated with biliary disease. Drugs that cause XEROSTOMIA by inhibiting SALIVA production & secretion. The xerostomia action produced by these drugs is reversible as normal salivary flow is regained after discontinuing the drug. 1. Amitriptyline (Elavil)-a tricyclic antidepressant (a drug class that causes significant xerostomia). They probably work through an anticholinergic action. 2. Diphenhydramine (Benadryl)-a sedating-type antihistamine (a drug class that causes significant xerostomia). They probably work through an anticholinergic action. 3. Atropine-a powerful anticholinergic that blocks saliva production in the salivary glands. Other anticholinergics have a similar action. 4. Diazepam (Valium)-a benzodiazepine tranquilizer that has moderate anticholinergic action to reduce the outflow of saliva.

Drugs that Control Salivary Secretions Sialagogues) to help obtain a dry field:

(Anti-

• Atropine Sulfate, Glycopyrrolate (Robinul), Belladonna derivatives, & Propantheline Bromide (Pro-banthine). • Anti-Sialogogues are anti-cholinergics; they block post-ganglionic cholinergic fibers. • Anti-Sialogogues Contraindications: patients with glaucoma, cardiovascular problems, GI tract obstruction, and asthma. • Anti-sialogogues also REDUCE SPASMS of smooth muscle in the bladder, bronchi, and intestine. They relax the iris sphincter, decrease gastric, bronchial, salivary secretions, and perspiration. They accelerate impulse conduction through the myocardium by blocking vagal impulses. Drugs that treat GLAUCOMA (an increase in intraocular pressure caused by poor drainage of the aqueous humor (fluid in the eye) and can cause blindness. 1. Pilocarpine (Isopto-Carpine or Salvogen)-eye drops that cause causes papillary constriction to allow aqueous humor drainage which reduces intraocular

pressure. 2. Latanoprost (Xalatan)-a prostaglandin analog. Eye drops reduce intraocular pressure by increasing aqueous humor outflow/drainage. 3. Betaxolol (Betoptic)-a beta blocker. Eye drops reduces intraocular pressure by reducing aqueous humor production. 4. Bimatoprost (Lumigan)-eye drops reduces intraocular pressure by increasing aqueous humor outflow/drainage.

CANCER CHEMOTHERAPY (ANTINEOPLASTIC DRUGS) 8 Classes of Chemotherapy Drugs: 1. Alkylating Agents-are most effective in treating chronic leukemias, lymphomas, myelomas, and carcinomas of the breast & ovary. These agents alkylate DNA so it cannot replicate. Alkylating agents contain a diverse group of compounds that form alkyl bonds to nucleic acids. All alkylating agents share a similar mechanism of action and mechanism of resistance, and form covalent bonds with nucleic acids and proteins. N-7 position of GUANINE is a common bonding site. Major Alkylating Agents: • Nitrogen Mustards: Mechlorethamine (Mustargen), Cyclophosphamide (Cytoxan), Chlorambucil (Leukeran), & Melphalan (Alkeran). Treats Hodgkin’s Disease & other lymphomas, and Chronic Lymphocytic Leukemia. • Nitrosoureas: Carmustine (BiCNU), Lomustine (CeeNU), & Semustine (Methyl-CeeNU). Treats Hodgkin’s disease, other lymphomas, and Myeloma. • Bisulfan (Myleran)-treats Chronic Granulocytic

Leukemia. • Cisplatin (Platinol)-a chemotherapeutic drug (Alkylating Agent) with adverse effects of nausea and vomiting (75%-100%; dose-related), alopecia, xerostomia, and changes within the oral cavity tissues (i.e. mucositis). • Mucositis-a common reaction to cancer chemotherapy involving inflammation of the mucous membranes. During chemotherapy and radiation therapy, mucosal tissues begin to desquamate and ulcerate. The mucosal integrity is broken and is secondarily infected by oral flora. Palliative treatment is indicated for mucositis. 2. Anthracyclines-these agents destroy DNA so the cell cannot replicate. Daunorubicin & Doxorubicin. Commonly associated with the development of oral mucosititis. 3. Antibiotics-these antibiotics are not used for antibacterial therapy, but are specifically designed for cancer chemotherapy. Dactinomycin. 4. Antimetabolites-cell cycle-specific drugs that act primarily in the “S” Phase of the cell cycle (DNA synthesis). Antimetabolites interfere with selected biomechanical reactions necessary for cell growth.

Antimetabolites are one of the oldest and most important classes of anti-neoplastic agents. Antimetabolites attack cells during the “S” period of the reproduction cycle by interfering with the biosynthesis of the PURINE & PYRIMIDINE BASES. 1. Folic Acid Analogs: Methotrexate (Amethopterin). Treats acute lymphoblastic leukemias in children. Adverse Effects: commonly associated with the development of oral mucosititis. 2. Pyrimidine Analogs: 5-Fluorouracil (5FU), Floxuridine (FUDR), Cytosine Arabinoside (Cytosar-U), 6-Mercaptopurine, & Methotrexate. Treats carcinoma of the breast, GI tract, ovary, cervix, and prostate. Induction of remission in acute leukemia in children and adults. Adverse Effects: commonly associated with the development of oral mucosititis. 3. Purine Analogs: Mercaptopurine (Purinethol) & Thioguanine (TG). Treats leukemia in children and adults. Acute leukemia and induction or remissions in acute granulocyctic leukemia.

5. Antimicrotubulars-affect microtubular assembly with cells to inhibit cell mitosis. Paclitaxel (Taxol). 6. Antiestrogens-block the tumors on which estrogen has a stimulatory effect. Tamoxifen (Nolvadex). 7. Vinca Alkaloids-these are mitotic spindle poisons. Vinblastine & Vincristine. 8. Gonadotropin Hormone-Releasing Antigens-these agents inhibit gonadotropin secretion (an action effective in reducing certain carcinomas). Leuprolide. Anti-cancer drugs Asparaginase (deprives tumor cells of certain amino acids to protein production is blocked) & Interferons (boost the immune system) do not fall within any category, but do treat certain cancers. Alopecia (hair loss)-the most common complication seen with chemotherapy treatment. Alopecia occurs with administration of most chemotherapeutic agents 1-2 weeks after treatment. GI upset, increased incidence of infection (especially Candidiasis), and degeneration of lymphatic tissue are other common site effects of chemotherapy. Most chemotherapy drugs are TERATOGENIC in humans, and should be avoided in pregnant women.

HYPOGLYCEMICS (ANTI-DIABETICS) HYPOGLYCEMIA-the most serious and common complication of insulin therapy. Symptoms: sweating, weakness, confusion, slurred speech, & blurred vision. Administration of a concentrated glucose source relieves mild hypoglycemia. • Humulin-the brand name for the human form of insulin. INSULIN-a pancreatic hormone secreted by pancreatic beta-cells of ISLETS of LANGERHANS. Insulin is essential for glucose metabolism and for homeostasis of blood glucose. Insulin is usually administered by subcutaneous injection. Various insulin preparations are prepared from beef or pork pancreas, and differ mainly in their onset and duration of action. Effects of Insulin: ↓ gluconeogenesis & ↑ triglyceride storage, glycogen synthesis, & protein synthesis. Insulin preparations mimic the activity of endogenous insulin, which is required for the proper utilization of glucose in normal metabolism. Insulin preparations are used in Type 1 & Type 2 diabetes that cannot be completely controlled by the oral anti-diabetic drugs or by diet alone. Insulins differ in their onset and duration of

action: 1. Ultra-rapid acting insulin (Insulin Lispro): peak onset of action is .25-.5hrs. Duration of action is 3-4hrs. 2. Short/Rapid-acting Insulin-peak onset of action of .53hrs, with a duration of action of 8-12hrs. Ex: Regular Insulin Injection & Prompt Insulin Zinc Suspension (Semilente Insulin). 3. Intermediate-acting Insulins-peak action 8-12hrs. Duration of action of 18-24hrs. Insulin Zinc Suspension (Lente Insulin) & Isophane Insulin Suspension (NPH). 4. Long-acting Insulins-duration of action is > 36hrs. Protamine Zinc Insulin (PZI) and Ultralente Insulin. Anti-Diabetics (Oral Hypoglycemics): Glyburide (DiaBeta), Metformin (Glucophage), Pioglitazone (Actos), Chloropropamide (Diabinese), Tolbutamide (Orinase). These drugs are used as adjuncts to diet to treat noninsulin dependent diabetes mellitus (Type 2 Diabetes) that cannot be controlled by diet alone. Glyburide & Chloropropamide-stimulate insulin release from the pancreas, and work by reducing glucose output from the liver and by increasing insulin sensitivity at peripheral target sites.

Metformin & Pioglitazone-increase insulin sensitivity at peripheral target sites. Tolbutamide-a sulfonylurea that stimulates the synthesis and release of insulin from the pancreas, increases the sensitivity of insulin receptors, and improves peripheral utilization of insulin.

REMOVABLE PARTIAL DENTURES (RPDs) Kennedy Classifications-based on the MOST POSTERIOR EDENTULOUS AREA to be restored. Although Class III & IV RPDs are entirely supported by abutment teeth, Class I & II RPDs are supported by abutment teeth, the residual ridges, subjacent tissues, and fibrous C.T. overlying the alveolar process. Alveolar ridge resorption under the distal extension RPD is a concern, but is reduced by maximizing coverage of these supporting areas. Periodontal damage to abutment teeth is avoided with firm tissue support (maintaining a stable basetissue relationship). 1. Class I: bilateral edentulous areas posterior to the natural teeth. BILATERAL DISTAL EXTENSION. 2. Class II: unilateral edentulous area posterior to remaining natural teeth. UNILATERAL DISTAL EXTENSION. Kennedy Class I & II: must have a MESIAL REST on the abutment next to the posterior edentulous space. 3. Class III: unilateral edentulous area with natural teeth both anterior and posterior to it. A tooth-borne RPD

because it depends entirely on abutment teeth for support . 4. Class IV: a single, but bilateral (it must cross the midline), edentulous area anterior to the remaining natural teeth. Anterior teeth are missing and across the midline. DOES NOT HAVE MODIFICATION SPACES. A tooth-borne RPD type because it depends entirely on abutment teeth for support . • OCCLUSAL RESTS ARE PLACED ON THE DISTAL OF THE FIRST PREMOLARS! Applegate’s Rules for Applying Kennedy Classification: 1. Rule 1: classification is done AFTER extractions are done. 2. Rule 2: if a 3rd molar is missing and will not be replaced, it’s NOT part of the classification. 3. Rule 3: if a 3rd molar is present and not used as an abutment, it’s NOT part of the classification. 4. Rule 4: if a 2nd molar is missing and will not be replaced, it’s not considered in the classification. 5. Rule 5: Most posterior area always determines the classification. 6. Rule 6: edentulous areas other then those determining the classification are “modifications”. 7. Rule 7: the extent of the modification is not considered, only the number of additional edentulous areas. 8. Rule 8: NO modification areas in a Kennedy Class IV.

MAJOR & MINOR connectors MUST BE RIGID to evenly distribute functional stresses applied to the RPD throughout the mouth. MAJOR CONNECTORS-connects components between both sides of the arches. Major connector must be RIGID so stresses applied to any area of the denture are effectively distributed over the entire supporting area. • Major connector is free of movable tissues and does not impinge on gingival tissues. Relief should be provided. Bony and soft tissue prominences are avoided during placement and removal. • Major connectors most frequently encounter interferences from lingually inclined mandibular premolars. Minor Connectors: a RIGID component that connects the major connector (RPD base) to the other components (direct retainers, indirect retainers). Also transfers functional stress to abutment teeth, and transfers the effects of the retainers, rests, and stabilizing components to the rest of the denture (“abutment-to-prosthesis” function). • 1.5mm thick, preserves tissue (does not impinge marginal gingiva) and is highly polished. • Maximum gingival exposure = joins at right angle. • Minimum of 5mm space between vertical component.

MANDIBULAR RPD MAJOR CONNECTORS Basic cross-section form is HALF-PEAR SHAPED located above moving tissues but as far below the gingival margins as possible. To determine which mandibular major connector to use, measure from the height of the floor to the lingual gingival margins. 1. LINGUAL BAR: superior border must be at least 4mm below the gingival margins (tooth-tissue junction) to prevent plaque collection and margin inflammation. There must be at least 7mm of space/clearance between the gingival margin and mouth floor. HALFPEAR SHAPED in cross-section. • Indication: Used when sufficient space exists between the slightly elevated alveolar lingual sulcus and lingual gingival tissues. Must be a minimum of 7mm vertical height between the gingival margin and mouth floor (inferior border of the bar). 3mm space between superior border of the bar and gingival margin + the bar must be 4mm wide = 7mm. • Contraindication: when severely tipped premolars and molars are present, an alternate framework or

crowns are recommended.

2. SUBLINGUAL BAR: used when there is INSUFFICIENT SPACE for a lingual bar. HALF-PEAR SHAPED. • Used when the height of the mouth floor is < 6mm from the free gingival margins or when it is desirable to keep the free gingival margins of the remaining anterior teeth exposed, and there is inadequate depth of the mouth floor to place a lingual bar. • Its bulkiest portion is to the lingual and it’s tapered is toward the labial. • Bar’s superior border MUST be at least 3mm below the free gingival margin. • Bar’s inferior border is at the height of alveolar lingual sulcus when the patient’s tongue is slightly elevated. • Requires a FUNCTIONAL IMPRESSION. • Contraindication: remaining natural anterior teeth

are severely lingually tilted.

3. CINGULUM BAR (CONTINUOUS BAR): • Used when a lingual plate or sublingual bar is indicated, but the axial alignment of the anterior teeth is such that excessive block-out of inter-proximal undercuts is required. • Contraindications: anterior teeth severely tilted lingually, wide diastemas between anterior teeth causing the metal cingulum bar to be displayed. • It’s a thin, narrow (3mm) strap located on the cingula of anterior teeth, scalloped to follow inter-proximal embrasures with its superior borders tapered to tooth surfaces. Originates bilaterally from rests of adjacent abutments.

4. LINGUOPLATE: Indications: • High floor of the mouth (< 7mm vertical height) or high lingual frenum. Preferred over a lingual bar when there is NO space in the floor of the mouth. • INOPERABLE lingual mandibular tori that cannot be removed. • Anticipated loss of one or more of the remaining teeth. • Used in Class I designs where residual ridges have undergone excessive vertical resorption. • Used to stabilize periodontally weakened teeth (splinting) or lingually tilted mandibular incisors. • Used when future replacement of 1 or more incisors is facilitated by adding retention loops to an existing linguoplate • Used to avoid gingival irritation or food entrapment, or to cover generously relieved areas that would irritate

the tongue. • Extends to the rests on the terminal abutments, to the contacts inter-proximally, and covers anterior cinguli. Do not use if there are wide open anterior contacts because it is un-esthetic or if anterior overlapping exists. • Superior border is at the middle 1/3 of the teeth’s lingual surface and extends upward to cover interproximal spaces to the contact point. • Mandibular lingual tori require a linguoplate because there is often not 7mm of vertical space for a lingual bar. Tissue covering the tori is thin and cannot tolerate vertical pressure from the major connector. DO NOT USE IF SEVERE ANTERIOR CROWDING EXISTS. 5. LABIAL BAR: • Indications: when severe lingual inclinations of remaining premolars & incisors cannot be corrected orthodontically, preventing placing a lingual bar. When severe lingual tori cannot be removed and prevent using a lingual bar or plate. Also used when severe and abrupt lingual tissue undercuts prevent using a lingual bar or plate. • Superior border is at least 4mm below the labial and buccal gingival margins. • Inferior border is in the labial-buccal vestibule at junction of attached (immobile) & unattached (mobile) mucosa.

• UNLESS TORI SURGERY IS ABSOLUTELY CONTRAINDICATED, interfering tori are removed to avoid using a labial bar. • Trauma and congenital deficiencies occasionally produce dental arrangements where only a labial connector is feasible.

6. DOUBLE LINGUAL BAR (WITH CONTINUOUS BAR): • Placed above the cingula and below inter-proximal contacts. • NEED 7-8mm above the mouth floor or cannot use. • Best indicated for PERIOSURGERY CASES FOR WIDE EMBRASURES. • Must have rests on the superior bar on at least the canines.

MAXILLARY MAJOR CONNECTORS 1. MUST BE RIGID. Superior border MUST be at least 6mm below the free gingival margins and parallel to the mean curve of the free gingival margin. 2. Use metal plating if < 6mm exists from the gingival margins. 3. Posterior border of the major connector must cross the palate at RIGHT ANGLES (90°) to the palate midline and extend backward PARALLEL to the residual ridges to prevent tongue sensitivity. 4. Anterior border of the major connector is perpendicular to the midline and buried in the VALLEY OF RUGAE. 5. NEVER PLACED ANTERIOR to indirect retainers (anterior palatal coverage is avoided). 6. Exposed borders are beaded to produce a positive contact with the tissue. 7. Metal is in intimate tissue contact. BEAD LINE (BEADED BORDERS): USED ONLY ON MAXILLARY MAJOR CONNECTORS to SEAL the interface of the maxillary major connector to the tissues. It tapers off as it approaches the marginal gingiva around the abutment teeth. • Beading is done along the border of the major

connector to seal it to the soft tissue. Bead is made by scoring the cast .75-1mm wide and deep. The groove fades as it approaches within 6mm of the gingival margins and fades over a hard midline suture. • Seal is 1mm thick (deep) to provide POSITIVE DENTURE CONTACT WITH TISSUE to prevent food entrapment under the maxillary major connector and retention when placed at the posterior border of a palatal plate major connector. ANTERIOR-POSTERIOR PALATAL STRAP: • MAXILLARY MAJOR CONNECTOR TO ALMOST ALWAYS USE FOR ALL KENNEDY CLASSES. • Major connector of choice for inoperable tori cases where there is 6-8mm room to the vibrating line. • GREATEST STRENGTH & RIGIDITY b/c of its circular shape and b/c its metal straps lie in different planes. • Primarily used for a LARGE EDENTULOUS SPAN Kennedy Class III mod I RPDs with good residual ridges and strong abutment teeth (not good to use when edentulous spaces are small). A better choice than a single palatal strap because it covers less

tissue (uses minimal palatal surface area). • Must be at LEAST 15mm between the anterior and posterior straps! • The anterior and posterior straps are 6-8mm wide, and must be 6mm below the free gingival margin to avoid rugae coverage and tongue interference. Anterior strap is just posterior to a rugae crest or in a valley between two crests. • Posterior strap is thin, at least 8mm wide, and located entirely on the hard palate at right angles to the midline (not diagonally) to protect the TONGUE. It is NEVER placed on mobile tissue (soft palate). • DO NOT USE when an inoperable palatal torus extends posteriorly onto the soft palate.

HORSESHOE (U-SHAPE PALATAL CONNECTOR): • LEAST DESIRABLE MAXILLARY MAJOR CONNECTOR because has the LEAST strength & rigidity. • Used only to go around INOPERABLE PALATAL TORI or with MAJOR GAGGERS. Used only when large inoperable palatal torus prevent using palatal coverage in the posterior area (interferences). • LACKS RIGIDITY (unless it is bulky), can cause lateral flexure of abutments, and can impinge tissue during occlusion. • Used in Class III mod I designs and/or when a

palatal torus is within 6-8mm of the vibrating line. • Requires good residual ridges and strong abutments. SINGLE WIDE (MIDPALATAL) STRAP: • Mainly used in CLASS III designs. • Its width is kept within the borders of the rests (does extend posterior or anterior to rests). • Used for a LARGE-SPAN Class III mod I when residual ridges are good quality, but molar abutments are weak. Provides additional palatal support to the RPD when abutments are weak. • With weak abutments in a Class III, more palatal support is desired and a wide palatal strap is preferred. PALATAL PLATE (COMPLETE COVERAGE): AKA “METAL PLATE” • Used mainly in Class I design. MAIN INDICATION: when the last abutment tooth on either side of a bilateral distal extension is a CANINE or 1st PREMOLAR complete palatal coverage is advised, especially when the residual ridges have undergone excessive vertical resorption.

• Used for long-span Class I BILATERAL DISTAL EXTENSIONS with poor residual ridges, and/or periodontally involved weak abutment teeth. • Can be used in a Class II where there are missing anteriors with a posterior modification. • Covers at least 50% of the hard palate and replicates the anatomy of the palate. • Posterior border is at the JUNCTION of the hard & soft palate, but does not extend to the soft palate. • Should be anterior to the posterior palatal seal. • Direct retention is a problem and can interfere with the patient’s tongue and taste (some patients cannot tolerate). It acts more in INDIRECT RETENTION.

MODIFIED PALATAL PLATE: used for MAXILLARY CLASS I & II DESIGNS and may or may not include lingual plating. Anterior border ends at the rugae no less than 6mm from the free gingival margin. Provides maximum support as the plate rests on tissue. Contraindicated with PALATAL TORI.

INDIRECT RETAINERS (Rests & Proximal Plates) Indirect Retainers: • Are placed as far away from the distal extension base as possible to PREVENT VERTICAL DISLODGEMENT of the base from the tissue (i.e. opening when eating sticky foods). • Increases the effectiveness of direct retainers (clasps) when the RPD tries to dislodge, and prevents the RPD from rotating around the fulcrum line (axis of rotation). • IR should be 90° (right angles) to the fulcrum line, and is placed in rest seats to direct forces along the abutment’s long axis. • The greater the distance between the fulcrum line and IR, the more effective the IR. • IR may include: RESTS, PROXIMAL PLATES, & MINOR CONNECTORS. Indirect Retainer Functions when the denture base tries to MOVE AWAY from the residual ridge:

1. MAIN FUNCTION: Prevents VERTICAL DISLODGEMENT of the distal extension base away from tissues (sticky food). 2. Protects soft tissues impingement by the major connector during downward movement (limits movement in a cervical/gingival direction). 3. Decreases antero-posterior tilting leverages when an isolated tooth is an abutment (but avoid this). 4. Helps stabilize against horizontal denture movement by contact of the minor connector with the axial tooth surfaces (guide planes). 5. Stabilizes against lingual movement of anterior teeth when used. 6. May act as an auxillary rest to support part of the major connector. 7. May provide the first visual indication for the need to reline a distal extension by acting as a reference for seating frameworks and making altered cast impressions (i.e. can tell you if the rest is not fully seated in its rest seat). Fulcrum Line (Axis of Rotation)-the axis the RPD rotates around when the denture base moves AWAY from the residual ridge.

• Fulcrum line is mainly determined by the placement (location) of primary rests. The fulcrum passes through the rigid metal above the tooth’s height of contour and closest to the edentulous space. • Fulcrum line is the center of rotation as the distal extension base moves toward supporting tissues when an occlusal load is applied. • Class I & II always have a fulcrum line! o Class I: fulcrum passes through the most posterior abutment next to the edentulous space. o Class II: fulcrum line is diagonal and passes through the most posterior abutment on the distal extension side, and most posterior tooth on the non-distal extension side. o Class III & IV: TOOTH-BORNE/SUPPORTED RPDs that do not move toward tissue during function thus (physiologic relief/adjustment is not required). In Class III & IV designs, rests are placed immediately next to the edentulous space. • If the framework is properly designed, the fulcrum line will pass through the most DISTAL REST on each side.

• CLASS III design does not have a fulcrum line (no axis of rotation).

• Class IV: fulcrum line passes through the MESIAL rests next to the edentulous space. RESTS-the indirect retainer united with the major connector by a minor connector. Rests augment mechanical retention. Rests should restore the original tooth topography that existed before the rest seat was prepared. Rests must be RIGID. Rest Functions: o PRIMARY FUNCTION IS TO PROVIDE RPD VERTICAL SUPPORT! Prevents vertical dislodgement. o Maintains components in position. o Maintains established occlusal relationships by preventing settling of the denture. o Prevents soft tissue impingement in a cervical

direction. o Directs and distributes occlusal loads (vertical forces) to the abutment tooth’s long axis. o Rests are prepared BEFORE the final impression is made and master cast is poured. o NEED A MINIMUM OF 3 RESTS FOR ANY PARTIAL DENTURE. o There MUST be a MESIAL rest on the most posterior abutment tooth with a distal extension. 1. OCCLUSAL REST: prepared only in enamel or any restorative material proven to resist fracture and distortion when a force is applied. • 2mm deep in the center (0.5mm deeper than the 1.5mm that the marginal ridge is lowered). • OUTLINE FORM: ROUNDED TRIANGLE with the apex toward the center of the occlusal surface. • CONCAVE (spoon-shaped) occlusal surface. • Occlusal rest is as long as it is wide, and the triangle base (marginal ridge) is at least 2.5mm wide for molars & premolars.

• Marginal ridge is reduced/lowered 1.5mm to provide sufficient bulk of metal for strength and rigidity of the rest and minor connector. • Rest floor is slightly INCLINED APICALLY (deeper), than the 1.5mm depth of the marginal ridge. • Angle formed by the occlusal rest and vertical minor connector that is originates from must be LESS THAN 90° to direct occlusal forces along the abutment’s long axis. • Occlusal rest is always be attached to a rigid minor connector. • Occlusal rest is ALWAYS PREPARED AFTER THE PROXIMAL GUIDE PLATES! Auxillary Occlusal Rest-the most common indirect retainer placed on the occlusal surface as far away from the distal extension base as possible. • Class I design: it is placed bilaterally on the Mmarginal ridge of 1st premolars. • Class II design: placed on M-marginal ridge of 1st premolar on the non-distal extension side. Extended Occlusal Rest-used in Class II (mod I) & III RPDs when the most posterior molar abutment is

MESIALLY TIPPED. Minimizes further MESIAL tipping to help direct forces down the abutment’s long axis. • Extends more than ½ the tooth’s M-D width and is 1/3 the B-L width. • Allows minimum of 1mm metal thickness. • Rounded rest seat preparation with no undercuts. 2. EMBRASURE/INTERPROXIMAL REST: • Used to prevent inter-proximal wedging of the framework and shunt food away from contact points. • Rest seat is preparation is extended LINGUALLY for bulk strength, but prepared just like an occlusal rest. • Marginal ridge is lowered 1.5mm on each abutment • Avoid creating a vertical groove to prevent the minor connector from torquing of the abutment tooth. 3. CINGULUM REST (LINGUAL REST): • The most satisfactory lingual rest for support is placed on a prepared rest seat in a cast restoration. • Canines are preferred over incisors. • Preferred to an incisal rest because it is more

esthetic. • Limited to maxillary canines and centrals with exaggerated cingulums. Rarely satisfactory on mandibular anteriors due to lack of enamel thickness. A risky preparation on lower incisors. • Preparation is a slightly rounded inverted V (semilunar) placed at the junction of GINGIVAL & MIDDLE 1/3 of the lingual surface (just above the cingulum, but low enough to minimize abutment torquing forces). • 2mm wide F-L. • 2.5-3mm M-D length. • Minimum 1.5mm deep (incisal-apically) 4. INCISAL REST: • Used mainly as an auxillary rest or indirect retainer. • Rounded notch is placed 3-4mm from either the MI or DI edge (canines or incisor). • LEAST ESTHETIC REST and most likely to cause orthodontic movement due to unfavorable leverage. • Preparation is 2.5mm wide and 1.5mm deep

(deepest portion is apical to the incisal edge). • Not used on maxillary incisors unless it is the only option. • Beveled labially and lingually. Maxillary RUGAE as indirect retention with RPDs: Broad coverage over rugae can provide some support, but is LESS EFFECTIVE than tooth support, is undesirable, and avoided. Rugae can provide indirect retention with a PALATAL HORSESHOE DESIGN because the horseshoe’s posterior retention is inadequate. GUIDE PLATE FUNCTIONS: • Helps establish a definite path of insertion/dislodgement of the RPD. • Stabilizes the RPD by controlling its horizontal position. • Provides contact with the adjacent tooth. • Should extend just past the DL line angles to provide 180° encirclement, bracing, and reciprocation. • Prepared in the occlusal 1/3 on the proximal surface. • Guide plane is ~2-3mm in height occluso-gingivally,

but its F-L width is determined by the tooth’s contour. • Guide plates for Class III & IV designs can extend above the abutment’s height of contour because there is no functional movement. • Guide plates MUST be below the abutment’s height of contour with Class I & II designs to prevent abutment torquing during functional movements. • When plates are used with I-bars and mesial rests on premolars to avoid lingual tori, the plate must end EXACTLY at the height of contour. • Guiding planes ensure predictable clasp retention. Failure of partials due to poor clasp retention design can be avoided by altering tooth contours. Guiding planes serve to assure predictable clasp retention.

DIRECT RETAINERS (CLASPING) ALL CAST CLASPS (DIRECT RETAINERS) both suprabulge & infra-bulge are NEVER placed anterior to the fulcrum line (axis of rotation) because they would release during function and TORQUE the abutment. Only indirect retainers and stress breakers (Wrought Wires) are placed anterior to the fulcrum line. To determine [number of clasps to use in an RPD = [Kennedy Class + 1] • Does not apply to Class IV designs Primary Retention-provided by mechanically placing retaining elements on the abutment teeth. Secondary Retention-provided by intimate relationship of minor connector contact with guiding planes, denture bases, and major connectors with underlying tissues. Vertical Arm of Surveyor: • Indicates areas of retention, areas of support on the abutment tooth, and tooth/tissue interferences to the path of insertion.

• A vertical arm represents the path of placement and removal of an RPD. • Carbon marker is used to determine the tooth’s height of contour (greatest convexity). • Zinc Stearate Powder can identify the survey line (height of contour) for a crown wax-up abutment tooth. It is brushed on the wax-up and the analyzing rod is passed over wax surface and removes the powder. DIRECT RETAINER: gives the RPD mechanical retention. Any RPD unit that engages an abutment to resist displacement AWAY from basal seat tissues using friction, engaging a depression, or undercut cervical (gingival) to the tooth’s height of contour. 2 DIRECT RETAINERS: .01 = .25mm Cast Clasp 1. Intra-Coronal Retainer (“Precision Attachment or “Internal Attachment”): • Advantages: the most esthetic direct retainer. Provides the best vertical support through a rest seat located more favorably relative to the abutment’s horizontal axis (does not allow horizontal movement). • It is cast or attached totally within the abutment’s restored natural contours. • Has a prefabricated key & keyway with

opposing vertical parallel walls to limit movement and resist removal by friction. • Not used with extensive tissue-supported distal extensions unless a stress-breaker is used between the movable RPD base and rigid attachment. 2. Extra-Coronal Retainer (Clasps): placed on EXTERNAL surfaces of abutment teeth. • Has a RETENTIVE clasp arm that is FLEXIBLE and placed in areas below (cervical) to the tooth’s height of contour (tooth’s gingival 1/3). Provides resistance to deformation from a vertical dislodging force. This generates the retentive action of the clasp. Which has a passive relationship with the abutment until a dislodging force is applied. • Lingual arms on molars are usually retentive b/c there are usually no usable facial undercuts on mandibular 2nd or 3rd molars. * Clasp Arm Flexibility: • Longer and thinner (smaller diameter) the clasp arm = more flexible. • Most clasps are ½ round in form. A round clasp form is the only circumferential clasp form that can be

safely used to engage an undercut on the side of an abutment away from the distal extension base. • Retentive arm must be flexible to provide stress relief for the abutment. • UNDERCUT LOCATION is the most important factor when selecting a clasp for distal extensions. DENTIST decides which clasp design is best based on the diagnosis and treatment plan established. • Has a BRACING (stabilizing/reciprocating) clasp arm placed OCCLUSAL to the tooth’s height of contour (crown’s middle 1/3). MUST BE RIGID. • Composed of chromium-cobalt alloys to give greater rigidity with less bulk. • RIGID because it is greater in diameter (thicker) than the retentive arm. • Tapered in one dimension only. • Horizontal force is transmitted by placing rigid portions of clasps in NONUNDERCUT areas of abutment teeth. RECIRPOCATION: occurs only when the retentive arm and bracing arm CONTACT the tooth at the same time during seating and removing the RPD. As the retentive arm tip passes over the height of contour and engages the

undercut, the rigid bracing arm must maintain contact with the abutment. TIMING IS CRITICAL IN RECIPROCATION. Clasp Assembly Components: Clasp assembly components provide 180° encirclement of the abutment (clasp arms, minor connectors, guide plates all contribute to the 180° encirclement. 1. 1-2 rests & at least 1 minor connector. 2. Retentive clasp arm (flexible) to engage and terminate in undercuts. 3. Reciprocating (bracing) clasp arm (rigid). When an RPD is fully seated, the clasp tips should NOT EXERT ANY PRESSURE against the abutment teeth. It must be totally passive. The retentive arm is activated ONLY when vertical dislodging forces attempt to unseat the RPD away from the basal seat tissues. CLASP ASSEMBLY: 1. Clasp should be completely passive and its retentive function is activated only when dislodging forces are applied. 2. Each retentive clasp must be opposed by a reciprocal (bracing) clasp arm or another RPD element capable of resisting horizontal forces exerted on the tooth by the retentive arm. 3. Each clasp must be designed to encircle more than 180° (more then ½ the circumference) of the abutment tooth. 4. Rest should only provide VERTICAL SUPPORT.

EXTRA-CORONAL RETAINERS: suprabulge & infrabulge clasps MUST have 1 retentive arm (flexible) and 1 rigid reciprocal bracing arm. SUPRABULGE RETAINERS: approach the retentive undercut from ABOVE the tooth’s height of contour (usually from an occlusal rest). 1. CIRCUMFERENTIAL CLASP (AKER’S CLASP): • Engages > 180° of the abutment’s circumference. • Terminal end of its retentive clasp arm provides retention (buccal) by engaging an undercut. • Has a non-flexible (rigid) lingual clasp arm for stabilization/reciprocation. Must always lie at or above the height of contour because it cannot flex to get in and out of undercuts. • Originates on or occlusal to the tooth’s height of contour, then crosses in the terminal third, and engages an undercut as its taper decreases and flexibility increases. • Consists of 1 retentive clasp arm + 1 non-retentive reciprocal arm. • Clasp of choice in Class III & IV (tooth-borne designs) when the most posterior abutment undercut is AWAY from the edentulous space (i.e. MB) surface. • Undercut must be on the opposite side of the tooth/rest from where the clasp originates. • DO NOT USE when an undercut is adjacent to the

edentulous space (DB or DL). • RPI Clasp Assembly: consists of mesial rest + distal guide plate + circumferential clasp. 2. RING CLASP: • Indicated to engage an undercut of a MESIALLYLINGUAL TILTED MOLAR when a severe tissue undercut exists that prevents using an I-bar. • Used almost exclusively on mandibular molars that drifted MESIALLY & LINGUALLY to engage a LINGUAL UNDRCUT. • Indicated in reverse on an abutment anterior to a “tooth-bound” edentulous space. • Encircles nearly all of a tooth from its point of origin. • Used to engage a proximal undercut (i.e. ML undercut on mandibular molar cannot be directly engaged b/c of its proximity to the occlusal rest and cannot be approached with a bar clasp (infrabulge) due to the molar’s lingual inclination. • Allows the undercut to be approached from the tooth’s distal. • Has a mesial primary rest & distal auxillary rest. Used almost exclusively on ML tilted molar abutments. Always used with a supporting strut on the nonretentive side with or without an auxillary rest on the opposite marginal ridge. • Used on protected abutments because it covers lots of tooth surface. • Used when caries risk is LOW and in NON-

ESTHETIC areas. • Used when a DB or DL undercut on a molar cannot be approached directly from the occlusal rest and/or when tissue undercuts prevent engagement with a bar clasp (infrabulge). • Clasp can originate on the MB surface to engage a ML undercut, or ML to engage a MB undercut. 3. REVERSE ACTION (HAIRPIN) CLASP: • Used only on abutments of “tooth-borne” dentures (Class 3 & 4) where a proximal undercut is BELOW the point of origin only when a bar clasp (infra-bulge) is contraindicated due to a tissue undercut, tilted tooth, shallow vestibule, or high tissue attachment. • Used when lingual undercuts prevent placing a supporting strut without tongue interference. • Only the lower part of the clasp arm (after the curve) is flexible to engage the undercut. 4. EMBRASURE CLASP: • Used on sound teeth with retentive areas or when multiple restorations are justified. • Used when no edentulous space exists on the opposite side of an edentulous Class II or III with no modifications. • Requires at least 1.5mm marginal ridge reduction to prevent fracture of the clasp assembly. • ALWAYS USED WITH DOUBLE OCCLUSAL RESTS to prevent inter-proximal wedging of the framework.

• A retentive arm and rigid reciprocating arm must be present for each abutment, but do not have to be on the same side. • Wrought wires are NEVER used with embrasure clasps. 5. HALF-&-HALF CLASP: • INDICATED FOR LINGUALLY INCLINED PREMOLARS (LINGUAL UNDERCUTS). • Consists of 1 circumferential retentive arm from one direction, and reciprocating arm that arises from the minor connector. 6. BACK-ACTION CLASP: • A ring clasp modification. Its use is difficult to justify because you could easily use a conventional circumferential clasp. • Can be used on a premolar abutment anterior to an edentulous space. 7. MULTIPLE CLASP: • Two opposing circumferential clasps joined at the terminal end of the two reciprocal arms. • Used when additional retention and stabilization is needed (tooth-supported RPDs). • Disadvantage: two embrasure approaches are necessary rather than a single embrasure for both clasps.

8. COMBINATION CLASP: • Most commonly used when an abutment next to a distal extension (Class II, mod I) where only a MB undercut exists or if large tissue undercuts prevent a bar clasp from being used. • Used when maximum flexibility is required (i.e. an abutment next to a distal extension, on a weak abutment when a bar direct retainer is contraindicated, or when esthetics is a concern). • Consists of a bracing arm, wrought wire retentive circumferential arm, and distal rest. • Use when the undercut is on the side of the abutment away from the edentulous space because it is more flexible than a cast clasp arm thus can dissipate functional stresses. 9. EXTENDED ARM CLASP: • NEVER used with Class I & II (distal extensions) b/c functional forces cause rotation around the rest and upward movement of the clasp tip. Used for abutment tooth-borne dentures next to an edentulous space. INFRABULGE RETAINERS (BAR CLASPS = ROACH CLASP): • Arises from the framework or metal base and approach the abutment’s retentive undercut from a GINGIVAL DIRECTION (BELOW THE HEIGHT OF CONTOUR).

• Indications: when a small undercut (0.01 inch) exists in the cervical third of the abutment, on abutment teeth for Class III & IV designs (tooth-supported), in distal extension base situations, and when esthetics is a concern. • Contraindications: when a DEEP cervical undercut exists or when a severe tooth or tissue undercut exists. Not used if a tissue undercut exists (because is bothers the tongue and cheek and traps food debris) or with high frenum attachments (shallow vestibule) or excessive buccal or lingual tilt of the abutment tooth. • Advantages: inter-proximal location for ESTHETICS, increased retention without abutment tipping, less chance of accidental distortion due to its proximity to the denture border. • They type of bar clasp is insignificant as long as it is mechanically and functionally effective, covers as little tooth surface as possible, and displays as little metal possible. • Vertical portion of the approach arm crosses the gingival margin at 90°. • Infrabulge Claps: T bar, Modified T bar, I bar, Y bar, Roach Clasp.

I BAR: • Always place the tip of I bar’s retentive arm MESIAL to the greatest M-D curvature on the abutment’s facial surface to ensure retention in the undercut. The undercut must be MESIAL (in front of) the greatest M-D curvature on the abutment’s facial surface. • I-bar retentive arm with a MESIAL rest and distal guide plate (RPI SYSTEM) is the BEST clasp assembly to be placed on the terminal abutment for distal extensions. • Indicated for a Class I or II RPDs using a MESIAL REST when there is no tissue undercut. • Used to engage a ML undercut when there is no tissue undercut below the abutment. • I-bar’s superior border is located more than 3mm from the free gingival margin. • Indicated for Class II mod I on a ML tilted molar with little tissue undercut. • The foot of the I-bar is completely below the height of contour (survey line) in distal extension designs so it can release during functional movements of the extension base. • When a patient bites down, the I-bar should release from the undercut. The retentive arm should only function when there is an attempt to dislodge the RPD (opening the mouth when chewing sticky food).

MODIFIED T BAR: • Bar of choice for DB undercuts below the height of contour immediately next to an edentulous space. • Primary indication is when abutment undercuts are immediately next to an edentulous area and no tissue undercuts. Can be used with a mesial or distal occlusal rest in tooth-supported designs since no functional movement occurs. • When used on a terminal abutment for a Class I design, it is used with a mesial rest and the arm tip is placed into a DB undercut. • Its vertical arm must approach and engage MESIAL to the greatest M-D curvature on the abutment’s facial surface to prevent the RPD from being dislodged up and back. “RPI” SYSTEM (Rest, Proximal plate, I-bar)-an I-bar clasp that consists of a MO rest, and distal guide plate with the minor connector placed into the ML embrasure, but not contacting the adjacent tooth.

• Used only with Kennedy Class I or II (DISTAL EXTENSIONS). • Must be 180 degrees around the tooth • Distal guiding plane extending from the marginal ridge to the junction of middle & gingival 1/3 of the abutment is prepared to receive a proximal plate. • MESIAL RESTS ARE PLACED ON THE TERMINAL ABUTMENT TOOTH FOR ALL DISTAL EXTENSIONS. RPI system is designed to allow vertical rotation of a distal extension saddle into the denture-bearing mucosa under occlusal loading without damaging the supporting structures of the abutment tooth. As the saddle is pressed into the denture-bearing mucosa, the denture rotates about a point close to the mesial rest. Both the distal guide plate and Ibar move in the directions indicated and disengage from the tooth surface. Potentially harmful torque is thus avoided.

RPD STRESS BREAKERS Stress-Breaker-a device that relieves the abutment teeth to which an FPD or RPD is attached, of all or part of the forces generated by occlusal function. When a stress-breaker is incorporated next to a free-end distal extension RPD, the functional stress is directed onto the residual ridge and only minimal transfer of functional stress to abutment teeth occurs. Since vertical and horizontal forces are concentrated on the residual ridge, increased ridge resorption frequently occurs. WROUGHT WIRE RETENTIVE CLASP (STRESSBREAKER): a stress breaker used because of its increased flexibility (it minimizing abutment torquing). The simplest form of stress relief. Has a flexible connection between the direct retainer and denture base. Advantages: higher yield strength, greater flexibility, more ductile and resilient. • Often used with a MESIAL rest in Class I & II designs on the most posterior abutment tooth (terminal abutment) when there is a tissue undercut, or high frenum attachment that prevents using an Ibar. • If occlusion prevents using a mesial rest on the most posterior abutment in a distal extension,

only a WW can be used with a distal rest because it is OK for its retentive tip to be in front of the axis of rotation (fulcrum). • Used on teeth with indirect retainers on them (both are anterior to the fulcrum line). Provides stress relief to the abutment tooth due to its flexibility when the distal extension moves toward the residual ridge. • Tip of its retentive arm should engage the undercut ANTERIOR to the fulcrum line (axis of rotation). Terminal end of its retentive arm is optimally placed in the middle of the gingival 1/3 of the clinical crown. However, it is acceptable to place it at the junction of the gingival and middle 1/3 of the clinical crown. When the partial is completely seated, the retentive arm should be passive and applying no pressure on the teeth. • NOT USED in Class III & IV (tooth-borne) designs because there is no functional movement of the RPD. • DO NOT USE wrought wires through embrasures or with embrasure clasps. • Has a tensile strength at least 25% greater than the cast alloy from which it was made. Wrought-wire clasps have greater flexibility and adjustability

than the cast clasps, are tougher and more ductile than cast clasps, and have greater tensile strength. Thus can be used in smaller diameters to provide greater flexibility without fatigue and fracture. • Wrought wire is incorporated into the RPD by soldering it to the minor connector, meshwork, incorporating it into the wax pattern, or is embedded into the acrylic resin (makes it the most flexible). 20gauge wrought-wire is 2x more flexible than an 18gauge wire. • .02 = .5mm wrought wire. Must be at least 8mm long and tapered round 18 gauge wire. • .03 = .75mm and IS NOT JUSTIFIED. FINISH LINES: 1. External Finish Line-the external junction (butt joint) of the metal framework and denture base plastic (acrylic). • The external finish line on a maxillary Class I RPD originates from the lingual of the guide plate of the terminal abutment and ands at the HAMULAR NOTCH. 2. Internal Finish Line-the butt joint between the metal and acrylic on the TISSUE SIDE of the edentulous area.

• Junction of the major and minor connector at palatal finishing lines are 2mm medial from the imaginary line that would connect lingual surfaces of missing posterior denture teeth. • Internal and External finish lines are normally OFFSET from each other to avoid framework weakness/fracture. • Location of the finishing line at the junction of the major and minor connector is based on restoring the natural palatal shape while considering the location of replacement teeth. • Finishing line junction with the major connector should be no greater than 90°, thus being somewhat undercut. • Junction of minor connectors and bar clasps are 90° butt-joints that follow the guidelines for base contour and clasp length.

SURVEYING RPD ABUTMENTS Surveying casts allows the dentist to record the dentures path of insertion, position of the survey line, and location of undercut and non-undercut areas. To do this, TRIPOD MARKS are placed on the cast to record the cast’s orientation to the surveyor. • Tripod marks-3 spots placed at 3 different locations around abutment teeth from a single point of view to ensure a reproducible orientation of the cast to the surveyor. Records the cast’s position. • Dental Surveyor-used to determine the relative parallelism of oral anatomy. Areas used for support CANNOT be determined by surveying. When surveying casts, the correct procedure is to first adjust the tilt to permit the establishment of guiding planes. The anterior edentulous space will frequently dictate the angulation needed. Normally, some re-contouring of the proximal walls of abutment teeth is needed to improve guideplane alignment by disking the proximal surface parallel to the path of insertion. • Tilting the cast during surveying changes the path of insertion, survey line position, and location of the undercut and non-undercut areas of each

tooth. • PURPOSE OF SURVEYING (Using Diagnostic Casts): • Determine the most desirable path of placement that will eliminate or minimize interference to removal and placement the RPD. • Identify surfaces for guiding planes for the prosthesis. • Locate and measure areas of teeth for retention. • Determine if a tooth and/or bony areas require surgical removal or if another path of insertion will suffice. • Determine the most suitable path of placement that will design of retainers and teeth to be most esthetic. • Prepare an accurate charting for any mouth preparation to be made. • Determine and delineate the tooth’s height of contour. FACE BOW-caliper-like device to record the patient’s maxilla/hinge axis relationship (opening and closing axis) and to transfer this relationship to the articulator during mounting of the maxillary cast. If the transfer is done properly, the arc of closure on the articulator should duplicate the patient’s true arc of closure.

Before an accurate face-bow transfer record can be made, the location of the hinge axis point (axial center of opening-closing), must first be determined. Facebow transfer is NOT a maxillo-mandibular record, but a record to orient the maxillary cast to the hinge axis on the articulator. The facebow transfers the maxilla/hinge axis relationship to the articulator during mounting of the maxillary cast. A face-bow transfer record DOES NOT: allow the dentist to locate the hinge axis, nor record CR more reliably, nor position the maxillary cast properly in relation to the mandibular cast, nor transfer the cast to the articulator maintaining the proper inter-occlusal relationships present in the mouth. Hinge-axis face bow transfer enables the dentist to alter VDO on the articulator. When altering VDO (either via restorations or with dentures), casts should be mounted on the hinge axis. Hinge Axis Face Bow-used to record opening and closing of the mandible. FRAMEWORK TRY-IN: • Before trying in a framework, inspect the master cast for damage and inspect the framework for sharp

metal fins. • When making maxillary & mandibular RPDs on the same patient, the dentist should first try each framework one at a time for fit, then adjust occlusion for each if needed, then adjust occlusion with both frameworks in place. • Damaged areas on the cast are the first areas adjusted if the framework does not fit. • If the framework fits the cast but not in the mouth, all other possible causes should be eliminated before making a new impression. • If attempts to fit the framework to the mouth are unsuccessful after adjusting, then assume the impression or cast is inaccurate and the impression will need to be remade. • A MASTER CAST for a RPD should be blocked out and duplicated BEFORE the framework is waxed up. Advantages of RPD chromium-cobalt alloys: corrosion resistance, high strength, low specific gravity & VERY RIGID (inflexible). No ductility or malleability after they are cast.

Composition of Chromium Base Metal Alloys for RPDs: 1. Chromium-responsible for CORROSION RESISTANCE in cobalt-chromium alloys. Ensures the alloy will resist tarnish and corrosion by forming a complex chromium oxide film. An RPD made of a base metal alloy is resistant to tarnish and corrosion because of its surface oxide layer. 2. Cobalt-increases the framework’s RIGIDITY, strength, and hardness. 3. Nickel-increases DUCTILITY. Measured as a percentage of elongation and determines how much margins can be closed via burnishing. Nickel is the metallic component of a RPD with the greatest potential for ALLERGIC REACTIONS in the mouth. ADA ALLOY CLASSIFICATION: • Type I: used for small inlays. • Type II: larger inlays & onlays. • Type III: onlays, crowns, and short-span FPDs. • Type IV: thin veneer crowns, long-span FPDs & RPDs. FLEXIBLE RPD: many RPDs today are more frequently made of thermoplastic materials in place of traditional acrylic or cast-metal partials. Duraflex, Valplast, and TCS flexible partial dentures offer unique physical and esthetic

properties to blend with the patient’s natural tissue and surrounding dentition. These are also know to be clinically unbreakable, are resistant to water absorption, so they are less likely to absorb stain and odors. They also remove the need for the dentist to cut rest seats into natural teeth. • Contraindications/Disadvantages: patients with poor oral hygiene, short or heavily worn mandibular anteriors, over a closed vertical dimension or protruding tori, or when there is minimal undercut on abutment teeth (at which time a metal clasp is advised). Most flexible RPDs cannot be relined chairside, thus must be sent to the lab for rebase. ALTERED CAST IMPRESSION: • Purpose is to obtain the maximum support possible from the edentulous ridges in Class I & II designs. • Captures edentulous ridge tissues in relation to the way the framework fits IN THE MOUTH (not cast). • Helps avoid overextension that is common when a stock-tray alginate impression is used. • A stock tray will perform adequately in areas where teeth remain but relatively poorer in edentulous areas. This is one of the major reasons an altered cast impression is done. • Altered Cast Technique-the purpose is to record the form of the edentulous segment without tissue displacement and to accurately relate the

edentulous segment of the teeth via the metal framework. The goal is to provide maximum support for the RPD denture base, thus maintaining occlusal contact to distribute occlusal load over both natural and artificial dentitions, while minimizing movement of the base that would create leverage on the abutment teeth. • Altered cast technique helps obtain soft tissue support to aid abutments in resisting functional stresses. It is a secondary impression system that uses the metal framework to hold customized impression trays for the edentulous areas. IMPRESSIONS: • If a mandibular RPD abutment must be crowned, the FPD impression should include a full arch impression is required to capture all abutment teeth and the retromolar pads. • When crowing an abutment tooth for an RPD, you must reduce more than the normal occlusal reduction for the rest seat.

COMPLETE DENTURES Complete Denture Design Characteristics: 1. Stability-the quality of a denture or prosthesis to be firm, steady, constant, and not subject to change position when forces are applied. In dentures, stability is the relationship of the denture base to bone that resists dislodgement of the denture in a HORIZONTAL direction. • Stability involves resistance to horizontal, lateral, & torsional forces (most important). • All RPD components, except the retentive clasp tip, contribute to denture stability. 2. Support-resists VERTICAL seating forces provided by rests and denture bases. For RPDs, support is provided by occlusal rests and edentulous ridge areas. Support is the MOST important design characteristic for oral health. 3. Retention-resists the force of gravity, sticky foods, and forces associated with mandibular movement. Direct & indirect retainers provide retention. CLASPS in undercut areas of abutment teeth provide retention. Factors that impact retention, stability, & support:

quality of oral mucosa, alveolar ridge contour, muscle attachments, saliva, and neuromuscular control (the most important as patient’s muscles memory learns to hold their denture in place to chew and speak effectively). BORDER MOLDING: • Masseter Muscle: powerful muscle whose fiber run superior-inferior that pushes the buccinator into the DB corner of the denture base during contraction. SHAPES DISTOBUCCAL AREA during an altered cast impression. Overextension of a mandibular denture base in the distofacial area causes dislodgement of the denture during function as the result of the action of the MASSETER. An overextended DB corner of a mandibular denture pushes against the MASSETER during function. • Superior Constrictor Muscle: shapes the DISTOLINGUAL BORDER MOLDING. Affects the most distal portion of the lingual flange. An overextended lingual flange can cause a sore throat. • MOST critical area in border molding a MAXILLARY DENTURE is the MUCOGINGIVAL FOLD above the maxillary tuberosity area as this area is extremely important for maximal retention. Other critical areas are the labial frenum in the midline, and frena in the bicuspid area. Overextension

in these areas often leads to decreased retention and tissue irritation. • When border molding a MANDIBULAR DENTURE for a final impression, the DB extension is determined by the position and action of the MASSETER MUSCLE. The DL extension of the mandibular impression for a complete denture is limited by the action of the SUPERIOR CONSTRICTOR MUSCLE. Frenum-folds of mucus membrane containing fibrous C.T. Must provide denture relief/space in this area because it can limit the denture’s extension. Frenum pull can also cause attachment loss. Custom Tray & Border Molding: ► A custom tray fabricated on a preliminary cast is trimmed ~2mm short of the mucosal reflection and frenae. This is done by first checking the borders in the mouth and then trimmed down to allow uniform thickness of 2mm of modeling compound when the borders are molded. However, the primary indicator of border molding accuracy is STABILITY and lack of displacement of the custom tray in the mouth. ► Custom tray for a final mandibular or maxillary

complete denture impression should have a spacer with tissue stops to ensure the tray seats in proper relationship to the arch, and to ensure adequate room for the impression material. The space is created with wax covered by aluminum foil over the master cast prior to forming the tray. ► Border molding is completed in two stages. The molding should approximate the tray borders and should be slightly overextended. Excess compound is trimmed from inside and outside the custom tray. The remaining modeling compound is then refined by repeating the process. The final form of the border molding should represent an accurate impression of the peripheral tissues. The border modeling compound should have a smooth, polished appearance. ► Palatoglossus, superior pharyngeal constrictor, mylohyoid, and genioglossus muscles are influential in border molding the LINGUAL border of the mandibular impression for an edentulous patient. After extraction, alveolar ridge resorption occurs because there is no longer bone stimulation. Atrophy of supporting structures occurs (residual ridge resorption). • Maxillary arch: bone loss/resorption occurs in a VERTICAL & PALATAL direction (UPWARD &

INWARD). (0.1mm/year is sustained). Initial loss in first year is greater, but varies. • Mandibular arch: bone loss occurs in VERTICAL direction (DOWNWARD & FORWARD/OUTWARD). Bone loss is oriented along the cross-sectional shape of the mandible. Mandibular bone resorption is 4x faster than in the maxilla, but varies. Severe bone resorption can cause a Pseudo Class III malocclusion appearance. OCCLUSION RIMS-the resultant product after adding baseplate wax to a record base to approximate the tooth position and arch form expected in the completed denture. Functions: • Determine and establish the patient’s VDO (vertical dimension of occlusion) and level of occlusal plane. • Make maxillo-mandibular preliminary jaw relation records. • Establish and locate the future position of denture teeth (arch for the lips, cheeks, tongue). • Maxillary rim is 22mm and mandibular rim 18mm. When recording CR for an RPD, the occlusion rim is attached to the completed partial denture metal

framework instead of to a record base as used with a complete denture. Inferior surface of the maxillary occlusion rim should be PARALLEL to CAMPER’S LINE (the line/plane running from the inferior border of the nose ala to the superior border of the ear tragus). Significance of Camper’s Line: the occlusal plane, established by the wax occlusion rims surfaces is parallel to Camper’s line & inter-pupillary line. Impression of Complete Dentures Should: ► Afford placement and control of impression material in recording border tissues (border molding). ► Result in minimal tissue displacement under the denture (registers tissues in its passive position). ► Depend on the oral conditions present. ► Best impression technique for a patient with loose hyperplastic tissue is to register the tissue in its PASSIVE position. There must be intimate contact of the impression material with the tissue.

MANDIBULAR COMPLETE DENTURES MANDIBULAR COMPLETE DENTURES: a primary support (stress-bearing) area is the BUCCAL SHELF because of its bone structure (resists resorption because it is dense cortical bone and does not change) and its trabeculation right angle (parallel) relationship to the occlusal plane. • A SECONDARY peripheral seal area for a mandibular complete denture is the anterior lingual border. • A SECONDARY relief area is the CREST OF RESIDUAL RIDGE WHEN IT IS SHARP. Mandibular Support Areas: 1. Buccal shelf-the primary support area for a mandibular denture. Buccinator muscle limits a denture’s extension in this area. The bigger the buccal shelf, the more denture support. • Masseter muscle affects the mandibular denture. • The boundaries of the buccal shelf are the buccal frenum to the retromolar pad crest of the residual ridges to the external oblique line (

2. Alveolar ridge- SECONDARY area of mandibular denture support. • RESIDUAL RIDGES if large and broad, may be primary support areas, but are usually SECONDARY STRESS-BEARING AREA because it is cancellous bone. 3. Retromolar pad-does not change or resorb. NOT a primary support area, but must be captured in the impression. Lies at the crest of the mandibular residual ridge. 4. Keratinized tissue- the more keratinized tissue, the better denture support and comfort. • When fabricating a mandibular complete denture for a patient with a “knife-edged ridge” you need maximum extension of the denture to help distribute occlusal forces over a larger area. • Marked RESORPTION of the alveolar ridge will occur if a mandibular complete denture base terminates short of the retromolar pad. Underextension of the peripheral border of a complete mandibular denture decreases tissuebearing surfaces, thus affecting denture STABILITY. • Underlying basal bone (under the retromolar pad)

resists bone resorption (does not resorb). Covering this area also provides some border seal. An overload of the mucosa occurs if the bases covering the area are too small in outline. • Mandibular dentures do not rely on suction from a peripheral seal for retention like maxillary dentures, but rely on denture stability in covering as much basal bone possible without impinging on muscle attachments. The active border molding performed by the lips, cheeks, and tongue determines the peripheral areas of a mandibular arch, thus establishing maximal basal bone coverage. • DO NOT PLACE mandibular molars over the ascending area of the mandible because occlusal forces over the inclined ramus DISLODGE the mandibular denture. Thin mucosa is found in mylohyoid area & over mandibular tori (mandibular dentures) and on the midline of the palatal vault and over a torus palatinus (maxillary dentures). Mandibular tori, sharp prominent mylohyoid ridges, and Epulis Fissuratum are evaluated for surgical removal before the fabrication of new dentures begins. Tongue position affects denture stability & retention in the

mouth. A patient with a RETRACTED/RETRUDED tongue is a poor denture candidate. Mylohyoid Muscle: affects the slope of the lingual flange of the impression in the molar region (at its most POSTERIOR (distal) aspect) causing the flange to slope toward the tongue. FORMS THE MUSCULAR FLOOR OF THE MOUTH. Arises from the mylohyoid ridge of the mandible near the mandible’s inferior border in the INCISOR REGION and becomes higher on the mandible’s posterior body until it terminates just distal to the lingual tuberosity. • Connects at the midline and can LIFT THE MANDIBULAR DENTURE when the TONGUE IS PROTRUDED. It influences the molar region and slopes toward the tongue. • Sublingual gland and anterior portion of the mylohyoid muscle cause the LINGUAL FLANGE to be lower in height in the anterior region. • RETROMYLOHYOID CURTAIN-composed of the superior pharyngeal constrictor & palatoglossus. Determines how far posteriorly the lingual flange can go. • RETROMYLOHYOID FOSSA-located at the distal end of the alveolar LINGUAL SULCUS. This is

where the lingual flange turns toward the ramus making the famous “S-CURVE” with the flange sloping toward the tongue in the molar region of the mylohyoid muscle. Bordered medially by the anterior tonsillar pillar and posteriorly by the retromylohyoid curtain. o S-CURVE is seen in a mandibular impression due to the MYLOHYOID MUSCLE (slopes toward the tongue) and RETROMYLOHYOID FOSSA which slopes toward the buccal. Pterygomandibular Raphe-a TENDON lying between the buccinator & superior constrictor muscles. Lingual Tubercle (Lingual Tuberosity)-the bulge at the distal of the mylohyoid ridge. Alveolingual Sulcus-the space between the ridge and tongue. Submaxillary Salivary Caruncle-eminence on either side of the frenum on which the major sublingual ducts and submandibular ducts open.

MAXILLARY COMPLETE DENTURES PRIMARY support denture bearing areas (stressbearing area) is the RESIDUAL RIDGES & PALATE. SECONDARY support areas is the PALATAL RUGAE. Secondary RETENTIVE area is the glandular region on each side of the midline. POSTERIOR PALATAL SEAL-extends through the HAMULAR NOTCHES in the maxilla, and passes 2mm in front of the FOVEA PALATINAE. It is in an area of immovable tissue and compensates for denture processing errors. Posterior palatine salivary glands help maintain peripheral seal. ► Mark it in the mouth with a Thompson stick and carve/scribe this area into the cast. ► Compensates for acrylic shrinkage and is in IMMOVABLE TISSUE. ► Butterfly shape and in shallower in the center and hamular notch areas. ► Carried ~5mm ANTERIOR TO THE VIBRATING LINE.

• Posterior palatal seal outline and depth differs for every patient according to the palatal form of the patient. • A posterior palatal seal is necessary when fabricating a complete denture on a patient with a flat palate. THE FLATTER THE PALATE, THE WIDER THE POSTERIOR PALATAL SEAL. • Posterior palatal seal should never be removed. • Middle of the posterior palatal seal is 0.5mm deep extending 3mm on both sides of the midline. • Seal is 1.5mm deep lateral to the middle of the seal and should extend up to the medial boundary of the pterygomaxillary (hamular) notches. Width is 11.5mm high and 1.5mm broad at its base. • Excessive depth of the posterior palatal seal usually results in unseating of the denture. • WIDTH of the seal anterior to posterior is characterized by a concave surface, 3mm wide in the midline, and 6mm wide in the mid-lateral areas. • Placement of the posterior palatal seal is always done by the DENTIST. Posterior Palatal Seal Functions:

1. Completes the border seal of the maxillary denture. 2. Prevents food impaction beneath the denture’s tissue surface. 3. Improves the denture’s physiologic retention. 4. Compensates for polymerization and cooling shrinkage of the denture resin during processing. Hamular Notch (Pterygomaxillary)-a cleft of loose C.T. that extends from the maxillary tuberosity to the hamulus of the medial pterygoid plate. Fovea Palatini-a group of mucous gland ducts whose location varies, but is usually slightly posterior to the junction of the hard & soft palates near the midline. • VIBRATING LINE: ~2mm anterior to the fovea palatinae and ALWAYS ON THE SOFT PALATE. The imaginary line across the posterior palate that marks the division between the movable & immovable tissues. Posterior Palatal Seal Landmarks: • Posterior outline is formed by the “ah” line (vibrating line) and passes through the two pterygomaxillary (hamular) notches and is 2mm anterior to the fovea palatini. Vibrating line IS AN AREA (imaginary line) that dictates the distal palatal termination of the maxillary complete

denture record base. In determining the posterior limit of a maxillary denture base the hamular notch is ON the posterior border. • Anterior outline-formed by the “blow” line (valsalva line), located at the distal extent of the hard palate. The blow line is anterior to the vibrating line which freely moves when the patient attempts to blow through the nose when it is squeezed tightly. Blow line is a close approximation to the junction of the hard and soft palate. PALATAL TORI-bony enlargements at the hard palate midline, occurring in 20-25% of the population, more prevalent in women. Usually covered by thinner and less resilient mucosa than the residual ridge, so it may act as a fulcrum and cause rocking of the maxillary denture. ► Because the soft tissues over the torus are generally thin with a poor blood supply, postoperative healing is slow. It is best to cover the operated site with a surgical stent lined with a sedative dressing. If a patient is having all maxillary teeth extracted at once, it is best to also remove the tori at that time. ► NOT usually removed for denture fabrication. However, mandibular tori ARE usually removed prior to denture fabrication.

Indications for Removing Palatal Tori: 1. Impinges on the soft tissue or the tori is undercut. 2. So large it fills the vault and prevents formation of an adequate denture base. 3. Extends too far posteriorly thus interferes with the posterior palatal seal. 4. Psychologically disturbing to the patient (cancer phobia). 5. Large palatal tori can cause problems with posterior palatal seal. Use a Y-incision to remove palatal tori directly over the tori. If a patient complains “when I smile, my upper denture doesn’t hold”, the area of the denture base needs to be adjusted is the buccal notch & buccal flange due to EXCESSIVE THICKNESS of the area. As the buccal frenum moves posteriorly during smiling or other facial expressions, it encroaches on the denture border that is too thick, causing the denture to become loose. MEDIAN PALATAL RAPHE (SUTURE)-area of very thin and tight attached tissue extending from the incisive papilla to the end of the hard palate. This is where the palatine processes of the maxilla join together. MAXILLARY DENTURE SHOULD BE RELIEVED IN THIS AREA.

ORAL PATHOLOGY DUE TO ILLFITTING COMPLETE DENTURES 1. Angular Chelitis-cracking in the corner of the lips secondary to chronic candidiasis, or caused by a LOSS of VERTICAL DIMENSION, or vitamin B deficiency. • Treat with anti-fungal therapy: Nystatin powder applied to denture undersurface 3x/day for 3-4 weeks, or reline or remake the denture. Nystatin rinse in ineffective! • Closed vertical dimension is the most likely cause of cheilosis in complete denture patients with a noncontributory medical history. 2. Inflammatory Fibrous Hyperplasia (Epulis Fissuratum)-begins as a traumatic ulcer secondary to an ill-fitting denture flange. Can be caused by continued denture wear and irritation. Treat: with SURGICAL EXCISION. First thing you must do is CUT THE DENTURE FLANGE BACK because the denture is OVEREXTENDED. Then consider tissue conditioning. • Denture-induced fibrous hyperplasia (Epulis Fissuratum), due to clefts found in the hyperplastic tissue, is also related to chronic trauma produced by

an ill-fitting denture. It occurs in the vestibular mucosa where the denture flange contacts tissue. Appears as PAINLESS FOLDS of fibrous tissue surrounding the overextended denture flange. If the amount of hyperplasia is minimal, tissue conditioning, fabrication of new dentures, and a change in denture habits may be sufficient to arrest tissue changes. However, surgical excision is usually required. The patient can also leave the denture out of the mouth. • The most likely tissue reaction to gross OVEREXTENSION of a complete denture that has been worn for a long time is an EPULIS FISSURATUM (caused by an ill-fitting denture flange). This cleft-like lesion are caused mainly by overextension of denture flanges. The overextension may result from long-term neglect or settling subsequent to residual ridge resorption. Traumatic occlusion of natural teeth opposing an artificial denture may also cause an epulis fissuratum. 3. Inflammatory Papillary Hyperplasia-secondary to illfitting MAXILLARY DENTURES and sometimes complicated by chronic candidiasis. Treatment: condition the tissues. Therapy: topical anti-fungal medication. In extreme cases, surgical excision. Frequently found under an ill-fitting denture, especially dentures with

a relief chamber produced in response to irritation from denture movement and accumulating food debris. The masses present as PAINLESS, FIRM, pink or red nodular proliferations of the mucosa. Candida Albicans may contribute to the inflammation. • Most patients are unaware of its presence. It usually involves only the HARD PALATE, but may also involve the residual ridges. IPH treatment depends on the lesion size. Although the nodules are not completely reversible, smaller papilla usually regress with treatment (removing the denture, soft relines, good oral hygiene, and Nystatin therapy). 4. Denture Stomatitis-a localized or generalized chronic inflammation (redness and burning) of the denturebearing mucosa. Discomfort may be or may not be present. Caused by denture trauma and secondary fungal infections. Treatment: improved oral hygiene, tissue rest, anti-fungal therapy (Nystatin), resilient tissue conditioners, and new, well-fitting dentures. The most important reason to treat hyperplastic tissue before making a complete or RPD is to provide a firm, stable base for the denture. Treat hyperplastic tissue: tissue rest, soft reline of existing dentures, change denture habits (not wearing them 24hrs/day), or surgical removal of tissue (for extensive tissue changes).

Combination Syndrome-caused when an edentulous maxilla is opposed by a partially dentate mandible (anterior teeth only), causing SEVERE BONE RESORPTION OF ANTERIOR MAXILLA. Thus,, during chewing, the denture tips anteriorly compress the mucoperiostieum of the pre-maxilla. • Characteristics: maxillary tuberosity hypertrophy (fibrous hyperplasia), occlusal plane problems, & premaxilla resorption. When a previously edentulous aged patient who now wears a complete maxillary denture against the 6 mandibular anterior teeth for many years, it is very common to have to do a reline due to loss of bone in the ANTERIOR maxillary arch. This is evident by a FLABBY MAXILLARY ANTERIOR RIDGE (loss of osseous structure in the anterior maxillary arch). A flabby, maxillary anterior ridge under a complete denture is frequently associated with RETAINED NATURAL MANDIBULAR ANTERIORS. • Flabby ridges are due to unstable occlusion and or excessive loading of tissues. Causes replacement of bone by fibrous tissue. Use a VERY FLOWABLE IMPRESSION MATERIAL to record flabby ridges like ZINC OXIDE EUGENOL PASTE. • RESILIENT LINER is indicated to improve/promote

healing, where there is very little ridge left, or when a maxillary natural teeth oppose a mandibular full denture (decreases pressure on the ridge and causes less damage to vascularity which decreases bone resorption. If you put pressure on incisive papillae, the patient feels burning. To treat relieve the incisive papillae area. A patient who wears a complete maxillary denture complains of a burning sensation in the palatal area of their mouth. This indicates too much pressure being exerted by the denture on the INCISIVE FORAMEN. INCISIVE PAPILLA-soft fibrous C.T. elevation that covers the incisive foramen (opening). RELIEF areas are the MEDIAL PALATAL SUTURE & INCISIVE PAPILLA when burning occurs or to not compromise blood supply. Burning sensation in the mandibular anterior area is caused by pressure on the MENTAL FORAMEN Patient returns to your office a few days after delivery of new dentures and complains of generalized irritation of the basal seat. Potential Causes: • PREMATURE OCCLUSAL CONTACTS = MOST common cause of generalized irritation of the basal seat. • Lack of denture hygiene, nutritional and hormonal imbalance.

• Excessive VDO METAL DENTURE BASE: metal is LESS irritating and MORE accurate fit to oral tissues than acrylic. • Has better thermal conductivity, so is better for tissues. • Increased strength allows for less bulk to allow more tongue space and better phonetics. • Increased weight for better mandibular denture stability.

IMMEDIATE DENTURES All new dentures should be evaluated 24hrs after delivery to correct any undetected errors. Tissue trauma attributed to denture function manifests as hyperemia, inflammation, ulceration, and pain. The basic sequence of the clinical procedure for a 24hr recall appointment is: 1. Remove the dentures from the mouth and thoroughly examine the mouth. 2. Ask the patient about the areas of tissue trauma observed. Permit the patient to describe additional complaints. 3. After collecting all diagnostic information, the dentist can determine the source of the problem and cure. During the first few days after inserting complete dentures, the patient should expect some difficulty in masticating most foods and excessive saliva due to reflex parasympathetic stimulation of the salivary glands. Over time, this will subside and return to normal. Ideal treatment is to fabricate the maxillary & mandibular immediate dentures simultaneously to avoid setting maxillary teeth to the likely malpositions of the remaining mandibular teeth.

• If the master casts are altered in an immediate denture procedure (e.g. elimination of gross undercuts), it is advisable to construct a second transparent denture base using a surgical stent or template. The stent is placed over the ridge after the teeth are extracted. Pressure points and undercuts are readily visible and surgical ridge correction can be performed. • Duplicating a master cast to construct a surgical stent/template that is to be used at the time of immediate denture insertion is best made after wax elimination and after the cast is trimmed. Do not make a 2nd denture set for at least 6 months. EXTRACTION STEPS PRIOR COMPLETE DENTURE DELIVERY:

TO

IMMEDIATE

1. Step 1: extract all posterior teeth EXCEPT a maxillary first premolar and its opposing tooth to provide a posterior “stop” to maintain the VDO. 2. Step 2: after the posterior residual ridges exhibit acceptable healing, the 2nd treatment phase (denture fabrication) can begin. Anterior teeth are extracted at the time of denture insertion. To help the patient get through the first day of wearing immediate dentures, instruct them to NOT REMOVE

THE DENTURES, eat soft foods, and return in 24hrs for the first adjustment/evaluation. KEEP THEM IN THE MOUTH FOR 24hrs after delivery. Immediate Complete Denture ADVANTAGES: • MAIN INDICATION IS ESTHETICS. • Ability to duplicate the position of the natural teeth. • Continuously acceptable esthetics as the patient is never without either natural or artificial teeth. • Improved speech adaptation. Immediate dentures require only one period of speech adaptation, while conventional denture treatment requires two (one after extractions and another after the dentures are delivered). • Protects extraction sites from trauma by acting as a bandage over the clot-filled sockets. • Continuously acceptable masticatory function. Patient retains some perception of chewing during healing. • Prevents tongue enlargement. When natural teeth are lost and not replaced, the tongue expands into the available space.

Immediate Complete Denture DISADVANTAGES: MAJOR disadvantage of immediate denture therapy is not being able to have an anterior tooth try-in to evaluate esthetics (anterior tooth try-in is impossible). Relining/rebasing the denture is required in 8-12 months. Relining is simple, but must be carried out within 8-12 months depending on the rate of alveolar ridge resorption. Also, increased post-delivery soreness for a few days can be encountered. Increased post-insertion care (including relining or remaking the dentures). Contour changes occur in the residual ridge during the 8-12 month healing period. Increased post-delivery soreness. The combination of post-extraction pain and denture-related trauma often produces greater discomfort during the first few days after insertion. Greater complexity of clinical procedures (i.e. border molding & final impressions are more difficult when natural teeth remain). Higher total cost of treatment due to the need for relines and repeated equilibration of the occlusion. Immediate dentures should be scheduled for RELINES

at 5 months and 10 months post-extraction to compensate for contour changes. Re-contouring of the healing ridge progresses rapidly for 4-6 months and does not stabilize in form until 10-12 months post-extraction. Due to this, immediate dentures become progressively more illfitting. This is a general timeline, as each case must be evaluated monthly and if necessary, relines performed. Reline Indications: any denture when diagnostic information indicates a reline a will effectively solve the patient’s chief complaint (when the denture record base adaptation is the major defect in the prosthesis. Reline Contraindications: excessive over-closure of vertical dimension (a large decrease in VDO). In this case, new dentures are indicated at the proper vertical dimension. After relining dentures, if a patient constantly returns for adjustments due to sore spots on the ridge, check the occlusion because the relining may have changed the CR contacts. INTERIM PARTIAL DENTURES CAN CAUSE TISSUE IRRITATION Immediate dentures: remove posterior teeth and allow tissue to heal to provide an area for support. Can keep the premolars to provide a VDO reference point.

OVERDENTURES COMPLETE OVERDENTURE-a denture whose base is constructed to cover all of an existing residual ridge and selected roots. Retained roots help prevent resorption of the alveolar ridges, improve denture retention, and allow the patient some proprioceptive sense of “naturalness” in function of the dentures. MOST important benefit of an overdenture (root-retained denture) is PRESERVATION OF THE ALVEOLAR RIDGE. • It is not always necessary to cover a root beneath an overdenture. However, if a root is not covered, the exposed surfaces are highly susceptible to decay. The patient’s oral hygiene must be impeccable to prevent root decay. • Retained roots are the most common findings when taking routine panoramic radiographs or patients who wear complete dentures (not necessarily over-dentures). ► For overdentures, retain mandibular canines bilaterally because they provide SUPPORT, NOT retention (a locator provides retention).

OCCLUSION ARCON ARTICULATOR (Articulated Condyle)-an articulator with its condylar elements on the LOWER member of the articulator and condylar path elements on the UPPER member. The angle between the condylar inclination and occlusal plane is FIXED on this articulator. ANGLE BETWEEN CONDYLAR INCLINATION & OCCLUSAL PLANE REMAINS CONSTANT (MORE ACCURATE). • Commonly used for diagnostic mounting of study casts to allow examination of occlusal contacts in the retruded contact position and analysis of tooth contacts during excursive movements of the mounted models. • Occlusal records in right and left lateral excursions are necessary for setting both the medial and superior condylar guides. • Fabrication of cast and porcelain restorations to ensure correct tooth contacts in occlusion and mandibular movements.

NON-ARCON ARTICULATOR (Non-Articulated Condyle)-has condylar elements on the upper member and condylar path elements on the lower member. • Angle between condylar inclination and occlusal plane is NOT CONSTANT (NOT FIXED) WHEN OPEN vs. CLOSED (LESS ACCURATE). This design is more popular to fabricate dentures. • Non-adjustable-has a SMALL AXIS OF ROTATION. • Semi-adjustable-gives a closer approximation of the axis of rotation & teeth and does not allow

intermediate tracking of condylar elements. • Fully adjustable: reproduces ALL border movements including progressive side shift and immediate side shift (BENNET’S MOVEMENT).

WORKING SIDE: teeth on the side the mandible is moving toward. When the mandible moves to the right and maxillary and mandibular teeth on the RIGHT side is the working side. For acceptable working contact, a denture must have the canine and at least 4 other cusps contacting the opposing teeth. BALANCING SIDE: the side OPPOSITE to the side the mandible is moving toward. When the mandible moves to the right, the maxillary and mandibular teeth on the LEFT is the balancing side (with natural teeth, the balancing side =

non-working side). For acceptable balancing side contact, at least 3 cusps must touch, but NOT THE CANINE. PROTRUSIVE-forward movement of the mandible during which there must be at least 3 points of contact (the anterior incisor, and one tooth on EACH side of the arch as far posterior as possible). • Protrusive record, records the relation of the maxilla & mandible, and is used to set the horizontal condylar guidance on the articulator. Made with mandibular anterior teeth 6mm forward of CR, or with mandibular & maxillary anterior teeth edge-to-edge). • Christensen’s Phenomenon-the space that opens b/t posterior teeth during anterior movement of the mandible. Amount of posterior separation is affected by both the incisal guidance and the horizontal condylar guidance. • Protrusive Movement-accomplished when the mandible is moved straight forward until the maxillary and mandibular incisors contact “edge-to-edge”. This movement is bilaterally symmetrical in that both sides of the mandible move in the same direction. The mandible can protrude ~10mm. CURVE OF SPEE-the two-dimensional ANTERIORPOSTERIOR CURVATURE of the MANDIBULAR

OCCLUSAL PLANE starting at the canine cusp tip, continuing through all posterior buccal cusp tips and anterior ramus, and ending at the mandibular condyle. • Compensating Curve-the anteroposterior and lateral curvature in the alignment of the occluding surfaces and incisal edges of artificial teeth used to develop a balanced occlusion. The form of the compensating curve is entirely under the dentist’s control (i.e. if during a try-in evaluation a dentist notes that a protrusive excursion movement results in the separation of posterior teeth, the problem can be corrected by simply increasing the compensating curve). • Compensating curve allows the dentist to alter the effective cusp angulation without changing the form of the manufactured denture teeth. The function of this curve is to help provide a balanced occlusion. A prominent compensating curve is required when there is a steep condylar path associated with a low degree of incisal guidance. CURVE OF WILSON-the MEDIOLATERAL U-SHAPED CURVE of the occlusal plane of maxillary and mandibular posterior teeth. BILATERAL BALANCED OCCLUSION-the stable simultaneous contact of opposing upper & lower teeth

in CR position with a smooth bilateral gliding contact to any eccentric position within the normal range of mandibular function, developed to lessen or limits tipping or rotation of denture bases in relation to the supporting structures. MAXIMUM NUMBER OF TEETH CONTACT IN ALL EXCURSIONS (CENTRIC & ECCENTRIC POSITIONS) for DENTURE STABILITY. Posterior Teeth Contacts in a Balanced Occlusion: ► Cusp-to-fossa contact in centric occlusion (MICP) in an ideal Class I occlusion. ► During lateral excursions, opposing cusps contact on the WORKING SIDE. ► During lateral excursions, on the balancing side, maxillary lingual cusps (lingual inclines) contact mandibular facial cusps (lingual inclines). Balanced occlusion occurs when a complete denture has balance on the working side, non-working side, and in protrusion. This assumes the denture has been constructed in proper CR and VDO. BILATERAL BALANCED IS THE OCCLUSION FOR COMPLETE DENTURES. 5 Factors Govern Establishing Balanced Articulation: 1. Inclination of the condylar guidance which is completely dictated by the PATIENT.

2. Inclination of the incisal guidance (horizontal and vertical overlap). 3. Inclination of the occlusal plane (plane of orientation). 4. Convexities of the compensating curve. 5. Angle and height of cusps. GROUP FUNCTION (“Unilateral Balanced Occlusion”)ALL TEETH ON THE WORKING SIDE CONTACT DURING WORKING MOVEMENT. ONLY working side contacts from anterior and posterior teeth and no nonworking side contacts. ALL posterior teeth on a side contact evenly as the jaw moves toward that side (working side). All teeth on the non-working side DO NOT contact. Only teeth on the working side contact during a lateral excursion. Non-working (balancing) interferences: occur on inner inclines of FACIAL cusps of mandibular molars. Working side (non-balancing) interferences: occur inner aspects of LINGUAL cusps of maxillary molars. Protrusive Interferences-occur between DISTAL inclines of FACIAL cusps of maxillary posterior teeth and MESIAL inclines of FACIAL cusps of mandibular posterior teeth. Purpose of making a

record of protrusive relation is to register the condylar path and adjust the condylar guides of the articulator so they equal the patient’s condylar paths.

VERTICAL DIMENSION OF OCCLUSION In a complete denture patient, when the teeth, occlusal rims, and central bearing point are in contact, and the mandible is in CR, the LENGTH of the face is the occlusal vertical dimension. Correct VDO is evaluated using 4 methods: 1. evaluating overall appearance of facial support. 2. visual observation of space between the occlusal rims at rest. 3. measurement b/t dots on the face (placed on the tip of nose and chin with a Thompson stick) when the jaws are at rest and when the rims are in contact. 4. observation when “s” sound is enunciated accurately and repeatedly to ensure adequate speaking space between the occlusal rims/occlusal plane. Excessive VDO may result in trauma to underlying supporting tissues. A CLOSED vertical dimension is the most likely cause of CHEILOSIS in patients who wear a complete denture with a non-contributory medical history. • Establish the VDO BEFORE making a CR record. • Teeth contact during swallowing, but NOT during speech. If teeth touch during speaking, VDO is too

great. When the mandible is in it’s physiologic rest (postural position), TEETH DO NOT CONTACT. Physiologic rest position occurs when the mandible and all of its supporting muscles (8 muscles of mastication + suprahyoids & infrahyoids) are in their resting posture (there is muscular equilibrium). This lack of tooth contact is the “freeway space” or “interocclusal distance” and averages 26mm. This position is a “muscle-guided” position and is the beginning and end point of most mandibular movements. Interocclusal Distance (“Freeway Space”)-the vertical distance or space created when the mandible is in its physiologic rest position between incisal and occluding surfaces of maxillary and mandibular teeth or occlusion rims. NEVER INCREASE the freeway space more than 1.5mm. Rest position of the mandible (postural position) is determined mostly by musculature. The usual reflex cited as the basis for the mandible’s postural position is the tonic stretch reflex of the elevator muscles. The rest position is a “muscle-guided” position. Vertical Dimension of Occlusion (VDO)-the vertical length of the face as measured between tow arbitrary selected points (one above and one below the mouth) when the teeth or any substitute material (occlusal rims) are in contact in

CR. Phonetics and esthetics help verify a patient’s vertical dimension of occlusion. Excessive vertical dimension (VDO): may result from trauma to underlying supporting tissues (denture patient), straining of the elevator/closing muscles, and adversely affects interocclusal clearance (decreased freeway space) causing loss of interocclusal distance in the rest position. Excessive vertical dimension is the usual causes of CLICKING OF DENTURE TEETH (to treat remount and fabricate a new complete denture). • Clicking of dentures can also be caused by lack of retention of the maxillary & mandibular dentures. To treat, if due to underextension, border mold and reline. If due to overextension, reduce as indicated with PIP and disclosing wax. • Porcelain teeth can also cause denture clicking. To treat, use acrylic resin teeth. Increased VDO is the usual cause of contacting/clicking of posterior teeth when a patient speaks. Decreased Vertical Dimension (VDO): an occluding vertical dimension causing EXCESSIVE INTEROCCLUSAL DISTANCE (increased freeway space) when the mandible is in the physiological rest position. • Example: people with no teeth or who have worn

dentures for a long time present with the lower portion of the face scrunched up, or do not show their lips anymore (poor facial profile). To correct, make new dentures and increase VDO. This decreases interocclusal distance (freeway space). • DECREASED VDO often results in CHEEK BITING. Factors to consider when verifying VDO: ♦ Pre-extraction records ♦ Amount of interocclusal distance (freeway space) to which the patient was previously accustomed. ♦ Esthetics (facial harmony and facial expression are considered). ♦ Phonetics (speech sounds). ♦ Length of the lip in relation to the teeth. ♦ Condition and amount of shrinkage of the ridges. Vertical Dimension of Rest (VDR)-the vertical length of the face measured between two arbitrary points (1 point above & 1 below the mouth) when the mandible is in the rest position. In a physiologically healthy individual, there is always a vertical space between the teeth (freeway space) when the mandible is in the rest position. This position is important in complete denture fabrication because it provides a guide to the VDO. VDR = Vertical Dimension of Occlusion (VDO) + Interocclusal Distance.

Balanced Centric Occlusion in partial dentures is necessary for appliance stability. The framework’s design and relationship of the teeth to the ridges also influence RPD stability. Bilateral Eccentric Occlusion is NOT an objective in RPD construction, UNLESS the partial prosthesis is opposed by a complete denture. The vertical relation for RPDs is usually determined by the remaining natural teeth (unlike complete dentures). Temporomandibular Joint (TMJ)-a combined hinge & gliding joint (ginglymoarthrodial joint) that permits both hinge-like rotation and gliding (sliding) movements. Ginglymus means “rotation” and arthrodial means “freely movable”. • In the lower (condyle-disc) compartment, only a hinge-type (rotary motion) can occur. This rotational or terminal hinge-axis opening of the mandible is possible only when the mandible is retruded in CR with a conscious effort by the patient or by the dentist’s control. A pure hinging movement is possible only in the terminal hinge position. • In the upper compartment (mandibular fossa-disc) only sliding movements (translation) can occur. When the lateral pterygoid muscles contact simultaneously, the discs and condyles slide forward

down over the articular eminence (protrusion), or can move backwards together (retrusion) during opening and closing of the mouth, respectively. Muscles Acting on the TMJ: • Elevator Muscles (Close) mandible: masseter, medial pterygoid, & temporalis (anterior fibers). Posterior fibers of temporalis retract the mandible. If the mandible fractures, UPWARD displacement of the fractured segment would be caused by the elevator muscles (masseter, medial pterygoid, & temporalis). • Depressor Muscles (Open) mandible/mouth: lateral pterygoid, anterior belly of digastric, & omohyoid. • Protrusion Muscles: lateral pterygoids together (individually, lateral pterygoids cause lateral excursion). Lateral pterygoids are mainly responsible for positioning and translating the condyles. CENTRIC RELATION (“Retruded Contact Position”)-a “ligament-guided” position that is the supero-anterior position of the condyle along the articular eminence of the condyle with the articular disc interposed between the condyle and eminence. This position is an optimum relative position between all anatomic components, and is a REPEATABLE reference position to mount casts on the

articulator. CR is the most unstrained, retruded anatomic and functional position of the mandibular condyle heads in the mandibular glenoid fossae of the TMJs. CR is a “bone-to-bone” relationship (bones of the upper and lower jaws) independent of tooth contact. The presence or absence of teeth and type of occlusion are not factors. Malposed or super-erupted teeth can cause a discrepancy between CR & CO, so opposing teeth should not contact when making a CR record to mount diagnostic casts because the contact causes the mandible to deflect or move away from CR. Mandible cannot be forced into CR from the rest position because the patient’s reflex neuromuscular defense would resist the applied force. Rather, the mandible should be relaxed and gently guided into CR. To place a patient in CR, have the patient swallow, turning the tongue upward towards the palate, relax the jaw muscles, or protruding and retruding the mandible can be effective ways to help record CR. In fixed and removable prosthodontics, CR should be established PRIOR to designing the frameworks.

When a CR record is taken in the natural dentition, imprints of the teeth should be confined to CUPS TIPS and the registration material should not be perforated. CR is a “ligament-guided” position. CR is the closing end-point of the retruded border movement (terminal-hinge movement). Transverse Horizontal Axis (Terminal Hinge Position)-the one relation of the condyles to the fossae in which a pure hinging movement is possible. In complete denture prosthodontics, the position of the planned MIC of the teeth in centric occlusion is established to coincide with the patient’s CR (CO = CR). Primary Requirements for making a CENTRIC RELATION record when fabricating a RPD: • Record the correct horizontal relation of the mandible to the maxilla. • Stabilize the lower record base with equalized vertical pressure. • Retain the record in an undistorted condition until the casts have been accurately mounted on the articulator or until a previous record can be verified.

Materials used to record jaw relationships have varied widely over the years. An ideal recording medium is easy to handle, uniformly soft while the record is being made, rapid setting, and totally rigid but not brittle when set. Rapid setting plaster zinc oxide & eugenol pastes, and modeling plastic are ideal. Avoid soft waxes as a recording material because they never become rigid and are likely to distort during the cast mounting procedure. If sufficient natural posterior occlusion exists, the mandibular cast may be mounted in CO using a ZOE reinforced wax bite. In the case of a distal extension RPD, base plates and occlusion rims should be placed on the framework, and the patient closed into softened recording wax or zinc oxide-eugenol paste (preferred). Whether this record will be in CO or CR depends on the case, and is dictated by the presence or absence of any natural posterior occlusion in the patient. Centric Occlusion (MICP)-a “tooth-guided” position defined as the maximum intercuspation of the teeth. During typical “empty mouth swallowing” the mandible is braced in the inter-cuspal position. • Empty mouth swallowing occurs frequently during the day and is an important to rid the mouth of saliva, and moisten the oral structures. The hourly rate of non-masticatory swallowing is related to the amount

of salivary flow and is usually an involuntary reflex activity. • Masseter muscles contract and the tongue tip touches the roof of the mouth during normal swallowing. • Tooth contacts are longer during swallowing than chewing, but this varies among people. Articulation-relationship of teeth during movements into and away from the eccentric position, while the teeth contact. CONDYLAR GUIDANCE-a factor TOTALLY dictated by the patient. It is the mechanical device on an articulator intended to produce similar guidances in articulator movement that are produced by the condyle paths during mandibular movements. • Condylar guidance is completely dictated by the patient and cannot be varied or “adjusted” by the dentist. • The inclination of condylar guidance depends on: shape & size of the bony contour of the TMJ (fossae and disc), action of the muscles attached to the mandible, limiting effects of the ligaments, and the method used for registration.

• The incline (angulation) of the condylar element on the articulator is anatomically related to the slope of the condylar articular eminences (condylar inclination). • When adjusting the condylar guidance for protrusive relationship, the incisal guide pin on the articulator should be raised out of contact with the incisal guide table. The protrusive record is probably the LEAST reproducible maxillomandibular record. • When restoring the entire mouth with crowns, the protrusive condylar path inclination influences the mesial inclines of the mandibular cusps. • RETRUSIVE MOVEMENT requires the condyles to move BACKWARD & UPWARD. • LATERAL MOVEMENTS, the working condyle moves down, forward, and laterally, and nonworking condyles move down, forward, and medially. • The inclination of the condylar path during protrusive movement forms an average angle of ~30° with the horizontal reference plane. If the protrusive inclination is steep, the cusp height may be obviously longer. If the inclination is shallow, the cusp will be shorter. This factor is the MOST important aspect of

condylar guidance that affects the selection of posterior teeth with appropriate cusp height. • In complete dentures, the condyle path during free mandibular movements is governed mainly by the SHAPE of the fossa and meniscus (articular disc) and the muscular influence. DETERMINANTS OF OCCLUSION: • Right & left TMJs • Occlusal surfaces of teeth and the neuromuscular system. The concepts of occlusal arrangement aim to place artificial teeth in harmony with the TMJ and neuromuscular system. If done properly, it results in minimum stress on the teeth and requires minimal effort by the neuromuscular system when performing mandibular movements. Optimal occlusion requires minimum adaptation by the patient. 4 Features that Directly Effect PDL Health & Hard Tissue Anchorage to Resist Occlusal Force: 1. anterior teeth have slight or no contact in MICP (intercuspal position). 2. occlusal table is < 60% of the overall F-L width of the tooth. 3. occlusal table is at right angles to the tooth’s long axis. 4. mandibular molar crowns are inclined 15-20° toward

the lingual. 4 Determinants Required to Restore a Complete & Functional Occlusal Surface of a Tooth: 1. Amount of vertical overlap of anterior teeth. The anterior determinant of occlusion is the horizontal and vertical overlap relationship of the anterior teeth. 2. Contour of the articular eminence. 3. Amount and direction of lateral shift in the working side condyle. 4. Tooth position in the arch. However, the jaw relationship most commonly used in the ACTUAL design of restorations is the ACQUIRED centric occlusion. The height of the pulp horn of a particular tooth is NOT a required determinant to restore a complete and functional occlusal surface. OCCLUSAL PLANE-an imaginary surface related anatomically to the cranium and theoretically touches the incisal edges of the incisors and tips of the occluding surfaces of the posterior teeth. It is not really a plane, but represents the mean curvature of the surface. • Anterior point of the occlusal plane is determined by the position of the anterior teeth. • Posterior determinants are anatomical landmarks (2/3 the heights of the retromolar pads). Thus, it is

debatable as to the extent of control the dentist may exercise over the orientation of the occlusal plane. CUSP INCLINATION-angle made by the slopes of a cusp with a perpendicular line bisecting the cusp, measured mesiodistally or buccolingually. Cusp inclination is under the dentist’s control (choosing 30° teeth, monoplane teeth, etc.). • In a protrusive condylar movement (protrusion), interferences can occur between DISTAL inclines of maxillary posterior cusps and MESIAL inclines of mandibular posterior cusps. • In a protrusive movement, the mandibular condyles move DOWNWARD & FORWARD. • During protrusive movement, there are occlusal contacts occurring on the maxillary distal inclines and mandibular mesial inclines. Anteriorly, the facial surface of the mandibular incisors will contact the guiding inclines (lingual) of the maxillary incisors and canines. • In any restorative case involving ALL teeth in the mouth, the protrusive condylar path inclination will have its primary influence on the same inclines (distal of maxillary & mesial of mandibular).

• The pathway followed by the anterior teeth during protrusion may not be smooth or straight because of contact between the anterior teeth and sometimes the posterior teeth. Centric Interference (Forward Slide)-corrected by grinding MESIAL INCLINES of maxillary teeth and DISTAL INCLINES of mandibular teeth. This group function of teeth on the working side evenly distributes the occlusal load. While, the lack of contact on the non-working side prevents those teeth from receiving destructive, obliquely directed forces found in non-working interferences, and saves the centric holding cusps (i.e. mandibular buccal cusps and maxillary cusps) from excessive wear. The advantage is the maintenance of the occlusion. MUTUALLY PROTECTED OCCLUSION (“Canine Guided” or Organic Occlusion)-anterior teeth protect the posterior teeth in all mandibular excursions. Canines DISCLUDE the posterior teeth during working and nonworking movements. • Canine Guidance is an occlusal relationship exists where the vertical overlap of maxillary & mandibular canines causes disclusion (separation) of ALL posterior teeth when the

mandible moves to either side. All other teeth do not contact once they move from CR. If there is contact of other teeth, “working side” or “non-working side” interferences occur depending on which side the mandible moves towards. • When placing a crown on a maxillary canine, if you change a canine protected occlusion to group function, you increase the chance for “non-working side” interferences. ANTERIOR GUIDANCE (Anterior Coupling)-result of horizontal & vertical overlap of anterior teeth. A tightly overlapping relationship of opposing maxillary and mandibular incisors and canines that DISCLUDE the posterior teeth when the mandible protrudes and moves in lateral excursion. Anterior guidance also affects the surface morphology of posterior teeth. The greater overlap, the longer the cusp height. INCISAL GUIDANCE-a measure of the amount of movement and the angle at which the lower incisors and mandible must move from the overlapping position of centric occlusion to an edge-to-edge relationship with the maxillary incisors. Incisal guidance is the second endcontrolling factor in articulator movement and is to some degree, under the dentist’s control. Influencing factors are esthetics, phonetics, ridge relations, arch space, and inter-ridge space.

• Esthetics & phonetics are the main factors the LIMIT a dentist’s control of incisal guidance. • Incisal guidance on the articulator is the mechanical equivalent of horizontal and vertical overlap. SUPPORTING CUSPS (“Stamp Cusps” or “Centric Cusps”)-these cusps contact the opposing teeth in their corresponding F-L center on a marginal ridge or fossa. These cusps are more robust and better suited to CRUSH food. When posterior teeth are in a normal ideal relationship, maxillary lingual cusps + mandibular buccal cusps are considered the SUPPORTING CUSPS. Centric stops-areas of contact that a supporting cusp makes with opposing teeth (i.e. the ML cups of the maxillary 1st molar (a supporting cusp) makes contact with the central fossae (centric stop) of the mandibular 1st molar. 5 Characteristics of Supporting Cusps: 1. contact the opposing tooth in the intercuspal position. 2. support the vertical dimension of the face. 3. are closer to the F-L center of the tooth than non-supporting cusps. 4. their outer incline has a potential for contact. 5. have broader, more rounded cusp ridges than non-supporting cusps.

NON-SUPPORTING CUSPS (“Guiding” or “Shearing” Cusps): maxillary buccal (facial) cusps + mandibular lingual cusps. These cusps overlap the opposing tooth without contacting the tooth and have narrower and sharper cusp ridges that serve to SHEAR food as they pass close to the supporting cusp ridges during chewing strokes. • The inner occlusal inclines leading to the guiding cusps are “guiding inclines” because in contact movements, they guide supporting cusps away from the midline. Thus, there are the bucco-occlusal inclines (lingual inclines of buccal cusps) of the maxillary posterior teeth, and the linguo-occlusal inclines (buccal inclines of lingual cusps) of the mandibular posterior teeth. In a posterior cross-bite situation, supporting and guiding cusps are opposite. The maxillary buccal and mandibular lingual cusps are now the supporting cusps and the maxillary lingual and mandibular buccal are the guiding cusps. SELECTIVE GRINDING-the reduction of occlusal interferences usually done BEFORE constructing a fixed bridge or denture for a patient to PREVENT duplicating the deflective occlusal contacts in the final restoration. The purpose of selective grinding is to remove all interferences without destroying cusp height. Thus, when interferences exist in centric, but not in

lateral excursions, the fossa or marginal ridge opposing the premature cusp is deepened. It is important that whenever a prematurity is found, the occlusion be checked in all centric positions before any adjustment is made. If cusps interfere with each other in excursions, then only the non-holding cusps are ground to prevent a decrease in VDO. • A common case where it is preferable to selectively grind AFTER a fixed bridge or RPD is in place is when a FPD or RPD is to be constructed for a space over which the opposing tooth has extruded slightly. The bridge or partial is frequently constructed to the ideal plane of occlusion and the opposing tooth is adjusted after insertion. • The most common complaint after cementation of a fixed bridge is sensitivity to hot & cold and indicates a deflective occlusal contact. The involved teeth may be sensitive to touch and when brushing. In these cases, an immediate correction of occlusion must be made. • Important: if you plan on changing a patient’s vertical dimension using crowns, it is critical to mount the casts on the true hinge axis (face bow). With Selective Grinding in Complete Fabrication in Centric Relation (CR):

Denture

► Secondary centric holding cusps are the mandibular buccal cusps. Grind these cusps only if there is a balancing side interference. ► Primary centric holding cusps are the maxillary lingual cusps. Never grind these cusps. ► Ideally, selective grinding should result in harmonious cusp-fossa contacts of all upper and lower fossa (and marginal ridges of premolars). Do NOT grind the upper lingual or lower buccal cusps. A forward slide from CR can be corrected by grinding the mesial inclines of maxillary teeth and distal inclines of mandibular teeth. ► Selective grinding of inner inclines of secondary centric holding cusps is done if a balancing (nonworking) side interference exists. ► Only grind cusp tips on maxillary buccal & mandibular lingual (BULL) cusps if they are premature in centric, lateral, or protrusive movements. Check before grinding. ► Selective Grinding in Working Side (nonbalancing side) Relation: the rule for selective grinding interferences during working side movements follows the rule of BULL (buccal cusp inner inclines of upper teeth & lingual cusp inner

inclines of lower teeth). ► Selective Grinding in Non-Working Side (balancing side) Relation: grind the inner inclines of mandibular buccal cusps, and NEVER GRIND MAXILLARY LINGUAL CUSPS (primary centric holding cusps). Basic Principles of Occlusal Adjustment: 1. The maximum distribution of occlusal stresses in centric relation (CR). 2. Forces of occlusion should be borne as much as possible by the long axis of teeth. 3. When surface-to-surface contact of flat cusps occurs, it should be changed to a “point-to-surface” contact (the cusp tip of the tooth occludes with the flat surface of it’s opposing tooth’s cusp). 4. When centric occlusion is established, NEVER take the teeth out of centric occlusion. 5. Never adjust cusp tips. Only marginal ridges and fossa.

DENTURE TEETH SELECTION SMILE LINE (INCISAL CURVE)-composed on the incisal edges of maxillary anteriors and parallels the inner curvature of the lower lip. Parallel with the inter-pupillary axis and perpendicular to the midline (middle of the face). • PHI (1.618:1) EACH NUMBER IS THE SUM OF THE TWO NUMBERS PRECEEDING IT. The size of the front teeth should become progressively smaller as you proceed toward the back of the mouth in the ratio of 1.6 to 1.0 to 0.6 = Golden Proportion.

• Long axis of posterior teeth are inclined toward the LINGUAL. • Long axis of maxillary incisor crowns CONVERGE slightly toward the midline.

• There should be slight irregularities on either side of the midline, even though the teeth are similar in size, shape, and alignment. • In younger patients, maxillary incisors are more prominent, while mandibular incisors are more visible with age. Guides for selecting artificial denture teeth for edentulous patients: pre-extraction records, teeth of close relatives, diagnostic casts, radiographs, photographs, extracted teeth, and following the lines placed on the occlusal rims. RULES FOR SETTING DENTURE TEETH: • Incisal edges of maxillary central incisors and cusp tips of canines lie on the SAME CURVED LINE with the incisal edges of lateral incisors 1mm above the same line. • Interproximal contacts of the maxillary anterior teeth are situated progressively closer to the gingiva the more distal they are from the midline. • Incisal embrasure become progressively LARGER from the central incisor, lateral, to canine (the more posterior you go). Incisal embrasures in younger patients become smaller, sometimes to the point of disappearing as the teeth wear.

• MANDIBULAR POSTERIOR teeth are placed over the crest of the residual ridge. MAXILLAR POSTERIOR functional cusps are placed in the fossae of mandibular teeth, and can be no farther than the facial vestibule. • MAXILLARY ANTERIOR teeth are set FACIAL to the ridge for phonetics and esthetics. Canine cups tips should be parallel to the posterior border of the incisive papilla. MANDIBULAR ANTERIOR teeth are set base on the maxillary anterior teeth. Primary role of ANTERIOR TEETH on a denture is ESTHETICS. Spaces, lapping, rotation, and color changes can be judiciously used to create a natural appearance. Setting anterior teeth too far lingually or facially to satisfy esthetic concerns should NOT be done. When selecting teeth, pre-extraction records are extremely valuable. Maxillary and mandibular anterior teeth should NOT contact in centric relation. Setting maxillary and mandibular anterior teeth so they contact in CR produces an unsatisfactory arrangement of artificial teeth for complete dentures. Error that most often contribute to poor denture esthetics is placing maxillary anterior teeth directly over the edentulous ridge. Maxillary anterior denture teeth should be placed slightly ANTERIOR to the ridge.

The outline of anterior teeth should harmonize with the face form. Convex profile faces should have a similar convex labial surface of anterior teeth. Broader contact areas of teeth look more natural on dentures as they are more compatible with aging. For best esthetics, maxillary anterior teeth in a complete denture are arranged FACIAL TO THE RIDGE. Setting anterior teeth directly over the ridge causes poor esthetics. Also, it is important to have accurate adaptation of the border seal and adequate bulk of the maxillary facial flange for good esthetics. VDO also affects the lip support. Maxillary central incisors are the MOST important teeth for esthetics. Their placement controls the midline, speaking line, lip support, and smiling line composition. For most patients, the labial surface of the central incisor should be ~8mm anterior to the center of the incisive papilla. The labioincisal 1/3 of maxillary central incisors should support the lower lip when the teeth are in occlusion. FUNCTIONAL needs overshadow esthetic needs when selecting POSTERIOR TEETH. Do not set mandibular molars over the ascending area of the mandible as occlusal forces in the area dislodge the mandibular denture. Size of POSTERIOR TEETH for a RPD is determined primarily by the amount of useful posterior tooth space and characteristics of the denture-supporting tissues.

Other factors relevant to selecting posterior RPD teeth include: 1. Occluso-gingival length: MOST important factor to determine posterior tooth length is available inter-arch space. 2. M-D width: the total MD space available for posterior teeth is determined by measuring from the distal of the lower canine to the point where the mandibular residual ridge begins to slope upward. 3. B-L width: the BL width is narrowed in relation to the missing natural tooth. Reducing the area of the occlusal table decreases stress transferred to the denture support area during food bolus penetration. Also, reducing the B-L width increases tongue space. 4. Shade: posterior tooth shade of is usually selected to harmonize with the anterior teeth. 5. Occlusal surface form: no superior tooth form or arrangement is identified. Thus, it is logical to use the least complicated approach that fulfills the patient’s needs. 6. Materials: plastic bonds well to acrylic resin. Thus, plastic teeth are retained better than porcelain teeth. Primary reason for using PLASTIC teeth in a denture is because plastic teeth are retained well in acrylic

resin. Common Errors Made When Arranging Denture Teeth: ♦ Setting mandibular anterior teeth too far forward to meet the maxillary teeth. ♦ Failure to make canines the turning point of the arch. ♦ Setting mandibular first premolars buccal to the canines. ♦ Establishing the occlusal plane by an arbitrary line on the face. ♦ Not rotating anterior teeth enough to give an adequately narrower effect. Lower 1/3 of a patient’s face appears too short and there is apparent loss of the vermilion border of the lips. The procedure indicated to correct this situation is increasing occlusal vertical dimension (VDO). Errors in occlusion are checked most accurately by REMOUNTING the dentures on the articulator using remount casts and new inter-occlusal records. Potential Problems with New Dentures: 1. Cheek Biting is caused by: • Posterior teeth set edge-to-edge. Treat by reducing BUCCAL CUSPS of mandibular molars to create proper horizontal overlap.

• Inadequate VDO. Treat by relining dentures at the corrected VDO, CR remount, and fabricate a new denture. • Biting corners of the mouth. Treat by RESET CANINES & PREMOLARS. 2. Lip Biting-caused by reduced muscle tone and/or a large anterior horizontal overlap (overbite). 3. Tongue Biting-caused by having posterior teeth set too far lingually. 4. Generalized speech difficulty with complete dentures is usually caused by faulty tooth position and/or faulty palatal contours. 1. Speech problems due to faulty tooth position are avoided by placing the denture teeth as close as possible to the position of the natural teeth. Note: the most effective time to test for phonetics is at the time of the wax try-in of the trial denture (usually the 4th appointment). 2. Faulty palatal contours are corrected by trial and error. Add wax to increase contours and remove wax to improve articulation of sounds.

PHOENETICS Patients edentulous for many years often have more distorted speech than patients edentulous for a short time due to a loss of tonus of the tongue musculature. “S” Sound: mandibular incisal edges should be even with or just lingual the maxillary teeth incisal edges. Formed when the tip of their tongue approaches the anterior palate and lingual surfaces of maxillary teeth. These sounds bring the mandible and maxilla close together. “S” sounds are the speech sounds that bring the mandible closest to the maxilla. • If a patient complains when he/she tries to make an “s” sound, it sounds like “th”, the two most probable causes is either the maxillary incisors are set too far palatally, or the palate is made too thick. • Words with the sibilant sound (hissing sounds) are pronounced correctly with the incisal edges of maxillary and mandibular almost touching. These sounds are usually produced between rest and the occluding position. • Incisal edges of mandibular incisors are established by occlusal contact with maxillary incisors and by their

position 1mm behind (lingual) and 1mm below the maxillary incisal edges when saying “S”. Incisal edges of mandibular incisors are 1mm anterior and lingual to maxillary incisal edges when making “S” sound. • SIBLANTS (CH, J, S)-produced by maxillary and mandibular incisors approximating each other. Palate and tongue controlling valve. “Th” LINGUODENTAL sound: tongue should protrude slightly (2-4mm) between maxillary & mandibular anterior teeth to form this sound. Made by putting tongue b/t max and mandibular teeth (1/8 inch (3mm) tip of tongue should be visible. “F” & “V” LABIODENTAL sounds: formed by incisal edges of the maxillary incisors and lower lip (incisal edges should just touch the wet/dry line of the lower lip). “P” & “B” LABIAL sounds: are formed totally by the lips. Made by pressure behind the lips. If teeth are not set correctly, it can affect this seal of lips to build-up pressure. P & B sounds are affected by: anterior-posterior position of teeth, incorrect VDO, and labial flange thickness. “T” & “D” (ANTERIOR LINGUAL PALATAL SOUNDS): If teeth are set too FAR LINGUAL, “t” sounds like “d”. If the teeth are set too FAR LABIAL, then “d” sounds like “t”. A patient who wears complete dentures is having difficulty

trouble pronouncing the letter “t” due to incorrect positioning of the maxillary incisors. Made by tip of tongue touching the anterior palate. “K” & “G” VELAR SOUNDS (POSTERIOR LINGUAL PALATAL): produced when tongue touches posterior palate. Tooth set-up DOES NOT AFFECT VELAR SOUNDS. A high palatal vault or a constricted palate can cause whistling sounds. Whistling during speech with dentures (complete or RPD that replaces the incisors) can be caused by either insufficient vertical overlap (overbite), excessive horizontal overlap (overbite), or the area palatal to the incisors is improperly contoured.

FIXED PARTIAL DENTURES (FPDs) TEMPORARY (PROVISIONAL) RESTORATIONS MUST PROVIDE: 1. Pulpal protection: restoration must be fabricated from a material to prevent conducting temperature extremes. Margins should be adapted well enough to prevent saliva leakage. 2. Positional stability: the tooth should not extrude or drift so INTERPROXIMAL CONTACTS PROVIDE THIS. 3. Occlusal function: the temporary’s ability to function occlusally aids in patient comfort & prevents tooth migration, and prevent joint/neuromuscular imbalance. 4. Easily cleaned: the temporary must be made of a material and contour that the patient can keep clean. 5. Non-Impinging Margins: it is VERY important that the temporary’s gingival margins do not impinge on the gingival tissues to prevent inflammation that can cause hypertrophy, gingival recession, bone loss. Margins should be well polished. An OVERHANG can result from

a preformed metal or resin provisional improperly contoured, while a CUSTOM PROVISIONAL can cause horizontal overhang if improperly trimmed. 6. Strength & retention: temporary must withstand the forces it is subjected to without breaking or coming off. 7. Esthetics: if the temporary is on an anterior tooth, it must provide a good cosmetic result. CUSTOM INDIRECT TECHNIQUE (outside the mouth in the LAB) FOR MAKING TEMPORARY CROWNS IS PREFFERED BECAUSE IT IS MORE ACCURATE, BETTER FIT, & PROTECTS THE PULP because when poly(methyl-methacrylate), is placed on freshly cut dentin, (as in a direct technique), it can cause thermal irritation and acute pulpal inflammation. Can make custom provisional (temporary) using over-impressions, templates, or a thin shell crown.

CROWNS RETENTION: prevents removal of the restoration along the path of insertion or long axis of the preparation. • Achieved by two opposing vertical surfaces (i.e. buccal & lingual walls). • Tooth preparation taper is kept to a minimum to enhance retention (as taper decreases, retention increases). Always try to preserve as much tooth structure as possible when preparing the tooth for a crown using ideal prep dimensions. • Greater surface area of a preparation, the greater retention (boxes and grooves increase surface area. • Maximum retention is achieved when there is only one path of draw. • PINS INCREASE RETENTION by increasing length internally and apically (not externally). RESISTANCE: prevents DISLODGEMENT of the restoration by forces directed apical or oblique and prevents any movement of the restoration under occlusal (vertical) forces.

A crown must be STRUCTURALLY DURABLE to withstand occlusal forces, it must have marginal integrity (always check clinically with explorer and take a pre-cement radiograph to ensure all margins are closed). IDEAL TAPER FOR A CROWN is 5-6° (2.5-3.0° inclination on each opposing axial wall). Axial walls of a crown preparation should taper no more than 3-6°). Guiding grooves placed in the crown preparation provide: Resistance to rotation, Retention, & path for seating the crown. MAIN purpose of a buccal or lingual groove in a single crown preparation is IMPROVE CROWN RETENTION. OCCLUSAL CLEARANCE is one of the most important features to provide adequate bulk of metal and strength. GOLD Crown = 1.5mm clearance functional cusps (lingual max, buccal mand). 1mm for non-functional. PFM Crown = 1.5-2mm functional cusps; 1-1.5mm non-functional cusps. ALL-CERAMIC Crowns = 2mm clearance on preparations. A wide FUNCTIONAL CUSP BEVEL provides space for adequate bulk of metal and prevents perforation of the

metal due to heavy occlusal contact. PFM COPING ALLOY: 1. High Noble alloys: used to fabricate metal-ceramic restorations (PFMs) consist of 98% gold, platinum, & palladium (with trace elements). These noble alloys (gold, platinum, palladium) do not oxidize on casting. This feature is important in a metal substrate so that oxidation at the metal-porcelain interface is controlled by adding trace elements to the metal (silicon, indium, iridium). This is the BEST TO USE. • Gold-Platinum-Palladium; Gold-Palladium-Silver, Gold-Palladium. 2. Palladium-Silver alloys (Noble): 50-60% palladium + 30-40% silver (not a noble metal, thus oxidizes on casting). 3. Nickel-Chromium alloys (Base metal alloys): 70-80% nickel + 15% chromium. These base metal alloys readily oxidize and can create porcelain-to-metal interface problems. Nickel-Chromium; NickelChromium-Beryllium; Cobalt-Chromium. Practical Note: many dentists have shifted from using PFM copings for single unit and 3-unit bridges, and are now using zirconium. PFMs are still used for larger unit bridges.

4 Coping Features: 1. Thickness of metal and joining porcelain. Noble metal (0.3-0.5mm thickness). Base metal (can be as thin as 0.2mm, and must have a higher yield strength and melting temperature). 2. Placement of occlusal and proximal contacts. 3. Extension of the areas to be veneered for porcelain. 4. Facial margin design ABSOLUTE MIMIMUM PORCELAIN THICKNESS is 0.7mm (ideal is 1mm). CROWN MARGINS: 1. Bevel (Feather-Edge) Margin-the best finishing margin for CAST FULL GOLD restorations, allowing burnishing and adaptation of the gold to the tooth. However, in practice it is difficult to read on the impression and die, and may lead to inaccurate extension and distortion of the wax pattern, and subsequent casting, as a result of the thin wax. It also has the LEAST MARGINAL STRENGTH to the casting. An acute edge/angle with a nearby bulk of metal is the optimum margin for a casting because it is easily burnished to improve crown fit. 2. Chamfer Margin-the PREFERRED FINISHING LINE for cast full gold restorations. The resultant casting has sufficient marginal strength while allowing the sliding

joint at its periphery to minimize the gap between the tooth and preparation, thus reducing the cement thickness. Combines the advantage of an easily definable margin on the impression and die, with minimal tooth preparation. Preferred gingival finish line for veneer metal restorations. 3. Shoulder Margin (Butt Joint)-finishing line of choice for ALL PORCELAIN/EMAX crowns. Edge strength of porcelain is low, thus a BUTT JOINT is required. Shoulder provides resistance to occlusal forces and minimizes porcelain stresses. The margin is easily read on the impression and die. Main disadvantage is any inaccuracies in the crown fit are reproduced at the margin, causing increased thickness of cement. SHOULDER MARGIN (butt joint) is the POOREST finish line used with cast metal restorations. • Unlike the PFM restoration which accepts any marginal design (bevel, chamfer, shoulder), marginal tooth preparation for the ALL-ceramic crown or porcelain jacket crown MUST BE A SHOULDER. • Radial Shoulder-a modified form of a shoulder used on all-ceramic crowns that combines maximum support of the ceramic, with a stress-reducing rounded gingioaxial angle.

• Heavy Chamfer-can be used on all-ceramic crowns (but not as good as a shoulder). A bevel can be added for metal restorations. • All-Ceramic Crown margin design is INTERNALLY ROUNDED SHOULDER. • Main reason to use porcelain jacket crowns and allceramic crowns is ESTHETICS. These crowns can mimic the optical properties of a natural tooth. However, the guidelines for usage, such as tooth preparation are more critical and more complicated than for PFM restorations. It is advisable to use all ceramic crowns only in the anterior region where esthetics is critical. • All-ceramic crowns are known for their LOW FLEXURAL STRENGTH (this inability to flex is the major weakness of all-ceramic crowns). Their relative tendency to fracture at a minimum deformation. Microscopic surface defects, under load lead to crack propagation and eventually to failure. 4. Shoulder with a Bevel-this margin allows a sliding fit to occur at the margin, thus may be used on the proximal box of inlays and occlusal shoulder of the mandibular ¾ crowns, or labial margins of PFM crowns (metal ceramic). If these margins are placed in the

gingival crevice (subgingival), little display of metal is seen. Can be used for metal-ceramic (PFM) with metal collars. Used as the finish line on the proximal box of inlays and onlays, and occlusal shoulder of onlays and ¾ crowns. Also used for the facial finish line of PFM restorations where gingival esthetics is not critical. Periodontium remains healthier when crown margins are ABOVE THE GINGIVAL CREST (SUPRAGINGIVAL), however, supragingival margins are often not possible due to esthetics or caries, so the margins must be placed subgingivally. If a margin must be placed subgingivally, the major concern is NOT TO EXTEND the preparation into the tooth’s attachment apparatus (invade biologic width). If the margin extends into the biologic width, a constant gingival irritant occurs and ultimately the crown will fail. In this case, the tooth should have crown lengthening performed PRIOR to final crown preparation. CROWN LENGTHENING may be done to surgically move the ALVEOLAR CREST 3mm apical to the proposed finish line (margin) to maintain biologic width and prevent periodontal pathology. EMERGENCE PROFILE: the axial contour that extends from the base of the sulcus, past the free gingival margin. It extends to the tooth’s height of contour to produce a STRAIGHT LINE PROFILE in the gingival 1/3 of the axial surface. A STRAIGHT LINE ACCESS (EMERGENCE

PROFILE) IS THE TX GOAL WHEN RESTORING TEETH, because it facilitates ACCESS FOR GOOD ORAL HYGIENE (toothbrush bristles can reach into the sulcus). • The most common error is creating a bulge or excessive convexity. FEATURES OF THE ANTERIOR PFM PREP: zirconium, PFZ, and all porcelain are more commonly used today for anterior crowns, although it is still important to know features of an anterior PFM preparation. 1. Radial shoulder: periodontal preservation and structural durability. 2. Chamfer: marginal integrity and periodontal preservation. 3. Axial reduction: retention and resistance and structural durability. 4. Incisal notch: structural durability. 5. Wing: retention and resistance, & preservation of tooth structure.

PORCELAIN SHADE SELECTION Dental Porcelain-a mixture of FELDSPAR (main constituent), QUARTZ, & metallic oxides used to impart proper shade to the porcelain. When feldspar undergoes fusion, it forms a glassy material, which gives porcelain its translucency. It acts as a matrix for the high-fusing quartz, which then forms a refractory skeleton for the other materials to fuse around. Porcelain’s compressive strength is GREATER than it’s tensile or shear strengths. Dental porcelain restorations are BRITTLE and are not capable of much plastic deformation. 3 Types of Dental Porcelains: 1. High-fusing porcelains-used to manufacture DENTURE TEETH. 2. Medium-fusing porcelains-used for all-ceramic and porcelain jacket crowns. Medium fusing porcelains also contain oxides of lithium, magnesium, and phosphate (in addition to silicone dioxide, aluminum oxide, potassium oxide, and sodium oxide). 3. Low-fusing porcelains-used for metal-ceramic (PFM) crowns. Aluminum oxide-agent added to low-fusing porcelains during its manufacture to increase its

resistance to “slumping down” during firing. Metamerism-a phenomenon that causes teeth/porcelain to appear color matched under one light source, but appear very different under another light source (appears different under different lights). This property is important in matching the shade of a PFM crown to a natural tooth. SHADE of a ceramic crown is matched first based on the color’s value, chroma, then hue. 1. Value-a color’s brightness. The most critical characteristic that is matched first. Value is the relative amount of lightness or darkness in a color (intensity of a color). In esthetics, the value of a denture tooth depends on the relative whiteness or blackness of its color. Staining a porcelain restoration or using a complementary color will reduce the value. It is almost impossible to increase the value. A 60-year old patient, compared to a 25-year old patient is most likely to have teeth with a color that is lower in value and higher in chroma. 2. Chroma-a color’s strength or saturation. Aspect of color that indicates the degree of SATURATION of the hue. The single most important factor in shade matching that is successfully increased by using stains.

3. Hue-the basic colors (color families) like red, blue, yellow, green. Drastic changes of hue (color or shade) are often impossible. Orange stain is most often used to change the hue. 3 light sources used in the dental office: 1. Natural Light-the best light to select porcelain shade. 2. Incandescent-lacks blue, but increase in RED & YELLOW. 3. Fluorescent-decrease in red, but increase in BLUE & GREEN. DO NOT place the rubber dam on before selecting tooth shade. Teeth should be clean and make-up removed from the patient’ face.

PONTIC DESIGN

PONTIC: the suspended member of a fixed bridge that replaces a missing tooth that MUST provide patient comfort, convenient contours for hygiene, and be esthetic. • Proper design is more important to cleanability and good tissue health than is choice of materials. • Excessive tissue contact is a major factor in the failure of FPDs. • The contact area between the pontic & ridge should be small and the part touching the ridge should be CONVEX. • Pontic tip should not extend past the mucogingival junction to prevent ulceration. • Pontic should only touch ATTACHED KERATINIZED GINGIVA to prevent ulcers. • Mesial, distal, and lingual gingival embrasure of the pontic should be open for easy cleaning access. • Pontics placed in the non-appearance zone are there to restore function and prevent drifting. • Success or failure of a bridge depends mostly on the pontic design which is dictated by function, esthetics, ease of cleaning, patient comfort, and the maintenance by the patient of healthy tissues on the

edentulous ridge. 1. MODIFIED RIDGE LAP PONTIC: • Pontic of choice in the “APPERANCE ZONE” for maxillary & mandibular FPDs. Highly esthetic. • Uses a ridge lap for minimal ridge contact, but gives the illusion of being a tooth. • ALL CONVEX SURFACES FOR EASY CLEANING & TO PREVENT FOOD IMPACTION. • Ridge contact cannot extend farther lingually than the midline (CREST) of the edentulous ridge. • Contact with the tissue should not fall just along the gingivofacial line angle (there should be no space between it and the crest, a debris trap results. • Pontic contact with the ridge should be compact, facial to the ridge crest, slightly wider M-D at the facial, and narrower at the lingual aspect. 2. SADDLE PONTIC (RIDGE LAP): • Forms a large CONCAVE CONTACT with the ridge. Overlaps the facial and lingual aspects of the ridge. • UNCLEAN & UNCLEANABLE & CAUSES TISSUE INFLAMMATION SO DO NOT USE!

3. HYGIENIC PONTIC (SANITARY PONTIC): “FISH BELLY” • DOES NOT CONTACT THE EDENTULOUS RIDGE. • PONTIC OF CHOICE IN A NONAPPERANCE ZONE (replaces mandibular 1st molars). Poor esthetics. • Occlusogingival thickness must be at LEAST 3mm with adequate space under it for cleaning. • Restores occlusal function and stabilizes adjacent and opposing teeth (prevents drifting). • CONVEX in all areas (F-L & M-D) for easy cleaning. • Floss passes over smooth round surfaces more easily than it does flat surfaces with sharp angles. • “Arc-Fixed Partial Denture” (Perel Pontic)an esthetic modification of the hygienic pontic that veneers visible parts of the pontic with porcelain (occlusal surface and occlusal half of the facial surface which is all of this pontic’s facial surface). • Pontic has a concave archway M-D and convex underside F-L (HYPERBOLIC PARABOLOID). • Added bulk in the connectors decreases stress to the connectors with diminished

deflection in the pontic’s center with less gold used. Increased access for cleaning. 4. CONICAL PONTIC: • PONTIC OF CHOICE FOR A THIN MANDIBULAR RIDGE in a non-appearance zone. • ROUNDED & CLEANABLE but the tip is small relative to its overall size. When used on a broad, flat ridge it creates large triangular embrasure spaces that collect food. 5. OVATE PONTIC: • PONTIC OF CHOICE with a BROAD, FLAT RIDGE to give the appearance it is GROWING FROM THE RIDGE. Superior Esthetics, negligible food entrapment (easy to keep clean). Used where esthetics is a primary concern. Recommended Location: maxillary incisor, canines, and premolars. • It is bluntly-rounded (round-ended design that fits into a ridge depression) where it contacts the tissue and is set into a ridge concavity. Can extend ¼ into the socket after an extraction. EXTRACTION SITE PONTIC. Requires surgical preparation.

CANTILEVER BRIDGES CANTILEVER BRIDGE has an abutment at ONLY ONE END with a pontic attached at the other end. INDICATIONS: • MUST HAVE A VERY STRONG ABUTMENT & MINIMAL OR NO OCCLUSAL CONTACT ON THE PONTIC. • Pontic can replace a missing MAXILLARY LATERAL, but the CANINE MUST be the abutment. • Pontic can replace a FIRST PREMOLAR if full crowns are used on the 2nd premolar and 1st molar abutments. • Pontic can replace 1st MOLAR to avoid UNILATERAL RPD OR PREVENT SUPRA-ERUPTION. 3 SITUATIONS TO USE CANTILEVER BRIDGE: 1. MISSING MAXILLARY LATERAL: CENTRAL INCISOR IS NEVER AN ABUTMENT IN A CANTILEVER BRIDGE UNLESS a rest seat is prepared on the DISTAL of the central (inlay or metallic rest) AND MESIAL OF THE PONTIC TO

PREVENT ROTATION. 2. MISSING MANDIBULAR 1st PREMOLAR: • 2nd premolar and 1st molar are the abutments and must have full coverage crowns on them. • Occlusion must be in the MOST DISTAL FOSSA OF THE PONTIC (1st premolar) b/c shorter lever. • Ideal when the 1st molar needs full coverage and the canine is virgin and full coverage is not desirable (cosmetic concern). 3. Pontic can replace MISSING 1st MOLAR to avoid UNILATERAL RPD OR PREVENT SUPRAERUPTION. • BOTH PREMOLARS MUST BE ABUTMENTS & THE PONTIC MUST RESEMBLE A PREMOLAR (not a molar) to decrease the length of the lever arm and minimize stress on the premolar abutments. In a posterior FPD, a pontic should be: in contact in centric occlusion, may or may not be in contact in workingside movements, and should NOT be in contact in nonworking side movements. • Be non-porous, smooth, with a polished surface. • Make passive pinpoint contact with the gingival

tissue. • Not be concave in two directions. • Be readily cleanable by the patient. • Be narrower at the expense of the lingual aspect of the ridge. • Be on as straight a line as possible between the retainers to prevent any torquing of retainers or abutments.

PIER ABUTMENTS A traditional rigid SOLDERED 5-unit bridge is NOT desirable because of physiological tooth movement, arch position of abutments, and the retentive capacity of the retainers. So, you need a STRESS-BREAKER, NON-RIGID CONNECTOR to prevent the “PIER ABUTMENT” from acting as a FULCRUM or LEVER. PIER ABUTMENT-freestanding abutment with edentulous spaces on each side that requires a NON-RIGID CONNECTOR. A pier abutment does not require a rigid connector (i.e. solder joint) which is the PREFERRED way to connect the abutments and pontic of a bridge. NON-RIGID CONNECTOR-a broken stress mechanical union of a retainer (abutment) & pontic. • USED ONLY FOR A SHORT-SPAN BRIDGE REPLACING ONE TOOTH. • NON-RIGID CONNECTORS DECREASE OR NEUTRALIZE DISPLACING FORCES ON ABUTMENTS BY ELIMINATING A FULCRUM EFFECT ON THE PIER ABUTMENT. • DO NOT USE IF ABUTMENTS HAVE SIGNIFICANT

MOBILITY (PERIODONTALLY COMPROMISED). • TRANSFERS SHEAR STRESS TO SUPPORTING BONE, NOT TO THE CONNECTORS. • MINIMIZES M-D TORQUING OF ABUTMENTS, BUT ALLOWS THEM TO MOVE INDEPENDENTLY. • Stress-breaking device in a five-unit FPD is PLACED ON THE MIDDLE ABUTMENT (PIER) to eliminate a lever and fulcrum by ISOLATING THE FORCE ONLY TO THE FPD SEGMENT WHERE IT IS PLACED. • DOVETAIL KEYWAY of the connector is placed on the DISTAL SIDE of the pier abutment (middle abutment tooth) because 98% of posterior teeth TILT MESIALLY when subjected to vertical occlusal forces. Thus, mesial tooth movement seats the key into the keyway more solidly. If the keyway were placed on the mesial side, the key would unseat during mesial movements. • KEY is placed on the MESIAL OF THE DISTAL PONTIC. The most common key design is a TSHAPED KEY. The path of insertion of the key into the keyway is parallel to the pathway of the retainer that is not involved with the keyway.

MARYLAND BRIDGE A conservative restoration (etched-material prosthesis) with solid metal retainers that relies on the etched inner surface in the enamel of the retainers for its RETENTION. The grooves give increase RESISTANCE FORM. • Requires an abutment MESIAL & DISTAL to the edentulous space. • Requires a shallow preparation in enamel (useful in children with large pulps who are at risk for exposure). • Both abutments inclination M-D difference cannot be > 15° and both abutments must have the same F-L inclination. • Preparations demand additional RESISTANCE via long, well-defined grooves. • Can be moderate resorption with no gross soft tissue defects. • Abutment teeth are basically left intact • Grooves for a resin-bonded FPD (Maryland Bridge) provide mainly RESISTANCE FORM by preventing B-L rotation. The grooves can also provide RETENTION on crowns. INDICATIONS: • RESTORATION OF CHOICE to replace 1-2 missing

MANDIBULAR INCISORS when abutments are unblemished (caries-free). • Replace MAXILLARY INCISORS if patient has an open-bite, end-to-end, or moderate overbite. • Used as a PERIODONTAL SPLINT (but abutment mobility can cause failure). • Can replace molars if child’s masticatory muscles are not well developed. • Replace SINGLE POSTERIOR TOOTH (10% higher risk of failure if more than 1 pontic). • Not used for FPDs > 3 units unless a mitigating treatment plan consideration exists (i.e. opposing RPD which results in less occlusal stress).

Preparation Features: • Should encompass at least 180° (guide surfaces/planes interproximal and extend onto the facial to achieve a facial-lingual lock). Want to extend as far as possible to provide maximum surface area for bonding.

• Vertical stops are placed on all preparations for RESISTANCE & RIGIDITY. • Grooves increase RESISTANCE TO DISPLACEMENT ON ANTERIOR PREPARATIONS. • Occlusal clearance is needed on very few teeth prepared for abutments (.5mm is needed for maxillary incisors). • Light chamfer (1mm) finish line is placed SUPRAGINGIVAL throughout the length to minimize deleterious effects to the periodontium. CONTRAINDICATIONS: patients with DEEP VERTICAL OVERBITE (VERTICAL OVERLAP), EXTENSIVE CARIES, & NICKEL SENSITIVITY, MOBILITY. Advantages of Maryland Bridges: reduced cost, no anesthesia required, supragingival margins (mandatory), minimal tooth preparation, and rebonding is possible if the wings are not bent or sprung. Disadvantages of Maryland Bridges: • IRREVERSIBLE and uncertain longevity. • No space correction (if M-D width is very wide, only so much porcelain can be added to fill the embrasure space) • No alignment correction (cannot correct alignment of teeth due to not restoring facial, proximal, & incisal areas). • Difficult to temporize (cannot make a provision FPD).

• May have a tendency to dislodge over time.

BRIDGES & BRIDGE ABUTMENTS IDEAL ABUTMENT is VITAL TEETH with NO MOBILITY. Abutment is evaluated for 3 factors: crown-to-root ratio, root configuration, and periodontal surface area. • OPTIMUM CROWN-ROOT RATIO FOR A TOOTH TO BE USED AS A FPD ABUTMENT IS 2:3. • 1:1 is the MINIMUM acceptable abutment under normal circumstances. • Crown-to-root ratio: 1:2 is the IDEAL crown-to-root ratio of an abutment tooth for a bridge ABUTMENT. This high a ratio is rarely achieved, thus a ratio of 2:3 is more realistic. A 1:1 ratio is the minimum acceptable ratio for a prospective abutment under normal circumstances. Crown-to-root ratio alone is NOT adequate criteria for evaluating a prospective abutment tooth. • Secondary Retention-double abutments (secondary abutment) to overcome unfavorable crown:root ratios and long spans. The secondary abutment MUST have at least as much root surface area and as favorable a crown:root ratio as the primary abutment (abutment next to the edentulous space). A canine is a good secondary abutment vs. a first premolar,

while a lateral is NOT a good choice as a secondary abutment to a canine. • ROOT CONFIGURATION WITH THE WIDEST F-L DIMENSION IS THE BEST ABUTMENT. • 1ST MOLAR IS THE BEST ABUTMENT & CANINE IS THE 2ND BEST ABUTMENT BECAUSE THEY HAVE THE LARGEST ROOT SURFACE AREA. • SINGLE-ROOT TOOTH WITH AN IRREGULAR CONFIGURATION OR CURVATURE IN ITS APICAL THIRD IS PREFFERED TO A ROOT WITH A PERFECT TAPER. • ROOTS THAT ARE BROADER F-L THAN M-D ARE PREFERRED TO ROOTS THAT ARE ROUND. • DIVERGENT ROOTS ARE BETTER ABUTMENTS THAN FUSED/CONCIAL ROOTS. ANTE’S LAW-the combined abutment teeth root surface area must be equal or greater than the edentulous space (pontic space). Any FPD replacing more than two teeth is high risk. TILTED MOLAR ABUTMENTS: A 3-unit bridge will not seat if the distal abutment intrudes (tilts mesially) on the line of draw. 98% of posterior teeth TILT MESIALLY when

subjected to occlusal forces. The long axis of FPD abutments must converge no more than 25-30°. Any greater mesial tilt requires either: 1. orthodontics (uprighting) is the TREATMENT OF CHOICE to better position a mesially tilted FPD abutment, distribute forces, and helps eliminate bony defects along the root’s mesial surface. Takes around 3 months. 2. PROXIMAL ½ crown: used if orthodontics is impossible, (a ¾ rotated 90° so the distal surface is uncovered). Only used if the distal is caries-free. Contraindicated if there is a severe marginal ridge height discrepancy between the distal of the 2nd molar and mesial of 3rd molar due to the tipping. 3. Telescoping crown and coping. A full crown preparation follows the tipped molars long axis and an inner coping fits the prep, and a proximal ½ crown fits over the coping. Allows full coverage while compensating for the discrepancy between the paths of insertion of the abutments. The coping provides the marginal adaptation. Indicated if extensive facial/lingual restorations on the tilted molar exists. 4. Non-Rigid Connector. Full crown preparation with a box placed in the distal of the premolar (keyway) Most useful when the molar has marked lingual and mesial inclination. Used if a post-core or DO amalgam exists on

the premolar abutment. RIGID CONNECTOR-(solder joints) the PREFERRED way to connect the abutments and pontic of a bridge (FPD). A rigid connector distributes occlusal load more evenly than a pier abutment (non-rigid connector), thus is PREFFERED FOR TEETH WITH DECREASED PERIODONTAL ATTACHMENT (PERIODONTALLY INVOLVED CASES). Factors that Determine a Fixed Bridgework Design: 1. Root configuration: important when assessing an abutment’s periodontal suitability. Roots broader F-L than M-D are preferred to roots round in cross-section. • Multi-rooted posterior teeth with wide separated roots provide better periodontal support than roots that converge, fuse, or are conical. • Single-rooted teeth with an irregular configuration or some curvature in the root’s apical 1/3 are better abutments than teeth with a nearly perfect taper. • Root surface area on a prospective abutment should also be evaluated. 2. Crown-to-root ratio: 1:2 is the ideal crown-to-root ratio of a tooth to be used as a bridge ABUTMENT. This high a ratio is rarely achieved, but a ratio of 2:3 is more realistic. A 1:1 ratio is the minimum acceptable ratio for a prospective abutment. Crown-to-root ratio alone is NOT adequate to evaluate a prospective abutment

tooth. 3. Axial alignment of teeth: parallelism of abutment prep is BEST determined by the LONG AXIS of the preparations. 4. Length of Lever Arm (Span). REPLACING 3 TEETH IS THE MAXIMUM! The absolute MAXIMUM number of posterior teeth that can be safely replaced with a fixed bridge is THREE, and only under ideal conditions. Any bridge replacing more than two teeth is high risk. • An edentulous space involving 4 adjacent teeth other than four incisors is usually best treated with a RPD. If more than one edentulous space exists in the same arch, even though each of them could be individually restored with a bridge, it may be desirable to restore them with a RPD, especially if the spaces are bilateral and each space involves two or more missing teeth. • 3rd molars can rarely be used as abutments since they often display incomplete eruption, short-fused roots, and a marked mesial inclination in the absence of a 2nd molar. To use a 3rd molar, it must be completely erupted, periodontally sound, longseparated roots (multi-rooted), and must display little or no mesial inclination.

• A simple bridge replaces 1-2 teeth, while a complex bridge replaces 2 or more teeth. • Edentulous areas involving 4 or more missing teeth (except 4 incisors), should be restored with an RPD. Fixed Bridge Contraindications: • Poor oral hygiene, high caries rate, or multiple spaces in the arch or teeth likely to be lost in the near future. • Space not detrimental to the maintenance of arch stability or dental health. • Unacceptable occlusion or bruxism. • Anterior fixed bridge is contraindicated when considerable residual ridge resorption exists. Use an RPD. ELECTROSURGERY-an acceptable method of gingival tissue retraction that passes small currents of electricity through the gingival tissues, causing cells to desiccate (scorch). Usually results in some delayed healing because of the lack of proper clot formation, but is very good at stopping hemorrhage. Too low of an electrical current in an electrosurgical electrode is detected by tissue drag. Electrosurgery Objectives: coagulation, hemostasis, access to cavosurface margins, and reduce the inner wall of the gingival sulcus (removing a thin layer of crevicular gingival tissue).

Indications: • Remove hyperplastic gingival tissue where it has proliferated into preparations or over crown margins. • In place of gingival retraction cord where substantial attached gingiva is present. • Crown-lengthening procedures prior to fabricating a provisional crown. Contraindications: areas of thin attached gingiva, or underlying dehiscence because gingival recession occur in these areas after electrosurgery. CARDIAC PACEMAKERS, METAL INSTRUMENTS, FLAMMABLE AGENTS (N2O), KEEP THE ELECTRODE MOVING, and do not touch a metal rest, tooth, or bone. Great care is used during electrosurgery due to potential serious damage to the PDL and surrounding bone, resulting in loss of attachment. ELECTROSURGERY WAVEFORMS: 1. Unrectified, Dampened: recurring peaks of power that diminishes rapidly. Causes dehydration and necrosis, slow, painful healing, but good hemostasis. 2. Partially Rectified, Dampened: damping occurs in the 2nd half of the cycle. Good lateral heat penetration, greater tissue destruction with slow healing in deeper

tissues. Good coagulation. 3. Fully Rectified: continuous flow of energy. Good cutting and some hemostasis. 4. Fully Rectified Filtered: continuous flow of energy. Excellent cutting with less tissue injury and greater healing.

POSTS (DOWELS) & CORES Posts & Cores: Primary Function of Post is to PROVIDE A PLATFORM FOR THE CROWN (retain the core). • If 50% of the clinical crown is destroyed, an amalgam or composite core-build up is indicated. • Composite cores have greater microleakage than amalgam cores, and they are not as dimensionally stable. • A core must be anchored to the tooth (with pins or another retentive feature) and not just placed to fill the void. • Not all endodontically treated anterior teeth require posts and a full crown. Placing a post in a conservatively treated tooth weakens it. • Posterior teeth must have a CAST RESTROATION with occlusal coverage (at least an onlay). • Endodontically treated teeth should NOT serve as abutments for distal extension RPDs (4x greater failure rate than non-abutment RCT teeth). • Pulpless FPD abutments fail 2x more than vital abutments. They should not be abutments with a span longer than 1 pontic. • For a custom cast-post-core, place a KEYWAY or GROOVE to prevent rotation (anti-rotational device). Anti-rotational features (pot holes, slots,

channels), but NOT PINS. • Post’s diameter must not be > 1/3 the root’s diameter at the CEJ. Must be a minimum thickness of 1mm tooth structure at the mid-root and beyond. • All cusps thinner than ½ must be shortened or removed. • Posts do not strengthen roots – it’s a myth! They simply provide retention for a core. A poorly designed or fitted post will cause premature failure of the root. Posts can be metal or fiber (esthetic and bondable). Cast post is becoming passé. • Carbon & Glass Fiber Posts: flexible, absorbs and dissipates forces acting against the tooth. • Ceramic Posts: more rigid, thus more flexure. Resistance to remaining radicular tooth structure. POST WIDTH: should involve the minimum amount of dentin removal, but provide some passive engagement (non-contact) of the outer periphery of the post and inner canal space. POST LENGTH: no clear formula exists for determining the ideal depth of the post preparation. Post LENGTH must be 4-5mm from the apex. Post must be AT LEAST as long as the clinical crown. The post should equal the crown length or 2/3 the root length (whichever is greater). Must leave at least 4mm of gutta-percha at the apical end of the canal because within the apical 4mm is where

the apical delta anastomose with the exterior root surface. Must keep this area blocked with gutta percha and cement to prevent microleakage and root fracture. POST DESIGN: active vs. passive. Just because a post has threading, does not mean it is active. The diameter of the preparation drill and post diameter determine if the post is active. • Tapered: usually requires the LEAST internal dentinal structure and corresponds to the shape of the remaining root. • Parallel: can be just as conservative if selected within the minimum canal diameter. • 2nd Stage: have both parallel and tapered sections. POST COLOR: • Ideal color is that of DENTIN (ceramic restorations). • Translucent, may need to be masked with dentincolored core material. • Carbon-fiber or titanium posts are best for PFM, PFZ, or zirconium crowns with a completely opaque substructure. FERRULE EFFECT-the preparation margin (finish line) MUST extend at beyond (apical) to the core and into SOUND TOOTH STRUCTURE. Ferrule is the 1.5- 2mm or so of sound root structure apical to the core that the margins of the crown should engage to PROTECT AGAINST ROOT FRACTURE. A ferrule makes post-

retained full-coverage restorations significantly more retentive and dramatically strengthens the tooth to resist fracture. It surrounds the circumference of the tooth, holding it together like the metal bands around the head of a wooden mallet. • Preparation for a post-core should preserve solid tooth structure. The margin should be APICAL to the dowel-core margin to enable the crown to girdle the tooth and brace it externally. • If the tooth is flush with the gingiva, fabricating a postcore and crown without encircling the tooth structure by the crown walls can cause ROOT FRACTURE.

Advantages of using a post & core, rather than a post crown when restoring RCT treated teeth: ► Marginal adaptation and fit of the restoration is independent on the fit of the post. ► Restoration can be replaced in the future if needed,

without disturbing the post and core. ► If the endodontically treated tooth is to serve as a bridge abutment, it is not necessary to make the root canal preparation parallel with the line of draw of other preparations (it can be treated as an independent abutment). • Post and core is made separate from the final restoration. The crown is fabricated and cemented over the core just as a restoration is placed over a preparation done on tooth structure. • Post & core can be used for teeth with little or no clinical crown, but with roots with adequate length, bulk, and straightness. For posterior teeth with less extensive destruction of coronal tooth structure, or teeth with less favorable root configurations, a pin-retained amalgam or composite core can be used.

PORCELAIN VENEERS Porcelain Veneer Indications: 1. Covering labial surface defects like enamel hypoplasia. 2. Masking discolored teeth like tetracycline staining, discoloration after loss of tooth vitality. 3. Repair structural damage like fractured incisal edges. 4. Improve tooth contour (i.e. peg-shaped lateral incisors). 5. Reducing spaces in cases when orthodontics is inappropriate. Porcelain Veneer Contraindications: severe imbrication of teeth, traumatic occlusal contacts, unfavorable morphology, insufficient tooth structure and enamel. A patient with a high caries index, short clinical crown, and minimal horizontal overlap are not candidates for partial veneer crowns. Rather, the restoration of choice is a full PFM crown. Advantages of Partial Veneer Restorations (3/4 & 7/8 crowns): 1. Primary reason for choosing a ¾ crown over a full cast crown is TOOTH STRUCTURE IS SPARED. 2. A great deal of the margin is in an area accessible to the dentist for finishing and to the patient for cleaning. 3. Less of the restoration margin is in close proximity to

the gingival crevice, thus decreasing the chance of periodontal irritation. 4. Can be more easily seated completely during cementation. With at least part of the margin visible, complete seating of a partial veneer crown is more easily verified by direct vision. 5. If it is ever necessary to do an electric pulp test (EPT) on the tooth, a portion of the enamel is un-veneered & accessible.

IMPRESSION MATERIALS HYDROCOLLOIDS: have the advantage of WETTING INTRAORAL SURFACES well, but have very limited dimensional stability because they are composed of 85% water. 1. Reversible Hydrocolloid (Agar-Agar)-an impression material whose physical state is changed from a GEL→SOL by applying HEAT and is reversed back by removing heat. Reversible hydrocolloids are composed of 85% water, 12-15% agar, traces of borax, potassium sulfate, & sodium tetraborate. Agar impression materials and dental compounds do not involve a chemical reaction to set. ► Advantages: Easy to pour, no mixing is required (but a hydrocolloid-conditioning unit is required). No custom tray is required. Moisture tolerant, clean & pleasant with acceptable odor, excellent shelf-life, inexpensive. ► Disadvantages: must be poured immediately, finish line is difficult to read, weak in a deep sulcus, and potentially injurious to the patient if not handled properly. Very limited dimensional stability.

2. Irreversible Hydrocolloids (ALGINATE)-an elastic impression material with very limited dimensional stability. ► Advantages: inexpensive, can use stock tray, easily mixed and poured. ► Disadvantages: unstable, fragile, may affect the cast surface, and must be poured immediately. Sodium Phosphate-component found in alginate powder that controls alginate SETTING TIME. After taking alginate impressions, if you place the impressions in a bowl of water for a few hours to try and prevent them from drying up before pouring the casts, IMBIBITION can occur (the impressions absorb water and expands). When imbibition occurs, the impression is no longer accurate. Shrinkage occurs in alginate impressions even when placed under 100% relative humidity = SYNERESIS (occurs when exudate like droplets of the liquid medium forms on the impression surface). Since shrinkage is undesirable (causes distortion of impressions), alginate impressions should not be left in water (causes expansion) or exposed to air (causes shrinkage). Impressions should be poured immediately to ensure accuracy. When immediate pouring is not possible, alginate

impressions can be stored only briefly in a moist paper towel. Techniques to help prevent GAGGING while taking alginate impressions: ♦ Decrease the time to take an impression and have the patient breathe through their nose. ♦ Seat the patient in an upright position. ♦ Seat the posterior part of the tray first. Mixing the alginate rapidly causes it to set more rapidly. ♦ Decreasing water-to-powder ratio causes alginate to set faster (affects mix consistency as the mix is much thicker when less water is used). ♦ DO NOT use cold water to mix the alginate because it retards alginate’s setting time. Mandibular alginate impression is taken FIRST since gagging is more likely to occur while taking the maxillary impression. For the maxillary impression, seat the posterior portion of the tray first, then the anterior portion to help prevent alginate from being squeezed out of the tray back toward the patient’s throat. Always remove alginate impressions in one quick movement with a snap to help decrease permanent deformation. Do not over-seat the tray (0.25 inch) minimum of alginate should remain over all critical structures (especially occlusal surfaces). When taking an alginate impression, it is advised that the tray be placed in the mouth after all critical areas are wiped

with alginate. Critical areas are buccal to the maxillary tuberosities and retromylohyoid space. Rest seats and guide planes should be covered with alginate and any other soft tissue undercuts. When taking alginate impression for a RPD, it is best to apply some alginate directly on the teeth to eliminate bubbles and saliva from the rest seat preparations. ELASTOMERS-impression materials with elastic or rubber-like qualities used for crown & bridge, secondary impressions for dentures, and inlays/onlays. When removing elastomeric impressions, use steady force (a snap is not required) to minimize permanent deformation. Elastomers set via a chemical reaction. Elastomers are NON-AQEOUS polymer-based rubber impression materials with good elasticity. 1. Polysulfides (Rubber Base, Mercaptan, Thiokol)-the base contains a liquid polysulfide polymer (mercaptan polymer) mixed with an inert filler. The accelerator is usually lead dioxide. When these two pastes are mixed, the polymer chains are lengthened and cross-linked through oxidized thiol groups to form a rubber-like material. • A tray for a polysulfide rubber impression that lacks occlusal stops may result in an inaccurate final impression because of permanent distortion during polymerization.

• Sets in 12-14 minutes (the longest setting time). • Moisture tolerance in the mouth is acceptable. • Wettability with gypsum is poor, and it has poor taste and odor. Has an 18-month shelf-life. • Polysulfide polymerization of is exothermic and accelerated by an increase in temperature or humidity. • Polysulfides have good flow properties, high flexibility, and high tear strength. Polysulfides have the strongest resistance to tearing, but impressions can distort when removed from areas where deep undercuts exist. Polysulfides have a long working time and relatively long polymerization time, which may add to patient discomfort. They have a low resistance to deformation. 2. Polyvinyl Siloxanes VPS (Additional Silicones or Vinyl Polysiloxanes)-one tube contains silicone with terminal silane H+ groups and an inert filler. The other tube is a vinyl silicone with terminal vinyl groups, chloroplatinic acid catalyst, and filler. Upon mixing, there is an addition of silane hydrogen groups across vinyl double bonds and does not form by-products, resulting in a very dimensionally stable material. PVS can be poured up to 1 week. • Latex gloves should not be worn when mixing polyvinyl siloxanes because sulfur in the latex retards the setting of addition silicone materials. Sulfur in ferric and aluminum sulfate reaction solution

may also inhibit polymerization of PVS. Some latex gloves might inhibit the setting of polyvinyl siloxane. • Mixing time (30-45 sec), Moderate working time (2-4 min); moderate setting time (6-8 min). There are now FAST SET products which set intra-orally within 2 minutes (Ex: Aquasil). • Excellent dimensional stability and very low permanent deformation. • Poor tear strength, lowest temperature rise, very high stiffness, very poor wettability by gypsum. Addition silicones are temperature sensitive (increases in temperature shorten working & setting times). Easy to mix, easy to clean-up, and acceptable odor and taste. • Polyvinyl siloxanes (PVS) are the MOST WIDELY USED & MOST ACCURATE elastic impression materials. They have less polymerization shrinkage, low distortion, fast recovery from deformation, and moderately high tear strength. Most PVS can be poured up to 1 week after impression making and are stable in most sterilizing solutions. 3. Polyethers (Impregnum/Premier & Polygel (Caulk)are two component materials. The rubber base includes a polyether polymer with ethylene imine groups, silica

filler, and plasticizer. The accelerator contains a crosslinking agent (aromatic sulfonic acid ester) which produces cross-linking by cationic polymerization. When mixed, a rubber forms by a cationic polymerization process. • Advantages: excellent dimensional stability (when dry), clean, pleasant taste & odor, FAST SETTING, dimensionally stable if more than one cast is poured, stable even if poured 24hrs after taking an impression (very low permanent deformation) as it can be poured up to 1 week, and are truly hydrophilic which results in superior wettability by gypsum. Polyether impression material tolerates moisture better than any other elastomer. • Disadvantages: the most difficult material to remove from the mouth (the most rigid/stiff material), tears easily (poor tear strength), may adhere to teeth, high water absorption (dimensionally unstable in the presence of moisture), and fine margins may break. Compared to other materials, the main disadvantage of using polyether elastomeric impression materials is they are much stiffer. Has the highest temperature rise and highest stiffness. • Polyethers have the SHORTEST WORKING & SETTING TIMES of the elastomeric impression materials. Mixing time is 30-45 seconds (mixes easily); Working time is 2-3 minutes; Setting time 6-7

minutes. • All elastomeric impression materials CONTRACT SLIGHTLY during setting (they do not expand). • For best results with elastomeric impression material, the prepared tooth should be free of surface moisture. Compared to hydrocolloids, elastomeric impression materials are easier to prepare, more resistant to tearing upon removal, and have a superior dimensional stability. • Custom Trays are an important part of rubber base impression techniques since elastomers are more accurate in uniform thin layers that are 2-4mm thick. With all elastomers, a custom tray should be fabricated with a plastic material, should be rigid, have occlusal stops to avoid permanent distortion during polymerization, and be coated with an adhesive that should dry completely before taking the impression to prevent the impression material from pulling away. ZINC OXIDE-EUGENOL-an impression paste whose setting time is accelerated by ADDING a drop of water to the mix. To retard the setting of ZOE, add inert oils (olive or mineral oil) during mixing. ZOE sets via a chemical reaction.

o Advantages: can record soft tissue at rest, sets hard in 5 minutes, stable, & less expensive than polysulfides. o Disadvantages: messy to mix, very sticky, tissue irritant, not elastic, difficult to manipulate, not recommended for gagging patients. IMPRESSION PROBLEMS: 1. Grainy Material: caused by improper or prolonged mixing, undue gelation, or too low a water-powder ratio. 2. Tearing of Material: caused by inadequate bulk, moisture contamination, premature removal from the mouth, or prolonged mixing. 3. Irregularly Shaped Voids: due to moisture or debris on tissue. 4. Rough or Chalky Stone Cast: caused by inadequate cleaning of the impression, excess water left in the impression, premature removal of the cast, leaving the cast in the impression too long, or improper manipulation of stone. 5. Distortion: impression not poured immediately, movement of tray during gelation, premature or improper removal from the mouth, or tray was held in the mouth too long (only with certain brands). Bite Registration Material used to make an accurate interocclusal record should offer a MINIMUM RESISTANCE to the patient’s jaw closure and have

LOW FLOW at mixing. Recently, addition-reaction silicone impression materials have dominated the interocclusal record (IOR) market since these materials have VERY LOW FLOW when mixed and become rigid after setting.

GYPSUM Main constituent of dental plasters and stones is Calcium Sulfate Hemihydrate. All gypsum is produced by calcination (heating gypsum and driving off part of the water of crystallization). • Type I: rarely used today. Dental plaster (plaster of Paris). Can be used for remounting casts. • Type II: used to make casts when strength is not important (orthodontics). Dental Model Plaster (Type II)-heating gypsum in an open kettle. This process produces porous and irregularly-shaped particles. Dental plaster is the WEAKEST GYPSUM PRODUCT. Beta-hemihydrate. o General lab use (articulating, flasking, repairs). • Type III: used for preparing casts of an alginate impression upon which dentures are processed. Produced by HEATING GYPSUM under pressure with water vapor in an autoclave to produce uniformshaped and less porous particles. Heating gypsum in a 30% solution of calcium chloride produces high strength (improved) die stone. Produces least porous and strongest particles. Alpha-hemihydrate.

o Used for orthodontic casts where whiteness, accuracy, and durability are essential, or with full or partial acrylic dentures. • Type IV: used when making stone “dies” (reproductions of teeth with prepared cavities). Use for fixed and removable bridges, cast RPDs, crowns, inlays, onlays. Dental stone die, high strength, low expansion. Modified alpha-hemi-hydrate. • Type V: Dental stone die; high strength and high expansion. Most trusted and popular die material today. DENTAL CEMENTS-cements do NOT increase crown retention. A tooth must be WIPED DRY before crown cementation, as opposed to drying the tooth with alcohol and warm air to decrease the possibility of pulp damage. Also, ALWAYS apply cement to both the restoration and the tooth. 1. Composite Resin-the luting material of choice to cement a ceramic crown and can provide the STRONGEST BOND. Ceramic crowns are bonded with composite resin after etching the internal surface of the crown, and are shown to be better in bonding strength than other materials. 2. Zinc-Phosphate Cement-may also be used to cement

porcelain crowns. It has good compressive strength (14,000-16,000 psi), but its high pH is a problem because two layers of varnish must be applied to protect the pulp. ZPC is one of the oldest and widely used cements for luting permanent metal restorations and as a used as a base. It is a high-strength cement base, mixed from zinc oxide powder and phosphoric acid liquid. Due to its low initial pH, it may cause pulpal irritation, especially where only a thin layer of dentin exists between the cement and the pulp. 3. Zinc Polycarboxylate or ZOE- biologically compatible cements used on teeth with preparations with adequate length and retentive features, or when the preparation depth raises some concern regarding pulp vitality. ZOE cements exhibit better resistance to solubility than zinc phosphate cement. Zinc polycarboxylate and GIC adhere to calcified dental tissue, and have superior biologic compatibility than zinc phosphate cements. 4. Glass Ionomer Cement (GIC)-a dental restorative material used in dentistry for restoring teeth and luting cements. These tooth-colored materials are based on the reaction of silicate glass powder and polyalkenoic acid. Introduced in 1972 for use as restorative materials for anterior teeth (particularly for eroded areas, Class III and V carious lesions). As they bond chemically to dental hard tissues and release

fluoride for a relatively long period modern day applications of GICs have expanded. The desirable properties of glass ionomer cements make them useful materials in the restoration of carious lesions in lowstress areas such as smooth-surface and small anterior proximal cavities in primary teeth. o GC Fuji PLUS is a resin reinforced glass ionomer luting cement designed for final cementation of metal, porcelain-fused-to-metal and metal free crowns, bridges, inlays and onlays. It bonds chemically and mechanically to tooth structure and to all types of core material. Its simple placement technique produces significantly higher bond strengths than conventional glass ionomer cements while maintaining the favorable characteristics of glass ionomers-fluoride release, low coefficient of thermal expansion, and biocompatibility to tooth structure and soft tissues. For indirect metal-free restoration that recommend a resin-reinforced / resin-modified glass ionomer for final cementation – cementable reinforced all ceramic crowns like PROCERA or cementable composite resin restoration like GRADIA. o Resin-Modified GIC: used in composites and luting cements. Based on the reaction of

silicate glass powder and polyalkenoic acid. Great for ZIRCONIUM crowns (i.e. BRUXZIRS). RMGICs are conventional GIC with the addition of HEMA and photoinitiators. Ex: Rely-X Luting Plus Cement by 3M. Benefits: proven bond strength, sustained fluoride release, low postop sensitivity, moisture tolerant, ideal for PFM, zirconia, metal and pediatric crowns. Prolonged sensitivity to heat, cold, and pressure after cementing a crown or fixed bridge is usually related to OCCLUSAL TRAUMA. If CR occlusion is high, patient complain of cold sensitivity and pain on biting down hard. All patients should have an appointment specifically to check the occlusion on all crowns and bridges. Excursive movements should also be evaluated, since often patients complain of pain on chewing soft foods (this indicates improper balancing or working contacts). The occlusion of gold restorations is best checked with SILVER PLASTIC SHIM STOCK.

DENTAL IMPLANTS IMPLANT-a prosthetic device made of alloplastic materials implanted into the oral tissue beneath the mucosal and/or periosteal layer and/or within the bone, to provide retention and support for a fixed or removable prosthesis. DENTAL IMPLANTS are classified based on their anchorage component as it relates to bone: 1. Eposteal- receives its primary support by resting on the bone. 2. Endosteal (used mainly today), placed into alveolar and/or basal bone of the mandible or maxilla, and transects only one cortical plate. Ex: Blade, ramus frame, or root form implant. 3. Subperiosteal-placed directly beneath periosteum overlying the bony cortex. 4. Transosteal (Staple Bone Implant)- penetrates BOTH cortical plates and the full thickness of alveolar bone. Combines the subperiosteal and endosteal components. 5. Intra-mucosal Implants-inserted into the oral mucosa which is used as the attachment site for the metal

inserts. Dental Implant Systems: subperiosteal, transosteal, & ENDOSSEOUS (most common). Placing endosseous implants is a predictable procedure. IMPLANT MATERIALS: 1. Metallic: most popular material today (TITANIUM). Other metallic implants are stainless steel, cobalt chromium molybdenum alloy, and vitallium. 2. Ceramic & Ceramic Coated: can coat metallic implant with a plasma spray or coated to produce a bioactive surface. Non-reactive ceramic materials are also present. 3. Polymeric: only used as adjuncts stress distribution along with implant, not used as implants themselves. 4. Carbon: made of carbon + stainless steel with a modulus of elasticity equal to bone and dentin. Brittleness can lead to fracture. Criteria for Implant Success: • No persistent signs/symptoms of pain, infection, neuropathies, paresthesia. • Implant immobility and no continuous peri-implant radiolucency.

• Negligible progressive bone loss (< 0.2mm annually) after physiologic remodeling during the first year of function. • Patient/dentist satisfaction with the implant restoration. Biointegration-direct biochemical bond of bone to the titanium implant surface at the electron microscope level. Independent of any mechanical interlocking. Osseointegration-direct attachment or connection of osseous tissue to an inert, alloplastic material without intervening with connective tissue. Direct contact between bone and implant surface.

Implants have a peri-implant soft tissue seal where

junctional epithelium attaches to the implant surface by HEMIDESMOSOMES. Main reasons implants fail to integrate: premature loading, apical migration of junctional epithelium, overheating during placement, loose fitting implants, and patients with medical risk factors or contraindications. Implants have > 90% success rate for both maxillary & mandibular implants. Implants with rough surfaces offer advantages than smooth surface implants, and implants placed in the mandible have higher success rates than in the maxilla. 3-D computerized tomography (CT) scans provide the most accurate information about regional anatomy (maxillary sinuses, foramina, mandibular canal, adjacent teeth/roots). LOWER success rates are associated with CANCELLOUS BONE (20-25% bone density) than with cortical bone (80-90% volume bone density). Thus, CORTICAL BONE provides greater implant-bone contact and fixation. Surgical considerations require evaluating the anatomy and location of vital structures, bone quality, quantity, and contour, and soft tissues. Diagnostic aids used in presurgical considerations to determine the number, location,

type, and angulation of the implants and abutments (mounted or unmounted diagnostic casts, CT imaging, surgical template). Placing implants involves analyzing: number & location of missing teeth, inter-arch distance, number, type, & location of implants to be placed, existing/proposed occlusal scheme, design of the planned restoration. ASSESSING PATIENT HEALTH STATUS FOR IMPLANTS: • Patient’s health status is the FIRST & FOREMOST priority when assessing a candidate for implants. Additionally, knowing past and current medications that can affect healing of bone and implant integration after surgery is critical. • Pre-Treatment Considerations: patient’s oral health and medical/psychological status, patient motivation and home care ability, patient expectations, and assessing habits & conditions that increase the risk of implant failure (alcoholism, smoking, high ASA score, bruxism, periodontal disease, and radiation therapy). Absolute Implant Contraindications: recent MI or cerebrovascular accident, valvular prosthesis surgery, immunosuppression, hemostatic issues that impair normal healing cascade, active malignancy treatment, drug abuse,

psychiatric illness and IV bisphosphonates. Relative Implant Contraindications: aging, adolescence, osteoporosis, smoking, uncontrolled diabetes, alcoholism, + interlukin-1 genotype, HIV positive, cardiovascular disease, hypothyroidism, post-irradiated mandible or maxilla, and poor oral hygiene. • Patients with a strong susceptibility to periodontitis CAN BE SUCCESSFULLY TREATED WITH IMPLANTS with proper maintenance and home care. • Age is not an important factor that affects implant survival, but implants placed after age 15yrs in girls and 18yrs in boys have a better prognosis than when placed in younger children. Implant Complications: prosthesis instability, fixture mobility, occlusal trauma, fractured/loose components, inflammation/infection, excessive progressive loss of hard/soft tissues, pain, neuropathy/paresthesia. Additional negative outcomes: implant mobility or loss, persistent pain/loss of function, progressive bone loss, persistent peri-implant radiolucency and uncontrolled inflammation/infection, implant fracture, increased probing depths. 3 Required Measurements to Determine Adequate Bone

Volume with a Radiograph: 1. Horizontal distance between adjacent roots. Minimum distance between roots is 1.5-2.0mm for adequate interimplant-radicular bone space for proper blood supply to nourish the PDL of adjacent roots and bone surrounding the implant-bone interface. 2. Vertical bone height, measured from most coronal aspect of the surgical site to the desired apical height. Probing the periodontal spaces of adjacent teeth helps determine site health. Periodontal therapy for adjacent teeth should be performed prior to implant surgery and then re-evaluate the site after healing. 3. Width (thickness) of available bone Implant Home Care & Maintenance: effective aids include interdental brushes with nylon-coated wire, soft toothbrushes (manual and power), end-tuft brushes, gauze, floss (plastic, nylon braided, coated floss with stiffined end to clean under bridges, dental tape). • Patients should present for hygiene every 3, 4, or 6 months depending on need as determined by the dentist. • Scaling implant surfaces with non-metal, unfilled resin instruments with optimum rigidity. This

produces the least surface alteration/rougness. Selective polishing with non-abrasive pastes also helps prevent plaque formation and retention, and reduces plaque-induced inflammation. • Removing excess cement after crown delivery and checking for loose implant crowns at patient recare, helps prevent peri-implantitis. If gingival surgery is performed when seating a final restoration (crown), probing is delayed until the tissue completely heals which takes about 6 weeks. A staged approach can be used to place ENDOSSEOUS IMPLANTS. Implants can also be placed at the time of extraction. Mechanical failures of the implant components and prosthetic superstructures have been associated with OCCLUSAL OVERLOAD. The desired outcome of implant therapy is maintenance of a stable, functional, esthetic tooth replacement. Pre-Surgical Site Preparation is critical for implant success. If the tooth was extracted in anticipation of an implant, bone preservation is the most important factor. The GOLD STANDARD is to graft the socket at the time of extraction and place a guided tissue membrane to facilitate growth of any missing bone walls or defects. Always try to preserve the buccal plate during extraction. • On the day of implant surgery, scientific evidence shows that antibiotic prophylaxis helps reduce implant

failure when placed under ordinary circumstances. 2g of Amoxicillin orally 1 hour preoperatively greatly reduces implant failure. • Intra-oral bacterial reduction at the surgical site is advised and patients should use a bacteriostatic rinse postoperatively. Patient should be seen 7-10 days post-operatively for evaluation and examination. Implant Placement & Location: • A surgical stent helps create the initial bone depression and pilot drill orientation. • Place implant entirely in bone, ideally at least 10mm. • Must be at least 3mm both mesially and distally to the implant if next to another implant, or at least 1.0 mm if the implant is placed next to a natural tooth. • Ideally want total space of >7mm mesial-distally. • Buccal and Lingual of implant should have at least 1.0 mm of bone. • Caution with existing anatomic structures (i.e. inferior alveolar nerve, mental foramen, and maxillary sinus). • Ideally, place the implant at least 2.0 mm away from vital structures. • When placing implants in esthetic regions, if the distance between bone and papilla is greater than 6 mm, papilla will be lost. SURGICAL PHASE: accessing alveolus using a flap or flapless approach depending on the aesthetic result and

surgical site location. • A flapless approach is predictable and aided using a tissue punch to gain bone access in the smile zone. If a flap is needed in an aesthetic area (smile zone), a papillae sparing incision is made on the palatal aspect of the ridge extending several mm into the buccal mucogingiva with a 15C scalpel blade, tissue forceps, and periosteal elevator. A papillae sparing incision maintains adjacent periodontium functional and aesthetic integrity. • Full thickness flap may be raised in a non-aesthetic zone (posterior maxilla/mandible). An implant placed in the posterior maxilla that has been grafted, may increase the risk of implant failure and migration to the maxillary sinus. To decrease this risk, carefully study radiographs to analyze the quality of bone, implant stability, and proper surgical technique. Implants have mechanical retention where the substrate engages with undercuts, threads, and slots contact surrounding bone, and bioactive retention, where hydroxyapatite bonds to bone. Interface Phase Development: • Stabilization- calluses form and adhere to implant surface (6 weeks). • Strength- osteoclasts resorb vital bone and replace w/ lamellar bone.

• Durability: remodeling of the site. Restoring Implants: • For crown impressions, a closed impression tray is best for single implant, while an open-tray impression works best for multiple implants. • Abutments can be screw or cement retained. For cement-retained abutments, at least 5mm of abutment height is advised. • From abutment to opposing dentition should be at least 1.0mm of space for restoration. Screw-Retained Abutment main PROBLEM IS it CAN LOOSEN. Tighten to 35Ncm and cement with a provision cement, so if it comes loose, you can remove the crown and retighten the screw.

IMPLANT OVERDENTURES An overdenture supported by two implants (implantretained overdenture) is the PRIMARY prosthetic approach for an edentulous mandible. Attachments provide overdenture retention and stability via 4 attachments, and selecting the optimal mandibular overdenture attachment depends on required retention, jaw morphology/anatomy, oral function, and patient recall compliance. 1. O-ring 2. Bar-clip attachments 3. Ball attachment: highly reliable, independent attachment that allows rotational freedom to allow stress release. Ball attachments place less stress on implants and produces less bending movement than bar-clip attachments. More favorable in distributing stress around an implant than a locater attachment. Ball attachments with minimum collar heights are preferred over locators when there is restricted vertical space to address stress concerns. Transfers the least stress to peri-implant bone when a unilateral load is applied. More resilient than a locator, thus causes more uniform and

less maximum stress. 4. Locator attachment: a self-aligning independent attachment with dual retention, comes in different vertical heights, and provides good resiliency, retention, and durability. Transfers more stress than a bar-clip. Very rigid to restrict overdenture movement and can generate more stress than ball attachments on the peri-implant bone. Adequate restorative space such as interocclusal distance, phonetics, and esthetics are critical in successful implant overdenture treatment. Minimum vertical restorative space required for implant-supported overdentures attachments: • Locator attachments = 8.5mm. • Ball attachments = 10-12mm. • O-ring attachment = 10-12mm. • Bar clip attachments = 13-14mm. IDEAL LOCATION FOR A MANDIBULAR TWO IMPLANT OVERDENTURE IS THE LATERAL INCISOR AREA WITH SHORT ATTACHMENTS.

At least 12mm vertical restorative space is needed for mandibular implant-supported overdenture. Crown Height Space (CHS): distance from crest of alveolar bone to the occlusal plane. CHS biomechanics is related to lever arm mechanics. • Non-axial loading creates lateral movement that equally increases with increased CHS, causing increased stress at the bone surrounding the implant neck. • Each 1mm CHS increase causes a 20% increase in cervical load on a fixed-implant prosthesis. Implant splinting is advised to prevent cervical overload. • Increasing vertical restorative space decreases the maximum stress around implants in both unilateral

and bilateral implant loading. • CHS > 15mm can be biomechanically unfavorable causing increased stress on the bone surrounding the implant. • Increasing occlusal plane height, decreases the stress in bone surrounding implants. • Attachment system height to the crest of bone, and distance from the attachment to the occlusal plane, are two heights to consider with any removable prosthesis that have mobility and soft-tissue support. INCREASING vertical restorative space and DECREASING bar height, DECREASES the maximum stress around implants when a unilateral load is applied.

DENTAL CEMENTS (LUTING AGENTS) Glass Ionomer Cements (GIC)-are hybrids of silicate and polycarboxylate cements designed to combine the fluoride releasing properties of silicate particles with the chemically adhesive and more biocompatible characteristics of the polyacrylic acid matrix compared to the extremely acidic matrix of silicate cement. GICs are mixed powder-liquid component systems. The powder (calciumaluminofluorosilicate glass) reacts with a liquid (polyacrylic acid) to form a cement of glass particles surrounded by a matrix of fluoride elements. • Advantages of GIC: ► Releases fluoride (anti-cariogenic). Upon setting, GICs can inhibit recurrent caries development at its margins because it releases fluoride from its surface. GIC can also absorb fluoride when local ionic concentrations are high, then slowly release fluoride when the environment concentration decreases, thus acting as a FLUORIDE SPONGE. ► Chemical adhesion to the prepared tooth and certain materials. Micromechanical bond to composite resins. ► High biocompatibility, thus with enough dentin remaining (.5-1mm) no pulpal protective agent

(calcium hydroxide) is required. ► Good thermal insulators (equal to natural dentin).Thermal expansion is similar to that of tooth structure. ► After the initial setting period, GICs have low solubility in the mouth. It is the least soluble cement (compared to zinc phosphate and zinc polycarboxylate cements). • Disadvantage: has a higher cement film thickness than zinc phosphate cement. • Only GIC is used as a cement (luting agent) AND permanent restorative material. GIC are often used for root surface carious lesions (Class V restorations) due to the potential advantage of fluoride release to help control spread of caries. GIC is used as a luting agent and for Class V restorations with composite “sandwich technique”. • Glass ionomer cements are composed of aluminosilicate powder and polycarboxylate liquid. CLASSES OF GLASS IONOMER CEMENTS (GIC): 1. Conventional GIC-used as a luting agent. Ex: KetacCem. 2. Light-cured GIC-used as a liner or base (the liquid version has HEMA added to it). Advantages: extended

working time, short on-demand setting times, as a set mass it is stronger, more adhesive, and more resistant to desiccation than self-cured glass ionomers. Ex: Vitrebond & XR Ionomer. 3. Resin-Modified (Hybrid) Light Cured GIC-used for any application where glass ionomers are good choices (the liquid has HEMA added to it). Ex: Fuji-II LC. Advantages of hybrid light cured ionomers: esthetics, bond strength, and coefficient of thermal expansion. GLASS IONOMER RESTORATIONS: both self-cured and light cured versions of GI restorations are available. Lightcured glass ionomers allow extended working time and have improved physical properties. Due to their limited strength and wear resistance, GI restorations are indicated to restore LOW STRESS AREAS where caries activity potential is a significant concern. • Are moderately esthetic, but do not polish as well as composites. • Material of choice for restoring root surface caries in patients with high caries activity. • Best surface finish for a glass ionomer restoration is obtained against a surface matrix. • Adheres to mineralized tooth tissue. • Glass ionomers have a lower compressive strength, tensile strength, and hardness than resin composites. They are very technique sensitive due to their high solubility when first mixed. However,

with newer hybrid or light-cured resin-modified glass ionomers, these properties have been improved. GLASS IONOMERS ARE IDEAL LINERS OR BASES due to these properties: adhesive bond to tooth structure, snap set in the light-cured form (ex: Vitrebond), anti-cariogenic due to fluoride ion release, and they BOND TO COMPOSITE. GIC are excellent liners for Class V root caries restorations or “sandwich technique” which achieves all the benefits of the GIC plus high polishable, surface hardness, and strong bond to enamel and composite resin. ZINC OXIDE EUGENOL CEMENTS (ZOE)- soft, sedative type cement used as a SEDATIVE or TEMPORARY filling material, as an insulating base, and in interim caries treatment. Eugenol has a palliative effect on PULP, which is one of the main advantages of using ZOE cement. ► Powder is zinc oxide and the liquid is eugenol. A conventional mixture of zinc oxide + eugenol is relatively weak. Recently, “reinforced” or “improved” ZOE cements have been introduced (“Reinforced ZOE or ZOE-EBA). The particles of the zinc powder are surface treated to produce better bonding of the particles to the matrix. ► ZOE is soluble in oral fluids and is difficult to remove from cavity preparations. ZOE is used as

a base, temporary restoration, and as a cement (luting agent). ► ZOE, “reinforced” ZOE, ZOE-EBA, silicate, & zinc silicophosphate cements are no longer routinely used to permanently cement restorations. When considering ease of cementation, the reinforced ZOE cements are preferred to zinc phosphate because they do not require use of a varnish for pulp protection (zinc phosphate is very acidic). Properties of improved zinc oxide-eugenol materials: they provide excellent marginal seal, have a palliative effect on the dental pulp, and are easily removed from the cavity preparation. They do NOT have thermal insulation qualities that compare to dentin. ► ZOE Contraindications: o Cannot apply to dentin or enamel prior to bonding as it inhibits bonding. o Cannot use as a base or liner under composites as eugenol interferes with composite polymerization. o Eugenol or “oil of cloves” allergy, as this is fairly common. o Direct pulp capping as eugenol is a pulpal irritant when in direct pulpal contact.

o Cannot be used under composites. 4 Types of Zinc Oxide Eugenol (ZOE) Materials: 1. Type I ZOE: temporary cement. 2. Type II ZOE: permanent cement. 3. Type III ZOE (Reinforced ZOE): a temporary filling material (IRM) and thermal insulating base. The powder is composed of zinc oxide and finely divided polymer particles (polymethyl-methacrylate) in the amount of 20-40% by weight. The zinc oxide powder is surface treated by an aliphatic monocarboxylic acid like propionic. Eugenol is the liquid. This combination of surface treatment + polymer reinforcement results in a material that has good strength and toughness which markedly improves abrasion resistance. Good for basing large and complex cavities as it withstands the pressure of amalgam condensation, with minimal effect on the pulp. 4. Type IV ZOE: a cavity liner. Basic composition of all ZOE materials (whether a cementing medium, surgical dressing, temporary filling material, or impression paste) is zinc oxide, eugenol, & resin. Plasticizers, fillers, accelerators, and other additives are incorporated as needed to provide desired properties for the particular use of the product. ZINC PHOSPHATE CEMENT: the oldest luting cement with the longest “track record” serving as the standard

to which newer systems can be compared. It is a powder-liquid system. The powder is mainly zinc oxide, while the liquid is orthophosphoric acid. ► Its primary use is as a cement (luting agent) for cast restorations. It is also used as a base when high compressive strength is needed. The initial mixture of ZPC is very acidic (pH 3.5), thus it can cause irreversible pulpal damage if a cavity varnish (2 coats) is not placed on the tooth prior to cementing the crown. ► If a zinc phosphate cement base is used when restoring a tooth, the varnish is applied PRIOR to placing the base. The use of a base in conjunction with amalgam or gold foil does not alleviate the need for a varnish as an aid in sealing the cavity margins against leakage. However, the type of base governs the order of applying the varnish and base. If a zinc phosphate cement base is used, then the cavity varnish is applied to the cavity walls PRIOR to placing the base. However, if a biocompatible agent (i.e. calcium hydroxide, ZOE, or polycarboxylate cement base is used, then these are placed against dentin, and the varnish is not applied until the base material has hardened. The varnish reduces the initial microleakage of an amalgam restoration. ► ZPC is used as a luting agent (cement) for gold

restorations and orthodontic appliances, and as a base under certain restorations. ► ZPC may also be used as a liner when a particularly strong one is needed. It should always be preceded by at least 2 coats of cavity varnish to prevent pulpal irritation. Can be used under composite materials. ► ZPC has been extensively replaced by polycarboxylate or GIC. These cements are based on ion-cross linked polyacrylic acid matrices with the potential to react chemically with residual powder particles and the tooth structure surface. ► ZPC has superior strength compared to other cements, and its retention depends on mechanical interlocking (as opposed to GIC and polycarboxylate cements which adhere to tooth structure by the polyacrylic acid in the liquid). ► ZPC liquid that has lost some of its water content causes the setting time of the mix to increase (slows setting). ZPC shrinks slightly upon setting. Mixing ZPC very rapidly decreases the cement’s final compressive strength. ► Clean zinc phosphate cement from crown margins after the cement is completely set.

When mixing ZPC, a cool glass slab is used to increase the powder-liquid ratio. A cool mixing slab should be used (caution: the slab’s temperature should not be below the dew point of the room). Mixing should be started by adding a small amount of powder to the liquid. This procedure, along with the cool slab, increases the cement’s working time. Advantages of using the cool slab method: substantial increase in working time of the mix on the slab, and shorter setting time of the mix after placement in the mouth. ► Small increments of powder are added ~ every 20 seconds with vigorous mixing until a creamy consistency is achieved to promote a high powderliquid ratio and superior cementation medium by providing a lower viscosity of the mix, stronger final set, and lower solubility of the set cement. ► Zinc phosphate cements provide good pulpal protection from thermal, electrical, and pressure stimuli, but can damage the pulp due to its initial acidic/low pH. However, this can be beneficial since it provides an anti-bacterial effect which reduces the number of viable microorganisms in the cavity floor, thus decreases pulpal irritation. Factors that govern setting rate of zinc phosphate cement: powder-liquid ratio, rate of powder incorporation,

powder particle size, manner of spatulation, and water content in the liquid. 1. In mixing zinc phosphate cement the liquid-powder ratio is the clinical variable with the greatest effect on the cement’s strength. 2. Prolonged, vigorous mixing of zinc phosphate powder into the liquid on a cool glass slab promotes a higher powder-liquid ratio and superior cementation medium by providing a lower viscosity of the mix, stronger final set, and lower solubility of set cement. ZINC POLYCARBOXYLATE CEMENT-first cement system developed with a potential for adhesion to tooth structure. The polycarboxylate cements are powder/liquid systems. Its liquid (an aqueous solution of polyacrylic acid + copolymers), and its powder (zinc oxide + magnesium oxide). ► Their compressive strength is slightly lower than zinc phosphate cements, but they have greater tensile strength. Its final strength depends on the powder-liquid ratio (the more powder, the greater strength). The strength of the set material is sufficient for amalgam condensation and its effect on the pulp is mild enough to eliminate the need for sub-lining. Zinc polycarboxylate cement is used strictly as a cement (luting agent).

► Thermal conductivity is low, so it provides good protection against thermal stimuli applied to metallic restorations. When cementing a cast restoration, always apply cement to the restoration and the tooth. ► When using polycarboxylate cement as a liner, it is not required to first place two coats of cavity varnish since this cement does not irritate the pulp (apply varnish AFTER placing the polycarboxylate cement liner). ► Advantage: it can bond to tooth structure due to the ability of carboxylate groups in the polymer molecule to CHELATE TO CALCIUM in the tooth. ► Disadvantage: extremely short working time. Glass ionomer and polycarboxylate both contain polyarcylic acid. Polycarboxylate cements may form a chemical bond to enamel. Compared with zinc phosphate cement, polycarboxylate cement has lower compressive strength, has equal tensile strength, elicits less pulp response, and can bond to tooth structure.

PREPARATION BASES BASES-materials 1-2mm thick that function as barriers against pulpally irritating agents, provide thermal insulation below a restoration, and provide adequate resistance to compressive forces of mastication. In essence, bases serve as a replacement or substitute for the protective dentin destroyed by caries and/or cavity preparation. • Materials used as bases: ♦ Zinc phosphate cement: remember to seal dentinal tubules with varnish prior to application. ♦ Zinc polycarboxylate cement: provides adhesion and does not irritate the pulp. ♦ ZOE (Type III or Reinforced ZOE): not used under composite restorations since eugenol inhibits composite’s setting reaction. ♦ GIC: provides fluoride release and adhesion. ♦ Calcium Hydroxide (“hard setting”) Dycal: thicker than when used as a liner. It is very effective in promoting secondary dentin formation (an important aid in pulpal repair). All of these materials are suitable bases under AMALGAM & COMPOSITE restorations. However, for composites, ZOE cannot be used because eugenol inhibits the composite setting reaction.

Primary Bases-placed on dentin in close proximity to the pulp to provide protection from toxic and thermal irritants. ♦ Under amalgam and composite restorations, calcium hydroxide is the primary base. ♦ Under gold restorations, the primary base is zinc phosphate or zinc polycarboxylate cement, or GIC. ♦ A primary base is not needed under zinc polycarboxylate cements, since these cements do not irritate the pulp. Secondary Bases-the most common use is placing zinc phosphate cement over a calcium hydroxide base that has been placed over a pulpal exposure (direct pulp cap). Important: Cements used as bases should be mechanically stronger than when used as luting agents and are mixed with the maximum powder content possible. A low powder-liquid ratio produces a low viscosity cement that is needed for luting agents. Selecting a base to be used under a permanent restoration is governed by the cavity design, type of permanent restorative material used, and the pulp’s proximity in relation to the cavity wall. The most important consideration for pulp protection in restorative

techniques is the thickness of the remaining dentin. • Cements thicker than 2mm are bases and function to replace lost dentin structure under restorations. A base can be used to provide thermal protection under metallic restorations, increase resistance to forces of amalgam condensation, or to block out undercuts when taking impressions for cast restorations. A base should not be used unnecessarily. • The only difference between a base, cement, and a cavity liner is their final application thickness. • Cements for luting have a desired final film thickness of 15-25 microns. • Cavity liners (solution or suspension liners) have a desired final film thickness of 5 microns. • Bases have a final application thickness of 12mm (they can be thicker depending on the amount of dentin that was destroyed). Selecting the appropriate base or liner to restore the axial wall of a Class II restoration depends on the biological effect required, and thickness of remaining dentin.

CAVITY LINERS CAVITY LINERS-used to PROTECT THE PULP, placed as thin coatings over exposed dentin. Their main purpose is to protect the pulp by creating a barrier between dentin and pulpally irritating agents (i.e. acids from etchants or cements, restorative materials, etc.) by SEALING the dentinal tubules. Two types of cavity liners: 1. Cavity Varnish (Solution Liner or Copalite)-a natural gum like copal, rosin, or synthetic resin dissolved in an organic solvent like acetone, chloroform, or an ether. It seals dentinal tubules without adding bulk. Copalite is the most commonly used and reduces initial microleakage of a restoration. Copalite is NOT used under composite because it inhibits the polymerization of the resin. Functions of cavity varnish (solution liner): • Reduces marginal leakage to improve the marginal seal for the short term. • Helps prevent acid penetration. • Protects pulpal tissues from phosphoric acid in zinc phosphate cements. • Prevents mercury penetration into the dentinal tubules from amalgam restorations, which prevent dentin discoloration. • Cavity varnish (copalite) does not act as a thermal

barrier (does not provide thermal protection), and should not be used with composite restorations because it inhibits polymerization. • With any restorative resin, cavity varnish or ZOE is NOT used because they can inhibit polymerization. Using cavity varnish might prevent direct contact between the composite and tooth structure, preventing bonding. 2. Suspension Liner (Zinc Oxide Eugenol & Calcium Hydroxide)-a liquid where calcium hydroxide & sometimes zinc oxide are suspended in a solution of natural or synthetic resins. Suspension liners are thicker (15 microns) than solution liners (1-5 microns). Suspension Liner Functions: • prevents penetration of acids and thermal shock and has adequate strength for use under permanent restorations. • ZOE is a very good liner since it has a palliative (soothing) effect on the pulp, thus is commonly used for temporary fillings. • Calcium Hydroxide (Dycal)-the most commonly used suspension liner that prevents thermal shock, prevents passage of acid from restorative materials, and has enough strength to resist forces used in placing restorations. Calcium hydroxide can stimulate secondary dentin formation when placed

near or in direct contact with pulp. When viewed radiographically, it can be easily confused with caries as both are radiolucent. • Cavity liners (solution and suspension) are being replaced by new dentin bonding agents. Cavity varnish functions in an amalgam restoration: to improve marginal seal, prevent dentin discoloration, and prevent acid penetration to the pulp from cements. When both a cavity varnish and polycarboxylate cement base are being considered for use in conjunction with an amalgam restoration, the cavity varnish should be applied to the cavity preparation after placing the base. Leakage after insertion of an amalgam is reduced when a cavity varnish is used. Cavity varnish is indicated for use under amalgam restorations to improve the marginal seal of the restoration. Cavity varnish is indicated under amalgam restorations because it improves the marginal seal of the restoration (it does not prevent galvanic currents from reaching the pulp nor does it completely seal the dentinal tubules). 3. Calcium hydroxide, U.S.P. is the dental material easily confused with caries when viewed

radiographically. 4. When using the acid-etch technique to restore a Class IV fracture, exposed dentin should first be covered with a calcium hydroxide liner. 5. The copal resin varnish that is placed in the cavity preparation before the amalgam is condensed provides short-term sealing of the margins. WHITENING (BLEACHING): the most conservative way to lighten VITAL teeth is bleaching. Two bleaching methods: 1. Office Bleaching: most use a light-activated solution of 35% H202 in 4-10 minute cycles. Other methods to lighten vital teeth: • Direct composites; useful for gray tetracyclinestained teeth (veneers are also an option). • Porcelain Veneers: useful when shape, size, and arrangement of teeth are esthetically poor. • Full-Coverage Crowns: the most invasive (least conservative). Materials include all porcelain, zirconium, PFZ, or PFM. 2. Home Bleaching: the active ingredient contained in all home tooth whiteners that has ADA approval is a 10% carbamide peroxide concentration. The active ingredient in most OTC home bleaching products is NOT

carbamide peroxide, but H202. Bleaching can affect the color of dentin and enamel. EXTRINSIC STAINS respond best to vital bleaching. Yellow stains respond best (then brown and orange stains). The worst response to vital bleaching is gray stains (tetracycline staining).

CARIES DENTAL CARIES-an infectious microbiological disease process that causes localized dissolution and destruction of tooth structure (calcified tissues). For caries to occur there are essential factors required to initiate a carious lesion: 1. Susceptible host (tooth). Fluoride and occlusal sealants modify the susceptible host (tooth). 2. Microflora with cariogenic potential (plaque). 3. A suitable substrate (dietary carbohydrates). These three factors interact to promote the disease severity. The greatest percentage of tooth loss in the first 20 years of life (except from natural loss of deciduous teeth) is due to untreated dental caries. The reason that the rate at which carious destruction of dentin progresses slower in older adults than in younger people is because of the generalized dentinal sclerosis that occurs with aging. Dental caries is initiated at the tooth surface by the growth of Streptococci (S. mutans, S. mitis, S. sanguisthe most frequently isolated Streptococcus in the oral cavity, & S. salivarius). These bacteria produce DEXTRAN SUCRASE (Glucosyltransferase), which catalyzes formation of extracellular glucans from dietary sucrose.

Glucan production contributes to dental plaque formation which then holds the lactic acid produced by Streptococci against the tooth. The lactic acid then eats through enamel, creating caries. Predominant Bacteria Found in Plaque: ♦ Streptococcus sanguis (found the earliest). ♦ Actinomyces viscosus & naeslundii. ♦ Streptococcus mutans, mitis, salivarius. ♦ Veillonella, Lactobacilli casei, & Fusobacterium. Dental Plaque-the soft, white film of organized bacterial colonies (main component), salivary glycoproteins, and inorganic material that readily forms on teeth surfaces. *The strong correlation between the presence of dental plaque and appearance of dental caries and periodontal disease has been recognized for many years. Cariogenic Bacteria: the evidence of bacteria’s role in the genesis of dental caries is overwhelming. Organisms that cause caries are “cariogenic”. Mutans Streptococci: Streptococcus mutans & sobrinus are the two most common cariogenic bacteria found in man. Properties of Cariogenic Bacteria: ► Acidogenic (produce acid) and aciduric (able to tolerate an acidic environment). Lactic acid is formed in large quantities after the degradation of sucrose by mutans

streptococci. Lactic acid produced by acidogenic bacteria is the main cause of enamel decalcification. ► The ability to attach to the root surface. *Streptococci species have special receptors for adhesion to the surface and produces a sticky matrix that allows them to cohere to each other. ► The ability to form a protective matrix. Streptococci species produce an extracellular insoluble dextran that protects them from being removed from the teeth by saliva, liquids, foods, and masticatory forces. *Glucans occur as dextrans or mutans, and are synthesized from sucrose by plaque bacteria as extracellular polymers of glucose. Levans are polymers of fructose. S. sanguis is etiologically related to caries, but is NOT a primary etiological agent in caries. Main cause or etiology of caries is bacteria or plaque formation. There is abundant evidence that the initiation of dental caries requires a high proportion of Streptococcus mutans within dental plaque. The first in caries development is the deposit of plaque

on teeth. Dental plaque is a highly organized gelatinous mass of bacteria that adheres to the tooth surface. Streptococcus mutans produce large amounts of lactic acid (acidogenic), are tolerant of acidic environments (aciduric), are vigorously stimulated by sucrose, and are the primary organisms associated with dental caries. However, other organisms are required to initiate caries. Other mutans streptococci species in humans can also do this (i.e. S. sobrinus). Factors the tooth surface is directly exposed to and that contribute to caries development: 1. Amount of plaque: large amounts of plaque on teeth (many bacteria that can produce acids) create a low pH or acidic environment to cause tooth demineralization. 2. Type of Bacteria: large proportions of “cariogenic” bacteria cause a lower pH, sticky plaque, and prolonged acid production. 3. Type of Diet: high amount and frequency of carbohydrates, and sucrose. A “sticky” diet leads to a low pH for a longer time period. High sugar frequency results in longer timer per day with a low pH. 4. Saliva Secretion: reduced saliva flow leads to prolonged sugar clearance time and a reduced amount of other saliva protective systems.

5. Saliva Buffer Capacity: low saliva buffer capacity results in prolonged time with a low pH. Saliva helps prevent caries by diluting acid, acting as a reservoir for Ca and PO4 ions for remineralization, and acts as a reservoir for Ca, PO4, fluoride, and other ions for hypermineralization of the enamel surface. 6. Fluorides: if too low or absent results in reduced remineralization. Developmental pits & fissures are the most susceptible areas on a tooth for plaque retention. Pit and fissure caries has the highest prevalence of all caries. Smooth surface areas (especially proximal enamel surfaces) immediately gingival to the contact area are the 2nd most susceptible areas to caries. Streptococci and lactobacilli species are common in this area. Facial and lingual root surfaces may have plaque-containing filamentous actinomyces species which can cause root surface caries. Fluoride treatments dramatically reduce smooth surface caries, although they are not as effective in preventing pit and fissure caries. Sealing pits and fissures after tooth eruption may be the single most important procedure to help protect these areas from caries destruction. Changes of pulp and dentin depend on the rate of carious progression. The pulp’s response to carious

attack or trauma from operative procedures depends on the pulp’s blood supply & cellular activity. Pulp defense mechanisms to protect it from irritation include: ► sclerotic dentin (peritubular dentin) formation (pulp’s initial defense mechanism). ► reparative dentin (irritation dentin) formation (pulp’s 2nd line of defense). ► vascularity (inflammation). Acute Caries (Rampant Caries)-characterized by rapidly progressing, little or no staining, often multiple, soft-totouch lesions, & most common in children. The lesion entrance is small, but the lesion is deep and narrow in large lesions. Pain may be feature. Chronic Caries (Slow or Arrested Caries)-common in adults, progresses slowly, and the lesion entrance is wide. Characterized by dark pigmentation with leathery dentin, shallow lesion (small lesion), and pain is uncommon. Chronic caries should be completely removed when found in enamel and close to the DEJ. Root Surface (Senile Caries)-often found in older patients and attacks cementum & radicular (root) dentin. The rising incidence of root caries is attributed to the aging population and most adults are retaining more teeth. In this population, there is increased gingival recession with exposure of root surfaces, leading to root surface caries which generally spreads more on the

surface, rather than in depth. In older patients, rampant caries can be caused by poor oral hygiene, decreased salivary flow, and medication side effects. Bacterial Etiology: the same as for coronal caries (S. mutans, S. sanguis, A. viscosus, A. naeslundii, lactobacillus, and Veillonella). Best way to prevent root caries is to maintain the periodontal attachment. Gingival recession is most related to the initiation of caries in the elderly. Microorganism most commonly associated with root surface caries is Actinomyces viscosus. Most current research suggests the microbial etiology of root caries is very similar to coronal caries (in the past it was thought Actinomyces species (viscosus and naeslundii) were most commonly associated with root surface caries. Residual Caries-caries that remains in a completed preparation either by the dentist’s intention or accident. Secondary (Recurrent) Caries-decay appearing at and under restoration margins. Zones of Carious Dentin (Innermost Zone→Outermost

Zone): 1. Zone I (Normal Dentin)-the innermost zone of normal dentin with no bacteria in the tubules. 2. Zone II Carious Dentin (Sub-transparent Dentin)-zone of demineralization created by acid from caries. Damage to the odontoblastic process is evident, but this zone does not contain bacteria. Capable of remineralization. 3. Zone III (Transparent Dentin)-softer than normal dentin and show further demineralization. No bacteria are present. Capable or remineralization. 4. Zone IV (Turbid Dentin)-the zone of bacterial invasion as the dentinal tubules are filled with bacteria. This dentin cannot remineralize and must be removed prior to restoration. 5. Zone V (Infected Dentin)-the outermost zone consisting of decomposed dentin filled with bacteria. This dentin must be completely removed prior to restoration. 4 Zones of an Incipient Carious Lesion in Enamel: 1. Translucent Zone-the deepest zone that represents the advancing front of the enamel lesion. 2. Dark Zone-does not transmit polarized light. Areas of demineralization and remineralization. 3. Body of the Lesion-the largest portion of the incipient

lesion that has areas of demineralization. 4. Surface Zone-relatively unaffected by the caries attack. Saliva helps prevent caries by diluting acid and acting as a reservoir for Ca, PO4, fluoride, and other ions for remineralization and hypermineralization of enamel. It also affects caries through its anti-microbial properties. Enamel demineralization occurs at pH 5.5. Remineralization of damaged tooth structure occurs as pH rises above 5.5. Cementum demineralization occurs between 6.0-6.7 pH. • Prevalence of caries is decreasing in children. However, a decline in adult caries is not as evident. Fluoridation has received the most credit for the decline in caries development. • Pregnant patients, compared to similar non-pregnant patients, are likely to have the same degree of caries, but more inflamed gingival tissues. • In radiographs of an incipient carious lesion limited to the enamel on proximal surfaces of a posterior tooth, the lesion appears a radiolucent area that is smaller in the radiograph than actually exists clinically. • An incipient carious lesion on an interproximal surface is usually located gingival to the contact

area. • Three essential factors for the initiation of a carious lesion: bacteria, suitable substrate, and susceptible tooth. • A radiograph of a first molar shows a very extensive carious lesion that may involve a horn of the dental pulp. The treatment of choice is to remove a major portion of the decayed tooth tissue and place an interim sedative dressing. • A tooth elicits a sharp stabbing pain caused by mastication of sweets or application of cold. Pain is relived by warmth and frequently by direct pressure. The cause of the pain is a moderate carious lesion with resultant pulpal inflammation. • Dental caries incidence is higher in children and adolescents because their dietary habits are erratic, there are more unrestored surfaces in patients of this age range, their teeth have not had the advantage of fluoride’s cumulative effect, and their oral hygiene is erratic and inefficient. • Streptococcus mutans is considered to be a principal etiologic agent of caries because it produces organic acids and forms a gelatinous matrix material.

• The rate at which carious destruction of dentin progresses tends to be slower in older adults than in young persons is because of the generalized dentinal sclerosis with aging. • Maxillary first molar is the tooth most likely to benefit from occlusal sealant placement. • A large carious lesion on the distal surface of a maxillary central incisor involving the incisal angle is a smooth surface lesion.

GOLD

DUCTILITY-a metal’s ability to easily be worked into desired shapes (i.e. ability to form a wire from a metal). These materials undergo extensive plastic deformation prior to fracture (in tension). Ductility depends on plasticity & tensile strength. • Ductility is usually expressed in terms of the

percent elongation (the higher the value, the more ductile the alloy). Ductility decreases as temperature increases. MALLEABILITY-a metal’s ability to be hammered (compression) into a thin sheet without rupture. Malleability depends on plasticity, but is not as dependent on tensile strength as ductility. Malleability increases as temperature increases. • Pure gold can be contaminated when it comes in contact with moisture, sulfur or eugenol vapors, or an oxygen-rich flame. ALLOY-a mixture of two or more materials mutually soluble in the liquid state. A pure metal solidifies at a constant temperature, while alloys solidify through a range of temperatures. Alloys are used in dentistry for cast restorations. 1. Base Metal Alloys (non-precious metals)-are based on active metallic elements that corrode, but which develop corrosion resistance via surface oxidation that produces a thin, tightly adherent film that inhibits further corrosion (i.e. cobalt-chromium alloys form a Cr2O3 oxide film that passivates the surface). • Base metals are LESS resistant to corrosion than noble metals. Their advantages are mainly based on their strength and low density. They are stronger and less

dense than noble metal alloys. Base metal alloys are also harder to cast and finish than noble metals. 2. Noble Metals (Precious Metals)-are very resistant to corrosion and do not oxidize on casting. Noble metals used in dentistry are based on the noble or precious metal elements of gold, silver, palladium, and platinum. 3. Solid Solution Alloys-the metals freeze without segregation of the individual constituents. These are generally used in dentistry because they have a very homogenous structure and provide maximum strength. 4. Eutectic Alloys-separate into individual grains of the respective constituents. Exhibit complete liquid solubility, but limited solid solubility (i.e. silver-copper system). KARAT- number of “pure gold” parts of a gold alloy, based on 24 parts as a unit (24 karat is 100% gold, 18 karat 75% gold). FINENESS-measured based on the parts of pure gold per 1,000 (1,000 fineness is 100% gold; 500 fineness is 50% gold). Pure gold is only used in the gold foil restoration. 4 High-Gold Alloys: high-gold alloys used for cast restorations are > 75% gold or other noble metals.

1. ADA Type I: highest gold content (83% noble metals). Used for small inlays and easily burnished due to high ductility. 2. ADA Type II: > 78% noble metals. Used for larger inlays & onlays, and can also be burnished. 3. ADA Type III: > 75% noble metals. Used for onlays & crowns. Capable of being heat-treated. When a Type III dental casting gold alloy is heated to a cherry red color and quenched immediately, malleability and ductility are increased (surface hardness and strength are decreased). 4. ADA Type IV: > 75% noble metals. Used for bridges and RPDs (hardest high-gold alloy). Can be heat-treated. Medium-Gold Alloys-contain 25-75% gold or other noble metals. Low-Gold Alloys-contain < 25% gold or other noble metals. Gold Substitute Alloys-do not contain gold, but are called “passive” because they form a protective surface oxide film layer that provides maximum corrosion resistance (i.e. palladium-silver alloys and cobalt-chromium alloys). Advantages of Cast Gold Restorations: strong and able

to withstand mastication forces, ideal for occlusal rehabilitation, and are kind to gingival tissue. Gold alloy against gold alloy is the combination that most likely results in the LEAST occlusal wear. Disadvantages of Cast Gold Restorations: gold has a high thermal conductivity, is expensive and non-esthetic, time consuming, technique sensitive, and the need to use cement which is the weakest point in the cast gold restoration. For maximum retention of cast gold restorations, the axial walls should be as parallel and as long as possible. Retention is directly proportional to the area of the axial walls and their parallelism. Axial walls should converge slightly from the gingival walls to the pulpal wall. • Cement’s main function in a cast gold restoration is to seal the cavity (not for retention). Retention is designed within the preparation and results from friction between the cavity wall and the casting. ONLAYS: main advantage is it can permanently restore and reinforce a tooth by a conservative technique. While conserving tooth structure is desirable, it is offset by a lack of retention. Conservative onlays have inferior retention than full crowns due to the crown’s greater axial surface area. Cast Gold Onlay Indications:

♦ Restore large lesions that involve more than 1/3 intercuspal dimension, extensive loss of supporting structure where at least 50% of the clinical crown remains, or loss of cusp(s) with at least 1mm of dentin supporting the remaining cusps. ♦ Restore ideal occlusion in cases of drifting, hypoeruption, & hypereruption. ♦ Restore optimal contour and proximal contact. ♦ Restore brittle teeth (RCT treated teeth). ♦ Restore an abutment tooth for an RPD to create ideal guiding planes, rest seats, and undercuts. ♦ Restore teeth to meet patient preference for gold. Parallelism of vertical surface (axial walls) is the primary retentive feature in an onlay preparation. Sharp point and line angles increase onlay retention. Auxillary retentive features include a box or groove (indicated where inadequate surface area of vertical walls is present). A box offers a greater increase in surface area, thus greater retention than a groove, but is also more costly in terms of lost tooth structure. The location of the gingival margin when preparing proximal surfaces is influenced by the amount of retention required, need to extend gingivally to clear the contact area, and convenience form. With respect to ONLAY preparations, “shoeing” a functional cusp is NEVER INDICATED (shoeing is never

indicated on functional cusps). There are two types of cuspal protection for onlay preparations to achieve proper resistance form: 1. Cap-complete coverage of the cusp. Except in situations demanding a minimal display of gold (primarily facial cusps of maxillary molars and premolars), capping is always preferred over shoeing. 2. Shoe-minimal or partial cusp coverage via a finishing bevel on the cusp crest. Axial walls in an MOD cavity preparation for a cast gold onlay should converge from the gingival walls to the pulpal wall. When preparing a tooth for an MOD onlay, occlusal reduction is influenced by ideal location of subsequent centric contacts, amount of clearance existing before reduction, minimal thickness needed to satisfy physical requirements of the restorative material (it is not influenced by the thickness of enamel present on the occlusal surface). Always bevel or plane the cavosurface margins or wall junctions of an onlay cavity. Near parallel axial walls is the factor that contributes the greatest amount of retention to an onlay restoration.

For an onlay preparation, the most effective means for verifying adequate occlusal clearance is a wax bite chew-in. When placing temporary restorations for several onlays in a quadrant, acrylic resin is an acceptable interim material if cemented with ZOE. When making an acrylic resin temporary restoration for a large MOD onlay preparation, necessary precautions to take are to avoid open margins, remove overextended resin, and polymerization should not go to completion in the mouth. In preparing a tooth to receive an onlay, a gingival bevel is used to remove unsupported enamel and compensate for casting inaccuracy. The dentist bevels the gingival margins of a gold onlay preparation. Beveling removes loose enamel rods, facilitates finishing, and minimizes marginal opening. Beveling does not serve to minimize the need for gingival extension. GOLD INLAYS: • Lack of undercuts (avoid undercuts) is the characteristic common to all Class II gold inlay preparations. The restoration will not seat if undercuts exists. This is true for all cast metal

restorations. • When designing a Class II inlay preparation, an occlusal lock (dovetail) should be established to prevent proximal dislodgement. Also, marginal ridges of posterior teeth restored with cast gold should be rounded to help form the occlusal embrasures and be in contact with opposing tooth cusps. Marginal ridges should be the same height as the adjacent tooth’s marginal ridge (to prevent an interference in retrusive movement). • When removing a Class II inlay, the best method is to cut through the isthmus to remove the occlusal and proximal pieces one at a time. Class II Gold Inlay Advantages or Indications: • Desire for permanency, low caries index, and for moderate size lesions with conservative outlines. • Esthetics (posteriorly where amalgam staining is to be avoided). • Tooth contours (where optimum contour and surface finish is desired to maintain periodontal health). • Rest seat retainers on abutment teeth. Class II Gold Inlay Disadvantages or Contraindications: • Expensive (gold is 6-7x more expensive than amalgam). • Time (at least two visits are required).

• Minimal lesions (best restored with gold foil). • Large lesions (if the cavity width exceeds 1/3 the intercuspal width, the tooth should receive cuspal coverage). 3 Types of Inlay Casting Waxes that differ in melting point and flow: 1. Type A: hard or low-flow wax rarely used except in some indirect techniques. 2. Type B: medium-flow wax used in some direct techniques. 3. Type C: soft or high-flow wax (the softest dental inlay casting wax) used in indirect techniques to construct inlays, onlays, and full crowns. Essential ingredients of a successful inlay wax: paraffin wax, gum dammar, and carnauba wax with some coloring material. ► Paraffin wax-the main ingredient (usually 40-60% concentration). ► Gum dammar-added to paraffin to improve smoothness in the molding and increases the wax toughness. ► Carnauba wax-hard and decreases wax flow. However the wax pattern is prepared, it should be an accurate reproduction of the missing tooth structure. The casting cannot be more accurate than the wax pattern. The inlay wax pattern should be invested ASAP after

fabrication to minimize changes in shape caused by relaxation of internal stresses in the wax. 1. First step when fitting a gold inlay casting in the mouth is to ADJUST PROXIMAL CONTACT AREAS (this applies to all crowns too). 2. When preparing an MO cavity for an inlay on a maxillary first molar, the oblique ridge should be crossed when the ridge has a defective fissure, is undermined by a carious lesion, and extension into the distal pit is necessary for retention of the mesial restoration (not if there is an incipient caries in the distal pit). 3. Forces for seating an inlay should be applied with a sustained heavy force from an instrument and by properly directing occluding forces of the patient prior to initial set of the cement. Cavosurface Angle Configurations used preparing a tooth for a Cast Gold Restoration:

when

1. Bevel-a diagonal cut across the cavosurface margin that is flat in one dimension only, and is curved in its other dimensions. It involves the external ends of enamel prisms, and follows a continuous curved outline. It can be a short bevel (cuts only the external 1/3 of enamel prisms), a full bevel (involving the entire thickness of enamel), or wide bevel (involving full thickness of

enamel and some dentin). 2. Chamfer-essentially a hollow ground bevel (not a flat diagonal cut across the cavosurface margin). The chamfer is “scooped-out” to create more bulk of restorative material near the margin and provide a greater cavosurface angle. 3. Plane-a diagonal cut across the cavosurface margin that is flat in all dimensions, and may involve the entire enamel thickness (which is usually does) or most of it, but cannot be curved in any direction. A cavosurface bevel is used when preparing a tooth for a cast gold inlay or onlay mainly to improve marginal adaptation. The cavosurface margin bevel permits closer adaptation of the gold margin because the thinner margin of gold overlying the bevel is more ductile and can be burnished. During cementation, the finishing (burnishing) of the margins of a cast gold restoration is started as soon as the restoration is well-seated into the preparation. • While preparing an inlay or onlay, a gingival bevel is used to remove unsupported enamel and compensate for casting inaccuracies. Gingival margin trimmers, carbide finishing burs, or fine tapered diamonds can place the cavosurface bevel. This gingival margin is always placed gingival to the contact area.

• When preparing teeth with short clinical crowns, facial and lingual walls should have a minimal gingival to occlusal divergence angle for maximum retention. • From facial to lingual, the axiopulpal line angle of an onlay preparation is longer than the axiogingival line angle (if it were not, the preparation would be undercut and the onlay would not seat). For an MOD onlay preparation, axial walls must converge from the gingival walls to the pulpal wall (for the same reason the onlay would not seat if the axial walls diverged). Crystalline Gold (Mat Gold)-formed by electrolytic precipitation yielding a crystalline structure resembling trees or links of chain. Mat gold is used for bulk filling of cavities. The flow and adaptation of mat gold is not as good as gold foil or powdered gold. Powdered Gold-gold formed by atomizing. Most of the granules in this material have an overall spherical shape. Advantages: Powdered gold can be placed in a very short time period and a gold foil veneer is not required. Also, it is denser than foil, thus easier to manipulate and condense, which saves time. ► Cohesion of direct gold at room temperature is an example of “atomic attraction”. Gold Foil-the traditional and oldest type of gold formed

by rolling and beating gold into thin sheets. This process causes elongation of grains which give a fibrous appearance. Gold foil is available in sheets, cylinders, and pellets, and is used for bulk filling and as a finishing veneer for mat gold. ► There are always microscopic voids in any compacted gold, no matter which type is used. The cause of porosities or pits in a gold foil restoration is related to improper condensing technique and using oversized pellets. ► The most important factor in securing adaptation of gold foil to all parts of the preparation is the direction that the force is applied (and using a small condenser point). ► During condensation, surface hardness of gold foil is always increased. Direct Filling Gold: MAIN indication for using a direct filling gold is a small initial Class III lesion. Reconciling permanency and esthetics are the key limitations and challenges to this classification. Lesions on the distal surfaces of all anterior teeth are relatively invisible and are less of an esthetic concern than mesial lesions. In most cases, the distal restoration in a canine is not visible. • Has a high tensile strength (edge strength). • Is the most nearly permanent of all restorative

materials. • Provides good adaptation to cavity walls. • Its coefficient of thermal expansion is close to that of tooth structure. • It will not corrode. • Direct filling gold is considered by many operative dentists to be the finest and most desirable restorative material. However, the additional skill required, demand for close attention to detail, and the limitation in versatility are disadvantages of the technique which have lowered it popularity, resulting in a utilization rate much lower than what the material deserves. Disadvantages of direct filling gold: poor esthetics, demanding technique, high cost, high coefficient of thermal conductivity (12x that of amalgam). • The amount of force required to compact direct gold is influenced mainly by the surface area of the condenser. A decrease in diameter of a condenser point produces a proportional increase in energy concentration. For this reason, good concentration with less force and trauma is accomplished with a SMALL POINT. During condensation, the surface hardness, tensile strength, and yield strength of direct filling gold increase. • Direct filling gold is heated prior to condensation to

drive off moisture and volatile compounds that have been placed on its surface. • Direct Filling Gold Indications: ♦ Ideal lesion: no greater than 1-2mm into dentin, and of minimal outline form. ♦ Ideal pulp: at least 2mm of dentin between the restoration and pulp. Pulp should be vital with no history of trauma or tooth sensitivity. ♦ Ideal periodontium: no tooth mobility, no inflammation or degenerative processes are present. Class III Cavity Preparation for Direct Filling Gold: • Outline form is a horizontal slot. The incipient lesion and subsequent preparation are usually positioned gingival to the contact area. The preparation has a flat labial wall and straight, parallel incisal and gingival walls. The axial wall is flat and there is no lingual wall. • Retention form is attained by a sharp internal anatomy with well-defined line and point angles. Retentive grooves are placed along the inciso-axial and gingivo-axial line angles. • Resistance form is provided by flat walls and a flat well-supported labial wall.

• A 26-year old woman has a Class III lesion on the distal aspect of a canine. The restorative material of choice is amalgam or direct filling gold. • Mechanical separators used in Class III direct filling gold procedures should routinely be stabilized to minimize trauma to the gingival tissues, provide more rigid resistance to condensation forces, and provide uniform support during instrumentation. Class V Cavity Preparation for Direct Filling Gold: not very common today, but may be tested. • Sharp internal line angles and small retentive undercuts placed in the axio-occlusal and axiogingival line angles. • Mesial & distal walls flair and meet the cavosurface at a 90° angle. M & D walls are placed at the tooth line angles. • A convex axial wall that follows the tooth’s external contour and is placed 0.5mm into dentin (this makes the occlusal wall slightly deeper than the gingival wall because there is a thicker layer of enamel making up the occlusal wall). The axial wall is convex in a M-D direction to conserve tooth structure and minimize pulpal irritation. • Trapezoidal (most popular) or kidney-shaped outline form. For any Class V preparation (amalgam,

composite, or direct filling gold), the outline form is determined by the extension of the carious lesion. • RETENTIONN FORM is attained by sharp internal line and point angles (axio-gingival & axio-occlusal). Proper retention form in a Class V cavity prepared for direct filling gold is dependent upon the angulation of gingivoaxial and occlusoaxial line angles. Retention placed in a Class V cavity preparation for direct filling gold should be at occlusoaxial & gingivoaxial line angles. RESISTANCE FORM is provided by FLAT mesial & distal walls, and a CONVEX axial wall that parallel the tooth’s external surface. • In the standard Class V cavity preparation for direct filling gold, the mesial and distal walls diverge facially and are placed at the tooth’s line angles. • Placing the axial walls of a Class V cavity preparation for dental amalgam or gold too far axially could result in pulp exposure, sensitivity from thermal conduction, or sensitivity owing to damage of odontoblastic cells. • Proper retention form in a Class V cavity prepared for direct gold filling is dependent upon the degrees of

the axiogingival & axio-occlusal line angles. • The position of mesial and distal cavity margins in a Ferrier Class V direct filling gold restoration should be at the respective line angles of the tooth. INVESTING-process of surrounding a wax pattern with a material that can accurately duplicate its shape & anatomical features. • Dental Investment Functions: a detailed reproduction of anatomical form, enough strength to withstand the heat of burnout and the actual casting of the molten metal, and compensation expansion equal to the alloy solidification shrinkage. Gypsum Bonded Investments-used with Types I, II, III gold alloys. Main Components of Gypsum Bonded Investments: 1. Refractory Filler-a form of silicon dioxide (SiO2) like quartz or cristobalite that comprises 60-65% of the investment. These two compounds have different crystal structures, thus have different thermal expansion coefficients. This refractory filler regulates and provides thermal expansion for the investment. 2. Binder-a gypsum matrix of α-calcium hemihydrate that comprises 30-35% of the investment. This material

hardens after being mixed with the liquid, thus holds the investment together (adds strength). The actual or effective setting expansion depends on the gypsum content and water-powder ratio. Using a thinner mix that contains more water of a gypsum-bonded investment will decrease the setting expansion, increase setting time, increase porosity of the set material, and ultimately weaken the set material. 3. Modifiers-are added to modify various physical properties of the investment. Ex: magnesium oxide, NaCl, boric acid, graphite, or potassium sulfate. The strength of dental investments for gold alloys is dependent on the amount of gypsum. Gold alloys used for cast gold restorations shrink upon solidification. Thus, it is necessary to compensate for solidification shrinkage of the specific alloy used by expanding the mold enough to equal the shrinkage. The dimensional compensation necessary is accomplished by 2 methods of expansion: 1. Setting expansion-occurs as a result of normal crystal growth, but can be enhanced by allowing the investment to set in the presence of water, producing hygroscopic expansion. 2. Thermal expansion-achieved through normal

expansion that occurs upon heating silica (quartz or cristobalite). The amount of expansion depends on the type of refractory material used (cristobalite produces greater expansion than quartz). Thermal expansion is the main cause of mold expansion. Variables that Affect Gold Expansion: • Older the investment, the less it will expand. • Increasing the water-powder ratio decreases expansion. • Longer spatulation time, the greater expansion. • Longer time between mixing and immersion in a water bath, the less expansion. • During alloy solidification, the number of grains forming depends on the rate of cooling & presence of nucleating agents. SPRUE: the best angle to attach the sprue pin to the proximal wall of a wax pattern is 45°. The purpose of the sprue former (sprue pin) is to provide an ingate or sprue in the investment through which the molten alloy can reach the model after the wax has been eliminated. • The size of the sprue former depends on the type & size of the pattern, type of casting machine used, and dimensions of the flask or ring in which the casting is made. Generally, for the average size pattern, sprue formers smaller than 1.5mm diameters are contraindicated. If a sprue is too small, the molten metal freezes completely in this area first, and

localized shrinkage porosity results. The general rule for sprue pin diameter when using a centrifugal type casting machine is the sprue pin diameter should be equal to or greater than the thickest portion of the pattern. • It is desirable to attach the sprue at the point of greatest bulk in the pattern as there is less chance of distortion upon attaching the sprue, and the molten metal is more apt to remain liquid in this area until the entire mold is filled. The direction of the sprue former is also important, as it is never attached at a right angle to a broad, flat surface of the mold because the entering hot metal impinges the mold surface at this point causing turbulence of the metal, which creates shrinkage void or suck-back porosity. When the same pattern is sprued 45° to the proximal wall, a satisfactory casting is obtained. • Placement of the sprue is most related to turbulence of molten gold in the casting process. • Primary reason for using a casting-ring liner is it allows uniform setting expansion of the investment. VACUUM INVESTING-a method of investing that is more dependable in preventing surface nodules/defects on a casting. Nodules are caused by the collection of air bubbles during investing. The best way to eliminate these

nodular defects is to subject the water-investment mixture to a vacuum during the investing procedure to remove the air bubbles. ► Porosity of the investment is reduced by vacuum investing due to the increased density obtained. As a result, the casting’s surface texture is smoother, with better reproduction of fine detail. The investment’s compressive strength is increased slightly by the vacuum investment (the investment does not fracture as easy). ► Not all of the air is removed by the vacuum treatment. The amount removed depends somewhat on the consistency of the mix. The more viscous the mix, the more air bubbles remain in the investment. However, a thick mix is usually necessary because of the desired shrinkage compensation, and because of the poor surface texture obtained with a thin mix. Parts of a Flame Obtained from a Gas-Air Blowtorch to Melt Alloys: 1. Mixing zone-1st zone that is cool & colorless. The air and gas are mixed before combustion in this zone. 2. Combustion zone-2nd zone that is greenish-blue and surrounds the inner cone. This is an oxidizing zone where partial combustion occurs.

3. Reducing zone-3rd zone which has a dim blue tip and is the hottest area in the flame and is the only part of the flame that should be used to heat the alloy. When this zone contacts, the alloy surface is bright and mirror-like. The proper zone of a gas-air blowpipe flame used for melting casting gold alloys is the reducing zone. 4. Oxidizing zone-outer zone where final combustion between the gas and surrounding air occurs. When the oxidizing part of the flame contacts metal, a dull film of “dross” (scum on molten metal) develops over the metal surface. The proper zone in contact with the metal can be readily detected by the condition of the metal surface. If a casting fails to completely seat in a cavity preparation, you should first check for residual temporary cement or other debris in the cavity. Once all temporary cement and debris are removed, the first step in fitting the casting is to adjust proximal contact areas. 1. When seating a casting, the initial interferences are usually proximal contacts. Complete seating of the restoration is verified by an x-ray and sharp explorer used at the gold tooth margin.

2. When seating cast gold restorations, the restoration’s occlusion should be to the same degree that teeth contact in that quadrant and on the opposite side (use shim stock to check occlusion). 3. Initially, if a cast restoration is in hyperocclusion, the patient complains of cold sensitivity and pressure in the tooth. If the restoration is not adjusted, the tooth becomes very cold sensitive, show signs of mobility, and there might be recession on the facial gingival tissue. MINIMAL reduction of working cusps (functional cusps) for protection from forces of mastication is 2.5-3mm for amalgam, and 1.5mm for cast gold restorations. Working cusps are lingual cusps on maxillary teeth, and buccal cusps on mandibular teeth. MINIMAL reduction for non-working cusps (nonsupporting cusps) for amalgam is 2mm while forming a flattened surface (this provides resistance form), and 1mm for cast gold. Non-working cusps are buccal cusps of maxillary teeth and lingual cusps of mandibular teeth. For metal-ceramic (PFM) restorations: occlusal clearance is 1.5-2mm, and 1.5mm facial and lingual reduction. The difference between tooth preparation for a PFM restoration and porcelain jacket crown is mainly related to the configuration of the finishing line (margin). Chamfer or bevel for PFM restorations, and a butt joint

(shoulder) for porcelain jacket crowns. The most effective way to verify adequate occlusal clearance is a WAX BITE CHEW-IN. Cavity Classification (Class I through VI): developed by Dr. G.V. Black in 1908 as a standardized method of recording the need for restoration exists to facilitate communication among clinicians, researchers, and dental educators. The most commonly accepted way to classify cavities is by naming the surfaces (anatomical areas) involved. Cavity classification relates to the cavity location (not cavity size). Class I, V, & IV can involve any teeth (anterior or posterior). • Class I-involves pits and fissures (all other classifications involve smooth surfaces of teeth). • Class II-involve proximal and occlusal surfaces of premolars and molars. • Class III-involve proximal surfaces of incisors and canines that do not involve the incisal angle. • Class IV-located on the proximal surface of incisors and canines and involves the incisal angle. • Class V-located on the facial or lingual surface of all teeth, and do not involve a pit or fissure.

• Class VI-located on the incisal edges of anterior teeth or occlusal cusp heights of posterior teeth. *The best method to definitively detect incipient carious lesions on the interproximal surfaces of posterior teeth (distal surfaces of canines through molars) is by BITEWING RADIOGRAPHS.

COMPOSITES Enamel is total-etched with PHOSPHORIC ACID (37%) to roughen the enamel surface which forms little tags ~10-25 micrometers long to provide mechanical retention. Enamel rods are most effectively etched at the enamel rod ends. When using the acid etch technique, all enamel cavosurface margins should be chamfered/beveled (to form obtuse angles to afford more surface area for etching and to enhance the seal and retention to reduce microleakage). Also, the bevel improves esthetics and exposes enamel rod ends for acid attack. A properly acid-etched enamel surface appears DULL WHITE & CHALKY. One of the most effective ways to improve marginal seal and mechanical bonding of composite resins to tooth structure is to condition/pre-treat enamel with acid prior to inserting the composite resin. This procedure is the “acid-etch” technique. • The acid cleans the surface of debris left after cavity preparation to provide an opportunity for better wetting of the enamel by the resin. More important, a selective dissolution of the enamel occurs during etching. While enamel is usually porous, acid removes calcium salts to increase the size and number of microspaces present. This “acid-

etch” technique conserves tooth structure, reduces microleakage, improves esthetics, and provides micro-mechanical retention. • Studies show acid-etched composite resin restorations have the best initial seal (microleakage). However, over time this seal weakens (amalgam has the best seal over time). • Once the tooth is etched, it cannot be contaminated with saliva or you must repeat the entire etching procedure. The reduction of microleakage represents the most significant advantage of the acid-etch technique. The specific purposes of acid etching enamel before insertion of a composite restoration or a sealant are to provide more surface area and a roughened surface. According to microleakage studies, acid-etched composite resin is the restorative material that shows the best initial seal when placed in a cavity preparation, thus protects the pulp from the effects of microleakage. Enamel is etched usually with buffered phosphoric acid prior to placing a resin restoration to allow increased retention of the resin restoration, allow better sealing of margins of the resin restoration, and produce enamel irregularities ranging from 5-30

micrometers in depth. Acid etching enamel increases retention and adaptation of resin restorations by increasing surface area, conditioning the surface for better wetting, and creating surface irregularities for better mechanical locking. The correct technique of enamel etching prior to placing a composite resin restoration is etching with 30-50% phosphoric acid and rinsing thoroughly with water and blot drying with air only. 37% phosphoric acid in water is the solution used to etch enamel when using total-acid etch technique with composite resins. DENTIN BONDING SYSTEMS:-complex and multi-step systems that consists of: 1. Etchant-used to roughen the enamel surface to help provide mechanical retention. Removes the smear layer and open up the dentinal tubules to demineralize enough dentin to allow resin tags to form within the dentin structure. 2. Dentin Conditioner-the component of a dentin bonding system that functions primarily to remove the smear layer of dentin and etch the intertubular dentin to

produce microspaces within the dentin surface. Conditioner is placed after the enamel is etched. 3. PRIMER-a wetting agent applied after conditioning that provides micromechanical and chemical bonding to the microspaces created by the conditioner. 4. UNFILLED RESIN ADHESIVE (Bonding Agent)-a restorative material applied after the primer. The resin is then cured (light, self, or dual-cured). This layer can now bond to composite or amalgam. ► Unfilled resin has the lowest thermal conductivity and diffusivity (compared to amalgam, gold, and filled resin). This characteristic offsets the undesirable effects of the relatively high coefficient of thermal expansion (7-8x that of the tooth). Due to its low thermal conductivity and diffusivity, the unfilled resin restoration changes temperature quite slowly. Thus, it takes considerably longer for an unfilled resin restoration to become hot or cold compared to metallic restorations, which have a high thermal conductivity and diffusivity. Remember: A low coefficient of thermal conductivity is most characteristic of currently available cement bases. ► Unfilled resin compressive strength is low, and its yield and tensile strengths are even lower.

► Unfilled resins have a higher coefficient of thermal expansion, but lower modulus of elasticity than filled resins. ► Unfilled resins are the softest of all restorative materials. ► Compared to amalgam, filled resin, direct gold, and silicates, unfilled resins have the greatest extent of marginal leakage related to temperature change. Marginal leakage related to temperature change occurs to the greatest extent with unfilled resin. DENTAL ADHESIVES (3 MAIN ETCH TECHNIQUES): 1. TOTAL ETCH (ETCH & RINSE)-classic technique using 30-40% phosphoric acid to prepare the enamel and dentin for the adhesive. Creates the most post-operative sensitivity as the dentinal tubules are opened by the phosphoric acid etch, and unless adequately closed by bond, the pulp-irritating composition of resin irritates the pulp. Placing a glutaraldehyde-containing liquids (GLUMA) contain 5% glutaraldehyde and 35% HEMA. They coagulate dentin to obturate the tubules to reduce dentin’s permeability to the placed resin. Applying two, 1-minute applications of GLUMA, then suction off the GLUMA off (don’t wash it off or get it on soft tissue). • Advantages: prepares enamel, dentin, and sclerotic

dentin for bonding yielding HIGH BOND STRENGTHS. Can be used universally as it does not interfere with polymerization of dual-cured resins. • Disadvantages: etching dentin longer than 15 seconds can yield post-operative sensitivity and decreased bond strength if the demineralization penetrates deeper into the tubules than the resin tags, thus forming a gap. To help prevent, etch the enamel first for 10 seconds then placed the etch into the prepared dentin for 15 seconds and rinse. 2. SELF-ETCH-rely on 10% maleic acid or acidic monomers to remove the smear layer and demineralize dentin and enamel. Main Advantage: LEAST POSTOPERATIVE SENSITIVITY. Disadvantage: lower bond strength than total-etch and possibly selective etch. 3. SELECTIVE-ETCH-similar to total-etch by selectively etching enamel only, rinse, place GLUMA and suction off, then place the composite. Helps reduce postoperative sensitivity. UNIVERSAL DENTAL ADHESIVES-work with all three techniques (total-etch, self-etch, and selective etch).

7th Generation bonding agents (introduced in early 2000s), eliminate the need for separate etching, rinsing, and mixing for light-cured products. Advantages of 7th Generation Bonding Agents: • High bond strength to enamel and dentin (20-30 MPa) • Thin film thickness to ensure complete seating of restorations • Fluoride-releasing to prevent secondary caries. • Tolerant to moist and dry environments. • MINIMIZES POST OPERATIVE SENSITIVITY. • Available in light-cured and dual-cured formulations. • Disadvantage: decreased shelf-life due to acidic

formulation. 3rd Generation dentin-bonding agents: (late 1980s), the first generation to bond to tooth structure and dental metals/ceramics, but their adhesive retention decreased after 3 years intra-orally and high post-operative sensitivity. can produce bonding strengths nearly comparable to that of resin to etched enamel. These depend on difunctional coupling agents being able to bond to inorganic or organic compounds in dentin. Unfilled Resins (Acrylic) temporaries fabricated for inlays and onlays should restore and maintain proximal contacts, occlusion, and tooth contours. Margins should be closed and flush with the tooth. Most importantly, methyl methacrylate maintains the occlusal and interproximal contact relationships. For inlays & onlays, plastic (acrylic) provisional restorations are fabricated prior to the final restoration being cemented. Their physical properties enable them to withstand occlusal forces and the adverse oral environment for short time periods. Methyl methacrylate, ethyl methacrylate, and ethylene imine resins have been used to produce provisional restorations. However, methyl methacrylate (MMA) is the most common (the liquid monomer that is mixed with the polymer polymethyl methacrylate (the

powder). The MMA monomer partially dissolves the polymer to form a plastic dough. The monomer is polymerized by the action of an initiator (benzoyl peroxide). Polymerization should not go to completion in the mouth for fear that the provisional will not be able to be removed from the tooth. Provisionals are usually cemented with a ZOE cement. Main disadvantage of using methyl methacrylate as a permanent restorative material are its low resistance to abrasion and high coefficient of thermal expansion. • Most current composite resins are based on BISGMA as the primary monomer (a difunctional monomer) or UDMA (urethane dimethacrylate) difunctional monomer. Since both monomers are very viscous, they are diluted with another difunctional monomer (TEGDMA) to reduce the overall viscosity. • The high filler content and BIS-GMA resin matrix greatly reduce the coefficient of thermal expansion (compared to unfilled acrylic resins). The filler also reduces polymerization shrinkage and increases hardness. Bisphenol A-glycidyl methacrylate-the component common to most composite resins, sealants, bonding and glazing agents, and resin cements for orthodontic

bands. An unfilled BIS-GMA glazing resin may be used on the surface of a polymerized composite restoration because unfilled BIS-GMA glazing resin seals the margins and smoothes the restoration surface. SEALANTS generally are comprised of Bis-GMA. There are filled and unfilled sealants. Fillers (glass and quartz) provide hue, strength, and increase wear resistance. Unfilled sealants typically do not require occlusal adjustment (ex: 3M Clinpro), while filled sealants (ex: VOCO Grandio) must be adjusted if hyperoccluded. LOW WEAR RESISTANCE is the property of filled resins that is primarily to blame for the failure of Class II composite restorations. Ideally, composite resins should only be used to restore minimal cavities in posterior teeth (instances where it will not be subjected to excessive occlusal forces and when teeth are in occlusion, there is cusp-to-cusp contact (not cusp to restoration). • Posterior composite restorations are often indicated to treat occlusal lesions which allow conservative preparations. • Posterior composite restorations are contraindicated patients with heavy occlusion or parafunctional habits (bruxism).

• Posterior composite restorations may be indicated to restore Class II cavities in premolars where esthetics is important, the cavity margins are in enamel, and the occlusal contacts are on the enamel. • The major indication for posterior composites is a demand for esthetics by the dentist and patient. Other criteria are non-involvement of cusps, minimal occlusal contact, no excessive wear, and the isthmus must be no wider than 1/3 of the inter-cuspal distance. CONSERVATION OF TOOTH STRUCTURE MAIN ADVANTAGE. • Composite is the material of choice if the patient has a documented allergy to mercury. • In the past, posterior composite restorations were contraindicated in patients with a caries-active mouth. New thinking says you should manage the disease (dental caries) before or while you are treating the consequence of the disease (i.e. by placing restorations). Thus, current literature does not see a problem for these restorations in caries-active patients. These have as bad a prognosis as any other restorative treatment if the disease is not managed simultaneously. Although the ADA does not endorse composite resin as a substitute for amalgam in posterior teeth, composite

restorations can be excellent if strict guidelines are followed for tooth selection and if done properly. However, composite resin restorations are inferior to amalgam in compressive strength and abrasion/wear resistance, and they do not provide any anti-cariogenic effects as do freshly placed glass ionomer or resin-modified glass ionomers. Composites are classified based on FILLER PARTICLE SIZE (diameter in microns) & polymerization method. Composite resins are dimethacrylate monomers and polymerize by an addition mechanism initiated by free radicals that are produced by chemical activation or external energy (heat or light). • Chemically-activated (self-cured) composites: a two-paste system (benzoyl peroxide initiator & tertiary amine activator). • Light-activated composites: visible light replaced UV light. A one-paste system that contains a photoinitiator molecule (camphor quinone) and amine activator. The most common classification method for composite resins is based on filler content, filler particle size, and method of filler addition. Almost all important properties of composite resins are improved by using higher filler levels. However, as the filler level is increased, fluidity decreases.

FILLED COMPOSITE RESINS-replaced unfilled acrylic resins. A filled resin has inorganic inert filler (silica or quartz) added to its resin matrix. Filled resins are harder, stronger, more resistant to abrasion, with a lower coefficient of thermal expansion than unfilled resins. • The first materials used as esthetic materials were based on silicate cements. Due to solubility problems, silicate cements were replaced by unfilled acrylic resins which contracted excessively during polymerization creating subsequent marginal leakage, and were not strong enough to support occlusal loads. • Filled resins are harder, with higher compressive and tensile strengths than unfilled resins. Filled Resin Particles Composition: colloidal silica, crystalline silica (quartz), or silicate glasses (noncrystalline). Ions are now added to the filler to produce desirable physical changes. Lithium and aluminum ions make the glass easier to crush to produce small particles. Barium, zinc, boron, zirconium, and yttrium ions produce radiopacity in the filler particle. Macrofillers/traditional composite (10-100 microns diameter), Midfillers (1-10 microns diameter), Minifillers (0.1-1 micron diameter), and Microfillers/fine particles (0.01-0.1 micron diameter) that develop the smoothest finish.

Small size filler particles in composite resins results in better finishing and greater resistance to occlusal wear. Highly filled resins contain larger filler particles, but this composition results in a rough finished surface. Smaller filler particles are used to produce a resin with a relatively smooth finished surface. The first composite resins developed contained large filler particles (10-100 microns diameter) = macrofill material. However, larger size filler particles reduce surface smoothness and resistance to wear. Thus, manufacturers continue to produce smaller and smaller particles that yield better finishing characteristics and greater wear resistance. Newer microfilled and hybrid resins are 40-60% filler by weight, compared to the 70-80% filler of macrofill materials. Hybrid Resins-contain a mixture of particles (usually midfill or minifill with microfill) with different diameters that allow higher filler levels while permitting good finishing. The principal particle size is in the 0.1-1 micron range (minifiller). Hybrid and microfill resins use colloidal silica fillers to increase hardness and wear resistance of the base resin material while maintaining high

polishability and esthetics. New resins with nanofillers that range from .0050.01 microns are being developed. These particles are so small that very high filler levels are achieved while maintaining workable consistencies. Composite Filler Particle Functions: Decrease coefficient of thermal expansion & polymerization shrinkage. Increase tensile and compressive strengths, hardness, and improve wear resistance. As the overall filler content increases, the physical, chemical, and mechanical properties generally improve. However, there is a limit as to the amount of filler that can be added to a resin because as the filler level increases, fluidity decreases. *Restorative composites have higher filler content, while flowable composites have a low filler content. MATRIX: difunctional monomers (BIS-GMA) or recently UDM (urethane dimethacrylate). Both of these monomers are diluted with another difunctional monomer (TEGDMA) to reduce viscosity. • Once proper finishing is complete, a thin layer of unfilled resin can be applied as a glaze to seal the margins and smooth the surface. Difficulty in

finishing composite resin restorations is mainly due to the softness of the resin matrix and hardness of the filler particles. The most desirable finished surface for composites is obtained with aluminum oxide disks. COUPLING AGENT (SILANE): synthetic hybrid inorganicorganic compounds that promote ADHESION between the inert filler and organic matrix (ex: silica-based materials like porcelain). Also used for ceramic repairs. Not used for zirconia, metals, alloys. The normal wear mechanism of composite resins occurs by abrasion of the matrix which then exposes the filler particles causing subsequent dislodgement of the filler particles. The most popular way to polymerize matrix monomers is VISIBLE LIGHT-CURED (VLC unit). • Visible light-curing involves light energy in the range of 410-500nm with a peak intensity of 470nm. The curing light is used at wavelengths 400-500nm for adequate composite curing. • Hold the light as close to the resin as possible (within 2mm to be effective). • Place a shield between the light tip and operator’s eyes. Patients who have had recent cataract removal should also have eye protection. Studies show visible

light used in polymerization of photo-activated materials can cause RETINAL DAMAGE, so always use a shield and eyeglasses for protection. Light from visible-light polymerization units can cause retinal damage (VLC units are most hazardous to the retina). • For deep restorations, cure the composite in increments to ensure that deeper areas are cured. • The most serious limitation of visible light-cured posterior composite restorations is polymerization shrinkage which can cause internal stresses and gap formations at butt-joint interfaces (seen at the gingival floor of Class II & V restorations). • Ensure the bulb in the light is still powerful (commercially available products test the curing bulb). • With darker resin shades, may need to cure a little longer. • For large restorations (those wider than the light tip’s diameter), cure each area for the full required time. Do not back off the light tip until it lights up the entire restoration surface. • Advantages of visible light curing systems compared to old ultraviolet light curing systems: * Greater depth of resin can be cured by visible light.

* Resin can be polymerized through enamel (especially advantageous in Class III restorations). * Intensity of visible lights remains relatively constant until the bulb fails completely. Visible light curing systems have totally displaced the UV light systems, and are now much more widely used than the chemically activated systems (self-cured). An advantage of light curing systems as a whole is the dentist has complete control over the working time and is not confined to the built-in curing cycle of the self-cure (this is especially beneficial when large restorations are placed). To deal with problems of incomplete curing with VLC due to the thickness of restorations and filler particles scattering light, manufacturers have developed composite resins that are dual-cured (combines self-curing and visible lightcuring). Another polymerization method is “staged curing” (a two-staged cure). However, VLC composites remain the most popular today. Visible light cured composites are single component pastes, and the polymerization process is activated by an external energy source. The alpha-diketone initiator (camphor quinone) absorbs energy from a visible (470nm blue light) light source. The ketone absorbs energy and reacts with an amine (added to the system to enhance the effect of the light-sensitive catalyst) to produce free radicals.

When using a light to cure light-activated composite resin the light should be held as close to the resin surface as possible, a shield should be placed between the light tip and operator’s eyes, and the curing time is increased with darker shade resins. D-L of canines is a Class III lesion that should NOT be filled with composite resin. Composite resin is not recommended for Class III lesions on the D-L aspect of canines (use amalgam or direct gold). Composite material will not maintain the M-D dimension of the tooth (this may not be entirely true today since there are much better wearresistant resins). A lingual approach is made when preparing a Class III dental amalgam preparation for the distal of a canine to preserve the esthetic value of the facial surface. When two adjacent teeth to each other have Class III lesions, the dentist should prepare the larger lesion first and fill the smaller lesion first. Access to the preparations and shade matching are easier when you do both at the same time. In a Class III composite preparation, retention points should be placed entirely in dentin. The retentive grooves are placed along the gingivo-axial & inciso-axial line angles (entirely in dentin) to provide mechanical lock in the preparation. Small, rounded retentive areas are preferred (not sharp angles) since it is difficult to insert

viscous composite material into the sharp angles. When placing composite material in a Class III preparation, the wooden wedge is placed to provide some separation of teeth for contact, stabilize the mylar strip, and avoid creating excess gingival flash. Restoring the contact area must be done properly and diligently. A diagnostic aid used as a last resort to confirm caries on the proximal surface of an anterior tooth is mechanical separation with a wedge. After the dentist completed acid etching a Class III composite preparation, the preparation becomes contaminated with saliva. In response, the dentist should rinse away the saliva with water, dry the preparation with air, then repeat the etching procedure. For Class III preparations using resins, the rule of “extension for prevention” into embrasures is DISREGARDED for Class III esthetic restorations. This compromise is for esthetic reasons and the unnecessary removal of tooth structure that will often involve the incisal edge. If possible, the outline form should place the gingival margin incisally from the crest of the gingiva. The outline form of a Class V composite preparation resembles a Class V amalgam preparation except that the internal line angles are much more ROUNDED. When restoring with composite resin, it is much easier to

compress the material into rounded line angles. • The outline form of a Class V restoration is not always uniform and varies depending on the location and amount of caries or decalcification (the size and location of the carious lesion determines the outline form of the cavity preparation). When carious tissue is removed and the margins are on sound enamel or dentin, the outline is usually rectangular with the corners round, ovoid, or kidneyshaped, very much resembling an amalgam Class V preparation except that the internal line angles are much more rounded. • The cavosurface margin for a composite restoration is chamfered when placed on enamel (a major difference between composite & amalgam preparations). This chamfer is etched and provides retention for the restorative material, improves marginal seal, and maintains the resin’s strength with sufficient bulk. Retentive grooves supplement the etched enamel retention (these grooves are placed in both incisal and gingival axial line angles). Whenever possible, use a composite syringe to place composite resin in the restoration to minimize trapping air in the final restoration. The most important factor in preparing and restoring a Class II lesion with composite resin is moisture

control. Resin cements are the cement of choice for composite inlays. With a composite inlay preparation there should be divergent internal walls and all margins should be finished in sound enamel. Beveling the enamel margin of a composite resin preparation is accomplished to improve esthetics and expose the ends of enamel rods for acid attack. Compared with self-cured resins, light cured composite resins provide denser restorations because no mixing is required that introduces air bubble porosity. Composite resin is the material most likely to cause an adverse pulpal reaction when placed directly in a deep cavity preparation. The cavosurface angle of a cavity preparation for an acid-etched composite is beveled to afford more surface area for etching to enhance seal and retention. Retention in a cavity prepared for composite resin usually is gained by undercuts in dentin and acid etching.

The normal wear mechanism of the composite resins is best explained by abrasion of matrix, exposure of filler, and dislodgement of filler particles. The use of composite resin for Class II restorations is not generally recommended because its clinical performance is inferior to amalgam with respect to occlusal wear. Whenever possible, a syringe should be used for placing composite resin because the possibility of trapping air in a restoration is minimized. In preparing a cavity for restoration with composite resin combined with an acid etch technique, all enamel cavosurface angles should be obtuse angles. Difficulty in finishing composite resin restorations is due primarily to the softness of the resin matrix and hardness of the filler particles. Composite resins are not generally recommended for restoration of Class II cavities because of excessive occlusal wear. A patient presents with numerous carious lesions. To restore these lesions, the dentist will place composite resins and use a visible-light curing unit. The patient’s history of cataract removal will necessitate that the

dentist take extra precautions when using this unit.

AMALGAM OUTLINE FORM-the part of a cavity preparation that is the shape or form of the cavity on the surface of the tooth. Important: margins are placed in areas of lessened caries susceptibility (extension for prevention which can be restricted in patients with very low caries susceptibility). Also, all undermined enamel (enamel not supported by sound dentin) should always be removed. These important points are influenced by: lateral spread of decay at the DEJ, type of restorative material to be used, and the tooth and its relative position in the arch. When establishing “ideal outline form” caries remains on any of the preparation walls, the next step is to extend the outline form before excavating any caries. Convenience Form-the form the cavity preparation takes to aid the operator in preparing, placing, or finishing the restoration. RETENTION FORM-that form the cavity preparation takes to resist dislodgement or displacement of the restoration Ex: buccal and lingual walls of a Class II amalgam preparation converge occlusally to prevent amalgam dislodgement.

• For Class II restorations, this resistance to dislodgement is provided by the occlusal dovetail and retention grooves in the proximo-axial line angles. Retention grooves are placed in axiobuccal and axiolingual line angles, and extend to the height of the axial wall (resistance to dislodgement). Resistance to proximal displacement in the ideal Class II restoration is provided by occlusal dovetail and retention grooves in proximoaxial line angles. RESISTANCE FORM-that form the cavity walls take to resist forces of mastication to prevent fracture of the restoration and tooth. • Ex: severely undermined cusps should be reduced to prevent fracture. The axiopulpal line angle in a Class II amalgam preparation is rounded or beveled to reduce concentration of stresses which prevents fracture of brittle amalgam. • Ex: proper angulation of cavity walls (converging) and pulpal and gingival walls that are prepared perpendicular to occlusal forces (flat walls at right angles to the tooth’s long axis) help achieve resistance form. • Most common cause of fracture at the isthmus of

a Class II amalgam restoration is inadequate depth at the isthmus area (must be adequate depth to obtain resistance form). Most detrimental to the strength of a posterior tooth in a cavity preparation is an increase in F-L width. Axiopulpal line angle is rounded or beveled to reduce concentration of stresses (resistance form). Fracture of a Class II amalgam restoration at the junction between the occlusal and proximal portions is the result of inadequate resistance form. TRITURATION: increasing trituration time, reduces the setting expansion of amalgam (the longer the trituration time, the smaller the setting expansion). If the trituration is carried to the extent that the amalgam is shiny and wet, the strength will be maximal and the smooth, carved surface will retain its luster long after polishing. A properly triturated amalgam is shiny, wet, smooth, and homogenous. An over-triturated amalgam mix is better than an undertriturated mix. • Inadequate trituration creates a low strength amalgam mix and rough surface that accelerates corrosion. An undertriturated amalgam mixture appears dry and grainy. Condensing this mixture results in poor adaptation to the preparation walls, lacrimation between condensed increments, and reduced strength.

• The discolored, corroded, superficial layer often seen on the surface of an amalgam restoration is most likely a sulfide. • When carving an amalgam restoration, trim the margins with a sharp instrument that rests on tooth structure to prevent “ditching” the margins. If an amalgam chips during carving, it is because the amalgam was condensed AFTER its working time had elapsed. The principal purpose of trituration is to coat the alloy particles with mercury. The object of trituration is to bring about an amalgamation of the mercury and alloy. Each individual alloy particle is coated with a slight oxide film that prevents mercury penetration. During trituration, this oxide film is rubbed off and the clean metal is then readily attacked by the mercury. ► Once amalgamation occurs, no free (unreacted) mercury is associated with the amalgam restoration (it now has no toxic properties). However, if the amalgam is heated above 80°C, liquid mercury can form on the amalgam surface, and its vapors can present a health hazard. PINS: function to retain the restorative material (not used much today). Pins do not increase the strength of the restorative material. Pin retention increases as the pin diameter increases. The largest pin that can safely be

placed should be selected in any situation. As a rule, one pin per missing axial line angle is used. Optimal pin placement is at the line angles (corners) of the tooth where the tooth-to-root mass is greatest and the risks of perforation into the pulp or furcation are minimal. PIN INDICATIONS: Class II amalgam preparation where one or more cusps have been lost, or where the outline form otherwise far exceeds that which is considered normal. A very large Class III amalgam preparation. Class V amalgam preparation that far exceeds minimal dimensions (especially in the gingivo-incisal area). A preparation for an amalgam buildup over which a cast restoration will be fabricated. The main advantage of PINS is to IMPROVE RETENTION of large restorations. Pin Disadvantages: placement can cause pulpal exposure, perforation, or tooth fracture. When placing pins in endodontically treated teeth, use only self-threaded pins or cemented pins (not friction-locked pins). Pins can

weaken the restorative material into which they are placed. If placed by force, pins can create stresses that cause crazing of tooth structure. Pins may provide an additional deep path for microleakage. If placed in close proximity to the pulp, they may aggravate an existing pulp problem or create one. ► Pins may be contraindicated in young teeth with very large pulps and teeth with reversible pulp pathology, which might be aggravated by instrumentation. Placement is always influenced by the limitations of access and vision. ► If when attempting to drill a pin hole the drill enters a vital pulp chamber, the proper treatment is to allow the bleeding to stop, dry with a sterile paper point, and place calcium hydroxide into the hole. Proceed with a better pin hole location. If a pin channel perforates the tooth’s external surface and all factors are favorable, a pin can be placed if there is no extension beyond the tooth surface. When preparing a pin hole for a pin-retained amalgam restoration, the spiral drill tip enters a vital pulp chamber. The next step is to apply calcium hydroxide in the pin channel and proceed. ► Pins are inserted into DENTIN ONLY and placed 1.0-1.5mm inside the cavosurface margin and at least 0.5mm inside the dentinoenamel junction (DEJ),

if present. Placing the pin channel at least 0.5mm away from the DEJ helps prevent crazing or complete fracture of remaining enamel. The optimal pinhole depth into dentin is 2mm. ► Pins should be 2mm into dentin, 2mm within amalgam, and 1mm from the DEJ (to be safe) with no bends in the pins. ► Cusps restored with dental amalgam should be reduced 2mm while forming a flat surface perpendicular to occlusal forces. ► The twist drill used to prepare the pin channels must be angled so it remains in dentin only. The channel should be prepared parallel to the external surface of the tooth. ► After restoring a tooth, check occlusion very carefully. If a restoration is left in hyperocclusion, the patient will return complaining of discomfort when biting, usually with no other symptoms. ► The most frequently used pins are SELFTHREADED PIN SYSTEMS (TMS, Whaledent) which use holes sized just under the screw diameter. The elasticity (resiliency) of the dentin function to retain the screwed pin. This system comes with a selflimiting drill of optimal 2mm depth and self-shearing

pins that guard against overtightening. This type of pin system is the most retentive of the 3 types of pins. TMS system has 4 pin sizes (regular, minim, minikin, & minuta) which are available in titanium or stainless steel plated with gold. Threaded pins used in a dental amalgam restoration should be placed ~2mm in depth at a position axial to the DEJ and parallel to the external surface between the pulp and tooth surface. ► Cemented pins-serrated stainless steel pins cemented into pinholes that are larger than the pin diameter. ► Friction-locked pins-are tapped into pinholes that are smaller than the pin diameter. They are retained by dentin’s elasticity. ► New systems are now available for bonding amalgam to dentin and enamel that may make the use of pins obsolete. Ex: Amalgambond (Parkell), All Bond 2 (Bisco), & Dentastic (Pulpdent). These systems allow adhesion to pre-conditioned substrate with the added benefits of retention and sealing of the restoration, and a stronger total cohesive mass to support all remaining cuspal segments of the tooth. Inclusion of pins in an amalgam restoration results in

an increase in retention only. Class I Amalgam Preparation: Preparation’s MESIAL & DISTAL walls must DIVERGE slightly towards to the occlusal surface to provide support for mesial and distal marginal ridges. This divergence also applies to Class I preparations involving direct filling gold and gold inlays. Class II Amalgam Preparation Features: Class II amalgam preparations have independent retention and resistance form for the proximal box and occlusal portion of the preparation. Occlusal dovetail provides resistance to proximal displacement. Pulpal floor is flat (same as with a Class I preparation). Lingually, it is often necessary to create a reverse curve in the outline (a reverse “s” curve is a curve put into the buccal or lingual wall so the wall meets the external tooth surface at a 90° angle. All walls should meet the tooth surface at a 90° angle (butt joint). A conservative Class II amalgam preparation has a proximal cavosurface margin that forms a 90° angle with the external surface. Buccal and lingual walls of the proximal section should

converge occlusally (the extension of these walls is determined primarily by the position of the adjacent teeth in relation to the tooth being restored). Buccal, lingual, and gingival walls are extended into embrasures enough to allow easy cleaning (to decrease caries susceptibility). Mandibular 1st premolar (bicuspid) requires special attention when preparing the occlusal aspect for a restoration. Bur is tilted lingually to prevent encroachment on the facial pulp horn and to maintain dentinal support of the lingual cusp. The pulpal floor should be parallel to the occlusal plane F-L and should slope to coincide with the slope (height) of the cusps. The most sensitive area of the tooth during cavity preparation is the DEJ. In a Class II cavity prepared for dental amalgam, the facial and lingual proximal walls should be formed slightly diverging as the wall approach the proximal surface. A conservative Class II preparation for dental amalgam should have independent retention and resistance form for both the proximal and occlusal portions.

Gingival cavosurface margin of the proximal box and occlusal cavosurface margin are the only ones beveled when preparing a Class II amalgam preparation. • The gingival cavosurface margin is beveled only if it is placed in enamel. It should be beveled to remove any unsupported enamel, and is usually placed with a gingival margin trimmer (beveling is not necessary if the gingival margin is within cementum). This gingival margin must be below any existing contact with the adjacent tooth to allow proper finishing of the gingival margin. • The bevel is no steeper than necessary to ensure full-length enamel rods forming the gingival margin, and is no wider than the enamel. *Enamel rods in the gingival third of primary teeth extend occlusally from the DEJ which eliminates the need in Class II preparations for the gingival bevel that is required in permanent teeth. • Primary molar teeth have marked cervical

constriction. Thus, when preparing the proximal portion of a Class II cavity preparation, a satisfactory gingival seat may be difficult to obtain if the preparation extends too deeply gingivally. • The position of the gingival margin of a Class II amalgam restoration is dictated primarily by the extent of the carious lesion. The gingival margin should clear the contact area to allow for adequate finishing of the enamel margins and matrix band placement. • Instruments used for placing gingival bevels on inlay preparations: margin trimmers, carbide finishing burs, and fine, tapered diamond stones. • Gingival cavosurface margin of a Class II inlay in a patient with gingival recession should ideally terminate gingival to the contact area and gingival to the lesion. • A deficient margin at a proximogingival cavosurface angle of a freshly packed Class II amalgam restoration may have been caused by poor condensation of the amalgam, using too large an initial increment of amalgam, or debris in the corner of the proximal box. MATRIX BAND: is removed from the tooth prior to final

carving of the restoration. The reason for placing the matrix for a Class II amalgam restoration to protrude above the cavity preparation is to allow for overfilling to enhance adequate cavosurface coverage. • Contact areas are always carefully restored in all restorations to protect the gingival tissue. The matrix band is burnished into contact with adjacent teeth to help ensure contact. • One of the most difficult teeth to adapt the matrix band to is the mesial of a maxillary 1st premolar due to its developmental depression/groove. • The larger circumference of the matrix band is always placed toward the tooth’s occlusal surface. This accommodates the larger tooth circumference at the contact level. Also, the diagonal slot opening on the Tofflemire matrix retainer (“Universal Matrix System”), is always placed facing the gingiva to permit easy separation of the retainer from the band in an occlusal direction. • Wedges are inserted from facial or lingual embrasure (whichever is larger), slightly gingival to the gingival margin. The wedging action between the teeth should provide enough separation to compensate for the thickness of the matrix band.

The thickness of a good Class II matrix should be approximately 0.002 inches. Matrix Function for Placing Class II Amalgam Restorations: establish proximal contour, limit moisture contamination during condensation, and aid in preventing gingival overextension of the amalgam. ► Primary function of the matrix is to RESTORE anatomical contours and contact areas. ► Other Matrix Functions: provide a rigid wall to condense filling material against, prevent excess filling material from going subgingivally, and to limit moisture contamination during condensation. A properly placed wedge also protects the gingival tissue and helps reduce moisture leakage into the cavity preparation. ► Before inserting amalgam into an MOD cavity preparation, a matrix is placed around the tooth. The procedures that should be performed next are the band should be burnished into contact with adjacent teeth and tapered wedges should be placed interproximally to obtain close adaptation of the matrix band at gingival margins. ► A matrix for a Class II dental amalgam restoration is extended occlusally to the cavity preparation to allow

for overfilling the amalgam.

Outline Form of a Class V Amalgam Preparation is a deformed trapezoid (“kidney-shaped”). This outline form is determined mainly by the location and size of the carious lesion/area. • The Class V amalgam restoration is used to restore lesions from caries, erosion, and abrasion. • The non-parallel mesial & distal walls of the preparation are also straight and parallel, but never extend beyond the transitional line angles. The direction of these walls is determined by the direction of the enamel walls (as is the decay pattern). • Mesial, distal, gingival, and incisal walls of the cavity preparation DIVERGE outward. • Retention form is provided by the gingival retention groove along the gingivoaxial line angle and an incisal retention groove along the

incisoaxial line angle. • A cervical clamp is usually necessary to RETRACT gingival tissues. • Occlusal and gingival walls of the preparation should be gently curved arcs as determined by the contour of the free margin of the gingival tissue. These arcs should be as parallel to each other as possible. The occlusal arc will normally be the longer of the two arcs. The gingival margin will normally be at or slightly below the margin of the free gingiva. • For incipient lesions, the axial wall should be uniformly deep into dentin. • Be careful not to “ditch” cementum when finishing and polishing. Occasionally, you will notice the gingival tissues have receded apically from the gingival margin that was previously polished due to irreversible tissue changes caused by inadvertently traumatizing the tissue when the restoration was being polished, so be careful. • In a Class V amalgam preparation for an incipient lesion, the ideal internal form of the preparation has the axial wall uniformly deep into dentin. DELAYED EXPANSION of amalgam restorations is

associated with insufficient trituration & condensation, and amalgam contamination by moisture during trituration and condensation (MAIN cause of failures). • If moisture is incorporated into an alloy that contains zinc, the water reacts with the zinc to produce hydrogen gas. The resulting pressure from the liberated gas produces severe expansion of the amalgam, causing amalgam to protrude from the cavity preparation, post-operative pain, and excessive corrosion. • Amalgam’s compressive strength is greatly reduced when contaminated with moisture. • Amalgam has a coefficient of thermal expansion ~2x that of tooth structure, thus percolation occurs during temperature changes. • Amalgam’s tensile strength is 1/5-1/8 of its compressive strength (this is why enamel is needed to support amalgam at the restoration margins). Amalgam is more abrasion-resistant than composite resin or unfilled resin. High mercury content is manifested in the clinical amalgam restoration by SEVERE MARGINAL BREAKDOWN. The most important consideration in amalgam’s strength is its mercury content. If the mercury content exceeds 55%, a dramatic loss in strength occurs.

Amalgam restoration that contains mercury levels of ~55% exhibits a high incidence of marginal breakdown, fracture, corrosion, and the surface finish of the restoration is not good. • Factors that influence the final mercury content of a restoration: original mercury-alloy ratio, amount of trituration, and condensation pressure and time involved in carrying out condensation. • Removing the mercury-rich matrix by proper condensation and carving produces a stronger and more corrosion-resistant amalgam because it minimizes formation of the matrix phases of amalgam (the least desirable parts of the set material). • Strongest phase of set amalgam is the GAMMA phase (original silver-tin particles→unreacted powder). Gamm-two is the weakest phase and the most susceptible to corrosion in the mouth. • The reaction that occurs between the alloy particles and mercury is: Silver-tin alloy (Ag3Sn) + Mercury (Hg)→Silver-tin alloy (Ag3Sn) + Silver-mercury (Ag2Hg3) + Tinmercury (Sn3Hg) • Gamma (Strongest Phase) > Gamma > Gamma 1 > Gamma 2 (Weakest Phase)

AMALGAM FACTS: * Smaller particle size results in higher strength, lower flow, and better carvability. * Spherical amalgams high in copper have the best tensile and compressive strengths. * Amalgam has a coefficient of thermal expansion 2x that of natural tooth structure. * Amalgam’s tensile strength is 1/8 of its compressive strength. * Weakest phase of the set amalgam is Gamma-2. DENTAL AMALGAM COMPOSITION: 1. Silver (Ag): 40-70%. Decreases setting time, increases setting expansion and strength. 2. Tin (Sn): 25-27%. Increases setting time, and decreases setting expansion (tin influences amalgam opposite of silver). Tin (Sn)-the component in amalgam that causes contraction. 3. Copper (Cu): < 6%. Increases hardness, strength, and resistance of amalgam to corrosion. Low Copper alloys have 6% or less (this is traditional alloy). Comminuted (irregular, filing, or lathe-cut) or spherical particles.

4. Zinc (Zn): < 1%. Prevents oxidation of the other metals while the alloy is being prepared by the manufacturer. Moisture contaminated zinc-containing amalgam manifests grossly delayed expansion & reduced compressive strength. 5. Mercury (Hg): 3% maximum. Reacts with the other alloy particles to produce different phases. • Most amalgam restorations show slight setting expansion, but not of clinical significance. The more free mercury, the more setting expansion (and vice versa). • Amalgam’s Strength: amalgam is BRITTLE, but possesses good compressive strength. The most important consideration in amalgam’s strength is its mercury content. Mercury content > 55% greatly decreased strength (should be within 45-53% by weight). ► Higher condensation pressure, smaller particle size, longer trituration time, and fewer voids all increase amalgam’s strength. Brittle describes a material with high compressive strength, but low tensile strength. Brittleness is the property of amalgam that makes it undesirable to bevel occlusal margins of an amalgam cavity preparation.

The amount of MERCURY remaining in dental amalgam after condensation directly affects: restoration porosity, compressive strength, corrosive resistance, and surface finish of the restoration. • The amount of mercury remaining in a set amalgam restoration is related to how much of the mercury-rich matrix is left in the amalgam after condensation. The key is to minimize the amount of mercury-rich matrix that forms during the reaction. By condensing the amalgam mixture in the cavity preparation, the mercury-rich matrix rises to the surface and is removed by subsequent condensing and carving. The final amalgam restoration will be composed of mostly residual alloy and very little of the mercury-rich matrix. • Mercury is used to initiate the reaction with the alloy. Although an amalgam restoration is non-toxic, mercury is poisonous. Free mercury, in the form of vapor or liquid droplets, is a significant health hazard in the dental office. The greatest potential hazard of chronic mercury toxicity comes from inhaling mercury vapor. The vaporization is most likely to occur during amalgam condensation (always use high speed suction). Mercury hypersensitivity (allergy) is very rare (1 in 100 million). • The area of the condenser point and force exerted on it by the operator determine the condensation

pressure. The smaller the condenser point, the greater pressure exerted on the amalgam. By doubling the diameter of the condenser and doubling the hand pressure applied to the instrument, the operator actually exerts less condensation pressure. Mercury vaporization in an amalgam restorative procedure utilizing well-sealed pre-measured capsules is most likely to occur during condensation. High Copper Amalgam Alloys: newer alloys that contain 10-30% copper in their composition. The main purpose of adding copper to an amalgam alloy is to decrease the gamma-2 phase (tin-mercury). The copper reacts with tin to prevent the formation of gamma-2 phase. High copper alloys have less marginal breakdown and are less likely to corrode. • Spherical: sets faster and attains final mechanical properties more rapidly. • Comminuted: can have zinc or be zinc-free, and can be fine cut or microcut. • Combination (Admix): a mixture of spherical and comminuted particles. ► Dispersed phase alloy: the original admix alloy (a mixture of comminuted traditional silver alloy and spherical particles of silvercopper eutectic alloy (made of elements that

when solid are insoluble and form a layered structure). Most commonly used alloy today. Copper contents over 6% (“high-copper” alloys) eliminate gamma-2 phase by forming a copper-tin phase with superior properties. High copper dental amalgams are superior to other amalgams because high copper dental amalgams are less likely to corrode and have less marginal breakdown. High copper content of amalgams are superior to conventional amalgams because high copper dental amalgams are less likely to corrode and have less marginal breakdown. Amalgam Setting Expansion: • Increasing amalgam’s trituration time increases compressive strength and decreases setting expansion. • Increased condensation pressure increases compressive strength, and decreases setting expansion. • A decrease in particles size increases compressive strength and decreases setting expansion.

Creep (time-dependent deformation or strain relaxation)-deformation with time in response to a constant stress. Creep is one of the main causes for margin fractures on amalgam restorations. Creep is timedependent. ► High copper and low mercury content of an amalgam restoration DECREASE CREEP. Increasing condensation pressure also decreases creep rate AND the restoration’s final mercury content. ► Altering the trituration time and condensation pressure can change the creep rate of an amalgam restoration. Undertrituration and overtrituration increase the creep rate. If a delay between trituration and condensation occurs, creep rate increases. ► Higher the creep, the greater marginal deterioration. ► Creep of a metal indicates that the metal will deform under a static load. ► Creep is a property of dental materials that is timedependent. Marginal leakage of an amalgam restoration DECREASES as the restoration ages. Corrosion products

are helpful to reduce marginal leakage around amalgam restorations. These corrosion products (tin oxide & tin sulfide) accumulate in the gap between the restoration and tooth to provide an excellent seal. The proper amount of time to wait until an amalgam restoration can be finished and polished is 24-48 hours. The final finish of the amalgam restoration should not be done until after the amalgam is fully set. It should be delayed for at least 24hrs after condensation and preferably longer. By waiting, you ensure the reactions between the alloy and mercury is complete, creating a more corrosionresistant surface. • Amalgam restorations should be finished and polished to reduce marginal discrepancies, create a more hygienic restoration, reduce marginal breakdown which reduces the chance of recurrent decay, to prevent tarnishing, and improve the restoration’s appearance. • Heat generation during polishing must be avoided. Using dry polishing powders and discs can easily raise the surface temperature above 60°C (140°F) danger point. Thus, a wet abrasive powder in a paste form is the agent of choice. Heat damages the pulp and draws mercury to the restoration surface to create an inferior restoration. • When checking occlusion on a newly condensed

amalgam restoration, the marks left by the articulating paper should be the same intensity as other markings in the same quadrant. Amalgam’s thermal insulation properties explains why cold sensitivity is the most common problem encountered after placing a dental amalgam restoration. Amalgam is a POOR THERMAL INSULATOR (this is why a base of calcium hydroxide or ZOE is placed under most amalgam restorations to provide thermal protection).

In preparing a Class I cavity for dental amalgam, the dentist will diverge the mesial & distal walls toward the occlusal surface to prevent undermining of the marginal ridges. The margins of a dental amalgam restoration are trimmed by carving along the margins with a sharp

instrument that rests on the tooth surface. A patient presents with an amalgam restoration fractured at the isthmus six months after placement. The most likely cause is inadequate preparation depth. Proper proximal contour is given to an amalgam restoration placed in a Class II cavity preparation by carving the restoration and adapting a contoured matrix (not by the matrix retainer, or overfilling the cavity preparation with amalgam, or heavy condensation of the amalgam). Restoring a cusp with dental amalgam requires at least 2mm of the cusp be removed to provide resistance form. An MO amalgam restoration is more resistant to fracture if the axiopulpal line angle is beveled or rounded. A deficient margin at a proximogingival cavosurface angle of a freshly condensed Class II amalgam restoration may have been caused by poor condensation of the amalgam, use of too large an initial increment of amalgam, or debris in the corner of the proximal box (not from neglecting to wedge the matrix band).

Direction of mesial & distal walls of a Class V amalgam cavity preparation is determined by the direction of enamel rods. Use of water spray and high volume evacuation is recommended when removing old amalgam or polishing dental amalgam restorations because mercury vapor is released during these procedures. When attempting to place a self-threaded pin to retain an amalgam restoration, the spiral drill enters a vital pulp chamber. The next step should be to cover the pin channel with calcium hydroxide and proceed. When comparing the pulpal depth of a standard Class II inlay cavity preparation with a Class II dental amalgam cavity preparation, the pulpal wall should be the same depth for both restorations. Cusps to be restored with dental amalgam (MOD) should be reduced by 2mm while forming a flat surface (perpendicular to the occlusal forces). Self-threading pins offer the greatest degree of retention into the dentin. Threaded pins are used in large dental amalgam restorations to provide retention form. Threaded pins used to retain amalgam should NOT be

parallel to each other only and should not be parallel to each other and parallel to the long axis of the crown. When pins are included in an amalgam cavity preparation, the strength of amalgam is decreased. If the proximal box is too wide to allow facioaxial & linguoaxial grooves to oppose one another, pins or slots on the gingival floor should be used. When pins are included in an amalgam cavity preparation, the strength of the amalgam is decreased. A patient has sensitivity in a mandibular premolar. A well-condensed Class V amalgam restoration was placed five months prior with no discomfort for the first four months. Since then, it has become painful. The problem probably relates to irreversible pulpal damage. A newly condensed amalgam restoration seems to chip away when being carved. The likely cause of this problem is the amalgam was condensed after its working time elapsed. RUBBER DAM FUNCTIONS: the best method to use when attempting to isolate an operating field in the oral

cavity. STANDARD OF CARE DURING ROOT CANAL THERAPY. 1. Retracts soft tissue (lips, cheeks, tongue) and provides a clean, dry field. “Woodbury” rubber dam frame provides more retraction of the soft tissues. However, the Young’s frame is the most popular (Ushaped metal frame). 2. Protects the patient by eliminating the possibility of swallowing debris or instruments. It protects the dentist by isolating him/her from possible infectious conditions in the patient’s mouth. 3. Provides for maximum physical properties of materials (i.e. it provides a dry field which is essential for placing amalgam and cements. *Cements placed under dry conditions have maximum strengths. A dry field prevents delayed expansion of amalgam. 2. Saves time because the operator can work more efficiently in a clean, dry field where visibility is not impaired. 3. The use of a rubber dam is the STANDARD OF CARE when performing ENDODONTICS. Rubber Dam Advantages: dry clean field, access and visibility, improved properties of dental materials, protection

for the patient and operator, and operating efficiency. Rubber Dam Disadvantages: time consumption & patient objection. However, if you become proficient in placing the rubber dame and explain to patients the advantages of using it, you can eliminate these disadvantages. Conditions that preclude the use of Rubber Dam: severely tilted teeth, some 3rd molars, partially erupted teeth, some respiratory problems (asthma or severe colds) where breathing through the nose is difficult. Important Points About the Rubber Dam: Apply lubricant to the lips and corners of the patient’s mouth. Plot the hole on the rubber dam. Always isolate at least 3 teeth. Punch the appropriate size hole for a particular tooth. For a tooth bearing a clamp, the hole should be one size larger than the holes over teeth without a clamp. An appropriate clamp is selected that will fit the most distal tooth to be isolated. The dam can either be stretched over the clamp with the clamp in place on the tooth or may be carried with the dam and placed on the tooth in one step.

Once the dam is placed, it is secured with either a Woodbury or Young’s holder (frame). The rubber dam is INVERTED into the gingival sulcus with floss and/or a blast of air and a plastic instrument to prevent seepage of saliva. When removing the rubber dam, the removal process is the reverse of placing the dam, except all ligatures (interdental septum of the dam) must be CUT and removed before the dam is removed. For a rubber dam clamp to be stable, all four points of the clamp jaws must contact the tooth gingival to the height of contour. They should not extend beyond the line angles to prevent impingement of interdental papilla and possible interference with wedge placement. In order for a rubber dam clamp to be stable on the anchor tooth, all four points of the clamp jaws must contact the anchor tooth and the clamp must contact the anchor tooth gingival to the tooth’s height of contour. Wrinkling the rubber dam between the teeth isolated means the holes were punched too far apart. Punching holes too close together in the rubber dam may cause damage to the gingival papilla. A rubber dam is essential to prevent contamination of the gold with saliva. A cervical clamp is usually necessary

to retract the gingiva (#212 ivory clamp). The hole to be punched in the rubber dam for the tooth being restored should be facial to the normal alignment with the adjacent teeth. The application of a rubber dam for a Class V facial preparation to include #212 clamp on a mandibular 2nd premolar requires the hole for the 2nd premolar punched larger than usual and slightly to the facial of the other holes in the arch. When punching holes in a rubber dam for a facial Class V cavity, the hole for the tooth in which the restoration is to be placed should be located facial to the normal tooth alignment. Jaws of a rubber dam retainer should NOT extend beyond the line angles of the anchor tooth to prevent impingement on the interdental papilla and possible interference with wedge placement. When removing a rubber dam, the first step should be to cut the interseptal rubber with scissors. In addition to maintaining a dry field, the rubber dam functions to retract soft tissue, protect the patient and operator, save time, and improve access. EMBRASURES-triangular-shaped

spaces

located

between the proximal surfaces of adjacent teeth. There are 4 embrasures for each contact area (buccal/facial, lingual, occlusal/incisal, cervical/gingival). In posterior teeth, gingival tissues fill the cervical embrasure. It is normally “col” shaped from a F-L cross section view. • Embrasure Functions: make a spillway for food during mastication, make teeth more self-cleansing, and protect gingival tissue from undue frictional trauma while providing the proper degree of tissue stimulation. • Contact Area-an area where the mesial and distal surfaces of adjacent teeth in the same arch contact. • Height of Contour-the thickest portion or point of greatest circumference of the tooth when viewed from the incisal/occlusal surface. Its functions to form the contact area on the mesial and distal surfaces and protects the gingiva surrounding the tooth. • Proximal contact area functions: to support neighboring teeth (stabilize the dental arch), prevent food particles from entering interproximal areas, protect the periodontium, and form embrasures. • Loss of proximal contact between teeth can cause periodontal disease, malocclusion,

food impaction, or drifting of teeth. Thus, it is important to restore proper proximal contact when restoring teeth to: 1. minimize periodontal pocket formation & food impaction. 2. maintain proper height of interproximal papillae & maintain the tooth’s M-D dimension of the tooth. • Contact Point-a point where teeth of opposing arches meet or touch in occlusion or closure. From the facial view, all premolars have their contacts at the junction of the occlusal and middle third. From this same view, molars have a proximal contact in the middle third. From an occlusal view, all posterior teeth have contacts located slightly buccal to the middle third (mesial & distal). This creates a wide lingual and narrow facial embrasure.

DENTAL BIOMATERIALS ADHESION-the attraction of unlike molecules. Adhesive potential is predicted by measuring the spreading or wetting of the adhesive over a substrate surface. This is done by determining the contact angle of the drop of adhesive as it spreads out. The smaller the angle, the greater wetting & potential for adhesion. 2 types of adhesion: • physical forces (van der Waals forces). • chemical forces (chemisorption). COHESION-the attraction of similar molecules. STRAIN-the actual change in shape or deformation that accompanies any stress. 3 Types of Stresses: 1. Compression-the squeezing of material by external forces. If the wire were a thin metal rod on a firm (hard) surface with a weight on the top, it would spread the weight (stress) across the entire rod. Here the rod’s atoms are being pushed together. 2. Tension-the pulling of a member or part of a member, resulting in an increase in length. If a long metal wire were hung from the ceiling with a weight on one end, the weight would place a force on the wire.

Inside the wire, the force spreads evenly through the whole wire. If enough force or weight were put on the end of the wire, it could cause the wire to stretch. The wire’s atoms are being pulled apart by the weight or force on the wire. 3. Shear-the sliding of one layer of a material relative to another layer of material. Ex: two blocks of wood nailed together. If one block was pushed to the right, and the other block pushed to the left, the nails experience a shear stress at the point where the two blocks of wood meet. If pushed hard enough, the nails could bend or break. TOUGHNESS-the total energy absorbed to the point of fracture (the property of being difficult to break). It is affected by yield strength, percent elongation, and modulus of elasticity. The mechanical property of toughness is affected by yield strength, tensile strength, percent elongation, and modulus of elasticity. BRITTLENESS-the opposite of toughness. A brittle material is vulnerable to fracture at or near its proportional limit. A brittle material has a high compressive strength, but low tensile strength (i.e. amalgam). This is why amalgam preparations doe NOT have beveled margins (they need butt joints).

MODULUS OF ELASTICITY-a measure of a material’s stiffness or rigidity (the ratio of stress to strain below the elastic limit). The higher the modulus of elasticity, the stiffer (more rigid) the material. RESILIENCE-the energy that a material can absorb before the onset of any plastic deformation. GALVANIC SHOCK-a clinical phenomenon where an electrical charge is created when two dissimilar metals come into contact. Ex: a dental patient may experience sharp pain when two restorations constructed of different materials in opposing arches contact in the wet environment of the oral cavity. ► Ex: an amalgam restoration is placed on the occlusal surface of a lower tooth directly opposing a gold inlay of a maxillary tooth. Because both restorations are wet with saliva, an electric couple exists, with a difference in potential between the dissimilar restorations. When the two fillings contact, the potential is short-circuited through the two alloys, resulting in sharp pain. Such post-operative pain usually occurs immediately after inserting a new restoration and it gradually subsides and disappears in a few days.

► Within an hour after cementation of a cast gold onlay on an unanesthetized tooth, the patient complains of a “shooting pain” every time the teeth come together. The most probable explanation is a galvanic current caused by the gold onlay occluding with a large amalgam restoration. COEFFICIENT OF THERMAL EXPANSION-a measure of the tendency of a material to change shape when subjected to temperature changes (i.e. eating or drinking hot or cold items). A possible break in the marginal seal of any restoration becomes imminent when there is a marked difference in the CTE between the tooth and restorative material. The closer the CTE to the tooth, the better (direct gold is the best). A consequence of thermal expansion and contraction differences between a restorative material and adjacent tooth structure is percolation (the cyclic ingress and egress of fluids at restoration margins). The possibility of recurrent decay at the margins increases with increased percolation.

ELASTIC LIMIT-the greatest stress a material can be

subjected to and still return to it original dimensions when the forces are released. Ex: if a small tensile stress is induced in a wire, the resulting strain might be such that the wire returns to its original length when the load is removed. If the load is increased progressively in small increments then released after each additional stress, a stress value finally is found at which the wire does not return to its original length after the load is removed. In this case, the wire is stressed beyond its elastic limit. The elastic limit (proportional limit) on a stressstrain diagram, the modulus of elasticity is the ratio of stress to strain. Elastic limit and proportional limit are properties so equal in magnitude that they may be used interchangeably even though they are defined differently. PROPORTIONAL LIMIT-the greatest stress produced in a material such that the stress is directly proportional to the strain. A material with a high proportional limit has more resistance to permanent deformation than a material with a lower proportional limit. While elastic limit & proportional limit are defined differently, their magnitudes are so identical that

these two terms are used interchangeably. YIELD STRENGTH-a stress slightly higher than the proportional limit. Before adhesion can occur between a liquid and solid, it is essential that the solid surface be wetted by the liquid. When a liquid wets a solid completely, the contact angle between the liquid and solid is 0°. A percentage elongation of a metal is a measure of ductility and is related to the permanent strain at fracture. The property that most closely describes the ability of a cast gold inlay to be burnished is percentage elongation.

RESTORATIVE HAND INSTRUMENTS & BURS Modified Pen Grasp-the most common grasp used with hand instruments to allow the greatest intricacy or delicacy of touch. With this grasp, the middle finger, index finger, and thumb all rest on the handle close to the junction of the handle and shank. The middle finger rests on the shank, and thumb and index finger are opposite each other on the handle. Inverted Pen Grasp-very seldom used, but is sometimes used for cavity preparations using the lingual approach on anterior teeth. Palm & Thumb Grasp-the most powerful grasp, and is most effective on the maxillary arch. It is similar to the grasp used for holding a knife while peeling the skin of an apple. Modified Palm & Thumb Grasp-allows much of the power of the palm and thumb grasp, but also permits more delicate control. It works best when you can rest the thumb on the tooth being restored, or on the adjacent tooth. Also works best on the maxillary arch.

EXCAVATORS-designed to remove caries and refine the internal parts of a preparation. There are 4 types of excavators: 1. Hatchet excavator-has the cutting edge of the blade directed in the same plane of the handle and is bibeveled. Used primarily on anterior teeth for preparing retentive areas. 2. Hoe excavator-has the cutting edge of the blade perpendicular to the axis of the handle. Commonly used in Class III and V preparations for direct gold. 3. Angle Former-has the cutting edge at an angle (other than 90°) to the blade. Used for sharpening line angles and is useful to form convenience points for gold foil preparations. 4. Spoon excavator-has a curved blade with a rounded cutting edge. Used to remove carious dentin and to carve amalgam. These can be sharpened with handpiece stones. Handpiece stones can be used primarily to sharpen spoon excavators. CHISELS-used mainly to cut enamel. Chisels can be grouped as: 1. Straight, slightly curved, or bin-angle-primarily used for planing or cleaving enamel. Characterized by a blade

that terminates in a cutting edge formed by a one-sided bevel. 2. Enamel Hatchets-chisel bladed instruments with the cutting edge in the plane of the handle. They come paired left and right. Designed to most effectively plane enamel of the facial and lingual walls of a Class II amalgam preparation. It is the only instrument that allows the dentist to have proper access to the margins and that imparts the proper cavosurface angle to the margins. 3. Gingival Margin Trimmers-similar in design to the enamel hatchet, but has a curved blade and angled cutting edge. Primarily used for beveling gingival margins, and for rounding or beveling the axiopulpal line angle of Class II preparations. The number of bevels that make up the cutting edge can classify hand cutting instruments (i.e. enamel hatchets and chisels have single bevels, while ordinary hatchets like excavators have two bevels (bi-beveled). • Dental hand cutting instruments are angled to provide better manipulative control, produce a better distribution of force, increase efficiency, and establish proper balance when in use. • 13-80-8-14 hand instrument is most applicable for placing retention grooves in the distal box of a Class

II amalgam preparation on a mandibular left 2nd premolar. Instruments Used to Trim Restorative Materials (not for cutting tooth structure): 1. knives (finishing, amalgam, gold)-used to trim excess filling material on the facial and lingual surfaces. 2. files-used to trim excess filling material, especially at gingival margins. 3. discoid-cleoid-used mainly for carving occlusal anatomy in unset amalgam restorations. 3 Major Parts of Hand Cutting Instruments: 1. Handle-the part of the instrument held or grasped during activation of the blade. It can be single or double ended. 2. Shank-connects the blade to the handle. It can be straight or angled (mon-angle, bi-angle, or triple-angle), meaning one, two, or three angles in the shank. *Proper balance of the instrument occurs by angling the shank so the cutting edge of the blade is within 2mm of the handle’s long axis. To keep the blade within 2mm of the long axis, the shank is angled. 3. Blade-the working end of the instrument, connected to the handle by the shank. Blades are of many designs and sizes, depending on their function.

Nib-is not a major part of a hand cutting instrument. Rather it is the working end of a non-cutting instrument (i.e. burnisher or condenser). It corresponds to the blade of a hand cutting instrument. Cutting instruments have formulas that describe the dimensions and angles of the working end: • 1st number: blade width in tenths of a millimeter. • 2nd number: primary cutting edge angle in centigrades. • 3rd number: blade length in millimeters. • 4th number: blade angle in centigrades. Many instruments have 3 measurements in their formulas. #12 in formula 12-5-6 indicates the blade is 1.2mm wide.

BURS BUR PARTS: 1. Shank-the part that fits into the handpiece. 3 most common shanks (straight, latch-type angle, and frictiongrip angle). 2. Neck-the intermediate portion of a bur that connects the head to the shank. Functions to transmit rotational and translational forces to the head. 3. Head-the working part of the bur, the cutting edges perform the desired shaping of tooth structure. A bur’s shape refers to the contour of the head. Basic head shapes (round, inverted cone, elliptical, and fissure (plain, cross-cut, or tapered). Within a given series of burs, the smaller numbers represent smaller burs, larger numbers represent larger burs. BUR TYPES: 1. Steel Bur-used mainly for finishing procedures. 2. Carbide Bur-used for cavity preparation and performs best at HIGH SPEEDS. 3. Diamond Burs-used during crown and veneer preparation.

BUR BLADES: each bur blade has two sides and three important angles: 1. Rake face-side toward the direction of cutting. The surface (side) of the blade that contacts the tooth surface and faces in the direction of bur rotation. 2. Clearance face-the surface (side) of the blade that faces away from the direction of bur rotation. 3. Rake angle-the angle made between the line connecting the edge of the blade to the axis of the bur and rake face. This angle can be positive or negative. Rake angle is the most important design characteristic of a bur blade. • Soft materials like acrylics are cut most effectively with (+) angle burs, while extremely hard and brittle materials (amalgam) are best cut with (-) rake angle burs. • Carbide burs-used for cutting tooth structure, and usually have (-) rake angles and edge angles of ~90°. To be most effective, carbide burs should be rotating rapidly before contacting the tooth. On a carbide bur, a greater number of cutting blades indicates/results in less efficient cutting and a smoother surface. 4. Edge angle-the angle of a bur blade formed between

the rake face and clearance face. 5. Clearance angle-the angle formed between the clearance face and a tangent to the path of rotation. Another important factor in blade design, that serves to eliminate friction between the clearance face and new tooth structure exposed by the cutting edge. The greater clearance angle, the less friction. Factors that Influence Tooth Temperature During a Cutting Procedure: bur diameter and sharpness, bur-totooth contact time, amount of force applied to the bur, and type of COOLANT used (WATER IS BEST. Air may dehydrate the tooth causing hypersensitivity by drawing odontoblasts into the dentinal tubules).

THE PERIODONTIUM & FIBERS PERIODONTIUM-tissues that surround and support the teeth consisting of the gingiva, PDL, cementum, alveolar & supporting bone. The main functions are to support, protect, and nourish the teeth. • Attachment apparatus-consists of the alveolar bone proper, PDL fibers, and cementum that attaches the root to alveolar bone. • Gingival apparatus-describes the gingival fibers and epithelial attachment. • Gingival ligament-consists of dentogingival, alveologingival, and circular fibers. • Indifferent Fiber Plexus-small collagen fibers located in the PDL that run in all directions, and are associated with the larger principal collagen fibers. ALVEOLAR PROCESS-the part of the maxilla and mandible that HOUSES TEETH and consists of two main parts. 1. Alveolar bone proper-the only essential part of the bone socket (alveolar process) that is always present. It is a thin inner layer of compact lamellar bone that immediately surrounds the root where PDL fibers attach. It is a perforated cribiform plate through which vessels and nerves pass between the PDL and bone

marrow. It consists of two layers of bone (compact lamellar bone & layer of bundle bone (the layer that PDL fibers insert into). Alveolar bone proper helps form the attachment apparatus. 2. Supporting alveolar bone-is not always present, but surrounds the alveolar bone proper to provide SUPPORT to the socket (alveolar process) and consists of: • Cortical plate (compact lamellar bone)-forms the outer & inner plates of alveolar processes, and is thicker in the mandible than in the maxilla. • Spongy bone (cancellated bone)-fills in the area between the cortical plates of bone. Spongy bone is NOT present in the anterior region of the mouth (here the cortical plate is fused to the cribiform plate). This is also true over the radicular buccal bone of maxillary posterior teeth. GINGIVA ANATOMY: 1. FREE GINGIVA (unattached or marginal gingiva)-the collar of tissue not attached to the tooth or alveolar bone composed of: 1. Gingival margin-the most coronal part of the free gingiva. 2. Free Gingival Groove-separates free gingiva from attached gingiva; only present in 33% of adults.

3. Gingival Sulcus-the shallow groove between the marginal gingiva & tooth surface, bound by sulcular epithelium laterally, and by JE apically. 4. Interdental (interproximal) gingiva-occupies the interdental spaces coronal to the alveolar crest. GINGIVAL FIBERS (Supracrestal C.T. Fibers)-Type I collagen fibers that support and attach the gingiva to tooth and alveolar bone. Gingival fibers are found ONLY within the FREE GINGIVA (not in attached gingiva or mucogingival junction). Gingival fibers are continuous with the PDL (which is also C.T that surrounds the root and connects the root to alveolar bone by its principal fibers). Gingival fibers are designated by their orientation: 1. Alveologingival fibers-insert in the crest of the alveolar process and spread out through the lamina propria into the free gingiva. Also helps form the gingival ligament. 2. Circular fibers (Circumferential fibers)-resists ROTATIONAL FORCES applied to a tooth, and help form the gingival ligament. These fibers encircle the tooth around the most cervical part of the root, and insert into cementum and lamina propria of the free gingiva and alveolar crest. 3. Dentogingival fibers-extend from cementum apical to the epithelial attachment (JE) and travel laterally and

coronally into the lamina propria of the gingiva. Also help form the gingival ligament. 4. Dentoperiosteal fibers-extend from cervical cementum over the alveolar crest, to the periosteum of the cortical plates of bone. 5. Transseptal fibers: connect two adjacent teeth (tooth-to-tooth) and are sometimes classified within the PDL principal fibers, although they are actually tooth-to-tooth fibers, not tooth-to-bone fibers. These fibers extend from tooth-to-tooth, coronal to the alveolar crest, and are embedded in cementum of adjacent teeth. They are not found on the facial aspect, and have no attachment to the alveolar crestal bone. They maintain integrity of the dental arches. 2. ATTACHED GINGIVA-the gingiva attached to underlying periosteum of alveolar bone and to cementum by C.T. fibers and epithelial attachment. Present between the free gingiva and more movable alveolar mucosa. Attached gingiva contains KERATINIZED EPITHELIUM & LAMINA PROPRIA of dense, well-organized fiber bundles with few elastic fibers. It is firmly joined to underlying tooth structure, periosteum, & bone, and structured to withstand frictional stresses of mastication and brushing.

♦ NARROWEST BAND of attached gingiva is found on FACIAL SURFACES of the mandibular canine & first premolar, and lingual surfaces adjacent to mandibular incisors & canines. Narrow attached gingival zones may also occur at the MB root of maxillary first molars (associated with prominent roots and sometimes with bony dehiscences), and at mandibular third molars. ♦ Width of FACIAL attached gingiva ranges from 19mm. It is WIDEST on the facial surface of the maxillary lateral incisor, & narrowest on facial surfaces of mandibular canine & first premolar. ♦ “functionally adequate” zone of gingiva-a zone that is keratinized, firmly bound to tooth and underlying bone, ~2mm or more wide, and is resistant to probing and gaping when the lip is distended. BOUNDARIES that define the ATTACHED GINGIVA extend from the mucogingival junction (MGJ) to the free gingival groove (base of the sulcus). • Mucogingival Junction-separates attached gingiva from alveolar mucosa. However, the free gingival groove separates the free gingiva from attached gingiva. • Free gingiva-extends from the free gingival groove (sulcus base) to the gingival margin.

ATTACHED GINGIVA is normally CORAL PINK in color and firmly bound down to underlying tooth structure, periosteum, and bone. Its color varies depending on the degree of keratinization, thickness of epithelium, presence of melanin, and number of blood vessels. STIPPLING-the irregular surface texture of the attached gingiva that resembles an orange peel’s surface. Stippling occurs at the intersection of epithelial ridges that cause the depression, and the interspersing of C.T. papillae between these intersections that form the small orange peel bumps. • In the absence of stippling, edema of underlying C.T., inflammatory degradation of gingival collagen fibers, and normal variation in gingival topography may result in areas of the attached gingiva. ALL ORAL MUCOSA is STRATIFIED SQUAMOUS EPITHELIUM regardless if it is keratinized or nonkeratinized. • Non-Keratinized Oral Mucosa: buccal & alveolar mucosa, tongue’s inferior (ventral) surface, soft palate, floor of mouth, specialized mucosa, and lining mucosa, gingival col, and crevicular epithelium, o Alveolar Mucosa-functions as a LINING TISSUE, located APICAL to the attached gingiva on the facial & lingual surfaces. Consists of a thin, NON-KERATINIZED

epithelium, loosely textured, contains elastic fibers in the mucosa and submucosa, and is loosely bound to periosteum of alveolar bone. Alveolar mucosa is well-adapted to permit movement, but cannot withstand frictional stresses. • Keratinized Oral Mucosa: hard palate & attached gingiva. Functional Oral Mucosa: 1. Masticatory mucosa-composed of free & attached gingiva, and mucosa of the hard palate. The epithelium of these tissues is KERATINIZED, and the lamina propria is a dense, thick, firm C.T. of collagenous fibers. Important: the surface epithelium of the gingiva is highly impermeable, which makes it resistant to bacterial invasion. 2. Lining Mucosa (Reflective Mucosa)-mucosa that lines most of the oral cavity EXCEPT the gingiva, anterior palate, and dorsum of the tongue. Lining mucosa is a thin, movable tissue with a thin, non-keratinized epithelium, and a thin lamina propria. • Mucogingival junction-the junction of the lining mucosa with the masticatory mucosa. 3. Specialized Mucosa-consists of non-keratinized

epithelium that covers the tongue dorsum and taste buds PERIODONTAL LIGAMENT (PDL)-a complex, specialized, soft, fibrous C.T. containing numerous cells, blood vessels, nerves, & extracellular substances consisting of fibers (gingival & principal), & ground substance. PDL is a HIGHLY VASCULAR & CELLULAR C.T. that surrounds the roots of teeth and bridges root cementum with alveolar bone. Most PDL fibers are collagen, while ground substance consists of various proteins and polysaccharides. When the tooth loses its function, the PDL becomes very thin and loses its regular arrangement of fibers. ♦ PDL is hour-glass shaped with the narrowest part at the middle of the root. ♦ PDL occupies the space between cementum and the periodontal surface of alveolar bone. ♦ Cementum and alveolar bone are the tissues immediately adjacent to the PDL. ♦ MOST ABUNDANT CELLS in the PDL are FIBROBLASTS. Fibroblasts are ovoid or elongated cells oriented along the PDL principal fibers and exhibit pseudopodial-like processes.

♦ Epithelial Rests of Malassez-remnants of Hertwig’s epithelial root sheath, found as groups of epithelial cells in the PDL. Some epithelial rests degenerate, while others become calcified (cementicles). PDL FUNCTIONS: 1. Physical Functions: attaches tooth to bone via its principal fibers and absorbs occlusal forces by acting as a cushion to reduce the impact of forces generated during mastication on alveolar bone. Orthodontic treatment is possible because the PDL continuously responds and changes based on the functional requirements imposed on it by externally applied forces. 4 human dentition features that directly affect PDL health & its hard tissue anchorage to resist occlusal forces: 1. anterior teeth have slight or no contact in MIC (intercuspal position). 2. occlusal table is < 60% of the overall F-L width of the tooth. 3. occlusal table is generally at right angles to the tooth’s long axis. 4. mandibular molar crowns are inclined 15-20% toward the lingual. Thus, mandibular molars root apices are positioned more facially, while the crowns are positioned more lingually. 2. Formative Functions: formation of C.T. components by

the activities of C.T. cells (cementoblasts, fibroblasts, and osteoblasts). 3. Resorptive Functions: by activities of C.T. cells (fibroclasts, osteoclasts, and cementoclasts). 4. Nutritive Functions: through blood vessels to cementum, bone, and gingiva. 5. Sensory Functions: carried by the trigeminal nerve (CNV), proprioceptive and tactile sensitivity is imparted through the PDL (sensation of contact between teeth).PDL contains 2 types of nerve endings: 1. free, unmyelinated endings: convey PAIN. 2. encapsulated, myelinated nerve endings: convey PRESSURE. PDL’s average thickness in an adult is ~.25mm in normal function. PDL thickness depends on: 1. Age (PDL becomes thinner, thus decreases in thickness with age) due to increased deposition of cementum and bone which occurs mainly in the mandibular canine region. 2. Stage of eruption 3. Function of the tooth (PDL is thicker in functioning teeth, than in non-functioning teeth, and is thicker in areas of tension, and thinner in areas of compression) 4. Trauma history (PDL may not exist due to ankylosis (fusion of the tooth to bone).

Specialized cells of the PDL: function to resorb and replace cementum (cementoclasts or cementoblasts), PDL (fibroclasts or fibroblasts), and alveolar bone (osteoclasts or osteoblasts). PDL is composed primarily of Type I COLLAGEN FIBERS. PDL does not contain MATURE elastin fibers, but contains two IMMATURE elastin forms (Oxytalan & Eluanin). • Oxytalan fibers-run parallel to the root surface in a vertical direction, and bend to attach to cementum in the cervical third of the root. Oxytalan fibers regulate vascular flow. PDL is a specialized form of C.T. derived from the DENTAL SAC that surrounds the root and attaches the root to its bony socket. PDL is composed mainly of C.T. fibers (Type I collagen fibers) that are arranged into two groups: 1. Gingival Fibers-C.T. collagen fibers found in the free gingiva, but are continuous with the PDL C.T. fibers, thus are often considered part of the PDL. Gingival fibers SUPPORT the marginal gingiva and interdental papillae. • Gingival fibers: circular, dentogingival, dentoperiosteal, alveologingival, & transseptal fibers (sometimes considered PDL principal fibers). 2. Principal Fibers-collagen fibers of the PDL that include:

alveolar crest fibers, horizontal, oblique, apical, & interradicular fibers. Principal fibers CONNECT ROOT CEMENTUM TO ALVEOLAR BONE. They are distinguished by their location and direction. • Sharpey’s Fibers-the terminal portions of the PDL principal collagen fibers that are embedded into the cementum and alveolar bone. PDL Principal Fibers (tooth-to-bone fibers)-bundles of Type I collagen fibers grouped based on the direction they extend from the root cementum to alveolar bone. The terminal portions of the PDL collagen fibers that are embedded/anchored into cementum and alveolar bone are SHARPEY’S FIBERS. The diameter of Sharpey’s fibers is much greater on the BONE SIDE than cementum side. 1. Horizontal fibers-run perpendicular from alveolar bone to cementum, and RESISTS LATERAL FORCES. 2. Alveolar crest fibers-extend from the cervical cementum of the tooth to the alveolar crest, and function to counterbalance the occlusal forces on the more apical fibers and resist lateral movements. 3. Oblique fibers-the most numerous fibers, obliquely situated with insertions in cementum and extend more apically in the alveolus. Oblique fibers RESIST FORCES ALONG THE TOOTH’S LONG AXIS (masticatory forces). Mostly commonly found in the root’s middle

third. 4. Apical fibers-radiate apically from cementum to bone. Apical fibers offer INITIAL RESISTANCE to tooth movement in an OCCLUSAL DIRECTION. 5. Interradicular fibers-are found ONLY ON MULTIROOTED TEETH, extending from cementum in the root furcation area to bone within the furcation. GINGIVAL CREVICULAR FLUID (GCF)-in health, GCF is a transudate that emerges from the gingival sulcus. GCF may contain a variety of enzymes and cells, particularly desquamating epithelium & neutrophils being shed through the sulcus. An increase in GCF flow is the first detectable sign of inflammation. Once inflammation has occurred, GCF is called inflammatory exudate which contains a higher level of serum proteins and leukocytes. • Nutrient materials for the gingival epithelium cells are supplied mainly from capillaries in subjacent C.T. DENTOJUNCTIONAL EPITHELIUM-the gingival epithelium that faces the tooth, composed of non-keratinized stratified squamous epithelium and is divided into: 1. Sulcular epithelium-epithelium that lines the sulcus and connects directly with the JE.

2. Junctional Epithelium-a specialized epithelium surrounding each tooth. It begins at the sulcus base. It is a collar-like band of stratified squamous epithelium firmly attached to the tooth by HEMIDESMOSOMES. At its beginning, it is ~10-20 cell layers thick, while at its apical end it is only a few cell layers thick. JE has two layers (basal & suprabasal layers). In IDEAL GINGIVAL HEALTH, the JE is located entirely on enamel above the CEJ. ♦ Epithelial Attachment (Junctional Epithelium)-the special part/structure of the junctional epithelium and inner layer of cells of the JE that actually provides the attachment (it attaches gingiva to the tooth surface or to compatible restorative materials). Epithelial attachment consists of a lamina lucida, lamina densa, & hemidesmosomes. Histologically, the best way to distinguish free gingiva from the epithelial attachment is the epithelium of the epithelial attachment DOES NOT CONTAIN RETE PEGS while the free gingival does. Rete Pegs-epithelial projections that extend into the free gingival C.T. • C.T. papillae-projections that extend into the overlying epithelium. CERVICAL LINE (CEJ) CONTOURS: are closely related to the attachment of the gingiva at the tooth’s neck. The GREATEST contour of the cervical lines & gingival

attachments occur on the MESIAL SURFACE OF ANTERIOR TEETH (mesial surface of the maxillary central incisor has the greatest cervical line curvature). • All teeth generally have a greater proximal cervical line (CEJ) curvature on the MESIAL than the distal. Also, the proximal cervical line (CEJ) curvatures are greater on incisors and get smaller when moving toward the last molar where there may not be a curvature. • CEJ (cemento-enamel junction) of ALL teeth curves in 2 directions: toward the apex (on facial & lingual surfaces), and away from the apex (on mesial & distal surfaces). • In the absence of periodontal disease, the configuration of the crest of the interdental alveolar septa is determined by the CEJ’s position on adjacent teeth. • The width of the interdental alveolar bone is determined by the tooth form present. Relatively flat proximal tooth surfaces call for narrow septa, while in the presence of an extremely convex tooth surface, wide interdental septa with flat crests are found.

FLAPS, GRAFTS, & SURGERY Autogenous Free Gingival Graft-an autogenous graft of gingiva placed on a viable C.T. bed where initially buccal or labial mucosa were present. Usually, the donor site from where the graft is taken is an edentulous region or palatal area. The graft epithelium undergoes degeneration after it is placed, then sloughs. The epithelium is reconstructed in ~1 week by the adjacent epithelium and proliferation of surviving donor basal cells. In 2 weeks, tissue reforms, but maturation is not complete until 10-16 weeks. The healing time required is proportional to the graft’s thickness. The greatest amount of shrinkage occurs within the first 6 weeks. • Free gingival graft-involves removing a section of attached gingiva from another area of the mouth (usually the hard palate or an edentulous region) and suturing it to the recipient site. FGG success depends on the graft being immobilized at the recipient site. A FGG is used to increase the zone of attached gingiva and possibility of gaining root coverage. The difficulty in getting complete root coverage lies in the fact that an avascular graft is placed over a root surface also devoid of blood supply. • FGG retains NONE of its own blood supply and is

totally dependent on the bed of recipient blood vessels. The FGG receives its nutrients from the viable C.T. bed. • MAIN reason a FGG fails is disruption of the vascular supply before engraftment. Infection is the second most common reason of FGG failure. Free Gingival Graft Indications: • Prevent further recession and successfully widen (increase the width) of attached gingiva. • Cover non-pathologic dehiscences & fenestrations. • Performed with a frenectomy to prevent reformation of high frenal attachments. • Cover a root surface with a narrow denudation. FGG may or may not yield a successful result when used to obtain root coverage (the result is not highly predictable in root coverage cases). • Used therapeutically to widen attached gingiva after recession occurs, and to prophylactically prevent recession where the band of attached gingiva is narrow, and of a thin, delicate consistency. • Correct localized narrow recessions or clefts, but NOT DEEP WIDE RECESSIONS. In such cases, the

laterally repositioned flap (a pedicle graft) has a greater predictability. FGG is rarely used on facial or lingual surfaces of mandibular 3rd molars (especially facial). ♦ FGG Healing: involves revascularization of the graft. The graft’s top layers are re-vascularized last. Thus, the epithelium dies off (degenerates), producing the necrotic slough. During healing, the epithelium of FGG degenerates (necrotic slough), and reepithelization occurs by proliferation of epithelial cells from adjacent tissue and surviving basal cells of the graft tissue. Free Mucosal Autograft-a graft that differs from a FGG in that the transplant is C.T. without an epithelial covering. • Epithelial differentiation is induced by the underlying C.T. so that free grafts of dense C.T. taken from keratinized areas result in the formation of keratinized tissue even when transplanted to non-keratinized zones. This procedure is more difficult than FGG, and is often used on CANINES where little keratinized gingiva exists to create a band of gingiva-like tissue. • During healing, the epithelium of FGG degenerates (necrotic slough), and re-epithelization occurs by proliferation of epithelial cells from adjacent tissue and surviving basal cells of the graft tissue.

Root Amputation & Hemisection MUST be done in conjunction with endodontic therapy of a particular tooth. Endodontic therapy is performed first, followed by the periodontal therapy: • Root Amputations-the separation of an individual root from the crown. Most root amputations involve MAXILLARY 1st & 2nd MOLARS (commonly involved periodontal sites). Burs and diamond stones are used to sever the crown and root before extracting by root tip forceps. Once the root amputation is complete, the remaining apical area of the crown and furcation region are recontoured similar to the shape of a pontic so maximal access is provided for oral hygiene. • Hemisection-the vertical sectioning of the tooth through BOTH the crown and root. Usually, a hemisection is used in a mandibular molar region were the crown is divided through the bifurcation region. 50% of the tooth is extracted if one specific root has excessive loss in osseous support, and the remaining half of the molar tooth is now treated as a premolar. Distal Wedge (proximal wedge) Flap-the simplest distal flap procedure/approach used for retromolar reduction. This procedure is often performed after wisdom teeth are extracted because the bone fill is usually poor, leaving a

periodontal defect. This region is occupied by glandular and adipose tissue covered by unattached, non-keratinized mucosa. Only if sufficient space exists distal to the last molar, a band of attached gingiva may be present (in this case, as distal wedge operation can be performed). • Distal Wedge flaps are performed in these areas: maxillary tuberosity region, mandibular retromolar triangle area, and distal to the last tooth in the arch, or mesial to a tooth that approximates an edentulous area. • Many designs have been presented for the distal wedge flap procedure. However, the basic concept involves making at least 2 incisions distal or mesial to the tooth, and carrying these incisions parallel to the outer gingival wall to form a wedge (the wedge base is the periosteum overlying the bone, and apex is the coronal gingival surface). Detaching the wedge from the periosteal base and eliminating involved tissues in the distal pocket region reduces tissue bulk, and allows access to the underlying bone. Osseous Recontouring Surgery-main goal is to ELIMINATE PERIODONTAL POCKETS. The existing bony topography is changed to eliminate periodontal pockets. This surgery does not cure periodontal disease, but provides the patient the opportunity and access to maintain their own periodontium and dentition with routine oral hygiene procedures.

• Before using osseous resection or recontouring to treat an infrabony defect, the dentist should consider these treatment alternatives: maintenance with periodic root planning, bone grafts, reattachmentfill procedures, hemisection or root amputation. • In some surgical procedures, it is necessary to leave interradicular bone exposed. This usually results in bone loss of no clinical consequence. • Osseous resection surgery should NOT be done until etiologic factors that caused the osseous defects are arrested. Clinically detectable inflammation must be eliminated by SRP and by the patient’s ability to maintain optimal plaque control. • The MOST critical factor to determine if a tooth should be extracted or have surgery (prognosis) is AMOUNT OF ATTACHMENT LOSS (the amount of apical migration of the epithelium attachment). Amount of attachment loss is the most important factor in the determination of a prognosis of a tooth with periodontal disease (more accurate than probing depth, tooth mobility, and presence of furcation involvement). PRIMARY OBJECTIVE of Surgical Flap Procedures in treating periodontal disease is to provide access to root surfaces for debridement.

• Surgical Flap Goals to treat periodontal disease: to reduce or eliminate periodontal pockets, regrowth of alveolar bone, maintain biological width, and establish adequate soft tissue contours. • Flap techniques vary with the goal sought, but the common goal of all flap procedures is to PROVIDE ACCESS for instrumentation. It allows the clinician to visualize the roots so calculus can be removed more completely. • Without direct visualization provided by a flap, it is rare that a clinician can effectively root plane beyond 5mm of probing depth or into root furcations of lesser depth. Flaps also make removal of granulomatous tissue from the region of the periodontal defect difficult. It is important to remove this since it contains epithelium and potential presence of bacterial infiltration. • If a patient fails to demonstrate adequate oral hygiene during initial therapy (SRP), surgery is contraindicated because after surgery, the incidence of disease recurrence is greater if oral hygiene remains poor. The best course of action is to continue to stress oral hygiene, and maintain areas with SRP (scaling & root planing).

PERIODONTAL FLAP-a segment of marginal periodontal tissue that is surgically separated coronally from its underlying support and blood supply, and attached apically by a pedicle of supporting vascular C.T. Flap procedures are the MOST COMMON of all periodontal surgical techniques. Flaps should be uniformly thin and pliable, the flap base must be uniformly thin (~2mm thick), and all flap corners should be ROUNDED. 4 RULES OF FLAP DESIGN: 1. Flap base is WIDER than the free margin to allow sufficient blood circulation to the flap’s free margin. 2. Incision lines must not be placed over any bony defect to prevent delayed healing. 3. Incisions that traverse a bony eminence (canine) is avoided. The mucosa covering bony eminences is thin and healing is slow, and may result in an ugly scar formation. Incisions made in tissues that harbor uncontrolled infection may cause rapid infection spread (do not do this). Most periodontal surgical procedures are performed only after anti-infective therapy is complete. 4. All flap corners should be rounded, since sharp points delay healing (healing will occur without complication if basic surgical principals are followed).

Deep periodontal pockets are often treated by flap surgery. These cases often result in reduced pocket depth by formation of a long junctional epithelium (soft tissue reattachment), even if there is no change in the position of the gingival margins. The best indicator of success of a periodontal flap procedure is postoperative maintenance and plaque control by the patient. COMMON FLAPS: 1. Full-Thickness Mucoperiosteal Flaps-include the surface mucosa (epithelium, basement membrane, C.T. lamina propria), and contiguous periosteum of the underlying alveolar bone. This flap involves reflecting ALL of the soft tissue and periosteum to expose the underlying bone. A full-thickness flap is used where attached gingiva is thin (< 2mm wide). Apically & Coronally positioned flaps are full thickness flaps. • Modified Widman Flap (MWF)-a full-thickness flap that is a modification of the replaced flap. MWF is used in open flap debridement and regenerative periodontal procedures, and is a mainstay of periodontal surgery on single-rooted teeth and on flap surfaces of molars affected by moderate pockets and infrabony defects. • MWF objectives: gain access to underlying bone & root surfaces, reduced pocket depth by establishing a new attachment at a more coronal level, preserve an adequate zone of attached gingiva, and to provide an environment for

healing by primary closure. • MWF indications: pocket bases located coronal to the mucogingival junction, where there is little or no thickening of marginal bone, when shallow to moderate pocket depths can be reduced, and when esthetics is important (anterior region). • Repositioned Flaps: include replaced flaps, MWF, and excisional new attachment procedures. They all heal by repair (i.e. by a long junctional epithelium and C.T. adhesion or attachment), and are considered pocket reduction procedures. Pocket reduction is achieved mainly by gains in clinical attachment mediated by repair. 2. Partial-Thickness Periodontal Flap-includes ONLY the mucosa epithelium and a layer of the underlying C.T. The mucosa is separated from the periosteum by sharp dissection. Alveolar bone is not exposed. These flaps are used to prepare recipient sites for free gingival grafts, or when a dehiscence or fenestration is present on a prominent root. A partial thickness flap is used when attached gingiva is thick (> 2mm). One determinant of how the flap will be raised (either a full or partial thickness flap) is the thickness (amount) of attached gingiva prior to surgery. Full thickness & partial

thickness periodontal flaps can be displaced, but palatal flaps cannot be displaced (due to the absence of unattached gingival on the palate). POSITIONED FLAPS-procedures in which the flap’s coronal margins are advanced (lifted, but the flap is not freed-up) apically, coronally, or laterally from an area adjacent to the recipient site to a new location relative to the site they occupied before the procedure. Like repositioned flaps, most positioned flaps heal by repair. In positioned flap procedures, the flap’s vascular supply is maintained (unlike in the free gingival graft). Types of Positioned Flaps: 1. Pedical Flap (Laterally Positioned Flap)-a positioned full-thickness flap performed to correct defects in morphology, position, or amount of attached gingiva. It is usually a full-thickness flap that is attached at its base with its free end adjacent to the defect (recipient site). The defect is covered by stretching the flap laterally until the free end comes over it. • Indications: areas where narrow gingival recession is adjacent to wide band of attached gingiva that can be used as the donor site. Used to correct or prevent recession by providing root coverage, creating a wider band of gingiva, and in the absence of recession to widen the zone of gingiva.

• Positioned & Repositioned flaps are really pedicle flaps that are all physically attached at their apical base by a pedicle of lining mucosa and an intact blood supply. • Important factors to evaluate before performing a laterally positioned flap: presence of bone on the facial surface of the donor tooth, gingiva thickness and width of attached gingiva at the donor site. • Double Papilla Flap-a variation of the laterally positioned flap that uses the same principals, except the gingiva between the teeth (papilla) on either side are placed over the exposed root. 2. Apically Positioned Flap-a full-thickness, mucoperiosteal flap with a relatively high degree of predictability that is the “work horse” of periodontal surgery/therapy. This flap’s objective is to surgically eliminate deep pockets by positioning the flap apically while retaining the attached gingiva. Most commonly used in conjunction with osseous surgery as surgical access is obtained for osseous surgery, treatment of infrabony (intrabony) pockets, and root planing. • Indications: moderate or deep pockets, furcationinvolved teeth, and crown lengthening.

• Contraindications: patients at risk for root caries, as excessive root surfaces are often exposed AFTER performing an apically positioned flap, and where tooth exposure would be unesthetic. • In the course of flap surgery, after gaining access to underlying osseous tissue and performing the required therapy, the apically positioned flap is sutured to a place at a more apical level, exposing the alveolar margin. When this is done, additional attached gingiva granulates from the PDL and covers the barely exposed bone. This additional tissue joins the apically positioned attached gingiva to form a broader zone of gingiva. Note: on the palatal surface of maxillary molars, you need to trim the flap margin to proper length during the procedure. 3. Coronally Positioned Flap-a full-thickness mucoperiosteal flap almost exclusively used to restore gingival height and the zone of attached gingiva over isolated areas of gingival recession. There is no necrotic slough of positioned flaps because positioned flaps carry their vascular supply with them. In a FGG, healing involves revascularization of the graft. The graft’s top layers are revascularized last. Thus, the epithelium dies off (degenerates), producing the necrotic slough.

Internal bevel incision-basic incision in most periodontal flap procedures where the flap is reflected to expose the underlying bone & root. It has 3 objectives: 1. Remove the pocket lining. 2. Conserve the relatively uninvolved outer gingiva surface, which if apically positioned, becomes attached gingiva. 3. Produce a sharp, thin flap margin to adapt to the bonetooth junction. GINGIVOPLASTY-surgical procedure to reshape the gingiva & papilla of a tooth to correct deformities and provide the gingiva with normal & functional form. The overall goal is NOT to eliminate periodontal pockets, but to PROVIDE IMPROVED PHYSIOLOGICAL TISSUE CONTOUR. While portions of the gingiva are excised during a gingivoplasty, it is the reshaping (not the excision) of gingiva that defines a gingivoplasty. Commonly used to correct tissue contours that result from ANUG. GINGIVECTOMY-surgical procedure that REDUCES POCKET DEPTH by resecting the tissue coronal to the pocket base. The final aspect of the gingivectomy is to bevel (contour) the wound’s coronal margin to provide the most physiological shape and marginal thickness so adequate oral hygiene techniques can be performed. • Indications: treating pseudopockets, SUPRABONY POCKETS, and hereditary hyperplasia or druginduced gingival hyperplasia (ex: calcium channel blockers like Phenytoin).

• Contraindications: infrabony pockets (defects), lack of attached tissue, compromised esthetics with longer teeth, lack of access to bony defects, and presence of a broad, open wound post-surgically. • Factors to consider when electing to perform a gingivectomy rather than a periodontal flap: pocket depth (if the base of the pocket is located at the mucogingival junction or apical to the alveolar crest, do not perform a gingivectomy); need for access to bone; and amount of existing attached gingiva. • PERICORNITIS (OPERCULITIS)-soft-tissue (operculum) inflammation surrounding the distoocclusal surface of a partially erupted tooth (most often mandibular 3rd molars) resulting in erythema, edema, and foul taste. Treatments: Irrigation, antibiotics, wait for 3rd molar to erupt, gingivectomy, or extraction. OSTECTOMY-removal of osseous defects or INFRABONY POCKETS (pockets below the crest of bone) by eliminating bony pocket walls. This bone removed is supportive in nature, but is sacrificed because eliminating the pocket is worth the price of the loss in attachment apparatus. After removing the osseous pocket walls, some re-contouring is done to provide optimal osseous architecture for the overlying gingival tissues to

conform to and be maintained. • Major contraindication of removing crestal bone: if the removal will weaken adjacent tooth’s bony support. OSSEOUS DEFECTS: the pattern of bone loss from periodontitis can be horizontal or vertical: 1. HORIZONTAL BONE LOSS-the pattern of interproximal bone loss parallels the CEJ of adjacent teeth. Horizontal patterns are usually generalized by involving multiple teeth in a segment. • SUPRABONY POCKETS-the pattern of bone loss (destruction) is HORIZONTAL, and is not intraosseous. Transseptal fibers are usually normally arranged, but form at a more apical level, and the pocket base (epithelial attachment) is CORONAL to the crest of alveolar bone. Suprabony pockets may be further classified as a: • Gingival pocket (“relative or “pseudopocket”)-a condition where there is expansion of the marginal tissue coronally (not apical movement) of the epithelial attachment. Pocketing occurs WITHOUT attachment loss. • Periodontal pocket (“true pocket”)-a deepened gingival sulcus where the epithelium undergoes ulceration. Characterized by APICAL MIGRATION of the epithelial attachment (inner layer of cells of the JE) beyond its physiological

level which is normally at or near the CEJ. 2. VERTICAL BONE LOSS-pattern of interproximal bone loss does not parallel the CEJ, and are found around ISOLATED teeth. • Periodontal Osseous Defects (INFRABONY POCKETS)-are generally classified by the number of bony walls remaining that surround the tooth. With infrabony pockets, the pocket base (epithelial attachment) is APICAL to the crest of alveolar bone so there is actually a defect or hole within the bone (intraosseous). For a defect to have a bony wall, it must be intraosseous (i.e. partially or completely within alveolar bone; i.e. ramps, hemiseptums, interdental craters, intrabony defects, & moat defects). THE MORE BONY WALLS REMAINING, THE BETTER THE PROGNOSIS. Infrabony pockets are associated with VERTICAL (ANGULAR) BONE LOSS that creates holes/defects within the bone, and are classified as follows: 1. One-wall defects: Hemiseptum- if only the proximal wall is present. Ramp-if only a facial or lingual wall is present. The most difficult to treat. 2. Two-wall defects: (i.e. interdental crater). 3. Three-wall defects: an intrabony pocket; offer

the best opportunity for bone graft containment and periodontal regeneration procedures. Clinically, a bone grafting procedure is MOST likely successful with a 3-Walled Defect. 4. Four-wall defects: a circumferential or moat defects. Four-walled moat defects also offer the best opportunity for bone graft containment and periodontal regeneration procedures. 5. Zero-wall defects: alveolar dehiscences & fenestrations. Not treated with osseous surgery. 6. Combination defect: a complex combination lesion with more walls apically than coronally. Important: INFRABONY DEFECTS/POCKETS ARE CONTRAINDICATIONS FOR MUCOGINIGVAL SURGERY. With infrabony pockets, interproximally the transseptal fibers run in an angular direction not horizontally. Transseptal fibers extend in a sloping configuration from the cementum below the pocket base along the bone and down over the crest of bone to the cementum of the adjacent tooth. Osseous Craters-concavities in the crest of the interdental bone confined within the facial & lingual walls. Craters comprise 1/3 (35.2%) of all defects, and 2/3 (62%) of all mandibular defects. Osseous craters are MORE

COMMON in posterior segments than anterior segments, and are best treated with OSSEOUS SURGERY (recontouring). When evaluating an osseous defect, the ONLY WAY TO DETERMINE the number of walls left surrounding the tooth is by EXPLORATORY SURGERY. • Radiographs DO NOT SHOW the number of walls left surrounding the tooth, the exact configuration of the defect, or the location of epithelial attachment. This is because a dense buccal and/or lingual plate of bone tends to mask the defect and blocks it out on the radiographs. Thus, the number of osseous walls remaining can only be determined by exploratory surgery. • TWO MOST CRITICAL FACTORS to determine the prognosis of a periodontally involved tooth are MOBILITY & ATTACHMENT LOSS (the most critical). Periodontal osseous defects are classified by the number of osseous walls present/remaining at the time of their surgical exposure, and may have 1, 2, 3, or 4 walls. o To measure attachment loss, use a periodontal probe from an established reference point (CEJ or restoration margin) for initial measurement. Probings are repeated after appropriate time intervals and any changes are noted (i.e. if a probing depth is

4mm and recession is 3mm, total attachment loss = 7mm). OSTEOPLASTY-reshaping or recontouring the alveolar bone that does not provide attachment for periodontal fibers (non-supporting bone) without removing supporting alveolar bone. It is similar to a gingivoplasty in that it is not directed toward eliminating the pocket walls, but recontours & reshapes the underlying osseous structures. In this technique, the non-supporting bone is removed (this means the osseous reduction does not reduce the bone that the PDL is attached). • Non-supporting bone-alveolar bone not directed related to tooth support (i.e. bony exostoses, edentulous ridges, tori, flattened interdental contours and ledges). Walls of some osseous defects may be comprised of non-supporting bone. Clinically, a bone grafting procedure is MOST likely successful with a 3-Walled Defect. The success of periodontal bone grafting varies directly with the number of bony walls of the defect (vascularized, osseous surface area), and inversely with the surface area of the root against which the graft is implanted. • A narrow 3-walled infrabony defect yields the greatest success, then a 2-walled defect, then a 1walled defect (infrabony defect/hemiseptal defect = the least successful).

Sometimes the graft may take even when the bone graft is piled on the crest of the interdental septum. Clinically, the chances of success are best in a 3-walled infrabony pocket and least successful in a through-and-through furcation defect on a maxillary molar. ROOT RESORPTION is the most common side effect of an autogenous bone graft in managing an infrabony pocket and often extends into dentin and the pulp chamber. Other postoperative problems that sometimes occur after osseous or marrow transplants: infection, graft exfoliation, various and sometimes prolonged healing rates, and rapid defect recurrence. DEHISCENCE-alveolar bone loss (usually on the buccal/facial surface) that leaves a characteristic OVAL, ROOT-EXPOSED DEFECT from the CEJ apically (the defect may be 1-2mm long or may extend the full length of the root). Three characteristic features of dehiscence include: gingival recession, alveolar bone loss, and root exposure. FENESTRATION-an OPENING or WINDOW in the solid cortical plate of COMPACT BONE on the buccal/facial bony surface that exposes the tooth root. It can be distinguished from a dehiscence in that fenestration is bordered by alveolar bone along its coronal aspect. ALLOGRAFT-a graft taken from one human (donor) and

placed in another human (recipient). A freeze-dried decalcified bone graft taken from a human donor and placed in a periodontal defect in another patient is an allograft. Osseous Grafting Techniques: osseous coagulum, autogenous intraoral bone, iliac crest bone, Freeze-dried bone allograft (FDBA) with autologous bone, porous, coralderived hydroxyapatite, and Undecalcified freeze-dried bone allograft (UDFA). Hemopoietic marrow is the bone donor graft with the greatest osteogenic potential. GUIDED TISSUE REGENERATION (GTR)-placement of non-resorbable barriers or resorbable membranes & barriers over a bony defect. GTR BLOCKS the repopulation of the root surface by long junctional epithelium & gingival C.T. to allow PDL and bone cells to re-populate the periodontal defect (this technique assumes that only the PDL cells have the potential to regenerate the tooth’s attachment apparatus). • GTR’s physical blockade uses non-resorbable barriers (expanded polytetrafluoroethylene ePTFE) or resorbable membranes and barriers (Type I bovine collagen, calcium sulfate, or polyactic acid). • If a non-resorbable barrier is used, then 6-10 weeks after placement, a second surgical procedure is

performed to retrieve the non-resorbable barrier. • Tissue Regenerative Procedures are applicable and predictable under these circumstances: 1. patient exhibits exemplary plaque control both before and after regenerative therapy. 2. patient does not smoke. 3. there is occlusal stability of the teeth at the regenerative site. 4. osseous defects are vertical. The more walls of bone remaining, the greater likelihood of regenerative success. Surgical Dressing Materials: 1. Should be conveniently prepared and placed. 2. Should be soft and flexible for proper adaptation, while providing stability and flexibility when set. 3. Should be non-irritating to the oral tissues. 4. Should have a smooth surface to help resist plaque accumulation. Eugenol was previously the most popular and most effective agent in the periodontal dressing, but was found to cause its own tissue injury and necrosis. Today, commercially available periodontal dressings currently DO NOT CONTAIN EUGENOL. “Non-eugenol containing periodontal dressings include: • Chemical cured: PerioCare-a paste-gel & Coe-paka paste-paste.

• Visible-light cured: Baricaid that comes in a syringe. Advantages of Placing a Periodontal Dressing After Surgical Procedures: protects the surgical wound, minimizes patient discomfort, helps maintain tissue placement, and helps prevent post-operative bleeding. Periodontal dressings DO NOT enhance the healing rate of the tissues. • Periodontal dressings do not have a well-defined effect on wound healing processes or on surgical outcomes (i.e. gains of periodontal attachment or reduced probing depths). • All dressings must be used according to the manufacturer’s instructions and are removed within 7-10 days. However, before removing, ensure that the sutures are not embedded in the dressing and ensure the dressing is not locked interproximally. COLLAGEN found in the gingival has a turnover rate significantly greater than collagen found in tendons and the palate. • The most common form of the various collagen types in the mucosa is TYPE 1 collagen. Type I collagen in the gingiva is not the same biochemically as found in other parts of the body and skin. • Collagen turnover rate in normal gingiva is not as rapid as in the periodontal ligament (PDL), but is much greater than in other tissues (i.e. skin, tendons,

and palate). • Collagen accounts for ~60% of gingival protein. • Vitamin C is required for hydroxylation of praline and lysine which are essential for collagen formation.

GINGIVITIS & GINGIVAL CONDITIONS JUNCTIONAL EPITHELIUM-in health, the JE is a collarlike band of stratified squamous epithelium 10-20 cells thick near the sulcus, 2-3 cells thick at the apical end, and .25-1.35mm long. It is the stratified non-keratinizing epithelium that surrounds the tooth like a collar, and is attached by one broad surface to the tooth, and by the other to the gingival C.T. • JE has 2 basal laminas (1 faces the tooth (internal basal lamina) and 1 faces C.T. (external basal lamina). • Proliferative cell layer-responsible for most cell divisions and is in contact with C.T. (next to external basal lamina). • JE desquamative (shedding) surface-located at its coronal end, which also forms the bottom of the gingival sulcus. The JE is more permeable than the oral or sulcular epithelium, serving as the preferential route for the passage of bacterial products from the sulcus into the C.T. and for fluid and cells from the C.T. into the sulcus. LONG JUNCTIONAL EPITHELIUM-refers to the JE in disease, and differs from the JE in health. In disease,

migration of the JE occurs, along with C.T. degeneration under the attachment. As the JE proliferates along the root surface (gets longer), the coronal portion detaches. Barrier membranes (often used to treat bony defects) help prevent the long JE from forming. EPITHELIUM ATTACHMENT-the attachment apparatus (i.e. internal basal lamina & hemidesmosomes) that connects the JE to the tooth surface. The epithelium attachment is not the same as the JE (which is the entire epithelium). For a new attachment to form after periodontal treatment, complete removal of calculus, altered cementum, diseased JE, and pocket epithelium must occur, and there must be undifferentiated mesenchymal cells. CLINICAL CRITERIA TO DIAGNOSE GINGIVITIS: (bacterial plaque is the etiology of gingivitis). Without plaque, there is no gingivitis. Gingivitis is the PREDOMINANT periodontal disease. 1. Gingiva color: normal color ranges from coral pink to various stages of pigmentation. • Erythema-an intense red color of the gingiva. • Cyanosis-the most common color change (bluishpurple hue) found with periodontal disease. 2. Gingiva Contour: has a normal range, but is influenced by missing teeth, teeth position, etc. Papillae should fill

inter-proximal spaces, and gingival margins should be scalloped in form. 3. Gingiva Tone: the normal consistency of the gingival tissue should be resilient and fibrotic from the free gingival groove, apical to the mucogingival junction. Gingiva texture should include stippling (orange-peel appearance) of the attached gingiva. 4. Gingiva Size: healthy gingiva tissues should be wellcontoured to the underlying osseous architecture with the free gingival margin thin enough to allow a “knifeedge” thickness at the dentogingival margin. 5. Plaque & Calculus: the best way to evaluate the amount and distribution of plaque is using disclosing solution. There are NO radiographic features of gingivitis (radiographic appearance of bone appears normal). However, there are radiographic features of periodontitis. 3 Stages of Disease in Developing Gingivitis: 1. Transient (incipient) Stage-occurs within 2-4 days after cessation of oral hygiene. The earliest changes are visible microscopically consisting of a margination of leukocytes (PMNs) in vessels close to the junctional epithelium. Sloughed epithelium cells and bacteria are

found in the gingival sulcus. 2. Developing Stage-the area of collagen destruction becomes larger and is occupied by fluid that contains serum proteins: fibrin, immunoglobulins (especially IgG), complement, inflammatory cells, principally lymphocytes (B or T cells), and macrophages. In the developing stage, lymphocytes are the predominant cell. 3. Chronic Stage-the cytologic characteristics of the inflammatory infiltrate in the gingival lamina propria are changed. Plasma cells predominate the chronic stage. IgG is produced by most of the plasma cells, with a few cells present containing IgA (mostly in saliva). IgM containing cells are rarely seen. IgG-the immunoglobulin most abundant in the gingival exudates common in gingivitis. Significant levels of immunoglobulins are found in both epithelial and C.T. compartments. Although severe periodontal disease has been reported in people suffering abnormalities in neutrophil function (neutropenia), increased or more severe periodontal disease has not been reported in young people with other compromised immune functions (i.e. T or B cell deficient states). Acute Necrotizing Ulcerative Gingivitis (ANUG)-also called “Vincent’s infection” or “trench mouth”. ANUG is

a condition that presents pathognomic signs and symptoms. Fusiforms, spirochetes, & Prevotella intermedia are involved in ANUG’s etiology. Treponema denticola is the intermediate-sized spirochete associated with ANUG. Histologically, deeper areas of the lesion contain a zone of spirochetal infection. ANUG’s 2 Most Important Clinical Signs: 1. Interproximal necrosis & pseudomembrane formation on marginal tissues-only affects the gingiva. There is NO ATTACHMENT LOSS WITH ANUG. 2. History of soreness/pain and bleeding gums caused by eating and brushing. ANUG Signs & Symptoms: fetor oris (offensive odor), lowgrade fever, lymphadenopathy, and malaise. NEUTROPHIL is the dominant WBC involved in the inflammatory infiltrate of ANUG. • ANUG occurs most often in adults ages 18-30. Factors that predispose someone to ANUG: a history of gingivitis, tobacco smoking, gross neglect (poor oral hygiene), fatigue, stress, poor nutrition, and immuno-compromised patients. • ANUG Treatment: debridement, hydrogen peroxide (or warm saline) rinses, and antibiotic therapy (penicillin V) if there is systemic involvement (i.e.

fever, malaise, lymphadenopathy). Patients with HIVassociated ANUG require gentle debridement and antimicrobial rinses. Acute Gingivitis-gram (+) bacteria predominate (Actinomyces filament & Streptococci species). • As gingivitis develops (early lesion is 4-7 days), the area of destruction becomes larger with persistence of inflammation. Leukocytes invade C.T. and are dominated by lymphocytes (3/4 of all cells), macrophages, plasma cells (secrete IgG), and mast cells (release histamine). 3 Major Phenomena of Inflammation: 1. Increased vascular permeability-inflammatory tissue exudates forms due to increased permeability of blood vessels at the inflammatory site (edema). 2. Leukocytic cellular infiltration-involves mainly neutrophils, caused by chemotactic agents C5a & leukotriene B4 (LTB4) that function to phagocytize bacteria and elaborate proteolytic enzymes. 3. Repair-occurs either by regeneration or replacement. ACUTE INFLAMMATION-the INITIAL protective response of tissue to irritation or injury (especially bacterial infections) involving vascular and cellular

responses. Polymorphonuclear Neutrophils (PMNs) = neutrophilic leukocytes are initially the most abundant inflammatory cells in acute inflammation, which predominate in the Cellular phase. • PMNs (neutrophilic leukocytes) are the first line of defense and first cells to migrate into the gingival sulcus when inflammation caused by plaque formation in the initial lesion of gingivitis occurs. PMNs are the first line of defense and migrate toward the injured site (gingiva) as a result of the chemotactic properties of plaque. • Cells involved in acute inflammation PHASES OF ACUTE INFLAMMATION (2 PHASES): 1. Vascular Phase-involves basophils, tissue mast cells, & platelets which release HISTAMINE. Vasoconstriction (temporary)-narrowing of blood vessels caused by contraction of smooth muscle in vessel walls that are visible as blanching of the skin. Vasodilation-widening of blood vessels to increase blood flow to the infected area. Increased vascular permeability-allows diffusible components to enter the infected site. Inflammatory tissue exudates forms due to

increased permeability of blood vessels at the inflammatory site (edema). Note: Eosinophils are NOT in the vascular phase, but are the predominant inflammatory cells in allergic reactions and parasitic infections. 2. Cellular Phase: Polymorphonuclear neutrophils (leukocytes) predominate and are the first defense cells to migrate to the injured tissue via chemotaxis (C5a & leukotriene B4 (LTB4) are the chemotactic factors for neutrophils). PMNs engulf matter by phagocytosis. The engulfed matter becomes a phagosome and combines with lysosomal granules to form a phagolysosome that digests the engulfed particle. Leukocytic cellular infiltration-involves mainly neutrophils caused by chemotactic agents C5a & leukotriene B4 (LTB4) that function to phagocytize bacteria and elaborate proteolytic enzymes. Macrophages appear late in the cellular phase and represent a TRANSITION between acute and chronic inflammation. 4 LOCAL Signs of Acute Inflammation usually accompanied by loss of function:

1. Redness (rubor)-caused by dilation of capillaries. 2. Heat (calor)-capillary dilation allows increased blood flow through vessels, with the associated high metabolic activity of neutrophils and macrophages. 3. Swelling (tumor)-caused by increased capillary permeability. 4. Pain (dolor)-caused by lysis of blood cells which triggers bradykinin and prostaglandin production. Pain is a cardinal symptom of inflammation that may result from histamine and bradykinin release, and from direct nerve damage or pressure by the tissue exudates. 5. SYSTEMIC effects of acute inflammation: fever, tachycardia, and leukocytosis (especially neutrophils). MAST CELLS-participate in the early phase of inflammation (i.e. an early lesion of gingivitis), and are found in C.T., contain numerous basophilic granules, and release substances (heparin & histamine) in response to injury or inflammation of body tissues. Mast cell content in human gingiva is high, and increases as gingival inflammation increases. • Histamine-MAJOR HISTAMINE STORAGE SITES are mast cells, platelets, and basophils. Histamine is released when tissues are damaged, and is important in the VASCULAR PHASE of acute

inflammation (causing vasodilation and increased vascular permeability). • Anaphylactic response-characterized by the degranulation of mast cells when antigen-antibody complexes affix to cell surfaces. CHRONIC GINGIVITIS-the number of gram (-) anaerobic organisms increases. Fusobacterium, Prevotella intermedia, and Capnocytophaga species comprise ¾ of the flora. Spirochetes become evident, and Actinomyces species (filaments) remain. • As gingivitis reaches the chronic stage (established lesion whose time varies from weeks, months, or years), there is an increase in PLASMA CELLS (still secreting IgG) and B-LYMPHOCYTES that invade deep into C.T. This stage can persist for years with or without moving to the advanced lesion (periodontitis). • Chronic Inflammation-the protective tissue response to irritation or injury that persists LONGER than several days, with an accumulation of lymphocytes, plasma cells, but mostly macrophages (macrophages dominate). There is more proliferative than exudative, and necrosis occurs and recurs (as opposed to repair by regeneration). Fibrosis occurs causing progressive tissue damage and loss of function. Polymorphonuclear

Leukocytes (PMLs) are the main cell components that cause chronic inflammation. • When chronic gingivitis progresses to periodontitis (advanced lesion), lymphocytes, plasma cells, and macrophages continue to invade the C.T. along vascular pathways and destroy the gingival fibers. Changes within the supporting bone occur as the inflammatory process continues. *Neutrophils are the MOST NUMEROUS cell in the inflammatory exudates of an acute periodontal abscess. PELLICLE-a glycoprotein deposit (dental plaque) derived from salivary constituents that rapidly covers and is selectively adsorbed onto a freshly cleaned tooth surface. Pellicle formation is the 1st step in plaque formation. Pellicle Components: albumin, lysozyme, amylase, immunoglobulin A, praline-rich proteins, and mucins. • Primary Pellicle Colonizers: gram (+) bacteria like Streptococcus sanguis, Streptococcus mutans, & Actinomyces viscosus. • Secondary Pellicle Colonizers: gram (-) species like Fusobacterium nucleatum, Prevotella intermedia, and Capnocytophaga species. • Tertiary Pellicle Colonizers: Porphyromonas gingivalis, Campylobacter rectus, Eikenella

corrodens, Actinobacillus actinomycetemcomitans, and oral spirochetes (Treponema species). The overall pattern observed in dental plaque development is a very characteristic SHIFT from an early predominance of gram (+) facultative aerobes, to a later predominance of gram (-) anaerobes. The MAJOR factor in determining the different bacteria is OXYGEN. The redox potential of the gingival sulcus greatly influences the bacterial composition. The MOST abundant bacteria in a HEALTHY SULCUS are STREPTOCOCCUS & ACTINOMYCES species. Gram (+) cocci (Streptococci) & filamentous bacteria (Actinomyces) are most abundant in a healthy sulcus. • Streptococcus Viridans-consist of a variety of alpha-hemolytic streptococci (i.e. S. salivarius, mutans, sanguis, and mitis) that are all COMMON ORAL FLORA. • Normal Flora of the Oral Cavity: o Gram (+): Streptococcus, Peptostreptococcus, Actinomyces, Lactobacillus. o Gram (-): Veillonella, Fusobacterium, Corynebacterium, Campylobacter, Eikenella. At birth, the oral cavity is usually sterile, but microorganisims appear ~10-12hrs after birth. After one year, these bacteria are present: • Streptococci (S. salivarius is most abundant). S.

mutans and sanguis do not appear until teeth are present. • Staphylococci, Neisseria, Actinomyces, and Fusobacterium. • By age 4-5, the oral flora resembles an adult’s oral flora. PREGNANCY GINGIVITIS-a nonspecific gingivitis and definitive diagnosis is made on the physical state of pregnancy. There is an exaggerated and modified response of the gingiva to local factors (plaque), considerable loss in tissue tone, a color change to bright red, with marginal rolling and papillary enlargement that obliterates embrasure spaces. The prominent clinical finding is gingival hemorrhage (bleeding) upon gentle pressure. • If a women is in her 1st or 2nd trimester, scaling, polishing, and OHI can be performed. If she is well into her 3rd trimester, prudent treatment may be to just give OHI and reappoint her after childbirth for scaling and polishing. • Gingival changes evident DURING PREGNANCY probably (but not definitely) result from the effect of PROGESTERONE, and an increase in the number of MAST CELLS found throughout the gingival tissues.

• Pregnancy gingivitis is associated with increased levels of PREVOTELLA INTERMEDIA in the inflamed sites. These bacteria crave steroid hormones (i.e. Progesterone) for their own metabolism. • Gingival pathology IS NOT found during pregnancy if no etiologic factors accumulate (plaque) or are not present prior to the pregnancy. Localized Acute Gingivitis-the most common form of gingival disease in school-aged children. DESQUAMATIVE GINGIVITIS (DG)-a chronic gingival disease characterized by erythematous, erosive, vesiculobullous, and/or desquamative involvement of the free and attached gingiva. Most patients are postmenopausal FEMALES ages 40-70yrs. DG is a poorly understood, painful condition where the outer gingiva layer desquamates (peels away), exposing an acutely red surface. • Many diseases and conditions are associated with this gingival disease, but most are dermatologic. • Plaque’s role is vague in desquamative gingivitis. • Usually occurs due to an allergic reaction or is associated with skin diseases (i.e. lichen planus, BMMP, bullous pemphigoid, and pemphigus vulgaris).

This condition generally resolves when the allergic reaction or skin disease is treated and cleared up. • Treatment: depends on the underlying condition. Pemphigus, pemphigoid, and psoriasis may require systemic steroid therapy. Topical corticosteroids (Triamcinolone, Kenalog in Orabase) are effective in treating the oral lesions. • Histologically, where non-ulcerated areas are found, stratified squamous epithelium is significantly atrophic. Rete pegs are short or absent. Inflammatory cells (mainly plasma cells), may be found on the basal layer. HEREDITARY GINGIVOFIBROMATOSIS-a rare GENETIC DISEASE that is a progressive proliferation of the gingiva (especially collagenous elements). Clinically, there is generalized diffuse gingival enlargement, often extensive enough to cover the teeth. The tissue is dense and firm, with considerable distortion of normal contour. Gingival color is normal, but erythematous changes are a result of secondary bacterial involvement. • There is a striking lack of inflammatory cells, proliferating capillaries, and vascular engorgement commonly seen with most types of gingivitis. INFLAMMATORY GINGIVAL ENLARGEMENT-a gingivitis

form easily differentiated from simple gingivitis. Clinical findings include an increase in gingival size, distortion of normal form, and change in tissue tone. There is significant increase in sulcular depth and pocket formation occurs (pseudopocket or relative pocket). It is a pseuodopocket because it is caused by expansion of the marginal tissue coronally, rather than apical movement of the epithelial attachment beyond its physiological level. Histologically, there is a greater degree of sulcular proliferation through rete pegs than in simple gingivitis. Medications (specifically Phenytoin/Dilantin, Cyclosporine A, & Nifedipine) cause the highest incidence of fibrous gingival hyperplasia. Drugs that induce OVERGROWTH (hyperplasia) of gingival tissues: Phenytoin (Dilantin), Cyclosporine A (Sandimmune), & Nifedipine (Procardia). Gingival overgrowth is related to the level of plaque accumulation. The highest incidence of drug-induced hyperplasia occurs with PHENYTOIN (DILANTIN). Dilantin Hyperplasia-a progressive proliferation response to the gingiva associated with the use of sodium Dilantin (Phenytoin). Studies show administration of Dilantin does not cause a significant increase in fibroblastic activity. The occurrence of the gingivitis IS NOT automatic with the drug therapy if the level of oral hygiene is maintained and no pre-

existing gingival disease exists. Studies show between 50-60% of patients taking Dilantin will develop hyperplasia. A significant difference in comparing this hyperplasia to hereditary gingivofibromatosis, is the finding that there is an increased accumulation of inflammatory cells in Dilantin-induced hyperplasia. • 20% of patients on calcium channel blockers (i.e. Nifedipine), and 20-30% of patients on cyclosporine A (an immunosuppressant), are susceptible to drug-induced gingival overgrowth/hyperplasia. Pseudopocketing (“gingival pocket” or “relative pocket”)-a condition where pocketing occurs WITHOUT ATTACHMENT LOSS due to expansion of the marginal tissue CORONALLY (not apical movement of the epithelial attachment beyond its physiological level). A GINGIVECTOMY can be done to correct the gingival contours for Hereditary Gingivofibromatosis & Inflammatory Gingival Enlargement that is drug-induced.

PERIODONTITIS PERIODONTITIS-inflammation that affects and destroys the attachment apparatus (bone destruction). Periodontitis history is marked by APICAL MIGRATION of the junctional epithelium from the CEJ, loss of C.T. attachment & PDL, and bone destruction. Periodontitis usually progresses slowly and painlessly, but is arrested with proper therapy. • Periodontitis ALWAYS begins with gingivitis due to local irritation, primarily plaque, and the inflammation then spreads from the gingiva and soft tissues into the underlying structures. Gingivitis and periodontitis cannot be induced without bacteria (plaque). • Chronic gingivitis/Gingivitis does not always lead to periodontitis, and may exist for long periods without advancing to periodontitis. However, gingivitis & periodontitis MUST ALWAYS be induced by bacteria (plaque). • The presence of periodontal pockets CANNOT be determined from radiographs. To diagnose periodontitis, radiographic evidence of bone loss MUST be evident. Bitewing x-rays are the

MOST accurate to assess alveolar bone resorption. If extensive bone loss has occurred, vertical bitewings should be taken because they reveal more of periodontium. • MORE THAN 30% of the bone mass at the alveolar crest must be lost for a change in bone height to be recognized on radiographs. • A reduction of only 0.5-1.0mm thickness of cortical plate is sufficient to permit radiographic visualization of destruction of the inner cancellous trabeculae. The crest of alveolar bone is affected in periodontal disease. In health, the crest lies 1-2mm below the CEJ of the adjacent teeth. • In periodontitis, these radiographic changes are visible: loss of lamina dura, horizontal or vertical bone resorption, and thickening (widening) of the periodontal ligament space. • Bleeding, pocket depths > 5mm, and changes in tissue color and tone cannot lead to a diagnosis of periodontitis WITHOUT radiographic evidence of bone loss. • Radiographs as a diagnostic tool in periodontal disease CANNOT determine a definitive diagnosis of furcation involvement, and CANNOT accurately depict the morphology or depth of interdental craters that sometimes appear as vertical defects.

Radiographic Changes in Periodontal Disease: 1. Early periodontitis-areas of localized erosion of the alveolar bone crest (blunting (cupping) of the crest in anterior regions, and rounding of the junction between the crest and lamina dura in posterior regions). 2. Moderate periodontitis-destruction of alveolar bone extends beyond early changes in the alveolar crest, and may include buccal or lingual plate resorption, generalized horizontal erosion, or localized vertical defects and possible clinical evidence of tooth mobility. 3. Advanced periodontitis-bone loss is so extensive that the remaining teeth show excessive mobility and drifting, and are in danger of being lost. There is usually extensive horizontal bone loss or extensive bony defects. 4 STAGES OF THE PERIODONTAL LESION (INITIAL, EARLY, ESTABLISHED, & ADVANCED) 1. Initial Lesion (2-4 Days): no clinical changes, vasodilation of small capillaries, increase in leukocytes (PMNs), and increased gingival fluid flow. PMNs (neutrophilic leukocytes) are the first line of defense and first cells to migrate into the gingival sulcus when inflammation caused by plaque formation in the initial lesion of gingivitis occurs.

2. Early Lesion (Gingivitis 4-7 Days): clinical signs of GINGIVITS appear. Leukocyte infiltration into C.T. Sulcular lining develops RETE PEGS. Collagen destruction occurs and the sulcular lining is ulcerated. The area of destruction becomes larger with persistence of inflammation. Leukocytes invade C.T. and are dominated by lymphocytes (3/4 of all cells), macrophages, plasma cells (secrete IgG), and mast cells (release histamine). PMNs in sulcus. 3. Established Lesion (2-3 weeks): gingival erythema occurs due to capillary proliferation. Changes in gingival color occur starting at the papillary and marginal tissues, then progressing to attached gingiva. Gingival enlargement (may increase probing depths); plasma cells become prominent, and widened intercellular spaces in the pocket lining. 4. Advanced Lesion: the IRREVERSIBLE transition from gingivitis to periodontitis. JE becomes detached from the root surface as it migrates apically. Osteoclasts and bone loss. Clinical Criteria To Evaluate During a Periodontal Exam (besides color, tone, contour, & gingiva size): 1. Level of Free Gingival Margin relative to the CEJ: the normal level of epithelial attachment should be on enamel or at the CEJ (this places the free gingival

margin 2-3mm coronal to the sulcular base). 2. Periodontal Pocket Depth: all measurements > 3mm are recorded for sulcular depth, including any reading that locates the free gingival crest < 2mm at or below the CEJ. • Shallow pockets attached at the level of the apical third of the root connote more severe destruction than deep pockets attached at the root’s coronal third. When the gingival margin coincides with the cementoenamel junction (CEJ), the loss of attachment = the pocket depth. • Provides the most accurate assessment of periodontal pocket depth. 3. Loss of Attachment: determined by measuring the distance between the CEJ to the base of the attachment. • Attachment level-the position of the junctional epithelium at the base of a sulcus (pocket). In health, the JE is on enamel or at the CEJ. In disease, the JE migrates apically along the root surface. It is measured from an established reference point (CEJ or restoration margin) to the attachment with a periodontal probe. The periodontal pocket is measured from a changeable point (margin of free gingiva) to the attachment level. 4. Mobility: grades 1, 2, & 3. Use two instruments to test mobility. A depressible tooth has grade 3 mobility.

5. Bleeding: physiologically, bleeding from the gingival sulcus should not be caused by gentle provocation. Bleeding in the absence of local irritants may indicate a systemic disease. Bleeding is the most reliable indicator of gingival or periodontal inflammation. 6. Exudate: the presence of exudates (specifically suppuration) is evaluated by digital pressure on the buccal and lingual of each tooth. 7. Mucogingival complications: the involvement of the gingival component in the disease state and the presence of imminent involvement of the alveolar mucosa. TWO MOST critical parameters for the prognosis of a periodontally involved tooth are ATTACHMENT LOSS (most critical) and TOOTH MOBILITY. • Attachment loss is the MOST significant clinical criteria in regard to the prognosis of a periodontally involved tooth (much more significant than periodontal pocketing). Attachment loss is the most significant factor because with attachment loss, supportive structures are being destroyed. Pocketing can increase or decrease depending on the amount of inflammation without attachment loss. However, extensive attachment loss and gingival recession can be accompanied by shallow pockets (poor tooth prognosis).

Clinically, the most common sign of occlusal trauma is tooth mobility. PERIODONTAL POCKET PROBING-provides the MOST ACCURATE assessment of periodontal pocket depth. ♦ Periodontal pocket-a pathologic space between a tooth and pocket wall. Its depth is measured from the gingival margin to the clinical attachment level. Pocket depth is measured using a calibrated periodontal probe. The most important reason for using the periodontal probe is it DETERMINES LOSS OF ATTACHMENT. Probing measurements are taken BEFORE & AFTER SRP procedures to evaluate the tissue response and effectiveness of treatment. ♦ Periodontal probing provides the MOST ACCURATE ASSESSMENT of periodontal pocket depth. The true topography of vertical osseous defects cannot be determined by radiographs alone (as bone levels may be high, but pockets may be deep). Extensive bone loss may exist, yet be unaccompanied by pockets if the gingiva has receded. During PROBING, the periodontal probe is adapted in the interproximal areas so it touches the contact area with the tip angled SLIGHTLY BELOW & BEYOND the

contact area. The probe is angled ~10° on each interproximal surface so the probe tip is placed apical to the contact point of adjacent teeth and may detect any interdental crater. However, usually the probe is directed PARALLEL to the tooth’s long axis. ♦ Periodontal measurements are taken by inserting the probe UNDER the marginal gingival and gently moving the tip down to the junctional epithelium (JE) which feels soft, elastic, and resilient. In a healthy site, the probe tip stops within the JE and in a diseased site it penetrates into C.T. In severe disease, the probe tip may penetrate to the alveolar bone. ♦ The probe is inserted along the long axis of the tooth into the pocket with gentle pressure (~25gm of force) until resistance is met. This force depresses the thumb pad ~1-2mm. The probe is walked around each tooth surface. This method is less painful and more efficient, provides a complete and accurate assessment of the depth of epithelium attachment, and detects bony defects better. ♦ Clinical probing depth is ALWAYS GREATER than the histologic sulcus or pocket depth. Probing accuracy is only within +/- 1mm. ♦ The calibrated periodontal probe should have a

tapered shaft ~0.5mm in diameter at the tip. It is important to have uniform instruments throughout the practice to ensure standardization. Naber’s 2N (Hamp Probe)-used to detect and clinically diagnose 4 types of FURCATION involvement according to the GLICKMAN FURCATION CLASSIFICATION: 1. Grade I: incipient bone loss. Furcation probe can feel the depression of the furcation opening. 2. Grade II: partial bone loss (Cul-de-sac). Furcation probe tip enters under the roof of the furcation. 3. Grade III: total bone loss/destruction with through-andthrough opening of the furcation. The furcation entrance is not visible. The bone loss is at the furcation is to the extent that a probe can be passed through the furcation from buccal to the lingual (and vice versa). In trifurcated teeth, the probe can be passed from one aspect to another (from the ML to the buccal). In these cases, the furcation is still covered by gingival. 4. Grade IV: a Grade III furcation where the furcation entrance is visible. Findings That Complicate Furcation Involvement and account for some painful symptoms:

♦ Caries of cementum & dentin, tooth resorption in the furcation, abscess formation in the furcation, and involvement of pulp via lateral canals in the furcation. ♦ A definitive diagnosis of furcation involvement is made by careful clinical probing with a Naber’s (2N) or Hamp probe. Radiographs are helpful, but are used ONLY as an adjunct to the clinical examination. MAIN objective of treating involved furcations is to ELIMINATE FURCATION INVOLVEMENT using various treatment methods (however, not all treatments can eliminate the furcation, as some treatments only increases accessibility for plaque removal). Bone grafts have relatively little effectiveness in treating furcations. However Guided Tissue Regeneration (GTR) can treat Grade II furcations with good success. Furcation involvement of maxillary 2nd molars have the POOREST PROGNOSIS after therapy. MOST COMMON ERROR during periodontal probing is EXCESSIVELY ANGLING the probe when inserting it interproximally beyond the long axis of the tooth. This gives greater probing readings than are actually present. ♦ Tilting the probe can affects measurement accuracy. If the probe is angled too much it will extend past the contact area. If it is not inserted far enough, it will be at the line angle rather than under the contact area.

Both mistakes result in inaccurate readings. Thus, the probe tip should be FLAT against the tooth near the gingival margin with probe approximately PARALLEL WITH LONG AXIS OF THE TOOTH FOR INSERTION. ♦ Probe tip should ALWAYS be kept in contact with the tooth to prevent soft tissue injury. The probe is gently “walked” along the JE in an up and down motion (circumferential probing) always remaining under the gingival margin. It is critical that the probe be walked along the entire gingival sulcus since the depth of the epithelial attachment varies. ♦ SIX probing measurements for each tooth are recorded (3 on the buccal (DB, B, MB) & 3 on the lingual (DL, L, ML). These measurements are the distance in mm from the pocket base (junctional epithelium) to the free gingival margin. ♦ In the presence of inflammation, the probe may extend apical to the most coronal extent of the JE (pocket base) and give a slightly greater probing depth than is actually present. Patients at risk for subacute bacterial endocarditis must be pre-medicated BEFORE periodontal probing. Bacteremia can occur even with mastication or brushing, but it does not last long. The important

consideration is the presence or absence of periodontal inflammation. Inflammation leads to longer duration bacteremia with resultant risks for patients at risk of acute bacterial endocarditis. If the periodontal probe contacts an obstacle once inserted into the gingival sulcus, attempt to MOVE PAST THE OBSTACLE and then continue to move the probe apically. Often when probing, probe passage may be blocked by a hard, unyielding ledge of calculus. Thus, gently lift the probe away from the tooth and place it against the tissue wall of the pocket and attempt to proceed apically again. If the obstacle was calculus, and has now been bypassed, the probe will move deeper into the pocket until the JE is reached (sulcus base). The probe tip should then be placed back against the tooth once the obstruction has been bypassed. Important: insert the probe PARALLEL to the tooth’s vertical axis and “WALKED” circumferentially around each surface of each tooth to detect the areas of deepest penetration. GINGIVAL RECESSION-measurement of the migration of the free gingival margin APICAL to the CEJ (measured as a positive value). The recession measurement added to the probing depth indicates the amount of periodontal attachment that has been lost at that site. When the gingival margin is coronal to the CEJ,

recession measurement has a negative value. • MOST COMMON etiologic factor/cause of gingival recession is TOOTHBRUSH ABRASION (injury). This type of recession is common on the left canines of right-handed people (or right canines of left-handed people). • Gingival recession can also occur secondary to periodontal therapy. This may have additional significance in the older patient, namely increased risk for cervical abrasion, dentinal sensitivity, and most important, predisposition to root caries. • The hygienist or dentist should evaluate the brushing technique and monitor hard and soft tissue conditions at each recall visit. Faulty placement, overaggressive movement or pressure, or use of a hard toothbrush can cause hard and soft tissue damage. • Sometimes these areas will become sensitive if the root is exposed and the patient complains of COLD SENSITIVITY. This hypersensitivity will sometimes subside over time with daily plaque removal using a SOFT TOOTHBRUSH to help desensitize the root surface by allowing remineralization of the root surface. • The rationale of in-office desensitization procedures is

not completely understood. Some procedures may depend on denaturation of the superficial end of TOMES FIBERS or of nerve endings in dentin. Other procedures are designed to deposit an insoluble substance on the nerve or fiber endings to act as a barrier to stimuli, while others procedures stimulate secondary dentin formation (reparative dentin), thus insulating the pulp from external stimuli. TOOTHBRUSH ABRASION-usually occurs on CANINES & PREMOLARS (the most prominent teeth in the dental arch). Oral health maintenance is enhanced by using a SOFT-BRISTLE toothbrush with a dentifrice of optimum abrasiveness and dental floss. A hard-bristle toothbrush can cause gingival recession and abrasion of enamel or cementum, thus is avoided. • The abrasive quality of dentrifices affects enamel, but abrasion is a greater concern for patients with exposed dentin since dentin is abraded 25x faster, & cementum 35x faster than enamel. This can lead to root surface abrasion and root sensitivity. • Hard tissue damage from oral hygiene procedures is mainly due to abrasive dentifrices, while gingival lesion can be produced by a toothbrush alone. • Toothbrush trauma (abrasion) can cause recession

of the marginal gingiva, soft tissue lacerations (including attached gingival and alveolar mucosa), vshaped notches in the cervical areas of teeth, & gingival clefts (narrow grooves that extend from the gingiva crest to the attached gingiva). The location of these alterations if usually INVERSELY related to the right-or-left handedness of the patient. Important: in gingival atrophy (recession), tissues appears otherwise normal, the gingiva is thin, finely textured, and pale pink with normal papillae. In addition, the gingival sulci are very shallow, and plaque is minimal. HYDRODYNAMIC THEORY-the most accepted theory as to the CAUSE OF ROOT SENSITIVITY which postulates that the pain associated with root sensitivity is caused by indirect innervation caused by DENTINAL FLUID MOVEMENT IN THE TUBULES that stimulates mechanoreceptors in the pulp. DENTINAL HYPERSENSITIVITY-a problem in patients immediately after periodontal surgery that causes clinical exposure of root surfaces (dentinal tubules are not adequately sealed). Cold sensitivity is the main symptom. Plaque and food debris that remain on exposed root surfaces, often lead to increased sensitivity. To reduce dentinal sensitivity to thermal change after removing a periodontal dressing, KEEP THE ROOTS FREE OF PLAQUE. There are other treatments to reduce dentinal

hypersensitivity, however none of the following are totally effective. Often, diligent oral hygiene alone solves the problem of sensitivity. 1. topical fluorides: sodium fluoride (2.0%) & stannous fluoride (8%). Acidulated phosphate fluoride is contraindicated due to its high acid pH. 2. fluoride mouth rinses: OTC sodium fluoride (0.05%) or prescription acidulated phosphate-fluoride (0.04%) and sodium fluoride (0.2%). 3. desensitizing toothpastes: contain either strontium chloride, potassium nitrate, or sodium citrate. Recommend patient to use with a low abrasive dentifrice. 4. iontophoresis: electroplating fluoride to root surfaces. 5. dentin bonding agents: applied to the root surface (i.e. All-Bond, Scotchbond, Gluma, Amalgam-Bond). 6. root coverage with free-gingival graft surgery ATTACHED GINGIVA-measured from the projection of the sulcus base (periodontal pocket) onto the gingiva surface to the mucogingival junction (MGJ). The amount of attached gingiva is calculated by subtracting the sulcus/pocket depth from the width of gingiva from the free gingival margin to the mucogingival margin.

AGGRESSIVE PERIODONTITIS (formerly Juvenile Periodontitis or Early Onset)-occurs in two forms: 1. Generalized Aggressive Periodontitis (formerly Rapidly Progressive Periodontitis)-Prevotella intermedia & Eikenella corrodens predominate. Occurs between ages 12-25, and is characterized by rapid, severe periodontal destruction around most teeth, with episodic, rapid, and severe attachment loss. 2. Localized Aggressive Periodontitis (formerly Localized Juvenile Periodontitis)-gram (-) anaerobes Actinobacillus Actinomycetemcomitans (AA) and Capnocytophaga species (ochraceus) predominate. Prevotella intermedia and Eikenella corrodens may also be present to a lesser extent. Occurs in otherwise healthy adolescents ages 12-19, characterized by rapid and severe attachment loss confined to incisors and 1st molars. • One outstanding negative feature is the relative absence of local factors (plaque) to explain the severe periodontal destruction that is present. • Possible etiologic factors are a genetic predisposition or a dysfunction of neutrophils (a chemotactic defect). • AA & Capnocytophaga species (specifically C. Ochraceus) are also associated with periodontitis

in juvenile diabetes.

PLAQUE AND CALCULUS BACTERIAL PLAQUE-is the KEY ETIOLOGIC AGENT in the cause/initiation of gingivitis and periodontal disease. Plaque is an accumulation of a mixed bacterial community in a DEXTRAN MATRIX. It is formed on a cleaned tooth surface within minutes, and is composed of 80% water & 20% solids (95% of this 20% is bacteria). The clinician’s goal and the goal of oral hygiene is to remove this etiologic agent. After a prophy, plaque is most likely to accumulate on interproximal tooth surfaces first. • Plaque is a SOFT deposit that accumulates on teeth. Plaque is a complex microbial community with > 1010 bacteria/mg. In addition to bacteria cells, plaque contains a small number of epithelial cells, leukocytes, and macrophages. The cells are contained in an extracellular DEXTRAN matrix formed from bacterial products and saliva. The extracellular matrix contains protein, polysaccharide, and lipids. • Dextrans-insoluble & sticky, and contribute to plaque’s ability to adhere to teeth. Plaque also contain inorganic compounds (largely calcium and phosphorus) which are derived from saliva. Plaque’s inorganic content is greatly increased with the development of calculus. Calculus formation involves the calcification of dental plaque.

• Plaque can be supragingival or subgingival, and different bacteria may be found in plaque (cocci, rods, & filaments), and their properties change with time, diet, and location. Constituents of PLAQUE: the overall pattern seen in dental plaque development is a very characteristic shift from an early predominance of GRAM (+) FACULTATIVE bacteria, to a later domination of GRAM (-) ANAEROBIC bacteria, as the plaque mass accumulates and matures. As PLAQUE AGES, the number of gram (+) aerobic bacteria (i.e. cocci) DECREASES, while the number of gram (-) anaerobic bacteria (i.e. rods, fusiform filaments, spirochetes) INCREASES. STAGES OF PLAQUE FORMATION: 1. Acquired Pellicle Formation: the pellicle is derived from salivary constituents that are selectively adsorbed onto the tooth surface. Pellicle is composed of albumin, lysozyme, amylase, immunoglobulin A, proline-rich proteins, and mucins. The pellicle is essentially structureless and bacteria-free, and forms on a clean tooth surface within minutes due to its salivary origin. It also forms on crowns, dentures, porcelain teeth, etc. 2. Bacterial Colonization: within a short time after cleaning a tooth, the pellicle-coated tooth surface is colonized by gram (+) bacteria (i.e. Streptococcus sanguis, Streptococcus mutans, & Actinomyces

viscosus). These are the primary colonizers. 1. Primary Plaque Colonizers: GRAM (+) facultative bacteria that are the constituents of EARLY (YOUNG) plaque, appearing within a short time after cleaning a tooth. Streptococcus sanquis, Streptococcus mutans, & Actinomyces viscosus. 2. Secondary Plaque Colonizers: are GRAM (-) bacteria (Fusobacterium nucleatum, Prevotella intermedia, & Capnocytophaga species) found in plaque after 1-3 days of accumulation. 3. Tertiary Plaque Colonizers: consists of GRAM (-), ANAEROBIC RODS that appear 1 week after plaque accumulation (Porphyromonas gingivalis, Campylobacter rectus, Eikenella corrodens, Actinobacillus actinomycetemcomitans/AA, and oral spirochetes (Treponema species). 3. Maturation Stage: saliva continues to provide agglutinating substances and other proteins to the intercellular matrix, and bacterial intercellular adhesion results. The crystalline structure will increase and eventually calcifies into calculus. Supragingival Plaque: plaque that is attached or tooth associated, dominated by gram (+) aerobic facultative cocci (Streptococcus sanguis, mutans, & Actinomyces

viscosus). Saliva and diet alter its bacterial composition. Subgingival Plaque: plaque that is unattached or loosely adherent (more closely associated with the wall of subgingival tissues than with attached (supragingival plaque). Dominated by anaerobic, gram (-) rods. Saliva and diet do not alter bacterial composition. As pockets form, gram (-) anaerobic rods prevail. Bacterial species found are: • Porphyromonas gingivalis, Fusobacterium nucleatum, Prevotella intermedia, Bacteroides forsythus, Campylobacter rectus, and Treponema denticola and other Treponema species. Bacteria produce low molecular weight compounds (i.e. hydrogen sulfide & butyrate) that can induce some of the early signs of inflammation. However, the chronic signs of periodontitis (i.e. inflammation, collagen & bone loss) are mainly caused by bacterial products entering the tissue and activating inflammatory and immune processes. CALCULUS-MINERALIZED, MATURE PLAQUE covered on its surface with non-mineralized plaque, material alba, desquamated epithelial cells, and formed blood elements. Calculus is the MOST important plaque retentive factor. The microbiologic etiologic factor in periodontal diseases is DENTAL PLAQUE, while dental calculus is the most significant LOCAL contributing factor. The primary reason to remove calculus from teeth is because calculus harbors plaque microorganisms. Structurally,

calculus retains much of the histologic morphology of its plaque precursor. • Calculus forms on natural teeth, dentures, and other dental prosthesis (crowns, bridges). • Calculus is classified as SUPRAGINGIVAL or SUBGINGIVAL depending on its location. • Calculus is classified as SALIVARY or SERUMAL depending on the source of the inorganic salts that comprise it. • INORGANIC materials comprise ~70%-90% of calculus composition, while ORGANIC material and WATER comprise the remainder. Calcium and phosphates with small amounts of magnesium and carbonate (derived almost entirely from saliva). At least 2/3 of the inorganic matter in calculus is CRYSTALLINE SALT(mainly appatite/hydroxyapatite). Calculus also contains octacalcium phosphate, tricalcium phosphate (whitlockite), & brushite. • ORGANIC components of calculus comprise (1015%) and include microorganisms (same as in plaque), desquamated epithelial cells, leukocytes, and mucin. • Calculus exerts its pathogenic potential as a

contributing factor that fosters plaque formation and promotes it retention on teeth. • The rough surface of calculus is usually covered with a layer of plaque biofilm. Calculus “presents” plaque via the biofilm to periodontal soft tissues. 3 PHASES OF CALCULUS FORMATION: AVERAGE TIME for entire calculus formation process to occur is 12 days. 1. Pellicle formation: begins to form within minutes. 2. Plaque maturation: microorganisms grow together to form a cohesive plaque layer. 3. Plaque mineralization: occurs both supragingivally and subgingivally. Calculus is mineralized plaque that is formed by bathing the plaque in a highly concentrated solution of CALCIUM & PHOSPHORUS (saliva). SUPRAGINGIVAL CALCULUS-the main source of its minerals is from SALIVA. Supragingival calculus occurs ABOVE the free gingival margin and is white or pale yellow, and easily removed by a professional cleaning. It occurs most often on the tongue side of mandibular incisors and cheek side of maxillary molars due to the presence of the salivary ducts that secrete saliva rich in minerals needed for its formation.

• SALIVARY PELLICLE is the most COMMON MECHANISM that allows supragingival calculus to attach to smooth enamel surfaces. SUBGINGIVAL CALCULUS-is darker due to blood breakdown pigments, harder, and more dense than supragingival calculus. The source of minerals for subgingival calculus is CREVICULAR FLUID. Subgingival calculus occurs BELOW the free gingival margin and is much darker due to blood breakdown products. It is more difficult to remove than supragingival calculus, and is usually distributed EVENLY throughout the mouth, and is FORMED FROM GINGIVAL CREVICULAR FLUID SECRETION. • The attachment of subgingival calculus is complicated by irregularities like cemental tears, cemental voids once occupied by Sharpey’s fibers, resorption bays, and other CEMENTUM DEFECTS. Other Contributing/Complicating Factors in Periodontal Disease: • Food impaction/retention: overlapping, malposed, tilted or drifted teeth are often associated with food impaction or retention. If food is not removed, it leads to inflammatory periodontal disease. • Open & loose contacts: leads to food impaction and possible retention.

• Overhanging margins of restorations and improperly designed prostheses: can provoke or initiate periodontal disease, as there is a direct correlation between surface roughness or marginal irregularities of a tooth, and plaque retention. • Soft or sticky consistency of diet: food debris collects between teeth and along the gingiva, and can be a prominent cause of inflammation. • Violation of biologic width: if restorative materials invade biologic width permanent periodontitis will produce apical migration of the junctional epithelium. • Occlusal traumatism PLAQUE SCORE-obtained using plaque disclosing solution to help visualize plaque for the patient and clinician. Also used at multiple visits to show patients improvement in their level of oral hygiene. ENDOTOXIN (LPS = lipopolysaccharide base)-a constituent of gram (-) microorganisms that is an important agent in the pathogenesis of inflammatory periodontal disease. Gram (-) bacteria cell walls consists of a lipopolysaccharide base (endotoxin) that has great pathogenic potential. Typically, LPS containing gram (-) cell wall extracts can promote bone resorption, inhibit osteogenesis, chemotaxis of neutrophils, and other

events associated with active periodontitis. Free endotoxin is present in dental plaque and inflamed gingiva. Plaque accumulation has a direct effect on the severity of gingivitis. • Plaque bacteria produces ENZYMES (hyaluronidase, collagenase, chondroitin sulfatase, elastase, and proteases) that may initiate periodontal disease. o Collagenase-enzyme produced by Bacteroides species that catalyzes collagen degradation (hydrolysis). o Hyaluronidase-enzyme produced by Streptococcus mitans & salivarius that destroys the amorphous ground substance. o Chondroitin Sulfatase-enzyme produced by Diptheroids that also destroys amorphous ground substance. • Antibodies (immunoglobulins)-are produced by plasma cells in response to oral bacteria or their byproducts. The most numerous are IgG which neutralizes bacterial toxins by enhancing phagocytosis. • The most likely source of bacteria found in diseased periodontal tissue is SUBGINGIVAL PLAQUE. The probability that bacterial endotoxins

play a major role in gingival inflammation is evident by a reduction in inflammation upon removal of plaque and a reduction of the inflammatory state with antibiotic treatment. The frequency of maintenance visits for a patient who had previous periodontal treatment depends on 2 factors: 1. Appearance and clinical condition of the gingival tissues. This will determine if the patient is maintaining adequate plaque control. 2. Ability and performance in home care. This will determine the effectiveness of the patient’s oral hygiene. The main goal of the dentist, hygienist, and patient is to maintain good oral health to prevent disease recurrence. This is the main objective of the MAINTENANCE PHASE of periodontal therapy. The primary cause of disease recurrence is the dental team’s failure to motivate the patient to practice effective plaque control. The first year after treatment is critical, since the patient has already demonstrated susceptibility to periodontal disease (the cause of which tends to be persistent and recurrent). The appearance and condition of the gingival tissues determines if the patient is maintaining adequate plaque control.

• BLEEDING during circumferential probing indicates that crevicular epithelium is ulcerated due to active periodontal disease. Bleeding scores (bleeding) is the MOST RELIABLE indicator of the presence of gingival or periodontal inflammation. • After periodontal treatment, the first recall visit should be scheduled at 3 months. With excellent plaque control and maintenance of periodontal health, the interval can be lengthened to 4-6 months. The new periodontitis classification system is more descriptive and not as temporal as the previous system. The terms adult, juvenile, early onset, and pre-pubertal ARE NOW REPLACED with various forms of chronic and aggressive disease. The term refractory periodontitis has been removed as a distinct disease entity, since the current thinking is that any type of periodontitis may be refractory. Bacteria associated with PERIODONTAL HEALTH are gram (+), non-motile, facultative anaerobes. In the healthy mouth, more than 350 species of bacteria are present. Periodontal infections are linked to less than 5% of these species. Healthy and disease-causing bacteria are grouped into two categories: 1. Harmless/Helpful bacteria-are gram (+) non-motile, facultative anaerobic bacteria (coccal and rod

forms). Streptococcus species (Streptococcus gordinii) and Actinomyces species are associated with periodontal health. 2. In periodontal disease, the bacterial balance shifts to gram (-), motile, strictly anaerobic bacteria. Inflammatory disease and injury cannot develop without these gram (-) motile bacteria. Bacteria most implicated in periodontal disease and bone loss include: • Actinobacillus actinomycetemcomitans (AA)associated with aggressive periodontal disease (formerly early onset) and localized aggressive periodontitis (formerly localized juvenile periodontitis). • Porphyromonas gingivalis-associated with chronic periodontitis (formerly adult periodontitis). • Bacteroides forsythus-strongly linked to periodontal disease. • Treponema denticola, sokranskii-associated with deep periodontal pockets & ANUG. • Prevotella intermedia-associated with deep periodontal pockets & ANUG. • Eikenella corrodens, Campylobacter rectus, Fusobacterium nucleatum, Peptostreptococcus micros, Prevotella nigresens, enteric rods. • Pseudomonas & Eubacterium species have also been implicated as periodontal pathogens. Conditions that PREDISPOSE a patient to the development of inflammatory periodontal disease or

exacerbation of an existing disease: pregnancy, neutropenia, agranulocytosis, and leukemias. While bacterial and immune factors play a major role, genetic, systemic, and functional factors are also involved. • Genetic factors may predispose or trigger periodontitis. Factors governing leukocytic levels, migration, and chemotaxis may predispose a person to infection and influence host response. • Individuals with a defective immune response (i.e. as seen in agranulocytosis & neutropenia) are notably susceptible to crestal alveolar bone loss and abscesses. Occasionally, the periodontium is the site of lesions with initial foci elsewhere, (i.e. as in tuberculosis or cancer). • Systemic disease (i.e. diabetes mellitus) may lead to an increased incidence and severity of periodontitis. • Agranulocytosis-an abnormal condition of the blood, characterized by a severe reduction in the number of granulocytes, causing fever, prostration, and bleeding ulcers in the mouth, rectum, and vagina. • Neutropenia-an abnormal decrease in the number of neutrophils in blood. Associated with acute leukemia, infection, rheumatoid arthritis, vitamin B12 deficiency, and chronic splenomegaly.

• Leukemias-malignant neoplasms of immature white blood cells. Patients with acute leukemias have more oral complications that people with chronic leukemias. The gingiva is grossly enlarged, bluish-red in color, has a soft, spongy consistency, and the papillae are blunted. Evidence suggests that periodontal disease may be an AUTOIMMUNE DISORDER where the body’s immune factors (cytokines) attack the person’s own cells and tissues. These possible immune factors are interleukin-1 beta, interleukin-4, tumor necrosis factor-alpha, and prostaglandin E-2. • Bacteria that form plaque and calculus release toxins that stimulate the immune system to overproduce powerful infection-fighting factors called CYTOKINES (i.e. TNF-alpha, IL-1 beta, IL-4, and PGE2). Usually, cytokines are important for healing. In excess however, cytokines cause inflammation, severe damage, and overproduce the enzyme COLLAGENASE that breaks down proteins and the C.T. that supports the teeth. This inflammatory response may have damaging effects on the C.T. that supports the teeth and in organs throughout the body (including the heart). • A wide variation in host response to bacterial challenge between patients exists. Some people with heavy plaque accumulation and high proportions

of pathogenic organisms appear relatively resistant to bone and attachment loss. Others develop extensive periodontal destruction just by small amounts of plaque and low proportions of putative pathogens. • Patients with abnormally exuberant inflammatory responses often have a hyper-inflammatory monocyte/macrophage phenotype, thus secrete greatly increased levels of pro-inflammatory mediators like IL-1 beta, IL4, TNF-alpha, and PGE2 in response to bacterial byproducts. Periodontal disease can be associated with these SYSTEMIC DISEASES & CONDITIONS: • Down’s syndrome, HIV/AIDS, hormone imbalances, uncontrolled Type 1 & II Diabetes mellitus. • People suffering from Types I & II Diabetes have 15x the risk of developing periodontal disease as compared to the non-diabetic population. Much evidence exists on the link between DM-I and DM-II and periodontal disease, as diabetes causes abnormalities in blood vessels. • Several rare white blood cell disorders, genetic predisposition, medications, smoking, and osteoporosis. Most systemic diseases and conditions that may affect

periodontal diseases generally alter host barrier and host defense mechanisms. While many conditions cause gingival inflammation and ulcers, not all people develop periodontal disease. Certain factors place people at higher risk: • Smoking-the single MAJOR preventable risk factor for periodontal disease that can cause bone loss and gingival recession even in the absence of periodontal disease. Studies indicate that smoking and nicotine increase inflammation by reducing oxygen in gingival tissue and trigger an overproduction of immune factors (cytokinesspecifically interleukins). This overproduction damages cells and tissues. The risk of periodontal disease increases with the number of cigarettes smoked per day. Smoking cigars and pipes carries equal risk as cigarettes. • Evidence supports the link between Diabetes Mellitus Types I & II and periodontal disease (these people have 15x greater risk than the nondiabetic population). Diabetes causes abnormalities in blood vessels and high levels of specific inflammatory chemicals (i.e. interleukins) that greatly increase the risk of periodontal disease. • Osteoporosis-loss of bone density associated with periodontal disease in post-menopausal women. Some treatments for osteoporosis (i.e.

bisphosphonates) may reduce bone loss, including the bony structures that support teeth. • Autoimmune conditions-Crohn’s disease, rheumatoid arthritis, lupus erythematosus, and CREST syndrome are associated with a higher incidence of periodontal disease.

SCALING & ROOT PLANING MAIN OBJECTIVE OF ROOT PLANING is to provide optimally smooth root surfaces to reduce the potential for bacterial accumulation, which is done to achieve soft-tissue reattachment. Root planing is the instrumentation applied to the root surface to remove deposits and smooth the root. SCALING & ROOT PLANING (SRP)-techniques of instrumentation applied to the root surface to divest it of plaque, calcified deposits, and softened or roughened cementum. When thoroughly performed, these techniques produce a smooth, clean, hard polished root surface. Cementum, dentin, and calculus are removed during root planing. • SRP is the primary treatment for periodontal inflammation. In simple cases, SRP reduces shallow pockets and the number of bacteria in these shallow pockets, and may be the only treatment necessary. • In severely advanced periodontal disease where surgery may not be possible, SRP is the only feasible treatment. • Since plaque and deposit removal is the definitive treatment for periodontal inflammation, SRP is more often used than any other type of therapy. Commonly

observed clinical changes one week after SRP include reduced pocket depths and reduced gingival inflammation. • BEST CLINICAL AID to determine if subgingival calculus is removed is using an EXPLORER & BITE-WING X-RAYS which shows the presence of any interproximal calculus. • The purpose of SRP is to remove calculus, bacteria, and endotoxins. • When extensive SRP must be performed, the best approach is to schedule a SERIES of appointments to scale and root plane a segment or quadrant of teeth at a time thoroughly and completely. DO NOT do gross debridement (subgingival and supragingival) of the entire mouth, then schedule a series of appointments for fine scaling and polishing. SRP TIPS: • There is potential for abscess formation in a deep pocket only when a superficial scaling is performed. • OHI may be more effective if a patient can see healing tissue in an area that has been completely debrided and can compare it to tissue in an untreated area.

• A patient who had a gross debridement will see a marked visual improvement of the oral cavity, and may not understand the importance and necessity of the deep SRP appointments. This may cause the patient not to follow through with scheduled treatment, and the patient’s periodontal condition will further deteriorate. • Post-evaluation therapy should take place ~4-6 weeks (the time for repair of the dentogingival junction) after periodontal therapy has been complete. • If bleeding or swelling is noted in localized areas of the mouth during the re-evaluation appointment, check for and remove any residual calculus deposits that may remain.

HYGIENE INSTRUMENTATION & POWER DRIVEN SCALERS INSTRUMENT SHARPENING: the sharpening technique is important to maintain the instrument’s original shape. Manual sharpening is preferred over using mandrel mounted stones, as manual sharpening helps prevent unnecessary blade reduction that can occur when using a rapid cutting mounted stone. • A wire edge is produced when the sharpening stroke direction is away from, rather than into or towards, the cutting edge. Avoid producing a “wire edge” by always finishing with a down stroke. • Sharpen instruments at the first sign of dullness. • When sharpening hygiene instruments with a FLAT STONE, the angle between the blade face and stone is kept at 100-110°. • The technique to sharpen Gracey curettes is essentially the same as for Universal curettes. However, the Gracy’s cutting edge is offset and curves, unlike the universal’s cutting edge. • Lubricate the stone. Oil is used with natural stones (Arkansas & India stones) and water with

artificial/synthetic stones (Carborundum, Ruby, & Ceramic stones). Lubricant allows metallic particles removed from the instrument during sharpening to become suspended in the lubricant to help prevent scratching and “glazing” of the stone. • Choose sharpening stones appropriate for the instrument (flat, cone, coarse). Ensure that the sharpening stone is sterile when sharpening any instrument while working on a patient. • Establish proper angle between the stone and instrument. Maintain a firm grasp of the stone and instrument, but avoid excess pressure. • A properly sharpened instrument with NO rounded surface area WILL NOT REFLECT LIGHT. • The best way to prevent curette breakage is to use proper sharpening techniques to maintain the original instrument design and discard instruments when the blade starts to thin out. A thinner blade is weaker and breaks more easily. • Instruments must be sharp if SRP is to be completed efficiently with minimal tissue trauma. A sharp instrument requires fewer strokes, provides greater control, and increases tactile sensitivity. With a dull instrument, the handle must be grasped

much firmer with more pressure applied to the tooth surface, causing fatigue due to heavyhandedness, burnished calculus, decreased tactile sensitivity, and inefficient use of time. If the curette tip breaks off when scaling subgingivally, the appropriate actions to take to remove the tip are: • Gently examine the gingival sulcus using a curette in a spoon-like stroke attempting to pull the fragment out, take a periapical radiograph of the area, and place the patient in an upright position. NEVER use a push stroke to force the tip out of the sulcus as this could force the tip deeper into the sulcus. • The procedure should be stopped immediately and patient placed in an upright position. Before proceeding subgingival, check the mouth floor and mucobuccal fold for the fragment, then take a periapical radiograph before going any further to locate the fragment. Do not alarm the patient, to prevent the patient from becoming frantic. • Tactile sensitivity-the ability to distinguish degrees of roughness and smoothness on the tooth surface. Factors to Consider in Selecting Root Planing Instruments: calculus deposit location, area of the mouth to be instrumented, and adaptation of gingiva to the tooth.

Anatomic features of teeth often limit the effectiveness and efficiency of SRP calculus removal. It is difficult to perform a thorough SRP on the mesial surfaces of maxillary premolars, proximal surfaces of mandibular incisors, and trifurcations of maxillary molars. • Mesial surfaces of maxillary premolars and proximal surfaces of mandibular incisors are most likely to have flutings. Also, root proximity is a major problem when performing SRP on mandibular incisors. Trifurcations on maxillary first molars are the most difficult of all to root plane. • If while root planing you find only a thin ring of calculus in the bottom third of a deep pocket, you can assume the calculus previously extended the full length of the pocket, but the top part was previously removed. Likewise, if after SRP the patient returns in one week with hard, black deposits of calculus around the gingival margin, this indicates a reduction in inflammation occurred after the procedure, and old calculus is now exposed. • The BEST CRITERION to evaluate the success of SRP is NO BLEEDING ON PROBING (since BOP indicates active inflammatory periodontal disease). Periodontal Hoes & Files are used almost EXLUSIVELY for HEAVY accessible SUPRAGINGIVAL CALCULUS removal as their cutting edges are designed to function at

RIGHT ANGLES (90°) to the tooth surface. Due to their blade thickness, they lack adaptability and tactile sensitivity. They may be used subgingival for gross calculus removal ONLY if the tissue is flexible and easily displaced. Since they are for gross calculus removal only, they should always be followed by a curette. 1. HOES-have a wide, single straight cutting edge that cannot adapt to curved tooth surfaces (injury to tooth and tissue can occur). The entire cutting edge length must be kept against the tooth and only vertical pull-type strokes are used. The cutting edge (blade) is angled 90° to the tooth surface, and is turned at a 90-100° angle to the shank, and the cutting edge is beveled at a 45° angle to the end of the blade (this angle is maintained when sharpening the hoe). • Hoes are most effective on buccal and lingual surfaces. However, mesial & distal adaptation is poor, if not impossible. Maintaining a two-point contact with the tooth (cutting edge and side of shank) increases hoe stability. 2. FILES-primary function is to CRUSH or FRACTURE heavy calculus, but the entire deposit is not removed. The file is always followed by root planing with a curette, and works best on buccal and lingual surfaces, next to edentulous areas, and to reduce amalgam overhangs. While their size and lack of adaptability make interproximal use difficult, files work well on the

distal surface of the 3rd molar where access is often limited. Vertical pull-strokes are used with files

PERIODONTAL CURETTES PERIODONTAL CURETTE (Universal or Area Specific)-a narrow, delicate instrument with either 1 or 2 cutting edges (working surfaces) with rounded edge ends. Usually, curettes are in paired forms allowing access to opposite surfaces. Curettes are usually smaller than scalers and are designed to permit atraumatic entry into the subgingival space. The tactile sensitivity of most curettes is greater than scalers. Thus, curettes are well suited for subgingival calculus detection and removal, and root planing. The most effective instrument for subgingival SRP is a sharp curette. • Periodontal curette is the instrument that is least traumatic and most effective for NON-SURGICAL ROOT PLANING. • Most effective and efficient way to perform root planing is using curettes with short, even working strokes, followed by longer strokes. • Final root planing strokes are LONGER & LIGHTER than scaling strokes, and become lighter as the cementum becomes smoother. • Exploratory SRP strokes differ in angulation, pressure, length, and direction.

Curettes can be used with vertical, oblique, horizontal, or circumferential pull-type strokes. Curettes are used for subgingival SRP, supragingival calculus removal, and gingival curettage. 1. UNIVERSAL CURETTES Curettes (Crane-Kaplan 6, McCall 17/18, Columbia 4R/4L)-designed primarily to remove SUBGINGIVAL CALCULUS deposits on ALL tooth surfaces. Universal’s have 2 cutting edges at 90° to the root surface, and can be used in any region of the mouth. 2. GRACEY CURETTES- Designed to SMOOTH AND DEBRIDE ROOT SURFACES DURING ROOT PLANING in SPECIFIC regions of the mouth. Have only 1 cutting edge per working end with its blade face offset and angled 60°-70° to the terminal shank. Starting stroke is always CORONAL to the edge of the JUNCTIONAL EPITHEILUM. • Gracey 1-2: has a short shank distance to clean ANTERIOR INTERPROXIMAL SURFACES AND BUCCAL/LINGUAL SURFACES OF POSTERIORS. • Gracey 3-4: designed with a short modification of the shank, and are used in the same regions as Gracy 12. • Gracey 5-6: comes in two different shank lengths to clean ANTERIORS & PREMOLARS.

• Gracey 7-8: direct buccal and lingual of posteriors. They have limitations on the distal surfaces of posterior teeth. • Gracey 11-12: cleans the MESIAL, BUCCAL, LINGUAL surfaces of posterior teeth. The cutting edge is on the outside of the elbow. • Gracey 13-14: cleans the DISTAL SURFACES OF POSTERIOR TEETH. The cutting edge is on the inside of the elbow. • Gracey 15-16: access to the MESIAL surfaces of posterior teeth. • Gracey 17-18: access to the DISTAL surfaces of posterior teeth. A 60° angle of the curette’s facial surface against the tooth’s surface is ideal to remove PLAQUE, while a 75° angle is ideal to debride CALCULUS. Although the angulation for calculus debridement ranges from 4590°. An instrument’s SHANK design influences the instrument’s intended use: • Rigid, thick shank: is stronger, less flexible, and provides less tactile sensitivity. Stronger, more rigid shanks are used to remove heavy calculus

deposits. • Less rigid, more flexible shank: provides more tactile sensitivity. Used to remove fine calculus and for root planing. • Straight shanks are used in anterior areas, while longer (contra-angled) shanks are used in posterior areas. IDEAL ANGULATION of a curette’s facial surface is between 70-80° to the tooth surface to allow the curette blade to be properly activated against the tooth surface. • STROKE-the action of an instrumenting the performance of the task for which it was designed. Instruments are often given the name of the task being performed. Probing Stroke (Walking Stroke)upward and downward movement within a periodontal pocket. • EXPLORATORY STROKE (Assessment Stroke)used to assess the smoothness or roughness of the tooth surface and the effectiveness of instrumentation. Instrument handle is grasped lightly to increase tactile sensitivity. o Explorer-when using the periodontal explorer to detect calculus, a LIGHT instrument grasp (not firm) is used to increase tactile sensitivity. The lateral side of the instrument tip is placed

in contact with the tooth surface when exploring for calculus. • SCALING STROKE-a short, powerful “pull” stroke to remove calculus (more pressure). The scaling motion is preferably initiated in the forearm and transmitted from the wrist to the hand with a slight flexing of the fingers. Wrist rotation is synchronized with movement of the forearm. o The sequence and control of strokes is important. If heavy lateral pressure is continued with long, even strokes, it will produce a smooth, but “ditched” root surface. To avoid this, deliberate transition from short, powerful scaling strokes, to longer, lighter root planing strokes must be made as soon as the calculus and initial roughness are removed. o Preferably, the motion to initiate a SCALING STROKE is generated from the FOREARM. Although fingers may be used to initiate the scaling stroke in some situations, a powerful scaling stroke cannot be initiated in the wrist or fingers. Hence, is not carried out independently without using the forearm. The modified pen grasp is the most useful grasp for periodontal instruments.

• ROOT PLANING STROKE-long, overlapping strokes for final smoothing of the root surface, usually in a “pull” motion (less pressure). Pulling strokes are safer than pushing strokes, as pushing strokes can cause calculus to become embedded in soft tissue. Strokes can be directed vertically, horizontally, or obliquely (diagonally). The pressure applied becomes lighter as the surface becomes smoother. When root planing, vertical strokes should be used first, then oblique, then horizontal strokes. Light pressure should be used with root planing strokes to maximize tactile sensitivity. In root planing, the working stroke begins at the apical edge of the junctional epithelium (base of the pocket). AIR SYRINGE-applies compressed air to DETECT supragingival & subgingival calculus. Supragingival calculus is easier to see when the tooth is dried (saliva conceals it), and the air deflects the gingival tissue to make the subgingival calculus easier to detect. • AIR is NOT used during scaling to locate the epithelial attachment. • Use radiographs to detect supragingival and subgingival calculus. The best way to evaluate the amount and distribution of plaque is with DISCLOSING SOLUTION.

GINGIVAL CURETTAGE-a surgical procedure performed when the cutting edge of the curette is directed AGAINST THE SOFT TISSUE WALL of the pocket. Curettage-is only the treatment of the pocket wall (removal of sulcular epithelium and inflamed C.T.). The objective of gingival curettage is to remove chronically inflamed, diseased epithelial lining and microorganisms from the pocket to reduce edema and pocket depth. • Eliminating the cause of periodontal inflammation is NOT an objective of curettage. However, the objective of scaling & root planing includes the elimination of the cause of inflammation. In addition to removing plaque, stain, and calculus, SRP also removes “disease affected” cementum containing embedded calculus, whole bacteria, and toxic bacterial debris (i.e. endotoxin). • Curettage removes the diseased soft tissue lining the periodontal pockets. It is a manual process that permits a deeper and more complete cleansing than ultrasound. It does not add any significant benefits for shallow pockets. Local anesthesia is often used. • In gingival curettage, the curette’s blade face is positioned at a 70° angle to the soft tissue pocket wall or sulcular epithelium. The curette is positioned at the base of the pocket and pressure is applied with the finger on the outside of the pocket with smooth, even vertical strokes.

• Coincidental Curettage (Inadvertent/Incidental/Accidental)-performed when the soft tissue wall of a pocket is removed by the offset cutting edge during a root planing procedure. FINE SCALING WITH CURETTES SERVES TWO PURPOSES: 1. Scrape and clean root surfaces. 2. Plane root surfaces to smooth and remove the outer layer of diseased material. • Contraindications: Firm and fibrotic gingival tissues, areas of acute periodontal inflammation, an extremely thin lateral gingival wall, wide or tortuous infrabony pockets, and bi-and trifurcation involvement. • Indications: when gingival inflammation persists after careful and thorough root planing, treating shallow (up to 4mm) pockets where gingival tissues are highly edematous (patients with edematous & granulomatous inflammation respond better to curettage than patients with conditions of fibrous hyperplasia). o As maintenance treatment for areas of recurrent inflammation and where pocket reduction surgery has previously been performed.

o To reduce inflammation prior to pocket elimination and to remove chronically inflamed periodontal tissues o Often performed with root planing to promote soft-tissue attachment (re-epitheliazation) which occurs within 7-10 days. o MOST important factor to determine the amount of shrinkage is the DEGREE OF TISSUE EDEMA. Healing begins with blood clot formation. Neutrophils predominate immediately after curettage (for the first 12hrs). • To establish the correct working angle once a curette is inserted subgingivally, the shank is moved away from the tooth to open the blade angle to the tooth surface. At proper working angulation (70-80°), the lower shank of a Gracey curette is PARALLEL to the tooth surface, while a Universal curette’s lower shank is tilted slightly toward the tooth. • The curette blade is held at an angle > 90° (compared to ~70° for SRP). Digitally support the free gingival/pocket wall increases the cutting stroke effectiveness. Use Horizontal strokes to treat facial and lingual surfaces (stroke length is determined by tooth contour; usually long and continuous strokes achieve best results). Irrigate frequently while

performing the procedure. • Post-operative healing involves shrinkage of the gingival wall, pocket depth reduction, and formation of a long junctional epithelium. • Chisel-best designed to remove supragingival calculus deposits in interproximal areas (especially on anterior teeth). It has a single, straight cutting edge, and the end of the blade is flat and beveled at a 45° angle.

POWER DRIVEN SCALERS POWER DRIVEN SCALERS-use ULTRASONIC or SONIC ENERGY TO DEBRIDE SURFACES by converting electrical energy (ultrasonic) into high frequency sound waves that produce rapid vibrations in scaler tips (mechanical power). Power driven instruments must be adequately cooled, be kept in constant motion, and the side (not the tip), must be applied against the tooth structure at all times. Many ultrasonic units may be used with a variety of irrigants (i.e. chlorhexidine or providoneiodine). Ultrasonic scaling devices use either magnetostrictive or piezoelectric technology to convert electrical energy to physical energy at the instrument tip. The tip vibrates from 25,000-40,000 cycles/sec at an amplitude between 1013µm. When the tip is held against calculus, stain, cement, and other substances that adhere to the tooth surface, they are broken up and removed. SIDE OF THE TIP (not the pointed end) is applied so the tooth surface is not gouged or injured. Heat generated by magnetostrictive instruments requires copious amounts of water for cooling. In the cooling process, water cavitation occurs which releases dissolved gases. The

resulting spray and the water facilitates the removal of dislodged debris. WATER CAVITATION-that action created by the formation and implosion of air bubbles in water by high frequency sound waves; causes little effect on hard deposit removal, but destroys surface bacteria and root endotoxins. • In the cooling process, water “cavitation” occurs which releases dissolved gases. The resulting spray and the water facilitates the removal of dislodged debris. ULTRASONIC MAGNETOSTRICTIVE SCALERS: produce an ELLIPTICAL OR ORBITAL VIBRATION PATTERN of the tip (all tip sides are active and work when adapted to the tooth) to break apart calculus. • High frequency sound waves convert high frequency electrical energy into mechanical energy for form rapid vibrations. Waves dissipated in the form of heat and the heat is reduced by cooled water. • Operate at 20,000-45,000 cycles per second. • Uses stack of metal strips in insert to convert energy from handpiece to insert tip. • Consists of and electric generator, handpiece, scaling tip insert, and foot control. • All sides of tip are active-most active is the tip point, followed by concave face, convex back, then lateral sides.

• Heat generated by magnetostrictive instruments requires copious amounts of water for cooling. ULTRASONIC PIEZOELECTRIC SCALERS: produce a rapid LINEAR (BACK & FORTH) VIBRATION PATTERN of the tip (only two tip sides are active and work when adapted to the tooth) at a rate of 25,000-50,000 cycles/second (above the audible range). • A quartz or metal alloy crystal transducer in the handpiece converts electrical energy into ultrasonic vibrations. Operates by alternating electrical current applied to reactive crystals using tip-like scalers. • Less water is needed to cool the tooth surface because less heat is generated than with magnetostrictive scalers. • Uses ceramic rod to produce mechanical vibrations. • The LATERAL surface of the insert tip is MOST ACTIVE due to its linear motion. ULTRASONIC & AIR POLISHER PRECAUTIONS: when an ultrasonic scaler or air polisher is used to help CONTROL AERSOLS, use barrier protection (wear PPEs), high volume evacuation, and pre-procedural rinsing. • Ultrasonic instruments ARE USED for scaling, curetting, and removing stains. Ultrasonic scalers can remove both supragingival and subgingival calculus (never touch the tip of the ultrasonic on the tooth, use only the sides).

• Contraindications: patients with cardiac pace makers unless the pacemaker is shielded (newer models). First consult with patient’s cardiologist. • Use Caution: creates an aerosol spray (patients with known communicable diseases), compromised patients susceptible to infection, patients with a respiratory risk, patients prone to gagging (excessive water), children, demineralized areas, exposed cementum, restorative materials (i.e. porcelain crowns, titanium implants), patients with sensitive areas, and subgingivally (due to lack of visibility and potential narrow pockets). ULTRASONIC CONTRANDICATIONS (MAGNETO & PIEZO): patients with cardiac pace makers unless the pacemaker is shielded (newer models, still consult with patient’s cardiologist), demineralized areas, restorative materials, children due to developing tissues and primary and newly erupted teeth due to large pulp chambers, and communicable diseases that spread via aerosols. If a patient experiences SENSITIVITY during scaling with an ultrasonic scaling device, appropriate actions to prevent sensitivity are: Move the instrument faster (power driven instruments work best with quick hand movement or rapid, controlled movements), decrease the device’s power, proceed to another tooth and return to the sensitive tooth later in the appointment, adjust the water spray

(increase water to cool the tip), and use less pressure. • Subgingival root surface roughness does not interfere with healing after SRP. Thus, it is not useful to reinstrument root surfaces with hand instruments after a clinically detectable smooth surface is created with a sonic or ultrasonic scaler. SONIC SCALERS: do not release heat like ultrasonic instruments, but are AIR-TURBINE instruments that uses COMPRESSED AIR PRESSURE from the high-speed handpiece to produce ELLIPTICAL or ORBITAL tip vibrations from 3,000-7,500 cycles/second. • All sonic tip surfaces are active and effective. • Less power and less action to remove calculus because fewer vibrations are produced. • No heat is generated, but water is required to the cool the tooth surface. • Do not release heat the way ultrasonic instruments do, but are AIR-TURBINE instruments that use AIR PRESSURE from high-speed handpiece lines to produce tip vibrations from 2,000-6,000 cycles/second.

OCCLUSAL TRAUMA OCCLUSAL TRAUMA-trauma to the periodontium from functional or parafunctional forces causing damage to the attachment apparatus of the periodontium by exceeding its adaptive and reparative capabilities. There are 2 forms of occlusal trauma: 1. Primary Occlusal Trauma-excessive occlusal forces applied to a tooth/teeth with normal supporting structures (no periodontal disease). It’s a condition where pathologic occlusal forces are the primary etiology for observed changes in the periodontium. It is usually reversible once the forces that produced it are controlled. An early effect of primary occlusal trauma is HEMORRHAGE & THROMBOSIS OF PDL BLOOD VESSELS. 2. Secondary Occlusal Trauma-occurs when the periodontium is already compromised by inflammation and bone loss. Consequently, occlusal forces that may otherwise be well tolerated in a healthy periodontium now have deleterious effects due to preexisting periodontal disease. Teeth with a reduced adaptive capacity and compromised periodontium may then migrate when subjected to certain occlusal forces. Factors like frequency, duration, and velocity of those occlusal forces (not just their magnitude), may be of

greater significance in the development of tooth hypermobility (a common clinical sign of occlusal trauma). BRUXISM-an aggressive, repetitive, or continuous grinding, gritting, or clenching of the teeth (cupping/hollowed out areas on the occlusal table of the most terminal tooth) during the day and/or night in other than functional activities (i.e. chewing or swallowing). Opinions differ on the primary cause of bruxism, but occlusal prematurities, muscle tension, and emotional factors have been implicated. • Signs & symptoms: PDL widening and thickening of lamina dura, sore muscles and jaw pain, TMJ dysfunction and difficulty opening the mouth, increased tooth mobility (especially in the morning), occlusal wear facets. • Excessive forces produced by bruxism can cause increased tooth mobility. • Treatment: behavioral, emotional, and interceptive modalities. TOOTH MOBILITY-the movement of a tooth in its socket due to an externally applied force. Mobility is measured by the examiner pushing the tooth gently in a F-L direction using the blunt ends of two metal instruments. The use of a

finger is not acceptable to assess mobility. Mobility Scale: 1. 0 mobility = tooth is not mobile. 2. 1 mobility = tooth moves 0.5-1mm. 3. 2 mobility = tooth moves 1-2mm. 4. 3 mobility = tooth moves > 2mm or is mobile and depressible in an occlusal apical direction. Abnormal/Pathologic Mobility-tooth mobility past the physiologic range because it exceeds the limits of normal mobility values. The periodontium may not necessarily be diseased at the time of examination. OCCLUSAL SPLINTING-the primary reason for splinting teeth is to IMMOBILIZE excessively mobile teeth for patient comfort. Temporary stabilization is achieved by splinting one or more mobile teeth to each other and to more stable teeth in a position that facilitates a more AXIAL and EVEN distribution of occlusal forces (generally performed on teeth with reduced periodontal support). The rationale for splinting is improved patient comfort, function and plaque control, better distribution of occlusal forces, and improved tooth stability during clinical procedures. • There is no reason for splinting non-mobile teeth as a preventive measure. Splinting is only one type of measure used to treat periodontal disease, and should be used with other needed measures like root planning, OHI, pocket elimination, and occlusal

adjustment. • Loose teeth splinted to adjacent teeth may become stabilized. When many teeth are loose, adjacent sextants should be included in the splint. Teeth tend to loosen B-L, yet may remain firm M-D. Even when teeth do not tighten, the splint serves as an orthopedic brace that permits useful function to loose teeth. • A variety of methods can be used to achieve temporary stabilization. Whichever method used, special attention is paid to make the splint amenable to oral hygiene procedures and instructing the patient on plaque control around the splint. OCCLUSAL SPLINTS: 1. External Splints-includes ligatures, tooth-bonding plastic splints, welded band splints, continuous clasps, and night guards. These devices may be unesthetic or unhygienic, lack durability, rigidity, and/or fit. However, these splints are prepared easily and economically, and tooth structure is not removed (very conservative). • Night Guards-an acrylic splint whose primary purpose of a night guard in periodontal trauma is to modify and control BRUXISM or to REDIRECT forces into a non-

traumatic pattern. Night guards should be made of hard acrylic, have no sharp edges and be comfortable. Night guards should occlude evenly with mandibular teeth and permit the patient to move the mandible freely in all excursions. Also called centric relation splints, occlusal appliances. Goal is to allow condyles to seat in their most anterior superior position (centric relation) and to have posterior (point contacts) in CO and no posterior contact in any eccentric movement. • Functions: stabilize the dentition, modify the nature of the habit (bruxism), act as a splint after periodontal surgery, and assist in controlling recurrence of the drug-induced gingival enlargement. • Night guards are usually worn at night, but may also be worn during the day. • Bruxism clinical signs: TMJ symptoms, muscle soreness, cracked teeth/fillings, wear facets on teeth, and widened PDL spaces on radiographs. 2. Intracoronal Splints-include acrylic or amalgam with an embedded wire and acrylic or acrylic-and-gold provisional splints. Tooth structure is removed, and these splints are more serviceable than external splints. However, the materials wear and break, depend on the bonding medium strength, and tend to create plaque harbors that lead to caries, calculus deposition, and inflammation.

MOST COMMON SIGN OF OCCLUSAL TRAUMA IS TOOTH MOBILITY. Other clinical signs are migration of teeth and teeth tenderness to percussion.: ♦ Radiographic signs of trauma due to occlusion: widening of the PDL space, thickening of the lamina dura, angular bone loss and infrabony pocket formation, root resorption, and hypercementosis. ♦ Radiographic changes seen on teeth that are no longer in function: reduced trabeculation of bone and narrowing of the PDL space. ♦ Trauma from occlusion is reversible (the body can repair the damage if the excessive occlusal forces are eliminated). Periodontal pockets are not caused by occlusal trauma. A local irritant and inflammation are necessary to cause apical shift of the epithelial attachment (JE). ♦ Other findings associated with excessive occlusal forces: alternating areas of resorption and repair of alveolar bone, fibrosis of alveolar bone marrow spaces, cemental resorption leading to dentinal resorption, cemental tears, ankylosis, occasional pulpal necrosis and calcification. Reasons to Perform Selective Grinding in the Natural Dentition:

• Achieve a more favorable direction and distribution of forces. • Coordinate the median occlusal position with the terminal hinge position of the mandible. • Eliminate prematurities in excursive movements to gain group function or canine protected occlusion. • Direct occlusal forces centrally along the long axis of the tooth. • Improve or maintain masticatory performance. • Accomplish occlusal adjustment without reducing vertical dimension and by retaining an acceptable interocclusal distance. • Reduce or eliminate fremitus. Contraindications to Selective Grinding in the Natural Dentition: • When pulp chambers are large or in the presence of tooth sensitivity. • When major occlusal discrepancies may require orthodontics or reconstruction. • Patients who are poor candidates for full-mouth reconstruction due to psychologic factors. ADJUSTING OCCLUSION: eliminate prematurities in

centric relation (CR), in protrusive movements, and in lateral excursive movements. Then re-establish the physiologic occlusal anatomy and carefully polish all ground surfaces.

ABSCESSES ♦ PERIODONAL PROBING will reveal deep pockets associated with the periodontal abscess. Vitality tests, thermal and electric tests will exclude the pulp as the unlikely cause of symptoms as the tooth with a periodontal abscess is usually vital. Methods to distinguish a periodontal abscess from a pulpal (periapical) abscess is done via periodontal probing, EPT, thermal testing. However, a periapical radiograph is not a good diagnostic method to distinguish a periodontal and a pulpal abscesses. ♦ MOST COMMON symptom a patient will report with a periodontal abscess is ACUTE PAIN that is constant, severe, and dull throbbing. Thermal changes do not elicit or modify the discomfort. The onset of this discomfort is rapid and becomes progressively more intense. The patient may also notice an increase in tooth mobility, and say it is difficult to close their teeth together without striking the involved tooth first, causing increased pain. ♦ The periodontal abscess can be an acute

exacerbation of chronic periodontal disease and may occur when the infection passes into the tissue through the pocket epithelium. Such abscesses are often the result of blockage of the narrow openings of tortuous or deep infrabony pockets. ♦ Radiographic findings associated with the periodontal abscess are NOT specific. There may be no change radiographically in the early acute lesion. However, often there is a localized discrete radiolucency lateral to the root or in a furcation which can cause rapid alveolar bone destruction. ♦ Treatment of the acute periodontal abscess is determined initially by whether there is localization of the abscess (if there is, drainage is the treatment). If the abscess is not localized, the patient is placed on antibiotics (Penicillin V) and instructed to rinse with warm saline. Clindamycin can be used in penicillin allergic patients. Periodontal-Endodontic Abscess: Signs & symptoms are not always consistent, but include radiographic involvement of the periodontium and periapex, significant probing depths, percussion and pulpal sensitivity. Each case may or may not present with all of these signs and symptoms. In the presence of both a chronic endodontic and periodontal lesion, both lesions must be treated to achieve complete healing.

♦ Treatment: RCT (pulp is treated first and after the periodontal condition is re-evaluated 2-3 months after the completion of endodontic therapy). Antibiotic therapy, SRP, and periodontal surgery if needed, is performed 2-3 months after the completion of RCT. Periodontal Cyst-is usually asymptomatic and without grossly detectable changes, but may present as a localized tender swelling. Radiographically, an interproximal periodontal cyst appears on the side of the root as a radiolucent area bordered by a radiopaque line. Its radiographic appearance cannot be differentiated from a periodontal abscess. Apical Periodontal Cyst-has a predilection for mandibular canine-premolar area. Teeth are vital, usually asymptomatic, but may be slightly uncomfortable. There are no periodontal pockets, and radiographs reveal a small, well-defined oval-to-round radiolucency on the lateral surface of the root. Treatment includes surgical removal without extraction of surrounding teeth. Periodontal Treatment Planning: Preliminary Phasetreatment of EMERGENCIES ONLY whether a pulpal, periodontal, or other emergency. 1. Phase I (Initial Therapy)-involves plaque control, extraction of hopeless teeth, mouth preparation (initial full-mouth scaling, and definitive root-

planing). Also includes: ♦ Oral hygiene instructions (OHI) which is the most important aspect of initial therapy. The hygienist or dentist teaches, motivates, and guides the patient in the performance of measures for disease control. During a prophy appointment, the hygienist or dentist can establish a rapport with the patient and individualize instructions for a particular patient. The patient is shown proper brushing/flossing techniques, and these techniques are repeated to see if they understand what you are showing them. If oral hygiene is poor, surgery is CONTRAINDICATED. ♦ Occlusal adjustments, night guards (if bruxism exists), splinting (stabilizing loose teeth). ♦ Re-examination that involves charting probing depths. 2. Phase II (Periodontal Surgery) 3. Phase III (Restorative Phase) 4. Phase IV (Maintenance Phase)

ORAL HYGIENE INSTRUCTION Indications for using Super Floss: plaque removal around isolated teeth, teeth separated by a diastema, wide embrasures where interdental papillae have been lost, FPD (bridgework), orthodontic appliances, and implants. Superfloss consists of a nylon floss, ultrafloss, and regular floss. Toothpaste Ingredients: 1. Polishing (Abrasive Agent)-may be silica, calcium carbonate, or alumina. Polishing agent REMOVES stain, stained pellicle, and plaque. When selecting a polishing agent to remove generalized staining, consider tooth sensitivity, type of stain and restorations present, and the tooth surface condition. • Factors that REDUCE the abrasive action (effectiveness) of a polishing agent: Using an agent containing dull and round particles, using a thin, watery mixture of polishing paste, and/or polishing at a low speed with light pressure. • Factors that INCREASE a polishing agent’s abrasive action (effectiveness): flexing the polishing cup into proximal areas, or using an agent with nice

large particles. • Contraindications of using abrasive polishing agents and/or a rotary polishing instruments: • Patients with a communicable disease or respiratory problems due to the production of aerosols is likely that can disseminate the disease. • Patients with “green stain” (usually the tooth surface under the stain is demineralized). • Patients with newly erupted teeth that have not completely mineralized. • Any patient at risk for dental caries (patients with xerostomia, amelogenesis imperfecta, rampant caries, or receiving radiation therapy to the head/neck). • However, the use of a porte-polisher (manual polisher) may be helpful in some of these cases. Dental tape and finishing strips may be used to polish interproximal tooth surfaces. 2. Binder (Thickener)-can be carrageenates, alginates, or carboxymethylcellulose. The binder gives toothpaste its consistency and flowability when expressed from the tube. 3. Surfactant-detergent like sodium lauryl sulfate that foams to aid in debris removal.

4. Humectant-consists of glycerin, sorbitol, or polyethylene glycol. Humectant provides moisture to the paste, and keeps it from drying out. Serves to retain the paste’s moisture. 5. Flavoring-may be spearmint, wintergreen, or peppermint. 6. Active ingredient-may be fluoride (caries protection), triclosan (extremely potent antibacterial agent found in Colgate’s Total Toothpaste), as an antiplaque agent, pyrophosphate (an anti-calculus agent), potassium nitrate (a desensitizing agent), or peroxide compounds (whitening agents). Tooth Brushing Methods: the effectiveness of tooth brushing is BEST measured by the amount and location of plaque. No matter which brushing method is used, the manual toothbrush should have SOFT, NYLON bristles and a small head. 1. Bass Method (“Sulcular Technique”)-toothbrush bristles are placed 45° to the tooth surface at the gingival margin to try and get the bristles into the gingival sulcus. The brush is then moved in a back-andforth motion for ~20 strokes. This is currently the preferred method of manual tooth brushing. Sulcular technique is theoretically the most

effective tooth brushing technique. 2. Modified Stillman Method (“Roll Technique)-brush bristles are resting partially on the cervical area of the teeth and partially on the gingiva pointing toward the gingival margin. Pressure is applied to the brush to cause the gingiva to blanch. The brush head is then moved in short back-and-forth strokes with the brush moving coronally simultaneously. 3. Charter’s Method-toothbrush is placed against the surface of the teeth with the bristles pointed away from the gingival margin at a 45° angle. The backand-forth motion is a massaging stroke for the gingiva. Frequent brushing/flossing helps prevent calculus formation by breaking up the plaque matrix. New plaque growth occurs shortly after brushing and flossing (starts interproximally and works its way around the tooth). Orange, green, and brown stains on anterior teeth are usually caused by poor oral hygiene. Other Homecare Aids: Important: NOTHING REPLACES BRUSHING & FLOSSING to disrupt and remove plaque. 1. Perio-Aid-a homecare aid that consists of a plastic handle that will receive round polished TOOTHPICKS and permit the patient to cleanse the teeth at gingival margins, where accessible, and in

areas of difficult access. A tapered, round toothpick is inserted into the hole in the carrier, and is then broken off. The tip is left in and used in a tracing motion along gingival margins. Peri-Aid is also helpful in cleaning accessible furcations. 2. Stim-U-Dent (balsa wood wedges)-primarily important in gingival massage and good for patients with interdental recession. These picks are triangular in cross-section and are small enough to fit into most interdental spaces. As a supplement to brushing, StimU-Dents are useful for dislodging interproximal debris often missed by meticulous brushing, and for massaging the underlying interproximal gingiva. 3. Interproximal brushes (proxabrush)-replaceable brushes used for interdental cleansing when the interdental space is wide. 4. Interdental stimulator-consists of a rubber tip of smooth or ribbed conical-shape attached to a handle or to the end of a toothbrush. Its action massages and stimulates circulation of the interdental gingiva and may increase tissue tone. It is not recommended for areas where papillae are normal and fill interproximal spaces. It may cause injury to the gingival tissue. 5. WATER IRRIGATION DEVICES (“flushing or lavage”)-useful around fixed bridgework and orthodontic

appliances where debris tends to accumulate. The water pressure helps remove food debris, dilutes bacterial by-products and even some plaque, but does not remove all dental plaque. Water-irrigation devices may be injurious if used forcefully in deep periodontal pockets. ♦ Oral irrigation devices are CONTRAINDICATED IN PATIENTS WITH PERIODONTAL INFLAMMATION. Water irrigation devices may be contraindicated in patients requiring antibiotic premedication prior to dental treatment since these devices have the potential for causing a bacteremia. The patient’s physician should be consulted. ♦ Oral irrigation devices can remove NONADHERENT bacteria from subgingival & supragingival sites. This can be done at home or in the office. ♦ Water irrigation is also called “flushing” or “lavage” of periodontal pockets. ♦ When used with water alone, irrigation can reduce gingivitis, but not as much as if used with chlorhexidine gluconate 0.12% (works best). ♦ Oral irrigation devices do not remove acquired tooth pellicle and cannot remove adherent plaque better

than toothbrushes. Irrigation Benefits: reduce gingivitis, reduce/alter microbial flora, subgingival access (penetrates below the gingival margin), and delivery of antimicrobial agents (i.e. chlorhexidine gluconate, stannous fluoride, & sanguinaria). Fluoride, antibiotics, and chlorhexidine effectively inhibit microbial plaque. Chlorhexidine Gluconate (0.12%) in an alcoholcontaining vehicle: ♦ Stains teeth, tongue, and resin restorations. The stains are reversible. ♦ Impairs taste perception, but the taste perceptions are transient. ♦ Has retention properties that are concentration and time dependent and is not influenced by the temperature or pH of the rinsing. ♦ Has a LOW systemic toxicity and is not associated with teratogenic alterations. Two basic agents that have a significant effect on gingivitis over a 6-month period: CHLORHEXIDINE GLUCONATE (0.12%) PERIDEX-the

MOST effective antimicrobial and ANTI-GINGIVITIS rinse due to its high substantivity. It reduces gingival inflammation and plaque accumulation by damaging the cell membrane of gram (-) and gram (+) bacteria within 20- 30 seconds upon contact. An alcohol-containing vehicle and phenolic compound/essential oil-based mouthrinses can be used to help control gingivitis. • Chlorhexidine kills bacteria in situations when used for 30sec 2x/day. The most effective anti-microbial agent for reducing plaque and gingivitis longterm. Its effectiveness is explained by the fact that is leaves the greatest residual concentration in the mouth after its use. • Peridex is rapidly absorbed onto the teeth and pellicle, and is slowly released. It is approved by the ADA as an antimicrobial and antigingivitis agent (i.e. Peridex & PerioGard). • The most effective anti-microbial ORAL RINSE to reduce long-term plaque and gingivitis. • Drawback: Chlorhexidine can stain teeth and oral tissues YELLOWISH-BROWN to BROWN color after prolonged use. • Chlorhexidine stains oral tissues if used for a prolonged duration. Chlorhexidine stains impart a YELLOWISH-BROWN to BROWN color. It does

NOT impart a greenish black-to-black color of the oral cavity tissues. • Appears in the cervical and interproximal regions of the teeth more commonly. • The presence of aldehydes & ketones (intermediates of mammalian & microbial metabolism), are essential for formation of this discoloration. It is not a permanent stain, and can stain restorations. Essential oils (thymol, menthol, eucalyptol, and methyl salicylate for flavoring are the active ingredients in mostphenol-based mouthrinses. These rinses also contain 2027% alcohol in the vehicle (i.e. LISTERINE). Other Topical Antimicrobial Agents: 1. Stannous fluoride-its antimicrobial action appears to be related to the stannous (tin) ion rather than to the fluoride ion. Available in gel form (i.e. Stop, Gel-Kam). ADA accepts stannous fluoride in anticaries activity, but NOT for anti-plaque or antigingivitis purposes. 2. Quaternary ammonium compounds-are not as effective as others in reducing plaque or gingivitis, seem to be best at eliminating bad breath (halitosis) contains cetylpyridinium (i.e. Scope & Cepacol).

LOCALLY APPLIED ANTIBIOTICS (LAA) TO TREAT PERIODONTAL DISEASE 1. ATRIDOX (Doxycycline Hyclate 10%)- used to treat periodontal disease. It is a locally applied antibiotic (LAA) placed gently below the gum line into periodontal pockets where bacteria thrive. It is bioabsorbable, so it does not have to be removed once placed. • ATRIDOX is a gel applied with a syringe into the infected tooth pocket. It contains an antibiotic, doxycycline hyclate which flows to the bottom of pockets and fills even the smallest spaces between teeth and gums. After it is applied into the infected pocket, it hardens upon contact with oral fluids (saliva) to a wax-like substance. The antibiotic is slowly released into the surrounding infected tooth pocket for 21 days. In clinical studies, ATRIDOX has been shown to improve periodontal pocketing to help arrest the advancement of periodontal disease. 2. ARESTIN (Minocycline Hydrochloride)-an effective antibiotic treatment in a powder form easily placed inside infected periodontal pockets just after the dental professional completes scaling and root planing (SRP). ARESTIN contains microspheres (tiny, bead-like particles that are smaller than grains of sand not visible to the eye). The Microspheres are filled with the antibiotic minocycline, and they release the drug over time into the infected periodontal pocket, killing bacteria

that live there for up to 21 days. • In clinical studies, ARESTIN has been proven to be more effective than using SRP alone, and it significantly reduced the size of periodontal pockets compared to SRP alone, killed the bacteria most commonly associated with periodontal disease, and reduced bleeding on probing. 3. ACTISITE (Tetracycline Hydrochloride)-indicated as an adjunct to periodontal therapy to reduce periodontal pockets and bleeding on probing. Contraindicated in patients sensitive to tetracycline. Non-bioabsorbable, so must be removed after 10 days. 4. PERIOCHIP (Chlorhexidine Gluconate)-an antimicrobial agent; a small orangish-brown rectangular chip that is inserted into periodontal pockets as an adjunct to SRP. It is bioabsorbable, and also contains glycerin and purified water. LANAP (Laser Assisted New Attachment Procedure): laser surgical therapy to treat periodontitis through REGENERATION, not resection. Approved by FDA in 2004 for use in periodontal surgery. Laser energy removes diseased/necrotic or infected pocket epithelium from underlying C.T. at the histologic level of the rete ridges. Underlying pleuropotential C.T. is spared, to permit healing and regeneration, rather than forming a pocket seal by long junctional epithelium. No probing for 9-12 months after

LANAP to allow healing of cementum-fiber PDL interface.

DENTAL STAINING ENDOGENOUS INTRINSIC STAINS-originate from a source outside the tooth then become incorporated inside tooth structure or originate from inside the tooth. CANNOT BE REMOVED BY SCALING and POLISHING. Causes: • Dentinogenesis imperfecta-causes a translucent or opalescent gray to bluish-brown hue. • Erythroblastosis fetalis- causes intrinsic stain that is bluish-black, greenish-blue, tan, or brown. • Porphyria-causes an intrinsic stain that is red or brownish. • Fluorosis-causes white opacities, or light brown to brownish-black. • Pulpal Necrosis- intrinsic stain starts pink, then becomes orange-brown to bluish-black. • Internal resorption-causes a PINKISH intrinsic stain. • Tetracyclines-intrinsic stain varies from light-gray, yellow, or tan to darker shades of gray. • Excessive Systemic Fluoride during tooth development. • Amalgam EXOGENOUS EXTRINSIC STAINS-are REMOVABLE BY SCALING & POLISHING. After a period of time, green and tobacco stains may become incorporated with the tooth, at

which point they become endogenous intrinsic stains. 1. Brown Stain: a thin, translucent, acquired pigmented pellicle. Color is due to TANNIN. Occurs in patients who do not brush sufficiently or who use a dentrifice with inadequate cleansing. 2. Tobacco Stain: dark brown or black resulting from coal tar combustion products and from penetration of pits & fissures, enamel, and dentin by tobacco juices. 3. Black Stain: a thin, black line on the facial and lingual surfaces of teeth near the gingival margin, and as a diffuse patch on proximal surfaces. Caused by chromogenic bacteria (especially Actinomyces species; also implicated in orange and green stain). 4. Orange Stain: caused by chromogenic bacteria in plaque due to POOR ORAL HYGIENE. Common on anterior teeth. 5. Green Stain: associated with poor oral hygiene; caused by chromogenic bacteria, fungi, and gingival hemorrhage. 6. Green-Greenish-Yellow Stain: common in children. Attributed to fluorescent bacteria. 7. Metallic Stain: common in industrial workers who inhale metal dust or due to orally administered drugs containing

metals or metal salts. Metal stains vary from green due to copper and nickel, to black stains due to iron, silver, and manganese.

NERVE ANATOMY Trigeminal Nerve (CN V)-the LARGEST of the 12 cranial nerves and principal general sensory nerve to the head and face. Trigeminal exits the inferolateral pons as a sensory and motor root. The larger sensory root enters the trigeminal (semilunar, gasserian) ganglion in the middle cranial fossa. * Trigeminal nerve 3 sensory divisions arise from the ganglion and leave the cranial cavity through foramina in the sphenoid bone. The smaller motor root passes under the ganglion and joins the mandibular division (V3) as it exits through FORAMEN OVALE. Mandibular division (V3) innervates 8 muscles. * CN V (trigeminal) contains NO parasympathetic component at its origin. Somatic SENSORY bodies of the ganglion’s sensory fibers enter the: 1. Ophthalmic Division (V1) supplies general sensation to the ORBIT & SKIN of the face above the eyes.

2. Maxillary Division (V2) supplies general sensation to nasal cavity, maxillary teeth, palate, & skin over the maxilla. 3. Mandibular Division (V3) supplies general sensation to the mandible, TMJ, mandibular teeth, floor of mouth, tongue, and skin of mandible. The axons of the neurons enter the PONS through the sensory root and terminate in 1 of 3 nuclei of the trigeminal sensory nuclear complex. 1. mesencephalic nucleus-mediates proprioception (i.e. muscle spindle). 2. main sensory nucleus-mediates general sensation (i.e. touch). 3. spinal nucleus-mediates pain and temperature from the head and neck. Proprioceptive fibers from muscles & TMJ are found only in trigeminal’s mandibular division (V3). Cell bodies of proprioceptive 1st order neurons are found in the mesencephalic nucleus, not the trigeminal ganglion. The TMJ (as with all joints), receives no motor innervation, however the muscles that move the TMJ receive the motor innervation. Branchiomeric motor fibers-innervate temporalis, masseter, medial & lateral pterygoids, anterior belly of

digastric, mylohyoid, tensor tympani, and tensor veli palatine (palati). Mandibular Division (V3) of Trigeminal Nerve-passes through FORAMEN OVALE and supplies MOTOR innervation to tensor veli palatine, tensor tympani, muscles of mastication (temporalis, masseter, lateral & medial pterygoids), anterior belly of digastric, & mylohyoid muscles. * V3 Sensory Innervation: ► Long buccal nerve (sensory only) to the cheek and mandibular buccal gingiva. ► Auriculotemporal nerve (sensory only) to TMJ, auricle, and external auditory meatus. ► Lingual nerve (sensory only) to floor of mouth, mandibular lingual gingiva, and anterior 2/3 of tongue. ► Inferior alveolar nerve (sensory & motor) to mandibular teeth, chin skin, and lower lip. Masseteric Nerve (nerve to the masseter)-branch of mandibular division (V3) that carries a few sensory fibers to the TMJ’s anterior portion. Auriculotemporal Nerve-a branch of mandibular division (V3) that provides the major SENSORY innervation to the TMJ’s posterior portion. Transmits pain in the TMJ capsule and disc periphery.

Nerve to Mylohyoid Muscle-a branch of the mandibular nerve (V3). Mylohyoid muscle elevates the hyoid bone, base of tongue, and floor of mouth. The sublingual gland is superior to the mylohyoid muscle. * When placing the film for a periapical view of mandibular molars, the mylohyoid muscle can obstruct the view if it is not relaxed. * When the floor of the mouth is lowered surgically, the mylohyoid & genioglossus muscles are detached.

Hypoglossal Nerve-a motor nerve supplying all intrinsic & extrinsic tongue muscles (except palatoglossus), which is supplied by the vagus nerve. It leaves the skull through the hypoglossal canal medial to the carotid canal and jugular foramen. Soon after it leaves the skull through the hypoglossal canal, it is joined by C1 fibers from the cervical plexus. It passes above the hyoid bone on the lateral surface of the hyoglossus muscle deep to the

mylohyoid muscle. It loops around the occipital artery and passes between the external carotid artery and internal jugular vein. * Unilateral lesions of the hypoglossal nerve cause deviation of the protruded tongue towards the affected side due to the lack of function of the genioglossus muscle on the diseased side. Injury of the hypoglossal nerve eventually produces paralysis and atrophy of the tongue on the affected side with the tongue deviated to the affected side. Dysarthria (inability to articulate) may also occur. * If the genioglossus muscle is paralyzed, the tongue has a tendency to fall back and obstruct the oropharyngeal airway with risk of suffocation. * Motor innervation to the tongue comes from the Hypoglossal nerve (CN XII). CAROTID SHEATH-located at the lateral boundary of the retropharyngeal space at the level of the oropharynx on each side of the neck deep to the SCM muscle. It extends from the skull base to the first rib and sternum. It contains the carotid arteries, internal jugular vein, vagus nerve, and deep cervical lymph nodes. • During surgical procedures of the neck, structures within the carotid sheath can be retracted (pulled aside) as a unit (common carotid artery, internal

jugular vein, internal carotid artery, & vagus nerve). However, the cervical sympathetic trunk would remain in place when the carotid sheath is retracted because it is not within the sheath. • Facial vein unites with the retromandibular vein below the border of the mandible and empties into the main venous structure of the neck (internal jugular vein). • Internal jugular vein descends through the neck within the carotid sheath and unites behind the sternoclavicular joint with the subclavian vein to form the brachiocephalic vein whose right and left segments unite in the superior mediastinum to form the superior vena cava (this returns blood to the right atrium of the heart). Maxillary 1st molar is innervated by the MSA & PSA nerves (middle superior and posterior superior alveolar nerves). To extract the maxillary 1st molar, you MUST numb both the PSA & MAS nerves, and the greater (anterior) palatine nerve for palatal anesthesia (soft tissue). * PSA nerve-innervates the DB root of the 1st molar and distal to it (2nd and 3rd molars). * MSA nerve-innervates the MB root of the 1st molar and the two premolars. * ASA nerve (anterior superior alveolar nerve)innervates canines and incisors.

The canine and 1st premolar on the injection side can be removed without pain after administering an inferior alveolar and lingual nerve block. You must give a long buccal injection to extract all molars and 2nd premolar. For operative procedures, a long buccal injection may not be needed for these teeth. * Incisors may need local infiltration for extractions. * Bone of the maxilla is more porous than the mandible, thus it can be infiltrated anywhere. Posterior Superior Alveolar (PSA) nerve block-provides anesthesia for 1st, 2nd, 3rd molars, but not mucoperiosteum of the palate. PSA injection does not always anesthetize all roots of the maxillary 1st molar. Thus, if anesthesia of this tooth for restorative or extraction is required, a local infiltration injection is given over the second premolar. • Patients experience few subjective signs of anesthesia after receiving a PSA nerve block, compared to an IA nerve block (numb lip). • If the patient’s face becomes distended and swollen after a PSA block due to an intravascular injection, place cold packs and pressure on the affected side, and explain that he/she may become black and blue on that side. Greater (Anterior) Palatine Nerve-a branch of the maxillary (CN V-2) nerve that provides soft tissue

innervation to the posterior 2/3 of the hard palate. It emerges into the hard palate via the greater palatine foramen and passes forward halfway between the alveolar crest and midline. Supplies soft tissues of the palate as far anteriorly as the canine where it overlaps with nasopalatine nerve branches. It is necessary to anesthetize the GPN if extraction or surgical procedures are contemplated in this area of the palate. Anesthetize the greater palatine nerve, deposit local anesthetic at the greater palatine foramen (between the 2nd and 3rd maxillary molars ~1cm from the palatal gingival margin toward the midline). Greater Palatine Injection-used if extracting any or all of the 3 molars and to anesthetize mucoperiosteum of the palate.

TONGUE SENSORY INNERVATION: * Lingual nerve (branch of V3)→supplies anterior 2/3 of tongue. * Glossopharyngeal nerve (CN IX)→supplies posterior 1/3 of tongue (taste & sensation) & vallate papillae. Also supplies sensory to the tonsil, nasopharynx, and pharynx. * Vagus nerve (CN X) via internal laryngeal nerve→supplies the area near the epiglottis. * Facial nerve (CN VII) via chorda tympani→supplies taste to anterior 2/3 of tongue.

FACIAL NERVE (CN VII)-originates in the PONS, traverses the facial canal of the temporal bone, and exits the cranium through the stylomastoid foramen where it then extends laterally around the neck of the mandible through the PAROTID GLAND. *If you cut the facial nerve just after it exits the foramen, it causes loss of innervation to the muscles of facial expression. Branches of the facial nerve to muscles of the face are enmeshed in the parotid gland. Functions: 1. motor innervation: muscles of facial expression, posterior belly of digastric and stylohyoid muscles, stapedius muscle within the middle ear. Lower motor neuron lesions of CN 7 cause ipsilateral (same side) flaccid paralysis of the facial muscles. 2. sensory innervation (proprioception): from the same muscles of facial expression that receive motor innervation. 3. motor (parasympathetic innervation): tear secretion from the lacrimal gland and salivation from the sublingual and submandibular glands. 4. sensory (taste impulses & sweet sensation): from taste buds on the anterior 2/3 of the tongue, floor of mouth, and palate. • Bell’s Palsy-facial paralysis (a functional disorder of the

facial nerve) caused by nerve irritation or viral infection, thus is usually temporary.

ARTERIES & GLANDS EXTERNAL CAROTID ARTERY-supplies most of the head & neck (except the brain, which gets its blood supply from the internal carotid and vertebral arteries). External carotid passes through the parotid salivary gland and terminates as the maxillary & superficial temporal arteries (terminal branches). The superficial artery supplies the scalp. MAXILLARY ARTERY-supplies maxillary & mandibular teeth, muscles of mastication, palate, and almost the entire nasal cavity. 1. Inferior alveolar artery-a branch of the maxillary artery that supplies blood to the mandibular teeth. 2. PSA artery-a maxillary artery branch that supply blood to the POSTERIOR maxillary teeth. 3. ASA & MSA arteries-maxillary arch branches that supply blood to the ANTERIOR maxillary teeth. VENOUS RETURN of both dental arches is the PTERYGOID PLEXUS of veins. GREATER (Descending) PALATINE ARTERY-descends

through the greater (descending) palatine canal to the greater palatine foramen. It supplies the hard palate, gingiva of maxillary teeth, & lateral nasal wall. * GPA gives rise to the greater and lesser palatine arteries which pass through the greater and lesser palatine foramina (respectively), and supply the hard & soft palates. Lesser palatine artery also helps supply the tonsils (along with the tonsillar artery-a branch of the facial artery). * GPA sends a branch to anastomose with the nasopalatine branch of the sphenopalatine artery in the incisive foramen to supply the mucosa of the hard palate anterior to the maxillary canine. LINGUAL ARTERY-supplies blood to the TONGUE (which also receives blood from the tonsillar branch of the facial artery & ascending pharyngeal artery). • Lingual artery arises from the external carotid artery at the level of the tip of the greater horn of the hyoid bone in the carotid triangle (below the facial artery). Branches include dorsal lingual artery, suprahyoid artery, and sublingual artery (supplies the sublingual gland). It terminates as the deep lingual artery, which ascends between the genioglossus & inferior longitudinal muscles. Floor of the mouth also receives its blood supply from the lingual artery.

• Lingual artery arises from the EXTERNAL CAROTID ARTERY. It passes deep to the hyoglossus muscle to supply the tongue, and DOES NOT accompany the corresponding nerve throughout its course. Lingual artery supplies structures of the mouth floor and posterior and inferior surface of the tongue. • Lingual Artery Branches: 1. suprahyoid artery: supplies suprahyoid region. 2. dorsal lingual artery: supplies dorsum of tongue (top of the tongue). 3. deep lingual artery: the terminal branch that supplies the anterior 2/3 of tongue. 4. sublingual artery: supplies mouth floor and sublingual gland. Inferior alveolar nerve & artery, and lingual nerve are found in the PTERYGOMANDIBULAR SPACE between medial pterygoid muscle and ramus of the mandible. IA nerve passes lateral to the sphenomandibular ligament. Ophthalmic Artery-supplies blood to the orbit and its contents. Facial Trauma-trauma to the facial nerve destroys the ability to contract facial muscles on the affected side of the face and distorts taste perception. The affected side of the face sags because muscle tonus is lost.

PAROTID GLAND-the largest salivary gland and pure SEROUS gland supplied by general visceral efferent (motor) nerve fibers of the glossopharyngeal nerve. The parotids are located below and just anterior to the ear, and are divided into deep and superficial lobes with the stylomandibular tunnel (which encloses the facial nerve) being the dividing line. Thus, a portion of the parotid lies superficial to the mandibular ramus and another portion lies deep. * Parotid gland is drained by STENSON’S DUCT which pierces the buccinator muscle and crosses the masseter muscle where it opens into the vestibule of the mouth opposite the maxillary 2nd molar. * Von Ebner’s glands-the only other adult salivary glands that are PURELY SEROUS. These glands are located around circumvallate papilla of the tongue. Their main function is to RINSE FOOD AWAY FROM PAPILLA after it has been tasted by the taste buds. * Parotid gland receives its parasympathetic secretomotor innervation from the glossopharyngeal nerve via the lesser petrosal nerve, otic ganglion, and auriculotemporal nerve (branch of V3). * External carotid artery and its terminal branches within the parotid gland (superficial temporal &

maxillary arteries) supply the parotid gland. Lymphatic drainage of the parotid is through the parotid nodes to the deep cervical lymph nodes. * Mumps-a viral disease of the parotid gland. Parotitis-inflammation of the parotid gland. * An injection into the parotid gland (capsule) when attempting to administer an IA nerve block may cause Bell’s Palsy facial expression (paralysis of the forehead muscles, eyelid, and upper and lower lips on the same side of the face that the injection was given. SUBMANDIBULAR GLANDS-located submandibular triangle (digastric triangle.

in

the

* Wharton’s Duct (Submandibular Duct)-emerges from the anterior end of the deep part of the submandibular gland, and passes forward along the side of the tongue, beneath the mucous membrane of the mouth floor. It opens into the mouth on the summit of a small papilla situated at the side of the frenulum of the tongue. Clinically, the submandibular duct and deep part of the gland can be readily palpated through the mucous membrane of the mouth floor alongside the tongue. Saliva can usually be seen emerging from the duct orifice. * During its course, Wharton’s duct is closely

related to the LINGUAL NERVE which eventually crosses over the duct. This is important because if you incise the mucous membranes of the mouth floor, you can expose the lingual nerve, Wharton’s duct, and the sublingual gland. To expose the duct intraorally, only mucous membrane needs to be cut through. * Submandibular glands are innervated by parasympathetic secretomotor fibers from the FACIAL NERVE, which runs in the chorda tympani and lingual nerve (V3 branch) and synapse in the submandibular ganglion (this is the same as the sublingual glands). Blood supply to the glands comes from the FACIAL ARTERY (a branch of external carotid artery). * Lymphadenopathy is the most common cause of swelling of the submandibular triangle tissues (not cysts or sialoliths). SUBLINGUAL GLAND-the smallest salivary gland that contains mostly MUCOUS ACINI. Sublingual glands are in the floor of the mouth below the tongue, close to the midline. Mylohyoid muscle supports the individual sublingual glands inferiorly. They have many small ducts (Rivian Ducts) that open onto the mouth floor. Most of its secretory units are mucous-secreting with serous demilunes.

* Innervated by parasympathetic secretomotor fibers from the FACIAL NERVE, which run in the chorda tympani and lingual nerve (V3 branch) and synapse in the submandibular ganglion. Blood supply is from the SUBLINGUAL ARTERY (branch of the lingual artery, which is a branch of the external carotid artery). Lymphatic drainage from the sublingual & submandibular glands goes to the submandibular and deep cervical lymph nodes. * Sometimes the numerous sublingual ducts join to form a single main excretory duct (Bartholin’s Duct) that usually empties into the submandibular duct. In the head and neck, all lymph ultimately drains into the DEEP CERVICAL LYMPH NODES which form a chain along the course of the internal jugular vein, from the skull to the root of the neck. These nodes receive lymph from neighboring structures and from all other regional lymph nodes in the head and neck. The efferent lymph vessels join to form the jugular lymph trunk (this vessel drains into either the thoracic duct or right lymphatic duct). REGIONAL LYMPH NODES: 1. Parotid lymph nodes-receive lymph from a strip of scalp above the parotid salivary gland, anterior wall of

the external auditory meatus, and from lateral parts of the eyelids and middle ear. Efferent lymph vessels drain into the deep cervical lymph nodes. 2. Submandibular lymph nodes-receive lymph from the front of the scalp, nose, and adjacent cheek, upper and lower lips (except the center part), paranasal sinuses, maxillary & mandibular teeth (except mandibular incisors), anterior 2/3 of the tongue (except the tip), floor of the mouth and vestibule, and gingiva. Efferent lymph vessels drain into the deep cervical lymph nodes. • Paranasal Sinuses-a series of mucous membranelined air spaced that lighten the skull and enhance voice resonance. Located within the frontal, ethmoid, maxillary, & sphenoid bones. 3. Submental lymph nodes-receive lymph from the tip of the tongue, floor of the mouth below the tip of the tongue, mandibular incisors and associated gingiva, the center part of the lower lip, and skin over the chin. Efferent lymph vessels drain into the submandibular and deep cervical lymph nodes. MAXILLARY SINUSES-open into the HIATUS SEMILUNARIS (groove in the middle meatus of the lateral nasal cavity that contains openings of the frontonasal duct & anterior ethmoid air cells). The maxillary sinuses develop after the permanent teeth have erupted, and growth continues throughout adulthood. Maxillary sinus is

innervated by the maxillary division of the trigeminal nerve (CN V-2 which includes ASA, PSA, MSA, & infraorbital nerves). • Clinical signs of Acute Maxillary Sinusitis: severe, constant, localized pain (any unusual motion or jarring accentuates the pain). Tenderness to percussion of the maxillary posterior teeth and tenderness over the anterior sinus wall, and mucopurulent exudates also manifest. • Antibiotics that treat sinus infections: * Ampicillin: treats sinusitis due to upper respiratory infections. * Penicillin & Amoxicillin: treats sinusitis caused by odontogenic foci. Maxillary Sinus Communication: if a small communication is made with the maxillary sinus while extracting a maxillary 2nd molar, no additional surgical treatment is needed, simply allow a blood clot to form. Advise the patient to avoid vigorous mouth washing, frequent nose blowing, smoking, sucking on a straw, and violent coughing or sneezing. * Medications that can be prescribed for 1 week: antibiotics (penicillin or erythromycin), decongestant nasal spray, and/or an oral decongestant. * If the communication opening is moderate sized (2-

6mm), a figure-eight suture is placed over the tooth socket. * If the communication opening is large (7mm or larger), the opening is closed with a flap. * If a tooth or large fragment is displaced into the maxillary sinus, it should be removed. If the tooth fragment is irretrievable through the socket, it should be retrieved using a “Caldwell-Luc” approach ASAP. However, only perform this technique if you know what you are doing. If not, refer the patient to an OMS. * The integrity of the maxillary sinus floor is at greater risk with surgery involving removing a single remaining maxillary molar due to possible ankylosis. Pterygopalatine Fossa-a small space behind and below the orbital cavity that lies between sphenoid bone’s pterygoid plates and palatine bone below the orbit apex. Maxillary nerve (V-2) & artery pass through the pterygopalatine fossa. * communicates laterally with the infratemporal fossa through the pterygomaxillary fissure. * communicates medially with the nasal cavity through the sphenopalatine foramen. * communicates posteriorly with foramen lacerum

through the pterygoid canal. * communicates superiorly with the skull through foramen rotundum. * communicates anteriorly with the orbit through the inferior orbital fissure. * openings into pterygopalatine fossa: pterygomaxillary fissure, inferior orbital fissure, sphenopalatine foramen, pterygoid canal, pharyngeal canal, and foramen rotundum. * Pterygopalatine Ganglion-lies in the pterygopalatine fossa just below the maxillary nerve (V2). It receives preganglionic parasympathetic fibers from the facial nerve (CN VII) via the greater petrosal nerve. It sends postganglionic parasympathetic fibers to the lacrimal gland and glands in the palate and nose.

MUSCLES OF MASTICATION & TMJ Buccinator Muscle-a muscle of mastication that forms the muscular substance of the cheek just lateral to the teeth. The maxillary & mandibular alveolar processes, and pterygomandibular raphe are the buccinator attachments. Buccinator muscle originates from 3 areas: 1. pterygomandibular raphe-a thin, fibrous band or tendon running from the hamulus of the medial pterygoid plate down to the mandible. Pterygomandibular raphe lies between the buccinator and superior constrictor muscles. 2. maxillary & mandibular alveolar processes. Buccinator muscle INSERTS at orbicularis oris and skin at the angle of the mouth. Facial and maxillary arteries supply blood to the buccinator muscle. The buccinator’s action is to compress the cheeks against the molar teeth for sucking and blowing. When draining purulent exudates from an abscess of the pterygomandibular space from an intraoral approach, the buccinator muscle is most likely to be incised.

Lateral Pterygoids (Right & Left)-acting together are the PRIMARY PROTRACTORS of the mandible. *In addition to opening and protruding, they move the mandible from side-to-side (lateral excursions). For right lateral excursive movements, the LEFT lateral pterygoid is the primary mover (and vice versa). Thus, when a patient attempts PROTRUSION, the mandible deviates markedly to the right causing the buccinator, temporalis, and left lateral pterygoid to contract. However, the right lateral pterygoid cannot contract during protrusion. ► Mandible deviates toward the injury side in cases of: condlyar ankylosis (the most common cause of TMJ ankylosis is TRAUMA), and with a unilateral condylar fracture. Also, with a lateral pterygoid injury, the mandible deviates toward the side of injury). * A patient who sustained a subcondylar fracture on the left side is unable to deviate the mandible to the right. This is usually treated by a closed procedure involving intermaxillary fixation which immobilizes the concomitant fractures and corrects jaw displacement associated with the condylar fracture. This corrects the shift of the midline toward the side of the fractured condyle and slight premature posterior occlusion on that side.

► Mandible deviates away from the affected side of injury in cases of: condylar hyperplasia (malocclusion is a common occurrence with this injury). ► Lateral pterygoid muscles form the ROOF of the PTERYGOMANDIBULAR SPACE. Masseteric, Pterygomandibular, & Temporal spaces = Masticator Space. These spaces are bound by the muscles and fascia of mastication. Masticator space infections are almost always of dental origin, especially from the mandibular molar region. *Needle tract infections after and IA block initially involve the pterygomandibular space. • Masticator Space is formed by the splitting of the superficial layer of cervical fascia to enclose the ramus, masseter, medial pterygoid, and lower portion of the temporalis muscle. It lies largely among the muscles of mastication. • It is traversed by the mandibular nerve (V3) and maxillary blood vessels. • Infections of the zygomatic or temporal bones may pass to the masticator space, and so may abscesses from mandibular molars. • Abscesses within the masticator space may point at

the anterior aspect of the masseter muscle, either into the cheek or mouth, or they may point posterior below the parotid gland. • Clinically, the picture of the masticator space infection is dominated by TRISMUS, PAIN, SWELLING occurring within a few hours after a molar extraction, or mandibular trauma. These signs increase rapidly and peak in 3-7 days. Spontaneous intra-oral drainage usually occurs between 4-8 days. If intraoral drainage does not occur, then surgical drainage is indicated. • The MOST definite clinical sign indicating extension of an odontogenic infection into the masticator space is TRISMUS (difficulty opening the mouth due to a tonic spasm of the muscles of mastication). Trismus is also caused by passing the needle through the medial pterygoid muscle during an inferior alveolar nerve block.

TEMPOROMANDIBULAR JOINT (TMJ) TEMPEROMANDIBULAR JOINT (TMJ)-dual articulation between the condyle of the mandible and squamous portion (glenoid fossa) of the temporal bone (skull). • TMJ is a GINGLYMOARTHRODIAL JOINT (rotational and translational); it is a hinging AND sliding joint. It involves the condylar process of the mandible and glenoid (articular) fossa of the temporal bone. • During SLIDING (GLIDING), the disk and condyle move forward (protrude by lateral pterygoid) and backward (retrude by temporalis). • During HINGE (ROTATIONAL) MOVEMENT, the mandible is elevated (masseter, temporalis, medial pterygoid) and depressed (lateral pterygoid). Rotational movement occurs mainly between the disc and mandibular condyles in the lower synovial cavity. • During mandibular lateral excursions (deviation) to one side, the TMJ glides on one side and rotates on the opposite side.

• Each TMJ is covered by a fibrous capsule, and the non-articular TMJ surfaces are covered with periosteum. TMJ COMPONENTS: 1. Mandibular Condyles-elliptically-shaped mandibular bone, with its long axis oriented medio-laterally. Condyle ROTATES in the fossa, and slides forward along the articular fossa to the articular eminence (TRANSLATES); the TMJ disc normally moves with the condyle which is wrapped by the JOINT CAPSULE. 2. Articular Disc (Meniscus)-a FIBROCARTILAGE, saddle-shaped (bioconcave) tissue associated with the TMJ made of dense, irregular C.T. that SEPARATES THE CONDYLE & TEMPORAL BONE (it divides the TMJ into superior and inferior joint spaces and prevents bone-to-bone contact), and provides a smooth articulating surface. Thickest at the posterior, thinner in the center. It moves with the condyle under normal function. THE DISC IS SURROUNDED and PROTECTED by the FIBROUS C.T. JOINT CAPSULE. The disc is considered DISPLACED when it lies ANTERIOR to its usual position. • CLICKING and POPPING when opening is caused by DISK DISPLACEMENT WITH REDUCTION. • Painful and limited opening (< 30mm) WITHOUT

SOUND is caused by DISK DISPLACEMENT WITHOUT REDUCTION. Normal, full opening is 4050mm as measured from the maxillary and mandibular incisal edges. • The best diagnostic imaging tests to examine DISK POSITION and FUNCTION is MAGNETIC RESONANCE IMAGING (MRI) or CT scan. 3. Articular Capsule-thick, fibrous tissue that surrounds the TMJ and is attached above to the articular eminence (tubercle) and to the margins of the mandibular fossa and below to the neck of the mandible. Synovial Membrane-lines the articular joint capsule in the superior and inferior spaces of the joint and PRODUCES SYNOVIAL FLUID for lubrication. It does not cover the articular surfaces or articular disc. 4. Temporal Bone’s Articular Surface-composed of the concave articular fossa (glenoid fossa) and convex articular eminence (tubercle) that limits forward movement of the condyle. • SUBLUXATION (slipping of the condyle from its socket) occurs when the condyle head moves too far ANTERIORLY on articular eminence. Caused by injury to the articular capsule surrounding the TMJ. • GOMPHOSIS-a type of fibrous joint in which the conical process is inserted into a socket-like

portion, such as the styloid process in the temporal bone or the teeth in the dental alveoli. 3 TMJ LIGAMENTS: attach and determine the farthest boundaries of mandibular movement: 1. Temporomandibular ligament (lateral ligament)-runs from the articular eminence (tubercle) to the neck of the mandibular condyle. It provides lateral reinforcement for the capsule & PREVENTS POSTERIOR & INFERIOR DISPLACEMENT OF THE CONDYLE (prevents the mandible from excessive retraction or from moving backward). Resists posterior movement of the mandibular condyle, and is found on the TMJ’s lateral surfaces. 2. Stylomandibular ligament-separates the infratemporal region anteriorly from the parotid region behind. Runs from the styloid process of sphenoid bone to the angle of the mandible. Separates the parotid and submandibular salivary glands, and is taut when the mandible protrudes. 3. Sphenomandibular ligament-attaches to the spine of sphenoid bone & lingula of the mandible. Becomes taut when the mandible protrudes. A remnant of Meckel’s cartilage and a landmark when administering an IA nerve block.

TMJ SENSORY innervation is from V3 (mandibular branch of trigeminal) and bloods supply is from the EXTERNAL CAROTID ARTERY (superficial temporal branch). TMJ development occurs at 12 weeks in utero when the joint spaces and articular disc develop. Arteries that vascularize the TMJ: ♦ Middle meningeal artery (branch of maxillary artery (a terminal branch of external carotid artery). ♦ Ascending pharyngeal artery (branch of the external carotid artery). ♦ Deep auricular artery (branch of maxillary artery). ♦ Superficial temporal artery (terminal branch of the

external carotid artery). TMJ Syndrome is Divided into 3 Categories: 1. Myofascial Pain Dysfunction (MPD) Syndrome-the most common cause of TMJ pain. It is a disease primarily involving the muscles of mastication. • MPD is believed to be a stress-related disorder. An increase in stress produces an increase in mandibular muscle tension and in combination with teeth clenching causes muscle spasm, pain, and dysfunction. • MPD often responds to an acrylic night guard (occlusal separator or occlusal appliance) along with a soft diet, limited talking, and elimination of gum chewing. Moist heat applied to the face and nonsteroidal anti-inflammatory agents are also helpful during the acute phase. 2. Internal Derangement (Disc Displacement)-an abnormal relationship of the articular disc to the mandibular condyle, fossa, & articular eminence (or tubercle). The most common direction the TMJ’s articular disc can become displaced is ANTERIORLY. Internal derangement is present when the posterior band of the articular disc is anteriorly displaced in front of the condyle. As the articular disc translates anteriorly, the posterior band remains in front of the condyle and the retrodiscal tissue (bilaminar

zone) becomes abnormally stretched. Often the displaced posterior band returns to its normal position when the condyle reaches a certain point (this is anterior displacement with reduction). When the articular disc reduces the patient often feels a pop or click in the joint. ♦ Subluxation (Dislocation or Open Lock): patient opens wide or maintains open mouth for a long time, causing the posterior band to stretch and the joint to travel beyond the articular eminence (condyle is in front of the eminence). Patient cannot close (mouth is stuck open) after keeping mouth open for a long time in the dental chair. ♦ Disc Displacement WITH Reduction: “clicking joint” disc is out of place. If painless with no dysfunction (no treatment necessary), patient has normal opening or an “S” shaped opening. TMJ is only ROTATING (not translating). Reciprocal clicking on opening and closing is a sign. A reproducible reciprocal click. On closing, the disc is forward to the condyle. Disc should be between the condyle & articular eminence. ♦ Disc Displacement WITHOUT Reduction (Closed Lock): rarely clicking with occasional locking. Clicking and popping has disappeared with limited opening & pain (< 35mm). These patients

have a consistent limited opening (“hard-end feel”). Patient deviates on opening to affected side (jaw goes to the affected side). Treatment: always treat conservative (4-6 weeks first) before considering surgery (95% improve without surgery). Surgery is mainly done to restore disc position and increase opening (normal opening is 50mm). There is NOT a REPRODUCIBLE reciprocal click. Most disc displacements are ANTERIOR & MEDIAL. ♦ In some patients, the articular disc remains anteriorly displaced at full mouth opening (anterior displacement without reduction). The articular disc is reduced by inducing downward pressure on the posterior teeth and upward pressure on the chin, accompanied by posterior displacement of the entire mandible. ♦ The most common cause of restricted mandibular movement is DISC INTERFERENCE DISORDERS, which change the relationship of the disc and condyle. 2. Degenerative Joint Disease (Osteoarthritis)-natural degeneration of the TMJ’s articular surfaces. The best way to palpate the posterior aspect of the mandibular condyle is EXTERNALLY over the posterior

surface of the condyle with the mouth open. The TMJ should be evaluated for tenderness and noise. When checking for joint noises (clicking and crepitus), the joint is palpated laterally (in front of the external auditory meatus) while the patient opens and closes the mandible. * Tenderness is assessed by palpating the lateral aspect of the joints when the mouth is closed and during opening of the mouth. The joint should also be palpated for tenderness while the patient opens maximally, and the fingertip should be positioned slightly posterior to the condyle to apply force to determine if there is inflammation of the retrodiscal tissue. * By placing the fingertips in the patient’s external auditory meatus, this technique can produce false joint sounds during mandibular function because of pressure against the thin ear canal cartilage. TMJ Surgical Approaches: 1. Preauricular-the best incision to expose the TMJ. A perpendicular incision is made just anterior to the external ear parallel to the superficial temporal artery. The incision extends from 1 inch above the zygomatic arch to the lower extremity of the ear. The condyle is approached from behind. With this approach, care must be taken not to damage the facial nerve or the vessels

that richly supply this area. 2. Submandibular Approach (Risdom Approach)-this is the standard surgical approach to the mandibular ramus and neck of the condyle. Not the best approach for procedures within the joint space itself. The most common cause of TMJ ANKYLOSIS is TRAUMA. ANKYLOSIS is common with RA.

FRACTURES Radiographic views helpful to evaluate mandibular fractures: posteroanterior view, lateral oblique view, Towne view, and panoramic view. Control of airway is vital to treating any patient with a facial fracture. The maxilla and mandible are in a critical relationship to the upper airway. Thus, displacement of fractures can cause airway obstruction and cause respiratory arrest. • The highest incidence of fractures occurs in YOUNG MALES ages 15-24 usually from trauma (i.e. car accidents). 3 Muscle Groups Displace the Mandibular Condyles: 1. Masseter, medial pterygoid, & temporalis ELEVATE the mandible during mastication and cause upward displacement of the proximal segment. 2. Digastric, mylohyoid, geniohyoid, & lateral pterygoid DEPRESS the mandible and displace the distal fractured segment inferiorly and posteriorly. 3. Lateral pterygoid is responsible for forward displacement of the condylar head when the condyle neck is fractured. Anatomic Distribution of Mandibular Fractures:

* Angle-the most common anatomic site of fracture of the mandible (30% of fractures). Frequently, impacted 3rd molars are located in this region and further add to the weakness of the mandible in this area. * Condylar neck-represents 25% of fractures. It is a safety feature that allows the blow to the jaw to be dispersed at this point rather than driving the condyle into the middle cranial fossa. • Bilateral dislocated fractures of the condylar necks cause an anterior open bite and inability to protrude the mandible. • Unilateral fracture through the neck may cause forward displacement of the head of the condyle due to pull of the lateral pterygoid muscle. * Symphysis area (chin)-represents 22% of fractures, and is usually where blows are sustained. These blows often result in fractures of the subcondylar region. * Body of mandible (17%), Ramus (2%), & Coronoid Process (1%)-least common fracture site. Weak points in the mandible where fractures are most

common are the angle of the mandible, condylar neck, and symphysis area. The location and extent of mandibular fractures are determined by the direction and intensity of the blow and specific points of weakness in the mandible. * Patient’s mandible deviates to the side of injury upon opening. * Coronoid process is NOT a weak point in the mandible where fractures are common. Most common pathognomonic sign of a mandibular fracture is MALOCCLUSION. Other signs and symptoms of a mandibular body or angle fracture are lower lip numbness, mobility, pain, or bleeding at the fracture site. In a fracture involving the angle of an edentulous mandible, the proximal segment is usually displaced anteriorly & superiorly. OPEN REDUCTION-the reduction of a fractured bone by manipulation after incision into skin and muscle over the fracture site. The most common site for open reduction is at the angle of the mandible. Once the incision is made, an intraosseous wire is placed through holes made on either side of the fracture. Reduction is accomplished under direct vision, and immobilization is obtained by tightening the wires. This procedure is usually reserved for fractures that cannot be reduced and immobilized adequately by

closed methods. Best used to reduce a fracture when teeth are missing in one or more of the fractured segments. * Indications for open reduction are continued gross displacement of the bony segments and an unfavorable fracture that is likely to cause further displacement of the fractured segments caused by muscle pull. This type of reduction is commonly performed for displaced angle or body fractures. *Condylar neck fractures are usually treated by closed reduction. CLOSED REDUCTION-the reduction of a fractured bone by manipulation without incision into the skin. It is the simplest method of reduction and is used most often when both fractured segments contain teeth. After manipulation of the bone, it is usually maintained in place by intermaxillary fixation (IMF). * Intermaxillary Fixation-fixation obtained by applying wires or elastic bands between the upper and lower jaws in which suitable anchoring devices have been attached. The most common technique for IMF is the use of pre-fabricated arch bars. * Treatment of a mandibular fracture using only intermaxillary fixation (IMF) is a closed reduction because it does not involve direct opening,

exposure, and manipulation of the fractured area. Bilateral Sagittal Split Osteotomy-the most commonly performed mandibular orthognathic procedure to correct mandibular retrognathia (Class II malocclusion). When performed correctly, this technique allows for versatile, accurate, expeditious corrections of mandibular disharmonies. The mandible is split sagitally and can be used to either advance the mandible (i.e. retrognathia) or to set back the mandible (i.e. prognathia). It is the standard procedure used today. *Position of the condyle is unchanged when correcting mandibular prognathism or retrognathism. Vertical Ramus Osteotomy-used to correct mandibular prognathism. By vertically sectioning the ramus in a line from the lower aspect of the mandibular notch vertically downward over the mandibular foramen, or just posterior to the lower border of the mandible at the angle. Body Osteotomy-a procedure that involves extracting mandibular teeth bilaterally (usually premolars), a piece of bone is removed from the mandible, and everything slides back. Corrects mandibular prognathism (Class III malocclusion). FRACTURE IMMOBILIZATION: 1. Barton Bandage-the simplest form of immobilization. Primarily a first aid measure until definitive therapy can

be instituted. 2. Intermaxillary Fixation (IMF)-establishing a proper occlusal relationship by wiring the teeth together. The method successfully treats most mandibular fractures. The main methods for IMF are wiring, arch bars, and splints. This is the classical way to immobilize the fracture after closed reduction. 3. External Skeletal Fixation-used in most cases in which the management of a fractured bone segment is not satisfactorily accomplished by intermaxillary fixation. It involves placing screws or pins through the skin on each side of the fracture and a cold cure acrylic bar that holds the screws in proper relationship with the fracture in the reduced position. It is cumbersome and esthetically displeasing. 4. Direct Intraosseous Wiring combined with a period of IMF-the traditional method of bone stabilization after open reduction. This method of stabilization can be accomplished through various wiring techniques. The wire is placed through holes on either side of the fracture and immobilization is accomplished by tightening the wires. Greenstick Fracture-a mandibular fracture that extends only through the cortical portion of bone without complete fracture of the bone. It is a closed fracture involving

incomplete fractures with flexible bone, most common in children. Mandibular fractures are classified based on condition of the bone fragments at the fracture site, and possible communication with the external environment: 1. Simple fracture-divides a single bone into two distinct parts with no external communication (closed fractures with no lacerations of the oral mucosa or facial tissues). 2. Compound fracture-fracture that communicates with the outside environment (open fracture). This may occur by laceration of the oral tissues exposing the bone fragments, fracture of the maxilla into the sinuses, or via skin lacerations that would expose the fracture segments. The most common complication of an open fracture is INFECTION. 3. Comminuted fracture-multiple fractures of a single bone that can be simple or compound. The line of fracture determines if muscles will be able to displace the fractured segments from their original position. • unfavorable fracture-occurs if the fracture line results in a muscle pull displacing the fracture segment. • favorable fracture-occurs if the fracture line prevents displacement of the fracture by muscle

pull. MIDFACIAL FRACTURES-fractures affecting the maxilla, zygoma, & nasoorbital ethmoid complex. Types of midfacial fractures: 1. LeFORT I (Horizontal Fracture)-a horizontal segmented fracture of the alveolar process of the maxilla in which the teeth are usually contained in the detached portion of the bone, causing an OPEN BITE. A LeFort I osteotomy is most commonly used to correct maxillary retrognathia. 2. LeFORT II (Pyramidal Fracture)-a unilateral or bilateral fracture of the maxilla in which the body of the maxilla is separated from the facial skeleton and the separated portion is pyramidal-shaped. Signs: periorbital edema, ecchymosis, subconjunctival hemorrhage, and nose bleeding. A common finding is paresthesia over the distribution of the infraorbital nerve. 3. LeFORT III (Transverse Fracture or Craniofacial Dysfunction)-a fracture in which the entire maxilla and one or more facial bones are completely separated from the craniofacial skeleton. These patients have restricted mandibular movement. 4. Zygomatic complex fractures (most common midfacial fracture). Fractures of the facial bones (especially the zygomatic complex) may on rare

occasions be complicated by damage to the contents of the superior orbital fissure. Other possible complications of the zygomatic complex fractures: • Paresthesia-the most common complication, but usually subsides. • Antrum (sinus) may be filled with a hematoma, which usually evacuates itself. • Ocular muscle balance may be impaired because of fracture of the orbital process. 5. Zygomatic arch fractures-are nicely demonstrated by the submental vertex view. This fracture may not cause any problem other than a possible slight sinking of the cheekbone area. There may be some encroachment and impairment in closure of the jaw if it comes down and entraps the coronoid process of the mandible. The first step to treat mid-facial fractures that affects the occlusal relationship is similar to treating mandibular fractures (re-establish a proper occlusal relationship by placing the maxilla into proper occlusion with the mandible. Zygomatic bone fractures is the 2nd most common fracture of facial bones behind nasal bone fractures (most common facial bone fracture). The mechanism of injury usually involves a blow to the side of the face from a fist, object, or secondarily to motor vehicle accidents (studies show 80% of these injuries are due to motor vehicle accidents).

Signs & Symptoms of a ZYGOTMATIC FRACTURE: • Binocular diplopia (can be secondary to muscle entrapment, neuromuscular injury, or intramuscular hematoma). • Difficulty with mastication (trismus) due to masseter spasm or bony impingement of the coronoid process. • Ipsilateral epistaxis occurs in some patients to a lacerated maxillary sinus mucosa. Fracture of the infraorbital rim presents with the symptoms of numbness of the upper lip, cheek, and nose on the affected side. *Water’s view is best to evaluate orbital rim areas. Water’s view, PA skull view, & submental vertex view are radiographic views helpful to evaluate midfacial fractures. Maxillary fractures have a greater tendency to produce facial deformities than mandibular fractures. Due to the slope of the sphenoid bone comprising the cranial vault floor, blows to the maxilla drive the maxilla backwards and downwards, resulting in a potential open bite or impingement of the airway. Segmental Osteotomy-a maxillary procedure where the maxilla is sectioned into two or more pieces. Bone may or may not be removed.

FRACTURE HEALING: • Endosteal Proliferation-occurs within a bone. • Periosteal Proliferation-occurs within the C.T. covering all pones (periosteum). • Primary (Bone-to-Bone) Healing-involves both endosteal and periosteal proliferation. • Secondary Bone Healing-involves mostly endosteal proliferation into the void/space between 2 pieces of bone). The space fills in with callus. Bone Healing (3 Phases): 1. Hemorrhage: occurs first and is associated with clot organization & proliferation of blood vessels. This nonspecific phase occurs during the first 10 days of healing. 2. Callus Formation: a primary callus is formed in the next 10-20 days. A secondary callus forms in 20-60 days. 3. Functional Reconstruction: mechanical forces are important in this phase. Haversian systems are lined up according to stress lines and excess bone is removed. The shape of the bone is molded to conform with functional usage so that bone can be added to one surface, and removed from another surface. It takes 2-3 years to completely reform a fracture.

4 Reasons Fractures Do Not Heal: 1. Ischemia: navicular bone of the wrist, femoral neck, and lower third of the tibia are all poorly vascularized, thus are subject to ischemic necrosis after a fracture. 2. Excessive Mobility: healing is prevented and pseudoarthrosis or a pseudo-joint may occur. 3. Interposition of soft tissue: occurs between the fractured ends. 4. Infection: compound fractures have a tendency to become infected. *A FAT embolism is most often a sequela of fractures. 3 Types of Inappropriate Healing: 1. Delayed-union: satisfactory healing that requires greater than the normal 6-week period. May be caused by infection, interposition of soft tissue, or muscle between the fractured segments. 2. Non-union: failure of the fracture segments to unite properly. May be caused by infection, improper immobilization, or interposition of soft tissue. 3. Mal-union: can be delayed or complete union in an improper position. May be caused by improper immobilization or imperfect reduction.

GENERAL ANESTHESIA Geudel’s Stages of General Anesthesia: 1. Stage 1 (Amnesia & Analgesia): describes conscious sedation that begins with the administration of anesthesia and continues until the loss of consciousness. Respiration is quiet, though sometimes irregular, and reflexes remain present. Patient has decreased awareness of pain, sometimes with amnesia. Consciousness may be impaired, but is not lost. The best monitor of the level of analgesia is the verbal response. Altered consciousness, loss of sensory from cerebral cortex occurs. 2. Stage 2 (Delirium/Disinhibition & Excitement): begins with loss of consciousness and includes the onset of total anesthesia. The patient may move their limbs, chatter incoherently, hold their breath, or become violent. Vomiting with the attendant danger of aspiration may occur. Patient appears delirious and excited, amnesia occurs, reflexes are enhanced, and respiration is typically irregular; retching and incontinence may occur. The patient is brought to Stage 3 as quickly and smoothly as possible. Loss of consciousness-motor centers. • Respirations irregular, lateral nystagmus, pupillary reflex intact, muscle tonus decreases as this stage

progresses. Laryngeal and pharyngeal reflexes become obtunded. 3. Stage 3 (Surgical Anesthesia): begins with establishing a regular pattern of breathing, total loss of consciousness, and is the period when signs of respiratory or cardiovascular failure first appear. Patient is unconscious with no pain reflexes. Respiration is very regular and BP is maintained. This stage has 4 planes. Spinal reflexes are depressed and skeletal muscle relaxation occurs. Agents Useful for Surgical Anesthesia: * Cyclopropane: 20-35% for Stage 3 causes rapid induction and recovery, good muscle relaxant, and sensitizes the heart to catecholamines. * Halothane: 3% induction and 1-2% maintenance causes relatively slow induction and recovery. Not a good muscle relaxant. Side effects: sensitizing heart to the catecholamines, hypotension, and cardiac arrhythmias. Not a good analgesic, so used as an adjunct. Halogenated hydrocarbons are associated with liver damage if toxic doses are used. * Methoxyflurane: 1-3% causes slow induction and recovery, good muscle relaxant, sensitizes the heart to catecholamines, a respiratory depressant, and

good analgesic. 4. Stage 4 (Premortem or Medullary Depression)-signals danger (characterized by maximally dilated pupils and cold, ashen skin). BP is extremely low and often unmeasurable. Cardiac arrest is imminent. *Eyes are greatly enlarged in size and do not react to bright light when functional circulation to the brain has stopped. Patient experiences severe respiratory and cardiovascular depression/paralysis requiring mechanical & pharmacologic support. American Society of Anesthesiologists (ASA) Classification of Patient Physical Status: 1. ASA-I: normal, healthy young patient with unremarkable medical history and no systemic disease. 2. ASA-II: patient with mild systemic disease or significant health risk factor (smoking, excessive alcohol use, obesity). 3. ASA-III: patient with severe disease that is not incapacitating. 4. ASA-IV: patient with severe systemic disease that is a constant threat to life. 5. ASA-V: a moribound patient who is not expected to survive without the operation. 6. ASA-VI: patient is declared “brain-dead” and their organs are being removed for donor purposes. ELEMENTS OF GENERAL ANESTHESIA: analgesia,

relaxation, hyporeflexia, and narcosis (not hyperpyrexia = elevated body temperature). * Medulla-the last area of the brain depressed during general anesthesia. This area is the most vital part of the brain and contains the cardiac, vasomotor, and respiratory centers of the brain. * The most reliable sign of “oxygen want” while monitoring a patient during general anesthesia is increased pulse rate. Cyanosis may also be present. * The emergency most often experienced during outpatient general anesthesia is respiratory obstruction. * The best anesthetic technique used in oral surgery to avoid aspiration of blood or other debris when a patient is under general anesthesia is endotracheal intubation with pharyngeal packs. * A patient with an acute respiratory infection is contraindicated for general anesthesia. * The eyes are taped shut prior to draping a patient before surgery to prevent corneal abrasion. INDUCTION-phase of anesthesia that begins with the administration of anesthetic and continuing until the

desired level of patient unresponsiveness is reached. The depth of general anesthesia by inhalation varies with the partial pressure (tension) of the anesthetic agent in the brain, and the rates of induction and recovery depend on the rate of change of tension in this tissue, blood supply to the lungs, pulmonary ventilation, and concentration of the anesthetic influence the induction rate. The signs and stages of anesthesia are most likely seen with an anesthetic with a slow rate of induction. Maintenance-process of keeping a patient in surgical anesthesia. RECOVERY-phase of anesthesia commencing when surgery is complete and delivery of the anesthetic is terminated, and ending when the anesthetic has been eliminated from the body. Dissociative Anesthesia-a unique method of pain control that reduces anxiety and produces a trance-like state where the person is not asleep, but feels separated from their body. Used in emergency situations (i.e. injury/trauma) and can be used for short, painful procedures (i.e. changing bandages). This method is safe and lasts only a short time. Since the person does not usually remember the procedure, dissociative anesthesia is useful in CHILDREN. A person who received dissociative anesthesia usually does not remember the procedure, especially if a sedative was given with the pain medication.

Most people feel normal within a few hours. As the medication wears off, an individual may have intense dreams or hallucinations. • KETAMINE is the primary medication used in dissociative anesthesia, but a sedative is often given before the Ketamine to reduce anxiety. A single dose of ketamine produces a trancelike state for 1030 minutes, and pain control for 30-45 minutes. The patient’s eyes are open during the procedure, but he/she is in a daze and feels no pain. * Ketamine ↑ secretions of salivary and bronchial glands, BP, HR, and muscle tone (not respirations). * Side Effects: hypertension, increased pulse, delirium. General Anesthetics most commonly used are inhalation alone, barbiturates alone, barbiturates with oxygen, and nitrous in combination with a more potent agency like Halothane. Also, a local anesthetic is sometimes used for vasoconstriction and to decrease the amount of barbiturate used in lengthy procedures. * The most effective agent in initial treatment of respiratory depression due to overdose of barbiturates is OXYGEN under positive pressure.

* The behavior of patients under GENERAL anesthesia suggests that the most resistant part of the CNS is the MEDULLA OBLONGATA (cardiac, vasomotor, and respiratory centers of the brain). The most controllable route of administration of a general anesthetic is inhalation. Barbiturates DEPRESS the CNS, which lasts 3-8 hours depending on the dose. The barbiturates of choice for the dentist are usually the short-acting drugs because the onset of sedation is short (~30 minutes to 1hr), and the duration of effect (3-4 hours) is more than sufficient for most dental procedures. * Barbiturates have two major effects: sedative (decreases anxiety) and hypnotic (helps sleep). * Drugs to avoid in patients taking barbiturates: phenothiazines, alcohol, antihistamines, & antihypertensives. These drugs enhance the CNS depression of barbiturates. * After IV administration of an ultra-short acting barbiturate (i.e. Brevital or Pentothal), the last tissue to become saturated as a result of redistribution is FAT (because it is not as vascular as liver, brain, and muscle tissue). * In low doses barbiturates reduce anxiety, respiration,

BP, HR, and REM sleep. * In higher doses, barbiturates can increase some types of behavior and act like a stimulant. These effects may be caused by depression of inhibitory brain circuits (barbiturates at these doses act to remove inhibitory behavior). * Barbiturates can lead to excessive sedation and cause anesthesia, coma, even death. Barbiturate overdoses may occur because the effective dose of the drug is not too far away from the lethal dose. * Properties of Barbiturate Hypnotic Agents: respiratory depression, induction of liver microsomal enzymes, tolerance development, & suppression of REM sleep. Barbiturates can cause hyperanalgesia (↑ sensitivity to pain). Brevital (Methohexital)-the drug most commonly used to attain GENERAL anesthesia (an IV barbiturate prescribed to induce anesthesia in short surgical procedures as a supplement to other anesthetics. • Brevital is metabolized in the liver and excreted by the kidney. • General anesthesia induction and recovery using Brevital is rapid.

• Hiccoughs is the most common side effect due to rapid injection of Brevital. • A primary advantage of IV sedation is the ability to titrate individualized dosages. Malignant Hyperthermia-an autosomal dominant inherited condition and life-threatening, acute pharmacogenetic disorder occurring in patients undergoing general anesthesia. Classic MH usually manifests in the operating room, but can also occur within the first few hours of recovery from anesthesia. Characterized by a sudden, rapid rise in body temperature associated with signs of increased muscle metabolism (i.e. tachycardia, tachypnea, sweating, and cyanosis, increased CO2 production, and muscle rigidity). * Usually occurs in apparently healthy children and young adults at an average age of 21 years (equally in males & females). * When MH is diagnosed early and treated promptly, the mortality rate should be near zero. Whenever anesthesia is administered, Dantrolene should be readily available and the protocol for MH management (100% oxygen, cooling procedures, and correcting acidosis and hyperkalemia). Dantrolene is currently the only known drug that treats MH by impairing calcium-dependent muscle contraction,

and controlling hypermetabolism manifestations. * People with MH are informed of their condition and advised that 50% of their first-degree relatives are likely to have the trait. SEDATION INDICATIONS: apprehensive or scared patients, decrease stress in a medically compromised patient, or to perform several procedures on a patient who is extremely anxious. The optimum site for IV sedation for an outpatient is the MEDIAN CEPHALIC VEIN which lies in the lateral aspect of the antecubital fossa (anterior to the elbow). Avoid entering the brachial artery to avoid immediate burning at the injection site (arm appears blotchy, and the arm pulse is weak compared to the other arm). • IV sedation is usually done with a 21-gauge needle with a Valium (Diazepam). The rate of injection of valium is 1ml/minute (1ml of injectable valium contains 5mg of valium. The injection is discontinued when the eyelids droop (ptosis). • 3 common signs that indicate the correct level of sedation has been reached with Valium are blurring of vision, slurring of speech, and 50% ptosis of the eyelids (Verrill’s sign). Phlebitis (Thrombophlebitis) of a vein after administration of IV valium is usually attributed to the presence of

propylene glycol in the mixture. Phlebitis is irritation or inflammation of a vein sometimes seen after IV administration of valium due to the propylene glycol in the mixture. Phlebitis is more likely to occur if a vein in the hand or wrist is used and may be more common after repeated injection, especially in heavy smokers, elderly, and women taking oral contraceptives. Treatment: elevate the limb, apply moist heat, and possibly use anti-coagulants. Clinical Observations of Phlebitis: ♦ Vessels feel hard, thready, or cord-like. ♦ It is extremely sensitive to pressure. ♦ The surrounding area may be erythematous and warm to touch. ♦ The entire limb may be pale, cold, and swollen. Scopolamine-a drug structurally similar to Acetylcholine. Scopolamine acts by interfering with the transmission of nerve impulses by acetylcholine in the PNS and produces symptoms typical of parasympathetic system depression (dilated pupils, rapid heartbeat, dry skin and mouth, and dry respiratory passages). • Scopolamine is very effective for preventing MOTION SICKNESS and this indication is its most common clinical use. Since scopolamine depresses the CNS, it is used as a sedative before anesthesia and as an anti-spasmodic in certain disorders characterized by restlessness and agitation (i.e.

delirium, tremors, psychosis, mania, and Parkinsonism). • Premedication with Scopolamine prolongs certain effects (similar to premedication with Morphine) like amnesia, psychic sedation, and decreased salivation. • Reduction of secretions occurs by competitive blockade of acetylcholine and other cholinergic stimuli at cholinergic receptor sites on salivary and bronchial glands. Antagonism of acetylcholine on the sphincter and ciliary body in the eye causes mydriasis (dilation of pupils). Anticholinergic Drugs-interfere with binding of acetylcholine at its receptor. The most common method for categorizing anti-cholinergic drugs is to identify the ionization state of nitrogen (i.e. tertiary or quaternary) because this affects the drug’s ability to penetrate the CNS. * Atropine, Benztropine, Scopolamine, Dicyclomine, Trihexyphenidyl are tertiary compounds (these penetrate the CNS more readily than quaternary (i.e. ionized) compounds. However, at normal doses Atropine penetrates the CNS poorly. Atropine is contraindicated for nursing mothers and patients with glaucoma. * Scopolamine, Atropine, and Benztropine decrease

saliva flow and secretion from respiratory glands during general anesthesia. * Glycopyrrolate, Ipratropium, & Probanthine are quaternary compounds that cannot penetrate the CNS. * Principal therapeutic uses of anti-cholinergic drugs in dentistry: ► Decrease saliva flow during dental procedures (anti-sialogue) and the secretion from respiratory glands during general anesthesia. Atropine is the most commonly used anti-cholinergic drug for these purposes. Scopolamine penetrates the CNS more readily than Atropine, but is rarely used. ► A patient pre-medicated with Atropine will exhibit mydriasis (dilated pupils). AIR VOLUMES: 1. Tidal Volume (TV)-volume of air normally inhaled or exhaled by quiet breathing. 2. Residual Volume (RV)-volume of gas in the airway that does not participate in ventilation. 3. Expiratory Reserve Volume (ERV)-amount of air that can be exhaled in addition to the TV.

4. Inspiratory Reserve Volume (IRV)-amount of air that can be inhaled in addition to the TV. 5. Vital Capacity (VC)-total lung capacity. VC = TV + ERV + IRV. 6. Functional Residual Capacity (FRC)-the amount of air remaining in the lungs at the end of a normal expiration. This air is used to provide air to alveoli to aerate the blood evenly between breaths. A larger than normal functional residual capacity (FRC) causes N2O sedation to take longer. FRC = ERV + RV. Pulmonary volumes and capacity are 20-25% less in females than males, and are greater in large and athletic people. N2O sedation varies accordingly. Laryngospasm-sudden acute spasm of the vocal cords and epiglottis that can result in airway occlusion and death. A patient under general anesthesia loses the laryngeal reflex. If blood and saliva collect near the vocal cords, this stimulates the patient to go into spasm (laryngospasm) and the vocal cords close. When this happens, air cannot pass through. The two most important steps in initial management of a laryngospasm are applying oxygen under positive pressure and administering succinylcholine (a skeletal muscle relaxant used when performing endotracheal intubation and endoscopies).

UNIVERSAL sign of laryngeal obstruction is STRIDOR (crowing sounds). Stridor is a high-pitched, noisy respiration, like blowing of the wind. It is a sign of respiratory obstruction, especially in the trachea or larynx. * Because total airway obstruction usually occurs during inspiration, there is usually adequate oxygen left in the cerebral blood to permit up to 2 minutes of consciousness. If the obstruction is not recognized, managed, and oxygen is not delivered to the victim’s lungs, blood, and brain, permanent neurologic damage occurs in 3-5 minutes. * Non-invasive Procedures for Obstructed Airway: back blows, manual thrusts, Heimlich maneuver, chest thrust, and finger sweep. * Invasive Procedures for Obstructed Airway: these procedures are only performed by people trained in these techniques and with the proper equipment: ► Tracheotomy-used more for long-term airway maintenance, not for emergency airway. ► Cricothyrotomy-a procedure to establish EMERGENCY AIRWAY when other methods are unsuitable or impossible. The access site is the cricothyroid membrane of the trachea, located on the anterior neck between the

cricoid and thyroid cartilages. * A cricothyrotomy may be life-saving in an anaphylactic reaction where a patient shows signs of laryngeal obstruction. If a patient shows signs of laryngeal obstruction (stridor/crowing sounds), epinephrine and oxygen is administered. * If a patient loses consciousness and is unable to breathe, an emergency cricothyrotomy may be required to bypass the laryngeal obstruction. Post-Operative Hypotension may be due to the effect of transfusion reactions, fat embolism, anesthetic or analgesics on the myocardium (the most common cause), liver failure, or anaphylaxis. Treatment: Narcan (narcotic antagonist) if hypotension is due to narcotics. Use Atropine (anti-cholinergic) if bradycardia is present. Post-Operative Hypertension-most often due to postoperative pain, so treat with narcotics and sedatives. It is also caused by hypercarbia (> normal CO2 amounts in the blood), or administration of a vasopressor or catecholamine agents.

LOCAL ANESTHESIA & COMPLICATIONS Local anesthetics are MOST EFFECTIVE in tissues above a pH 7 (alkaline). Local anesthetics are alkaloid bases combined with acids to form water-soluble salts. A pH > 7 causes hydrolysis of the anesthetic salts. The potential action of all local anesthetics depends on the anesthetic salt’s ability to liberate the free alkaloidal base (the non-ionic lipophilic molecule). The potency of local anesthetics increases with increasing lipid solubility. * As the pH of the tissue decreases and [H+] increases, the cationic (water-soluble) form rises and free base decreases. Conversely, as pH is increases and [H+] is decreased, the free base (fat-soluble) form increases and the cationic form decreases. This free base form readily penetrates the lipid-rich nerve. * Inflammation and infection cause tissues to become acidic. The cationic (water-soluble) form of the anesthetic predominates (there is less free base available). Thus, the penetration through the membrane is decreased, giving the anesthetic poor effectiveness. * Local anesthetics affect the nerve membrane by

DECREASING the membrane’s permeability to Na+ and DECREASING the membrane’s excitability. Local anesthetics bind to inactivation gates of fast voltage gated sodium channels, stabilizing them in closed position, effectively prolonging the absolute refractory period. This decreases Na+ membrane permeability, thus reducing membrane excitability. When membrane excitability is reduced below a critical level, a nerve impulse fails to pass through the anesthetized area. • K+, Ca2+, & Cl- conductances are unchanged. • Local anesthetics “reversibly block” nerve impulse conduction and produce reversible loss of sensation at their administration site. The action site of local anesthetics is at the lipoprotein sheath of nerves. • Small, myelinated nerve fibers that conduct pain and temperature sensations are affected FIRST, then touch, proprioception, and skeletal muscle tone. • Local anesthesia works by reducing anxiety and sensitivity during the procedure. Local anesthesia acts by reducing sensitivity which thus reduces anxiety and stress related to treatment. Salivation is also decreased.

MAXIMUM allowable dose of 2% lidocaine with 1:100,000 EPI is 3.2mg lidocaine/ per lb. (or 7mg/kg). *1kg = 2.2 lbs. * Ex: 70lb patient x 3.2mg = 224mg (maximum allowable dosage) / 36mg (amount of lidocaine in 2% carpule) = 6.2 carpules (~6). * For Carbocaine without EPI, the maximum allowable dose is 3.0mg/lb. * Maximum allowable dose of EPI that can be administered to a cardiac-risk patient is 0.04mg. In terms of local anesthetics, this is equivalent to either: ► 1 cartridge (1.8cc) with anesthetic concentration of 1:50,000. ► 2 cartridges (3.6cc) with anesthetic concentration of 1:100,000. ► 4 cartridges (7.2cc) with anesthetic concentration of 1:200,000. 0.018mg of EPI are in each cartridge/carpule (1.8cc) of 2% lidocaine with 1:100,000 EPI. 0.018mg = 1.8cc x .01mg EPI. • 1cc of 2% lidocaine with 1:100,000 EPI contains: * 20mg of lidocaine, 0.01mg EPI, 6mg NaCl, 0.5mg sodium-metabisulfate (preservative to stabilize EPI).

* 1mg of methylparaben (a preservative) and NaOH to stabilize the pH. • 1.8cc of 2% lidocaine (1 carpule) with EPI 1:100,000 contains: * 36mg lidocaine (1.8 x 20mg), 0.18mg EPI (1.8 x .01mg), 10.8mg NaCl (1.8 x 6mg). * .90mg sodium-metabisulfate (1.8 x 0.5), 1.8mg of methylparaben (1.8 x 1mg), and NaOH to stabilize the pH.

ESTER LOCAL ANESTHETICS ESTERS: are metabolized in BLOOD PLASMA. Esters are metabolized by the plasma enzyme “plasma cholinesterase” or “pseudocholinesterase” which splits the ester linkage within the chemical structure rendering the anesthetic ineffective. • All esters have an “ester” grouping them within their chemical structure. An ester grouping is a bridge or link containing the –COOCH2- configuration. Drugs with 1 letter “i” in the name. • Due to their potential to cause allergic reactions, ESTER LOCAL ANESTHETICS ARE MAINLY USED ONLY AS TOPICALS (Benzocaine 20% is the most common topical). ALL TOPICAL ANESTHETICS are ESTERS except for 5% lidocaine (an amide). • Other ester anesthetics: Procain (Novocaine), Tetracain (Pontocaine), Propoxycaine (Ravocaine), Dibucaine). • Ester local anesthetics are no longer available as dental anesthetic injections due to their relatively HIGH ALLERGIC REACTION POTENTIAL & INCIDENCE.

• BENZOCAINE is the MOST COMMON TOPICAL ANESTHETIC used in dentistry. Because some local anesthetics are rapidly absorbed when applied to mucous membranes, prevent toxicity reactions by using only the needed amount of topical GEL (avoid topical sprays since they have a higher incidence of toxicity reaction). • PROCAINE (NOVOCAINE)-an original ester-type local anesthetics. When metabolized by plasma cholinesterase, a highly allergic compound “Paraaminobenzoic Acid” (PABA) was formed. Not used today in dentistry because many patients developed an allergy to PABA. Hydrolysis of procaine occurs mainly in plasma. • COCAINE-an addictive CNS stimulant and only local anesthetic that increases the vasoconstriction activity of EPI & NE. NOT routinely used in dentistry. Cocaine is a stimulant, anesthetic, and vasoconstrictor. Snorted as a powder, smoked as “crack”, or injected to produce symptoms for 5-30 minutes.

AMIDE LOCAL ANESTHETICS AMIDE LOCAL ANESTHETICS: all are metabolized in the LIVER (except Articaine), and the metabolites are then renally excreted. All “ have an “amide” grouping within their chemical structure. An amide grouping is a bridge or link containing the –CONHCH2- configuration. Amides are present in urine Amides are the only local anesthetics presently available as dental injectables (Lidocaine (Xylocaine), Prilocaine (Citanest), Bupivacaine (Marcaine), & Mepivacaine (Carbocaine), Etidocaine, & Articaine). All drugs with two letter “i” in the name. Amides are metabolized by the “hepatic microsomal enzyme system”, and the products formed to not have anesthetic actions and are excreted from the body by the KIDNEY. • Amides are used with caution, or not at all in patients with compromised liver function. • AMIDES have a longer duration of action, and are metabolized by P450 enzymes in the liver so toxicity is more likely if administered to individuals with liver dysfunction or with other drugs that may alter hepatic metabolism. • Allergic reactions to amides are RARE, but if they occur are usually caused by the PRESERVATIVE in

the anesthetic solution. There is typically NO cross allergenicity, although Lidocaine and Mepivacaine may show cross-allergy. • Amides are INACTIVATED BY MONOAMINE OXIDASE ENZYME. LIDOCAINE (XYLOCAINE): the most common local anesthetic used in dentistry, may be found in topical anesthetic agents, and an IV ANTI-ARRHYTHMIC AGENT to effectively treat (reverse) life-threatening ventricular arrhythmias to decrease cardiac excitability. • Lidocaine 2% with 1:100,000 EPI (vasoconstrictor) provides profound anesthesia of medium duration. • Lidocaine is SAFE during pregnancy and lactation. • LIDOCAINE is the local anesthetic that may manifest its toxicity clinically by initial depression and drowsiness (rather than stimulation and convulsion). The initial effect on the brain for local anesthetics is usually stimulation, then depression. However, it is possible that the excitatory phase of the reaction may be extremely brief, or may not occur (this is true especially with lidocaine and mepivacaine) causing patients to feel drowsiness. *Lidocaine & Mepivacaine can also show crossallergenicity.

• Usually, the FIRST clinical sign of mild lidocaine toxicity is NERVOUSNESS. Mild toxicity can be caused by an intravascular injection, unusually rapid absorption, or too large a total dose of the local anesthetic. Clinical manifestations of a mild lidocaine toxicity related to CNS excitation: Nervousness (increased anxiety), talkativeness, muscular twitching, perioral numbness, ↑ HR, BP, RR. • Lidocaine can skip the excitatory phase and go straight to the depression phase (drowsiness). • If the clinical manifestations do not progress beyond these signs with retention of consciousness, no definitive therapy is needed. The lidocaine will undergo redistribution and biotransformation, and the blood level will fall below the toxic level in a short time. • Treatment of a sustained convulsive reaction to a local anesthetic includes oxygen and Diazepam IV. If proper equipment and adequately trained staff are unavailable, do not attempt injections. • Possible side effects of lidocaine systemic absorption (not necessarily toxic levels) are tonicclonic convulsions, respiratory depression, and decreased CO.

ARTICAINE (SEPTOCAINE)-the only amide local anesthetic metabolized in the BLOODSTREAM (not in the liver). Chemically unique because its ester group is attached to a molecule that can be acted upon by plasma cholinesterase to make it ineffective. • Approved in the U.S. in 2000, but controversy within U.S. dentistry remains due to potential increased incidence of paresthesia with inferior alveolar nerve (mandibular) blocks. • Supplied as articaine HCL 4% solution with EPI 1:100,000 concentration. Comes in a lower dose due to its increased potency. • Indicated for local, infiltrative, or conductive anesthesia in simple and complex dental and periodontal procedures. The onset of anesthesia after administration is 1-6min after injection. Complete anesthesia lasts about 1 hour for local infiltration and 2 hours for nerve blocks (longer duration than Prilocaine, but shorter than Marcaine). • Contraindications: patients with hypersensitivity to local anesthetics containing sodium bisulfite, patients with impaired cardiovascular function, or with congenital or idiopathic methemoglobinemia. HEADACHE & PAIN are the most common adverse reaction (2%).

• Maximum dose of Articaine (Septocaine) recommended in one appointment is expressed as mg/kg body weight (not as total mg). 7mg/kg is the maximum recommended dose of Articaine in children and adults. In a typical kg adult male, the 7mg/kg dose = 490mg. Thus, the maximum recommended amount of Articaine for a 70kg adult in one appointment is 490mg. PRILOCAINE (CITANEST, CITANEST FORTE)-a local anesthetic amide used for nerve block, epidurals, and regional anesthesia. • Compared to lidocaine in equal amounts, Prilocaine has an intermediate duration of action that is slightly LONGER, LESS POTENT, and produces LESS vasoconstriction due to its lower EPI content of (1:200,000) compared to lidocaine (1:100,000). • Prilocaine is available as a 4% solution with or without EPI, which prolongs the anesthetic effect. Prilocaine is 50% as toxic as Lidocaine, since methemoglobinemia is a possible reaction. • Prilocaine is not used for patient with hypoxic conditions (oxygenation problems) or patients with Hepatic (liver) disease. Prilocaine is metabolized into orthotoluidine (a product than can produce methemoglobinemia, a condition

characterized by increased levels of methemoglobin in the blood which is less effective then hemoglobin in carrying oxygen in the blood. Thus, Prilocaine is contraindicated in patients with methemoglobinemia (a condition in which the iron in hemoglobin does not carry oxygen effectively), sickle cell and other anemias, and patients taking Tylenol (Acetaminophen). BUPIVACAINE (MARCAINE)-has the LONGEST DURATION OF ACTION of any dental amide anesthetic available. May be used with EPI. Good for extended procedures and when post-operative pain is expected. The long duration of effect does increase risk of systemic absorption & toxicity. Radiotoxic in some patients and used with caution if cardiovascular disease, elderly, or pediatric population. Often used in labor & procedures where motor control is essential because exhibits a strong preference for sensory fibers & is long-acting. MEPIVACAINE (CARBOCAINE)-equal to lidocaine in efficacy, but HAS THE SHORTEST DURATION OF ACTION of all amides. Not best (less useful) for dental procedures more than 25-30 minutes (short effects). Can be used without EPI (plain), or with EPI vasoconstrictor Levonordeferin (neo-cobefrin). Ineffective topically. TOXIC TO NEONATES, so avoid for labor and infants.

Inadvertent intravascular injection of a local anesthetic with a vasoconstrictor (EPI) may cause clinical signs of nervousness, tremors, dizziness, blurred vision, and excitation and/or depression of the CNS. These signs may be followed by drowsiness, convulsions, unconsciousness, and possible respiratory arrest. * Injections should always be made slowly with aspiration to avoid intravascular injection and prevent a systemic reaction to the local and vasoconstrictor used in many solutions. The presence of a vasoconstrictor does not prevent an intravascular injection or systemic absorption. The acute intravenous toxicity of a local anesthetic with a vasoconstrictor may be higher than that of the anesthetic agent alone. If drowsiness is apparent after administration of a local anesthetic, then the reaction is probably due to the toxic effect of the anesthetic, rather than a psychogenic reaction.

* In local anesthesia, depression of respiration is a manifestation of the toxic effects of the solution. Reasons Vasoconstrictors (EPI) are placed in Local Anesthetics: 1. Prolong the duration of action of the local anesthetic (most important reason). 2. Reduce the toxicity because less local anesthetic is necessary 3. Reduce the rate of vascular absorption by causing vasoconstriction. 4. Help make the anesthesia more profound by increasing the concentrations of the local anesthetic at the nerve membrane. Vasoconstrictors do NOT reduce the chance of developing an allergic reaction to the local anesthetic.

Vasoconstrictors act at ALPHA RECEPTORS to produce constriction of arterioles. Cocaine acts as an intrinsic vasoconstrictor that increases the PRESSOR ACTIVITY of both EPI & NE.

Local anesthetics depress small, non-myelinated nerve fibers FIRST, and depress large, myelinated nerve fibers LAST. Variations in susceptibility of nerve fibers to local anesthetics depend on nerve diameter and distance between the nodes of Ranvier. Clinically, the general order of loss of nerve function from a local anesthetic is: 1. pain 2. temperature (cold and warmth). 3. touch & pressure. 4. proprioception. 5. skeletal muscle tone (motor). Nerves regain their function in the reverse order. EPINEPHRINE-drug of choice to manage acute allergic reaction involving bronchospasm (acute narrowing of the respiratory airway) and hypotension.

INFERIOR ALVEOLAR NERVE BLOCK COMPLICATIONS During an inferior alveolar nerve block injection, the needle passes through the mucous membrane & buccinator muscle, and lies lateral to the MEDIAL PTERYGOID MUSCLE. If the needle accidentally passes posterior at the level of the mandibular foramen, it penetrates the parotid gland causing the patient’s cheek to feel numb (patient may develop paralysis of muscles of facial expression CN VII). • If the needle tip is resting well below the mandibular foramen, it penetrates the medial pterygoid muscle. • Trismus is most likely caused by irritation of the medial pterygoid muscle during an inferior alveolar nerve block. • After an inferior alveolar nerve block or mental nerve block, a prickly or tingling sensation (paresthesia), or complete numbness in the lower lip may result and persist for a considerable time due to direct trauma or piercing of the nerve trunk by the needle. This happens more often in a mental nerve block injection. The paresthesia symptoms gradually diminish (may last from two weeks to 6 months), but there is usually

a complete recovery. • The most common cause of paresthesia of the lower lip is extracting a mandibular 3rd molar (especially horizontally impacted 3rd molars). Psychogenic Reaction-caused by psychological factors (rather than physical factors like drugs). Signs: nausea, pallor and cold perspiration, widely dilated pupils, eyes rolled up, and brief convulsions. • Vasovagal syncope-a psychogenic reaction and the most commonly experienced complication associated with using local anesthetic solutions. The clinical signs closely resemble shock. These psychogenic reactions readily respond to placing the patient in a supine position. Practice Techniques that may prevent a vasovagal syncopal reaction after local anesthetic admin: * Slowly injecting the anesthetic solution. * Watching the patient’s color change during the injection. * Using a topical anesthetic prior to administration of the local anesthetic. * Using a low concentration of vasoconstrictor. * Premedicating extremely anxious patients. * Sympathetic, but confident handling of the patient. * Proper patient preparation.

The most common cause of transient loss of consciousness in the dental office is vasovagal syncope due to a series of cardiovascular events triggered by emotional stress brought on by the anticipation of or delivery of dental care. Any signs of an impending syncopal episode should be quickly treated by placing the patient in a FULLY SUPINE POSITION, or a position where the legs are elevated above the level of the heart (TRENDELENBURG POSITION), and by placing a cool, moist towel on the forehead. The most common early sign of syncope is PALLOR (paleness). Trendelenburg’s Position-a position where the patient is on an elevated and inclined plane (~45°) with the head down and legs and fee over the edge of the table. It is used in abdominal operations to push abdominal organs toward the chest, and is usually used to treat shock (i.e. anaphylaxis reactions). However, if there is an associated head injury, the head should not be kept lower than the trunk.

NITROUS OXIDE NITROUS OXIDE-inhalation anesthetic with the FASTEST ONSET of action. It is a colorless, non-irritating gas with a pleasant, mild odor and taste. It has a blood/gas partition coefficient of 0.47, thus is poorly soluble in blood. It is excreted unchanged by the lungs. N2O is stored under pressure in steel cylinders painted blue (oxygen is stored in green tanks). N2O is the oldest gaseous anesthetic used today, and the only inorganic substance used as an anesthetic. Nitrous oxide is a great drug to reduce anxiety. Nitrous oxide is a gas used as an ANESTHETIC in SURGERY to provide light anesthesia and is delivered in various concentrations with oxygen. Nitrous oxide alone does NOT provide deep enough anesthesia for major surgery, so it is supplemented with other anesthetic agents. It is often given for induction of anesthesia, preceded by the administration of a barbiturate or analgesic narcotic. Induction and recovery are rapid. *Sedation can be reversed rapidly when using inhalation as the route of administration of drugs. • Advantages of N2O Analgesia: good analgesia, nonflammable, suitable for all ages and therapeutic for many medically compromised patients, has

virtually no adverse effects in the absence of hypoxia, titratable, and produces euphoria. • Disadvantages of N2O Analgesia: misuse potential with patients and dentists, is not a complete pain reliever (a local anesthetic is still required for most dental procedures), nausea is the most common patient complaint, and diffusion hypoxia can occur (so give 100% oxygen at the end of dental procedures to prevent it). *Inhalation of 100% oxygen is contraindicated in a patient with COPD (chronic obstructive pulmonary disease). Nausea is the most common SIDE EFFECT of nitrous oxide analgesia. Primary disadvantage of N2O as a general anesthetic is its LACK OF POTENCY. The most common complication of N2O sedation is a behavioral problem (laughing, giddy). N2O inhibits methionine synthetase which is required for vitamin B12 production (can cause fatigue). • N2O Contraindications: * Hypoxemia: abnormal deficiency of oxygen in arterial blood. * Respiratory disease: emphysema, asthma, upper respiratory obstruction. * Emotional instability and contagious

diseases because cannot sterilize the entire tube. * N2O is acceptable for a pregnant patient, but from a risk management point it may be prudent not to use nitrous oxide on any pregnant patient. • N2O works on the CNS, and is the only INORGANIC gas used by the anesthesiologist. Room air contains 21% oxygen (so the patient MUST receive at least this much oxygen). Maximal safe concentrations of N2O (70% nitrous & 30% oxygen) produce intoxication, analgesia, and amnesia. • N2O is carried in the bloodstream in physical solution. There is no metabolism or degradation of N2O in the body. Rather, it is excreted solely by the lungs, unchanged. High blood levels of N2O can be achieved quite quickly. N2O is non-toxic to body tissues (the only toxicity using N2O is the lack of oxygen that can result from operator error). The gag reflex is only slightly obtunded with N2O analgesia. • N2O has its main effects on the RETICULAR ACTIVATING and LIMBIC SYSTEMS. • First symptom of nitrous oxide analgesia is TINGLING OF THE HANDS.

• Correct total liter flow of nitrous oxide/oxygen is determined by the amount needed to keep the reservoir bag 1/3-2/3 full. • Nitrous oxide is very appropriate for children who are fearful and timid. • The difference between conscious sedation vs. general anesthesia with respect to patient response is the patient retains all reflexes under conscious sedation, but not under general anesthesia. Neuroleptic + Narcotic analgesic + N2O→Neurolept Anesthesia (a state of neurolept anesthesia & unconsciousness produced by combined administration of a narcotic analgesic and neuroleptic agent with inhalation of N2O & oxygen). • The induction of anesthesia is slow, but consciousness returns quickly after nitrous oxide inhalation is stopped. Neuroleptic agent (Droperidol) + Narcotic analgesic (Fentanyl)→Neurolept Analgesia (conscious). Under the influence of this combination, the patient is sedated and demonstrates psychic indifference to the environment, but remains conscious and can respond to questions and commands. Neurolept analgesia only produces an unconscious state if nitrous oxide is also administered.

Nitrous Oxide & Ethylene are useful ONLY for sedation & analgesia. 1. N2O: 10-20% (maximum 35%) causes rapid induction and recovery. *Ventricular fibrillation is LEAST LIKELY to occur during anesthesia with nitrous oxide. The primary danger associated with using N2O anesthesia in concentrations exceeding 80% is HYPOXIA. 2. Ethylene: 25-35% causes rapid induction & recovery, not often used due to explosiveness and disagreeable odor. Agents currently used in INHALATION ANESTHESIA are nitrous oxide (a gas) and several easily vaporized liquid halogenated hydrocarbons (Halothane, Desflurane, Enflurane, Isoflurane, Sevoflurane, & Methoxyflurane). These anesthetics are liquid at room temperature and are vaporized in precisely controlled concentrations in a metered stream of oxygen and N2O. Inhalation anesthetics are absorbed and primarily excreted through the lungs. * Desflurane-an inhalation anesthetic with a low blood: gas partition coefficient, but is not used to induce anesthesia because of its pungency (irritates the airway) which causes patients hold their breath. The pungency of desflurane leads to a high incidence of coughing, bronchospasm, and can cause centrally mediated tachycardia and ↑BP.

* They are administered as GASES (their partial pressure or “tension” in the inhaled air or in blood or tissue is a measure of their concentration). Since the standard pressure of the total inhaled mixture is atmospheric pressure (760mm Hg at sea level), the partial pressure can also be expressed as a %. Thus, 50% N2O in the inhaled air would have a partial pressure of 380mm Hg. * SPEED of inhalation induction of anesthetic effects depends on: 1. Gas solubility (the primary factor). The more soluble the gas in blood, the slower rate of induction. 2. Inspired gas partial pressure. 3. Ventilation rate. 4. Pulmonary blood flow. 5. Arteriovenous concentration gradient. Administration of an inhalation anesthetic (except N2O) is usually preceded by IV or intramuscular administration of a short-acting sedative hypnotic drug (often a barbiturate). The procedure almost always requires endotracheal intubation. Drugs that when administered 1 hour prior to the dental appointment are safe and effective ways to reduce fears of an apprehensive ADULT dental patient and avoid a psychogenic reaction in the dental chair include:

* Diazepam (Valium): 5-10mg orally (PO). * Promethazine (Phenergan): 25mg orally (PO). * Pentobarbital (Nembutal) or Secobarbital (Seconal): 50-100mg orally (PO). * These drugs are NOT recommended unless the dentist has experience with them and can handle any complications from their use. For a dentist to use “enteral sedation” (use of a pharmacological method that produces a minimally-depressed level of consciousness), some states require special training and registration with the state. The most common emergency seen after using local anesthetics is SYNCOPE (fainting). Syncope often occurs when upright, but can also occur while sitting. Syncope never occurs when lying down. The patient may complain of feeling generalized warmth with nausea and palpitations. * The initial event in a vasovagal syncope episode is the stress-induced release of increased amounts of catecholamines that cause ↓ peripheral vascular resistance, tachycardia, and sweating. * As blood pools in the periphery, ↓ BP and cerebral blood flow occurs, causing the patient to complain of

feeling dizzy or weak. Compensatory mechanisms attempt to maintain adequate BP, but they soon fatigue, causing vagally-mediated bradycardia. Once BP drops below levels necessary to sustain consciousness, syncope occurs. * Placing the patient in the supine position while administering 100% oxygen is often all that is required. Additional treatment is based on symptoms. * The primary airway hazard for an unconscious dental patient in a supine position is tongue obstruction (so head tilt and lift the chin). After receiving injecting a local anesthetic containing 2% lidocaine with 1:100,000 EPI, the patient loses consciousness. The most probable cause is SYNCOPE, caused by transient cerebral hypoxia. Proper Management of Syncope: 1. Place the patient in the supine position with their feet slightly elevated. 2. Establish airway (head tilt/chin lift) and administer 100% oxygen via a face mask (O2 is indicated for treating all types of syncope EXCEPT hyperventilation syndrome). 3. Monitor vital signs and support the patient. Pupils may dilate from lack of oxygen to the brain.

4. Maintain your composure. Apply a cool, wet towel on the patient’s forehead. 5. Follow-up treatment. Determine factors causing the unconsciousness. *Hyperventilation in an anxious dental patient leads to CARPODEDAL SPASM (spasm of hand, thumbs, foot, or toes). Somatogenic Reaction-the development of a reaction from an organic pathophysiologic cause. Shock Symptoms: tiredness, sleepiness, confusion. Skin is cold, sweaty, bluish, and pale. Pulse is weak & rapid, and BP drops. Reduces cardiac output (CO) is the MAIN factor in all types of shock. TYPES OF SHOCK: 1. Cardiogenic Shock-most commonly caused by MYOCARDIAL INFARCTION. Shock consists of a set of hemodynamic changes that diminish blood flow below a level that provides adequate oxygen for the metabolic needs of organs and tissues. Shock is circulatory collapse resulting from pump failure of the LEFT VENTRICLE, most often caused by a massive myocardial infarction. 2. Hypovolemic Shock-produced by a reduction in blood

volume due to severe hemorrhage, dehydration, vomiting, diarrhea, or fluid loss from burns. 3. Septic Shock-due to severe infection caused by endotoxin from gram (-) bacteria. 4. Neurogenic Shock-results from severe injury or trauma to the CNS. 5. Anaphylactic Shock-occurs from a severe allergic reaction. STAGES OF SHOCK: 1. Compensatory stage-the early stage where compensatory mechanisms (↑HR and peripheral resistance) maintain perfusion to vital organs. 2. Progressive stage-metabolic acidosis occurs and compensatory mechanisms are no longer adequate. 3. Irreversible/refractory stage-organ damage occurs and survival is impossible. A patient who fractured his jaw in a car accident received an IM injection of 60mg of Meperidine, then subsequently developed a severe reaction characterized by tachycardia, hypertension, hyperpyrexia, and seizures. When questioned, the uninjured father revealed his son takes a drug for a psychiatric condition. The psychiatric drug most

likely responsible for this untoward reaction with Meperidine is PHENELZINE. Concomitant administration of Meperidine and MAO inhibitors can cause lifethreatening hyperpyrexic reactions that may culminate into seizures or coma. * Meperidine (Demerol)-a potent narcotic analgesic prescribed to relieve moderate-to-severe pain, and as a cough suppressant. Demerol is probably the most widely used narcotic in U.S. hospitals. It compares favorably with morphine (the standard for narcotic analgesics). Meperidine is the most abused drug by health professionals. * Morphine-the standard drug to which all analgesic drugs are compared. It causes euphoria, analgesia, drowsiness, miosis, and respiratory depression. Complete Blood Count (CBC) & Urinalysis are two tests that should be performed prior to administering general anesthetic for surgical procedures. If a bleeding problem is suspected, PTT (partial thromboplastin time) is also evaluated. A CBC test includes: 1. Hematocrit: the volume % of RBC in whole blood. Normal male (45-50%), normal female (40-45%). Hematocrit is the amount of your blood that is occupied by red blood cells (its like chocolate milk; how much is chocolate vs. milk). Hematocrit is the volume % of RBC in whole blood. Normal male (45-

50%) & normal female (40-45%). *The minimal acceptable value of hematocrit is 30% for elective surgery. 2. Hemoglobin: normal men (14-18g/dL) and normal women (12-16g/dL). 3. Total Leukocytes (WBC): normal (5000-10,000/mm3); dental infection (15,000-20,000/mm3). 4. Total Erythrocytes (RBC): normal men (4.5-6.0) x 106/mm3; normal women (4.3-5.5) x 106/mm3. Normal Values for Coagulation: • Template Bleeding Time: 1-9 minutes. • Prothrombin Time (PT): 11-16 seconds (compared to normal control). • Partial Thromboplastin Time (PTT): activated, 3246 seconds (compared to normal control). • Platelets: 140,000-440,000/ml. Urine Values: pH 6 (4.7-8.0); specific gravity (1.005-1.025).

EXODONTIA General Surgery Concerns: patient’s nutritional status, body fluids and electrolytes, pre-operative and postoperative information, wound healing (primary and secondary), and most important—infection. *The difference between acceptable and an excellent surgical outcome depends on how the surgeon handles the tissue. Discipline of oral surgery is “the diagnosis and surgical treatment of injuries, disease, and malformation of the mouth and jaws”. Major Oral Surgical Procedures: • Treatment of maxillary and mandibular fractures. • Pre-prosthetic surgery (tuberosity reductions and vestibuloplasty). • Reconstructive surgery (orthognathic surgery and facial deformities). • Traumatology (treatment of wounds, injuries, and resulting disabilities). • Exodontia (including routine, multiple, & surgical extractions) is NOT a major oral surgery procedure. Minor Oral Surgical Procedures:

* Exodontia (routine extractions, multiple extractions, surgical extractions) * Treatment of dental infections (periapical, periodontal, pericornitis, facial infections (cellulitis). * Treatment of hard tissue (alveoplasty) and soft tissue (biopsy, benign lesions) pathologies. Local Contraindications to Tooth Extractions: * ANUG, irradiated jaws, and malignant disease. * Acute infection with uncontrolled cellulitis. * Acute infectious stomatitis. Systemic Contraindications to Tooth Extractions: * Uncontrolled diabetes mellitus * Uncontrolled cardiac disease and dysrythmias. * Uncontrolled leukemias and lymphomas. * Debilitating diseases. * Severe bleeding disorders. * Patients taking certain medications (i.e. immunosuppressives, corticosteroids, & cancer chemotherapeutic agents). An acute dentoalveolar abscess should NOT be a contraindication to extraction because infections can resolve very quickly when the affected tooth is removed. However, it may be difficult to extract the tooth either because the patient cannot open sufficiently wide enough, or because adequate local anesthesia cannot be obtained.

Conditions that require Antibiotic Prophylaxis prior to oral surgery: 1. Prosthetic heart valve 2. Rheumatic valve disease 3. Most congenital heart malformations Patients with cardiac pacemakers do not require antibiotic prophylaxis since the endocardium is not involved. If antibiotic prophylaxis is necessary, these medications & dosages are recommended by the AHA:

Antibiotic Prophylaxis REPLACEMENTS:

for

TOTAL

JOINT

• No scientific evidence currently supports antibiotic prophylaxis to prevent HEMATOGENOUS INFECTIONS before dental treatment in patients with total joint prosthesis. 2014 panel of experts convened by the ADA Council of Scientific Affairs developed an evidence-based clinical practice guideline (CPG) to clarify the 2012 panel of the American Academy of Orthopaedic Surgeons (AAOS) and American Dental Association (ADA). The final recommended is in general, for patients with prosthetic joint implants, prophylactic antibiotics are NOT recommended before dental procedures to prevent prosthetic joint infection. The reasoning behind the 2014 recommendation is evidence shows dental procedures are not associated with prosthetic joint infections, antibiotics provided before oral care do not prevent joint implant infections, the potential harms of antibiotics like anaphylaxis, antibiotic resistance, opportunistic infections like Clostridium difficile, and the benefits or antibiotic prophylaxis may not exceed the risks for most patients. • Antibiotic prophylaxis is NOT indicated for dental patients with PINS, PLATES, & SCREWS, nor is it routinely indicated for most dental patients with total joint replacements. PREMEDICATION IS NO LONGER REQUIRED for patients after artificial

joint replacement surgery. However, the dentist can consult with the patient’s physician if the patient has a compromised immune system (diabetes, RA, cancer, chemotherapy, and chronic steroid use) which may increase the risk of orthopedic implant infection. • If unanticipated bleeding occurs, administer an antimicrobial prophylaxis within 2 hours after the procedure! Most frequently IMPACTED teeth are MANDIBULAR 3rd MOLARS (followed by maxillary 3rd molars and maxillary canines). One system describes the angulation of the long axis of the impacted 3rd molar w/r/t the 2nd molar’s long axis: * Mesioangular = 43% of all impacted teeth. * Vertical = 38% of all impacted teeth. * Distoangular = 6% of all impacted teeth. * Horizontal = 3% of all impacted teeth. Additionally, teeth can be angled buccally or lingually (*most mandibular 3rd molars are angled toward the LINGUAL). Impacted teeth are also classified based on their relationship to bone and tissue: * Soft tissue impaction = tooth is impacted only by soft tissue. * Partial bony impaction = crown is partially covered

by bone. * Full bony impaction = tooth is completely covered by bone. The ideal time to remove impacted 3rd molars is when the root is approximately 2/3 formed. The patient is around 17-21 years old. At this time, bone is more flexible and the roots are not formed well enough to have developed curves and rarely fracture during extraction. * When the root is fully formed, the possibility increases for abnormal root morphology and fracture of root tips during extraction. * If root development is insufficient (1/3 or less), the tooth is more difficult to remove because it tends to roll in its crypt (like a ball in a socket), preventing easy elevation. * Young patients tolerate surgery very well, with minimal post-operative complications. Older patients have the most post-operative difficulties, as the bone is more dense and the patient usually responds more slowly to the entire process (anesthesia and surgery). * Complications most often seen after extraction of an isolated residual maxillary erupted molar are fracture of the tuberosity or sinus floor. *Warning: beware of the lone molar.

During extraction of a maxillary 3rd molar, if you realize the tuberosity has also been extracted, the proper treatment is to smooth sharp edges of the remaining bone and suture the remaining soft tissue. A fracture of the maxillary tuberosity most commonly results from extraction of an erupted maxillary 3rd molar (or 2nd molar if it is the last tooth in the arch). If the tuberosity is fractured but intact, it should be manually repositioned and stabilized with sutures. Cavernous Sinus Thrombosis (CST)-usually caused by a late complication of an infection (Staphylococcus Aureus) of the central face or paranasal sinuses. CST is an unusual occurrence that is rarely the result of an infected tooth. It is usually a fulminant process with high rates of morbidity and mortality. * CST incidence has decreased greatly with the advent of effective antibiotics. Most cases are due to an acute infection in an otherwise healthy individual. However, most patients with chronic sinusitis or diabetes mellitus may be at a slightly higher risk. * Infections of the face can cause a septic thrombosis of the cavernous sinus. Furunculosis and infected hair follicles in the nose are frequent causes. Extractions of maxillary anterior teeth in the presence of an acute infection and especially curettage of the socket under such circumstances can cause CST. The infected thrombus ascends in the veins against the

usual venous flow. It usually occurs in the OPHTHALMIC VEIN because of the absence of valves in the angular, facial, & ophthalmic veins. * CST is not usually caused by a bacteremia, trauma, or ear infections. LUDWIG’S ANGINA-most commonly encountered neck space infection that involves sublingual, submandibular, & submental spaces. Submandibular Space-a potential space of the neck bound by the oral mucosa and tongue anteriorly and medially, superficial layer of deep cervical fascia laterally, and hyoid bone inferiorly. It comprises two spaces (sublingual & submaxillary spaces), divided by the mylohyoid muscle. * Submandibular space usually drains infections from mandibular premolars & molars since their apices lie BELOW the mylohyoid muscle attachment. * While extracting a mandibular 3rd molar, if you notice the distal root tip is missing, it is most likely to be found in the SUBMANDIBULAR SPACE. To prevent this, avoid all apical pressures when removing roots or root tips of all mandibular molars. If a mandibular molar root tip is displaced inferiorly, it may be in the mandibular canal, or through the lingual cortical plate.

Sublingual Space-superior part of the submandibular space that contains the sublingual gland & loose C.T. surrounding the tongue. Submental Space-medial part of the submaxillary space that contains submental lymph nodes that drain the median parts of the lower lip, tip of tongue, and mouth floor. It usually drains infections from mandibular incisors and canines because their apices lie ABOVE the mylohyoid muscle attachment. MOST COMMON SITE for a supernumerary tooth is the MAXILLARY INCISOR AREA. When it occurs here, it is called a MESIODENS (these teeth are usually small, pegshaped, and do not resemble the teeth normal to the site). Treatment of mesiodens is by surgical removal. The main reason to use water irrigation when cutting bone is because heat generated by the drill affects bone vitality (you do not want to burn the bone). Irrigating the surgical wound during and after the procedure is critical. Copious amounts of coolant spray are crucial in minimizing osseous necrosis caused by heat generated from the bur. Irrigation also cleans the crypt and areas beneath the flap of bony debris, tooth fragments, and blood. Class II lever classification is used during tooth extractions. Teeth are extracted by luxation forces perpendicular to the long axis of the tooth (not by pulling

along the long axis). The fulcrum is as close to the tooth apex as possible. *Rotational forces can be used on single rooted teeth. • The extraction forcep beak is designed so most of the pressure exerted during extraction is transmitted to the tooth root. • Luxation-the loosening of the tooth in the socket by progressive severing of the PDL fibers using patience and controlled force, NOT brute strength. The force is applied as far down on the root as possible. You should also support the jaw with your other hand, with a thumb and finger on either side of the tooth being extracted. #15 Scalpel is UNIVERSALLY used for oral surgery. 3 incisions are used in oral surgery: 1. Linear incision-a straight-line incision used for Apicoectomies. 2. Releasing incision-used to add a vertical leg to a horizontal creation incision for extractions, augmentations. The correct position for ending a vertical releasing incision is at the TOOTH LINE ANGLE (not over the tooth’s buccal surface). If the incision ends over a buccal surface, the edges are difficult to approximate, which can cause periodontal problems.

3. Semi-lunar incision-a curved incision used mostly for Apicoectomies. FLAP-a section of soft tissue that is outlined by a surgical incision, carries its own blood supply, allows surgical access to underlying tissues, can be replaced in the original position, and maintained with sutures and is expected to heal. • Flap design should ensure adequate blood supply (flap base is always larger than the flap apex). • Flap reflection should adequately expose the operative field. • Flap design should permit atraumatic closure of the wound. SUTURES: can be resorbable or non-resorbable * Interrupted Suture Pattern/Method-offers strength and flexibility because each suture is independent to one another. This is advantageous because if one suture is lost or loosens, the integrity of the remaining sutures is not compromised. The major disadvantage of this pattern of suturing is the extra time required for placement. * Continuous Suture Pattern/Method-provides ease and speed of placement, distributes tension over the entire suture line, and a more watertight closure than the interrupted pattern or method.

* Sutures should never be over tightened or closed under tension, should be 2-3mm apart, placed from mobile tissue into fixed tissue, and from thin into thick tissue. * Suture size is based on strength and diameter. This system uses “0” as the baseline average size suture. As suture diameter decreases, “0s” are added or numbers followed by a “0” (i.e. 000 and 3-0 are the same size). As suture diameter increases above “0”, numbers are assigned to the suture material. * Because suture material is foreign to the body, the smallest diameter suture sufficient to keep the wound closed properly should be used. Most OMS procedures require 3-0 or 4-0 sutures. 9-0 suture size has the least strength and smallest diameter. The most severe tissue reaction occurs with PLAIN CATGUT SUTURE material. Resorbable sutures evoke an intense inflammatory reaction (thus, plain and chromic gut are not used to suture the surface of a skin wound).

Impacted maxillary 3rd molars are occasionally displaced into the maxillary sinus (antrum), from which they are removed via a Caldwell-Luc approach. If a large root fragment or the entire tooth is displaced into the maxillary sinus (antrum), it should be removed using a Caldwell-Luc approach (a surgical procedure where an opening is made into the maxillary sinus via an incision into the canine fossa above the level of the premolar roots. The tooth or root is then removed. *An oral surgeon to whom the patient should be referred should perform this procedure. * Strong apical pressure with a small straight

elevator may displace root tips of maxillary premolars & molars into the MAXILLARY SINUS. If the root tip is small (2-3mm), non-infected, and cannot be removed through the small opening in the socket apex, no additional surgical procedure is performed through the socket, and the root tip is left in the sinus. If the root tip is left in the sinus, measures should be taken similar to those taken when leaving any root tip in place. The patient must be informed of the decision and given proper followup instructions. * Maxillary 3rd molars can also be displaced into the infratemporal space during elevation of the tooth, the elevator may force the tooth posteriorly through the periosteum into the infratemporal fossa. If access and light are good, the tooth may be retrieved with a hemostat. If the tooth is not retrieved after a short amount of time, the area should be closed, and the patient informed that the tooth was displaced and will be removed by an oral surgeon who will use a special technique to remove it. Distoangular Impaction-the maxillary 3rd molar impaction most likely displaced into the antrum (maxillary sinus) and infratemporal space if correct extraction techniques are not used. Mandibular 3rd molars with a distoangular impaction are the most difficult impaction to remove.

Mesioangular Impaction-most common and LEAST difficult impaction to remove, comprising 43% of all impacted teeth. • For impacted mandibular 3rd molars, mesioangular impactions are the least difficult to remove, followed by horizontal and vertical impactions, then the most difficult to remove which is the distoangular impaction. • For impacted maxillary 3rd molars, mesioangular impactions are the most difficult to remove, while vertical and distoangular impactions are the easiest to remove. Once sufficient amounts of bone are removed from around the impacted tooth, the tooth is usually sectioned to allow portions of the tooth to be removed separately with elevators through the opening provided by bone removal. Bone is rarely, if ever, removed on the lingual aspect of the mandible because of the likelihood of damaging the lingual nerve. * Reasons to Section Teeth: allows for minimal bone removal, minimal force to remove the tooth, and shortens the entire surgical procedure. Tooth sectioning is performed with a bur or chisel, but the bur is used by most surgeons. Dead space in a wound usually fills with BLOOD which creates a hematoma with a high infection potential.

Dead space in a wound is any area that remains DEVOID OF TISSUE after closing the wound. Dead space is created by removing tissues in the depths of a wound or by not re-approximating tissue planes during closure. Dead space is eliminated by closing the wound in layers to minimize the postoperative void, applying pressure dressings, using drains to remove any bleeding that accumulates, and placing packing into the void until bleeding stops. While attempting to remove a grossly decayed mandibular molar, the crown fractures. The recommended next step to facilitate removal of the tooth is to separate the roots with a chisel, elevator, or most easily with a bur. Teeth with two or more roots often need to be sectioned into single entities prior to successful removal. A popular sectioning method is to make a bur cut between the roots, then inserting an elevator in the slot and turning it 90° to cause a break. • Roots can be removed by using a closed technique. The surgeon should begin a surgical removal if the closed technique is not immediately successful. • Teeth are resistant to crush, but are not resistant to shear. Thus, place the forcep beaks opposite to each other at the same level on the tooth. Forcep beaks are applied in a line parallel with the long axis of the tooth.

• When luxating a tooth with forceps, the movements should be firm and deliberate, primarily to the facial with secondary movements to the lingual. • Maxillary 1st premolar is the least likely to be removed by rotational forces due to its root structure (molars are obviously NOT removed using rotation). PALATAL is the primary direction of luxation for extracting maxillary primary/deciduous molars, while the BUCCAL is the primary direction for adult maxillary molars. This is because primary molars are more palatally positioned, and the palatal root is strong and less prone to fracture. • In general, removing primary teeth is not difficult. It is facilitated by the elasticity of young bone and resorption of root structure. Do NOT use a “cowhorn” forcep to extract mandibular primary molars because the sharp beaks of these forceps can damage unerupted permanent premolar teeth. • If the preoperative radiograph shows that the permanent premolar is tightly wedged between the bell-shaped roots of the primary tooth, the best treatment is to section primary molar crown and remove the two portions separately. This helps to not disturb the permanent tooth.

• After extracting mandibular teeth on a child after a mandibular block was given, always advise the child not to bite his/her lip while he/she is numb. Also inform the parents to watch the child to prevent this. When extracting maxillary teeth, the patient’s maxilla is AT THE SAME HEIGHT as the dentist’s shoulders. Stand in front of and to the side of the patient for maximum visibility and leverage. For mandibular extractions, the patient is positioned so the occlusal plane of the mandibular arch is PARALLEL to the FLOOR when the mouth is opened, and the chair should be as low as possible. Stand directly to the side or behind the patient. Fingers on the left hand (for a right-handed dentist) serve to: • Retract soft tissue, provide the operator with sensory stimuli to detect expansion of the alveolar plate and root movement under the plate. • Help guide the forceps into place on the tooth, protect teeth in the opposite jaw from accidental contact with the back of the forceps, and support the mandible while performing mandibular extractions. GENAIL TUBERCLES-located on the lingual surface of

the mandible midway between the superior & inferior borders. There are 4 (two are situated on each side and adjacent to the symphysis). While they are usually relatively small, they may be fairly large and extend outward from the surface as spinous processes. Geninal tubercles are the area of muscle attachment for the SUPRAHYOID MUSCLES. In preparing the edentulous mandible for dentures, each may be safely excised by the OMS (labial and lingual frenums, mylohyoid ridge, and exostosis). Genial tubercles are never excised, because it they were removed, the tongue would be flaccid. • When removing the mylohyoid ridge, be careful to protect the lingual nerve. • When removing a mandibular exostosis (mandibular torus), an envelope flap design (has no vertical components) should be used. MAXILLARY TORUS (Torus Palatinus)-most frequently located on the MIDLINE of the hard palate. They usually appear before age 30, and affect females more often than males. Maxillary tori present few problems when the maxillary dentition is present and only occasionally interferes with speech or become ulcerated from frequent trauma to the palate. * Indications for Removal: a large, lobulated torus with a thin mucoperiosteal cover extending posteriorly to the vibrating line of the palate that prevents seating of

a denture and formation of a posterior seal at the fovea palatine. Chronic irritation, interference with speech, rapid growth, and patients with a cancer-phobia are also indications. * Removal Technique: it should not be excised en masse to prevent entry into the nose (the palatine bone will come out with the torus). Subdivide the tori into segments with a bur and remove the segments with an osteotome. Any protuberances are smoothed out with a bone file. The flap is then loosely sutured and place a palatal splint to prevent hematoma formation and to support the flap. NORMAL POST-EXTRACTION PROCEDURE: 1. All loose bone spicules and portions of the tooth, restoration, or calculus are removed from the socket and from the buccal and lingual gutters and tongue. 2. Compress the socket must with the fingers to reestablish the normal width present before the buccal plate was surgically expanded. *The natural recontouring of the residual ridge occurs primarily by resorption of the labial-buccal cortical bone. 3. Sutures are usually not placed unless the papillae have been excised, there is severe bleeding from the gingiva, or if the gingival cuff is torn or lose, only then would you place a suture over a single extraction socket.

4. Socket is covered with a gauze sponge folded and moistened slightly at its center with cold water. 5. Patient is instructed to bite down on the pressure dressing for 30-60 minutes. 6. Printed instruction sheet is given to the patient. *The most common cause of post-extraction bleeding is failure of the patient to follow post-extraction instructions. 7. Prescription for pain is given if the need is anticipated. If bleeding persists for some time after an extraction, instruct the patient to bite on a tea bag. Tannic acid in the tea bag helps promote hemostasis. DRY SOCKET (Post-Extraction Fibrinolytic Alveolar Osteitis or Localized Alveolar Osteitis)-most common complication after surgical removal of a MANDIBULAR MOLAR resulting from a pathologic process combining the loss of a healing blood clot with a localized inflammation. * A patient with DRY SOCKET develops a severe, dull throbbing pain 2-4 days AFTER a tooth extraction. The pain is often excruciating, may radiate to the ear, and is not relieved by oral analgesics. There may be an associated foul odor and taste, and the extraction site is filled with necrotic tissue, which is delayed wound healing.

* Smoking, spitting, or drinking through a straw creates negative pressure in the oral cavity, thus encouraging this condition. *Careful technique and minimal trauma reduce the chance of dry socket. * Treatment: gently irrigate debris with saline solution and place a sedative dressing (eugenol) in the socket (eugenol is the active component in most sedative dressings). Gauze provides an attachment for the obtundent paste so it stays in the socket. Prescribe analgesics if needed. PERICORONITIS-inflammation of the soft tissues (operculum) associated with the crown of a PARTIALLY erupted tooth seen most commonly in relation to the MANDIBULAR 3rd MOLAR. Maxillary 3rd molar is the most frequent contributing factor to pericoronal infections found around mandibular 3rd molars, so always examine the maxillary 3rd molar because it may be supererupted or malaligned. * Signs & Symptoms: pain, bad taste, inflammation, pus expressible from beneath, and the pericoronal tissues are aggravated by trauma from the opposing tooth. Pericornitis is a criteria by the NIH (National Institute of Health) for removing 3rd molars. * Unless the cause is removed, pericoronitis may present as a recurrent condition requiring multiple treatments. In severe episodes, an acute pericoronal

abscess may develop which may remain localized or spread to involve one or more of the adjacent deep surgical spaces, and may be associated with systemic and local signs and symptoms. * Treatment: irrigate the area (if possible, establish drainage). Place the patient on antibiotics and instruct the patient to rinse with warm saline mouthwashes. As soon as acute symptoms are relieved, definitive treatment may be instituted (extraction). Incision for Drainage (I&D) in an area of acute infection is only performed after LOCALIZATION of the INFECTION. Physiologically, it is at this time that nature has constructed a barrier around the abscess, walling it off from the circulation and making it possible to palpate the presence of purlent material within the abscess cavity (“fluctuance”). ► After you incise and drain the fluctuant mass, it may be prudent to culture for antibiotic sensitivity. This should always be done if after I&D, the swelling does not subside despite large doses of antibiotics. ► Prior to actual abscess formation, however, the infection can produce a cellulitis in the soft tissues of the involved region. The palpable tissues take on a condition called induration (appearing hard, dense, and brawny). Treatment during this

period involves localizing the infection. Early administration of antibiotics may be extremely important in a severe and life-threatening infection. Localization of the infection may be aided by using warm compresses and warm mouth rinses at frequent intervals. 5 HEALING PHASES OF AN EXTRACTION SITE: 1. Hemorrhage and blood clot formation. 2. Clot organization by granulation tissue. *Glucocorticoids have the greatest effect on granulation tissue by retarding healing. 3. Replacement of granulation tissue by C.T. and epithelialization of the site. 4. Replacement of C.T. by fibrillar bone. 5. Recontouring of the alveolar bone and bone maturation. The same stages that occur in normal wound healing of soft tissue injuries (inflammation, fibroplasias, & remodeling) also occur in repair of injured bone. However, osteoblasts and osteoclasts are also involved to repair damaged bone tissue. 3 STAGES OF WOUND HEALING: 1. Inflammatory stage-consists of a vascular and cellular phase where neutrophils and lymphocytes predominate. The macrophage is the most important inflammatory cell for wound healing.

2. Proliferative Stage (Fibroblastic Stage)-this stage is mediated by fibroblasts which form collagen and new blood vessels. 3. Maturation Stage (Remodeling Stage)-foreign material, necrotic tissue, ischemia, and tension. 3% H2O2 is the agent of choice to debride intraoral wounds. Bone & Soft Tissue HEAL by Primary or Secondary intention: ► Primary Intention (Primary Closure or First Intention): bone repair that involves both endosteal & periosteal proliferation. Primary intention occurs when bone is either incompletely fractured, or a surgeon closely reapproximates the fractured ends of a bone. Little fibrous tissue is produced with minimal callus formation. Primary intention occurs when wound margins are nicely apposed. Healing is more rapid with a lower risk of infection, with less scar formation, and less tissue loss than wounds that heal by secondary intention. Examples of primary intention (well-repaired and well-reduced bone fractures). ► Secondary Intention (Secondary Closure or Second Intention): occurs when a wound is large and exudative. This side fills in with granulation tissue. Healing is slower and produces more scar

tissue. Bone repair that involves mostly endosteal proliferation. If fractured bones remain more than 1mm apart, this type of repair occurs. Lots of fibrous tissue is formed and a callus is formed (which eventually ossifies). Examples (extraction sockets, poorly reduced fractures, and large ulcers). Factors that DELAY the healing process of an extraction site: patient with protein deficiency or on glucocorticoid therapy, older patients, and local infections.

GRAFTS IDEAL GRAFT is replaced by the host bone, withstands mechanical forces, produces no immunological response or rejection, and actively assists osteogenic (bone-forming) processes of the host. The greatest osteogenic potential occurs with an autogenous cancellous graft and hemopoietic marrow. * Bones, plates, biphasic pins, titanium mesh, and intraosseous wires are used to fixate bone grafts. Sutures are not generally used. * Costochondrial Rib Graft-may be used with the cartilaginous portion simulating the TMJ and condyle. When used for ridge augmentation, extensive shrinkage is noted. ALLOGENEIC GRAFTS (Allografts or Homografts)composed of tissue taken from a donor of the SAME SPECIES who is not genetically related to the recipient (usually cadaver bone). • The most commonly used allogenic bone is FREEZEDRIED. These grafts consist of freeze-dried bone and freeze-dried decalcified bone from another source (human cadaver bone). • Allogenic grafts are treated to reduce the anti-

genicity. However, these treatments destroy any remaining osteogenic cells it the graft. These grafts offer a hard tissue matrix only (this graft is eventually replaced by the host’s bone). • Host MUST produce ALL of the essential elements in the graft bed for an allogeniec bone graft to become resorbed and replaced. • Allogeneic Advantages: does not require another site of preparation in the host, and a similar bone or bone of similar shape to the bone being replaced can be obtained. AUTOGENOUS GRAFTS (Autografts)-composed of tissues taken from the SAME INDIVIDUAL. Most often used in oral surgery. Autogenous grafts, while often present surgical & technical problems, do not usually involve rejection or immunological complications. * OPTIMAL bone grafting material should be of autogenous origin. Autogenous bone is from the same person (from one part of the body to another). Autogenous grafts are usually used to restore large areas of lost mandibular bones after oncological surgery or trauma. Of all facial bones resected in oncological surgery, the mandible is the most frequently removed. * Bone marrow for grafting defects in the mandible &

maxilla is usually obtained from the ILIAC CREST. Also used for ridge augmentation. Isogenic Grafts (Isografts or Syngenesioplastic Grafts)composed of tissues taken from an individual of the SAME SPECIES who is GENETICALLY RELATED to the recipient. Xenogenic Implants (Xenografts or Heterografts)composed of tissue taken from a donor of ANOTHER SPECIES (i.e. animal bone grafted to man). Rarely used in oral surgery. Rejection of a graft is MOST common when Allografts or Xenografts of bone and cartilage are used in oral surgery. ALLOPLATIC GRAFTS-inert, man-made synthetic materials. Alloplastic materials used for augmentation genioplasty tend to MIGRATE from the position they were placed at the time of surgery. Two additional problems often experienced when using alloplastic materials for genioplasty are: 1. Erosion of the chin prominence contiguous with the implant. 2. Unpleasant sensation in the implant region when exposed to cold temperatures. Modern artificial joint replacement procedures use metal

alloplastic grafts. For bone replacement, a man-made material that mimics natural bone is used. Most often, this is a form of calcium phosphate (i.e. tricalcium phosphate, calcium carbonate, or hydroxyapatite) that is used for augmentation of the mandible. * Hydroxyapatite-a dense, biocompatible material produced synthetically or obtained from biologic sources like coral. The granular or particle form is most commonly used for alveolar ridge augmentation. When placed in a subperiosteal environment, HA bonds physically and chemically to bone. * Advantages of Restructuring an Atrophic Ridge with HA granules: ♦ It is a simple surgical technique suitable as an office procedure. ♦ No donor site is required to obtain autogenous bone graft material. ♦ HA is totally biocompatible and non-resorbable. * Disadvantages of Restructuring an Atrophic Ridge with HA granules: ♦ Migration of the hydroxyapatite granules. ♦ Poor ridge form (inadequate height). ♦ Abnormal color under the mucosa. ♦ Mental nerve neuropathy (excessive augmentation). INSTRUMENTS TO REMOVE BONE:

• Rongeur Forceps are the most commonly used instrument to remove bone. A chisel & mallet, and bone file can also be used. However, the technique most oral surgeons use to remove bone is the bur & handpiece. • Most high-speed turbine drills used in routine restorative dentistry are UNACCEPTABLE for oral surgery. Air exhausted from these drills goes into the wound and may be forced deeper into tissue planes and produces tissue emphysema, a potentially dangerous situation. An acute infected tissue emphysema is usually caused by indiscreet use of: 1. Air-pressure syringes: in drying out a root canal with a compressed air syringe, septic material can be forced through the apical foramen into the cancellous portion of the alveolar process, and ultimately out through the nutrient foramina into adjacent soft tissues, resulting in formation of a septic cellulitis & tissue emphysema. 2. Atomizing spray bottles activated by compressed air: a similar condition can be induced by using a compressed-air spray bottle for irrigating wounds (especially in the retromolar region). It is safer to use a hand-activated syringe when irrigating wounds or drying root canals since to reduce the chance of producing a tissue

emphysema. FRENUM-a membranous fold of skin or mucous membrane that supports or restricts movement of a part or organ like the small band of tissue that connects the underside of the tongue to the floor of the mouth. When a frenum is malpositioned and interferes with the normal alignment of teeth, or results in pulling away gingiva from the tooth surface causing recession (attachment loss), it is often removed via a FRENECTOMY. FRENECTOMY SURGICAL TECHNIQUES: 1. Simple excision and Z-plasty: are effective when the mucosal and fibrous tissue band is relatively narrow. These techniques relax the pull of the frenum. 2. V-Y plasty (localized vestibuloplasty)-often preferred when the frenal attachment has a wide base. This technique is good for lengthening tissue and usually results in less scarring. Local anesthetic infiltration is usually sufficient fro surgical treatment of frenal attachments. Care is taken to avoid excessive infiltration directly in the frenum area because it may obscure view of the anatomy during the excision. GINGIVOPLASTY-a surgical procedure to reshape the gingiva to create a normal, functional form.

OPERCULECTOMY-removal of operculum (flap of tissue over an unerupted or partially erupted tooth). GENIOPLASTY-a procedure that surgically alters the chin’s position. The most common techniques for genioplasty are osteotomy or augmentation using natural or alloplastic materials. The best way to enlarge the prominence of the chin for best long-term results is to reposition the lower border anteriorly by OSTEOTOMY (horizontal sliding osteotomy). ALVEOPLASTY-a surgical procedure used to recontour the supporting bone structures in preparation of a complete or partial denture. It is the surgical preparation of the alveolar ridges (i.e. removing undercuts) to receive dentures or shaping and smoothing the socket margins after extractions with subsequent suturing to ensure optimal healing. * Objectives of bone recontouring should provide the best possible tissue contour for prosthesis support, while maintaining as much bone and soft tissue as possible. CLOSED REDUCTION-closing the space between a fractured bone without cutting through soft tissue or surrounding bone. Systemic Contraindications to Elective Surgery: * Blood dyscrasias (i.e. hemophilia, leukemia)

* Uncontrolled diabetes mellitus (controlled diabetes is NOT a contraindication to elective surgery). * Addison’s Disease or any steroid deficiency * Fever of unexplained origin * Nephritis * Any debilitating disease * Cardiac disease (i.e. coronary artery disease, uncontrolled hypertension, and cardiac decompensation) can complicate exodontia. Usually a post-infarction patient is not subjected to oral surgery within 6 months of his infarction. However, emergency procedures can be performed, if the patient’s physician has been consulted. Patients with these systemic conditions can be treated, but you need to consult with their physician before treatment. In most cases, these patients are best treated in the hospital by an oral surgeon. Autogenous Tooth Transplantation-a tooth from the same individual is move to another socket in the mouth. The MOST common indication for tooth transplantation is SEVERE DECAY of a 1st MOLAR. (The first molar is atraumatically extracted, and replaced with the 3rd molar in its socket). • Transplant success is most predictable when the root apices to be transplanted are 1/3-1/2 formed with open apices and the bordering bony plates are intact. Adequate M-D width of the host implant

site, absence of acute periapical or periodontal inflammatory states, and the general good oral health are also required for successful tooth transplantation. • The most likely cause of transplantation failure is a chronic, progressive EXTERNAL ROOT RESORPTION. • Universal sequelae of an allogeneic tooth transplant is ANKYLOSIS & PROGRESSIVE ROOT RESORPTION. Allogeneic tooth transplant is when a tooth from one patient is placed into the socket of another patient.

BIOPSY TYPES OF BIOPSIES: 1. Incisional Biopsy-removes only part of the lesion. Most often used for oral lesions. A (-) incisional biopsy report of a highly suspicious oral lesion suggest another biopsy specimen is necessary in view of the clinical impressions. The key is a highly suspicious oral lesion. 2. Excisional Biopsy-removes the entire lesion. Most often used for oral lesions. 3. Needle Biopsy-aspiration biopsy. 4. Exfoliative Cytology-pap smear. Method of tissue removal depends on the type of biopsy: 1. In a needle (percutaneous) biopsy, the tissue sample is simply obtained by using a syringe. A needle is passed into the tissue to be biopsied, and the cells are removed through the needle. 2. In an open biopsy, an incision is made in skin, the organ is exposed, and a tissue sample is taken. 3. A closed biopsy involves a much smaller incision than

an open biopsy. The small incision is made to allow insertion of a visualization device that can guide the physician to the appropriate area to take the sample. 10% Formalin-the fixative of choice used for a routine biopsy specimen. After removal, the tissue is immediately placed in 10% formalin solution (4% formaldehyde) that is at least 20x the volume of the surgical specimen. The tissue must be totally immersed in the solution, and care is taken to ensure the tissue has not become lodged on the container wall above the level of the formalin. Biopsy Indications: * A lesion that persists for more than two weeks with no apparent etiologic basis. * Persistent hyperkeratotic changes in surface tissues. * Bone lesions not specifically identified by clinical and radiographic findings. * A lesion with malignant characteristics. * An inflammatory lesion that does not respond to local treatment after 14 days (i.e. removing a local irritant). * A persistent swelling (visible or palpable) below relatively normal tissue. Biopsy Techniques & Surgical Principles: * Anesthesia: block local anesthetic techniques are used when possible. If not, infiltration may be used, but the solution should be injected at least 1cm away from the lesion.

* Tissue stabilization: use fingers or clamps. * Hemostasis: gauze compresses (avoid high speed suction). * Incision: sharp scalpel * Extent of Tissue: obtain some normal tissue adjacent to the lesion is possible. * Handling of Tissue: use a traction suture through the specimen, not tissue forceps to avoid specimen trauma. * Specimen Care: after removal, the tissue should be immediately placed in 10% formalin solution that is at least 20x the volume of the surgical specimen. *No other solution is acceptable. Mandibular left 2nd molar of a 14-year old boy is unerupted. Radiographs show a small dentigerous cyst surrounding the crown. The treatment of choice is to uncover the crown and keep it exposed. Dentigerous Cysts (Primordial or Follicular Cysts)-cysts associated with the crowns of unerupted teeth, probably the result of degenerative changes in the reduced enamel epithelium.

Eruption Cysts-cysts that form when a tooth is erupting and interfere with normal eruption of teeth, and are more commonly found in the child or young adult, and may be associated with any tooth. If treatment is indicated, a simple incision or “deroofing” is all that is needed. Whether a bone cyst or other cysts are completely enucleated or treated by marsupialization depends on the SIZE & LOCATION to vital structures. 1. Enucleation-process by which the TOTAL REMOVAL of a cystic lesion is achieved. Enucleation is the treatment of choice whenever possible for congenital cysts, mucoceles, and most odontogenic cysts. 2. Marsupialization, Decompression, & Partsch Operation all create a “surgical window” in the wall of the cyst. The cyst is uncovered or “deroofed” and the cystic lining is made continuous with the oral cavity or surrounding structures. The cyst sac is opened and emptied. Marsupialization is the treatment of choice for ranulas (for a recurrent ranula, treatment also involves excision of the sublingual gland). Also the treatment of choice when a cyst is large and close to vital structures. Wait 14 days before obtaining a biopsy of an oral ulcer, since almost all oral ulcers caused by trauma heal within 14 days. Thus, any ulcer present for 2 weeks or

more should be biopsied. A biopsy is also indicated in these situations: • Pigmented lesions (black/brown). • When tissue is associated with paresthesia. This is often an ominous sign. • If a lesion suddenly enlarges. Always aspirate a central bone lesion to rule out a vascular lesion. If a lesion seems compressible, pulsatible, blue, or a bruit is heard using a stethoscope, beware of a vascular lesion and biopsy only under a controlled hospital setting.

ORAL SURGERY CONDITIONS & TREATMENT CONSIDERATIONS Dyspnea-the unpleasant sensation of difficulty breathing. Apnea-transient cessation or absence of breathing. Hypercapnea-excess CO2 in arterial blood. Hypocapnea-below normal CO2 in arterial blood. Hyperapnea-abnormally deep and rapid breathing. Respiratory arrest-permanent cessation of breathing (unless corrected). Hyperventilation-increased pulmonary ventilation in excess of metabolic requirements. Hyperventilation results in loss of CO2 from the blood (hypocapnea), thus causing a decrease in BP and sometimes fainting. Hypoventilation-underventilation in relation to metabolic requirements. Hypoventilation results in an increased level of CO2 in the blood (hypercapnea). DEHYDRATION-loss of water and important blood salts

like potassium (K+) & sodium (Na+). Vital organs (i.e. kidneys, brain, and heart cannot function without a certain minimum of water and salt). Causes include decreased intake (lack of water) and/or increased output (vomiting, diarrhea, blood loss, drainage from burns, diabetes mellitus, diuretic use, or a lack of ADH (anti-diuretic hormone) owing to diabetes insipidus (kidneys are unresponsive to ADH, or ADH is not being produced by the posterior pituitary). • Initially, a patient suffering from dehydration will clinically demonstrate only dryness of the skin and mucous membranes. However, a dehydration progresses, the turgor (fullness) of the skin is lost. If dehydration persists, oliguria (↓ urine output) occurs to compensate for the fluid loss. More severe degrees of fluid loss are accompanied by a shift of water from the intracellular space to the extracellular space (a process that causes severe cell dysfunction, especially in the brain). Systemic BP falls with continuous dehydration, and declining perfusion eventually leads to death. • Fluids in several forms should be continually urged on the patient. In severely dehydrated individuals, they must get to the hospital immediately. IV fluids quickly reverse dehydration, and is often life-saving in young children and infants. • Symptoms of Dehydration: ↓ BP, weight loss, ↑ HR, CO, body temperature, and sunken eyeballs.

CHRONIC OBSTRUCTIVE PULMONARY DISEASE (COPD)-a group of disorders characterized by airflow obstruction during respiration. COPD is a chronic airway obstruction resulting from emphysema, chronic bronchitis, asthma, or any combination of these diseases. In most cases, bronchitis and emphysema occur together. *Secondary pulmonary hypertension is most often caused by COPD. 1. Bronchial Asthma-disorder marked by DYSPNEA & WHEEZING expiration caused by episodic narrowing of the airways. 2. Emphysema-often coexists with chronic bronchitis. Labored breathing and increased susceptibility to infection. 3. Bronchiectasis-copious purulent sputum, hemoptysis, and recurrent pulmonary infection. 4. Chronic Bronchitis-a condition of excessive bronchial mucus and a productive cough that produces sputum (hypersecretion of mucus) for 3 months or more in at least 2 consecutive years without any other disease that could account for this symptom. A productive cough, often without wheezing, is the universal factor of chronic bronchitis. * Chronic bronchitis is a very common, debilitating respiratory disease, characterized by increased production of mucous by the glands of the trachea &

bronchi. It has a strong association with CIGARRETTE SMOKING. * Common results of Chronic Bronchitis: cor pulmonale (enlargement of the heart’s right ventricle), airway narrowing and obstruction, & squamous metaplasia of the bronchial tree. Patient’s with chronic bronchitis may be predisposed to lung cancer (bronchogenic carcinoma). * Patients with chronic bronchitis (or any COPD) can have difficulty during oral surgery. Many of these patients depend on maintaining an upright posture to breathe adequately. They often experience difficulty breathing if placed in an almost supine position or if placed on high-flow nasal oxygen. ATELECTASIS-the collapse of part or all of a lung by blockage of the air passages (bronchus or bronchioles), or by very shallow breathing. Atelectasis is the MOST COMMON anesthetic complication occurring within the first 24hrs after surgery under general anesthesia. Symptoms include: diminished breath sounds, fever, increasing dyspnea. Prolonged atelectasis can lead to PNEUMONIA. Pneumothorax-the presence of air in the pleural cavity. It can occur as a post-operative complication of aspiration of liquid vomitus into the trachea and bronchi. Onset of pneumothorax is accompanied by a sudden, sharp chest pain, followed by difficult, rapid breathing, cessation of

normal chest movements on the affected side, tachycardia, weak pulse, hypotension, diaphoresis, and elevated temperature, pallor, dizziness, and anxiety. * Pneumonitis (inflammation of the lung) & Atelectasis are the two most common causes of fever in a patient who has had general anesthesia. * Nausea is the most common pos-operative complication of outpatient general anesthesia. ASTHMA-a syndrome consisting of dyspnea, cough, and wheezing caused by bronchospasm, which results from a hyperirritability of the tracheobronchial tree. Two types of asthma exist (allergic asthma) the most common, and idiosyncratic asthma. • Avoid using aspirin, NSAIDs, barbiturates, narcotics, and erythromycin (if the patient is taking Theophylline). • Nitrous oxide is SAFE to administer in asthmatics, and is especially indicated for patients whose asthma is triggered by anxiety. If the patient is taking steroids, consult their physician for the possible need for corticosteroid augmentation. • Inhalation of a selective beta2-agonist (Terbutaline, Albuterol) is the preferred treatment for an acute asthmatic attack.

• Status Asthmaticus-the most severe clinical form of asthma, usually requiring hospitalization that does not respond adequately to ordinary therapeutic measures. If not managed properly, chronic partial airway obstruction may lead to death from respiratory acidosis (produced by hypoxemia & hypercapnea). Management of an Acute Asthmatic Episode during Oral Surgery: • Terminate all dental treatment and position the patient in an erect or semi-erect position. • Patient should administer their own bronchodilator using an inhaler. In most severe asthma attacks or when the patient’s bronchodilator is ineffective, EPI (0.3ml of a 1:1,000 dilution) can be injected IM or SC. • Administer oxygen and monitor vital signs. HEMOPHILIA-a hereditary BLEEDING DISORDER that mostly affects MALES, where it takes a long time for blood to clot and abnormal bleeding occurs. A true hemophiliac has prolonged partial thromboplastin time (PTT), but normal prothrombin time (PT) and bleeding time (BT). • Hemophilia A & B are inherited as a SEX-LINKED RECESSIVE trait where males are affected and females are carriers. Most people afflicted with

hemophilia have type A, and it presents under age 25yrs. Signs, symptoms, and clinical manifestations are excessive bleeding from minor cuts, epitaxis, hematomas, and hemarthroses. • Hemophilia A: represents most people with hemophilia. The classic type caused by a deficiency of coagulation Factor VIII (anti-hemophilic factor). • Hemophilia B (Christmas Disease): caused by a deficiency of Factor IX (plasma thromboplastin component). • Hemophilia C (Rosenthal’s Syndrome): is not sexlinked, and there is less severe bleeding due to a deficiency of Factor XI (plasma thromboplastin antecedent). von Willebrand’s Disease-inherited as an AUTOSOMAL DOMINANT BLEEDING DISORDER that occurs equally in males and females. Due to a deficiency in the vonWillebrand factor (a large glycoprotein with binding sites for Factor VIII, and facilitates platelet adhesion to collagen (important for platelet plug formation). THROMBOCYTOPENIA-the most common cause of hemorrhagic disorders (bleeding disorders). It is a blood disease characterized by abnormally low number of platelets in the bloodstream. Normal platelet count is

between 150K-450K cells per microliter of blood. When platelet numbers falls below 150K cells per microliter of blood, the person is “thrombocytopenic”. • Thrombocytopenia is common in people with Idiopathic thrombocytopenic purpura (ITP) where an autoimmune disease causes very low platelet counts. Abnormal reductions in the number of platelets are caused when abnormalities occur in any of 3 processes: 1. ↓ platelet production by bone marrow. 2. ↑ trapping of platelets by the spleen. 3. faster than normal destruction of platelets. Thrombocytopenia Clinical Features: • Spontaneous appearance of purpuric or hemorrhagic lesions of the skin which may vary in size from tiny, red pinpoint petechiae, to purplish ecchymosis and even massive hematomas. Patients also exhibit a tendency to bruise. • Nosebleeds, GI bleeding, and urinary tract bleeding. • Severe and often profuse gingival hemorrhage and petechiae on the oral mucosa. 2 Oral Surgery Concerns with Thrombocytopenia

Patients: post-operative hemorrhage caused by a decrease in blood platelets, and patients with the chronic form may be on steroids and have adrenal insufficiency. They may be unable to handle the stress of extractions. Drugs that can potentiate Bleeding post-extraction: aspirin, anti-coagulants, broad-spectrum antibiotics, alcohol, and anticancer drugs. Antianxiety drugs do NOT potentiate bleeding after extraction. • If a patient is taking any of these 5 drugs, be prepared to take special measures to control the bleeding. Patients with specific systemic diseases also have prolonged bleeding time (i.e. non-alcoholic liver disease, hepatitis, cirrhosis, and hypertension). • Excessive bleeding causes formation of hematomas, which increase the chance of infection. • 5 Ways to Obtain Wound Hemostasis: 1. Assisting natural hemostatic mechanisms: usually by placing a cotton sponge with pressure on the bleeding vessels or using a hemostat directly on the vessel. 2. Heat on the cut vessels (thermal coagulation). 3. Suture ligation of the vessel. 4. Placing a pressure dressing over the wound

(most bleeding from oral surgery can be controlled this way). 5. Vasoconstrictive substances like EPI in local anesthetics. TESTS TO MEASURE CLOTTING MECHANISMS: the most important consideration to rule out hemorrhagic disorders is history. 1. Normal Prothrombin Time (PT): < 11 sec (+ 2 sec). PT is the best test to determine if oral surgery can be safely performed on a patient taking COUMADIN (or any oral anti-coagulant). To be a good candidate for surgery, PT time should be within 5-7 seconds of the control sample. 2. Partial Thromboplastin Time (PTT): detects coagulation defects of the intrinsic system. The basic test for hemophilia. Normal value is 25-36 seconds. 3. Bleeding Time (Ivy Method): normal value is < 9 minutes. 4. Platelet Counts: normal value 150,000-450,000 per mm3 of blood. Minimal platelet count for oral surgery is 50,000. Routine Pre-Operative Tests for a patient being admitted to the hospital for surgery:

1. Complete blood count (CBC) that includes an evaluation of the hemoglobin and hematocrit indices. 2. Total WBC count with a differential count. 3. Assessment of circulating platelets 4. Urinalysis (a gross and microscopic urinalysis). 5. Patients scheduled for general anesthesia should have a chest x-ray (patients > 40yrs should also have an E.K.G.) Factors to Consider when deciding to hospitalize a patient for an Elective Procedure: • Medical problems compromising treatment (diabetes, hemophilia). • Difficulty and extent of surgery. • Consideration of the individual patient (emotionally disturbed, handicapped, etc.). • Hospitalization costs (time and money). Management of a Therapeutically Anticoagulated Patient: * Surgery is deferred until the platelet-inhibiting drugs have been stopped for 5 days. * Always consult with the patient’s physician to determine the safety of stopping the anti-coagulant for several days. Medical consultation is always indicated before surgery if the patient is taking anti-coagulant or anti-platelet therapy. * Take measures during surgery to help promote clot formation and retention.

* Restart the drug therapy the DAY AFTER surgery if no bleeding is present. Anti-Coagulants (Blood Thinners): Dicumarol, Heparin, Antithrombin III, Enoxaparin, & Warfarin. * Aspirin & NSAIDs inhibit platelet aggregation. Thus, if given to a patient already taking an anticoagulant, the effects can be life-threatening. * A patient on anticoagulant therapy (Warfarin, Heparin, Aspirin, or an NSAID) will most likely have a prolonged prothrombin time (PT) and bleeding time (BT). For elective extractions, this patient should stop taking the anticoagulant for 5 days prior to extractions. Always consult with the patient’s physician before recommending this. * When oral surgery is performed on patients taking anticoagulants, these steps may help prevent hemorrhage: ► Hemostatic agent is placed within the socket. ► Soft diet, intra-oral pressure packs, and avoid mouth rinses ► Multiple sutures are placed in the surgical area ► ice packs are applied extra orally. Ecchymosis-area of hemorrhage into the skin & subcutaneous tissue > 1cm in diameter often caused by injury, but clotting and bleeding disorders can predispose an

ecchymosis formation. Grossly, an ecchymosis presents as a bluish lesion in its earliest stages of onset. As RBCs in the lesion undergo progressive degeneration, & hemoglobin is converted through bilirubin→hemosiderin, the lesion progressively changes color from blue→green→purple→brownish discoloration. Post-operative Ecchymosis-a result of trauma to underlying blood vessels when blood escapes from the vascular tree and accumulates in the tissues. It is common after extractions in elderly patients due to the fragility of the vessel walls. All patients should be warned it can occur after extractions. Sometimes the patient will complain of a diffuse, non-painful, yellowish discoloration of the skin. Moist heat often speeds the resolution of postoperative ecchymosis. Osteoradionecrosis (ORN)-the most SERIOUS potential complication after extractions from areas previously irradiated. ORN is a condition of non-vital bone in a site of radiation injury, that can be spontaneous, but most often results from tissue injury. The absence of reserve reparative capacity is a result of the prior radiation injury. OSTEOMYELITIS-a bone infection characterized by progressive inflammatory destruction after formation of new bone. It is most often caused by STAPHYLOCOCCUS AUREUS (but can be caused by Streptococcus pyogenes, Pneumococcus species, Pseudomonas aeruginosa, and

Escherichia coli). Reduced blood supply predisposes bone to osteomyelitis. Osteomyelitis may be chronic or acute. It commonly results from a combination of local trauma (usually trivial, but causing a hematoma) and an acute infection originating elsewhere in the body. In children, long bones are usually affected, while in adults, the vertebrae and pelvis are most commonly affected. * Pus is produced in bone, which may cause a bone abscess that deprives the bone of its blood supply. * Chronic Osteomyelitis-results when bone tissue dies due to the lost blood supply. * Acute Osteomyelitis-occurs more frequently in the MANDIBLE than the maxilla because the blood supply in the maxilla is much richer and comes from many different arteries, while the mandible draws its main blood supply from the inferior alveolar artery. Also, the dense overlying cortical bone of the mandible prevents penetration of periosteal blood vessels, thus the mandibular cancellous bone is more likely to become ischemic and thus infected.

CPR GUIDELINES First step when initiating CPR is establish unresponsiveness by shaking the patient and shouting “are you OK”, then use the “ABCs”: Process is CAB: 1. Circulation: check the carotid pulse. 2. Airway: open the airway with the head tilt-chin lift. This is the easiest technique in most medical emergencies. 3. Breathing: look, listen, and feel. Cardiopulmonary Resuscitation (CPR): PROCESS: C-AB • C = CIRCULATION: Check for a carotid pulse by feeling for 5-10 seconds on the side of the victim’s neck. If there is a pulse, but the victim is not breathing, give rescue breathing at a rate of 1 breath every 5 seconds, or 12 breaths per minute. If there is no pulse, begin chest compressions as follows: • Place heal of one hand on the lower part of the victim’s sternum. With your other hand directly on top of the first hand, depress the sternum 1.5-2.0 inches.

• Perform 30 compressions for every 2 breaths (rate: 80-100 per minute). Check for pulse return every minute and continue uninterrupted until advanced life support (ALS) is available. • A = AIRWAY: * Place the victim flat on his/her back on a hard tissue. * Shake victim at the shoulders and shout “are you ok?” * If no response, call EMS (911) then, do a headtilt/chin lift (open the victim’s airway by tilting their head back with one hand while lifting up their chin with your other hand). • B = BREATHING: * Position your cheek close to the victim’s nose and mouth, look toward the victim’s chest and look, listen, and feel for breathing (5-10 seconds). * If patient is not breathing, pinch the victim’s nose closed and give 2 full breaths into the victim’s mouth. * If breaths will not go in, reposition the patient’s head and try again to give breaths. If it is still blocked, perform abdominal thrusts (Heimlich Maneuver). If CPR efforts are effective, the pupils will constrict. If too much pressure is incorrectly applied directly over the xyphoid process, the liver may be injured. The result of

interruptions in chest compressions while performing CPR is a reduction of blood flow and drop in BP to zero. Only discontinue CPR under these conditions: • Another trained person takes over the CPR for you. • EMS personnel arrive and take over care of the victim. • You are exhausted and cannot continue or the scene becomes unsafe. IF THERE IS AN AED ON THE SCENE, USE IT FIRST BEFORE STARTING CPR USING CAB

Most COMMON ERROR in recording BP is applying the blood pressure cuff TOO LOOSELY. This gives falsely elevated readings. Using the wrong cuff size can result in erroneous readings.

• A normal adult BP cuff placed on an obese patient’s arm produces falsely elevated readings. This same cuff applied to a very thin arm of a child will produce falsely low readings. • The compression cuff’s width should be ~20% greater than the extremity diameter on which the BP is being recorded. If you need to take additional readings, wait at least 15 seconds before re-inflating the BP cuff. CONGESTIVE HEART FAILURE (CHF)-heart failure resulting from progressive diseases that weakens the heart directly or cause an increased demand on the heart. CHF occurs most commonly in association with atherosclerotic coronary heart disease, valvular deformities, and hypertensive cardiopathy. *Usually the left ventricle fails first, follow soon after by right-sided failure. • Common CHF Signs: * Exertional dyspnea * Paroxysmal nocturnal dyspnea (patient wakes up gasping for air). Earliest & most common sign. * Peripheral edema (swollen ankles). * Cyanosis, Orthopnea (sitting or standing to breathe comfortably), and high venous pressure. • Treatment & Dental Management: prolonged rest, administer oxygen.

* Digitalis causes patient to be prone to nausea and vomiting. * Patients taking diuretics/vasodilators are prone to orthostatic hypotension, also avoid excessive EPI. * Dicumarol patients may have bleeding problems. • Nitroglycerin is given sublingually only to manage patients having chest discomfort or possible MI. It is not used for asthma attacks. Considerations When Checking Vital Signs: • The patient should not have had alcohol, tobacco, caffeine, or performed vigorous exercise within 30min of the exam. • Ideally the patient should be sitting with their feet on the floor and their back supported. The examination room should be quiet and the patient comfortable. • History of hypertension, slow or rapid pulse, and current medications should always be obtained. • Abnormalities of vital signs are often clues to diseases, and an alteration of vitals can be used to evaluate a patient’s prognosis. In complicated cases do not hesitate to contact a patient’s physician or previous dentist for a consultation.

ROUTINE VITAL SIGNS: 1. BP (normal 120/80) 2. Pulse Rate (normal 72) 3. Temperature can be measured several ways: ♦ Oral with a glass, paper, or electronic thermometer (normal 98.6°F or 37°C). ♦ Axillary (under arm) with a glass, or electronic thermometer (normal 97.6°F or 36.3°C). LEAST accurate. ♦ Rectal or “core” with a glass or electronic thermometer (normal 99.6°F or 37.7°C). MOST accurate. ♦ Aural (ear) with an electronic thermometer (normal 99.6°F or 37.7°C).

HORMONES, CALCIUM & BLOOD GLUCOSE Calcium levels are regulated by PARATHYROID HORMONE (increased hormone causes bone resorption) which then increases calcium levels. Calcium is also regulated to some extent by the kidney tubules and GI mucosa (lowering pH causes increased calcium absorption). Low serum calcium levels result in hyperirritability of nerves and muscles. Conditions Causing Increased Serum Calcium (Ca2+): • Hyperparathyroidism • Chronic Glomerulonephritis • Hypervitaminosis D • Malignant diseases of the skeleton (i.e. multiple myeloma) *Serum calcium is decreased in Diabetes Mellitus. Phosphorus concentration is also regulated by PARATHYROID HORMONE. Increased hormone causes the kidneys to increase the rate of phosphate excretion, which causes a decrease in plasma phosphate concentration.

In good health, the ratio of calcium to phosphorus in the blood is 10:4. If a glandular imbalance exists (especially with the parathyroid glands), then this ratio is maintained at a different level, causing long-term health deterioration. A high ratio of phosphorus to calcium sensitizes the body and increases inflammatory tendencies. BLOOD GLUCOSE concentration is regulated by INSULIN (↓ blood glucose) and GLUCAGON (↑ blood glucose). Glucose normally does not appear in urine, although it is freely filtered because it is reabsorbed in the proximal convoluted tubule of the kidney. • Serum glucose is increased in Diabetes Mellitus, adrenal tumors, ↑ growth hormone, and liver dysfunction. Normal serum concentration of glucose is 70-120mg/dl.

Adrenal cortex secretes 20mg of hydrocortisone daily. During stress, the cortex can increase its hydrocortisone

output to 200mg daily. Patients taking steroids or people with disease of the adrenals have a decreased ability to produce more glucocorticoids (hydrocortisone) during stress (i.e. extractions). This is because glucocorticoid secretion is stimulated by ACTH, which is produced in the anterior pituitary in response to stress by increasing ACTH output, thus glucocorticoid production increases. A relative lack of glucocorticoids also increases ACTH output. An overabundance of circulating systemic steroids inhibits ACTH production. Patients on large steroid doses repress ACTH production, causing atrophy of the adrenal cortex. A 52-year old women requests extraction of a painful mandibular 2nd molar. She tells you she has not rested for two days and nights because of the pain. Her medical history is unremarkable, except that she takes 20mg of Prednisone daily for Erythema Multiforme. To treat this patient, you would give steroid supplementation and remove the tooth with local anesthesia and sedation. • The fear here is that the patient may not have sufficient adrenal cortex secretion (adrenal insufficiency) to withstand the stress of an extraction without taking additional steroids (this hold true for any patient treated for any disease with steroid therapy).

• Erythema Multiforme-hypersensitivity syndrome characterized by polymorphous eruption and mucous membranes. Macules, papules, nodules, vesicles, or bullae and target (“bulls-eye-shaped”) lesions are seen. A severe form of this condition is “StevensJohnson Syndrome”. These patients may be receiving moderate doses of systemic corticosteroids, thus may be unable to withstand the stress of an extraction. Consultation with their physician is absolutely necessary before treating these patients. CUSHING’S SYNDROME (Hypercortisolism)-a relatively rare hormonal disorder caused by prolonged exposure of the body’s tissues to high levels of CORTISOL HORMONE. Most commonly affects adult’s ages 20-50yrs. 10-15 out of every 1 million people are affected each year. Most common cause of Cushing’s Syndrome is pituitary adenomas. Cushing’s Syndrome Symptoms: * Upper body obesity, rounded face, increased fat around the neck, thinning arms and legs. * Children tend to be obese with slowed growth rates. * Skin becomes thin and fragile, and bruises easily and heals poorly. * Purplish pink stretch marks may appear on the abdomen, thighs, buttocks, arms, and breasts. * Bones are weakened, and routine activities like bending, lifting, or rising a chair may cause backaches, rib and spinal column fractures.

* Most people have severe fatigue, weak muscles, high BP and high blood glucose levels. * Irritability, anxiety, and depression are common. * Women usually have excess hair growth on their face, neck, chest, abdomen, and thighs. Their menstrual periods may become irregular or stop. * Men have decreased fertility with diminished or absent desire for sex. A Cushing Syndrome patient’s cardiovascular status MUST be evaluated and treated if necessary prior to surgery. A person who has been on SUPPRESSIVE DOSES OF STEROIDS will take up to 1 year to regain full adrenal cortical function. Guidelines to help determine if a patient’s adrenal function is suppressed (if any doubt exists, consult with the patient’s physician): • People on small doses (5mg of Prednisone/day) will have suppression if they have been on the regimen for a month. • People taking an equivalence of 100mg Cortisol per day (20-30mg of Prednisone/day) will have abnormal cortical function in a week. • Short-term therapy (1-3 days) of even high dose steroids does NOT alter adrenal cortical function.

• A person on suppressive steroid doses will take as much as 1 year to regain full adrenal cortical function. • Stress or fatigue can cause an adrenal crisis in a patient with suppressed adrenal function. An abnormality in one or more of the pH control mechanisms can cause two types of major disturbances in the body’s acid-base balance. Depending on the condition cause, acidosis or alkalosis can be respiratory or metabolic: 1. Metabolic Alkalosis-a condition where the blood is alkaline because it has too much base or too little acid (high level of bicarbonate), occasionally causing an increase in blood pH. The major effect on the body is over-excitability of the nervous system, which may cause tetany (tonic spasm). • Major Causes: diuretics (thiazides, furosemide, ethacrynic acid) ingestion of alkaline drugs, vomiting gastric acid contents, or overactive adrenal gland as in Cushing’s Syndrome or use of corticosteroids). • Respiratory Alkalosis-a condition in which the blood is alkaline because rapid or deep breathing results in a low blood CO2 level. Major Causes: hyperventilation from anxiety, pain, liver cirrhosis, low levels of oxygen in the blood (high altitudes), & aspirin overdose. *Much LESS COMMON than respiratory acidosis.

• Treatment: ingestion of AMMONIUM CHLORIDE. 2. Metabolic Acidosis-a condition in which the blood has too much acid or too little base, often causing a decrease in blood pH (more acidic blood). When blood pH falls below normal (7.3), the CNS becomes so depressed the person first experiences disorientation, then is comatosed. *The normal blood bicarbonatecarbonic acid ratio is 20:1. A 10:1 ratio indicates uncompensated acidosis. Severe acidosis always occurs during CPR. Metabolic acidosis is excessive blood acidity characterized by an inappropriate level of bicarbonate in the blood. Major Causes: chronic renal failure, diabetic ketoacidosis, lactic acidosis, poisons, and diarrhea. • Respiratory Acidosis-excessive blood acidity caused by a buildup of CO2 in the blood due to poor lung function or slow breathing (decreased respiratory rate). Major Causes: hypoventilation, emphysema, chronic bronchitis, severe pneumonia, pulmonary edema, and asthma. • Treatment: ingestion of SODIUM BICARBONATE. DIABETES-the most common pancreatic endocrine disorder and metabolic disease involving mostly carbohydrates (glucose) & lipids owing primarily to a relative or complete lack of insulin secretion by beta cells of the pancreas. Diabetes is caused by an absolute deficiency of

insulin (Type I) or resistance of insulin’s action in the peripheral tissues (Type II). Classic triad of symptoms is polydipsia, polyuria, & polyphagia.

Symptoms of Diabetes Mellitus: glycosuria, polyuria, polydipsia, hyperglycemia, weakness, weight loss, ketoacidosis, and vascular abnormalities. • People with well-controlled diabetes are no more susceptible to infections than patients without diabetes. However, they have more difficulty containing infections due to altered leukocyte function. • Patients who take insulin daily and check their urine regularly for sugar & ketones (controlled diabetics), can usually be treated in the normal manner without additional drugs or diet alterations. If any doubt exists as to the patient’s medical status, consult with their physician and do not

assume anything! • Treatment of choice for hypoglycemia in an UNCONSCIOUS diabetic is IV injection of 50% dextrose in water. • Treatment of choice for hypoglycemia in a CONSCIOUS diabetic is administration of an oral carbohydrate (orange juice, cola beverages, candy bars) NUMBER ONE cause of KIDNEY DISEASE, responsible for 40% of all kidney failures is DIABETES. High BP is the 2nd cause (25%). Another form of kidney disease is Glomerulonephritis (a general term for many types of kidney inflammations). Genetic diseases, autoimmune diseases, birth defects, and other problems can also cause kidney disease. End-Stage Renal Disease (ESRD)-a condition in which there is a permanent and almost complete loss of kidney function. The kidney functions at < 10% of its normal capacity. In ESRD, toxins slowly build up in the body. Normal kidneys remove these toxins (i.e. urea and creatinine) from the body through urine. • Patients with ESRD are often on steroid therapy, are more susceptible to post-operative infections, and have an increased tendency to bleed.

• When oral surgery procedures are performed on these patients, meticulous attention to good surgical technique is necessary to reduce the risks of excessive bleeding and infection. When treating patients with Renal Insufficiency and patients on Hemodialysis: * Avoid using drugs metabolized or excreted by the kidneys. * Do not use NSAIDs, as they are nephrotoxic. * Perform oral surgery the day after dialysis. * Consult the patient’s physician for possible prophylactic antibiotics. Rheumatic Fever (RHD)-an acute inflammatory disease with systemic manifestations and particular involvement of the heart valves, which follows an upper respiratory infection with a Group A beta-hemolytic streptococcus. Although rheumatic fever may follow a streptococcal infection, it is NOT an infection, but an inflammatory reaction to an infection. * Rheumatic fever is most common in children ages 5-15yrs. The onset is sudden and often occurs 1-5 symptom free weeks after recovery from a sore throat or from scarlet fever. Mild cases may last 3-4 weeks, while severe cases may last 2-3 months. Treatment: Penicillin & Rest. * Clinical diagnosis or rheumatic fever is made when

two major (or one major and one minor) criteria (the “Jones Criteria”) are met. Major Jones criteria: carditis, arthritis, chorea, erythema marginatum, & subcutaneous nodules. Minor criteria include: fever, arthralgias, history of rheumatic fever, EKG changes, and lab tests. * Heart inflammation (carditis) disappears gradually usually within 5 months. However, it may permanently damage the heart valves, resulting in rheumatic heart disease. The mitral valve (valve between left atrium & ventricle) is most commonly damaged and may become leaky (mitral valve regurgitation), abnormally narrow (mitral valve stenosis), or both. The pulmonary valve is RARELY involved. * A history of rheumatic fever should lead the dentist to an in-depth dialogue history seeking the presence of rheumatic heart disease. If RHD is present, antibiotic coverage is indicated to minimize the risk of subacute bacterial endocarditis (SBE). 5 Major Areas Discussed when taking Patient Medical History: 1. Chief complaint 2. History of present illness 3. Specific drug allergies 4. Review of systems (heart, liver, kidney, brain) 5. Nature of systems

TOOTH FRACTURES Diagnostic Aids to Identify Vertical Root Fracture: 1. Fiberoptic light for transillumination. 2. Wedging the tooth in question and take an x-ray. 3. Persistent periodontal defects in an otherwise healthy tooth. 4. Have patient bite forcefully on a bite stick (tooth slooth). Root fractures are only visualized on a radiograph if the x-ray beam passes THROUGH THE FRACTURE LINE. Since the fracture line can extend diagonally, an additional radiograph is taken with a steep (45º) vertical angulation in addition to the conventional 90º degree. Inlays can cause fractures. If a patient complains of pain during mastication since inlay placement, suspect a fractured cusp (using a bite stick or tooth slooth helps determine which cusp is fractured). VERTICAL ROOT FRACTURE-symptoms and clinical tests show pulpal pathosis in a posterior tooth, but no decay or restoration in any proximity to the pulp on the radiograph is pathognomonic of a vertical tooth fracture. Vertical root fractures are cracks that BEGIN IN THE ROOT and

extend TOWARD the chewing surface and often present minimal signs and symptoms, thus may go unnoticed for a period of time. Often discovered when surrounding bone and gingiva become infected. Treatment: Extraction or endodontic surgery (ex: apicoectomy). ♦ Radiographic exam rarely reveals the fracture because the crack is usually parallel to the x-ray film. Radiographs (without first wedging the tooth) RARELY show vertical tooth fractures. ♦ Vertical fractures through root structures have an almost HOPELESS prognosis, unless the fractured segment can be removed, and gingivoplasty & alveoplasty are performed. However, unrealistic or overambitious case selection leads to failure. ♦ A tooth with a vertical root fracture has a POOR prognosis. Studies show most vertical root fractures are caused by using too much condensation force during obturation with guttapercha. Anterior tooth root fractures, usually occur in a more HORIZONTAL PLANE and may be visible on the radiograph. Anterior tooth fractures are usually due to accidental trauma (i.e. blow to the jaw/teeth). If the fracture line is not too far down the root, it may be saved with RCT

and a crown. Therapy for horizontal root fractures is always difficult. Root canal treatment (RCT) is NOT indicated if the fracture site remains in close proximity and if pulp retains its vitality. However, if clinical symptoms develop, or the segments appear to be separating on the x-ray, some treatment is necessary. CRACKED TOOTH SYNDROME-one of the most frustrating dental conditions involving the possible need for endodontic treatment, because its symptoms are usually characterized by a SHARP, but BRIEF PAIN occurring unexpectedly only when the patient is chewing. Having a patient bite forcefully on a bite stick (tooth slooth) and noticing the cusps that occlude when the pain occurs helps locate the cracked tooth. SPLIT TOOTH-often the result of the long-term progression of a cracked tooth. Identified by a crack with distinct segments that can be separated. The cracked portion is removed and restored with a restoration or crown unless the crack extends apical to the crest of bone, in which case extraction may be required. Submarginal Curved Flap (Semilunar Flap)-a half-moon shaped flap raised with a curved horizontal incision in the mucosa or attached gingiva with the concavity towards the apex. While it is simple and does not impinge on

surrounding tissue, its disadvantages outweigh its advantages, thus it is NOT used for anterior tooth rootend surgery. 1. limited access and visibility. 2. tearing of incision corners when trying to improve accessibility by stretching the flap. 3. if a lesion is larger than expected, the incision lies over the bony defect, and healing occurs by scarring. 4. incision extent is limited by attachments (i.e. frenum, muscles).

FLAPS & SLOB RULE Submarginal Triangular & Rectangular Flap (Ochsenbein-Leubke)-requires at least 4mm of attached gingiva and a healthy periodontium. This flap is raised by a SCALLOPED INCISION in attached gingiva with 1 or 2 vertical incisions. Less risk of incising over bony defects, with no post-surgical gingival recession. May be indicated for root-end surgery on an anterior tooth. • Advantages: better access & visibility than a semilunar flap, but NOT better than a full mucoperiosteal flap. • Disadvantages: hemorrhage from the cut margins & scarring. Full Mucoperiosteal Flap (Full-thickness)-allows MAXIMUM access & visibility. This wide flap is raised from the gingival sulcus (elevating gingival crest & interdental gingival), and its outline precludes any incisions over bony defects, and allows various periodontal procedures (i.e. curettage, SRP, bone re-shaping). Can be indicated for root-end surgery on an anterior tooth. Disadvantages: it’s a large flap that may be difficult to reposition, suture, make alterations, and post-surgical gingival recession is possible.

Electric Pulp Tester (EPT = Vitalometer)-usually elicits a response at a HIGHER current than normal if the tooth being tested has CHRONIC PULPITIS. EPT checks tooth sensibility/vitality by stimulating nerve endings with a low current and high potential difference in voltage. While EPT manufacturers give normal reference values of current the BEST way to check “normal/baseline” values is to use it on adjacent (non-pathological) teeth, then compare the normal values with the values obtained on the tooth in question. EPT Responses: 1. Acute pulpitis-indicated by a lower than normal current, as acute inflammation mediators lower the pain threshold. 2. Chronic pulpitis-indicated by a response at a HIGHER current than normal. 3. Hyperemia-indicated by a LOWER than normal current, but a higher current than with an acute pulpitis. 4. Pulp necrosis/Abscess-indicated by no response at any current level. EPT is NOT reliable in these circumstances: 1. Pus-filled canal or a nervous patient (gives a false +). 2. Recent dental trauma, an insulating restoration, or wearing gloves (gives a false (-) response). Do not wear gloves when using the EPT. 3. Secondary dentin deposits, moisture contamination, immature tooth (open apex), patient taking analgesics, improper application, or weak EPT batteries.

Buccal Object Rule (SLOB Rule = Same Lingual, Opposite Buccal)-a shift-cone technique/rule that allows the dentist to determine on the radiograph which canal is the buccal and which is lingual. Buccal object rule also allows one to determine working length of superimposed canals, root/canal curvatures, and the facial-lingual orientation of instruments or other anatomical objects. • The lingual surface is always CLOSEST to the cone, while the buccal surface is always FARTHER away from the cone. To apply SLOB rule, you MUST have a reference object. • If taken from the mesial, the lingual surface (ML canal) will appear more mesial (closer to the cone) than buccal surface (MB canal) which appears farther distally. • If taken from the distal, the lingual surface appears more distal (closer to the cone) than the buccal surface which appears father mesially. • According to buccal object rule, when the x-ray tube is repositioned either at a more mesial or more distal angulation and a film is exposed, the root/canal farther from the film (the buccal) moves in the SAME direction that the cone is directed. Thus, when the cone is aimed to the distal (angled from the mesial direction), the buccal root/canal moves to the distal and appears distal to the

lingual/palatal root/canal. • The object toward the lingual side (closer to the film) appears to shift on the film to the same direction as the repositioned x-ray cone. If the xray cone is angulated mesially, the lingual/palatal root shifts toward the same (mesial) side in the resultant radiograph, and is easily visualized. • When treating multi-canaled bicuspids & molars, it is often difficult to determine radiographically which canal is more toward the buccal. When a straight-on exposure is taken of a bicanaled tooth, the canals become superimposed on the film, and visualization of each canal is impossible. If the x-ray cone is moved to give an angled exposure, the roots will be separate on the film. Diagnostic tests indicated for recently traumatized teeth: 1. Soft tissue exam to observe lips, face, tongue, etc. 2. Hard tissue exam to visually look and then palpate the injured tooth and alveolus to reveal the extent of tooth mobility, any alveolar fractures, and areas of inflammation. Check for occlusal disharmonies to help detect tooth displacements and jaw fractures. 3. Radiographic examination to reveal tooth

displacement, root fractures, previous RCT, periapical radiolucencies. 4. Other diagnostic tests. EPT (pulp vitality testing) is CONTRAINDICATED since the traumatized pulp undergoes temporary paresthesia thus giving a false reading. A percussion test is NOT usually performed since it is painful. 5. Observe adjacent and opposing teeth for injury. Traumatized teeth may be fine for a long time, but many develop radiolucencies. Thus, do not indiscriminately do root canals before checking pulp vitality. Perform RCT only in teeth that do not respond to pulp testing (i.e. trauma to maxillary anterior teeth. A few years later x-rays reveal radiolucencies around incisor apices). Check pulp vitality of all anterior teeth before performing root canals. Trauma causing deep intrusion to a permanent tooth causes PULP NECROSIS and conventional RCT is necessary. Radiation Safety: Endodontic procedures involve taking multiple radiographs. 1. To protect the dentist and staff if there is no barrier available to stand behind, the stand at least 6 feet away in the area that lies between 90-135º to the xray beam (an area of minimal scatter radiation).

2. A fast (sensitive) film like “E-speed” film is preferred over slower films, since faster films require less radiation exposure while providing a quality image. 3. Dental units should operate at 70kVp or higher. The higher kVp = lower patient skin doses or radiation. 4. Collimation (i.e. restricting x-ray beam size so it does not exceed 2.5inch at the patient’s skin) decreases exposure. 5. Patients must be protected with a lead apron and thyroid collar for each exposure. 6. Dental personnel who may get exposed to occupational x-radiation MUST wear film badges to record exposure, and must never exceed the max permissible dose (MPD) of 50mSv per year per person. PULPOTOMY-removal of a portion of the pulp that is indicated for: 1. cariously exposed deciduous (primary) teeth with healthy radicular pulps. 2. traumatic or carious exposure of permanent teeth with undeveloped roots. 3. an alternative to extraction when endodontic treatment is not available. 4. emergency treatment in permanent teeth with acute pulpitis. Pulpotomy procedures performed on fully developed permanent teeth are NOT successful. Thus, it is only a temporary procedure for fully developed permanent

teeth. While doing a vital pulpotomy on a young, immature permanent tooth, if hemorrhage after pulpal amputation cannot be controlled with cotton pellets even after several minutes, the next step is to perform the amputation at a more apical level. • Uncontrolled bleeding is a sign of inflamed pulp tissue. Radicular pulp must be uninflamed for a successful pulpotomy. It is not uncommon to find uninflamed pulp at a more apical level, especially in cariously exposed teeth. • If bleeding does not stop even after more apical amputation, then hemostatic agents are used as a compromise treatment. These are closely monitored, and if pulp vitality is lost, then apexification (pulpectomy) procedures are performed. • Pulpotomy is the removal of the pulp chamber contents only. APEXIFICATION-goal is to INDUCE FURTHER ROOT DEVELOPMENT in a pulpless tooth (dead tooth) by stimulating the formation of a hard substance at the root apex to allow obturation of the root canal space. Apexification may be required after pulpectomy (as at age 7, the root apex must be open). The apex closes 2-3 years after eruption.

1. Isolate the field with a rubber dam, make an access cavity, and remove all pulp using reamers and files. 2. Premixed syringe of calcium hydroxidemethylcellulose paste (i.e. Pulpdent syringe) is injected into the canal and filled to the cervical level. The paste must reach the apical portion of the canal to stimulate tissues to form a calcific barrier. A double cement seal is made to close off the access cavity. 3. Patient is recalled after 3 months to see if apexification occurred. If not, a fresh supply of paste is placed. Once apexification occurs, only then is conventional RCT performed. Calcium Hydroxide (CaOH)-its action promotes formation of a hard substance at the root apex by creating an ALKALINE environment that promotes hard tissue deposition. If a permanent tooth fractures, but has a fully formed root and the pulp is exposed (large exposure), complete RCT is the treatment of choice. Apexification is NOT needed because the root is fully formed. If the pulp exposure is small and the length of time is short (30min-1hr), then direct pulp capping with CaOH followed by a restoration is the treatment of choice. Pulp Capping-placing a sedative and antiseptic dressing

on an exposed healthy pulp to allow it to recover and maintain normal function and vitality. Dycal (CaOH2 = calcium hydroxide) is the most commonly used dressing. • Pulp capping is overused in dentistry today, with very few indications for its use. Young pulps are more vascularized, thus more amenable to repair. Accidental exposure of the pulp and the pulp of a young child are two situations where pulp capping offers better success. Carious exposure of the pulp or the pulp of a middle-aged person are not indications for pulp capping. • Pulp cappings are more successful if the exposure was accidental (trauma or with a dental bur = mechanical) rather than carious. Also, the pulp exposure should only be pinpoint in size to expect success. Repair occurs when a dentin bridge forms at the pulp exposure site. Even a small carious exposure should have RCT for the best longterm prognosis. • Pulp capping-not recommended in primary teeth with carious pulp exposures due to its high failure rate and because pulpotomy (having similar time requirements) is very successful. Pulp capping can be done if a mechanical pulp exposure occurs. • A tooth can stay asymptomatic for several weeks

after pulp capping is performed, but this may only be temporary. If pulp capping fails and the tooth becomes symptomatic, it may be nearly impossible to treat with routine endodontics due to the severe calcifications in the root canal. Perforations may occur when attempting to follow the obliterated canal to gain apical patency (perforations into furcations of multi-rooted teeth have the poorest prognosis). • Traumatic blows to teeth also cause pulp space calcification sometimes to a point where locating the canal is very difficult. With primary teeth, trauma may cause calcifications in the pulp chamber, which can cause a yellowish discoloration of the tooth. • Pulp capping success is recognized by the formation of a complete barrier of dentin at the site of pulpal exposure. 1. INDIRECT PULP CAPPING-a calcium hydroxide base is placed on a thin layer of questionable dentin remaining over the pulp. It is performed when a carious exposure is anticipated. After a 3-4 month waiting period, the tooth is re-opened and remaining decay is removed. During the waiting period, hopefully secondary dentin formation occurred, allowing complete removal of the decay without pulp exposure. • Ex: a radiograph of a 1st molar shows gross decay that may involve a dental pulp horn. The ideal

treatment is to do an indirect pulp cap and place a sedative filling (IRM). If the tooth remains asymptomatic, in 3-4 months you can re-enter the tooth and remove all decay with subsequent placement of a permanent filling. *If this patient had tooth pain (aggravated by heat and tender to percussion), and excavation of the caries revealed pulp horn exposure without evidence of vital tissue, the emergency treatment pending eventual RCT is to place a small cotton pellet dampened with eugenol over the exposure and seal the cavity with a temporary material (IRM). 2. DIRECT PULP CAPPING-placed a calcium hydroxide base directly on a pulpal exposure. Favorable factors for direct pulp capping: * Visual evidence of un-inflamed (pink) pulp tissue. * Absence of copious hemorrhage through the exposure. * No previous symptoms of pulpitis. * Small non-carious exposure (mechanical pulp exposure) * Clean cavity uncontaminated with saliva. * Direct pulp capping is very successful in immature teeth. * Direct pulp capping involves prompt application of a setting calcium hydroxide cement to a small (< 1mm in diameter), well isolated traumatic pulp exposure. Direct pulp capping is expected in most instances,

to stimulate formation of a reparative “dentin bridge” over the exposure site and preserve underlying pulpal tissue in a healthy condition. * Direct pulp capping should NOT be attempted on teeth with a history of pain, sensitivity to percussion, or periapical radiolucencies. Instead, RCT may be indicated. * Failure of a direct pulp capping procedure is indicated by symptoms of pulpitis at any time, or the lack of vital response after several weeks or months. * Adverse responses that can occur after a direct pulp capping procedure: ► Physical or microbial insult to the pulp may cause persistent inflammatory changes that can culminate in partial or complete pulpal necrosis. ► Regulation of the mineralization process involved in dentin bridge formation may become deranged, causing extensive calcification and obliteration of the pulp canal space by mineralized tissue. ► Very rarely, the differentiation of odontoclasts may be induced with the development of internal resorptive lesions.

Criteria that must be met BEFORE a canal is obturated with gutta-percha: • Canal must be prepared in a manner that ensures optimum canal debridement and access to the apical area so the filling material can be condensed to obliterate the entire preparation. • Tooth must be asymptomatic, and canal must be dry at the time of fill. • If a bacteriologic culture test is being used, a negative culture must be obtained. A tooth prepared for RCT that responds to thermal tests, indicates inadequate debridement (cleaning), since a pulpless tooth should NOT respond to any stimuli. Thus, all nerve must be removed before the canal is obturated. Access Preparation Objectives: straight-line access, conservation of tooth structure, pulp chamber unroofing, and remove pulp horns. • Root canal access is the INITIAL STEP in canal preparation. Straight-line access to the apical foramen must be established to ensure free movement of the instrument during debridement and canal preparation. All subsequent treatment hinges on the correctness of the access preparation. • All access cavities are made through the

LINGUAL on anterior teeth & OCCLUSAL on posterior teeth. • A facial access is recommended for maxillary primary incisors. Access Preparation Mistakes: • During mandibular molar access preparation, two regions tend to be “overcut” resulting in undesirable over preparation of the canal access (mesial aspect under marginal ridge & lingual surface under lingual cusps). Mandibular molars tip mesially and lingually, thus if a bur is directed straight inferior, it may cause unnecessary loss of tooth structure in the mesial and lingual areas. • Mandibular incisors & maxillary 1st premolars are the EASIEST teeth to perforate during access preparation due the limited access mesiodistally. Thus, care must be taken when initiating treatment on these teeth. DEBRIDEMENT-removal of foreign material & contaminated or devitalized tissue from or adjacent to a traumatic infected lesion until surrounding healthy tissue is exposed. Chemomechanical debridement of the root canal system is the MOST crucial aspect of root canal treatment. • Achieving GLASSY, SMOOTH CANAL WALLS is

the BEST and most reliable indicator of adequacy of root canal debridement. • Obtaining clean canal shavings on a file or clean irrigating solution are INACCURATE measures to determine the end point of debridement. • MOST important consideration BEFORE filling a root canal is PROPER CLEANING (debridement) and SHAPING (instrumentation) of the canal. Once the canal is obturated, any organisms that enter periapical tissues from the canal are eliminated by the body’s natural defenses. The MOST common cause of root canal failure is incompletely and inadequately disinfecting the root canal system. • SECOND most common cause of root canal failure is LEAKAGE from a poorly filled canal. This is common even after apical curettage (i.e. RCT performed on a tooth with apical curettage of a lesion that was found to be a cyst. 3 years later the lesion was even bigger than before). Thus, the failure was due to LEAKAGE from a poorly filled canal. When a canal is prepared properly, any of the acceptable filling methods will produce a successful result as long as the canal is COMPLETELY filled. • Complete canal debridement is the MOST EFFECTIVE way to reduce canal microorganisms.

Debridement is done using various ways depending on the case, and may involve canal instrumentation, placement of medicaments/irrigants, and/or surgery. Objectives of Root Canal OBTURATION (Filling with Gutta-Percha): 1. Develop a fluid-tight seal at the apical foramen. 2. Complete filling of the root canal space. 3. Create a favorable biologic environment for the process of tissue healing. The importance of canal obturation (filling) is SECOND only to canal debridement. ~40% failures are caused by incomplete obturation of the canal (most failures are due to incomplete debridement). If the canal is not filled, tissue fluid and microorganisms from periapical tissues can enter the voids, causing endodontic failure. However, if an accessory (lateral) canal is not totally filled during obturation, the appropriate treatment is to observe the tooth and evaluate every 3 months. After RCT is completed on a tooth with a periapcial radiolucency, it takes 6-12 months before a marked reduction in the size of the radiolucency is evident on a radiograph. Desired periapcial tissue changes are regeneration of alveolar bone, deposition of apical cementum, and PDL re-establishment. Indications for using solvent-softened custom gutta-

percha cones: lack of an apical stop, abnormally large apical portion of the canal, or an irregular apical portion of the canal. Tugback within 1mm of working length is NOT an indication. • Studies show solvent softening DOES NOT ultimately result in a better apical seal. Thus, this time consuming procedure is not used if tugback (slight resistance to dislodgement) is achieved. The gutta-percha cone should also have a definite apical seat (should not be able to be pushed further apically). • If the canal preparation is properly flared, fitting the master cone is not a time-consuming procedure. A gutta-percha cone the same size as the last file used during preparation (MAF) is selected and placed as far as possible into the canal, but NOT past the working length. • Once tugback and apical positioning are obtained, a radiograph is taken to verify cone positioning. If an accurate determination and careful enlargement are performed, the x-ray will show the master cone reaching the most apical position of the preparation or extending just 1mm short of the apical foramen. When the cone is slightly short, the pressure of condensation & the sealer’s lubrication action are sufficient to produce complete seating of the cone. If the cone is > 1mm from the

radiographic apex, remove the cone and fit a smaller one, or instrument more in the apical third. • MAIN reason for recapitulation (using your MAF after each increase in file size) during canal instrumentation is to clean the canal’s apical segment of any dentin filings not removed by irrigation. WHEN REGAINING CANAL PATENCY: ♦ A “crown-down” sequence (larger to smaller) of instruments is used from coronal to apical. ♦ Rotary instruments work faster and improve the access early in the treatment compared to heated instruments. ♦ Very light apical pressure is applied when using Nickel Titanium (NiTi) rotary files. ♦ Over-extended gutta-percha cones had to be removed by extending the file periapically. ♦ Glass Bead Sterilizer-sterilizes endodontic files in 15 sec at 220º C (428ºF). If a gutta-percha cone passes past the apex, a file must then be used beyond the apex to avoid breaking the cone. A broken cone in the periapical area can cause orthograde re-treatment failure. ♦ Techniques to remove gutta-percha: rotary instruments, ultrasonic, heat, heat & instrument, and file & chemical.

CHLOROFORM-reagent of choice to DISSOLVE guttapercha. Highly concentrated chloroform is very effective, but use with caution because its vapor is potentially hazardous so it is dripped directly in the canal to avoid excessive flooding. ♦ Other chemicals that dissolve gutta-percha: xylol, halothane, benzene, carbon disulfide, essential oils, methyl chloroform, and white rectified turpentine. IRRIGANTS-destroy bacteria during endodontic therapy. Irrigant’s bactericidal action is much greater than the action supplied by intracanal medicaments, thus are used COPIOUSLY throughout the instrumentation phase of root canal procedures. 1. Sodium Hypochlorite (NaOCl)-the most commonly used IRRIGANT in endodontics to aid in canal preparation used in concentrations of 1%, 2.6%, or 5.25%. There is no agreed single-concentration value of NaOCl that is most effective while being the safest, thus these percentages are all acceptable. • 5.25% solution provides excellent germicidal solvent action, but is dilute enough to cause only mild irritation when contacting periapical tissue. Thus, gutta-percha points can be disinfected by placing them in 5.25% NaOCl solution for 1 minute. • NaOCl is a good tissue solvent, has some antimicrobial effects, and acts as a lubricant for root canal instrumentation. NaOCl is toxic to vital

tissue so ALWAYS use a rubber dam. 2. Hydrogen Peroxide (3% solution)-much less solvent action than NaOCl, but a widely used irrigant in endodontics as many clinicians use H2O2 & NaOCl alternately during treatment. H2O2 has two modes of action: • Bubbling action occurs when it contacts tissue, and certain chemicals physically foams debris from the canal (effervescent effect). • Liberation of oxygen strictly destroys anaerobic microorganisms. 3. Urea Peroxide (Gly-Oxide)-useful irrigant available in an anhydrous glycerol base (Gly-Oxide) to prevent decomposition. Better tolerated by periapical tissue than NaOCl, yet has greater solvent action and is more germicidal than H202, thus is an EXCELLENT IRRIGANT for treating canals with normal periapical tissue and wide apices. • Best use for Gly-Oxide is in NARROW and/or CURVED canals, utilizing glycerol’s slippery effect. CHELATING AGENTS-aid and simplify preparation of highly sclerotic canals after the apex is reached with a fine instrument. Chelating agents act on calcified tissues only with little effect on periapical tissue. Chelating agents

act by substituting Na+ ions that combine with dentin to form soluble salts for Ca+ ions that are bound in a less soluble combination. Thus, canal edges become softer, to facilitate canal enlargement. 1. Ethylene Diamine Tetra-Acetic Acid (EDTA)-a chelating agent that removes the mineralized portion (decalcify) of the smear layer. EDTA’s decalcifying process is self-limiting, and can decalcify up to a 50sm thin layer of root canal wall. o EDTA is normally used in 17% concentration. o RC-Prep & EDTAC are types of EDTA preparations. o EDTA has a limited value as an irrigation solution, since the decalcifying process induced by EDTA is self-limiting and stops as soon as the chelator is used up. o EDTA remains active in the canal for 5 days if not inactivated. Thus, upon appointment completion, the canal MUST be irrigated/inactivated with sodium hypochlorite (NaOCl) containing solution. 2. EDTAC-is EDTA + Cetavlon (a quaternary ammonium compound) with a greater antimicrobial action than EDTA, but has more inflammatory potential to tissues. NaOCl inactivates EDTAC. 3. RC-PREP-a foamy solution that combines functions of EDTA + urea peroxide to provide chelation & irrigation. RC-prep has a natural effervescence that is increased

by irrigation with NaOCl to help remove canal debris. Zinc Oxide-Eugenol (ZOE)-a cement with a long history of successful use as a based root canal sealer. The MAIN function of a root canal sealer is to FILL DISCREPANCIES between the core-filling material and the dentin walls. This function makes it even more important than the core filling material. ZOE (root canal sealer) functions: • Acts as a lubricant to facilitate gutta-percha cone placement. • Forms a bond between the gutta-percha and dentin walls. • Exerts antibacterial activity (some exert more than others) which is highest in the period of time immediately after its placement. • ZOE Disadvantages: staining, slow setting time, nonadhesion, and solubility. Most root canal sealers are some type of ZOE cement capable of producing a seal while being well-tolerated by periapical tissues. All sealers have some radiopacity (caused by metallic salts in the sealer), thus are visible on a radiograph. This radiopacity helps disclose any accessory canals, resorptive areas, root fractures, and the shape of the apical foramen. • Note: after filling a canal with gutta-percha, if there is a horizontal line of material (gutta-percha or sealer) extending both mesially & distally from the

canal to the PDL space, this indicates a ROOT FRACTURE. Mineral Trioxide Aggregate (MTA)-MOST superior retrofilling (reverse filling = retrograde amalgam filling) material and material of choice today. MTA seals the APICAL portion of the root canal, and is ALWAYS after an apicoectomy alone will not yield a good result. A reverse filling (MTA) MUST always be placed when an apical seal may be faulty (i.e. with a calcified root canal, it is impossible to obdurate most of the canal and get an apical seal. If just the root apex where cut off (apicoectomy), the incompletely filled canal might act as a source of reinfection. To prevent this, after the root tip is resected, the apical foramen is found, enlarged, and filled with zinc-free amalgam to create a seal. • Advantages: radiopaque, hydrophilic, biocompatible, non-toxic, induces hard tissue formation. • Disadvantages: difficult to manipulate & long-setting time. Apicoectomy (root resection = root amputation)-a procedure where the buccal tissue is flapped back, the buccal bone around the root apex and the root apex itself are removed, and the area is curetted out. Apicoectomy is a resection of the most apical portion of the root. It is best accomplished by obliquely RESECTING the most

apical portion of the involved root. • Indications: if a reverse filling (MTA) must be placed, it is necessary to gain access to an area of pathosis, the poorly filled apical portion of the root is to be removed to the level of canal obturation. Retreating teeth with posts are the MOST common reason for an apicoectomy and retrograde filling. • If a tooth with previous endodontic treatment becomes reinfected, it is best to retreat it conventionally by removing the filling material, debride the canals, and refill. However, if the tooth has been restored with a post, core, and crown, then apical curettage, then an apicoectomy and retrofill should be performed. PERIRADICULAR SURGERY INDICATIONS: 1. non-negotiable canal, blockage or severe root curvature in which non-surgical treatment is impossible. 2. complications arising from procedural accidents (i.e. instrument separation, ledging, or perforations) that cannot be handled without surgical exposure of the site. 3. failed treatment due to irretrievable posts or root fillings. 4. horizontal apical fractures where the apical end of the pulp becomes necrotic. 5. biopsy to diagnose non-odontogenic causes of symptoms (i.e. patient with a history of previous malignancy, lip parsthesia, or anesthesia).

PERIAPICAL CURETTAGE:-the same procedure as an Apicoectomy that flaps back the buccal tissue and buccal bone removal, but DOES NOT remove the root apex. Removal and examination of the diseased tissue and determining the extent of the lesion are the objectives of apical curettage. SUPEROXOL-the MOST COMMON bleaching agent for endodontically treated teeth that is a 30% aqueous solution by weight of H2O2 in distilled water. It is a potent oxidizing agent whose bleaching effect is due to direct oxidation of stain-producing substances. Tooth bleaching causes color change in enamel and dentin. • Chairside Technique: apply heat to Superoxolsaturated cotton pellets in the tooth chamber, and repeat until the tooth is lighter. This heat liberates the oxygen in the bleaching agent. • The most probable post-operative complication of tooth bleaching that has not been properly obturated is acute apical periodontitis. • Acute Apical Periodontitis (AAP)-characterized by pain commonly triggered by chewing or percussion. AAP alone does not indicate irreversible pulpitis. AAP indicates irritated apical tissues possibly associated with a vital pulp with a potential reversible pulpitis. In the absence of acute pain, a negative EPT or frank apical radiolucency, a carious

tooth with sensitivity to percussion may respond to caries control (temporary filling). If it does not respond to a sedative filling, RCT is indicated. • Walking Bleach Technique: place a thick paste consisting of sodium perborate and 2-3 drops of Superoxol in the tooth chamber with a temporary restoration. Several repetitions of this technique works well.

ENDODONTIC INSTRUMENTATION Broken Instruments: When a broken instrument (file) protrudes past the apex, surgery should be performed to remove it because it is a constant irritant (i.e. during cleaning & shaping of a canal, your K-file (#25) separates in the canal. Your first attempt to retrieve it results in a broken instrument through the apex). To manage this case, raise a flap and remove the instrument surgically, then fill the canal with gutta-percha. • It is easier to retrieve an instrument if it is wedged coronal or at the curvature of the canal, but very difficult if the instrument has passed the canal curvature. • When an instrument breaks off anywhere in the canal and a periapical radiolucency is present and minimal canal enlargement has been performed before the accident, surgery is indicated since the periapical tissues have had little opportunity for healing to be stimulated. You would prepare and obdurate to the point of blockage, and then perform an apicoectomy Sand retrofilling. • When an instrument breaks off in the canal’s apical third and is lodged tightly with no evident periapcial radiolucency, the remaining root canal space can be

filled with gutta-percha, the patient is informed, and placed on a 3-6 month recall. • Prognosis of a tooth with a broken instrument is best if the tooth had a vital pulp and no periapical lesion. Nickel titanium instruments (NiTi)-hand-operated or engine-driven to CLEAN & SHAPE canals. NiTi methods: • Push & pull stroke, reaming motion, or engine-driven rotary motion (uses only a reaming motion). • Hand instrumentation is done by either filing (push & pull) or reaming (repeated rotations). • The instrument’s action, NOT type of instrument used, determines the canal preparation’s general shape: o Filing-a push-pull action with emphasis on the withdrawal stroke. Its efficiency is greater with files than with reamers to remove dentin due to the greater number of flutes in contact with canal walls during the rasping motion of removing the file. Filing action produces a canal that is irregular in shape, thus a canal prepared with this push-pull filing action MUST be filled with gutta-percha in a condensation procedure.

o Reaming-repeated clockwise instrument rotation especially during insertion. A reaming action produces a canal that is relatively ROUND in shape. A reaming method is usually most efficient if using a silver cone to fill a canal. o Circumferential filing-a push-pull filing action that SCRAPES canal walls to create a smooth, tapered preparation. The file is moved first towards the canal’s buccal side, reinserted, then removed slightly mesially. This is done all the way around the canal until all dentin walls are planed. Circumferential filing technique enhances preparation when a flaring method is used. Important: A canal is instrumented and shaped so it has a continuously TAPERING FUNNEL SHAPE. The canal’s widest diameter should be at the orifice (opening), while its narrowest diameter is at the dentinocemental junction (DCJ or .5-1.0mm from the radiographic apex). Ideally, .5-1.0mm from the canal apex is where all teeth should be filed to and filled. Barbed Broaches-intracanal stainless steel instruments designed to remove pulp tissue, cotton pellet absorbent points, and other soft materials. It is NOT used for canal enlargement. The barbs are notched out of the instrument shaft and represent a weakened point. If the broach is not

used with utmost care or is forced apically, the barbs will bend and engage the canal walls, making removal difficult. Hedstrom files-an effective “H-Type” stainless steel CUTTING instrument made using a sharp, rotating cutter to gauge triangular segments out of a round blank shaft to produce a very sharp edge. Used carefully with ONLY a filing action, this file successfully planes dentin walls much faster than K-type files or reamers. S-file-a modified Hedstrom file. K-Type Instruments: 1. K-Files-the most useful instruments for removing hard tissue to enlarge canals. Files are made by twisting a blank (a square stainless steel rod producing a series of cutting flutes). The K-type file’s action in the canal is a clockwise-counterclockwise motion while directing pressure apically (can be a filing or reaming action). Ktype files are the STRONGEST and cut the LEAST aggressively. K-flex file-a modified K-type file. 2. Reamers-manufactured similar to files, but with fewer flutes. Reamers are used in canal preparations to SHAVE DENTIN using ONLY a reaming action to enlarge canals. Reamers remove intracanal debris with a CLOCKWISE reaming action, and place materials into the apical portion of the canal using a COUNTERCLOCKWISE rotation.

5 FACTORS TO MANAGE TRAUMATIC TOOTH AVULSION INJURIES: 1. Time: the time interval from injury until tooth replacement is a MAJOR factor in maintenance of PDL viability & subsequent root resorption. Teeth replanted within 30min exhibit very little resorption, while most teeth replanted after 2hrs have extensive external root resorption (MAIN cause of failure of replanted teeth is external root resorption). 2. Storage Media: if the tooth cannot be replanted immediately, proper storage can favorably influence PDL cell viability. MILK is the BEST storage media due to its near neutral pH (6.5-6.8) and osmolality which is conducive for cell survival. Physiologic saline and saliva are other tooth storage media. 3. Tooth Socket: should not be damaged by curettage or forceful replantation. 4. Root Surface: should not be scraped, dried, or manipulated with caustic chemicals. 5. Splint Stabilization: a splint that allows physiologic movement is placed for a maximum of 2 weeks to allow the initial reattachment of the PDL fibers. *When a tooth has been avulsed (out) of the mouth for > 2hrs, the treatment of the tooth socket, root surfaces, and

time required for splint stabilization changes. Proper management of an avulsed PERMANANT tooth replanted by the dentist within 2hrs of the accident: • 10-14 days after replantation, clean and shape (prepare) the root canal, and place calcium hydroxide paste into the canals. Replace this paste every 3 months for on year. After 1 year, if the root resorption has reversed or stopped, a permanent gutta-percha filling can be placed. Proper management of an avulsed PERMANANT tooth out for > than 2hrs of the accident: • Ankylosis & external root resorption will probably occur within 2 years. Ankylosis caused by the replacement gives a better prognosis than external root resorption which leads to failure. • RCT is performed in its entirety PRIOR to replantation. • Soak the tooth in 2.4% fluoride solution acidulated at 5.5pH for at least 20min. Fluoride slows the resorptive process. • Gently curette the blood clot out of the alveolar socket and irrigate with saline. • Rinse the tooth with saline, replant it into the socket, and splint for 4-6 weeks. Avulsed Tooth Management: • After 60 minutes of dry storage of an avulsed tooth,

few periodontal ligament cells survive. • Storage of an avulsed tooth in tap water is as bad as dry storage. • Saliva is hypotonic, thus allows storage for up to 2hrs, but MILK has a maximum storage time of 6hrs. • Teeth with complete root development should be treated endodontically ASAP even if replanted within 30 min. • Teeth with incomplete root development that are replanted within 30 min may not require endodontic treatment. ROOT RESORPTION-the MOST frequent sequela to avulsed tooth replantation. EXTERNAL ROOT RESORPTION-caused by periradicular inflammation, dental trauma (resulting in damage to attachment apparatus), excessive orthodontic forces, impacted teeth, bleaching of non-vital teeth. ALWAYS accompanied by bone resorption and is the chief cause of failure of replantation of permanent teeth (the main cause of failure of replanted teeth). 3 Types of External Root Resorption: 1. Surface Resorption-caused by acute injury to the PDL & root surface. If injury is not repeated, healing occurs forming new cementum and PDL. Root surface resorption is limited to cementum, may heal itself, and is not visible on a radiograph.

2. External Inflammatory Resorption-external resorption in which an infected pulp may further complicate the resorptive process. Characterized by bowl-shaped resorption areas involving cementum and dentin that rapidly progresses and continues if treatment is ignored. • Necrotic pulp and bacteria are necessary components of inflammatory resorption. Thus, this process is arrested with immediate RCT (open the tooth, clean & shape the canal, and place calcium hydroxide paste in the canal every 3 months for 1 year). If resorption stops after 1 year of treatment, then place a permanent root canal filling (gutta-percha). A calcium hydroxide-based root canal sealer is strongly recommended. • cervical root resorption does not occur exclusively at the cervical area of the root. Pulp does NOT play a role in cervical root resorption. 3. Replacement Resorption (Ankylotic Resorption)external root surface resorption that becomes substituted by bone, causing ankylosis (common in unsuccessful avulsed tooth replant cases). Replacement resorption accompanies dento-alveolar ankylosis due to extensive trauma to the tooth’s attachment apparatus. It is characterized by progressive replacement of the root by bone. Histologically, it shows direct contact between dentin and bone, with no intervening PDL or cemental layer.

• Pathognomonic signs of external resorption: lack of mobility, metallic sound to percussion, and infraocclusion of the involved tooth in the developing dentition. EXTERNAL CERVICAL INFLAMMATORY RESORPTION (ECIR)-cause if often unknown, but is correlated with orthodontics, trauma, and fractures. Typically presents as vascular granulation tissue that creates a cavity at the CEJ. Prognosis depends on degree of resorption, which can be determined with CBCT (Cone-Beam CT). Treatment: SRCT to gain access to the full extent of the lesion with surgical operating microscope. Trichloro-acetic acid may be used to cauterize the vascular lesion, and areas of resorption are removed with small burs. INTERNAL ROOT RESORPTION (INFLAMMATORY) caused by dental trauma, partial removal of pulp (pulpotomy), caries, pulp capping with calcium hydroxide, or a cracked tooth. Causes loss of pulp vitality & subsequent infection of coronal pulp causing inflammation. Inflammation due to infected coronal pulp is generally the cause of internal resorption. Teeth with internal resorption have a history of trauma, crown preparation, or pulpotomy. • Internal resorption is often precipitated by traumatic injury to the tooth. Undifferentiated reserve C.T. pulp cells are activated to form dentinoclasts that resorb the tooth structure in

contact with the pulp. • Internal resorption is usually asymptomatic, and discovered on routine radiographic evaluation. The root canal’s anatomic configuration is altered and increases in size, appearing as an irregular radiolucency anywhere along the canal space. Radiographic presentation of internal resorption is a fairly uniform enlargement of the root canal space. The root canal “disappears” into the lesion. • Sometimes on a radiograph, an external resorptive lesion can superimpose the canal space to mimic internal resorption. In such cases, another radiograph is taken at a different angle since the radiolucent lesion inside the canal space will not shift. • A tooth with internal resorption may respond to pulp vitality tests, but when detected, a pulpectomy should be performed. Once the pulp tissue responsible is removed, all resorption stops. To “wait and see” may cause sufficient destruction to perforate the root. • While internal resorption can occur ONLY when some pulp tissue is still vital, a NEGATIVE sensibility test does NOT rule out this etiology. • Pink Tooth-a pathognomonic sign of internal

resorption (not replacement resorption), and sometimes a sign of cervical root resorption, characterized by a pinkish appearance of the tooth due to granulation growth undermining the coronal dentin. Intentional Replantation (Replant Surgery)-a tooth that requires endodontic therapy is purposely REMOVED from its socket, some type of canal or apical preparation and/or filling is performed, and the tooth is RETURNED to its original socket. Replant Surgery Indications: 1. When routine endodontic therapy is impractical or impossible. 2. When a canal is obstructed via a broken instrument or calcification, and periapical surgery is impractical (a lower molar with the mandibular canal is close proximity). 3. When perforating internal or external is present, yet surgery is impractical. 4. When previous treatment has failed, but non-surgical treatment or surgery is impractical. Intentional replantation is considered ONLY when there is no other alternative treatment to maintain a “strategic” tooth. Long-term follow-up is required to monitor for complications, periodontal defects, and ankylosis with replacement resorption. Procedures

essential

for

successful

intentional

replantation of an avulsed tooth include: skillful extraction, minimum out-of-socket time, minimum root damage while the tooth is held during apical root end preparation, and repair of any perforation or resorptive defect before replantation. • Curettage of the socket to remove periapical pathosis is unnecessary because the socket wall should be MINIMALLY manipulated during replantation. Replantation of a PRIMARY TOOTH is NOT recommended due to potential danger to the permanent successor tooth from sequels of trauma (i.e. infection, ankylosis, or damage due to manipulation during the procedure). Thus, if a mother of a 4 year old child calls your office reporting that her child has suffered a trauma to one of his front teeth that has avulsed out of the socket, advise the mother to leave the tooth and come to the office immediately.

PULP, DENTIN, & CEMENTUM PULP-primary function of pulp is DENTIN FORMATION. Other pulp functions: 1. induction-forms dentin which then induces enamel formation. 2. nutrition-dentinal tubules are linked to the pulp to maintain pulp hydration and formation of peritubular dentin. MANTLE DENTIN-the first formed dentin that is laid before the odontoblast layer is organized. Thus, the pattern of deposition and size of collagen fibers differs from circumpulpal dentin. It is less mineralized than circumpulpal dentin and contains VAN KORFFS FIBERS (thick Type III collagen). Lays closest to the DEJ. INERGLOBULAR DENTIN-separates CIRCUMPULPAL DENTIN.

MANTLE

and

DENTIN TYPES: 1. Primary Dentin-forms the initial shape of the tooth, and is deposited before completion of the apical foramen. The majority of primary dentin is CIRCUMPULPAL DENTIN (forms most dentin surrounding the pulp, except the space that occupies

secondary dentin). Has more tubules than secondary dentin. 2. Secondary Dentin-dentin formed AFTER completion of the apical foramen at a slower rate than primary dentin as functional stresses are placed on the tooth. It is a regular and uniform layer of dentin around the pulp cavity. The junction between primary and secondary dentin is characterized by a sharp change in the direction of the dentinal tubules. 3. Reparative (Tertiary) Dentin-formed very rapidly in response to irritants. Throughout life, dentin responds to environmental changes (normal wear, caries, operative procedures). These changes initiate the deposition of reparative dentin which is limited to the site of irritation. The composition or reparative and secondary dentin is same, except reparative dentin is more irregular and they differ only in location and deposition. If the environment insult is strong enough, it will kill the odontoblast and its tubular process, leaving the tubule empty. If there is a collection of empty tubules (dead tracts), in time these dead tracts calcify and become sclerotic dentin. 4. Sclerotic Dentin-are dentinal tubules that become calcified. Hypermineralized peritubular dentin that reduces the width of dentinal tubules.

As PULP ages, the number of reticulin fibers DECREASES (pulp becomes less cellular and more fibrous), and the size of the pulp decreases due to the continued deposition of dentin. However, as pulp ages, collagen fibers & calcifications within the pulp (denticles or pulp stones) both increase. • Pulp stones-calcifications associated with chronic pulpal disease from advanced carious lesions or large restorations. Pulp contains myelinated (sensory) & unmyelinated (motor) nerve fibers that are afferent & sympathetic. Unmyelinated fibers regulate the lumen size of the blood vessels. However, proprioceptors (respond to stimuli regarding movement) are NOT found in dental pulp. • Free nerve endings-the only nerve ending found in pulp. Free nerve ending is a specific pain receptor. Regardless of the source of stimulation (heat, cold, pressure), the ONLY response will be PAIN. • Cells Found in Dental Pulp: fibroblasts (the main cell), odontoblasts, histiocytes (macrophages), & lymphocytes. • Cells Found in Diseased Pulp: PMN’s, plasma cells, basophils, eosinophils, lymphocytes, & mast cells (contain histamine & heparin).

At the ONSET of pulpal inflammation, these cells are involved in the cellular response: plasma cells, macrophages, & lymphocytes, but NOT Polymorphonuclear (PMN) Leucocytes. • The onset of pulpal inflammation is an insidious process characterized by a chronic cellular response of plasma cells, macrophages, and lymphocytes. There is no direct pulp exposure to caries, thus the cellular response is not acute. • After pulp exposure, acute inflammatory cells (mainly PMNs) are chemotactically attracted to the area. Histologically, the tissue is likely to show signs of acute inflammation near the exposure site, and a band of chronic inflammatory cells b/t the acute inflammation and the underlying normal pulp. • The response of vital pulp to microbial invasion is very resistant based on observation that even after 2 weeks of traumatic pulp exposure, only 2mm of coronal pulp may “give in” to microorganisms. However, non-vital pulp is a “fertile ground” for microbial growth. • Carious exposures in permanent teeth pulp usually require RCT. Immature (open apex) permanent teeth with carious exposures can be treated with pulp capping or a pulpotomy. o Pulp capping-not recommended in primary

teeth with carious pulp exposures due to its high failure rate and because pulpotomy (having similar time requirements) is very successful. Pulp capping can be done if a mechanical pulp exposure occurs. APICAL portion of pulp contains more collagen than the coronal portion. This facilitates a pulpectomy using barbed broaches or endodontic files. ♦ Type 1 & 3 collagen is mainly found in pulp in a 55%:45% ratio. Type 5 collagen is found in only small amounts. ♦ Type 1 collagen predominates in DENTIN. ♦ Odontoblasts synthesize Type 1 collagen. ♦ Fibroblasts in pulp synthesize Type 1 & 2 collagen. Central Zone (Pulp Proper)-area that contains large nerves & blood vessels, and is lined peripherally by a specialized odontogenic area that has 3 layers: 1. Cell-rich zone-INNERMOST pulp layer that contains fibroblasts. 2. Cell-free zone (zone of Weil)-pulp layer rich in capillaries, nerve networks, and contains the Nerve Plexus of Rashkow. 3. Odontoblastic layer-OUTERMOST pulp layer that contains odontoblasts, and lies next to predentin & mature dentin.

♦ The absence of the predentin layer predisposes dentin to internal resorption by pulp cells. Immediately adjacent to the odontoblastic layer in the pulp, 10-47.m of the dentin matrix remains unmineralized. If this unmineralized dentin layer is lost due to trauma or an infectious process, it predisposes the dentin to internal resorption by odontoclasts.

CEMENTUM CEMENTUM-hard dental tissue that is 50% mineralized like bone. Cementum covers the ANATOMICAL root surfaces of teeth. Cementum is formed by PDL cementoblasts and is nourished by PDL. CEMENTUM-main function of cementum is ATTACHMENT of the PDL principal fibers. Cementum also functions to compensate for loss of tooth surface due to occlusal wear by apical deposition of vital cementum throughout life. Cementum also protects the root surface from resorption during vertical eruption and tooth movement, and has a reparative function that allows reattachment of C.T. after periodontal treatment. CEMENTUM contains 2 Types of Collagen Fibers: 1. Sharpey’s Fibers-the terminal portions of PDL principal fibers embedded in cementum (run PERPENDICULAR to cementum) on one end and alveolar bone on the other. 2. Collagen Fibers-found within the cementum itself running PARALLEL to the cementum surface. Found in gingival C.T. Function to provide firmness to attached gingival tissue and underlying CEMENTUM & ALVEOLAR BONE.

Cementum Characteristics: • Cementum is slightly softer and lighter in color (yellow) than dentin and is formed by PDL cementoblasts (dentin is formed by pulp odontoblasts). • Contains cementocytes (trapped cementoblasts) in lacunae. • Cementum most closely resembles BONE, except there are no haversian systems or blood vessels (cementum is avascular, while bone is vascular). Cementum has NO NERVE INNERVATION. • 50% inorganic (hydroxyapatite), 40% organic (mainly collagen and protein), and 10% water. • Cementum is important in ORTHODONTICS as it is more resistant to resorption than alveolar bone, permitting orthodontic movement of teeth without root resorption. TWO TYPES OF CEMENTUM (functionally they are the same): 1. Acellular Cementum-contains no cells, and predominates on the coronal two-thirds of the root (found at the CEJ). It is thinnest at the CEJ. 2. Cellular Cementum-contains cementoblasts, inactive cementocytes, fibroblasts from the PDL, and cementoclasts. Found on the apical third of the root (apex), and is thicker to compensate for attritional wear of the occlusal/incisal surface and passive eruption of

the tooth. Root hypersensitivity decreases as the tooth forms CELLULAR CEMENTUM. CEMENTUM is THICKEST at the ROOT’S APICAL THIRD and THINNEST at the CORONAL THIRD. The MAIN FUNCTION of CEMENTUM is to ATTACH & ANCHOR THE PDL PRINCIPAL FIBERS TO THE ROOT from the surrounding alveolar bone. Additional Cementum Functions: 1. Compensates for the loss of tooth surface due to occlusal wear by apical deposition of cementum throughout life. 2. Protects the root surface from resorption during vertical eruption and tooth movement. 3. Reparative function that allows reattachment of C.T. after periodontal treatment. HISTOLOGICALLY, cementum differs from enamel because it has collagen fibers and cellular components in the mature tissue. CEMENTIOD-the peripheral uncalcified cementum.

layer

of

developing,

RADICULAR CEMENTUM-cementum found on root surfaces whose thickness INCREASES with age. It is

thicker apically than cervically, and its thickness ranges from 0.05mm-0.6mm. ♦ Cellular Cementum-found on the APICAL THIRD of the root, and contains cementocytes in lacunae in its cementum matrix. It occurs more frequently on the apical third of the root and in furcations. It is usually the thickest to compensate for attritional wear of the occlusal/incisal surfaces and passive eruption of the tooth. Cellular cementum is formed AFTER the tooth reaches the occlusal plane. ♦ Acellular Cementum-the FIRST cementum to be formed, does not contain any cells in its matrix, and usually covers the root’s CORONAL TWO-THIRDS (cervical third or half of the root). It is thinnest at the CEJ and plays a major role in TOOTH ANCHORAGE. Coronal Cementum-forms on the enamel covering the crown. If VITAL cementum is resorbed or nicked in surgical procedures, the defect is repaired by the deposition of new cementum. Repair cannot occur where pockets exist or where the gingiva has receded and cementum is exposed. Exposed cementum that forms part of the clinical crown is often removed during scaling, root planing, or brushing. Deposition of new cementum continues periodically throughout life, whereby root fractures may be repaired. Cementum is indistinguishable on radiographs.

MANDIBUAR AND MAXILLARY ROOT ANATOMY MANDIBULAR TEETH ROOT ANATOMY: the lingual wall of mandibular teeth is most easily perforated when preparing an access opening due to the lingual inclination of these teeth. • Mandibular 1st Premolar: 25% MAY have 2 canals with 2 apical foramina. Thus, treatment can be tricky. At least 23% can have 2 or 3 canals starting anywhere down the root. If a straight-on preoperative radiograph of a mandibular 1st premolar shows the pulp canal disappearing in midroot, this is an important indication that 2 canals are present. Pulpitis can cause referred pain to the MENTAL REGION OF THE MANDIBLE. • Mandibular 2nd Premolar: 97% have 1 canal at the apex. It has less variation than the mandibular 1st premolar. It usually has 1 root and 1 well-centered canal, with an oval access opening. The mental foramen lies in close proximity to the root apex, so avoid overinstrumentation and overfill, and do not misdiagnose the foramen as a premolar abscess on a radiograph. Thus, before performing root canal therapy, ensure that all diagnostic test

confirm your finding. Pulpitis can cause referred pain to the MENTAL REGION OF THE MANDIBLE. • Mandibular Molars: usually have a TRAPEZOIDAL outline of the pulp chamber formed by two canals in the mesial root, and one oval canal in the distal root. ~40% of cases, the distal root may have a 2nd canal (4th canal overall). The pulp chamber is located in the mesial 2/3 of the crown. Look for a 4th canal if the first canal found in the distal root lies more toward the buccal, rather than in the center. • Mandibular 1st molar requires endodontic treatment more than any other tooth in the oral cavity. Pulpitis can cause referred pain to the EAR. • Mandibular Canine: the root canal is THIN mesiodistally, but WIDE labiolingually. They usually have 1 root, but in rare cases may have 2 separate roots. The access opening is a large OVAL with the greatest width placed incisogingivally. This tooth usually has a slightly labial axial inclination of the crown. Thus, the access opening must be directed towards the lingual surface. Pulpitis can cause referred pain to the MENTAL REGION OF THE MANDIBLE. • Mandibular Central Incisor: has ONLY 1 ROOT that is narrow M-D, but relatively wide

labiolingually. The root may have a distal and/or lingual curvature, and 2 CANALS may be present (if so, the labial canal is straighter). Access opening for a mandibular central or lateral is a LONG OVAL, with the greatest width placed incisogingivally, and the incisal extent very close to the incisal edge. Pulpitis can cause referred pain to the MENTAL REGION OF THE MANDIBLE. MAXILLARY TEETH ROOT ANATOMY: • Maxillary Incisors & Canines: ALL have 1 root, 1 canal, & a distal axial inclination. Thus, when penetrating along the long axis of the tooth, the bur must be slightly angled toward the distal surface to avoid perforation of the mesial portion of the root. ♦ Maxillary Central Incisor: ALWAYS has 1 root & 1 canal. The root is bulky with a slight distal axial inclination, but rarely has a dilacerations. The pulp chamber access opening is OVALTRIANGULAR (it is somewhat triangular, as opposed to oval). The triangle base will be the facial, and triangle apex is lingual. If it is not triangular, then it must be oval. Pulpitis can cause referred pain to the FOREHEAD. ♦ Maxillary Lateral Incisor: is MOST likely to have a curved root. It ALWAYS has 1 root with 1 canal. The root is more slender than in the maxillary central incisor and often (55%) of the

time has a distal and/or lingual curvature or dilacerations. The access opening is OVAL. Pulpitis can cause referred pain to the FOREHEAD. ♦ Maxillary Canine-ALWAYS has 1 root & 1 canal, and is the LONGEST tooth in the maxillary arch. The access opening is OVAL. Pulpitis can cause referred pain to the NASOLABIAL AREA. • Maxillary 1st Premolar: ALMOST ALWAYS has 2 canals. ~60% have 2 roots (1 B, 1 Palatal) each with 1 canal. The buccal & palatal roots can be completely separate, or twin projections rising from the root’s middle 1/3 to the apex (this is more common). These 2 roots are usually equal in length from apex to cusp. However, the palatal root and canal may be wider. • ~40% of maxillary 1st premolars, only 1 root is present (usually with two separate canals). In cross section at the cervical line, the canal is shaped like a FIGURE EIGHT (ELLIPSE). The access is a THIN OVAL. Be careful not to perforate on the mesial (due to the mesial concavity). • Maxillary 2nd Premolar: have a higher incidence of accessory canals (60%) than maxillary 1st premolars. A pulpitis of this tooth MOST often refers PAIN to

the TEMPORAL REGION and sometimes NASOLABIAL REGION. If careful diagnosis does not reveal the affected tooth, other teeth and related anatomic structures become suspect. Referred painoccurs when a pulpitis in one tooth causes pain in another area. • Usually (85%) have 1 root, while (15%) 2 separate roots exist (each with 1 canal). The access opening is also a THIN OVAL (exactly like maxillary 1st premolars). • When only 1 canal is present in a maxillary 1st or 2nd premolar, it is usually in the CENTER of the access preparation. If only 1 canal is found, but is not in the center of the tooth, it is probable than another canal is present. Overfilling either premolar may force gutta-percha material directly into the MAXILLARY SINUS. • Maxillary Molars: have a TRIANGLE pulp chamber outline. The pulp chamber floor is formed by the buccal canals, and the apex is formed by the palatal canal. The line connecting the mesial & palatal canals is the longest. If a 4th canal exists, it is usually lingual to the orifice of the MB canal, and in the MB root. The 4th canal is more common than previously believed. • Maxillary 1st Molar: its 3 canal orifices are arranged in a TRIANGLE shape. ~59% have 4th canal (ML)

with its orifice located just lingual to the MB canal orifice. This 4th ML canal is in the MB root, and may join the MB canal or exit through a separate foramen. If a lesion exists on the MB root prior to RCT and does not heal in the usual time frame (6-12 months) after treatment, it is most likely due to a missed ML canal. • MB canal: the MB canal orifice is usually the MOST DIFFICULT to locate because it is under the MB cusp and must be accessed from a distolingual position. It is the small canal that often splits into two canals, and may be calcified and difficult to instrument. • Palatal canal: is the straightest, widest, and most tapering canal. The most common curvature of the palatal root is to the facial. The U-SHAPED radiopacity commonly seen overlying the palatal root apex is most likely the ZYGOMATIC PROCESS of the maxilla. • DB canal: also a small, tapering canal, but its orifice has no direct relation to the DB cusp. The DB orifice is usually located by its relation to the MB orifice. The DB canal is found 2-3mm distal and slightly palatal to the MB canal orifice. Important: nerve endings of CN VII, IX, & X are widely distributed in the SUBNUCLEUS CAUDALIS of

trigeminal nerve (CN V). A profuse, intermingling of these nerve fibers creates potential of referred dental pain to many sites.

RCT ADJUNCTS, INDICATIONS, & CONTRAINDICATIONS Adjuncts to Endodontic Treatment: ♦ Transplantation-the transfer of a tooth from one alveolar socket to another in the same person or into another person. Transplanted teeth with partial root development have a better prognosis than fully developed roots. Intentional replantation is NOT a substitute for endodontic surgery if it can be done. ♦ Orthodontic extrusion-force controlled vertical tooth movement occlusally in the socket. Orthodontic extrusion indications: PRIOR to implant placement, untreatable subgingival pathoses (i.e. cervical caries, cervical fracture, periodontal defects, resorptive lesions, and perforations in the cervical area). ♦ To stabilize an intentionally replanted tooth, a very effective method is to ask the patient to close in centric occlusion for the rest of the day. ♦ A major disadvantage of endodontic implants is the lack of an apical seal. ♦ Crown lengthening-a procedure to apically position the gingival margin and/or to reduce cervical bone.

Used to treat subgingival caries, perforations, and resorptions. ♦ Root submersion-involves resection of tooth roots 3mm below the alveolar crest, then cover with a mucoperiosteal flap. Submerged roots will prevent alveolar resorption and maintain better proprioception. • Indications: rampant caries, adverse periodontal conditions, repeated failure of prosthetic cases, and especially useful in medically compromised or handicapped patients requiring better denture control. Sometimes, root submersion is performed to avoid formation of an esthetic defect that may result after extraction. Thermal Sensitivity-the EARLIEST and MOST COMMON symptom of pulpal edema/inflamed pulp (acute pulpitis) is thermal sensitivity to hot and/or cold stimuli. Usually involves increased and persistent pain to cold. • Best method to elicit the most accurate thermal response is to INDIVIDUALLY ISOLATE the suspected teeth with a rubber dam, then bathe each tooth in hot or cold water. All other methods may stimulate the tooth in only one section of one surface, thus are less accurate. Thermal tests may yield a false-negative in immature, recently traumatized teeth, or due to premedication with an analgesic.

• As caries enter dentin, it spreads laterally at the DEJ due to the increased organic content and involvement of many dentinal tubules. Tomes fibers-react, cause fatty degeneration and later decalcification (sclerosis). • As caries progresses, dentin destruction is followed by bacterial invasion of the tubules and complete dentin destruction. Once odontoblasts are involved, pulpal changes occur (i.e. initially vascular dilation and local edema). • The only reliable clinical evidence that secondary dentin has formed is DECREASED TOOTH SENSITIVITY (seen a few weeks after placing a filling). When dentinal tubules become completely calcified, dentin is insensitive. Teeth conditions that usually DO NOT require endodontic treatment if managed properly because pulp remains vital: 1. Cementoma (periapical cemental dysplasia)-usually occurs in the anterior region of the mandible, starting as a radiolucent lesion that eventually calcifies. Cementoma DOES NOT affect pulp vitality. 2. Traumatic bone cyst-not a true cyst since there is no epithelial lining. Found mainly in young people and is asymptomatic. Appears as a radiolucency that scallops around teeth roots. Teeth are usually vital. No

treatment. 3. Globulomaxillary cyst-a developmental cyst found at the junction of the globules and maxillary processes of the maxilla, between the lateral incisor and canine roots. This cyst arises from cells in a fissural line of bone. Teeth are vital. CONTRAINDICATIONS for ROOT CANAL THERAPY (RCT): 1. Non-restorable or non-strategic tooth (i.e. tooth not in occlusion). 2. Poor-to-Hopeless periodontal support. 3. Vertical root fracture. 4. Extensive internal or external resorption. 5. Canal unsuitable for instrumentation or surgery (i.e. broken instruments, dentinal sclerosis, sharp dilacerations or extensive calcification). Any teeth not contraindicated are EXCELLENT candidates for successful endodontic therapy. Hemophilia-a medical condition that is NOT a contraindication to conventional endodontic therapy, but is strongly recommended that the dentist obtain clearance from the patient’s physician before treatment. Example of a Special Case: a previously traumatized tooth may show complete obliteration of the pulp chamber &

canal, but the PDL may appear normal. The patient will be asymptomatic and the tooth will not respond to pulp vitality testing. Treatment of choice: observe as long as the tooth remains asymptomatic & no periapical changes are evident. INDICATIONS for ROOT CANAL THERAPY (RCT): 1. Apical Scar-a periapical granuloma, cyst, or abscess that heal with scar tissue. It’s a well-circumscribed radiolucency resembling a granuloma. Tooth is nonvital, so endodontic treatment is necessary. 2. Radicular Cyst-usually occurs in a pre-existing granuloma, is rarely painful, and appears as an apex radiolucency of a non-vital tooth. Endodontic treatment is necessary. 3. Chronic Dental Abscess-often the cause of a sinus tract in the gingival tissue of children. It is often the result of a periapical granuloma, appearing as a radiolucent area at the apex of a non-vital tooth. A fistula is often found leading from an abscess cavity. Tooth pain stops upon drainage. Endodontic treatment is necessary. 4. Chronic Periapical Granuloma-asymptomatic, associated with a non-vital tooth, and the MOST common sequelae of pulpitis. Endodontic treatment is necessary.

ABSCESSES Periapical Abscess-a condition that results from a pulpal infection that extends through the apical foramen into the periapical tissues. MOST COMMON OF ALL DENTAL ABSCESSES. It is a localized collection of pus in the alveolar bone at the root apex after pulpal death with the infection extending into the periapical tissue. • FIRST SYMPTOM is slight tooth tenderness that later develops into a severe throbbing pain (acute abscess) with swelling of the overlying mucosa. • Tooth will NOT respond to EPT or cold tests, but may respond to heat. • Emergency treatment: establish drainage (ideally through the canal) and prescribe antibiotics & analgesics to relieve the acute symptoms, followed by conventional RCT at a later date. • For endodontic infections that do not respond to penicillin, clindamycin is recommended as it produces high bone levels, and is effective against anaerobic bacteria, but must be used with caution due to the potential for pseudomembranous colitis. • Acute Osteomyelitis-occurs in the jaws, most commonly caused by a DENTAL INFECTION. It is not a common disease, but is a serious sequela of a

periapical infection that often results in a diffuse spread of infection throughout the MEDULLARY SPACES, with subsequent necrosis of a variable amount of bone. o Acute or subacute osteomyelitis may involve the maxilla or mandible. The disease usually remains fairly well-localized to the area of initial infection in the maxilla. In the mandible, bone involvement is more diffuse and widespread. o Clinically, the afflicted person has severe pain, temperature/fever, and regional lymphadenopathy. The teeth in the involved area are loose and sore making eating difficult, if not impossible. o Radiographically: AO progresses rapidly and demonstrates little radiographic evidence of its presence until it has developed for at least 1-2 weeks. At that time, diffuse lytic changes in bone appear. A “moth-eaten” radiolucency is evident. o Treatment: establish & maintain drainage, and prescribe antibiotics to prevent further spread and complications. Periodontal Abscess-an acute abscess that develops through the periodontal pocket that involves alveolar bone

loss, pocket formation, and periodontal pathologic conditions. Tooth is usually palpation & percussion positive, and responds to EPT (unlike the periapical abscess). Bacteria associated with the periodontal abscess are gram (-) rods (i.e. Capnocytophaga species, Vibriocorroding organisms, Fusobacterium species). Gingival Abscess-a RARE abscess that occurs when bacteria invade through a break in the gingival surface. Caused by mastication, oral hygiene procedures, or dental treatment. Chronic Apical Abscess (suppurative apical periodontitis)-a long-standing, low-grade infection of the periapical bone with the root canal being the source of the infection. It is generally asymptomatic, and sometimes so painless that it may go undetected for years until revealed by an x-ray. Treatment is conventional root canal therapy. ♦ Chronic apical abscess may follow an acute alveolar abscess or unsatisfactory RCT. Radiographs reveal a diffuse radiolucency & PDL thickening. The tooth may be slightly loose, or tender to percussion. ♦ Chronic apical abscess is differentiated from cysts & granulomas because cysts & granulomas are welldefined radiolucencies. Phoenix Abscess (suppurative apical periodontitis = recrudescent abscess)-an apical lesion (acute abscess)

that develops as an acute exacerbation of a chronic apical abscess. It develops as the granulomatous zone becomes contaminated or infected by root canal elements. Diagnosis is based on acute symptoms (pain to percussion) and radiographic examination (reveals a large periapical radiolucency). • A massive invasion of pulpal contaminants will result in the formation of an acute abscess (Phoenix abscess). Important: 30%-50% of bone calcium must be altered before radiographic evidence of periapical breakdown occurs. This alteration occurs at the junction between the cortical and cancellous bone. Acute Apical/Alveolar Abscess (AA)-a localized collection of pus inside alveolar bone at the root apex after pulpal death, with the infection extending into the periapical tissue. FIRST SYMPTOM of AA may be slight tooth tenderness, that later develops into a SEVERE THROBBING PAIN to percussion with swelling of the overlying mucosa. ♦ The tooth becomes more painful, elongated, and loose. At times, the pain may decrease or completely disappear. The patient may appear weak, irritable, and have a fever. ♦ AA diagnosis is based on history, exam, and radiographs. An AA tooth will not respond to EPT or cold tests, but MAY respond to HEAT.

♦ Treatment: establish drainage & debride the canal system of necrotic tissue to relieve the acute symptoms. At a later date, perform conventional RCT. (Note: if the abscess ruptures through the periosteum into soft tissue, the patient’s symptoms will subside). GRANULOMA-a growth of granulomatous tissue continuous with the PDL due to pulpal death with diffusion of toxic products into the periapical area. Is usually asymptomatic. Radiographically, a well-defined area of rarefaction (radiolucency) with some irregularities is evident. Clinically, the tooth is NOT sensitive. CYST-inflammatory response of the periapex that develops from pre-existing granulomatous tissue (granuloma), characterized by a central, fluid-filled, epithelium-lined cavity, surrounded by a granulomatous tissue & peripheral fibrous encapsulation. Often associated with a chronically infected and potentially mobile tooth, but is usually asymptomatic. • Radiographically: evident as a well-defined area of rarefaction (radiolucency) that is limited by a continuous radiopaque, sclerotic border of bone. • Important: a granuloma or cyst is ONLY differentially diagnosed via HISTOLOGIC EXAMINATION.

Combined Periodontal-Endodontic Lesion-endodontic treatment takes precedence over periodontal management. Combined endodontic-periodontal therapy is widely used because the anatomic and clinical connections b/t the pulp and periodontal structures are close & numerous. In most combined endo-perio lesions, endodontic procedures are performed first, and when necessary, are followed by periodontal treatment. ♦ Ex: If bone loss extends from the cemento-enamel junction (CEJ) to the tooth apex on a radiograph, probing depth are above normal all around the tooth, but at one point the probe drops precipitously to an even greater depth, and vitality test is negative. This patient may require endodontic treatment followed by periodontic treatment. ♦ In these cases, the value of precise pocket probing and correct appraisal of pulp vitality is crucial. In some doubtful cases, it is best to wait until after RCT is complete to see if spontaneous resolution (pocket closure and osseous fill-in) will occur before surgical periodontal procedures are begun. ♦ Periodontal therapy is initiated first ONLY in the case of a primary periodontal lesion, with subsequent secondary endodontic involvement. ♦ A common clinical finding of a periodontal problem is PAIN to LATERAL PERCUSSION on a

tooth with a wide sulcular pocket. ♦ Periodontic-Endodontic Abscess-a combined lesion that usually shows radiographic involvement of the periodontium & apex of the involved tooth. PROBING LESIONS: 1. Conical-Shaped Probing Lesion-a periodontal probing defect that cannot be managed by endodontic treatment alone that is typical of a periodontal problem. Periodontal Lesions characteristically show bone loss that starts at the crestal bone level and progresses apically. Thus, probing defect is conical-shaped. Periodontal lesions may not be amenable to RCT alone even if it is associated with a pulpless tooth. However, endodontic treatment must be completed PRIOR to treating the periodontal problem. 2. Blow-Out (acute) Probing Lesion-a clue for diagnosis is a non-vital (necrotic) pulp that can completely heal after RCT. A tooth with this lesion shows normal sulcus depth all around the tooth until the area of swelling is probed. At this point, the probe drops to a level near the root apex. Probing depths in all other areas are within normal limits. 3. Narrow Sinus Tract Lesion-probing reveals normal depths all around the tooth except at one very narrow area. Thus, the probe can pass down the root surface to

some distance (sometimes to the apex). The tooth is pulpless (non-vital). Once RCT is complete, the lesion completely heals within one week. Another clue for diagnosis is a non-vital (necrotic) pulp. REVERSIBLE PULPITIS (Pulpal Hyperemia = Pulpal Inflammation)-most commonly caused by bacteria. Pain is not spontaneous, but requires an external irritant to evoke a painful response (i.e. cold, sweets). Pains are SHARP & BRIEF, stopping when the irritant is removed. Radiographs appear normal (may show deep caries or cavity preparation). • Tooth is usually percussion negative, and with thermal tests, the pulp responds more readily to cold than to hot (the response leaves shortly after the irritant is removed). • A short, painful response to cold suggest pulpal hyperemia. Pulpal hyperemia is an excessive accumulation of blood in the pulp due to vascular congestion. It is the engorgement of pulpal vessels with blood. Once the causative agent (i.e. bacteria or a restoration in hyperocclusion) is removed or adjusted, the pulp usually returns to normal. • Pulpal hyperemia is congestion of blood within the pulp chamber caused by physical, chemical, or bacterial insult. After restoration placement, teeth

often become hyperemic and sensitive to cold for a few days. The pain is not spontaneous, and does not last after the stimulus is removed. This short pain duration and low intensity is what distinguishes pulpal hyperemia from the pain of acute pulpitis (“irreversible pulpitis”). • Pulpal hyperemia caused by bacterial insult is a limited inflammation of the pulp. The tooth can recover if the caries is eliminated by timely operative treatment. • Treatment: usually a sedative filling or new restoration with a base. • Hyperemic teeth respond on a lower level of current on the EPT (electric pulp tester) than a normal tooth. • The most effective way to reduce pulp injury during tooth preparation is to minimize dehydration of dentin. IRREVERSIBLE PULPITIS (“Acute Pulpitis”)-condition characterized by SPONTANEOUS PAIN with periods of cessation (intermittent in nature). Clinical symptoms severity varies as the inflammatory response increases. Pain varies from a mild & readily tolerated discomfort, to a severe, throbbing, and excruciating pain that is

spontaneous, intermittent, and LINGERS AFTER THE IRRITANT IS REMOVED. • Pain is usually not readily localized by the patient, but is diffuse in nature. • Irreversible pulpitis is an acute inflammation of the pulp characterized by intermittent spasms of pain that becomes continuous. In the early stages, it may appear as a very severe hyperemia. As the condition continues, the pain is gnawing or dull-throbbing, and is generally increased by heat and relieved by cold. • Lying down or bending over intensifies the pain because the overall increase in cephalic blood pressure is relayed to the confined pulp tissue. • Tooth may be tender to percussion (tooth is usually percussion positive), heat may intensify the pain response, while cold may relieve it (in advanced stages). Usually HOT & COLD cause severe and lasting pain. Thus, thermal test is the best aid to diagnose an irreversible pulpitis. • When the pulp becomes severely inflamed as indicated by a thermal stimulus producing pain that lasts long after the stimulus is removed (longer than 10 seconds), this suggests “irreversible pulpitis” and the pulp is unlikely to recover after removing the caries. Pulpal pain (spontaneous or elicited by an

irritant) that lingers more than 10-15 seconds. • Radiographs do NOT disclose periapical pathology. • Sometimes it is hard to distinguish between reversible and irreversible pulpitis, in which case caries control (placing a temporary filling) is a conservative approach toward making the final diagnosis. If a tooth responds well to the temporary filling, the there is no need for RCT at this time. • Treatment: RCT (complete pulp debridement). PULPAL NECROSIS (Pulp Death)-may have no painful symptoms and does not respond to EPT at any current level, but the tooth sometimes responds to heat, but NO COLD RESPONSE. A tooth affected with a necrotic pulp may have no painful symptoms and may appear discolored. EPT is valuable because there will be no response at any current level. Treatment: RCT or extraction. Significant Bacteroides species involved in pulpalperiradicular infection are Porphyromonas & Prevotella which received a separate genus (undergone taxonomic revision) due to their distinct characteristics. Predominant bacterial species isolated from INFECTED ROOT CANALS include: • Eubacterium, Peptostreptoccus, Fusobacterium, Porphyromonas, and Prevotella species.

• Streptococcus species may not be as important in the progression of a carious lesion (leading to pulp exposure) as much as it is in the initiation of the lesion. Strict anaerobes play a major role in periapical pathoses. Virulence Factors involved in Periradicular Pathosis: • Lippolysaccharide (LPS)-found on the surface of gram (-) bacteria. • Enzymes-neutralize antibodies and complement components. • Extracellular vesicles-involved in bacterial adhesion, proteolytic activities, hemagglutination, and hemolysis. • Fatty acids-affect chemotaxis and phagocytosis. Vital pulp resists bacterial invasion. Even if vital pulp is exposed to microorganisms for 2 weeks, the bacterial penetration may extend no > 2mm into the pulp. In contrast, non-vital pulp is a fertile ground for bacterial growth and leads to necrosis. Restoring Endodontically Treated Teeth: • A major disadvantage of posts (dowels) is it WEAKENS ROOT/TOOH STRUCTURE. • All posts designs are predisposed to leakage. • At least 4mm of gutta-percha MUST remain to preserve the apical seal. • Threaded screw posts may increase root fractures.

Thus, parallel-sided posts and tapered posts are preferred. • Pins increase stresses and microfractures in dentin, thus should not be used. • Cusps adjacent to lost marginal ridges should be restored with an ONLAY. Restorative Options for Endodontically Treated Posterior Teeth: Posterior teeth with RCT are more prone to fracture than non-treated posterior teeth due MAINLY to the destruction of coronal tooth structure and reduced structural integrity. • Anterior teeth that have had RCT do not necessarily need a full coverage crown. If the access opening is conservative and there is not extensive caries compromising the clinical crown, a lingual composite or amalgam can simply be placed to conserve tooth structure. • ONLAYS: the minimum and most conservative preparation and restoration to cover cusps and marginal ridges, especially posterior teeth with prior root canal therapy (RCT). Goal is to prevent cracked tooth syndrome. • CROWNS (ZIRCONIUM, PORCELAIN, PFM): a fullcoverage crown is preferred when remaining coronal tooth structure does not provide enough sufficient tooth structure for an onlay. Goal is to prevent

cracked tooth syndrome. • POST-CORE: indicated when there is insufficient clinical crown remaining to retain a core build-up and allow for adequate crown preparation. Requires the tooth to have RCT then post-placement to provide suitable coronal structure for an optimum crown preparation. • If performing a pulp-chamber-retained amalgam, must place amalgam 3.0mm into each canal for retention.

ANTI-ANXIETY DRUGS INHALATION NITROUS OXIDE-the most frequently utilized route of administration for sedation in pediatric patients. ► Nitrous oxide is a slightly sweet smelling, colorless, inert gas that must always be coupled with at LEAST 20% oxygen. N2O is quickly absorbed from the lungs and is physically dissolved in the blood. When one turns on the N2O, the initial concentration is 20%. Initially, some start with 100% oxygen. Maximum concentration of N2O should not exceed 50% (some say 40%). When one starts conscious sedation, the flow rate is about 6 liters/minute. The correct total liter flow of nitrous oxide-oxygen is determined by the amount necessary to keep the reservoir bag 1/3-2/3 full. The earliest symptoms of conscious sedation is LIGHT HEADEDNESS. Laughing or crying, paresthesia of arms, legs, and oral cavity, and feeling of floating are seen in the second stage.

After sedating a child, always keep an eye on the status of the child. ► There is no biotransformation, and the gas is rapidly excreted by the lungs when the concentration gradient is reversed. It is recommended that the patient be maintained on 100% oxygen for 3-5 minutes after the sedation period. ► N2O creates an altered state of awareness with impaired motor function. It is a CNS depressant, but produces little analgesia. The combined volume of gases being delivered (oxygen and nitrous) should be at least 3-5 liters/min. The operator should encourage the patient to breathe through their nose with the mouth closed. The feeling of floating or giddiness with tingling of the digits is the proper response to nitrous oxide. ► For restorative dentistry, N2O and local anesthesia is usually all that is needed to treat a child who fears the dentist. SEDATIVE HYPNOTICS-their principal effect is sedation and sleepiness. 1. Chloral Hydrate-a drug widely used for pediatric sedation by acting on the CNS to induce sleep. At normal doses, the sleep induction does not affect breathing, BP, or reflexes. It may be used before some

surgeries or procedures to help relieve anxiety and induce sleep. When used with analgesics, it can help manage pain after surgery. Its onset of action is 15-30 min when given orally. Children often enter a period of excitement and irritability before becoming sedated. As with barbiturates, pain may cause paradoxical reactions. 2. Short-Acting Barbiturates: Secobarbital (Seconal) & Pentobarbital (Nembutal) are sedative drugs sometimes used for pediatric conscious sedation by oral administration, but are of very limited value. They are non-analgesic and may cause hyper-excitability rather than sedation in some children. MIDAZOLAM (Versed Injection, Syrup, or Nasal Spray)an anti-anxiety (benzodiazepine) drug that relaxes and makes child sleepy. Closely related to diazepam (Valium). • Used in adults and children to reduce anxiety and fear. In adults, it is given through a vein in the arm. During the last 20 years, it has almost totally replaced diazepam as the drug of choice for IV sedation. It causes less burning and pain during injection. In children, it is usually given orally as a sweet-tasting syrup. Some dentists may elect to give midazolam as a nasal spray. • Does of IV midazolam usually is in the range of 15mg just before the dental procedure. Children’s

dosing is based on body weight, ranging from 0.25mg-0.5mg per kilogram (2.2 pounds). Maximum dose is 20 milligrams. The syrup is given to a child 30 to 45 minutes before the dental procedure. • Review child’s existing allergies and medications (to include OTC vitamins and herbal supplements). • Must monitor patient’s blood pressure, heart, pulse and breathing. Pulse oximeter to measure heart rate and indirectly, breathing. Midazolam can sometimes affect breathing. Midazolam potential side effects: nausea, vomiting, xerostomia, confusion, decreased coordination. • Certain drugs may cause midazolam systemic build up, thus can increase risks of Midazolam side effects. These drugs include: • Erythromycin (Eryc) and Clarithromycin (Biaxin). • Azole antifungal drugs: Ketoconazole (Nizoral) and Fluconazole (Diflucan). • Ulcer medicines: Cimetidine (Tagamet) and Omeprazole (Prilosec). Midazolam Contraindications: • Pregnant women • Alcoholics • Narrow-angle glaucoma (increased eye pressure) • Sleep apnea (can cause respiratory depression)

PULP TREATMENT PULPECTOMY (RCT)-the suggested treatment when an 11-year old child traumatized a permanent maxillary central incisor and the tooth becomes painful, with swelling, and a periapical radiograph discloses a pathosis associated with the apex. ► This is treated the same way that you would treat the adult patient. At age 11, the root of a maxillary central incisor should be completely formed, thus an apexification procedure is NOT indicated. If the root were not fully formed, then an apexification process should be started. This involves the placement of calcium hydroxide paste into the canal to stimulate continued apical closure. ► Pulpectomy is the treatment of choice when the primary mandibular 2nd molar in a 4-year old has a large carious lesion with pulpal involvement. Radiographically there is periapical pathology on the distal root. There is also furcation involvement and slight mobility. Pulpectomy is the treatment of choice when there is periapical pathology. If there were no periapical pathology, a formocresol pulpotomy is indicated. If the child were older and there was a periapical radiolucency but successful

pulpectomy could not be accomplished, the treatment of choice is extraction and placement of a space maintainer to prevent damage to surrounding bone and the developing permanent tooth. ► When performing a pulpectomy on primary teeth, the canals are filled with ZOE paste because it causes only a minimal tissue reaction and it resorbs when the primary tooth begins its normal resorption process. ► Chronic pulpal infection in primary molars is usually first noted radiographically as a change in the bony furcation. A cooperative 5-year old child has a carious lesion in her primary mandibular second molar. There is no tooth mobility, but the dentist notes a small, draining sinus tract adjacent to the tooth. Treatment of choice is a pulpectomy. PULPOTOMY-preserves radicular VITAL pulp tissue when the entire coronal pulp is amputated, and allows resorption & exfoliation of the primary tooth, but preserves its role as a natural space maintainer. The reason to perform a pulpotomy using either of the following two techniques is that the coronal pulp shows evidence of inflammation and degenerative change due to microorganisms located there.

1. Calcium Hydroxide Pulpotomy Technique-treats permanent teeth when there is a pathological change in the pulp at the carious exposure site. CHT is especially indicated for permanent teeth with immature root development and with healthy pulp tissue in the root canals. The success of a pulpotomy in this case would be indicated when the root apex (if incompletely formed) completes its full development in time (apexogenesis). This technique is completed in one appointment. Only those teeth free of symptoms of painful pulpitis (severe toothache, lingering pain) are considered. A dentin bridge will form at a level below the level of amputation of the coronal pulp. Histologically, you would see a necrotic layer immediately under the calcium hydroxide. 2. Formocresol Pulpotomy Technique-treats primary teeth with a carious exposure. Teeth selection is the same as for the CaOH technique, and is performed in one appointment. After the coronal pulp is removed (amputated), a cotton pellet moistened with formocresol is placed in contact with the pulp stumps and remains for 5 minutes. The pellet is then removed and the pulp chamber dried. A thick paste of ZOE is placed over the chamber and the tooth is restored. The success of formocresol pulpotomy for a primary tooth depends primarily on a vital root tip. Formocresol (formalin and cresol) causes surface fixation of the pulpal tissue and odontoblast degeneration.

► Indications: tooth sensitive to sweets, pulp exposure during caries excavation, and radiographic evidence of deep caries approximating the coronal pulp. ► Contraindications: radiographic evidence of internal resorption is a contraindication for a formocresol pulpotomy on a primary molar. Also, a tooth that is painful with swelling is a contraindication to a pulpotomy. You need healthy ROOT pulp tissue for pulpotomy success. ► The success of a pulpotomy for a primary molar depends primarily upon a VITAL ROOT PULP. DIRECT PULP CAPPING-primarily used on permanent teeth (not widely used on primary teeth because of CaOH’s alkaline pH). To perform a direct pulp cap on a primary tooth, the tooth MUST be asymptomatic and must be a small exposure with little or no hemorrhaging. CaOH can effect (irritate) the pulp mildly or most often severely: • With a mild irritation, there is a mild inflammatory reaction that resolves itself and regroups as reparative dentin. • With severe irritation, internal resorption can occur. In primary teeth, severe irritation often results in internal resorption. However, in permanent teeth, this rarely occurs because the severe inflammatory response causes reparative dentin formation.

• Contraindications of performing a direct pulp cap on primary teeth: ► Spontaneous pain from the tooth (symptomatic). ► A large exposure (only used for pinpoint/small exposures). ► Excessive hemorrhaging (bleeding). ► Radiographic evidence of internal resorption. INDIRECT PULP CAPPING: primary goals are to preserve pulp vitality, prevent pulp exposure, save tooth structure, arrest caries, and promote reparative dentin formation. • Indications: mostly for permanent teeth with rampant caries and large carious lesions close to the pulp that are not chronically painful. • Contraindications: do not use in cases where there is spontaneous pain, furcation involvement, or pulpal involvement. Most pediatric dentists feel indirect pulp capping is contraindicated in the primary dentition. • Adolescent patients with rampant caries may require caries control prior to final restorations to arrest all deep carious lesions and halt their advancement toward the pulp. This involves removing gross caries then placing calcium hydroxide and an interim restoration like IRM (reinforced ZOE). • The greatest problem in pulpal diagnosis is

predicting the amount or extent of inflammation. Successful management of deep caries and pulpally involved teeth in children depends on the extent of pulpal inflammation. Emergency Treatment of Fractures of Permanent Teeth with Immature Apices: 1. Class I: smooth enamel edges, restore the tooth. 2. Class II: apply calcium hydroxide to exposed dentin and restore the tooth with a permanent restoration. 3. Class III: immediately after injury, apply calcium hydroxide over exposure and place a temporary restoration. If the exposure is large or the injury was several hours or days ago, perform a calcium hydroxide pulpotomy. Once the apex closes, do a pulpectomy. 4. Class IV: do a calcium hydroxide pulpotomy. Once the apex closes, do a pulpectomy. In an older child with a fully formed apex, if there is a pinpoint exposure and it has been a while (day) since the fracture, conventional RCT with gutta percha is the treatment of choice. If the child is seen immediately, then a direct pulp cap with calcium hydroxide is indicated, followed by a permanent restoration.

PEDIATRIC DISEASES & CONDITIONS Cleft Palate & Cleft Lip are the MOST COMMON craniofacial malformations, accounting for 50% of all defects. They can be unilateral or bilateral. 1. CLEFT LIP-a separation of the two sides of the lip that occurs during the 5th-6th week of embryonic life due to failure of the maxillary & frontonasal processes to merge. Cleft lips are more common in MALES, and more common on the left side than right side. • 4 Classes of Cleft Lip: 1. Class I: a unilateral notching of the vermillion NOT extending into the lip. 2. Class II: unilateral notching of the vermillion, but the cleft extends into the lip, but NOT to the nose floor. 3. Class III: a unilateral notching of the vermillion, but the cleft extends into the lip and floor of the nose. 4. Class IV: any bilateral clefting of the lip whether incomplete notching or complete clefting. • Cleft lip deformity occurs during 4-6 weeks of pregnancy. Cleft lip occurs following the failure of the permanent union between the maxillary process & frontonasal process.

2. CLEFT PALATE-an opening in the roof of the mouth where two sides of the palate did not unite. It occurs in 6th-8th week of embryonic life. Isolated clefts of the palate are more common in females. It is characterized by a fissure in the midline of the palate due to the failure of the two sides to fuse during embryonic development. • The most severe handicap imposed by cleft palate is an impaired mechanism preventing normal speech and swallowing. The child almost always needs orthodontic treatment once the palate is surgically repaired. Speech therapy is also required since these patients have problems related to the inability of the soft palate to close airflow into the nasopharynx. Orthognathic surgery may be needed to correct the concave appearance of the face due to deficient maxillary growth. • 4 Classes of Cleft Palate: 1. Class I: involves only the soft palate. 2. Class II: involves the soft and hard palates, but not the alveolar process. 3. Class III: involves the soft and hard palates, and the alveolar process on one side of the premaxilla. 4. Class IV: involves the soft palate and continues through the alveolus on both sides of the premaxilla.

• Cleft palate deformity occurs/develops during the 1st trimester of pregnancy (6-9wks). Speech problems associated with cleft lip & palate are usually the result of the inability of the soft palate to close air flow into the nasal area. Acute Necrotizing Ulcerative Gingivitis (ANUG, Vincent’s Angina or “Trench Mouth”)-an acute gingival disease characterized by painful hyperemic gingiva, punched-out erosions of interproximal papilla, covered by a GRAY pseudomembrane with an accompanying fetid/foul odor. • ANUG is an acute fusospirochetal infection of the gingiva caused by fusiform/fusobacterium, spirochetes, & Prevotella intermedia. ANUG is a progressively painful infection with ulceration, swelling, and sloughing off of dead tissue from the mouth and throat due to the spread of infection from the gums. • ANUG is usually associated with poor oral hygiene, and is most common in conditions where there is crowding and malnutrition. Emotional stress and smoking are also risk factors. • Debridement helps clear up the infection and antibiotics reduce acute symptoms.

• Rarely occurs in pre-school children and has a slow onset. • ANUG usually affects YOUNG ADULTS ages 15-35 years old. It is rare in preschool children, and is easily diagnosed due to involvement of interproximal papillae and presence of a pseudomembranous necrotic covering of the marginal tissues. • Clinical Manifestations: inflamed, painful, bleeding gingival tissues, poor appetite, fever, general malaise, and fetid odor. ANUG is necrotic, foul-smelling ulcers of the gums and throat. • Treatment: debridement, H2O2 mouth rinses, and antibiotic therapy. PRIMARY (ACUTE) HERPETIC GINGIVOSTOMATITIS: most commonly occurs in CHILDREN & YOUNG ADULTS. Patients develop FEVER, irritability, regional lymphadenopathy, and headache. Within days, the gingiva is intensely inflamed. Any part of the oral mucosa and lips may become involved. Vesicles then form and rupture shortly later to leave shallow VERY PAINFUL ulcers covered with a GRAY MEMBRANE and surrounded by a RED HALO. Ulcers heal on their own in 7-14 days. • Treatment: fluid intake, good oral hygiene, and gentle debridement of the mouth. In healthy individuals, the lesions heal in 7-14 days, and the

ulcers heal WITHOUT SCARRING. • After recovery from primary HSV, the virus is not cleared from the body, but lies dormant in a nonreplicating state in the sensory nervous system (trigeminal ganglion). Periodically, latency reactivates and allows the virus to return to the skin or mucous membranes, where it causes a recurrent infection. • Primary herpes occurs only in YOUNG PATIENTS (baby, children, adolescents). Fever, malaise, pain, or could be CHICKENPOX (varicella zoster virus) in a 6yr old child. Multiple vesicles, sick for one week. • The primary herpes infection (HSV-1) that mainly affects YOUNG CHILDREN (1-5 years old), but may also affect young adults (15-25yrs). Usually occurs in a child who has not had any contact with HSV-1, and who thus has no neutralizing antibodies. Nearly all primary infections are of the sub-clinical type that may only have flu-like symptoms, with 1-2 mild sores in the mouth that go unnoticed by parents. In other children, the primary infection may be manifested by acute symptoms (acute herpetic gingivostomatitis) these prodromal symptoms are (fever, malaise, irritability, headache, dysphagia, vomiting, cervical lympadenopathy) 1-2 days prior to local lesions. Then, fiery red gingival tissues and

small yellowish vesicles form that rupture quickly, causing painful shallow, round, discrete ulcers with an erythematous (red) halo on the FREE & ATTACHED MUCOSA. Thus, the primary HSV-1 infection ranges from sub-clinical (asymptomatic) to severe systemic infections. Dehydration is the most serious potential problem due to the child not wanting to eat or drink because of the pain. A generalized marginal gingivitis may precede the ulcers. Primary herpetic gingivostomatitis is MOST likely to occur in children ages 1-5 years. The virus that causes acute herpetic gingivostomatitis is closely related to the herpes virus that causes chickenpox (varicella zoster). Clinically AHG can result in spherical discrete vesicles. HERPANGINA–an acute infection (stomatitis/mouth inflammation) affecting YOUNG CHILDREN caused by the A strain COXSACKIE VIRUS. It is differentiated clinically from HSV-1 (cold sore virus) as herpangina oral ulcerations/vesicles typically occur on the BACK OF THE THROAT (SOFT PALATE, TONSILS, POSTERIOR PALATE). May also appear on the tongue. • Clinical Features: are mild and short duration

compared to HSV-1. Herpangina begins with a sore throat, fever, headache, and sometimes vomiting & abdominal pain. Herpangina usually runs its course and resolves without treatment in less than 1 week. Treatment: PALLIATIVE. ATROPHIC GINGIVITIS-characterized by gingival recession without a corresponding rate of alveolar bone loss. Minor marginal and papillary gingival inflammation is found. RECESSION is the main clinical finding. Acute Lymphocytic/Lymphoblastic Leukemia (ALL)-the most common PEDIATRIC CANCER (peaks around age 4) that is a life-threatening disease where cells that normally develop into lymphocytes (lymphoblasts) become cancerous and rapidly replace normal cells in bone marrow. ALL is the form of acute leukemia most responsive to therapy. • Early signs in a child are fatigue, pallor, weight loss, and easy bruising. This progresses to fever, hemorrhages, extreme weakness, bone/joint pain, and repeated infections. • The most common type of leukemia in children is lymphoblastic leukemia. • Oral Features: gingival oozing, petechiae, hematoma, ecchymosis, oral ulceration, pharyngitis, gingival infection that is unresponsive to conventional

therapy, and submandibular lymphadenopathy. • Children with leukemia are very susceptible to Candida fungal infections, thus, NYSTATIN rinses or popsicles are effective treatments. • Leukemia-a progressive, malignant disease of the blood-forming organs. APERT SYNDROME-a genetic defect that falls under the broad classification of cranial-limb anomalies, primary characterized by specific malformations of the skull, midface, hands, and feet. Blindness is NOT a clinical feature. ► Dental Considerations: often associated with supernumerary teeth (disrupt eruption of other teeth), severe crowding, and Class III malocclusion. ► Major Features: prematurely fused cranial sutures, retruded midface (often corrected with Lefort III surgery), fused fingers and toes. AUTISM SPECTRUM DISORDER (ASD)-a complex developmental disability that typically appears in the first 3 years of life due to a CNS disorder affecting brain function (4x more prevalent in boys than girls (1 in 42 boys and 1 in 189 girls in U.S.). Thought to be caused by a combination of autism risk genes and environmental factors

affecting early brain development. • Dental Management (ASD): goal is to get the child comfortable with the dental experience. Have parent simulate the dental visit with the child before the appointment if possible, and have child bring a favorite toy. Establish trust using “tell-show-do”, consistency, and positive reinforcement techniques. Have parents present in operatory. Short and frequent appointments in a relaxed setting (dim lights or sunglasses, keep sound and touch stimuli to a minimum). May require N2O or sedative drug. Higher risk for caries and gingivitis. Provide positive reinforcement during dental appointment and OHI. • Difficult to treat an autistic patient due to impaired verbal communication (inability to communicate with the child). New technology is improving communication through hand-held electronic devices. ATTENTION DEFICIT HYPERACTIVITY DISORDER (ADHD)-a developmental and behavioral disorder characterized by an attention span that is shorter than expected for the child’s age, hyperactivity, talkative, inattentive and impulsive behavior. These children are easily distracted, have difficulty paying attention, and may be unable to focus more than a few moments on mental tasks. They may be physically active and behave impulsive.

Cause is unknown, and is 10x more common in males than females. • Clinical Signs: poor school performance due to inability to attend to tasks or sit still during school. Short-term memory deficient, impulsive, hyperactivity, inattentiveness, and highly talkative inhibit learning. • Dental Management (ADHD): ADHD children typically do not require special treatment. However, SHORT-MID MORNING appointment after breakfast and taking RITALIN works best. Maintain eye contact, use tell-show-do, and let child assist and be involved during dental treatment (holding the suction, just as you would with most children). • Methylphenidate (Ritalin)-a mild CNS stimulant often used in ADHD children (children over age 6). Among the more serious adverse reactions are nervousness, insomnia, and anorexia. • Amphetamines (i.e. Dextroamphetamine) also treat ADHD. ACHONDROPLASIA-the most common form of short-limb dwarfism that occurs in all races, equally affected males and females. Many of these children die before age 1. Deficient growth in the cranial base is evident in many

children who survive. ► Clinical Features: disproportionate short stature (head is large, and arms/legs are short compared to the trunk length). Prominent forehead, depressed bridge of the nose, small maxilla causing overcrowding of teeth, and Class III malocclusion. GIGANTISM-oral manifestations are an enlarged tongue causing the teeth to be tipped either to the buccal or lingual. Mandibular prognathism and roots may be longer than normal are other oral features. PITUITARY DWARF-the eruption rate and shedding of the teeth are delayed. Clinical crowns and roots appear smaller, the dental arch is smaller causing malocclusion, and the mandible is underdeveloped. CELLULTIS-an acute spreading infection of the dermis and subcutaneous tissues causing pain/tenderness, erythema, edema, and warmth of the affected area (hallmarks of cellulitis). Cellulitis is a common infection that may progress to a serious illness by uncontrolled spread contiguously or via the lymphatic or circulatory systems. Group A Streptococci & Staphylococcus Aureus are the most common causative organisms. • Cellulitis may be caused by a necrotic primary or permanent tooth, resulting in considerable swelling of the face or neck, and the tissue appears discolored. It

is a very serious infection that can be life-threatening. The child appears acutely ill and may have a very high temperature with malaise and lethargy. • Cellulitis in a child is harder to treat because dehydration occurs more frequently, rapidly, and severely in children. • Treatment includes bringing the child to the hospital if the signs and symptoms warrant it. LUDWIG’S ANGINA-cellulitis that affects the submandibular, sublingual, & submental spaces, and causes elevation of the tongue and mouth floor resulting in obstruction of the patient’s airway and makes swallowing impossible. The patient MUST be taken to the hospital immediately. CRETINISM (Child Hypothyroidism)-a deficiency disease caused by the congenital absence of THYROXINE (hormone secreted by the thyroid gland). Cretinism is severe hypothyroidism in a child, characterized by defective mental and physical development. • Cretins have dwarfed bodies with curvature of the spine and a pendulous abdomen. • Limbs are distorted, their features are coarse, and their hair harsh and scanty.

• Severe mental retardation is caused by the improper development of the CNS. If cretinism is recognized early, it can be markedly improved with the use of thyroid hormones. • Dental Findings: underdeveloped mandible, overdeveloped maxilla, enlarged tongue causing malocclusion like anterior open bite and flared anterior teeth, delayed tooth eruption, and longer retention of the deciduous teeth. CYSTIC FIBROSIS-an inherited disease of exocrine glands, affecting ~30,000 children and adults in the U.S. Cystic fibrosis causes the body to produce an abnormally thick, sticky mucus due to a faulty transport of Na+ and Cl within cells lining organs like the lungs and pancreas. The glands most affected are in the pancreas, respiratory system, and sweat glands. • Cystic fibrosis has various symptoms. The most common are very salty tasting skin, persistent coughing, wheezing, or pneumonia, excessive appetite, but poor weight gain, and bulky stools. • An individual must inherit a defective copy of the CF gene (one from each parent) to get the disease. In CF cells, salt does not move properly because the protein product of the CF gene is defective and makes a faulty channel for the chloride to exit.

• Cystic fibrosis is usually recognized in infancy or early childhood, occurring mainly in Caucasians. Early signs are a chronic cough, frequent foul-smelling stools (steatorrhea), and persistent upper respiratory infections. The most reliable diagnostic tool is the SWEAT TEST (shows elevated levels of Na+ and Cl). • A high percentage of children with cystic fibrosis have dark-colored teeth. Cystic fibrosis is a combination of steatorrhea, chronic respiratory infections, and functional disturbances in secretory mechanisms of various glands. Cleiodocranial Dysplasia (Dysostosis)-an inherited disorder of bony development characterized by absent or incompletely formed clavicles, characteristic facial appearance, and dental abnormalities. Occurs equally in M & F, with the presence of supernumerary teeth being one of the most distinguishing features. • Oral findings: delayed permanent teeth eruption, peg-shaped teeth, and congenitally missing teeth. ECTODERMAL DYSPLASIA-a hereditary/congenital condition characterized by abnormal development of the skin & associated structures (hair, nails, teeth, sweat glands). Several types of ectodermal dysplasia exist, but the x-linked anhidrotic ectodermal dysplasia is the most

common. • Teeth develop abnormally with possible complete failure of the teeth to develop (anodontia) or oligodontia (partial anodontia). Conical-shaped anterior teeth characteristic of oligodontia and associated with ectodermal dysplasia. Atrophic skin, defective hair, partial anodontia, & hypoplastic sweat glands. • Anhidrotic Ectodermal Dysplasia-a condition that only affects males, characterized by lack of perspiration caused by partial or complete absence of sweat glands. Retained primary teeth are an outstanding oral manifestation of Ectodermal & Cleidocranial dysplasias. This prolonged retention of the primary teeth may cause a change in the path of eruption of the succeeding teeth. DIABETES MELLITUS-a disease where the body cannot properly use and store glucose. Glucose backs up in the bloodstream causing blood glucose levels to rise too high. The classic triad of symptoms is always present (polydipsia/extreme thirst, polyphagia/extreme hunger, & polyuria/frequent urination). Weight loss, poor wound healing, or extreme unexplained fatigue are other symptoms.

1. Type I Diabetes (Insulin-Dependent Diabetes or Juvenile-Onset): diabetes affecting young adults where the body completely stops producing insulin. Type I diabetics are totally insulin deficient, thus must take daily insulin injections to survive. Juvenile diabetics tend to be unstable, brittle, and prone to ketoacidosis. Blindness is a serious complication that may develop. • Oral Complications: xerostomia, infection, poor healing, increased incidence and severity of periodontal disease, and burning mouth syndrome. DIPHTHERIA-an acute, contagious disease caused by the bacterium Corynebacterium diphtheria, characterized by the production of a systemic toxin which is very damaging to tissues of the heart and CNS. Immunization is available to all U.S. children. DOWN SYNDROME (TRISOMY 21)–a genetic disorder caused by a chromosomal abnormality (TRISOMY 21) that occurs during the PREIMPLANTATION PERIOD of prenatal development. Marked by various degrees mental impairment, physical growth delays (short, flattened skull, slanting eyes (ophthalmic disorders), thickened tongue/fissured, broad hands/feet), potential heart defects, weakened immune system (increased infection susceptibility), risk of leukemia, early Alzheimer’s disease and hearing loss. Trisomy 21 is the most common chromosomal abnormality in humans, and its incidence

increases with maternal age (greater risk after 35 years age). • Oral Manifestations: mandibular prognathism, delayed teeth eruption, higher incidence of congenitally missing teeth (hypodontia), taurodontism, malocclusion, enamel dysplasia, increased gingivitis and periodontal disease, thickened or fissured tongue, MACROGLOSSIA, mouth breathing (open mouth and dry, cracked and protruding lips). DECREASED DENTAL CARIES. • Dental Management (Trisomy 21): practice “tellshow-do” and involve caregiver (some patients cannot verbally communicate depending on severity). May require antibiotic pre-medication if a congenital heart defect exist (affects 45% of newborns), due to compromised immunity and infection susceptibility. Avoid air polisher and ultrasonic scalers. Use positive reinforcement. Physicians consult regarding need for pre-medication with antibiotic due to susceptibility to infections due to congential heart defects. HEMANGIOMA-the most common benign tumor of INFANTS. Hemangiomas are vascular birthmarks where the proliferation of blood vessels leads to a mass that resembles a neoplasm. Hemangiomas differ from other vascular birthmarks because they are biologically active

(their growth is independent from the child’s growth). Most hemangiomas appear within a week or two after birth, and are 5x more common in girls. • Hemangiomas are common on the lips, tongue, and buccal mucosa, appearing as flat or raised lesions, usually deep red or bluish-red and rarely wellcircumscribed. Hemangiomas are removed surgically, while other do not require treatment. Asymptomatic, soft, bluish lesion on the tongue, present for 5 years and has had a minimal increase in size. LYMPHANGIOMA-a fairly well-circumscribed nodule or mass of lymphatic vessels that occurs most often in the neck and axilla. Compressible and spongy red-to-blue translucent enlargements/lesions. • Treatment: Excisional biopsy. NEUROFIBROMA-a moderately firm, encapsulated tumor caused by the proliferation of Schwann cells. Found on the tongue, buccal mucosa, vestibule, & palate. Neurofibromas appear as solitary or multiple sub-mucosal enlargements, and may become malignant (5-15%). Multiple lesions are associated with Neurofibromatosis (Von Recklinghausen’s Disease). NURSING BOTTLE CARIES (Baby Bottle Tooth Decay or Bottle Mouth Syndrome)-a widespread carious

destruction of the deciduous (primary) teeth most commonly affecting the MAXILLARY INCISORS. Inappropriate feeding of children may cause a typical nursing pattern decay. There is early carious involvement of the maxillary anterior teeth, followed by maxillary & mandibular 1st molars, and mandibular canines. Mandibular incisors are less affected since the tongue covers them. ► AKA: Early childhood caries, Nursing caries, Bottle caries, or Infant caries. ► Nursing bottle caries is RAMPANT DECAY due to sleep time bottle-feeding combined with Streptococcus mutans activity. The stagnation of milk around the necks of anterior teeth and fermentation of the disaccharide lactose (a sugar found in milk), also contributes to this caries process. ► Baby Bottle Caries Preventive Measures: ♦ Infants should not be put to sleep with a bottle containing a liquid other than water. ♦ Infants should be encouraged to drink from a cup prior to their first birthday, and should be weaned from the bottle at 12-14 months of age. ♦ Infants should start to supplement their diet with non-liquids at age 4-6 months. ♦ Juices should only be offered in a cup. ♦ Oral hygiene should be started with eruption of the first primary tooth. ♦ Within 6 months of eruption of the first tooth

(no later than the first birthday) it is time for the first dental visit. PIERRE ROBIN SYNDROME-a hereditary disorder that presents micrognathia (smallness of the jaws), glossoptosis (downward displacement or retraction of the tongue), & a high-arched or cleft palate. Most children require orthodontics. PORPHYRIA-a group of inherited disorders involving abnormalities in the production of heme pigments, myoglobin, & cytochromas. It can result in discoloration of teeth. Porphyrias are characterized by 3 major findings: 1. Photodermatitis (light sensitivity causing rashes). 2. Neuropsychiatric complaints. 3. Visceral complaints (abdominal pain, cramping). Diagnosis of Porphyria: a child present with red urine, purplish-brown teeth, sensitivity to sunlight, and develops blisters and swelling on the face and hands when exposed to sunlight. Causes of Tooth Discoloration: 1. Porphyria-a condition that results in discoloration of teeth. 2. Cystic Fibrosis: these children have teeth dark in color (ranging from yellowish-gray to dark brown), and may

be related to the usual high doses of tetracycline given to these children. 3. Erythroblastosis Fetalis: characterized by an excessive destruction of erythrocytes. Primary teeth may have a blue-green color. 4. Tetracycline Therapy: can cause tooth crowns to become discolored from yellow-to-brown, and gray-toblack. The drug stains permanent teeth that have not completed enamel formation at the time the drug is given (i.e. if a 5-year old child receives tetracycline therapy, the teeth affected will be the canines, premolars, and 2nd molars. Important: incisors and 1st molars have already completed enamel formation. RIEGER SYNDROME-a rare genetic disorder characterized by delayed sexual development, hypothyroidism, and dental features like hypodontia, underdeveloped premaxilla, cleft palate, and a protruding lower lip. Eye anomalies are is a min symptom of Rieger Syndrome. RECURRENT APHTHOUS ULCERS (Canker Sores)-the cause is UNKNOWN, but they can be triggered by stress, dietary deficiencies (iron, folic acid, vitamin B12), menstrual periods, hormonal changes, food allergies, etc. • Appear as painful white or yellow ulcers surrounded by a bright red area on non-keratinized oral mucosa and inner surface of the cheeks, lips,

tongue, soft palate, and base of gingiva. • Usually begin with a tingling or burning sensation, followed by a red spot or bump that ulcerates. Pain spontaneously decreases in 7-10 days, with complete healing in 1-3 weeks. • Recurrent aphthous ulcers & intra-oral herpes lesions are distinguished largely on their location. Recurrent aphthous ulcers occur mainly on mobile mucosa, while intra-oral herpes lesions occur on tissue bound to periosteum. • Occur more in women than men. May occur at any age, but usually first appear between ages 10-40yrs. 3 Classes of Recurrent Aphthous Ulcers: 1. Recurrent aphthous minor- < 1cm in diameter, are common, last over 2 weeks, and heal without scarring. 2. Recurrent aphthous major- > 1cm in diameter, are much less common, last over 2 weeks, and heal with scarring. 3. Recurrent herpetiform-consists of clusters and ulcers. Patients with frequent recurrences should be screened for diabetes mellitus or Bechet’s Syndrome. Topical Steroids are suggested to relieve aphthous

ulcer symptoms: Triamcinolone (Kenalog) in Orabase 0.1%, Disp: 5gm tube, Sig: coat the lesion with a thin film after each meal and at bedtime. Grand Mal Epilepsy-the most common seizure disorder (present in 90% of epileptics). ~60% of epileptics have this form alone, while 30% have other seizure types in addition to grand mal. Grand mal epilepsy can occur in any age. This tonic-clonic type seizure is produced by neurological disorders, or develops in a neurologically sound brain secondary to a systemic metabolic or toxic disturbance. Grand mal seizures usually last 2-5 minutes. • A young epileptic who has a grand mal seizure in the dental office generally recover if restrained from self-injury and oxygen is maintained. • Treatment for tonic-clonic seizures (grand mal) position the patient in the supine position, preventing injury to the convulsing patient, initiating basic life support (head tilt). If cyanosis occurs, oxygen is administered. Petit Mal Epilepsy (Absence Seizures)-almost always develops in childhood usually in children under age 16. Clinically, this seizure consists of a brief lapse of consciousness, normally lasting 5-10 seconds, and rarely lasts beyond 30 seconds. The patient makes no movement during the episode, and termination of the episode is equally abrupt. Management of petit mal and partial seizures is of a protective nature. The doctor merely

prevents any injury to the victim. There is little or no danger to the victim, so that even without assistance from staff members, morbidity seldom occurs. However, if this seizure should last a significant length of time, medical assistance is required. MEASLES (Rubeola)-a highly contagious viral illness characterized by fever, cough, and spreading rash due to PARAMYXOVIRUS. The incubation period is 1-2 weeks before symptoms usually appear. • Koplik’s Spots-1-2mm yellowish-white oral lesions pathognomonic of measles, appearing usually as necrotic ulcers surrounded by a bright red margin on the buccal mucosa. • Before immunization, measles were common during childhood, as 90% of the population was infected by age 20. GERMAN MEASLES (Rubella)-a fairly benign viral disease, with symptoms of a red, bumpy rash, swollen lymph nodes (around the ears and neck), and a mild fever. Some people feel achy. The virus can manifest in the oral cavity as small petechiae-like spots on the soft palate. The defects of congenital infection from an infected mother are more severe (enamel defects, hypoplasia, pitting, and abnormal tooth morphology). Hypoplastic primary incisors is a dental anomaly

resulting from maternal rubella in the first trimester of pregnancy. MUMPS-an acute contagious viral infection characterized mainly by unilateral or bilateral swelling of the salivary glands (usually the PAROTID = PAROTITIS). Mumps is usually a childhood disease, but may also affect adults. The papilla on the opening of the parotid duct on the buccal mucosa is often puffy and reddened. SMALLPOX (Variola)-an acute viral disease that manifest clinically by a high fever, nausea, vomiting, chills, and headache. The skin lesions begin as small macules and papules that first appear on the face, but rapidly spread to cover much of the body. Oral manifestations: ulceration of the oral mucosa and pharynx. Sometimes, the tongue is swollen and painful, making swallowing difficult. SCARLET FEVER-an exotoxin-mediated disease arising from group A-beta-hemolytic streptococcal infection that peaks in children 4-8 years old. Symptoms include strep throat, sudden onset of fever, sore throat, headache, nausea, vomiting, abdominal pain, muscle pain, fatigue, and STRAWBERRY TONGUE. • Enlargement of fungiform papillae extending above the level of the white desquamating filiform papillae gives the appearance of an “unripe strawberry”. During the course of scarlet fever, the coating disappears and the enlarged red papillae

extend above a smooth denuded surface, giving the appearance of a red strawberry or raspberry. • Penicillin is the drug of choice for treatment. Early diagnosis and treatment are important to prevent complications (i.e. local abscess formation, rheumatic fever, arthritis, and glomerulonephritis).

TOOH ABNORMALITIES Amelogenesis Imperfecta-an inherited condition in which teeth are covered with thin, malformed enamel. This genetic condition is transmitted as a dominant trait and causes SOFT, THIN ENAMEL. Teeth appear yellow due to dentin visible through the thin enamel. Teeth are easily damaged and susceptible to decay. The pulpal outline and root morphology appears normal. • Treatment: depends on its severity and demands of esthetic improvement. Since the dentin is normal, the teeth can be prepared for full crowns. Dentinogenesis Imperfecta-an inherited condition and dental anomaly transmitted as a dominant trait that causes undermineralized dentin. It can affect the primary & permanent dentition. Crowns are bulbous with short roots, and teeth may appear gray or brown with opalescent dentin that overgrows and obliterates the pulp cavity. Teeth wear rapidly. 3 types of DI exist: 1. Type I: often associated with osteogenesis imperfecta. Child may also have blue sclera, fragile bones, and hearing loss. 2. Type II: the most common type of dentinogenesis imperfecta.

3. Type III (Brandywine Type): characterized by multiple pulpal exposures in the deciduous dentition. Dens-in-Dente (Dens Invaginatus)-a “tooth within a tooth” caused by an invagination of all enamel organ layers into the dental papilla. It most frequently involves the MAXILLARY LATERAL INCISOR. The pulp is usually exposed, thus is necrotic or inflamed and the pulp canals should be cleaned and filled (RCT). This invagination can range in severity from an accentuated lingual pit to a “densin-dente”. Enamel Hypocalcification-a hereditary dental defect affecting the enamel of primary and permanent teeth. The enamel is soft and undercalcified in context, but normal in quantity due to the defective maturation of ameloblasts (defect in mineralization of the formed matrix). The teeth are chalky, the surfaces wear down rapidly, and a yellow-to-brown stain appears as the underlying dentin is exposed. Enamel Hypoplasia-a developmental defect in which the enamel is hard in context, but thin and deficient in amount due to defective enamel matrix formation with a deficiency in the cementing substance. There is a lack of contact between teeth, rapid breakdown of occlusal surfaces, and a yellowish-brown stain that appears where dentin is exposed. This condition affects both primary and permanent teeth, and can be transmitted genetically, or

caused by environmental factors (i.e. vitamin deficiency, fluorosis, or metabolic disturbances during the prenatal period). It is a common dental sequelae in a child with a history of generalized growth failure in the first 6months of life. Hypoplastic areas on teeth are seen if a child has illnesses in early childhood. Hypoplastic enamel is also a dental manifestation of hypoparathyroidism, preventable by early treatment with vitamin D. Concrescence-the state of teeth joined together only by cementum. A type of fusion that occurs after root formation is complete, possibly due to a traumatic injury. Gemination-a process where a single tooth germ splits or shows an attempt to splitting to form two completely or partially separated crowns, resulting in incomplete formation of two teeth. Most commonly occurs in the incisor region. Fusion-a condition produced when two tooth buds are joined together during development and appear as a macrodent (single large crown). Fusion usually occurs in the incisor area. The single crown may have two roots or a grooved root, but there are usually two root canals. It is often difficult to distinguish between gemination and fusion. Fusion or gemination of teeth occurs during the initiation and proliferation stages of tooth development.

ANODONTIA-a developmental abnormality characterized by the TOTAL ABSENCE OF TEETH. 1. Complete True Anodontia-a rare condition in which all of the teeth are missing. It may involve both primary and permanent dentitions; usually associated with hereditary ectodermal dysplasia. 2. Partial Anodontia (Congenitally Missing Teeth)-rather common and usually affects maxillary 3rd molars, maxillary lateral incisors, & mandibular 2nd premolars. As a rule, if only one or a few teeth are missing, the absent tooth is the most distal tooth of any given type (if molar, then it would be the 3rd molar). Oligodontia-congenital absence of MANY, but not all teeth. Hypodontia-absence of only a few teeth. Conditions that DELAY exfoliation of primary teeth and DELAY eruption of permanent teeth: • Systemic conditions: Cleidocranial dysostosis, Down syndrome, Ectodermal dysplasia, Gardner’s syndrome, Osteogenesis imperfecta, rickets, severe congenital heart disease, and mental retardation. • Localized pathologic conditions: abscess of a primary tooth, or ankylosis of a primary or permanent tooth.

• Hypothyroidism, Hypopituitarism, Hypoparathyroidism, and heredity/genetics. HEREDITY is most frequently responsible for the congenital absence of teeth. The roots of the primary tooth will resorb slower than normal without the presence of the permanent tooth. As a general rule, if only 1 tooth or a few teeth are missing, the absent tooth is the MOST DISTAL TOOTH of any given type (i.e. if a molar is congenitally missing, it is almost always the 3rd molar; if an incisor is missing, it is nearly always the lateral, if a premolar is missing, it is almost always the 2nd premolar). Rarely is a canine the only missing tooth. In the case of a congenitally missing 2nd premolar, you want to maintain the primary 2nd molar as long as possible. If it is still present, it may be ankylosed. Cessation of eruption is most diagnostic of an ankylosed primary molar. Over-retained primary teeth in the mixed dentition may prevent the normal eruption of permanent teeth, may be caused by the abnormal root resorption of the primary teeth, and are often treated by extraction. • Used caution when extracting over-retained primary teeth as the succedaneous tooth bud may be in close proximity. This is especially true when placing the beaks of forceps into bifurcations of primary molars in older children. The most common cause of fracture of root tips in extracting a primary

molar is root resorption between the apex and bifurcation. • If a permanent tooth bud is accidentally extracted while removing a primary molar, the best treatment is to immediately orient the tooth bud, replant the bud using digital pressure, and suture. The best way to extract a primary molar that has the permanent tooth bud close to it is to section the tooth and remove the parts individually. Child Periodontium vs. Adult Periodontium: • Child periodontium has greater blood and lymph supply. • Alveolar crest is flatter, and alveolar bone is thinner (especially in the lamina dura area) since there are fewer trabeculae in the alveolar bone of the pediatric patient. • Gingival pocket depths are larger and the attached gingiva is narrower. • Gingival tissues are redder since child’s gingiva is more vascular, thinner, and less keratinized. • There is lack of stippling because the C.T. of the lamina propria is shorter and flatter. • Child has rounded and rolled gingival margins due to normal eruption patterns. • Cementum is thinner and less dense than the adults, as cementum increases with age. • PDL fibers run parallel to the teeth. In adults, PDL

fibers are more horizontal against the tooth. The child’s PDL is also wider (this is why you may see more tooth mobility and decreased resistance to forces). PDL fiber bundles increase with age.

TOOTH DEVELOPMENT & ERUPTION Primary teeth begin to form at 6 weeks in utero & begin to calcify at 4 months in utero. Permanent teeth begin to develop at 4 months in utero. STAGES OF TOOTH DEVELOPMENT (Life Cycle): 1. Initiation (Bud Stage)-the initial interaction between oral epithelium & mesenchyme (ectomesenchyme) formation of dental lamina. Fused or geminated teeth occur during this stage. 2. Proliferation (Cap Stage)-the tooth’s shape is evident and the enamel organ is formed. Fused or geminated teeth occur during this stage. Fusion or gemination of teeth occurs during initiation & proliferation stages of tooth development. 3. Differentiation (Bell Stage or Histodifferentiation)final shaping of the tooth, cells differentiate into specific tissue-forming cells (ameloblasts, odontoblasts, cementoblasts, and fibroblasts) in the enamel organ. ► Dentinogenesis imperfecta & Amelogenesis imperfecta occur during the Bell Stage. 4. Apposition-cells that were differentiated into specific

tissue-forming cells begin to deposit the specific dental tissues (enamel, dentin, cementum, & pulp). The majority of cells of the dental pulp are fibroblasts. 5. Calcification (Mineralization)-primary teeth begin to calcify/mineralize during the 2nd trimester of pregnancy (14 weeks or 4 months in utero). In a 2 year-old child, 40 teeth have calcified. At birth, 20 deciduous and 4 1st molars have commenced mineralization (24 teeth). Maxillary and mandibular 1st molars begin to calcify at birth and are the first to begin calcification. Mandibular 3rd molars are the last teeth to begin calcifying at age 8-10 years. The cariostatic effect of fluoride is manifested during the calcification stage of tooth development. Tetracycline stain is incorporated and discoloration occurs during calcification. Calcification of the roots is completed by age 3-4. 6. Eruption-the emergence of tooth structure through the gingiva.

7. Attrition–physiologic wearing away of enamel & dentin due to NORMAL function or excessive GRINDING/GRITTING/CLENCHING teeth together (BRUXISM-pathologic attrition due to teeth grinding). The most noticeable effects are POLISHED FACETS (flat incisal edges that usually develop on the linguoincisal of maxillary canines & central incisors, and facioincisal of mandibular canines). Discolored tooth surfaces, and exposed dentin. Age-related process not due to caries Tooth development is initiated by mesenchyme’s inductive influence on the overlying ectoderm. Enamel is derived from ectoderm of the oral cavity. Ectodermal cells determine crown root and shape. All other tooth tissues differentiate from the associated mesenchyme (mesoderm). Tooth development depends on a series of sequential cellular interactions between epithelial & mesenchymal components of the tooth germ. Once the ectomesenchyme influences the oral epithelium to grow down into the ectomesenchyme and become a tooth germ, the following events occur: STAGES OF TOOTH HISTOGENESIS (formation and development of tooth tissues): 1. Elongation of the inner enamel epithelial cells of the enamel organ. This influences mesenchymal cells on the periphery of the dental papilla to differentiate into odontoblasts.

2. Differentiation of odontoblasts. 3. Deposition of the 1st layer of dentin then 1st layer of enamel, followed by deposition of root dentin and cementum. Korff’s Fibers-a rope-like grouping of fibers in the pulp periphery that are involved with formation of the dentin matrix. Thick collagenous fibers in the developing tooth that starts in the dental papilla. Hertwig’s Epithelial Root Sheath (HERS)-an epithelial diaphragm formed/derived from the joining of the inner + outer enamel epithelium of the enamel organ. After crown formation, the root sheath grows down and shapes the tooth root and induces root dentin formation. Uniform growth of HERS results in formation of a single-rooted tooth, while medial outgrowths or evaginations of the sheath produce multi-rooted teeth. When a tooth clinically erupts in the mouth, 2/3 of the root structure has developed. For primary teeth, the root is completely formed by 18 months (for permanent teeth, it takes 3 years). Lobes-primary centers of ossification always separated by developmental grooves that are very prominent in posterior teeth and form specific patterns. The minimum number of lobes from which any tooth may develop is 4.

• A lobe is one of the primary sections of formation in the development of the tooth’s crown (it represents a cusp on posterior teeth, and mamelons & cingula on anterior teeth). • Anterior teeth lobes are much less noticeable and are separated by developmental depressions. All anterior teeth have 4 lobes (3 labial, 1 lingual = cingulum). • Premolars have 3 buccal + 1 lingual lobe (except mandibular 2nd premolar which has 3 buccal + 2 lingual lobes). • 1st Molars have 5 lobes (5 cusps); one lobe for each cusp. The average age when the mineralization of permanent 1st molar crown is completed is 4-5years. • 2nd Molars have 4 lobes (4 cusps); 1 for each cusp. • 3rd Molars have at least 4 lobes (4 cusps) 1 for each cusp, but variation are seen. Maxillary 3rd molars usually have only 3 cusps. • Mamelons-rounded or conical prominences on the incisal ridge of newly erupted incisors that represent the 3 lobes from which the incisors developed from. Mamelons are usually worn off after the tooth comes into functional position. The presence of mamelons in a teenager or adult is evidence of malocclusion

(usually an anterior open bite relationship where the incisors do not touch).

“Rule of Four” enables you to determine the number of teeth present at any given time and implies the eruption of 4 teeth every 4 months beginning with 4 teeth at age 7 months. Deciduous dental formula: I 2/2 + C 1/1 + M 2/2 = 10 x 2 = 20 (5 per quadrant = 10/ arch = 20) Permanent dental formula: I 2/2 + C 1/1 + B 2/2 + M 3/3 = 16 x 2 =32 (8 per quad = 16/ arch = 32)

Closure of the apices of deciduous teeth occurs 18 months AFTER eruption. 3 Cardinal Rules of Tooth Eruption: 1. Girls teeth erupt before boys. 2. Mandibular teeth before maxillary. 3. Slender teeth before stocky (anterior before posterior teeth). MIXED DENTITION-a dentition phase during which some teeth present in the oral cavity are permanent, and some are deciduous (primary). The earliest indication of a mixed dentition consists of the primary dentition & permanent mandibular 1st molars. 3 periods of dentition in man: 1. Primary dentition (6 months to 6 years) 2. Mixed dentition (6 to 12 years) 3. Permanent dentition (12+ years) After the permanent teeth have reached full occlusion, small tooth movements occur to compensate for wear at the contact areas (by mesial drift) and occlusal surfaces (by deposition of cementum at the root apex).

Usual Exfoliation (Fall-out) Age of Deciduous Teeth: • Primary central incisors are exfoliated between ages 6-8. • Primary lateral incisors are exfoliated between ages 7-9. • Primary canines are exfoliated between ages 9-12. • Primary 1st & 2nd molars are exfoliated between ages 10-12. Primary Teeth vs. Permanent Teeth: • Primary teeth are lighter in color than permanent teeth. • Pulp cavities are proportionately larger in primary teeth. • Crowns of primary teeth are more bulbous and constricted than permanent teeth. • Crown surfaces of all primary teeth are much smoother than permanent teeth (less pits & grooves in primary teeth). • Crowns of primary anterior teeth are wider M-D and shorter incisocervically than permanent anterior teeth. • Crowns of primary molars are shorter and narrower M-D at the cervical third than permanent molars. • Roots of primary anterior teeth taper more rapidly than permanent anterior roots. • Roots of primary molars are longer and more slender than permanent molar roots. • Enamel ends abruptly at the cervical line on primary teeth, rather than becoming thinner as on permanent

teeth. • Buccal and lingual surfaces of primary molars are flatter above the crest of contour than on permanent molars. Primary Molars vs. Permanent Molars: • Primary crowns are shorter with pronounced B & L cervical ridges and a constricted cervical area. • Primary occlusal table is narrow F-L. • Anatomy is shallower (i.e. cusps are short, ridges are not as pronounced, and fossae are not as deep). • A prominent mesial cervical ridge (makes it easy to distinguish rights from lefts). • Roots are longer and more slender than the roots of permanent molars. The roots are extremely narrow M-D and very broad lingually. • Roots are very divergent and less curved. There is little or no root trunk. • The sum of the M-D widths of primary molars in any one quadrant is 2-5mm greater than the permanent teeth that succeed them (premolars). Also, the enamel on the occlusal surfaces of primary molars is of uniform thickness and is ~1mm thick, compared to permanent molars (2.5mm) thick.

PRIMARY DENTITION Succedaneous teeth-in each quadrant, 5 permanent teeth (incisor, canine, & premolars) succeed/replace the 5 primary teeth. Permanent molars do not replace primary teeth, thus are not succedaneous teeth. ♦ The last primary tooth to be replaced by a permanent tooth is usually the maxillary canine (the permanent maxillary canine usually erupts between age 11-12). Permanent mandibular canine usually erupts between 9-10yrs. Non-Succedaneous teeth-do not succeed/replace deciduous teeth (permanent maxillary & mandibular 1st, 2nd, & 3rd molars). A 6-year old should have all 20 primary teeth and 4 first molars (6-year molars) clinically visible in the mouth. • Permanent mandibular centrals erupt between ages 6-7. • Permanent maxillary centrals erupt between ages 78. A 7-year old should have 18 primary teeth (except the two mandibular centrals) and 6 permanent teeth (all 4 first molars & 2 mandibular central incisors) clinically visible.

Mandibular Central Incisor-the first deciduous (primary) tooth to erupt. MAXILLARY CENTRAL INCISOR-the first succedaneous tooth to erupt. ► The crown of the primary maxillary central incisor is larger M-D than the permanent maxillary central incisor. ► Primary maxillary central incisor has a shorter crown length inciso-cervically than the permanent maxillary central incisor. ► primary maxillary central incisor’s incisal edge is straighter than the permanent maxillary central’s incisal edge. ► No mamelons are present on the primary maxillary central incisor, only on permanent maxillary central incisors. ► Labial and lingual cervical ridges are prominent on all primary incisors. When extracting primary incisors where the roots have been partially resorbed due to pressure from the developing permanent teeth, the facial part of the remaining primary root is usually the longest and most securely attached to the gingiva. Primary Maxillary Lateral Incisor-the most common congenitally missing primary tooth (although this is rare). This tooth is most often atypical in size (i.e. peg-shaped).

► A patient who has permanent central incisors, permanent canines, and primary canines anterior to the premolars most likely has congenitally missing permanent lateral incisors. ► Most common congenitally missing permanent tooth is the 3rd molar, then the mandibular 2nd premolar, then the maxillary lateral incisor. Primary Maxillary Canine-the teeth most likely to be crowded out of the arch. ► Mesial cusp ridge is longer than the distal cusp ridge, and its cusp is much longer & sharper than the permanent maxillary canine. This is the opposite on all other canines. It also is very wide and short. Permanent Mandibular 2nd Premolars-the tooth most likely malposed in cases of mandibular arch space discrepancy. Primary Mandibular 1st Molar-this tooth does not resemble any other primary or permanent tooth. • It has an oval occlusal surface that is wider M-D than B-L. • From an occlusal view, the MB angle is acute and prominent due to the mesial cervical ridge on the buccal surface. The DB angle is obtuse. • Occlusal table is rhomboid-shaped, and is the

chewing surface inside the cusp ridges and marginal tables. • Has a prominent transverse ridge that unites the MB + ML cusps, and separates the mesial portion from the rest of the occlusal surface. • MB cusp is always the largest and longest cusp, occupying nearly 2/3 of the buccal surface. • ML cusp is larger, longer, and sharper than the DL cusp. • Crown is wider M-D than cervico-occlusally. • Mesial marginal ridge is very well developed and resembles a cusp. It has a prominent MB cervical ridge. • Class II cavity preparations are difficult due to its morphology, and it has NO central fossa. • Mandibular 1st Molar- first permanent tooth to erupt and first to begin calcifying. • Maxillary 1st Molar- second permanent tooth to erupt. • Maxillary and mandibular 1st molars are NOT succedaneous teeth. • Primary 1st molars are the teeth with the most noticeable morphologic deviations from permanent teeth. Primary Mandibular 2nd Molar: • Its morphology and amalgam preparation outline closely resembles the permanent mandibular 1st molar.

• Relative size of the distal cusp. The MB, DB, and distal cusps are nearly equal in size. However, the distal cusp of the permanent molar is smaller than the MB and DB cusps. • From the buccal aspect, it is has a narrow M-D dimension at the cervical portion of the crown compared to the dimension M-D on the crown at the contact level. The mandibular permanent 1st molar is wider at the cervical portion. • Groove patterns are different on the occlusal surface. • Primary molar has more divergent roots to allow eruption of the 2nd premolar. • Primary molar has a more prominent facial crest of contour. • Primary 2nd molar has the greatest F-L diameter of all primary teeth. Primary Maxillary 1st Molar-this is the most atypical tooth of all the molars (primary and permanent). It is intermediate in form and development between a premolar and molar. ► It is the smallest molar in all dimensions EXCEPT the labiolingual diameter. ► The crown of this tooth is bicuspid (two cusped). It has a wide MB (the longest cusp) and narrow ML cusp (second longest and sharpest). The indistinct cusps are the DB & DL. ► Its cervical line is higher mesially than distally. ► The cervical ridge stands out very distinctly on the

MB portion of the tooth. ► H-shaped occlusal pit-groove pattern. ► Primary maxillary 1st molar has THREE ROOTS and its root form closely resemble the permanent maxillary 1st molar. Primary Maxillary 2nd Molar-while smaller, this tooth morphologically resembles the permanent maxillary 1st molar. ► Crown’s F-L measurement is greater than its M-D measurement. ► May have a 5th cusp of Carabelli. ► Prominent MB cervical ridge and an oblique ridge. ► MB cusp is almost equal in size or slightly larger than the ML cusp. ► MB pulp horn is the largest and longest. ► Primary 2nd molars are larger than primary 1st molars and resemble permanent 1st molars.

Important Human Development Facts: • At age 6, a child’s head is 90% of its adult size. This is typical of all neural tissues in the body. At birth, the

cranial vault is very near the size it will eventually attain in adulthood (compared to the cranial base, mandible, mid-face, etc.). The brain and cranial base are fully developed by age 6. • At birth the jaw is large enough to accommodate all primary teeth if they were to erupt simultaneously. • At birth, the width of the face has reached its greatest percentage of its adult size (as opposed to height and depth). • At birth, the palate is flat. In adults, it is vault-shaped (this occurs by deposition of alveolar crestal bone). • In early life, tonsils function to filter bacteria and program the production of antibodies. • At birth, a newborn cannot differentiate between sour, salt, or bitter tastes. • From ages 6-12, the body’s lymph tissue is 200% of its normal adult mass. Consequently, enlarged tonsils in a 6-year old are at age 12, most likely smaller because lymphoid tissue in the nasopharynx decreases at puberty, while at the same time, the genital tissue is developing.

FLUORIDE FLUORIDE is the most effective caries prevention agent available. It is completely safe when used properly. However, ingestion of high fluoride concentrations can cause nausea, vomiting, dental fluorosis (mottling), or in extreme cases, death (especially in children). • Preventive dentistry for Adolescents: frequent dental visits, fluoride rinses at home, fluoride tray applications, brushing and flossing, sealants, and fluoride tablets. • Topical fluoride (with occlusal sealants) is the primary preventive agent during adolescence (past age 12) because the entire dentition (except 3rd molars), normally erupts by age 13. Thus, fluoride tablets may not be as beneficial. Fissure sealants succeed by altering host susceptibility. • Caries activity is directly proportional to the consistency, frequency, & oral retention of fermentable carbohydrates ingested. To prevent dental caries, the Centers for Disease Control & Prevention recommends at least 0.7ppm of fluoride be present in drinking water. The maximum fluoride amount is 1.2ppm.

The optimal concentration of fluoride for community water depends on AIR TEMPERATURE. A 15-year old female has lived in a non-fluoridated area her entire life. When she moves to a community where the drinking water naturally contains 6ppm of fluoride, she will experience an increase in the amount of fluoride stored in her bones. • Moderate fluorosis will not occur, because by age 15 her entire dentition has undergone complete enamel calcification (except possibly the 3rd molars). A 50% reduction in dental caries is not probable because her entire dentition has already undergone complete enamel calcification. • Water fluoridation and diet supplementation may affect tooth morphology, while self and professionally applied topical treatments will not. • Fluorides often added to water supplies are sodium fluoride, sodium silicofluoride, & hydrofluosilicic acid. • Deposition of fluoride occurs on the smooth surfaces of teeth. Fluoride’s main effect occurs AFTER the tooth has erupted above the gingiva (“post-eruptive”). This topical

effect happens when small amounts of fluoride are maintained in the mouth in saliva and dental plaque. Fluoride works by stopping or even reversing the decay process. It keeps enamel strong and solid by preventing the loss of and enhancing the re-attachment of important minerals from tooth enamel. Fluoridation of community water is credited for reducing tooth decay by 50-60% in the U.S. since WWII. Recent estimates show decay reduction at 18-40%, which reflects that even in communities that are not optimally fluoridated, people are receiving some benefits from other sources (i.e. bottled beverages, toothpaste). Professional Applied Topical Fluorides (applied 2x/year): 8% Stannous Fluoride, 1.23% Acidulated Fluorophosphates (APF gel), & 2% Sodium Fluoride. 1. SODIUM FLUORIDE (NaF 2% or 5% = 22,600 ppm)advantages are basic pH (9.2), more acceptable taste than stannous fluoride, and no adverse effect on restorative materials (does not etch or roughen composite and porcelain restorations). MOST EFFECTIVE professional fluoride delivery system for patients with RAMPANT CARIES, GERD, or BULEMIA is the TRAY METHOD WITH SODIUM FLUORIDE (NaF). 2. STANNOUS FLUORIDE (SnF2) (8%)- an unstable solution that must be freshly mixed, tastes bad (metallic due to tin ion), stains silicate restorations, and is acidic

(2.1-2.3 pH). SnF2 only advantage is it does not etch porcelain restorations. At one point was not used in any approved dentrifices in the U.S. • STAINED (DISCOLORED) demineralized areas and margins of enamel (pits, fissures, grooves), composite, and porcelain YELLOWISH-BROWN due to the reaction of the TIN ION. May also cause gingival sloughing. • Has bacteriocidal and bacteriostatic properties. Used as an ingredient due to its wide range of benefits against caries, plaque, gingivitis, dentinal hypersensitivity, and breath malodor. • Before 2006, stannous fluoride was not FDA approved in U.S. dentrifices, however, P&G produced a new stannous fluoride formulation that tastes better and causes less staining. New formulations like Crest-Pro Health combine stannous fluoride with sodium hexametaphosphate (fights stain and tarter). • The main advantage of topically applying an 8% solution of stannous fluoride instead of a 2% solution of NaF, is a single treatment may be given. 3. ACIDULATED PHOSPHATE FLUORIDE (1.23%)professionally applied, more acceptable taste than stannous fluoride, and can be applied to both arches

simultaneously. However, it may damage porcelain restorations. • pH of APF gels falls in the 1-4 range (pH is 3.0-3.5 which is acidic). • APF is contraindicated on porcelain and composite restorations because it causes pitting and etching of these materials. Acidulated fluorides should be avoided on implant patients because they can corrode the surface of titanium implants. • The most effective way to increase fluoride content in the external tooth layers is a daily application of 1.23% APF in fitted trays for 4 minutes. However, this is not realistic since we do not routinely do “daily” applications.

When painting fluoride on, it is critical to isolate the teeth with cotton rolls. When using fluoride trays, cotton rolls may be placed in premolar areas to increase patient comfort and help keep the fluoride in place. Patients are asked not to brush, rinse, eat, or drink for 30 minutes after a

fluoride treatment so the fluoride is left undisturbed and is able to continue reacting with the hydroxyapatite for some time after the initial application. Fluoride treatments should be applied for 4 minutes, although there are now some 1-minute products available on the market.

Beneficial Effects of Fluoride: 1. Interferes or inhibits plaque formation on enamel surfaces. Dental plaque adheres to teeth because DEXTRANS are insoluble and sticky. 2. Has antibacterial qualities depending on the concentration (fluoride is bactericidal in high concentrations). 3. Enhances enamel remineralization to help reduce and eliminate early carious lesions. 4. Decreases enamel solubility. 5. Inhibits glycolysis (the process by which sugar is metabolized by bacteria to produce acid). When glycolysis is inhibited, the caries process is inhibited.

Fluoride Mechanisms to Inhibit Caries: 1. Topical effect of constant infusion of a low fluoride concentration into the oral cavity ↑ enamel remineralization. • One of the most effective means of reducing caries activity is by decreasing the solubility of enamel to the acid attack of bacteria. Enamel demineralization begins when the pH reaches 5.5 (the pH threshold level at which enamel demineralization occurs). • The main anti-cariogenic effect of fluoride is by enhancing remineralization. Its secondary role is to decrease demineralization by increasing the resistance of the tooth’s outer surface (enamel). Lastly, the least understood property of fluoride are its antimicrobial properties. 2. Fluoride converts hydroxyapatite→fluorapatite by substituting OH- for Fl-. F- decreases the solubility of hydroxyapatite crystal, while increasing the crystal size. Fluoride ion is easily exchanged with the hydroxyl ion in the lattice structure of enamel because Fl- is slightly smaller than OH- with a greater affinity for hydroxyapatite crystals than OH-. 3. Fluoride inhibits glycolysis where sugar is converted into acid by bacteria (fluoride ion inhibits enzymatic production of glucosyltransferase).

Fluoride mouth rinses are shown to have the greatest effect on NEWLY ERUPTED TEETH, making it essential to have rinsing continued into the teen years to protect the 2nd and 3rd permanent molars. Fluoride rinses are most beneficial to SMOOTH tooth surfaces and there are some benefits to pits & fissures.. Systemic fluorides are least effective on root surfaces. During a routine prophylaxis, it is possible that the use of abrasive polishing agents may remove the fluoride-rich layer of enamel. The greatest concentration of fluoride ions exist on the OUTERMOST ENAMEL LAYER (not just beneath the enamel surface). Since this fluoride-rich layer may be inadvertently removed while polishing, all patients would benefit from a topical fluoride application after a routine prophylaxis. Dicalcium Phosphate-the toothpaste/dentrifice component most likely to inactivate fluoride ion. • Some believe the pumice prophylaxis may be omitted as studies show fluoride can make its way through the pellicle and plaque to the tooth surface. However, heavy stains may interfere with fluoride absorption, thus stains should be removed.

Factors to Consider when Prescribing a Child Fluoride Supplement: • Amount of fluoride in the child’s drinking water, child’s age, and how responsible the person administering the fluoride supplement is (whether it be the patient or patient’s parents). When determining the appropriate dose of systemic fluoride supplement for a child, it is most important for the dentist to consider the child’s age and fluoride content of the drinking water. Before prescribing topical and/or systemic fluoride, the dentist must consider the child’s AGE.

Table Reads: • The appropriate amount of DAILY fluoride supplement that should be given to a 6-16 year old patient in an area with a 0.25 ppm water fluoride concentration is 1.00 mg. • The appropriate amount of DAILY fluoride supplement given to a 6 month-3 year old child living in an area with a 0.5 ppm fluoride ion concentration in

drinking water is NONE. Emergency treatment in the dental office for a child who has accidentally ingested a large amount of fluoride includes: ► Induce vomiting manually or with the help of Ipecac syrup and call 911. ► Have the patient drink a large quantity of milk or another calcium-containing liquid to decrease stomach acidity and to form complexes with the fluoride to decrease its absorption. ► Do NOT have the patient drink large quantities of sodium bicarbonate or crush ammonia vaporole under the patient’s nose. Acute Fluoride Toxicity symptoms may appear within 30 minutes of ingestion and persist for up to 24hrs. Patients may experience nausea, vomiting, diarrhea, and abdominal cramping since 90-95% of ingested fluoride is absorbed through the stomach and small intestines. Fluorides are mainly eliminated from the body via the kidneys. However, the fluoride that remains in the body is found mostly in skeletal tissue. Acute Fluoride Poisoning-is rare, but the most common causes of death are cardiac failure & respiratory

paralysis. Fluoride toxicity shows up in the bones as osteosclerosis. Ingestion of 15mg/kg of fluoride can be lethal to a child. The adult lethal dose is 4-5gm. In mg/kg body weight, the LETHAL DOSE of fluoride falls in the range of 20-50mg/kg. If a 6-year old child were receiving fluoridated water in the amount of 3ppm, the result would most likely be fluorosis, but not systemic toxicity. However, if a child age 6-7 were receiving 8ppm of fluoridated water, there is a good chance of systemic toxicity and moderate-to-severe fluorosis. For an adult, the LETHAL DOSE of fluoride is between 2.5-10g, with the average lethal dose is 4-5g. Death has occurred in infants with as little as 0.25g (250mg). Death is likely in a child who ingests > 15mg of F-/kg body weight (i.e. 5 teaspoons of APF gel for a 44lb child). Thus, it is weight dependent. Most fluoride absorption occurs in the stomach (an acidic pH is required to facilitate this diffusion process). The minimum does that could cause toxic signs and symptoms (including death), and that should trigger immediate therapeutic intervention and hospitalization is 5mgF/kg weight = 2.27mgF/lb (this is the “probable toxic dose”)

Most effective method to reduce dental caries in the general population is FLUORIDATION of the communal water supply. The optimal concentration in the communal water supply varies with mean annual temperature, but is 1ppm in most states. Fluoride supplements are recommended if the water fluoride content is < 0.7ppm. Fluoride supplements are normally NOT indicated after age 13. • School water fluoridation optimal concentration is 4.5x that of city water supplies because of less water consumption at school. • A child should STOP taking fluoride supplements at 16-18 years old. • It is not necessary to prescribe fluoride supplements to a child who is consuming optimally fluoridated water. • Children in an elementary school exhibit a high interproximal caries rate. For these children, the school-based program that will be most effective is fluoride mouthrinse. In communities without fluoridated water supplies, the most cost-effective method of delivering fluoride to 612 year old children is via school water fluoridation (it is better than tablets, brushing with fluoride gel, or rinsing with fluoride mouthrinse).

Fluoride Therapies for a 13-year old child prone to decay who lives in a community where the water is appropriately fluoridated: • Professionally applied fluoride every 6 months, and fluoride toothpaste. • Low concentration fluoride mouth rinse. High concentration mouth rinses must be professionally prescribed and require a prescription. These rinses reduce root surface hypersensitivity. • Dietary fluoride supplements are contraindicated since the community water is appropriately fluoridated. DENTAL FLUOROSIS-an “irreversible” diffuse symmetric hypomineralization disorder of ameloblasts that only occurs with exposure to fluoride when enamel is developing (during the calcification period). Dental fluorosis is a toxic manifestation of chronic (low-dose, long-term) fluoride intake. Dental fluorosis is not caused by repeated topical application of fluoride. • Dental fluorosis typically causes “mottled” discoloration and pitting of the enamel of permanent and deciduous teeth. The severity of mottling increases with an increasing amount of fluoride in the drinking water. Thus, there is little mottling at a level below 0.9-1.0ppm of fluoride in the

drinking water, while children living in temperate zones where the water supply contains a higher content of fluoride are most often affected. • If a child lives in a temperate zone most of their childhood (up to age 9-10yrs.), all permanent teeth will most likely be affected (remember: 3rd molars begin calcification between ages 7-9yrs, so if the child moves before this age, the 3rd molars may not be affected). • Systemic distribution of fluoride may affect tooth morphology. Systemic distribution of fluoride is accomplished by fluoride in the water, fluoride tablets, and vitamins that contain fluoride. • Fluorosis may result from excessive fluoride consumed during the mineralization stage of tooth development, and can occur in permanent and primary teeth. When a 15-year old female who has lived in a nonfluoridated area all her life moves to a community where the drinking water naturally contains 6ppm of fluoride, an increase in the amount of fluoride stored in her bones is most likely to occur. Moderate fluorosis will not occur, because by age 15, her entire dentition has undergone complete enamel calcification (except possibly the 3rd molars).

3 Fluorides used in Public Drinking Water Supply: sodium fluoride, sodium silicofluoride, & hydrofluosilicic acid (supplied as an aqueous solution in 55 gallon drums and is commonly used at ground water well houses). The first two fluorides are supplied in bulk, crystalline form, and are regularly used at water treatment plants. * The use of hydrofluosilicic acid is advantageous because of the relatively inexpensive equipment required compared to the equipment used with powered components. Hydrofluosilicic acid is added directly to the water by a chemical dosing system that consists of a chemical feed pump, analyzer/controller, injector, drum scale, and 55-gallon drum of the acid solution. Diluting the acid prior to adding it to the water is not recommended because of the hazards involved in handling the acid, and errors associated with dilution. * When fluoride concentrations in excess of 1.5-3.0mg/L are consumed, a chemical combination can occur within tooth enamel, resulting in mottled and permanently discolored teeth. Concentrations > 4mg/L can be toxic. FLUORIDE “TEST PEARLS”: * Fluoride does NOT reduce caries by making enamel harder, but reduces its rate of solubility.

* Substantial reductions in dental decay in the young U.S. population are due to use of systemic & topical fluorides. * Fluoride concentration in body fluids is regulated by an equilibrium relationship between bone & urinary excretion. * Most fluoride is absorbed in the small intestine and excreted through the kidneys. * Fluoride passes the placental barrier slowly and is deposited in calcified tissues (i.e. bones). * At 1.0ppm fluoride is tasteless, colorless, & odorless. 1.0ppm is the optimum fluoride concentration in community drinking water. * U.S. Public Health Service sets the optimal fluoride level at 0.7-1.2ppm for public water. Optimal fluoride concentration in community drinking water depends on the average air temperature and water consumption. For temperate climates, the optimum is 1.0 ppm, and for warmer and colder climates the amount is adjusted from 0.7-1.2ppm, respectively. * Fluoride uptake on teeth depends on the amount of ingested or delivered fluoride in contact with the tooth during the day. * Fluoride is deposited in calcified tissues (skeletal), and normally accumulates slowly in bones as a person ages. * Proximal tooth surfaces derive the greatest benefit from fluoridation.

* Dental fluorosis can occur in permanent and deciduous teeth. * Fluoride’s cariostatic effect is produced during the calcification stage of tooth development. * Fluoride converts hydroxyapatite→fluorapatite to decrease enamel’s solubility. Fluoride helps teeth in two ways. When children under age 6 ingest fluoride in small doses, it becomes incorporated in their developing permanent teeth, making it harder for acids to cause demineralization. Fluoride also works directly on teeth in children and adults by speeding up remineralization and disrupting acid production by bacteria. FLUORIDE CHARACTERISTICS: • Fluoride concentration increases in the external layer of enamel throughout life. • Fluoride concentration increases during topical application, but decreases for a few days after treatment. • Increasing fluoride content in the external tooth layers increases enamel’s resistance to demineralization. • Fluoride uptake is greater in enamel than in dentin or cementum. When painting fluoride on, it is important to isolate the teeth with cotton rolls. When using fluoride trays, cotton rolls may be placed in the premolar areas to increase

patient comfort and kept the fluoride in place. Patients should not brush, rinse, eat, or drink for 30 minutes after fluoride treatment so that the fluoride is left undisturbed and can continue to react with the hydroxyapatite for some time after the initial application. Fluoride treatments should be applied for 4 minutes, although there are now some 1minute products available.

HEAD & NECK CANCER patients can benefit from using Sodium fluoride & Stannous fluoride for home-care custom tray use. The gel contains 1% sodium fluoride or 0.4% stannous fluoride. For maximum benefit, the gel must be in direct contact with teeth. Fluorides are recommended to protect cancer patients from postirradiation caries. • Fluoride found in commercial toothpastes is not adequate for people who had head and neck radiation. These patients must continue to use the fluoride gel as directed for their entire life to protect their teeth from rampant decay.

• Patient Instructions: the trays containing the fluoride are placed over the teeth for a prescribed period of time (usually 10 minutes), and the patient cannot eat or drink for at least 30 minutes. This is usually done at night after toothbrushing and just before going to bed. • Daily use of fluoride gel in custom trays at home is indicated in these situations: * Rampant enamel or root caries in any age group. * Xerostomia (can be due to medications or head/neck radiation therapy). * Use on abutment teeth under an overdenture. * Hypersensitive root surfaces. Components of Sodium Fluoride Paste to Treat Root Sensitivity: sodium fluoride, kaolin, glycerin (present in paste in equal parts). The rationale of desensitization procedures is not fully understood. Some techniques may depend on denaturation of the superficial ends of Tomes’ fibers or nerve endings in dentin. Other procedures are designed to deposit an insoluble substance on the ends of the fibers or nerves to act as a barrier to stimuli. Others are designed to stimulate secondary dentin formation to insulate the pulp from external stimuli. Hydrodynamic Theory-the most accepted theory to explain unusual sensitivity and response of exposed

root surfaces to various stimuli which postulates the pain results from indirect innervation caused by dentinal fluid movement in the tubules which stimulates mechanoreceptors near the predentin. • Technique: After cleaning the hypersensitive areas, rub the paste into the exposed root surfaces with a porte-polisher and orangewood stick or rubber cup for 1-5 minutes. Satisfactory results are usually obtained. No caustic effects on the gingiva or mucosa result from contact with the paste. However, it is toxic if accidentally ingested.

SEALANTS SEALANTS-a preventative barrier to protect tooth occlusal surfaces (PITS & FISSURES) from bacteria that cause dental caries by sealing unprotected pits and fissures with resin. Sealants also provide a smooth surface for saliva and toothbrush bristles to clean. • Sealants are generally comprised of BIS-GMA RESINS (bisphenol A diglycidyl ether methacrylate) that does not hydrolyze into BPA. The ADA currently states patients are NOT at risk from BPA exposure through dental sealants since BPA exposure is too low and infrequent (sinece the longest duration of salivary BPA was 3 hours). • Sealants can be filled or unfilled sealants. Fillers (glass and quartz) provide hue, strength, and enhance wear resistance. Unfilled sealants typically do not require occlusal adjustment (ex: 3M Clinpro), while filled sealants (ex: VOCO Grandio) must be adjusted if hyper-occluded. Placing sealants is a highly effective way to prevent pit and fissure caries. It is safe, but underused in the private and public dental health care delivery systems. The substantial reductions in dental decay in the U.S. youth population is due to systemic and topical fluorides. The

control of smooth surface caries provided by fluorides is critical to the additional effectiveness of sealants. Sealants are highly effective in preventing pit and fissure caries. • When sealants are applied correctly, there is a decreased development of new carious lesions and a decreased progression of pre-existing lesions. • A close correlation exists between a sealant retention and their effectiveness. The effectiveness of sealants is equal whether applied by dentists, hygienists, or dental assistants provided that they have received appropriate training. • Curing sealant materials occurs one of two ways. Some sealants are chemically cured via “autopolymerization”. These materials are dispensed as two components. As soon as the components are mixed, polymerization begins. Curing is complete in ~60 seconds. Other sealant material is cured with visible light, but the curing light must be of high quality and should be tested frequently for the value of light emitted. Retention rates for chemically cured and light cured sealants are similar. • Low viscosity sealants wet acid-etched tooth surfaces the best. • Sealants need micro-mechanical retention. The surfaces should be cleaned with a prophylaxis brush

or rubber cup and pumice with water. When the teeth are effectively isolated from saliva contamination, the surfaces are dried and acid etched by applying 3050% phosphoric acid solution for 1 minute. The solution should be gently agitated during application and is then washed away and dried to leave a frostyappearing etched surface. • The properties of sealants are closer to unfilled direct resins than to filled resins (composites). Sealants are weaker than filled resins, but their strength is sacrificed allow it to flow into pits and fissures (its viscosity must be low enough to flow into pits and fissures). • The most likely result of inadvertently sealing a small carious lesion in the occlusal surface is the caries is arrested. • Research indicates pit and fissure sealants are best retained on maxillary and mandibular premolars. However, maxillary and mandibular 1st molars benefit the most from sealants. The principal feature of a sealant required for success is ADEQUATE RETENTION. Sealant success is highly depends on obtaining and maintaining intimate adaptation of the sealant to the tooth surface to hopefully seal it. Research shows caries protection is 100% in pits and

fissures that remain completely sealed. Components of Pit & Fissure Sealants: 1. Bis-GMA: monomer that can be diluted with TEGDMA to reduce viscosity. 2. Initiator: Benzyol Peroxide in self-cured sealants and Diketone in visible-light cured. 3. Accelerator: amine is self-cured. 4. Opaque Filler: small amounts of TiO2 (titanium oxide) are added to make the appearance slightly different from the occlusal enamel. Sealant materials can be unfilled, filled with opaquer, clear, and colored. However, there is no difference in retention rates. Filled materials are easier to see and monitor, but clear materials allow the operator to continue to see the filled fissures. Sometimes operators prefer colored sealants to make monitoring retention easier. The newest sealant material is “fluoride releasing” where fluoride is released from the sealant after polymerization. The clinical significance of this is not yet determined, but the fluoride release is thought to occur at the base of the sealed groove, which provides a fluoride-rich layer available when remineralization of an incipient lesion occurs. If a topical fluoride is used in conjunction with a pit and fissure sealant, the fluoride is applied AFTER the sealant. When topical fluoride is used, it is applied either

before the conditioner (acid etchant) or after the sealant. When cleaning the tooth prior to application, do not use glycerine-containing products. Applying fluoride after the conditioner decreases the bond strength. Technique for Applying Sealants: 1. Gross debridement of enamel surfaces (use plain flour or pumice). Do not use anything that contains fluoride because it inhibits etching. Rinse thoroughly. 2. Isolate teeth with a rubber dam or cotton roll to keep the tooth dry. Dry isolated teeth thoroughly. 3. Acid-etch the teeth with phosphoric acid (30-50% concentration). Apply etchant by gentle dabbing of the enamel surface. Etching time for permanent teeth is 1 minute (2 minutes for primary teeth). 4. Rinse and dry thoroughly. A properly etched surface appears dull and chalky. 5. Apply sealant (procedure differs with the method of polymerization). 6. Evaluate results, floss the tooth, and check occlusion. If a child is returns for a periodic evaluation and prophylaxis and you find that two of the four sealants placed at the prior re-care were lost, it was most likely due to one or more of the following: 1. Etchant was not rinsed thoroughly off the tooth surface. 2. Dry field was not maintained and the tooth surface was contaminated with saliva after etching. 3. Contaminated air supply to air/water syringe.

4. Tooth was not thoroughly dried prior to applying the sealants. Sealants act as a PHYSICAL BARRIER to prevent bacteria accumulation in pits and fissures. • Care is taken not to mix the sealant resin too vigorously prior to placement or to over-manipulate the sealant resin upon placement. Either of these errors can incorporate air into the sealant resin, causing a void in the sealant surface. As long as the sealant remains in tact, decay will not develop under it. • Factors that may influence which teeth are candidates for sealants: presence of interproximal decay, patient age, how caries prone the patient is. • Sealants should be placed right after the tooth has fully erupted, before the decay process has begun. For permanent molar placement, this is usually around age 6 and 12 (give or take 6 months).

TOOTH TRAUMA The alternate loosening and tightening of a deciduous tooth before it is shed due to the alternate resorption and apposition of cementum and bone. If during a routine exam, you notice a permanent tooth trying to erupt while the primary tooth remains firmly in place, the best treatment is to extract the primary tooth and allow the permanent tooth to erupt. • In most cases, proper treatment for an intruded primary anterior tooth is NO TREATMENT. Immediate attention should be given to soft-tissue damage. However, an x-ray of the area should be taken. If the intruded incisor is contacting the permanent tooth bud, the primary tooth should be extracted. • If during the first 6 months after the injury, you observe pulpal necrosis (tooth darkens), the tooth can receive RCT if needed as long as the tooth is sound in the socket and no pathologic root resorption is evident. If the tooth is asymptomatic, leave it alone. • Repositioning displaced mobile primary teeth is not recommended. Extraction is recommended due to potential aspiration in young children.

Discolored primary teeth that are asymptomatic and show no radiographic changes should NOT BE TREATED. They should be examined periodically by taking a radiograph. Primary teeth often darken (become gray) after injury due to pulp bleeding and diffusion of biliverdin into the dentinal tubules. ► 80% of primary incisors that darken due to injury are asymptomatic. Occasionally, these teeth will lighten. • 15% of these teeth will need to be extracted within 1 year due to repeated trauma. • 85% of these teeth will remain until normal exfoliation. ► As a result of trauma to the primary dentition, there should not be problems with permanent successors unless the crown is not calcified. In this case, the tooth will be hypocalcified (most common with mandibular incisors). underdeveloped motor coordination is the most common cause of dental trauma in very young children’s primary dentition ages 1 ½ - 2 ½ years old. ► Recently traumatized teeth may give false negative responses to pulp vitality tests. This impaired nerve conduction may be temporary or permanent,

but only time will tell. Crown Fracture Classification: 1. Ellis Class I Fracture-a simple crown fracture involving little or no dentin. Treat with enameloplasty and/or bonding. 2. Ellis Class II Fracture-an extensive crown fracture involving considerable dentin, but not pulp. The standard of care used to involve covering exposed dentin with calcium hydroxide or a glass component cement to seal out oral flora. It is now recommended to also place a glass ionomer restoration. 3. Ellis Class III Fracture-extensive crown fracture with pulpal exposure. Treatment include pulp therapy via pulp capping, pulpotomy, or pulpectomy, followed by a permanent restoration. Usually, a Class III fracture in a primary tooth leads to pulpal necrosis. RCT using ZOE paste as a filling material is indicated. Unlike gutta percha, ZOE resorbs with resorption of the primary tooth roots. 4. Ellis Class IV Fracture-a fracture where the entire crown is lost. Treatment is pulpectomy. The tooth restoration involves using a stainless steel or celluloid crown. Root fractures of primary teeth are relatively

UNCOMMON because the more pliable alveolar bone allows primary tooth displacement. When a primary tooth root fractures, it is treated identical as for permanent tooth root fractures. However, the prognosis is LESS FAVORABLE. The pulp in a permanent tooth with a fractured root has a better chance to recover since the fracture allows immediate decompression and circulation is more likely to be maintained. • Root fractures in the APICAL THIRD are often repaired without treatment (root fractures in the apical third are more likely to undergo self-repair). Many dentists recommend a relatively long stabilization period of 2-3 months for teeth with fractured roots. A longer stabilization period encourages a more favorable type of healing with calcified tissue. HEAVY WIRES are recommended when stabilizing teeth with fractured roots. Splinting is NOT recommended in the primary dentition. • Occlusion should be adjusted so the injured tooth is not further damaged during normal mastication. Follow-up radiographs and pulp tests are done at frequent intervals during the 6-month period after the injury. These teeth often remain functional and vital. • Fractured maxillary anterior teeth occur most

often in children with Class II, Division I malocclusion (flared maxillary anteriors). There is no reliable method to determine pulp vitality in the case of a recently traumatized primary tooth. Often, traumatized teeth do not respond to vitality testing. Pulp vitality testing is NOT routinely performed in the primary dentition because primary teeth do not respond to such test reliability since the test requires a relaxed and cooperative patient objectively reporting a reaction. Congestion of blood within the pulp chamber a short time after injury can often be detected in the exam. Shining a bright light on the facial surface and holding the mirror to view the lingual usually shows a reddish hue which indicates pulpal hyperemia. If this red color change is evident after several weeks, it often indicates a poor prognosis. EPT is seldom reliable to determine pulp vitality if taken immediately after the injury. The thermal test is the MOST RELIABLE, especially in primary incisors. Failure of a tooth to respond to heat indicates pulpal necrosis. Panoramic Radiographs are EXCELLENT to demonstrate the following in a YOUNG PATIENT (CHILD): • Supernumerary teeth, congenitally missing teeth, and axial inclinations of teeth.

• Apical development of permanent teeth and impacted teeth • View pathologic jaw lesions and mandibular condyles. The nice thing about panoramic x-rays is they are taken without placing the film in the mouth so it does not alarm a nervous child. Rather, children are often “entertained” by the panoramic unit. A disadvantage of a panorex is the loss of image detail (it is hard to diagnose early carious lesions). Thus, bitewing x-rays are required to diagnose carious lesions. For a clinically caries-free child, the first bite-wing x-ray should be taken when the spaces between the posterior teeth have closed. Inter-proximal caries on primary teeth may result in eventual loss of the primary tooth, loss of tooth structure, and arch length loss. Decay in primary teeth must be treated the same as in permanent teeth. To just “watch” this decay can cause loss of tooth structure, and eventual loss of the primary tooth with resultant loss in arch length. • Caries is an infectious disease that must be eliminated by removing the caries and restoring or extracting the tooth. To ensure arch integrity, restoring the involved tooth, if possible is preferred.

• Amalgam is still used to some degree to restore primary teeth. The most important modification in its use is in the cavity preparation. Properly contoured restorations are important to maintain proximal contacts and length of the dental arch. • Composite resin & resin-modified glass ionomers are also commonly used to restore primary teeth. • Occlusal anatomy of primary teeth is not as defined as permanent teeth anatomy, thus amalgam preparations can be more conservative. • Enamel and dentin are thinner in primary teeth, thus amalgam preparations are deeper. The thickness of coronal dentin in primary teeth is ½ that of permanent teeth. Pulpal horns of primary teeth are longer and pointed, thus amalgam preps must be conservative to avoid pulpal exposure. • Primary molars have an exaggerated cervical bulge that makes the matrix band adaptation much more difficult. Primary molars have an exaggerated cervical constriction which requires special care in the formation of the gingival floor in Class II preparations. • Enamel rods in the gingival third of primary teeth extend occlusally from the DEJ. This eliminates the need in Class II preparations for the gingival bevel

which is always required when preparing Class II preparations on permanent teeth. When placing a Class II amalgam in a primary tooth, with isthmus width should be 1/3 of the intercuspal width. If an amalgam fracture occurs, it is most likely to occur here. Other principles in preparing cavities in primary teeth: • Class I & II preparations should include areas with caries and that retain plaque, and potential carious areas (pits and fissures). This “extension for prevention” is only done when restoring with amalgam. It is not necessary to “extend for prevention” when restoring with composite resin or resin-modified glass ionomer, it is possible to seal the remaining pits and fissures. • Flat pulpal floor. • Beveled axio-pulpal line angle to help reduce stress in the amalgam and provide greater bulk of material in this area. • Rounded angles throughout the preparation to result in less concentration of stresses and allows more complete condensation of amalgam into extremities of the preparation. • In Class II preparations, the facial and lingual walls of

the proximal box should be carried to self-cleansing areas and should be parallel to the external surfaces and converge slightly. • The gingival margin is not beveled in Class II preparations because enamel rods in this area incline occlusally. • In Class II preparations, the gingival floor is not ideal in most cases as the preparation gets deeper in this area due to the cervical constriction found in this area on primary molars. Stainless Steel Crowns-are considered superior to large multi-surface amalgam restorations, with a longer clinical lifespan. Two common stainless steel crowns are pretrimmed & pre-contoured crowns. • Primary Teeth Indications: extensive carious lesions, hypocalcified teeth, teeth with dentinogenesis or amelogenesis imperfecta, restoration after pulpotomy or pulpectomy, on an abutment tooth for a crown and loop space maintainer, or temporary restoration of a fractured tooth. • Tooth Preparation: ♦ When preparing a primary tooth for a stainless steel crown, the cusps are reduced 1.0-1.5mm to allow clearance with opposing teeth and to prevent traumatic occlusion after placing the

stainless steel crown. ♦ Reduce proximal surfaces (mesial & distal); vertical reductions are made and carried gingivally to the extent that contact with the adjacent tooth is broken. ♦ Remove all sharp line angles. It is usually not necessary to reduce the buccal or lingual surfaces. This aids in crown retention (undercut area). However, it may be necessary to reduce the distinct buccal bulge, especially on the primary 1st molar. ♦ When festooning and trimming the crown during fitting, greater length is needed in the mesiofacial bulge region on a primary 1st molar. When operative or surgical procedures are performed on the mandibular primary or permanent teeth, the INFERIOR ALVEOLAR NERVE must be blocked by administering the conventional mandibular block. The supraperiosteal injection technique (local infiltration) is sometimes helpful in anesthetizing mandibular primary incisors, but cannot be relied on for complete anesthesia of mandibular primary or permanent molars. • Local infiltration can be used to anesthetize maxillary primary teeth. Adequate diffusion of the local anesthetic readily occurs in children because their bones are less dense than in adults. • The mandibular foramen is located at a level lower

than the occlusal plane of the primary teeth in the child patient. Thus, the injection is made slightly lower and more posterior than for an adult. • Young children do not always understand what “numb lip” means when you ask them this after a mandibular block. The best indicator of a profound block is to probe the labial attached gingiva between the lateral incisor and canine with an explorer. If this is done without a reaction, the child is anesthetized.

BEHAVIOR MANAGEMENT A dentist has offered plaque-control instructions to a patient. To ensure the patient understands the instructions, the dentist should ask the patient to verbalize her understanding. Between boys & girls, the difference is onset of pubertal growth spurt is 2 years. Girls reach puberty 2 years before boys. The MOST personal behavior by the dentist is touching the patient gently on the arm. Rubber Dam: a main advantage of using a rubber dam is it aids in child management. It seems to quiet and calm the patient because it acts as a separation/barrier both physically and psychologically between the patient and dentist. Thus, the rubber dam works for a very nervous or anxious patient. Other Rubber Dam Advantages: Better access and visualization. Controls saliva and moisture in the operating field. Decreased operating time. Provides protection from aspiration or swallowing of

foreign bodies. Child becomes primarily a nasal breather when the dam is in place, enhancing the effects of nitrous oxide if applicable. Contraindications: presence of fixed orthodontic appliances, a patient with congested nasal passages/nasal obstruction, or a recently erupted tooth that will not retain a clamp. Management of a child who must undergo dental extractions is based on these factors: • Child’s age & maturity. This often determines the type of anesthesia best suited for the intended procedure. Children below the age of reason are best managed under general anesthesia, since a slight amount of discomfort is always associated with the administration of a local anesthetic. It is very important to have total anesthesia before starting the procedure. Use both buccal & palatal infiltration on maxillary teeth, and block anesthesia on mandibular teeth with infiltration. o A very young child is best managed under general anesthesia (inhalation or in combination with small doses of IV barbiturates). VERSED is the most common premedication prior to general anesthesia. o Premedication with a barbiturate may cause paradoxical excitement in a young child.

• Past medical and dental experiences that might influence the child’s behavior. • Child’s physical status. • Length of time and amount of manipulation necessary to accomplish the surgery. Post-anesthetic lip biting is a common post-treatment complication in children. After extracting a tooth on a child, the biggest post-operative concern is preventing lip biting. TELL-SHOW-DO-the most important technique of behavioral management in the pediatric dental patient used to manage an extremely apprehensive 5-year old child. Tell-show-do is the backbone of the educational phase of developing an accepting, relaxed child patient. ► In this technique, you tell the child what is going to happen, you show the child what is going to happen, then perform the actual procedure in the mouth. ► Tell-show-do works especially well when treating a child with a different cultural background. ► The clinical examination of the infant and toddler should be accomplished with the parent’s assistance in a non-threatening environment. Most often, it is not necessary nor recommended that the dental chair be used. The parent and dentist sit facing each other in a

knee-to-knee position, supporting the child with the head cradled on the dentist’s lap. ► Aggressive behavior in the dental office is usually a fear reaction. The most realistic approach to managing a difficult child in the dental office is to attempt to recondition the child through techniques of applied psychology. When treating a child who is obviously afraid, the dentist should permit the child to express his fear (identify the fear). All behavioral patterns are motivated by anger and fear. The crying child is NOT an abnormal child. Anger is easier to treat than fear. Fear is most likely exhibited by a young child on his first visit to the dentist, and is related to anxiety over being separated from a parent. The PARENT (not the dentist), has the greatest influence on the child’s reaction at this initial visit. Child Control Techniques: ► Angry Child: Separate the parent and child, and place the child in the chair abruptly and be firm. Use “hand-over-mouth” technique (HOME) after getting the parent’s permission! Display authority and command the child’s respect by continuing with treatment even if he/she is uncooperative. Comfort the parent at the end of the visit, and compliment the child at the end of the visit. ► Fearful Child:

o Have the parent stand quietly behind the chair. o Dentist must be consistent in tonal quality. o Allow the child to express his fears (identify the fear). o Change the child’s focus off fear. o Lastly, sedation. Shy, submissive children are often the product of parents who are OVERPROTECTIVE. INTELLECTUAL DISABILITY (MENTALLY CHALLENGED): limited intellectual functioning (IQ < 70) affecting a person’s ability to learn, reason, make decisions, and problem-solve. Adaptive behaviors like the ability to communicate and interact effectively with others, and selfcare are limited. • Oral Health Concerns: higher incidence of gingivitis, periodontitis, and caries due to personal neglect/noncompliant. Bruxing, mouth-breathing, open-bite from tongue thrusting. • Dental Management: speak simply and reassuringly, “tell-show-do”, positive reinforcement. Relative contraindication for use of nitrous-oxide/oxygen sedation. Keep appointments short, and schedule the patient early in the day, since staff, dentist, and patient are less fatigued earlier. Speak slowly and in very simple terms and listen carefully to the patient. Ask the patient if there are any questions about

anything you will be doing. • FETAL ALCOHOL SYNDROME increases the genetic risk of intellectual disability. • CYTOMEGALIC INCLUSION DISEASE is the most common VIRUS that can cause intellectual disability. • Give a tour to the patient before attempting any treatment. Introduce the patient to the office personnel. • Give only one instruction at a time. Reward the patient with compliments after the procedure is successfully completed. • Dentist should assess the degree of developmental disability by consulting with the patient’s physician before starting dental treatment.

ORTHODONTICS PRIMARY TEETH OCCLUSION: ♦ Flush Terminal Plane-the NORMAL relationship of the primary molars in the deciduous (primary) dentition. The terminal plane relationship of primary second molars determines the future antero-posterior position of the permanent first molars. ♦ Mesial Step-the primary dentition’s equivalent to an Angle Class I malocclusion. ♦ Distal Step-the primary dentition’s equivalent to an Angle Class II malocclusion (retruded mandible). ♦ An equivalent of Angle’s Class III is almost NEVER SEEN in the primary dentition because of the normal pattern of craniofacial growth where the mandible lags behind the maxilla. The EDGE-TO-EDGE position of permanent maxillary & mandibular first molar cusps is the MOST COMMON initial relationship (when primary molars are in a flush terminal plane). This will most likely become a Class I molar relationship by both molars drifting forward (“early mesial shift”), with the mandibular molar drifting two times

farther than the maxillary molar. While erupting, the permanent teeth move OCCLUSALLY & BUCALLY. Also, during active tooth eruption, there is apposition of bone on all surfaces of the alveolar crest and on the bony socket walls. o Important: the maxillary arch is slightly longer than the mandibular arch. The sum of the M-D diameter of the maxillary permanent teeth is ~128mm, and is 126mm for the mandibular permanent teeth. OVERBITE- VERTICAL overlapping of maxillary anterior teeth over the mandibular anterior teeth. OVERJET-HORIZONTAL projection of maxillary anterior teeth beyond the mandibular anterior teeth. Labial-axial inclination of the maxillary incisors. Reverse Overjetassociated with Class III skeletal patterns with more than 2 maxillary anterior teeth in linguoversion. Physiologic occlusion-while it is not necessarily an ideal Class I occlusion, it is an occlusion that adapts to the stress of function, and can be maintained indefinitely. Physiologic tooth movement-an example is mesial drifting of a permanent molar into a space created by the premature loss of a primary molar. Important: permanent molars have a natural tendency to drift MESIALLY.

Pathologic occlusion-occlusion that cannot function without contributing to its own destruction, and may manifest itself by any combination of excessive tooth wear without sufficient compensatory mechanisms, TMJ problems, pulpal changes ranging from pulpitis to necrosis, and periodontal changes. Pathologic tooth movement-tooth movement caused by pathologic conditions. MALOCCLUSION-malocclusion is MOST OFTEN HEREDITARY. There may be a disproportion between the size of the maxilla & mandible, or between the jaws and tooth size resulting in overcrowding of teeth or in abnormal bite patterns. Supernumerary teeth, malformed teeth, impacted or lost teeth that erupt in an abnormal direction may cause malocclusion. Less frequent causes of malocclusion are thumb sucking or tongue thrusting habits. • Incipient Malocclusion Signs: o Lack of interdental spacing in the primary dentition. The significance of the lack of spacing related to the increased M-D width of the permanent teeth. o Crowding of permanent incisors in the mixed dentition. Since arch perimeter increases after the incisors erupt and is small in the maxilla and essentially non-existent in the mandible, arch growth cannot usually contribute to further dental

alignment. o Premature loss of primary canines (especially in the mandibular arch). Premature loss of the mandibular primary canine reflects insufficient arch size in the anterior region. As such, the crowns of lateral incisors during eruption impinge on the primary canines’ roots and cause them to resorb. When the canine is shed, the midline will shift in the direction of the lost tooth, and there is lateral and lingual migration of the mandibular incisors. • 3 Planes of Space To Classify Malocclusion: Antero-posterior, Transverse, & Vertical. • The incidence of malocclusion in a HOMOGENOUS population is generally LOWER than in a HETEROGENOUS population (where the incidence is higher). Dental arch form is ultimately determined by the interaction of environmental influences on the genetic pattern. SEVERE malocclusion may compromise all aspects of oral function (speech, swallowing, and mastication). There may be difficulty in masticating if only a few teeth meet, and jaw discrepancies may force adaptive alterations in swallowing. It can be difficult or impossible to produce certain sounds, and speech therapy may require some preliminary orthodontic treatment. Referral to a speech therapist is helpful as both

patient and parents can benefit. Even less severe malocclusions can affect mastication, swallowing and speech, not so much by making function impossible, but by requiring physiologic compensation for the anatomic deformity. • Patients with anterior open-bite may have difficulty making speech sounds “th, sh, ch” due to distortion. An anterior open-bite can also create a lisp, making it difficult to pronounce “s” & “z” sounds. • A large diastema between maxillary incisors can create a lisp, making it difficult to pronounce “s” & “z”. • Irregular incisors (especially lingual position of maxillary incisors), creates a difficulty to produce sounds “t” & “d”. • Due to distortion, there may be difficulty in making speech sounds “F” & “V” in a Skeletal Class III. SKELETAL OPEN BITE (“Long Face Syndrome”)-a type of malocclusion that is most often associated with MOUTH BREATHING. Factors associated with chronic mouth breathing include narrow face & oropharyngeal space, chronic rhinitis (inflammation of the mucous membranes of the nose), chronic tonsillitis, allergies, and a deviated nasal septum. o The earliest possible diagnosis of a skeletal open-bite

is essential because the condition is not selfcorrecting, and usually worsens with time. Anterior open bites can be classified as a form of apertognathism (an open-bite deformity). AS A CHILD MATURES, THEIR FACIAL PROFILES BECOMES LESS CONVEX. There is no set rule as to when a malocclusion should be treated. The age of treatment depends on the problem. Malocclusions are more identifiable in children ages 7-9 because the eruption of permanent incisors reveals tooth-arch length discrepancies. Steiner Analysis: 1. SNA angle-the angle formed at the intersection of 2 lines (one line from sella turcica (S) to nasion (N), and the other line from nasion to Point A). These two lines show the maxilla’s position relative to the cranial base. Steiner indicates in a good skeletal pattern, the SNA angle is ~82°. • SNA > 82° = maxillary prognathism. • SNA < 82° = maxillary retrognathism. 2. SNB angle-the angle formed by the intersection of line SN & NB. It defines the sagittal location of the mandibular denture base. Steiner states an 80° angle is compatible with skeletal harmony. • SNB angle > 80° = mandibular prognathism. • SNB angle < 80° = mandibular retrognathism.

3. ANB angle-the norm for this critical angle is 2° (it is the difference between SNA & SNB norms). A Class I skeletal profile has a 2° ANB angle. • ANB angle > 4° = Class II skeletal profile. • ANB angle < 0° (negative angle) = Class III skeletal profile. ANGLE MALOCCLUSION CLASSIFICATION (Permanent Molar Relationships)-the MESIOBUCCAL CUSP of the maxillary first molar serves as the REFERENCE POINT in identifying Angle’s Class I, II, & III occlusions. 1. Class I (A) Malocclusion: MB cusp of the maxillary first permanent molar occludes with (lines up with) the buccal groove of the mandibular first permanent molar. Also, maxillary centrals overlap mandibular centrals. THE MOST COMMON OCCLUSION (~70% of the population) and is associated with an ORTHOGNATHIC FACIAL PROFILE where the nose, lips, & chin are harmoniously related. • Class I (A) canine relation: maxillary permanent canine occludes with the distal half of the mandibular canine & mesial half of the mandibular first premolar (maxillary canine lies between the mandibular canine & first premolar). • Cephalometric analysis of Class I malocclusions indicates an ANB angle < 4° (indicates a harmonious skeletal profile and sagittal harmony between the maxillary and mandibular dental arches).

• The MOST COMMON CAUSE of Class I malocclusion is a discrepancy between tooth structure and amount of supporting bone length. • The MOST PREVALENET characteristic of Class I malocclusion is CROWDING due to insufficient alveolar arch length to accommodate all teeth in ideal alignment and in a good sagittal position. When a crowding diagnosis is made (crowding > 4mm in the mandible), extractions are often required to attain an excellent, stable result. However, the decision to extract teeth depends greatly on performing a mandibular space analysis which is usually performed (referral) to the orthodontist. • When the space lacking is < 4mm, in most cases, it is obtained by carefully stripping some interproximal enamel from each anterior tooth. A space deficiency > 4mm indicates extraction to correct the malocclusion. 2. Class II (B) Malocclusion (Retrognathism = Overbite): mandibular first permanent molar’s buccal groove is DISTAL to the MB cusp of the maxillary first permanent molar (the mandible is “distal” to the maxilla). The MB cups of the maxillary first molar is between the mandibular first molar & second premolar. The lower jaw and chin may appear small and withdrawn. The mandibular incisors occlude even more

posterior to the maxillary incisors so that they may not touch at all. Associated with a RETROGNATHIC FACIAL PROFILE where the convexity is due to the relative prominence of the maxilla compared to the mandible. The mandibular incisors will most likely be tipped forward. This is less common (~25% of population). An SNB angle < 78° indicates mandibular retrognathism, while an SNA angle < 82° indicates maxillary retrognathism. The big difference between Division I & II is in a Division II, the maxillary laterals are tipped labially & mesially. • Class II, Division I: a distal relationship of the mandibular first permanent molar’s buccal groove to the MB cusp of the maxillary first permanent molar & maxillary incisors (maxillary central & lateral incisors are in extreme labioversion (protruded). o In most Class II, Division I malocclusions, the body of the mandible and its superimposed dental arch are in a DISTAL relationship to the maxilla, and the maxillary incisors are usually in a labial axial inclination. Also, the relationship of the maxillary first molars and canines to their mandibular counterparts is such that the DB cusp of the maxillary first molar occludes in the buccal developmental groove of the mandibular first molar, and the maxillary canines occlude mesial to the

mandibular canines. Besides the labial-axial inclination of the maxillary incisors (overjet), various aberrations in the individual alignment of the teeth (i.e. crowding) can be superimposed on this class. • Class II, Division II: a distal relationship of the buccal groove of the mandibular first permanent molar to the MB cusp of the maxillary first permanent molar and maxillary laterals being tipped labially and mesially (sometimes overlapping the centrals). Its a malocclusion where the body of the mandible and its superimposed dental arch are also in distal relationship to the maxilla, while the molar & canine occlusion are the same as Class II Division I. The DB cusp of the maxillary first molar occludes in the buccal developmental groove of the mandibular first molar, and maxillary canines occlude mesial to mandibular canines. The maxillary centrals are near normal antero-posteriorly or slightly in linguoversion (usually retruded), and maxillary lateral incisors are usually labiomesially flared and overlap the central incisors. An impinging overbite may also exist. • When the distocclusion occurs on ONLY ONE SIDE OF THE ARCH, this UNILATERALITY is called a “subdivision” of its division (Class II, Division I Subdivision where ONE SIDE of the

maxillary arch is in a Class II relationship with its occluding mandibular quadrant, while the other side is in a Class I relationship). The protruded maxillary incisors (centrals & laterals) and maxillary overjet and other anterior aberrations are usually confined to one side of the maxillary arch. • “Sunday Bite”-a term given to the forward postural position of the mandible adopted by people with Class II profiles in an effort to improve their esthetics. • Class II (B) canine relation: mandibular canine’s distal surface is DISTAL to maxillary canine’s mesial surface. The maxillary canine is mesial to the mandibular canine. ECTOPIC ERUPTION -occurs when a tooth erupts in the WRONG PLACE (most commonly occurs in the eruption of maxillary first molars & mandibular incisors), and is much more common in the maxilla, and often associated with a developing skeletal Class II malocclusion. • Ectopic eruption occurs in 2-6% of the population and spontaneously corrects itself in ~60% of cases. • Ectopic eruption of a permanent maxillary first molar is often treated by a BRASS WIRE separating device placed between the primary second molar & permanent first molar to cause the permanent

first molar to tip distally. • If the eruption path of the maxillary first molar carries far too mesially at an early stage, the permanent molar cannot erupt and the primary molar root can be damaged. The mesial position of the permanent molar means the arch will be crowded unless the child receives treatment. Important: this mesially inclined position of the permanent molar makes it susceptible to decay. If it shows signs of caries, extract the adjacent primary second molar immediately. The resultant space can then be maintained as part of orthodontic treatment. • Ectopic eruption of mandibular lateral incisors (which occurs more commonly than mandibular 1st molars), can cause transposition of the lateral incisor & canine. A poor eruption direction of the canine, sometimes leading to impaction is often observed, but is usually due to the eruption path being altered by a lack of space. 3. Class III (C) Malocclusion (Mandibular Prognathism = Underbite): occurs when the body of the mandible and its superimposed dental arch are in a MESIAL relationship to the skull base & maxilla. The maxillary first molar occludes distal to the mandibular first molar, while the maxillary canine is an exaggerated distal relationship to the mandibular canine. mandibular first

permanent molar’s buccal groove is MESIAL to the MB cusp of the maxillary first permanent molar (MB cusp is between the mandibular first & second molars). The chin may protrude like a bulldog. Mandibular incisors overlap anterior to the maxillary incisors (mandibular incisors are forward to the maxillary incisors). Maxillary incisors are usually tipped lingually. LEAST COMMON MALOCCLUSION (< 5%). Associated with a PROGNATHIC FACIAL PROFILE (MANDIBLE) where the mandible is markedly forward past the maxilla giving a concave midfacial appearance (the mandible protrudes forward, and the mandibular teeth extend over the maxillary teeth). • Class III subdivision: a Class III relationship of the teeth on one side, with a Class I relationship on the other side. • On a cephalometric analysis, all Class III malocclusions have an SNA angle > 84° (maxillary prognathism). • Due to distortion, there may be difficulty in making speech sounds “F” & “V” in a Skeletal Class III. • Reverse Overjet-associated with Class III skeletal patterns with more than 2 maxillary anterior teeth in linguoversion. Pseudo-Class III malocclusion: malocclusion where the

mandibular incisors are forward of the maxillary incisors in centric occlusion, but the patient can bring the mandible back without strain so that the mandibular incisors can touch the maxillary incisors (this ability is often considered diagnostic). Thus, this type is a milder form of the “true” Class III malocclusion, and is more amenable to conservative orthodontic movement, while a “true” Class III malocclusion often requires surgical correction. o The existence of a FORWARD SHIFT of the mandible during closure to avoid incisor interference is found in “Pseudo” Class III malocclusions. To avoid teeth interference, the patient can adopt a jaw position on closure that is forward to normal (this may look like a Class III position in the absence of a “true” skeletal Class III relationship. Thus, it is called a “pseudo” Class III malocclusion. In most cases they have an edge-toedge bite. o Treatment: involves eliminating the CO-CR discrepancy early in the treatment to avoid abnormal wear and abnormal growth influences. Occlusal interferences and anterior cross bites are treated subsequently. o Bimaxillary Dentoalveolar Protrusion-present when the teeth protrude in both jaws. In this condition you will see severe dental and lip protrusion

accompanied by severe lip strain which is needed to bring the lips into closure. o Class III (C) canine relation: mandibular canine’s distal surface is MESIAL to maxillary canine’s mesial surface. The maxillary canine is DISTAL to the mandibular canine. CROSSBITES-occur when some of the teeth move on the “wrong side of the track”. Crossbite can be unilateral (on one side), or bilateral (on both sides), and can occur anteriorly or posteriorly. Crossbites are associated with a jaw-size discrepancy, hereditary (genetics), reverse over-jet, and a scissor bite. Crossbites are NOT associated with tongue thrusting. • Anatomical (Skeletal) Crossbite-usually demonstrates a smooth closure into centric occlusion. • Functional Crossbite-usually caused by thumb sucking does not demonstrate a smooth closure into CO. Orthodontic treatment to correct a crossbite in children should start as early as possible. Maxillary expansion-is the first step of treatment to broaden the maxilla with an “expander” appliance. This palatal expander is fixed to the roof of the mouth and is widened each night for 1-2 months with a turn key, and remains in the mouth for ~3 additional months to allow the bone to harden in its new position.

Braces may also be placed on the maxillary teeth during the palatal expansion to eventually close the “gap-tooth grin” that develops as the maxilla is expanding. Once expansion is complete, the child may need to wear a full set of braces for 1-2 years to achieve an ideal occlusion. Scissor Bite (Bilateral Lingual Crossbite)-results from a narrow mandible or wide maxilla. Milder cases may involve only maxillary first premolars. Severe cases may require contraction of the maxilla or expansion of the mandible. ANTERIOR CROSSBITE in the primary dentition often indicates a skeletal growth problem & a developing Class III malocclusion. Anterior crossbite can be caused by a labially situated supernumerary tooth, traumatic injury, or an arch length discrepancy. • However, anterior crossbite of one or more permanent incisors may suggest a localized discrepancy and a condition that should almost always be treated in the mixed dentition state, or as soon as it is identified. It is NOT a self-correcting condition. • It is most often associated with prolonged retention of a primary tooth. Delayed treatment can lead to serious complications (i.e. loss of arch length). The most essential factor related to correcting an anterior crossbite is the amount of M-D space available. It is easily retained once it is

corrected. • Premature exfoliation of a primary canine may indicate an arch length deficiency. Premature loss of a primary mandibular canine may cause a lingual collapse of the mandibular anterior teeth. • A corrected anterior crossbite is BEST RETAINED by the normal incisor relationship that is achieved from the treatment (the overbite), not from appliances. • CLASSIC SYMPTOMS of a digit sucking habit: anterior crossbite, crossbite, proclination of maxillary incisors, constriction of the maxillary arch (not expansion) retroclination of mandibular incisors, and a Class II malocclusion. OPEN BITE-a malocclusion (abnormal bite) where some teeth (usually the front teeth) cannot be brought into contact with their opposing teeth. Tongue thrusting DOES NOT cause crossbite or an open bite. A tongue thrust swallow is the result of displaced incisors (not the cause). Recent studies show that “tongue thrust swallowing” does not cause open bite as there is no tongue force on the teeth during swallowing even though the tip of the tongue is placed forward. The tendency to place the tongue forward originates from the need to attain an oral seal. By placing the tongue between the teeth (in cases of anterior open bite) an oral seal is

formed during swallowing. Thus, the forward position of the tongue during swallowing is due to the anterior open bite, and the reverse is not true “a tongue thrust swallow thus is considered the result of displaced incisors, not the cause”. ANTERIOR OPEN BITE-is the most common sequelae of a digit sucking habit. Unilateral crossbites can also occur. Increased pressure from the buccinator muscles during sucking constricts the maxillary arch. Other mechanical forces cause the maxillary incisors to procline & mandibular incisors to retrocline. As the hand rests on the chin, it retards mandibular growth, causing a Class II profile. • Most of the time, anterior open bite is ASYMMETRICAL with normal posterior occlusion. • Anterior open bites are much more common in African Americans than Caucasians. • Deep bites are much more common in Caucasians. POSTERIOR CROSSBITE after prolonged thumb sucking & ANTERIOR CROSSBITE in mildly prognathic children are usually seen with displaced teeth related to functional shifts. • Prolonged sucking habits often produce a mildly narrow maxillary arch and a tendency toward bilateral crossbite. Children with this condition usually shift the mandible to one side on closure to gain better function, which can guide permanent molars, or later guide premolars into a crossbite relationship.

• A young child with a tendency toward a Class III malocclusion will have end-to-end contact of the primary incisors. A true anterior crossbite in the primary dentition is rare because mandibular growth LAGS BEHIND maxillary growth. The primary incisors wear down rapidly, and an anterior shift of the mandible to escape occlusal interferences rarely occurs until the permanent incisors begin to erupt. A pattern of anterior displacement of the mandible may develop when the permanent incisors come into contact, however, producing an anterior crossbite from the shift. POSTERIOR CROSSBITE in the MIXED DENTITION should be corrected ASAP and be thoroughly diagnosed as either a dental, functional, or skeletal crossbite and may be associated with a mandibular shift. • Posterior crossbite may be corrected with a palatal expansion which causes diastema formation between the central incisors and expands the nasal floor. Tooth movement and skeletal expansion are inevitable when the midpalatal suture is widened with a palatal expander. • It is important to correct posterior crossbites (which are related to the TRANSVERSE PLANE OF SPACE) and mild anterior crossbites in the FIRST STAGE of treatment, even if the permanent 1st molars have not erupted. Severe anterior crossbites

are usually not corrected until the SECOND STAGE of conventional treatment. • THE MOST COMMON ACTIVE TOOTH MOVEMENT in the primary dentition is to CORRECT A POSTEIROR CROSSBITE (a transverse plane of space problem). Maxillary mandibular plane angle (MMPA)-the angle between the mandibular plane (Go-Me line) & maxillary plane (ANS-PNS line) whose normal value is 27° (+ 4°). The greater MMPA, the longer the anterior facial height. • There is also an interaction between face height & antero-posterior position of the mandible. All other factors being equal, a long face predisposes the patient to Class II malocclusion, while a short face predisposes the patient to a Class III malocclusion. The mandibular plane angle is visualized clinically by placing a mirror handle or other instrument along the border of the mandible. • A STEEP mandibular plane angle correlates with LONG anterior facial vertical dimensions & anterior open bite malocclusion. A long face predisposes the patient to Class II malocclusion. • A FLAT mandibular plane angle correlates with SHORT anterior facial vertical dimensions (height) & anterior deep bite malocclusion. A short face

predisposes the patient to a Class III malocclusion. Poor Man’s Cephalometric Analysis-a facial profile analysis that delinates the same information as that obtained from lateral cephalometric radiographs. The difference is there is greater detail obtained from the lateral cephalometric. However, the Poor Man’s Cephalometric Analysis is a vital diagnostic technique for primary evaluation as it is a quick, simple, and inexpensive technique that readily gives the following information: 1. anteroposterior position/proportion of the jaws relative to each other. 2. lip posture (competent/incompetent) and incisor prominence. 3. vertical facial proportions. 4. inclination of the mandibular plane angle. *Within the lower 1/3 of anterior face height, the mouth should be ~1/3 of the way between the nose & chin. CEPHALOMETRICS in Orthodontics are used for diagnosis, analysis of treatment results, and longitudinal study of growth. Cephalometrics is useful to assess tooth-to-tooth, bone-to-bone, & tooth-to-bone relationships. Serial cephalometric film can show the amount and direction of growth. • The lateral head radiograph (cephalometric radiograph) must be compared with the “normal” lateral radiographs to form an accepted norm. Linear

& angular measurements are obtained using known anatomical landmarks in the lateral head radiography of the patient. These measurements are then compared with measurements considered within normal limits and in that way enable the orthodontist to assess aberration in the dentition and jaw structures that result in malocclusion. • Analysis of cephalometric radiographs is NOT limited to hard structures (i.e. bone & teeth), but include measurements of soft tissue structures (i.e. nose, lips, soft tissue chin). • Superimposition in longitudinal cephalometric analysis is on a reference plane and registration point. This best demonstrates the growth of structures furthest from the plane and the point. The most stable area to evaluate craniofacial growth is the anterior cranial base because of its early cessation of growth. • A lateral cephalograph usually shows magnification with up to 7-8% magnification which is considered acceptable. The resulting double shadows are traced, and the average is used for measurements. CEPHALOMETRIC LANDMARKS: 1. Bolten (Bo)-the highest point in the upward curvature of the retrocondylar fossa of the occipital bone.

2. Baison (Ba)-the lowest point on the anterior margin of the foramen magnum, at the base of the clivus. 3. Articulare (Ar)-the intersection of 3 radiographic shadows, inferior surface of the cranial base, & posterior surfaces of the necks of the mandibular condyles. 4. Porion (Po)-the midpoint of the upper contour of the metal ear rod of the cephalometer. 5. Sphenooccipital synchondrosis (SO)-the junction between the occipital & basisphenoid bones. 6. Sella (S)-the midpoint of the cavity of the sella turcica. 7. Pterygomaxillary fissure (Ptm)-the point at the base of the fissure where the anterior & posterior walls meet. 8. Orbitale (Or)-the lowest point on the inferior margin of the orbit (floor of orbit). 9. Anterior nasal spine (ANS)-the tip of the anterior nasal spine. The point above the root the maxillary central. 10. Point A (Subspinale)-innermost point on contour of the mandible between the incisor and bony chin. 11. Point B (Supramentale)-innermost point on contour of the mandible between the incisor and bony chin. 12. Pogonion (Pog)-the most anterior point of the chin’s contour (on the mandibular symphysis). 13. Menton (Me)-the most inferior point on the mandibular symphysis (the bottom of the chin). 14. Gonion (Go)-the lowest, most posterior point on the angle of the mandible with the teeth in occlusion. 15. Nasion (Na)-anterior point of the intersection between the nasal and frontal bones.

FRANKFORT-HORIZONTAL PLANE-constructed by drawing a line connecting PORION & ORBITALE. FHP is the best representation of the NATURAL ORIENTATION OF THE SKULL.

TO PREDICT THE TIME OF PUBERTAL GROWTH SPURT while treating jaw mal-relationships in a growing child, the orthodontist can get the most valuable information from a WRIST-HAND RADIOGRAPH. The physiologic age (developmental age) is judged by determining the skeletal development. Wristhand radiographs offers the best aid to do this by examining the ossification & development of the wrist’s carpal bones, hand’s metacarpals, and finger’s phalanges to give the orthodontist an idea regarding the chronology of skeletal development. Comparing the overall pattern observed in the hand-wrist radiograph, with age standards in a reference atlas, does this. • ulnar seaside or hamate bones are landmarks to obtain an estimate of the timing of the adolescent growth spurt. Wrist-hand radiographs in the dental office are obtained using a standard cephalometric cassette and dental radiograph. • The state of physical maturity or skeletal development co-relates well with the jaw growth. Orthodontist use this information to predict the amount of expected jaw growth. After sexual maturity, much less growth is expected, thus growth modification is not attempted. • Hand-wrist radiographs are LESS useful in evaluating if growth has stopped or is continuing (patient’s position on the growth curve). Serial

Cephalometric radiographs are used for this purpose. MIXED DENTITION-the dentition phase when some of the teeth in the oral cavity are permanent, and some are primary. The earliest indication of a mixed dentition consists of the primary dentition & permanent mandibular first molars. Supervising a child’s development of occlusion is MOST CRITICAL from ages 7-10yrs (mixed dentition stage). ♦ Mixed Dentition Analysis (transitional dentition analysis)-an analysis performed during the mixed dentition to predict the amount of crowding AFTER the permanent teeth erupt. It determines the space available vs. space required. It is performed using a boley gauge, study models, and a prediction table. The analysis is based on a correlation of tooth size (one may measure a tooth or group of teeth and predict accurately the size of the other teeth in the same mouth. Performing a Mixed Dentition Analysis: 1. Measure the M-D diameter of the mandibular incisors and ADD them together. 2. Measure the space available for the mandibular incisors. 3. Subtract #1 from #2 (a negative number indicates CROWDING in the incisor region). 4. Measure the space available for the canine and premolars on each side of the arch.

5. Calculate from the prediction table the size of the canine and premolars. 6. Subtract #6 from #5 on each side (again, a negative number indicates CROWDING). 7. By this step, there are 3 numbers that are added: (a (-) number = crowding; a (+) number = space). 1. number for incisor crowding or excess space. 2. number for the right canine and premolar crowding or excess space. 3. number for the left canine and premolar crowding or excess space. For the MAXILLARY ARCH, mandibular incisors are used to predict the size of maxillary canines & premolars. Follow the same steps as for mandibular teeth. MOYER’S Mixed Dentition Analysis-the size of unerupted canines & premolars is predicted from knowing the size (M-D width) of the mandibular incisors that have already erupted into the mouth early in the mixed dentition. Maxillary incisors are not used in any of the predictive procedures, since they show too much size variation. Mandibular incisors are measured to predict the size of maxillary and mandibular posterior teeth. If mandibular anterior crowding is noted during the mixed dentition phase, the most appropriate approach to management is to take study models and perform an arch length analysis. This mandibular incisor crowding usually

results from a tooth size-arch length discrepancy. PRIMATE SPACES-spaces found in the primary dentition. Spacing is normal throughout the anterior part of the primary dentition, but is MOST NOTICABLE in these two locations: ♦ Maxillary arch: primate space located between the lateral incisors & canines. ♦ Mandibular arch: primate space located between canines and 1st molars. Primate spaces are normally present from the time the primary teeth erupt. Developmental spaces between the incisors are often present from the beginning, but become larger as the child grows and as alveolar processes expand. Generalized spacing of the primary teeth is required for proper alignment of the permanent incisors. This spacing is most frequently caused by growth of the dental arches. If spacing is present, it is possible that DRIFTING of the adjacent teeth will occur if there is a loss of a primary incisor. However, if no spacing exists, and the primary anterior teeth were in contact before the lost incisor, a collapse in the arch after the loss of one of the primary incisors is almost certain. This is not true in the case of a lost permanent incisor as space closure occurs rapidly whether spacing is present or not prior to the primary tooth loss. Space maintenance would be indicated.

♦ Important: one of the MOST COMMON CAUSES OF MALOCCLUSION IS INADEQUATE SPACE MANAGEMENT AFTER EARLY LOSS OF PRIMARY TEETH. Relative to the primary mandibular canines, the permanent mandibular canines ERUPT IN A FACIAL (LABIAL) direction or are often right in line with the primary canines. If there are eruption problems, these teeth can be displaced either lingually or labially, but they are usually displaced LABIALLY if there is not enough room to accommodate them within the arch. The mesial inclined plane of the primary maxillary canine articulates with the distal inclined plane of the primary mandibular canine. This is the normal relationship. In both maxillary and mandibular arches, the permanent incisor tooth buds lie LINGUAL & APICAL (inferior) to the primary incisors. This results in a tendency for mandibular permanent incisors to erupt somewhat lingually, and in a slightly irregular position (this occurs in children who have normal dental arches and normal spacing within the arches). Permanent teeth normally move OCCLUSALLY & BUCCALLY while erupting. ♦ The maxillary arch is slightly longer (~128mm) than the mandibular arch (~126mm). LEEWAY SPACE-the DIFFERENCE in the total of the M-

D widths between the primary canine, first molar, and second molar, AND permanent canine, first premolar, and second premolar. The permanent successors are usually SMALLER than their primary predecessors. ♦ Mandibular leeway space averages 3-4mm & the Maxillary leeway space averages 2-2.5mm. The important factor is that some space will be available in the posterior part of the mouth. This leeway space serves to at least accommodate the permanent canines (which are usually larger than primary canines). ♦ During the canine-premolar transition period, the permanent first molars generally move MESIALLY into the leeway space after the primary second molars are shed, thus causing a loss in arch length. This is called “the late mesial shift of a permanent first molar”. SERIAL EXTRACTION-a procedure that involves the orderly removal of selected PRIMARY & PERMANENT TEETH in a predetermined sequence. Serial extraction is primarily indicated in SEVERE CLASS I malocclusion in the mixed dentition that has insufficient arch length. It benefits children who have an arch-length discrepancy. Important: severe arch space deficiency in the permanent dentition (> 10mm) almost always requires extractions to properly align teeth.

Serial Extraction Stages: 1. Primary Canines are removed 1st. 2. Primary 1st Molars are removed 2nd 3. Permanent 1st Premolars are removed last (usually). 6-15 months is the interval between extractions. To aid in support and retention during serial extraction treatment, a LINGUAL ARCH is used in the mandible and HAWLEY APPLIANCE in the maxilla. This is usually followed by full orthodontic treatment. THE KEY TO SUCCESS IS TO EXTRACT THE 1st PREMOLARS BEFORE THE PERMANENT CANINES ERUPT. In serial extraction procedures, concerns about eruption sequence are usually related to the eruption pattern of the permanent mandibular canines & first premolars. After extracting the maxillary first premolar in a serial extraction procedure, the maxillary canines’ path of eruption will usually be downward and backward. MOST COMMON IMPACTED ANTERIOR TEETH are MAXILLARY CANINES. Failure of a permanent tooth to erupt may damage the roots of other teeth and create a severe orthodontic problem. Orthodontic consultation is indicated when first observed on a radiograph. An impacted canine or other tooth in a teenage patient can usually be brought into the arch by orthodontic traction after being surgically exposed. In older patients, there is an increasing risk that the impacted tooth has become

ankylosed. Even adolescents have a risk that surgical exposure of a tooth will cause ankylosis. 3 Principals when Treatment Planning an Impacted Tooth: 1. The prognosis is based on the extent of displacement and surgical trauma required for exposure. 2. During surgical exposure, flaps should be reflected so that the tooth is ultimately pulled into the arch through keratinized tissue (NOT through alveolar mucosa). 3. Adequate space should be provided in the arch BEFORE attempting to pull the impacted tooth into position. Research suggests the association of impacted canines with missing lateral incisors or shortened roots of lateral incisors. The distal aspect of the lateral incisors root guides the eruption of canines. SUPERNUMERARY TEETH-extra teeth that develop in excess of the normal complement of teeth that can occur in the maxilla or mandible, but are most common in the maxilla in the midline of the anterior teeth, and sometimes distal to the molars. Supernumerary teeth have a 2:1 predilection for MALES. • MOST COMMON SITE of a SUPERNUMERARY TOOTH IS BETWEEN MAXILLARY CENTRAL

INCISORS. • Mesiodens-an extra tooth that occurs between the maxillary central incisors. Mesiodens are usually small peg-shaped teeth (microdontia) that do not resemble the normal teeth of the site. An impacted mesiodens can cause a diastema between the maxillary central incisors, while an inverted mesiodens can cause delayed eruption of the maxillary central incisors. • Supernumerary teeth can cause crowding of the normal teeth and can delay permanent tooth eruption. • Diagnosis: to localize a supernumerary or impacted tooth and its relationship to other teeth, take two or more periapical radiographs at different angles, and an occlusal view film. • Treatment: surgically removing the supernumerary teeth and observing the progress of the permanent teeth. • Conditions associated with multiple supernumerary teeth: Gardener’s syndrome, Down’s syndrome, Cleidocranial dysplasia, & Sturge-Weber Syndrome. Oligodontia-absence of one or more teeth. More

common in females, and often associated with a smaller than average tooth-size ratio. SPACE MAINTENANCE: A 9 year old patient had an extraction of the primary mandibular first molar. The ideal treatment at this time is PLACE A SPACE MAINTAINER. Although this can be done with either fixed or removable appliances, fixed appliances are preferred in most situations because they eliminate the factor of patient cooperation. If the space is unilateral, it can be managed by a unilateral fixed appliance (“band & loop” space maintainer). If molars on both sides have been lost, and the permanent incisors have erupted, it is usually better to place a lingual arch space maintainer. An 8 year old child with a PULPALLY INVOLVED primary second molar comes into your office The IDEAL and best approach to manage this case is to TREAT THE PULP and RETAIN the tooth as a space maintainer BECAUSE there is no prefabricated space maintainer that is as good as the natural tooth. In this case, proper pulpal therapy followed by a restorative procedure is needed on this tooth to allow it to function as a space maintainer. ♦ The natural tooth will preserve ARCH LENGTH & INTEGRITY better than any prefabricated space maintainer.

♦ If a primary tooth is lost, an orthodontic evaluation is indicated to determine whether space maintenance is needed. This decision is based on the patient’s skeletal and dental development (i.e. if a 10 year old child loses the primary first molar, no treatment is usually needed, since the permanent first molar usually erupts between ages 10-12yrs). Generalized causes of Failed or Delayed Eruption: Hereditary Gingival Fibromatosis, Down’s syndrome, & Rickets. Generalized eruption failure or “primary failure of eruption” is caused by the failure of the eruption mechanism itself. The involved teeth do not erupt spontaneously and are not amenable to any orthodontic recourse. Fortunately, the condition is rare. • Hyperparathyroidism, however, causes premature exfoliation (loss) of primary teeth. Localized causes of Failed or Delayed Eruption: congenital absence, abnormal position of the crypt, lack of arch space (crowding), supernumerary teeth, and dilacerated roots. Effects of Environmental Influences during Growth & Development of the face, jaws, and teeth: • Patients who have excessive overbite or anterior open bite usually have posterior teeth that are infraerupted or supra-erupted respectively.

• A non-nutritive sucking habit causes malocclusion only if it continues during the mixed dentition stage. • Negative pressure created in the mouth during sucking is not a cause of maxillary arch constriction. • Adenoids that lead to mouth breathing, cannot be with certainty considered an etiologic agent of a long-face pattern of malocclusion as studies show most of the long-face population do not have nasal obstruction. 98% of 6-year olds, & 49% of 11-year old children have a MAXILLARY (MEDIAN) DIASTEMA caused by either a tooth-size discrepancy, mesiodens, abnormal frenum attachment, or a normal stage of development. • The diastema closes as the permanent canines erupt. The greater the spacing, the LESS likely a maxillary central diastema will completely close on its own. • As a general rule, a maxillary central diastema of 2mm or less will usually close spontaneously, while total closure of a diastema initially greater than 2mm is unlikely. If the space is 2mm or less, and maxillary laterals are in a good position, it is most likely the result of a normal developmental process. • If the diastema is caused by an abnormal frenum, it is best to align the teeth orthodontically and then do a frenectomy (this is usually not done until

the permanent canines erupt). • Methods to Close a Diastema: using a lingual arch with finger springs, a Hawley appliance with finger springs, or using cemented orthodontic bands with inter-tooth traction. 6 Types of Tooth Movement Accomplished with Orthodontics: 1. Tipping-the tooth crown moves in one direction, while the root tip/apex is displaced in the opposite direction due to rotation or pivoting of the tooth around the axis of resistance (axis of rotation) which is located somewhere in the apical 1/3 of the root. Tipping is best accomplished with a removable appliance and most easily with anterior incisors. 2. Translation (bodily movement)-a coupled force is applied to the crown to control root movement in the same direction as crown movement (force is applied through the tooth’s center of resistance). Translation is very difficult to accomplish. 3. Extrusion-displacement of the tooth from its socket in the direction of eruption. 4. Intrusion-tooth movement into the socket along the tooth’s long axis. Intrusion is very difficult to accomplish.

5. Torque-controlled root movement F-L or M-D while the crown is held relatively stable (M-D root movement is also called “uprighting”). 6. Rotation-revolving the tooth around its long axis. Recurring tooth rotations after orthodontic correction occur due to the persistence of the elastic supracrestal gingival fibers (mainly free gingival & transseptal fibers). Need adequate retention to prevent relapse. Thus, supracrestal fibers are commonly associated with relapse after orthodontic rotation of teeth. • An appropriate candidate for post-orthodontic circumferential supracrestal fibrotomy is a ROTATED maxillary lateral incisor. Circumferential supracrestal fibrotomy-a minor surgical procedure where a simple incision into the sulcus is made to the crest of bone to incise all of the collagen fibers that are inserted into the tooth root. Cutting the collagen fibers eliminates relapse potential due to collagen fiber retraction and allows new fibers to form to help retain the tooth in its new position. On the side TOWARD where the tooth is being moved, osteoclasts are breaking bone down, while on the side of the root from where the tooth moves osteoblasts that are remodeling/forming bone. One of the most important aspects of orthodontic therapy is RETENTION. After malposed teeth have been

moved into the desired position they must be mechanically supported until the hard & soft tissues have been thoroughly modified in structure and function to meet the new position’s demands. Once the desired occlusal results are achieved, and the hard tissues are in normal function, the next step is to maintain or modify the soft tissues in the retention phase. Important: most clinicians believe the collagen fibers in the supra-alveolar tissue are primarily responsible for relapse of orthodontically rotated teeth and for redevelopment of spaces between orthodontically moved teeth. Collagen fibers are the main components of attached gingiva. When teeth are orthodontically moved, collagen fibers stretch like rubber bands to adjust to the new position. However, like rubber bands, the fibers have a strong tendency to return to their former position (pulling the teeth with them as they go). The rationale for RETENTION in orthodontics (accomplished with fixed or removable retainers) is to allow reorganization of gingival & periodontal tissues, minimize changes due to growth, and to maintain the teeth in unstable conditions. Maintaining the treatment outcome after orthodontic treatment is one of the most difficult aspects of the entire treatment process. Retention is necessary because: 1. gingival and periodontal tissues are affected by orthodontic tooth movement, and require time for

reorganization when the appliances are removed. 2. changes produced by growth may alter the orthodontic treatment result. 3. teeth may be in an inherently unstable position after treatment so that soft tissue pressures constantly produce a tendency for relapse. Thus, gradual withdrawal of an orthodontic appliance is of no value. The only possibilities are accepting relapse or using permanent retention. Fortunately, only the first two reasons apply to most orthodontic patients, and maintaining the teeth’s position until remodeling of the supporting tissues is complete and growth has stopped allows a stable orthodontic result without further retention. Anterior crossbite is EASILY RETAINED after orthodontic correction by the overbite achieved during treatment.

ORTHODONTIC APPLIANCES Orthodontic Appliances: can irritate the gingiva, act as plaque harbors, and make proper oral hygiene difficult to perform. Prolonged orthodontic treatment has long been associated with causing inflammatory periodontal disease. However, if meticulous oral hygiene is maintained at all times during orthodontic treatment, periodontal health can be maintained. ♦ When a patient (young or old) is in active orthodontic treatment, and the gingiva is inflamed, the dentist should encourage better oral hygiene. It may be useful to recommend using water irrigation devices to help flush food debris away from the orthodontic brackets. ♦ For an orthodontic appliance to be effective in TRANSLATING TOOTH ROOTS it MUST be capable of EXERTING A TORQUE. FIXED Orthodontic Appliances-offer controlled tooth movement in all 3 PLANES OF SPACE. 4 basic components of a fixed appliance are BANDS, BRACKETS, ARCHWIRES, & AUXILLARIES (elastics or ligatures to hold the archwire in the brackets). ALLOYS used for orthodontic archwires: stainless steel, chromium-cobalt,

& titanium. ♦ The properties of stainless steel wires used for archwires can be controlled over a wide range by varying the amount of cold working annealing during manufacturing. Steel is softened by annealing and is hardened by cold working (work hardening). ♦ Chromium-cobalt alloys are advantageous because they can be supplied in a softer, thus more formable state, and can be hardened by heat treatment after being shaped. The heat treatment increases strength significantly. ♦ Titanium alloys offer a highly desirable combination of strength, springiness, & reasonably good formability. Properties of an IDEAL WIRE MATERIAL for orthodontic purposes should possess high strength, high range, high formability, and low stiffness (stiffness of orthodontic wires is a function of the wire’s length & diameter, and the wire’s alloy composition. The material should be weldable or solderable, so hooks or stops can be attached to the wire. Loops & Helices are incorporated into archwires to increase the activation range. Quad Helix-a FIXED appliance that consists of 4 helices (2 anterior & 2 posterior) used for POSTERIOR CROSS-

BITE cases with a digital-sucking habit. Unbuffered Phosphoric Acid (35-50%)-used as an ETCHING AGENT for 1 minute before direct bonding of orthodontic brackets. After etching, the tooth surface has a FROSTED appearance. When etching, the tooth surface cannot be contaminated with saliva that promotes immediate remineralization, until bonding is complete to avoid re-etching. DO NOT USE topical fluoride before etching as FLUORIDE DECREASES ENAMEL SOLUBILITY. Indications for using BANDS instead of bonded brackets: ♦ To provide better anchorage for greater tooth movement. ♦ For teeth that need both lingual and labial attachment. ♦ Teeth with short clinical crowns. ♦ Tooth surfaces that are incompatible with successful bonding. Band Cementation: glass-ionomer cements (resin or non-resin based) due to their fluoride releasing properties and retentive strengths, are fast replacing

zinc phosphate cement. The cold slab (“frozen slab technique”) is used to mix the cement on regardless of the cement that is used to allow a greater amount of powder into the cement liquid, to produce a stronger cement. INDIRECT METHOD OF BONDING BRACKETS to a tooth over the direct method: • Advantages: reduced chair-side time, accurate placement on teeth, controlled resin thickness between the tooth & bracket interface, and easier clean-up during bonding & de-bonding. • Disadvantage: indirect bonding is more complex, more technique sensitive, & requires extra precautions than direct bonding. Indirect Bracket Bonding Procedure: 1. take an accurate alginate impression and pour it with orthodontic model stone to be used as a working model. 2. draw vertical lines on the teeth to aid in bracket placement, and a separating media is applied. 3. brackets are then loaded with a filled resin paste and cured. 4. after its initial set, individual positioning of a tray with silicone is prepared by applying it over the bracketed teeth on a plaster model. 5. entire set-up is placed in warm water to dissolve the separating media. 6. silicone tray is then removed from the plaster model with

brackets embedded in it. 7. brackets are cleaned under running water making sure that pads have cured resin. 8. enamel is etched, conditioned, and unfilled resin is applied. Unfilled resin is also applied to cured resin on the base of the bracket pads. 9. silicone tray with embedded brackets is then positioned on the teeth being bonded and held in position until the initial set of unfilled resin is reached. The control of “flash” (excess resin) makes clean-up easier, and the controlled thickness of it accurately expresses the built-in prescription of the appliance. Also, in situations where visibility is a problem (i.e. lingual appliances), this technique is almost always used. Types of Fixed Appliances: ♦ Lingual Archwire Appliance & Whip-Spring Appliance ♦ Fixed-Space Maintainers & Palatal-Separating Devices ♦ Edgewise Mechanism ♦ Light-wire, twin wire, & universal appliances FIXED EDGEWISE APPLIANCE -the MOST WIDELY USED APPLIANCE TODAY by orthodontists. In its essential form, the mechanism consists of bands on all teeth, tubes on the last molar, and brackets on all other teeth. One labial arch is used at a time. The ultimate labial arch wire is .0125 x .028 in diameter, and the narrow

dimension (edge) fits precisely into the bracket slot (which is .022inch wide from top to bottom). It finds its greatest application in treating comprehensive malocclusions of the adolescent permanent dentition. ♦ VARIATIONS of the basic Edgewise Appliance: includes double (tandem) brackets (Siamese twin brackets), & narrow slotted brackets, 0.18 from top to bottom. A straight wire appliance is a version of the edgewise appliance with several features that allow placement of an ideal rectangular archwire without bends. ♦ Edgewise Appliance Components: o Siameses twin bracket (used on maxillary anterior teeth). o Broussard buccal tube (allows for the use of the segmented arch technique used to intrude teeth). o Straight wire bracket and a bracket with a .0222 x .028 rectangular slot. The bracket slot size of .022 inch allows a wide range of wire sizes to be used. The alternate slot size is .018 inch, which can also upright the molar, but limits the wires sizes available. The tipped 2nd molar should be banded because the considerable posterior masticatory forces produced can easily

shear off bonded brackets. ♦ The time required to UPRIGHT A MOLAR takes between 6-12 months (a severely tipped molar or molar that requires mesial movement to shorten the pontic space requires longer treatment). A Fixed Edgewise Appliance is usually used for MOLAR UPRIGHTING. Molar Uprighting Facts: o A severely lingually tipped mandibular molar is MORE DIFFICULT to control and upright properly. o Molar uprighting treatment in high angles cases results in excessive bite opening (increases vertical dimension of occlusion VDO). o Stabilization (retention) should last until the lamina dura & PDL reorganize which takes ~2 months for simple uprighting, and up to 6 months for uprighting and osseous surgery or grafts. This retention (stabilization) is provided by an appliance or well-fitting provisional restoration to stabilize the tooth positions and allow for reorganization of the PDL. Slow progress in molar uprighting in an adult patient is most likely due to occlusal interferences.

Conditions that can complicate MOLAR UPRIGHTING: • High mandibular plane angle (one of the most significant complications because if the molar is uprighted unsuccessfully, it can cause an increased open bite, and loss of anterior guidance). • Presence of periodontal disease. • Poor crown-to-root ratio and/or short roots. • Root resorption. • Significant CR to MIC discrepancy. • Severe lingual inclination of the tooth in addition to the mesial tipping. • Occlusal plane disharmony (i.e. extruded maxillary & mandibular molars). • Open bite. • Severe skeletal discrepancies. A common dental condition that can benefit from orthodontic treatment prior to prosthetic treatment is the long-term loss of a mandibular permanent 1st molar which causes tipping, migration & rotation of adjacent teeth into the edentulous space. The best way to upright a 2nd molar that drifted mesially is tipping its crown DISTALLY and opening up space for a pontic to replace the missing first molar, rather than attempting to move the 2nd molar mesially to close the space. A NORMAL ANGULATION of a molar is desirable because it improves the direction & distribution of occlusal forces, alveolar contour, crown-to-root ratio, and the

periodontal environment by eliminating plaque-retentive areas. It decreases the amount of tooth reduction required for parallelism of the abutments and the possibility of endodontic, periodontic, or more complex prosthodontic procedures. It increases restoration durability due to better force distribution. Whip-Spring Appliances-used to DE-ROTATE one or two teeth. Space Maintainers that REPLACE ONE PREMATURELY MISSING PRIMARY TOOTH: 1. “Band & Loop” Space Maintainer-most often used when the PRIMARY FIRST MOLAR must be prematurely extracted. Used after a unilateral loss of a primary first molar. It consists of a band that is usually cemented to a primary second molar. Attached to the band is a loop that extends to the canine’s distal surface. The loop prevents mesial migration of the primary second molar. Note: limited strength allows only single tooth-space maintenance. 2. Distal Shoe Space Maintainer-used when a primary second molar is lost BEFORE the permanent first molar erupts (typically children under age 5 or 6). ♦ Premature loss of a primary maxillary second molar usually produces a Class II molar relationship on the affected side. DISTAL SHOE SPACE MAINTAINER

may help alleviate this potential problem. This maintainer extends backwards from the primary first molar crown, and subgingivally to the mesial line of the unerupted first permanent molar, thus preventing mesial migration. Space Maintainers that REPLACE MULTIPLE PREMATURELY MISSING PRIMARY TEETH: 1. Lingual arch space maintainer-the primary second molars or permanent first molars are banded. Typically, the “lingual arch” space maintainer is comprised of two bands that are cemented to the primary second molars or permanent first molars with a loop of wire that rests on the cingula of the incisors. Used to maintain space when multiple primary teeth are missing and the permanent incisors have erupted. It does NOT restore function, and should be made completely passive. 2. Nance appliance (transpalatal appliance)-used for BILATERAL LOSS of primary maxillary molars. An acrylic button rests on the palate, and is attached to bands that are bilaterally cemented on the permanent maxillary molars. This appliance prevents MESIAL ROTATION & DRIFTING of the permanent maxillary molars to which it is attached. 3. Partial Denture-most useful for bilateral posterior space maintenance when permanent incisors have not

erupted. Also used for missing anterior teeth when esthetics are a concern. REMOVABLE orthodontic appliances-are generally restricted to TIPPING teeth. 1. Active Removable Appliances: includes extra-oral traction devices (head gears, face masks, chin cups), lip bumpers, active plates (Schwartz appliance & anterior spring aligners), and vacuum formed appliances. 2. Passive Removable Appliances: bite planes, occlusal splints, and retainers. Removable Orthodontic Appliance Components: 1. retentive component-retains the appliance in function, and consists of various clasps (i.e. Adam’s crib). 2. framework (baseplate)-made of acrylic and provides anchorage. 3. tooth-moving elements-are either springs or screws. 4. anchorage component-resists force of active components (i.e. acrylic base-plate). Anchorage components can also cause desired or undesired tooth movements. 5. active components (tooth moving components)springs, screws, or elastics.

Indications of Removable Appliances: retention after comprehensive treatment, limited tipping movements, & growth modification during the mixed dentition. Hawley Retainer-the MOST COMMON REMOVABLE RETAINER used is orthodontics. It incorporates clasps on molar teeth and a characteristic outer bow with adjustment loops that span from canine-to-canine. ♦ Palatal coverage of a removable plate like a Hawley retainer makes it possible to incorporate a BITE PLANE lingual to the maxillary incisors to CONTROL BITE DEPTH. This design is important for any patient who once had an EXCESSIVE OVERBITE. Palatal coverage with acrylic is the major source of anchorage in the Hawley appliance. ♦ Hawley retainer can be made for the upper & lower arch. The lower retainer is fragile and may be difficult to insert because of undercuts in the premolar region. A patient may have difficulty pronouncing linguoalveolar consonants for a few days after receiving a maxillary Hawley until the tongue adapts to the palatal coverage. Begg Appliance-uses round wires that fit loosely into the bracket’s vertical slot.

Frankel’s Appliance-a REMOVABLE functional appliance used for abnormal (hyperactive) soft tissue patterns. HEADGEAR-used to maintain EXTRA-ORAL ANCHORAGE& TRACTION. The ~required force for anchorage is 250g for 10hrs/day. The required force for traction is 500g for 14-16hrs/day. One of the greatest advantages of using extraoral anchorage (headgear) is it PERMITS POSTERIOR MOVEMENT OF TEETH IN ONE ARCH without adversely disturbing the opposing arch. ♦ Headgear extra-oral components: neck strap, chin cup, & head cap. ♦ Headgear intra-oral component: facebow which has an outer & inner bow. The length & position of the outer bow is set according to the inner bow. The inner bow relates to the center of resistance of the tooth and effects anchorage and/or traction. ♦ For use with URA, the direction of force above the occlusal plane aids in retention. 4 Basic Headgears: cervical-pull, straight-pull, highpull, & reverse-pull. These are selected based on the direction of force (pull) needed. 1. High-pull headgear: produces a DISTAL & UPWARD force on the maxillary teeth & maxilla. This headgear

consists of a head cap connected to a face-bow. This type of headgear has a more direct effect on the anterior segment of the arch. Indicated for Class II, Division I malocclusions that have an open bite. 2. Cervical-pull headgear: consists of a neck strap connected to a face-bow to produce a DISTAL & DOWNWARD force against the maxillary teeth & maxilla. Major disadvantage: possible extrusion of maxillary molars. Likely results include an open bite, and move first molars distally, and decrease forward growth of the maxilla. Indications: Class II, Division I malocclusions. 3. Straight-pull headgear: similar to cervical-pull headgear, but this appliance places a force in a straight distal direction from the maxillary molar. Like cervical-pull headgear, straight-pull headgear is indicated for Class II, Division I malocclusions (when bite opening is undesirable). 4. Reverse-pull headgear: UNLIKE ALL OTHER HEADGEAR BECAUSE IT HAS AN EXTRAORAL COMPONENT supported by the chin, cheeks, forehead, or combination of these structures. Indicated for Class III malocclusions (where protraction of the maxilla is desirable). FINGER SPRINGS-best method for TIPPING maxillary &

mandibular ANTERIOR TEETH. Finger springs are attached to a removable appliance. The most common problems associated with these simple removable appliances are lack of patient cooperation, poor design leading to a lack of retention, and improper activation. An undesirable common side effect of a finger spring is the tendency for the root apex to move in the direction opposite from the crown. ♦ Z-Springs-can also be used for tipping anterior teeth, but deliver excessively heavy forces & lack range of motion. ♦ Maxillary incisor rotation is not commonly treated during the mixed dentition stage, but is BEST TREATED after all permanent teeth have erupted (in the early permanent dentition) with SIMPLE REMOVABLE APPLIANCES. However, if the incisor is in anterior crossbite, it should be corrected ASAP (while it is erupting). ♦ When using BUCCAL COIL SPRINGS to try and regain space by pushing a tooth mesially or distally, be careful because what commonly occurs is ROTATION of that tooth, rather than actual movement. ♦ The force generated in the spring is DIRECTLY proportional to the distance that an orthodontic

spring is deflected and the radius (r) of the wire. The force is INVERSELY proportional to the spring’s length.

Loose Removable Appliances: include functional appliances and functional jaw orthopedic appliances. FUNCTIONAL APPLIANCES: Frankel, Bionator, Clark’s Twin Block, Herbst, & Activator. Functional appliances are classified as either tissue borne or tooth borne. 1. Tissue-Borne Functional Appliance: Frankel’s Functional Appliance-the only TISSUE-BORNE FUNCTIONAL APPLIANCE whose function is to expand the arch by “padding” against the pressure of the lips & cheeks on the teeth and postures the mandible forward and downward. 2. Tooth-Borne Functional Appliances: • Activator: advances the mandible to an edge-toedge position to induce mandibular growth to correct a Class II malocclusion. The maxillary teeth are prevented from erupting by the acrylic shelf, while mandibular posterior teeth are free to erupt. This improves the deep-bite seen in Class II cases. • Bionator: similar to the activator in function, but its design is a trimmed-down version of the activator to

make it more comfortable to wear. • Herbst Appliance: can be fixed or partially removable. A metal-rod & tube-telescopic apparatus are attached bilaterally to the maxillary first molars & mandibular first premolars to help posture the mandible forward and induce growth. Jasper modified Herbst by replacing the telescopic apparatus with a flexible plastic open coil spring. • Clark’s Twin Block: a two-piece acrylic appliance that postures the mandible forward with help of occlusally inclined guiding planes and bite blocks. The vertical separation of the jaws is also configured by the height of the bite blocks. A 7 year-old child with good occlusion has a LINGUALLY LOCKED maxillary permanent central incisor. There is sufficient room for the tooth. To treat this condition properly, the dentist should CORRECT THE CONDITION IMMEDIATELY WITH A SIMPLE APPLIANCE. Ideally, this anterior crossbite should have been corrected BEFORE it reached the occlusal plane (while it was erupting). The most probable etiologic factor for this happening is PROLONGED RETENTION OF PRIMARY MAXILLARY INCISORS. ♦ Cross-elastics from the maxillary lingual to mandibular labial can be used to correct a single-

tooth crossbite. A maxillary removable appliance can also be used. When elastics are used to move teeth they should be ATTACHED DIRECTLY TO THE APPLIANCE COMPONENTS. ♦ Anterior Crossbite (especially crossbite of the incisors), is RARELY FOUND in children who do not have a skeletal Class III jaw relationship. A crossbite relationship of 1 or 2 anterior teeth, however, may develop in a child who has good facial proportions. Maxillary lateral incisors tend to erupt to the lingual, and may become lingually trapped especially in the presence of severe crowding. In this situation, extracting the adjacent primary canines usually leads to spontaneous correction of the crossbite. It is important to evaluate the space situation before attempting to correct any anterior crossbite. If enough space is available to accomplish this movement, a maxillary removable appliance is usually the best mechanism to correct a simple anterior crossbite that requires a tipping movement. Anterior crossbite in a primary dentition usually indicates a SKELETAL GROWTH PROBLEM. First Order Bend-a bend of an orthodontic wire in the HORIZONTAL PLANE. Cartilage Growth Occurs 2 Ways:

1. Appositional Growth-by the recruitment of fresh cells, chondroblasts, from perichondral stem cells and the addition of new matrix to the surface. The perichondrium consists of a fibrous outer layer and chondroblastic inner layer. 2. Interstitial Growth-occurs by the mitotic division & deposition of more matrix around chondrocytes already established in the cartilage. Examples of sites that grow by interstitial growth include the mandibular condyle, nasal septum, and spheno-occipital synchondrosis. ♦ Hyaline Cartilage-differs from bone in that hyaline cartilage may grow by INTERSTITIAL GROWTH. BONE FORMS by Endochondral ossification or Intramembranous ossification. Bone formation begins in the embryo where mesenchymal cells differentiate into either fibrous membrane or cartilage. Do not confuse bone growth & bone formation. Once bone is formed, it then GROWS by appositional growth (growth by the addition of new layers on top of previous formed layers). This leads to two paths of bone development. 1. Intramembranous ossification-takes place IN membranes of C.T. Osteoprogenitor cells in the membrane differentiate into osteoblasts, and a collagen matrix is formed which undergoes ossification. The maxilla & mandible are formed this way.

♦ FLAT BONES of the skull & part of the clavicle are formed by INTRAMEMBRANOUS OSSIFICATION. 2. Endochondral ossification-is how the remainder of the skeleton forms and takes place within a hyaline cartilage model. Cartilage cells are replaced by bone cells (osteocytes replace chondrocytes), organic matrix is laid down and calcium and phosphate are deposited. Endochondral ossification is mainly responsible for formation of SHORT & LONG BONES (i.e. ethmoid, sphenoid, & temporal bones of the skull). BONE GROWTH occurs ONLY BY APPOSITIONAL GROWTH which below the covering periosteal layer of bone. Periosteum consists of a fibrous outer layer & a cellular inner layer of osteoblasts, which lay down bone. Due to its rigid structure, interstitial growth is not possible. ♦ A major site of growth of the mandible is the CONDYLE. Mandible growth occurs by cartilage proliferation at the condyles, and by apposition & resorption of bone at the mandible surfaces. Resorption occurs along the anterior surface of the ramus (creates space for mandibular molars), while bone apposition occurs along the posterior surface of the ramus. However, the mandible’s MAIN GROWTH SITE is in the CONDYLAR CARTILAGE. The “V principal” of growth is best illustrated by growth of the mandibular ramus.

Important: growth at the mandibular condyle during puberty usually results in an increase in posterior facial height. ♦ Mandibular growth involves a synchronous & selective deposition and resorption of bone from membrane surfaces and interstitial & appositional growth changes in the condyle. The MAIN GROWTH THRUST is in an UPWARD & BACKWARD direction causing the body of the mandible to move DOWNWARD & FORWARD. In this process, bone is deposited along the posterior aspects of the ramus and in the condylar area. ♦ Bone deposition in the MAXILLARY TUBEROSITY REGION is responsible for LENGTHENING OF THE MAXILLARY ARCH. The maxillary arch elongates, moves posterior, and increases in height. Bone deposition in the tuberosity region is responsible for the lengthening (elongation) of the maxillary arch. The posterior movement is due to resorption of the labio-alveolar surface, and apposition of the lingual surface. Alveolar growth is responsible for an increase in the height of maxillary bones. o Posterior movement predominates in the tuberosity area. The primary movement of the alveolar region & palate is downward, the nasal region moves forward, and zygomatic process

moves posteriorly and laterally. o Growth of the maxilla and its associated structures occurs from a combination of growth at sutures, and direct remodeling of the surface of the bone. ♦ In a young child, the ALVEOLAR PROCESS grows in height & length to accommodate the developing dentition. Alveolar process bone exists ONLY TO SUPPORT TEETH. If a tooth fails to erupt, alveolar bone will never form in that area. If a tooth is extracted, the alveolus resorbs after the extraction until finally the alveolar ridge completely atrophies. ♦ The space between the jaws into where the teeth erupt is provided by growth at the mandibular condyles (especially the molars). Condyle is a major site of vertical growth in the mandible. Many arguments have been made about the condyle’s function in mandibular growth. Most agree that soft-tissue development carries the mandible forward and downward, while condylar growth fills in the resultant space to maintain contact with the base of the skull. ♦ In infancy, the ramus is located at about the spot where the primary first molar will erupt. Progressive

posterior remodeling creates space for the second primary molar and then for the sequential eruption of the permanent molar teeth. More often than not, however, this growth stops before enough space has been created for eruption of the 3rd permanent molar, which becomes impacted in the ramus. After age 6, the greatest increase the mandible size occurs distal to the first molars. LATE MANDIBULAR GROWTH is the theory that best explains why there is a strong tendency for mandibular anterior crowding in the late teens and early 20’s. The current concept is that late incisor crowding develops as the mandibular incisors and possibly the entire mandibular dentition, move DISTALLY relative to the body of the mandible late in mandibular growth. o Late incisor crowding occurs in individuals with no 3rd molars, so the presence of these teeth is not a critical variable. However, the extent of late mandibular growth is a critical variable. o Mandible undergoes more growth in the LATE TEENS than the maxilla. The MOST RAPID LOSSES IN ARCH PERIMETER are usually due to a MESIAL TIPPING & ROTATION of the permanent first molar after removal of the primary second molar. When the primary second molar is lost, ALWAYS

MAINTAIN SPACE until the second premolar arrives. • If a permanent 1st molar is extracted on a child before the permanent second molar erupts, the best approach is to allow the second molar to erupt and allow mesial drifting to occur naturally as this will usually fill in the space. • A space maintainer can be removed as soon as the permanent tooth begins to erupt through the gingiva. • MOST RELIABLE INDICATOR of readiness of eruption of a succedaneous (permanent) tooth (and the need for a space maintainer) is the EXTENT OF ROOT DEVELOPMENT determined by radiographic evaluation. A space maintainer is NOT required if eruption of the succedaneous (permanent) tooth is imminent.

BEHAVIORAL SCIENCE Psychosocial Factors that strongly influence behavior: attitudes, beliefs, values, family, society, culture, and education. Behavior is a determined, purposeful unit of activity. • Determined-the assumption that behavior is lawful and has determinants. • Purposeful-the assumption that behavior is goaloriented, that is seeks to achieve positive and reduce negative need or motivational states. • Unit of Activity-what a person does that can be reported or described as discrete elements. Ex: teeth do not behave, individuals behave. Observing that a pulpal or periodontal problem exists is a common behavior for the dentist. Avoiding the dentist, even though an objective need exists and the patient requires treatment, is a common behavior for patients. Both meet the criterion of being determined, purposeful units of human activity. Behavioral Development-any observable response mediated through the neuromotor system. To understand the development of human behavior, you must

understand the basic concepts of maturation and learning. There are 4 major fields of behavior: 1. Personal Social-is usually a function of environment, work, play, and society. 2. Motor-starting point to access maturity. 3. Language-vocalization, words, sentences, facial, and manual movements. 4. Adaptive-use of motor capacity and solutions to practical behavior. Behavior Management-the means by which the dental health team effectively & efficiently performs treatment for the patient while simultaneously instilling a positive attitude. Most researchers believe changes in behavior are a prerequisite to changes in attitude. Ex: Ashley, a 32-year-old women, comes in for her routine cleaning appointment. Stephanie, the hygienist finds that Ashley has not been following the home care program recommended six months ago. Stephanie believes Ashley’s problem is a management deficiency, not a skills deficiency. Thus, the best course of action for Stephanie to take is to meet with her supervising dentist to determine the future course of action. • Since this is not a skills deficiency problem, reviewing homecare techniques is not going to solve the problem. Ashley knows what to do. Now Stephanie and Ashley’s supervising dentist needs to find a way to motivate Ashley to find the time to brush

and floss. • The most effective way to teach oral hygiene skills is by having the patient participate in repeated, supervised training sessions. • The best time to determine a patient’s plaque index (to assess the effectiveness of patient homecare) is at the beginning of the appointment. Rather than just asking about the patient’s homecare skills, have the patient demonstrate their brushing/flossing skills. • Maintaining a 4-year-old child’s healthy dentition starts with educating the parent. • Having your teeth cleaned and examined regularly and keeping them clean daily at home is the best way to prevent periodontal disease. • Patient information required to plan dental hygiene care includes: health & dental history, dietary analysis, and periodontal examination. 7 Steps in the Educational Process: 1. Step 1 (Recognizing Needs)-the dentist recognizes educational needs as he checks for treatment needs. Then, the dentist helps the patient recognize his own needs. 2. Step 2 (Expressing Needs)-the dentist records

educational needs and helps the patient state his own needs. 3. Step 3 (Stimulating Motivation)-motivation arouses and maintains interest. The dentist may appeal to inner needs or use artificial stimuli. 4. Step 4 (Setting Goals)-goals may be short-term or longterm guides to activity. The goals must be meaningful, attractive, and attainable. 5. Step 5 (Acting to Achieve Goals)-activity is necessary to learning. The activity should be directed toward specific goals. 6. Step 6 (Reinforcing Learning)-review and repetition aid in retention of learning. 7. Step 7 (Evaluating Results)-aids in judging what the patient has learned and how effective the dentist’s teaching has been. This can help clarify or redefine the goals. Each learning situation will not follow these steps in this exact sequence, but most learning situations will include all of these steps in some form. NEEDS AND LEARNING: • Needs are driving forces that prompt a person to act.

Motivation stimulates a person to act on his needs and is a fundamental part of every learning situation. Motivation can be artificial or built-in. Needs and goals may provide motivation, and patients rarely learn without some motivation. Motivation arouses and maintains interests. • Short-range goals are less remote and more easily attained than long-term goals. Goals should be attractive and attainable to be meaningful. Goaldirected activity is required for learning. • The learning process is continual and multiple, and occurs as a person attempts to satisfy needs. • Telling a person what he needs may convince him that a behavior change is desirable. • Expressing needs helps pinpoint them for the dentist and patient. • Recording educational needs can be as important as recording treatment needs. Behavior Modification (Behavior Therapy)-a type of psychotherapy that attempts to modify observable, maladjusted behavior patterns by substituting a new response or set of responses to a given stimulus. Psychologists have developed many techniques to modify

patient behavior by using the principles of learning theory. Examples of techniques/methods used mainly in pediatric dentistry: 1. Classical Conditioning (Pavlovian or Respondent Conditioning)-a mechanism by which behavioral responses are learned. Operates by associating one stimulus with another. A stimulus leads to a response. If individuals in white coats give painful injections that cause crying, the sight of an individual in a white coat soon may provoke a crying outburst. If this is not reinforced, the conditioned response will no longer occur (extinction of the conditioned behavior). Classical conditioning is a form of learning in which a previously neutral stimulus comes to elicit a given response through associative training. 2. Operant Conditioning-the consequences of a behavior is in itself a stimulus that can affect future behavior. The consequences that follow a response alter the probability of that response occurring again in a similar situation. It is a form of learning in which the person undergoing therapy is rewarded for the correct response and punished for the incorrect response. There are 4 types of operant conditioning distinguished by the nature of the consequence: positive & negative reinforcement, omission, and punishment. • Behavior Shaping (Successive Approximation)-an operant conditioning technique used in behavior

therapy in which new behavior is produced by providing reinforcement for progressively closer approximations of the final desired behavior. It is a common non-pharmacologic technique. • Proponents of this theory believe that most behavior is learned, and learning is the establishment of a connection between a stimulus and a response. Thus, it is sometimes called the “Stimulus-Response Theory” (SR Theory). • When shaping behavior, the dentist assistant or dentist is teaching a child how to behave. Young children are led through these procedures stepby-step. • Behavior Shaping is regarded as a learning model. A general rule about learning models is the most efficient learning models are those that follow the learning theory model most closely. • Ex: attempting to change several aspects of a patient’s oral hygiene regimen should be done one aspect at a time (sequentially). Have the patient mimic the correct oral hygiene behavior to increase the chances of succeeding in changing the patient’s behavior.

3. Aversion Conditioning-a technique in which punishment, unpleasant or painful stimuli are used in the suppression of undesirable behavior. In dentistry it is called the “Hand-Over-Mouth” technique (HOME). 4. Observational Learning (Modeling or Behavior Shaping)-a behavior acquired through initiation of a behavior observed in a social context. There are two distinct stages in observational learning: acquisition of the behavior by observing the behavior and the actual performance of the behavior. Having an open dental office design may aid in the dentist in this method. Children can watch other cooperative children, and this may rub off on them. Modeling is a technique in which the person learns a desired response by observing it being performed. Relative maturity modifies expectations of a child’s behavior in that a child cannot be expected to learn a mode of behavior until he has matured to a stage at which he is ready for such learning. 5. Systemic Desensitization-a technique used to eliminate maladaptive anxiety associated with phobias. The procedure involves the construction by the person of a hierarchy of anxiety producing stimuli and the general presentation of these stimuli until they no longer elicit the initial response of fear. Communication is basic to all aspects of the dentist-

patient relationship. Without proper communication (verbal or non-verbal), dentist-patient relationships would fail. Acceptable non-verbal behavior varies with age, sex, ethnic background, geographical region, culture, and situation. • When communicating with children, try to reinforce positive behavior by telling the child exactly what he is doing well. • Non-verbal behaviors: posture, facial expression, eye-contact (most important), body position (physical proximity), and gesticulations. Eye Contact is the primary non-verbal cue that two or more people use to regulate verbal communication. Eyes are directed toward the patient and engage the patient’s eyes as frequently as is comfortable for the speaker and listener. Types of Questions: These are all VERBAL communication behaviors. 1. Open-Ended Questions: requests information in the patient’s own words and specifies a general content area. Ex: “How are you doing with your brushing and flossing?” When presenting treatment plans always use open-ended questions (questions that cannot be answered with a simple “yes” or “no”). Open-ended questions are the most effective in helping patients to express their understanding of the proposed treatment plan. When reviewing oral hygiene, have the

patient repeat what you have gone over with them. 2. Direct Questions: questions that ask the patient for a specific bit of information. Ex: “Is it easier to hold the brush this way?” 3. Probing Questions: questions that ask for more specific information that the patient offers spontaneously. Ex: “What else did you notice about your gums?” 4. Laundry-List Questions: questions that ask the patient to respond from among a list of alternative adjectives or descriptions provided. Ex: “Is the pain throbbing, aching, dull, or sharp?” 5. Leading Questions: questions that entice a patient to answer a specific way. Ex: “You are not afraid of needles, are you?” 6. Facilitating Questions: questions that encourage the patient to say more without specifying an area or topic. Ex: “How are you?” AVERSIVE CONDITIONS of interaction between the dentist & patient: psychophysiological reactions, stress, anxiety, fear, and pain. These conditions are perceived as aversive and the dentist-patient interactions seeks to MINIMIZE them.

NON-AVERSIVE CONDITIONS are perceived as nonaversive, and the dentist-patient interactions seeks to MAXIMIZE them. • Communication-gathering information, identifying problems, giving information (as in case presentations). • Preventive oral-health behavior. • Management of exceptional patients such as the physically or emotionally disabled. For these patients, gradually expose them to the dental office. Specific Ways to Effectively Communicate with Patients: • Describe, be specific, be responsive, time appropriate, and pay attention (the best way to show a patient you care about what he/she is telling you is to use EYE CONTACT). Do not evaluate, be general, be evasive or premature, too deep, or inattentive or wander. Aggression: 1. Constructive Aggression-an act of self-assertiveness in response to a threatening action for purpose of selfprotection and preservation. 2. Destructive Aggression-an act of hostility unnecessary for self-protection or preservation directed toward an external object or person.

3. Inward Aggression-destructive behavior directed against oneself. 4. Aggressive personality-a personality with behavior patterns characterized by irritability, tantrums, destructiveness, or violence in response to a frustration. Stress, Anxiety, & Fear are simultaneously negative or aversive emotional states, full of symptoms that can motivate through a process of threat appraisal. The interaction of the intensity of an emotional response with threat appraisal determines the content of the behavior that will follow (whether or not to show up at the dentist’s office, to submit to an injection, to accept the need for an extraction or filling, etc.) 1. Anxiety-a state or feeling of apprehension, uneasiness, agitation, or uncertainty resulting from the anticipation of some threat or danger, usually of intrapsychic rather than external origin, whose source is generally unknown or unrecognized. The ANXIOUS patient is usually considered the most difficult patient. Most dentists become anxious with an anxious patient. Most patients who are anxious have had a traumatic experience in a dental or medical setting. Ways to reduce patient anxiety: • Explain procedures before doing them and forewarn about the possibility of pain.

• Give the patient some control over the procedures and pain (i.e. raise your hand if you feel anything). • Build trust between you and the patient. • Watching a patient’s eyes or eyebrows is a good indication if the patient feels pain during dental treatment. 2. Fear-anticipation of a threat elicited by an external object generally agreed to be harmful. In evaluating a patient’s dental fears, take note of what the patient says, how he/she behaves, and how they appear while in the dental office. Fear is distinguished from anxiety on the basis of the person’s ability to locate the threatening agent “out there” and to recognize the clear presence of a behavior that will reduce perceived danger. Patients who are fearful or anxious will do anything to put off making dental appointments. 3. Stress-a general disturbance in psycho-physiological adaptation. It implies a person who is being maladaptively influenced by more than one negative or aversive factor. Stress is mostly associated with response aspects. PARENT TYPES: 1. Manipulative Parents: have excessively demanding attitudes that usually start with appointment times and

can extend to directing the course of diagnosis or treatment. 2. Overprotective Parents: insist on remaining with the child in the dental operatory, regardless of the situation or child’s age. Pointing to the lack of apprehension of a young child and the importance of establishing a one-toone relationship between the child and dentist, usually satisfies most overprotective parents. Overprotective parents usually have children who are shy, docile, and manageable. 3. Hostile Parents: question the necessity for treatment usually due to distrust, not curiosity. 4. Neglectful Parents: fail to maintain appointments, miss recall visits, or do not oversee the child’s oral hygiene. Children with defiant behavior are usually stubborn or spoiled. For hostile or angry children, try to identify the underlying source of these emotions (this holds true for adult patients too). HEALTH BELIEF MODEL-a conceptual framework that describes a person’s health behavior as an expression of his or her health beliefs. HBM was designed to predict a person’s health behavior, including the use of health services and to justify intervention to alter maladaptive health behavior. Health Belief Model suggests that

individuals will act to prevent disease only when they believe they are susceptible to disease. A patient’s compliance is affected by their perception of a disease’s severity and length of a treatment regimen. Components of the Health Belief Model: 1. person’s own perception of susceptibility to a disease or condition. 2. likelihood of contracting that disease or condition. 3. person’s perception of the severity of the consequences of contracting the condition or disease. 4. perceived benefits of care and barriers to preventive behavior. 5. internal or external stimuli that result in appropriate health behavior by the person. The ethical rules and principles of professional conduct for the practice of dentistry are set forth in the ADA’s publication “Principles of Ethics and Code of Professional Conduct”. The ethical principles found in this code are justice, autonomy, and beneficence. • Justice-the quality of being impartial and fair. • Autonomy-to inform patients about treatment, to be truthful, and protect their confidentiality. • Beneficence-to be kind and give the highest quality of care one is capable of providing. The dentist is responsible for providing information and dental care. However, the patient is ultimately responsible

for maintaining his/her own oral health (brushing, flossing, etc.). Good Samaritan Law-a law enacted in all states that provides immunity from suit for specified health practitioners who render emergency aid to victims of accidents, provided there is no evidence of gross negligence. Not all states include dentists in the Good Samaritan Law. Child Abuse: Dentists are morally, ethically, and legally obligated to report a suspected case of child abuse. Once an injury or a suspicious nature is observed, the dentist’s first and immediate responsibility is to protect the child. • Reports should be made to the designated state agency (Social Services or Police). Dentists must familiarize themselves with the exact procedures to be followed in their states. • Child abuse most commonly involves newborns and children up to age 3 years. • Dentists are also ethically obligated to identify and refer cases of domestic violence. Practitioners should be familiar with the physical signs of domestic violence, especially since 68% of battered women injuries involve the face, 45% the eyes, and 12% the neck.

Managed Care-an arrangement where a third-party payer (insurance company, federal government, or corporation) mediates between doctors and patients, negotiating fees for services and overseeing the types of treatment provided. • Types of Managed Care Practices: HMO, PPO, IPA (Independent Provider Association). • PPO (Preferred Provider Organization)-typically involves contracts between insurers and dentists. Patients can choose their dentist depending on if the dentist participates in the PPO arrangement. • Participants of HMOs are much more limited in their dentist selection because they have to stay within network.

PUBLIC HEALTH, RESEARCH METHODS, AND ORAL HEALTH INDICES PUBLIC HEALTH-the science & art of preventing disease, prolonging life, and promoting physical health & efficiency through organized community efforts. 3 principles of public health: a problem exists, solutions exist, and solutions to the problem are applied. Public health problem does NOT always result in government passing new laws against a problem nor result in public demand for immediate government intervention. A public health problem must meet these criteria: 1. A widespread condition or situation that is the actual or potential cause of morbidity and/or mortality. 2. Involves a perception by the public, public health authorities, and government that a public health problem is occurring. Most important concept of Winslow’s definition of public

health is PROMOTION COMMUNITY EFFORT.

THROUGH

ORGANIZED

DENTAL PUBLIC HEALTH-science and art of preventing and controlling dental disease, and promoting dental health through organized community efforts. A form of dental practice that serves the community as a patient rather than serving the individual. Dental public health involves providing dental education to the public, research, and applying research findings with the administration of dental care programs for groups, and the prevention and control of dental disease through a community approach. Fundamental principles of public health are prevention, cost-efficiency, and teamwork. Prevention is the major objective of public health programs because it entails ethics, teamwork, and cost-efficiency. It is more ethical to prevent disease than to cure it. Teamwork is necessary to handle large groups efficiently. Cost-efficiency plays a major role because prevention is cheaper than a cure. • Education plays an important role in public health because it decreases the need for government intervention. When people learn why regulations are of value, they will comply. Ex: when people learn how many lives are saved annually by seatbelts, they are more inclined to wear them. • Any school-based program to promote oral health should have these fundamental components:

1. Oral health services: involves preventive procedures, health screening and treatment, referral, and follow-up. 2. Health instruction: includes personal and community health topics. 3. Healthy environment: attention to all aspects of the school environment that could affect the health of students or school personnel. The more successful school-based programs use a high degree of active involvement of the participants. Quality Assurance-the measurement of the quality of care and implementation of any necessary changes to maintain or improve the quality of care rendered. Quality assurance includes the additional dimension of action to take the necessary corrective steps to improve the situation in the future. Concepts that relate to quality assurance: 1. structure-the layout and equipment of a facility. 2. process-involves the actual services that the dentist and assistant perform for patients, and how well they perform. 3. outcome-the change in health status that occurs as a result of the care delivered

RESEARCH METHODS, VARIABLES, & STATISTICS RANDOMIZED STUDY-a study where ALL subjects have an equal chance of being assigned to either the study or control group. Statistical probability is such that the assumption can then be made that the groups differ ONLY in terms of the agent under study. Any uncontrolled variables influencing the outcome are likely to affects subject in both groups equally. Thus, researchers prefer the “random assignment method” for placing subjects into either the study or control group. BLIND STUDY-a study where subjects are unaware of if they are in a test or control group. One way of achieving a blinded study is using placebos. • Double Blind Study-neither participants (subjects) nor examiners know the group allocations (test or control groups). RESEARCH STUDY VARIABLES: 1. Dependent variable-the variable whose value depends on those of others (i.e. in the formula x = 3y + z) x is the dependent variable.

2. Independent variable-the variable whose value determines the other variable values (i.e. x = 3y + z) y & z are the independent variables. RESEARCH SAMPLES: representative portion of the population: 1. Random Sample: every element in the population has an EQUAL chance of being selected; reduces the chance of bias. REDUCES BIAS. 2. Stratified Random Sample: selecting an element according to certain subgroups; accomplished by selecting a proportionate number of participants from each subgroup for the sample (ex: identify all of the dental schools studying from the DENTIN NBDE II BOARD PREP and select two members from each school). Subjects are randomly chosen from a previously subdivided population. 3. Systematic Sample: the researcher randomly chooses the first participant from the population, then selects every ‘nth” subject to participate. Ex: a researcher has a population of 100, but only needs 7 subjects. He first picks his starting number “5” then picks his interval as “4”. Thus, the members of his sample would be subjects: 5, 9, 13, 17, 21, 25, 29. There is a chance for researcher bias. 4. Judgment Sample: someone familiar with the

population selects the sample; HIGH chance of bias. (ex: your class president selects fellow students to be in one of four simulation lab groups). 5. Convenience Sample: sample group is chosen based solely on convenience (ex: the first 20 people to walk through the door today were asked to complete a questionnaire). ELIMINATES BIAS. INFERENTIAL STATISTICS-uses a random data sample from a population to describe and make general inferences based on what the population might think. Valuable when it is impractical or possible to examine each member of an entire population. Ex: 50 out of 100 people randomly sampled in a Virginia suburb said they floss. Thus, it is inferred that 50% of all people in that Virginia suburb floss their teeth. Uses three main tests to determine statistical significance: T-TEST-the most common method to evaluate the HYPOTHETICAL DIFFERENCE IN MEANS BETWEEN TWO GROUPS. • Unpaired t-test: compares differences between males and females oral hygiene indices in a sample group. • Paired t-test: compares the effects of two toothpastes to improve the oral hygiene index.

P-VALUE (STATISTICAL SIGNIFICANCE): a statistic used to test a hypothesis to help the researcher determine the SIGNIFICANCE of test results or probability that a condition or circumstance would occur by chance without any experimental manipulation. All hypothesis tests use p-value to weigh the strength of evidence. • SMALL p-values (< 0.05) indicate statistically significant results ( 0.05) indicate the occurrences were likely DUE TO CHANCE (cannot reject the null hypothesis). P-value greater than 0.5 (1-20) affects study results (this means there is a 5% chance that the relationship between the variables in the sample is due to chance). The higher the pvalue, the LESS likely a relationship exists between two variables. ANOVA (Analysis of Variance)-a test to compare the hypothetical statistical differences between THREE or more mean scores. FALSE NEGATIVE RESULT-a test result that erroneously excludes an individual from a specific diagnostic or reference group, due mainly to insufficient exact methods of testing. Simply stated, if a diagnostic test fails to identify a case of true disease, it is a false negative result.

FALSE POSITIVE RESULT-this result erroneously assigns an individual to a specific diagnostic or reference group, due particularly to insufficiently exact methods of testing. VALIDITY-the test or study measures what it is supposed to measure. Degree that a study or procedure can be conclusive yet sufficient realistic; Does the test measure what it claims to be measuring. Ex: increase in CAL loss has been shown to be the BEST predictor of advancing periodontal disease. It is a valid way to test periodontal bacteria. RELIABILITY-the degree to which a measurement is CONSISTENTLY reproducible. Determined by taking several measurements on the same study subjects PEER REVIEW (REFEREEING)-the process of subjecting a dental author’s scholarly research or work to the scrutiny of dental experts in the same field before the journal or book is published to improve research validity. Sensitivity & Specificity are INVERSELY proportional. As the specificity of a test increases, the sensitivity decreases. Sensitivity-ability of the test to diagnose correctly a condition or disease that actually exists. Sensitivity measures the proportion of people with a disease

who are correctly identified by a positive test. Sensitivity is defined as the number of true positives (TP) divided by total number of potential positive findings (true positives and false negatives) in the sample. Sensitivity = TP/TP + FN. Specificity-ability of the test to classify health. Specificity is defined by the number of true negative (TN) results divided by the total number of false positive (FP) + true negative (TN) results in a sample. Specificity = TN/FP + TN. Prevalence-the number of OLD cases of disease present in a population at risk at a specific period of time. It is the proportion of persons in a population suffering from a particular disease at a given point in time. A proportion expressed as a percentage of the population. Incidence-number of NEW cases of a specific disease occurring within a population at a certain amount of time. Expressed as a rate (cases)/(population)/(time). Incidence is a “rate” that requires a unit of time. Frequency-a count. Epidemiology-the study of the incidence, prevalence, mode of transmission, and control of diseases in a

defined human population to establish programs to prevent and control their development and spread. CASE CONTROL STUDY (RESTROSPECTIVE STUDY)the best approach to study a RARE disease. CROSS-SECTIONAL STUDY-different subjects of different groups are compared at ONE POINT IN TIME (snapshot in time). Researcher selects children previously treated with fluoride varnish who are currently in grades 1-12. The effect of prior fluoride varnish is evaluated once (a snaphot in time). Less expensive and fast compared to a longitudinal study. LESS VALID THAN LONGITUDINAL STUDY. RANDOMIZED CONTROL TRIALS-provide STRONGEST FORM OF EVIDENCE.

the

DOUBLE BLIND STUDY- a study where BOTH subjects and researcher are unaware of which subjects belong to the experimental group or the control group to prevent researcher and subject bias (BEST study method to prevent bias). Dental clinical boards are often double-blind examinations. ORAL HEALTH SURVEY-the first step in a public oral health procedure. A sound basis to assess the current oral health status of a population and its future oral health needs.

CORRELATION CO-EFFICIENT: determines the strength of relationship between two variables. Shows probable cause and effect. Given as a number between +1 and -1. • THE CLOSER THE CORRELATION CO-EFFICIENT IS TO +1 OR -1, THE STRONGER (MORE PERFECT) THE CORRLEATION. • Positive Correlation-as the value of x increases, the value of y increases AND as the value of x decreases, the value of y decreases; reflects a DIRECT association between the variables. Ex: The more dietary carbohydrates and sugar consumed, the greater chance of caries and obesity). • Negative Correlation-as the value of x decreases, the value of y increases; the score on one scale predicts an opposite score on the other scale. Ex: An increase in patient oral hygiene proficiency resulting in a decrease in plaque accumulation. MEASURE OF CENTRAL TENDENCY (AVERAGES)-a central or typical value of a probability distribution. The frequency of distribution is plotted on an x-y graph to create a pictorial representation of the data. The most common measures of central tendency are MEAN, MEDIAN, & MODE. • MEAN-arithmetic average of scores, and most common measure of central tendency. Mean is the sum of a collection of numbers DIVIDED by the

number or numbers in the collection. Add all the scores together and divide by the number of scores. The mean is sensitive to extreme values (outliers). Ex: If 10 dentists scored a total of 950 points out of 1000 possible points on the NBDE II after using the DENTIN study guide, then the mean score is 950/10 = 95%. • MEDIAN-divides the distribution of score into two equal parts. 50% of the scores will be above the median; 50% will be below. NOT affected by extreme high or low scores. To calculate, place scores in a data matrix and locate midpoint. • MODE-the MOST frequently occurring score within the data matrix. Affects the skew of the graph. To calculate: using an ungrouped tally system, identify the score that occurs most often. BELL CURVE-a normal “bell shaped” curve (Gaussian Curve) represents plotted data that are EQUALLY distributed around the mean, median, and mode. When sample data is plotted on a NORMAL BELL CURVE, the MEAN, MEDIAN, & MODE are EQUAL.

VITAL STATISTICS-quantitative data routinely collected by epidemiologists and public health professionals on mortality, morbidity, natality, birth-death ratios, and crude death ratios. Vital statistics are quantitative methods to monitor and evaluate the life history of a specific population. Vital statistics identify community health needs, estimate healthcare costs, and evaluate health program effectiveness. 1. Mortality (Death Rate)-number of deaths caused by a specific disease. It is the ratio of the number of deaths caused by the disease, to the total number of cases of the disease at a specific time. 2. Morbidity-the incidence of a specific disease within a

given population. 3. Natality (Birth Rate)-the ratio of births to the general population. 4. Birth-Death Ratio (Vital Index)-the number of births in a given year divided by the number of deaths in a given year. It is an indication of the population growth, stability, and reduction. 5. Crude Death Rate-ratio of the number of deaths occurring within a given time period and population to the total population during that time.

ORAL HEALTH INDICES DENTAL INDEX-a data collection instrument used to numerically express the oral health status of a population. 1. DMFT Index (Decayed-Missing-Filled Teeth Index)-an irreversible index (measures conditions that cannot be reversed like caries) applied only to permanent teeth. DMFT index results yield a group’s caries susceptibility. DMFT has received universal acceptance, and is probably the best known of all dental indices. DMFT is the best caries index to use if the major purpose of an epidemiologist’s research is to determine caries susceptibility as opposed to immediate treatment needs. DMFT determines the total dental caries experience (past and present) but is only used for permanent teeth. It has received universal acceptance and is the best know of all dental indexes. DMFT Index Limitations: • DMFT values are not related to the number of teeth at risk. • DMFT index can be invalid in older adults because teeth can become lost for non-caries reasons. • DMFT index can be misleading in children whose teeth have been extracted for orthodontic

reasons. • DMFT cannot be used for root caries and cannot account for sealed teeth. 2. DEFT Index (Decayed, Extracted, Filled Teeth)-used for the primary dentition. 3. DMFS Index (Decayed, Missing, Filled Surfaces)-the same as DMFT, but records involved tooth surfaces. 4. GINIGIVAL INDEX (GI)-a reversible index used to assess the severity of gingivitis based on color, consistency, and BOP. ♦ Gingivitis is most commonly scored with the gingival index of Loe & Silness which grades the gingiva based on the four surfaces of each tooth based on inflammation and bleeding. ♦ GI, Papillary, Marginal, & Attached Gingiva Index (PMA Index) are confined to measurements within the gingiva. PMA index is used to record the prevalence and severity of gingivitis in school children. The presence or absence of gingivitis is noted in the gingival papillae, gingival margin, and attached gingiva. 5. PERIODONTAL INDEX (PI)-a reversible index that measures conditions that can be changed (plaque and

bleeding). Measures the periodontal condition of a population. Used to assess the prevalence of periodontal disease in populations of people. The best time to perform this index is at the beginning of the appointment. ♦ The condition of the gingiva AND bone are estimated individually for each tooth. A progressive scale that gives little weight to gingival inflammation, and more weight to alveolar bone resorption is used for scoring. ♦ Scores from each tooth are added together, then is divided by the number of teeth in the mouth to give the patient’s periodontal disease index. This score reflects the average status of periodontal disease in a given mouth without reference to the type or causes of disease. The community’s score is the average of individual scores of persons examined. 6. SIMPLIFIED ORAL HYGIENE INDEX-a reversible index used to measure oral hygiene status by estimating the tooth surface covered with plaque and/or calculus. 7. PLAQUE INDEX (PI) of Silness & Loe-a reversible index to assess the THICKNESS of plaque at the gingival margin (measures oral debris). Used to determine plaque accumulation. The best time to perform a plaque index is at the beginning of the

appointment. The plaque index is a specific index for scoring plaque and oral debris. It was derived from the papillary, marginal, and attached gingiva index (PMA). The plaque index is based on an assessment of the severity and location of soft debris aggregates in terms of scores 0, 1, 2, 3. A plaque score of “0” means the gingival area of the tooth surface is plaque free. The examination is made by passing a probe over the tooth surface into the gingival sulcus. If no soft material adheres to the probe, the area is given a “0”. A plaque score of “1” is given when plaque cannot be observed on the tooth, but is observed on the probe. A plaque score of “2” is given when a thin to moderately thick plaque layer is visible to the eye. A plaque score of “3” is given when a heavy accumulation of plaque fills the area between the gingival margin and the tooth surface, or fills the interdental area. In this system, the most stress is placed on the THICKNESS OF PLAQUE at the gingival margin area on all four surfaces of each tooth. The scores of all three areas of all teeth are added then divided

by the number of teeth. The plaque index has been extensively used, but does NOT have universal acceptability. Some studies show 80-90% of children have inflammatory periodontal disease (gingivitis or periodontitis) by age 15. Localized acute gingivitis is the most common form. Epidemiologic studies show the strongest relationship between prevalence and severity of periodontal disease with oral hygiene and age. 8. Sulcus Bleeding Index (SBI)-used to determine the bleeding and gingival health.

OCCUPATIONAL SAFETY & HEALTH ADMINISTRATION (OSHA) INFECTION CONTROL Standard (“Universal”) Infection Control Precautions-a method of infection control in which all human blood and certain body fluids (saliva in dentistry) are treated as if known to be infectious for HIV, HBV, HCV, and other blood borne pathogens. The same infection control procedures are used for all patients. • A thorough medical history is obtained for all patients at the first visit and updated and reviewed at subsequent visits. However, since not all patients with infectious diseases can be identified from a medical history, physical exam, or readily available lab tests, the CDC introduced the concept of “Standard Precautions”. Universal precautions are effective in preventing disease transmission from dental worker to patient, patient to dental worker, and patient to patient. Occupational Safety & Health Administration (OSHA)-a federal agency created by Congress in 1970 to protect WORKERS from hazards in the work place. OSHA is

concerned with regulated WASTE within the dental office. According to OSHA, “regulated waste” includes: Blood or other potentially infectious materials (including saliva) in dental procedures. Items that would release blood and other potentially infectious materials (including saliva), if compressed. Items caked with dried blood or other potentially infectious materials capable of releasing these materials during handling. Contaminated sharps Pathological and microbiological waste containing blood or other potentially infectious materials (including saliva). Regulated waste MUST be placed in containers that are closable, constructed to contain all contents and prevent leakage, and labeled appropriately. The container must be closed prior to removal to prevent spillage or protrusion of contents. The rule also requires that if the outside of the container becomes contaminated, it must be placed in a second container with the same characteristics. The need for a second container is extremely unlikely in a dental office. Hazardous Waste-waste causing harm or injury to the environment (not all hazardous waste is toxic/poisonous). Toxic Waste-waste capable of causing a poisonous effect.

Infectious Waste-waste that contains strong enough pathogens in sufficient quantity to cause disease. All infectious waste is contaminated, however, not all contaminated waste is infectious (may not be able to cause disease). AIDS prompted OSHA to adopt the Bloodborne Pathogens Standard for Dentistry. OSHA is also concerned with Hepatitis B and other bloodborne diseases, but AIDS is the disease that prompted regulatory action. • In 1986, unions representing healthcare workers petitioned OSHA for an emergency rule to protect their members from workplace exposure to the human immunodeficiency virus (HIV) and hepatitis B virus (HBV). OSHA denied the petition, but agreed to adopt a permanent rule on exposure to bloodborne pathogens through the regular rule making process. It took five years to develop the rule which now applies to hospitals, physician offices, nursing homes, other healthcare settings, emergency response personnel, funeral homes, and dental offices. • Bloodborne Pathogens-pathogenic microorganisms present in human blood that can cause disease in humans. • Other potential infectious materials (OPIMs):

human body fluids (semen, vaginal secretions, CSF, synovial fluid, pleural and pericardial fluids, peritoneal fluid, amniotic fluid, & saliva in dental procedures). Only in dental procedures is saliva considered a potentially infectious material. Also any unfixed tissue or organ is considered an OPIM. OSHA Bloodborne Pathogens Standard-a comprehensive rule that sets forth the specific requirements OSHA believes will prevent the transmission of bloodborne diseases to EMPLOYEES (not patients or employers). OSHA requirements: • Employers covered by BPS must make exposure determinations and develop an exposure control plan. Employers must use engineering and work practice controls to prevent employee exposure, and develop a system to evaluate exposure incidents. • BPS requires training of all employees who provide or assist in providing patient care and training to those who clean operatories, instruments, and gowns. • Through the OSHA Bloodborne Pathogen Standard, OSHA directs that uniform clothing worn in the dental office is laundered at the dental office or by an outside service, NOT at an employee’s home. Offices must use barrier techniques, communicate hazards to employees (training), perform proper cleaning of the office, and offer

Hepatitis B vaccinations. • Standard (formerly “Universal”) infection control precautions for bloodborne pathogens requires changing gloves BETWEEN ALL PATIENTS. Gloves and gowns are required when you reasonably anticipate skin contact with blood or other potentially infectious materials, including saliva. Thus, if you reasonably anticipate the forearms will be spattered with saliva or blood, then the forearms must be covered. • Any gown or clinic jacket that prevents blood or other potentially infectious materials (including saliva) from reaching work clothes, street clothes, or skin is considered adequate. Fluid-resistant gowns are not required unless it is anticipated that large amounts of blood, saliva, or other body fluids will soak through the gown to the employee’s clothing. OSHA considers cotton or cotton/polyester clinic jackets or lab coats as satisfactory barriers for most routine dental procedures. When surgical procedures are performed involving large quantities of blood (i.e. trauma surgery), additional personal protective equipment like long-sleeved gowns are required. According to OSHA, the selection is to be based on the quantity and type of exposure expected.

• When handling chemical agents or cleaning a dental office, always wear protective eyewear, mask, and heavy-duty utility or nitrile gloves. • Wearing gloves, protective eyewear, and a facemask is recommended WHEN TREATING ALL PATIENTS. Masks and protective eyewear are required when splashes, spray, splatter or droplets of blood or other potentially infectious materials (including saliva), may be generated and eyes, nose, or mouth contamination can be reasonably anticipated. A face shield may be substituted for masks and eyewear if you prefer. • Face mask should be changed BETWEEN PATIENTS (every patient) or more often if heavy spatter is generated during treatment. CDC recommends the mask to be changed between patients or during patient treatment if the mask becomes wet or moist from within or without. • The rule states that if eyewear is required, if must be goggles or glasses with solid (not perforated) side shields. • Center for Disease Control suggests a new mask for EACH PATIENT. Masks should have at least 9599% filtering efficiency for small particle aerosols 13μm.

Human Immunodeficiency Virus (HIV) the most infectious target of Standard (Universal) Blood Precautions. While HBV is the most infectious bloodborne pathogen, it is not the most infectious target of Standard Blood Precautions. Infection control precautions are aimed at preventing its viral transmission and are also effective in preventing HIV & HCV cross-infection. Hepatitis B Virus (HBV)-poses the greatest occupational healthcare worker risk for bloodborne infection. The basis of the current standard (formerly “universal”) infection control precautions were first recommended by the CDC in 1987 to protect healthcare workers from occupational transmission of all bloodborne infectious diseases during the provision of patient care. While many health professionals focused on HIV, evidence demonstrated that HBV was responsible for infection in 10-30% of exposed, susceptible healthcare workers. HBV concentrations in blood of a chronic carrier can range between 1-100 million virions/ml, in contrast to significantly lower viral loads shown for both HIV and AIDS infected patients. Precautions that minimize potential HBV infection/spread prevent crossinfection of less infectious microorganisms. Exposed employees who have declined the Hepatitis B vaccination can change their mind at any time and receive FREE vaccination. Exposed employees who have begun their

Hepatitis B vaccination series can work at their jobs even though the series is not complete. Dentists must provide “at-risk” employees with protection from HEPATITIS B. The federal standard for occupational exposure to bloodborne pathogens requires employers to provide the Hepatitis B vaccination. Employers must offer the vaccination (and boosters if these are recommended in the future) to all employees who have occupational exposure at no cost to the employees. The employee may refuse the vaccination, but OSHA will require proof that an employee has refused. Employers must offer the vaccination to a new employee within 10 working days of initial assignment to a position involving exposure. OSHA requires the employee to be trained regarding Hepatitis B and the vaccination prior to being offered vaccination. ♦ Hepatitis B virus infection commonly occurs by sexual intercourse, prenantal transfer, and percutaneous inoculation. Hepatitis C Virus (HCV)-transmitted primarily in infected blood via accidental needlesticks, blood transfusions, or drug addicts sharing contaminated syringes. Historically, parenteral drug abusers, people receiving

transfusions, organ recipients, and hemophiliacs receiving Factor VII or IX are at high risk for HCV infection. More recently, people receiving tattoos or undergoing body piercings have been infected via contaminated, unsterilized needles. • Viral concentrations detected in hepatitis C virus infected patients range between numbers for HBV and HIV. Thus, we target the most infectious bloodborne pathogen with our infection control standards. Occupational Exposure-defined by OSHA as any reasonably anticipated skin, mucosal, eye, or parenteral contact with blood or other potentially infectious fluids during the course of one’s duties while at work. OSHA includes saliva in dental procedures in the definition of “other potentially infectious materials” because saliva can be mixed with blood in some dental procedures. Thus, OSHA concluded that saliva should be treated as potentially infectious even though scientists believe bloodborne diseases are not transmitted via saliva. The following are included in procedures to evaluate an exposure incident: • State the policies that were in place at the office at the time of the incident. • State the engineering controls (i.e. needle recapping device, sharps container, rubber dam) and work practices that were in place at the office at the time of the incident.

• State the personal protective equipment (gloves, lab coats, etc.) that were in use at the office at the time of the incident. Exposure Incident-a specific occupational incident involving the eyes, mouth, other mucous membranes, nonintact skin, or parenteral contact with blood or other potentially infectious materials (including saliva). An injury from a contaminated sharp is the most common exposure incident. After a report of an exposure incident is made, the employer must make immediately available at no cost to the employee, a confidential medical evaluation and follow-up that includes: • Documentation of route(s) of exposure and circumstances in which the incident occurred. • Identification and documentation of the source individual (patient), unless the employer can establish that identification is not feasible or prohibited by state or local law. • Results of testing of the source individual’s blood, it the patient is available. • Collection and testing of the employee’s blood AFTER consent is obtained. • Medically indicated prophylaxis • Counseling • Evaluation of reported illnesses in the weeks after the incident. The employer MUST provide the following information

to the healthcare professional who performs the evaluation: • A copy of the OSHA standard. • Description of the employee’s duties as they relate to the incident. • Documentation of route(s) of exposure and circumstances under which exposure occurred. • Results of the source individual’s blood testing if available. • All medical records relevant to treatment of the employee (including vaccination status) which are the employer’s responsibility to maintain. Exposure Control Plan-the standard requires that every employer have a written exposure control plan designed to eliminate or minimize employee exposure to bloodborne diseases. The plan must set forth office policies and protocols to protect employees from these diseases. OSHA requires the plan to contain these elements: how and what schedule your office is implementing OSHA’s requirements for barrier techniques, hepatitis B vaccination, housekeeping, disinfection of contaminated work surfaces/equipment, handling regulated waste, post-exposure evaluation and follow-up, communication of bloodborne pathogen hazards to employees, and record keeping. • The exposure control plan should outline office

protocol for when gloves, gowns, masks, and eyewear (or faceshields) are used. When and how you will provide training for employees, your specific practices for disinfection, how you launder gowns (onsite or using a service), where to keep your records (training and medical records, and OSHA standard). • Office policy on the hepatitis B vaccination, and office protocol for handling regulated waste. Exposure Control Plan must be accessible to employees and must be updated at least annually and whenever necessary to reflect office changes (i.e. new procedures that affect occupational exposure and new positions). The plan must be provided to OSHA upon request. Employers must ensure that ALL employees with occupational exposure participate in a training program at no cost, during working hours, with material appropriate to the education, literacy, and language of the employee. Training is critical to OSHA compliance and must include: • A copy of the standard and explanation of its contents. • A general explanation of the epidemiology, symptoms, and modes of transmission of bloodborne diseases. • An explanation of the office’s exposure control plan, and how the employee can obtain a copy of it. • Information about the office’s protocol for gloves,

gowns, masks, and eyewear (or faceshields), including the type of equipment available, its location and when it is to be used, and how it is to be removed, handled, decontaminated, and disposed. • An explanation of how to recognize tasks that may involve occupational exposure and how to prevent or minimize such exposures (i.e. how to use a sharps container properly). • Information on the hepatitis B vaccine, including efficacy, safety, how it is administered, vaccination benefits, and that the vaccination is free of charge. • Information on how to handle emergencies involving occupational exposure (what actions to take and who to contact). • An explanation of the office protocol for handling exposure incidents like injuries from contaminated sharps, how to report the incident, follow-up medical care, and evaluation. • An explanation of the biohazard labels used in the office. • An opportunity for interactive questions and answers with the trainer. OSHA regulates CONTAMINATED SHARPS. Contaminated sharps are any contaminated object that can penetrate the skin, including, but not limited to needles, scalpels, broken glass, broken capillary tubes, and exposed ends of dental wires. There may be other objects used in the dental office that are sharps, and if they

become contaminated with blood or other potentially infectious materials (including saliva), then they must be regulated. Recapping Sharps: the rule generally prohibits bending, recapping, breaking, shearing, or removing sharps. However, recapping with a one-handed method or using a mechanical device is permitted if the employer can demonstrate that not alternative is feasible, or that such action is required by a specific medical procedure. The rule’s introduction lists the administration of an anesthetic as a medical procedure that may require recapping (i.e. reinjection of the same patient to enhance the local anesthetic). Anti-Retraction Valves-valves used on handpiece and air-water syringe hoses to prevent the retraction of fluid back into the tubing (they prevent patient fluids from getting into the water lines) and reduce the risk of crosscontamination other patients. Center for Disease Control & Prevention (CDC) recommends a minimum of 20-30sec of flushing water lines between patients, and several minutes of flushing water lines if the system has been idle for a period of time (i.e. over the weekend). Hand-pieces and reusable air-water syringe tips must be autoclaved between patients. Disposable

air-water syringe tips are available. Disposable saliva ejectors cannot be reused. FDA (Food & Drug Administration)-a branch of the U.S. Department of Health & Human Services that determines which drugs and medical services can be marketed in the U.S. The FDA requires both drug safety and efficacy. FDA is also responsible for regulating handpieces and recommending sterilization procedures to the CDC. Drug Enforcement Administrations (DEA)-a branch of the U.S. Department of Justice that determines the degree of control for substances with abuse potential (Schedules I-V). The most commonly used dental materials deemed HAZARDOUS by OSHA are mercury, nitrous oxide, & chemicals used to develop film. When working with or handling mercury, work in a well-ventilated space, avoid direct skin contact with the metal, and store scrap amalgam in tightly sealed containers covered with sulfide solution (an effective agent to suppress or eliminate the emission of mercury vapor from the scrap). When working with x-ray processing solutions, use protective eyewear & rubber gloves, work in a well-

ventilated area, and store solutions in tightly covered containers. The acceptable maximum exposure level allowed by OSHA for nitrous oxide is 1000ppm. Material Safety Data Sheets (MSDS)-documents that contain information concerning a hazardous chemical. Chemical manufacturers and importers are required to obtain a material safety data sheet for each hazardous chemical they produce or import. Distributors are responsible for ensuring their customers are provided with a copy of these MSDS. Employers must have an MSDS for each hazardous chemical they use, and may rely on the information received from their suppliers. The role of MSDS under the rule is to provide detailed information on each hazardous chemical (including its potential hazardous effects, physical and chemical characteristics, and recommendations for appropriate protective measures). This information is useful to employers responsible for designing protective programs, and to workers. If you are not familiar with the MSDS and chemical terminology, you may need to learn to use them yourself. A glossary of MSDS terms may be helpful in this regard. Most employers using hazardous chemicals will primarily be concerned with MSDS

information regarding hazardous effects and recommended protective measures. MSDSs must be readily accessible to employees when they are in their work areas during work shifts. You must decide what is appropriate for your particular office. Some employers keep MSDSs in a binder in a central location. As long as employees can access the information they need, any approach can be used. Employees must have access to the MSDS themselves. Infection control training records and medical records if the employee is involved in an occupational exposure MUST be maintained. The employer must maintain a medical record for each employee whose job involves occupational exposure to blood and other potentially infectious materials. The employee’s medical record must include: 1. Employee name and SSN. 2. Copy of the employee’s hepatitis B vaccination status (dates and medical records regarding the employee’s ability to receive the vaccination). 3. Medical opinions and evaluations. 4. Test results 5. Details about exposure incidents (routes of exposure and how the exposure occurred). Medical records MUST be maintained for the duration of

the employment plus 30 years, and must be kept strictly confidential. However, each employee is entitled to review their medical record. Also, the OSHA-required records must be transferred to the new owner. If you go out of business and there is no new owner, you must notify the Director of the National Institute for Occupational Safety and Health (NIOSH) at least 3 months before you intend to dispose of the records and offer to transmit the records to NIOSH. Environmental Protection Agency (EPA)-regulates waste transportation from the dental office. CDC considers the following groups as “highrisk”: IV drug users, homosexual/bisexual males, people receiving transfusions or blood products, and healthcare workers who may come into contact with body fluids. OSHA considers part-time, temporary, and probationary workers as employees. It is important that the employer stress to employees that Hepatitis B has been a long-standing occupational threat in dentistry. HBV is a hardy virus that only takes a low concentration to transmit the virus and infect someone. Dental healthcare workers are at greatest risk for contracting HEPATITIS. As a dentist (employer),

you must offer to provide vaccination to all employees who have occupational exposure. Any FDA-approved hepatitis vaccine is acceptable (Recombivax HB & Engerix-B). Hepatitis B virus is usually transmitted by percutaneous inoculation, sexual intercourse, or prenatal transfer. Treat each patient and instrument as potentially infectious (Standard Infection Control Precautions).

INFECTION CONTROL Opportunistic Infection-an infection caused by normally non-pathogenic microorganisms in a host whose resistance has been decreased or compromised. • The percentage of people living with a wide variety of immune compromised conditions continues to increase. Along with the clinical manifestations of those types of diseases, there can be accompanying deficiencies in aspects of hostimmune defenses. The severity of deficiency can range from mild, to life-threatening, and predispose the compromised person to infections by organisms that would not usually occur in other people with intact innate and specific immunity. Infection Control: • Exposure is not synonymous with infection. • Do not disinfect when you can sterilize. • Known AIDS patients can be treated using Standard Bloodborne Precautions. • It is not possible or necessary to sterilize all environmental surfaces that become contaminated during patient care. In many instances, because of the relatively low risk of microbial transmission, thorough cleaning of surfaces

is sufficient to break the cycles of cross-contamination and cross-infection. Thus, sterilization of all clinical instruments and inanimate environmental surfaces is NOT mandatory. Personal protective equipment clinic jackets are required to be LONG SLEEVE, HIGH NECK for optimal protection and to minimize the potential for exposed skin contact that can become contaminated with a patient’s blood, saliva, or other potentially infectious material. Bactericidal-an antibacterial solution that directly KILLS bacteria. Bactericidal agents are preferred to “bacteriostatic” chemicals because bacteriostatic do not directly kill or inactivate microbes, but merely inhibit their metabolism and replication. Thus, these organisms can remain viable, but inactive for extended intervals. Application of a bactericidal agent or process ensures microbial inactivation. Sanitization-a type of antimicrobial treatment (i.e. used for drinking water) to lower the total microbial load to safe public health levels. Used to treat water supplies to reduce microbial levels to safe public health levels. STERILIZATION-the process of KILLING (or removing) all microorganisms (including bacterial spores) on an object or in a material (i.e. liquid media). Sterilization is the complete destruction of all forms of microbial life

(including SPORES) without particular reference to microbial organisms. The limiting requirement is destruction of heat-resistant spores. Sterilization involving the use of HEAT is recommended for all instruments used in the mouth. Heat is the most efficient, reliable, and biologically monitorable sterilization method. During a routine cycle using an autoclave, unsaturated chemical vapor sterilizer, or dry-heat unit, CELL DEATH OCCURS because the heat inactivates critical enzymes and other proteins within microbial cells. DO NOT DISINFECT WHAT CAN BE STERILIZED. • Sterilization is the use of physical or chemical procedures to destroy ALL microbial life (including bacterial endospores). Sterilization refers to the absence of all living forms. • The recommendation stating that all reusable items that come into contact with a patient’s blood, saliva, or mucous membranes must be sterilized using heat is now routinely accepted and used in dental facilities. • PRE-CLEANING is the most important step in instrument sterilization. Debris acts as a barrier to the sterilant and sterilization process. Ultrasonic

instrument cleaning is the safest and most efficacious method of precleaning. • Heat sterilization is the most EFFICIENT way to kill microbes. Heat is the most efficient, reliable, and biologically monitorable sterilization method. During a routine autoclave cycle, unsaturated chemical vapor sterilizer or dry heat unit, cell death occurs via heat inactivation of critical enzymes and other proteins within microbial cells. • The recommendation that all reusable items that contact a patient’s blood, saliva, or mucous membranes must be sterilized using heat is now routinely accepted and used in dental facilities. • The immersion of dental instruments in cold disinfectants will not destroy spores or hepatitis viruses (they are resistant to physical and chemical agents). • Liquids are generally sterilized by filtration. The most commonly used filter is composed of nitrocellulose and has a pore size of 0.22μm. This size retains all bacteria and spores. Filters work by physically trapping particles larger than the pore size. Rapid Heat Transfer Sterilization-provides a very fast cycle time, no dulling of cutting edges, & dry

instruments after the cycle. FDA-approved, forced air, dry heat convection ovens are appropriate for sterilizing heat-stable instruments and other reusable items used in patient care. They use a higher temperature than other dry heat units, and there is controlled internal air flow within the chamber. In contrast to traditional dry heat sterilizers, a rapid heat transfer unit can sterilize items in much shorter times, while offering the advantages of dry heat. Rapid Heat Transfer Sterilization requires 375°F (191°C) for 12 minutes for wrapped instruments, and 6min cycle time for unwrapped instruments. Dry Heat Sterilization-proper time & temperature for dry heat sterilization is 320°F (160°C) for 2hrs, or 340°F (170°C) for 1hr. Items usually sterilized by dry heat can be autoclaved. They should be removed immediately after cycle to diminish the possibility of instrument corrosion and dulling of sharp points or edges (carbon steel instruments). Dry heat destroys microorganisms by causing COAGULATION OF PROTEINS. • Instruments MUST BE DRY before using dry heat sterilization and ethylene oxide sterilization, because water interferes with the sterilization process. • Dry Heat Advantages: effective/safe for sterilizing metal instruments, and does not dull or corrode instruments.

• Dry Heat Disadvantages: long cycle, poor penetration, and ruins heat-sensitive materials. Autoclave (Moist Heat Sterilization or “Saturated Steam”)-proper time and temperature for autoclaving is 250°F (121°C) for 15-20 minutes. These conditions yield 15lbs of pressure of steam per square inch. MOIST HEAT destroys bacteria by denaturation of high proteincontaining bacteria. Autoclave provides sterilization when used at the 250°F (121°C) for 15-20 min because it applies the heat under pressure, which greatly speeds up the protein denaturation process (compared to boiling water). Usually only 10 minutes is required to destroy all bacteria, but the increased time is allowed for penetration when instruments are wrapped in thick towels. Autoclaving effectiveness is best determined by culturing bacterial spores. Spore testing of autoclave units is recommended weekly. • Spore-forming pathogens provide the ultimate test for efficacy of sterilization. Since bacterial spores are resistant to boiling (100°C at sea level), they must be exposed to a higher temperature. This temperature cannot be achieved unless the pressure is increased. For this purpose, an autoclave chamber is used where steam at a pressure of 15lbs/in reaches a temperature of 121°C for 1520min. This kills even the highly heat-resistant spores of Clostridium botulinum (causes botulism) with a

margin of safety. Saturated steam (autoclave) is the most practical, economical, most currently effective sporicide, and the most efficient method for destroying viral and fungal microorganisms. Steam autoclaves operate at either 121°C (250°F) at a pressure of 15lbs/square inch (psi) for 15-20 minutes, or at 134°C (270°F) at a pressure of 30psi for a minimum of 3 minutes (“flash cycle”). To effectively kill all living organisms, the minimum required temperature is 121°C (250°F). • Autoclaving time varies directly with the type of load placed into the chamber. The 3-minute “flash cycle” is best indicated for unwrapped instruments. For wrapped instruments, a longer sterilization cycle is required to permit adequate penetration of steam for proper disinfection. Unsaturated Chemical Vapor Sterilization (UCVS)-the proper time and temperature is 270°F (132°C) for 20-40 minutes. These conditions yield 20lbs of sterilizing vapor pressure. The main operation of CVS is similar to steam sterilizers, but rather than using distilled water, UCVS uses a solution of alcohol, formaldehyde, ketone, acetone, and water to produce the sterilizing vapor.

• The temperature and pressure required for chemical vapor sterilizers is greater than for autoclave. • MAIN advantage of this system is it does not rust or corrode metal instruments and carbon steel. • Disadvantages: instruments must be dried completely before processing, a special chemical solution must be used, and it destroys heat-sensitive plastics. Glutaraldhyde (2%)-an alkalizing agent highly lethal to essentially all microorganisms. It takes 10hrs to kill bacterial spores when a dental instrument is placed in a 2% glutaraldehyde solution. If the 10hr sufficient contact time is provided, there is absence of extraneous organic material. • Glutaraldehyde Advantages: most potent category of chemical germicide, capable of killing spores (after 10hrs), EPA registered as a chemical sterilant, and can be used on heat-sensitive materials. • Glutaraldehyde Disadvantages: long period required for sterilization, allergenic, not an environmental disinfectant, and extremely toxic to tissues. • Used in hospitals to sterilize respiratory therapy equipment.

Ethylene Oxide Gas Sterilization-used extensively in hospitals to sterilize heat-sensitive materials like surgical instruments and plastics. It kills by alkylating proteins and nucleic acids (it is an alkylating agent that irreversibly inactivates cellular nucleic acids (DNA) and proteins). It is a slow process (10-16hrs) depending on the material to be sterilized. This gas is fairly toxic to humans and is flammable, so its use is limited. • Ethylene oxide gas has been widely used as a sterilization agent, especially for pre-packaged, disposable plastic ware in hospitals. Exposure of materials to ethylene gas must be performed in special sealed chambers, and items must be cleaned and dried thoroughly before the sterilization process. • Advantages: highly penetrative, does not damage heat-sensitive materials (rubber, cotton, plastics), evaporates without leaving a residue, and works well for materials that cannot be exposed to moisture. ANTISEPTICS- chemically safe to be administered to external body surfaces or mucous membranes to decrease microbial numbers. Antiseptics cannot be taken internally. Antiseptics are chemical agents similar to disinfectants, but can be applied onto living tissues (i.e. hand-washing) to remove accumulated transient microorganisms, and temporarily lower the concentration of normal, resident flora. Soap only removes microorganisms.

• “Antiseptic” best relates to a handwash agent. Antiseptics are used for antimicrobial agents applied onto living tissues. • Liquid antimicrobial preparations (antiseptics) for handwashing: chlorhexidine gluconate, parachlorametaxylenol, idophors, and triclosan. • Alcohol-the most widely used antiseptic, and is used to reduce the number of microorganisms on the skin surface in a wounded area. Alcohol denatures proteins, extracts membrane lipids, and acts as a dehydrating agent, all of which contribute to its effectiveness as an antiseptic. Even some viruses (lipophilic viruses only) are inactivated by alcohol. • Isopropyl Alcohol-also used for hand hygiene procedures, but products containing 60-80% alcohol DO NOT use water, thus is a waterless hand hygiene agent. Isopropyl alcohol (90-95%) is the major form used in hospitals. • Phenol-the original disinfectant used in hospitals, but is rarely used today because it is too caustic. • Advantages of using alcohols (70% isopropyl & 70% ethyl alcohol) as surface disinfectants: alcohols are bactericidal, tuberculocidal, and are economical. Alcohols are NOT sporicidal.

• Alcohol Disadvantages: alcohol evaporates too quickly and has diminished activity against viruses in dried blood, saliva, and other secretions on surfaces due to the presence of tissue proteins and glycoproteins that render alcohol ineffective. Thus, alcohols are not effective surface cleansing agents (i.e. cleaning a dental operatory after treatment). • Ethanol (70%)-widely used to clean skin prior to immunization or venipuncture. • Iodine-the most effective skin antiseptic used in medical practice that acts as an oxidizing agent, and irreversibly combines with proteins. Disinfection-process of reducing the numbers or inhibiting growth of microorganisms (especially pathogens) to the point that they do not pose a threat of disease. Disinfection uses chemical agents to destroy pathogenic microorganisms, but not necessarily all pathogens or resistant spores on INANIMATE surfaces. Disinfection is less lethal than sterilization, and involves a chemical process of microbial inactivation that destroys virtually all pathogenic microorganisms on inanimate surfaces, but not necessarily all microbial forms (spores). Disinfection is the killing of many, but not all, microorganisms. It does not include the destruction of bacterial spores.

DISINFECTANTS-antimicrobial chemical agents (solutions, sprays, or wipes) used to destroy/kill microorganisms on when applied to inanimate objects/surfaces like countertops, lights, bracket trays, headrests, light handles, dishes, tables, floors. Disinfectants are not safe to use on living tissues (however antiseptics are safe). Thus, are only applied to inanimate objects. Disinfectants are germicidal (kill) and prevent pathogenic microorganism growth (microbiostatic) • Alcohols, Chlorhexidine, & Quaternary Ammonium Compounds are disinfectants. • Water-based disinfectants are better than alcoholbased disinfectants. • Pump spray disinfectants are better than aerosol spray disinfectants. • Quaternary Ammonium Compounds-cationic detergents used as disinfectants & antiseptics. Gram (+) bacteria are most susceptible to destruction. These compounds are NOT sporicidal, tuberculocidal, or viricidal, and are inactivated by anionic detergents (soaps and iron found in hard water). Ex: Benzalkonium Chloride. • Cationic Detergents-quaternary ammonium compounds widely used for SKIN ANTISEPTICS.

Detergents are “surface-active” agents composed of a long chain, lipid-soluble, hydrophobic portion, and a polar hydrophilic group that can be a cation, anion, or non-ionic group. These surfactants interact with the lipid in the cell membrane through their hydrophobic chain, and with the surrounding water through their polar group and thus, disrupt the cell membrane. • Anionic Surface-Acting Substances/Detergentsare synthetic anionic detergents and soaps that alter the nature of interfaces to lower surface tension and increase cleaning. Their primary value is their ability to remove microorganisms mechanically from the skin surface. *Non-ionic chemicals DO NOT have anti-microbial properties. • Cleaning surfaces PRIOR to disinfection in clinical settings is required to REDUCE the concentration of pathogens. The simplest way to approach environmental surface disinfection is to adhere to the aseptic technique of CLEANING IT FIRST. All disinfectant products include specific label instructions for cleaning PRIOR TO DISINFECTION. Cleaning physically removes debris, reduces the number of microorganisms present, and removes blood, tissue bioburden, and other debris that can interfere with disinfection. • Mycobacterium Tuberculosis is the marker

microorganism for intermediate surface disinfection. Antimicrobial activity against MT is recognized as a significant benchmark criterion for disinfectant effectiveness. While tuberculosis is not transmitted via inanimate environmental surfaces, its morphology and structure make tubercle bacilli relatively resistant to penetration by many lowlevel disinfectant chemicals. • Intermediate level agents like phenols, iodophors, hypochlorite, and certain preparations containing alcohols and other cleaner disinfectant chemicals can penetrate the wax and lipid outer layers surrounding mycobacteria. Chlorhexidine Gluconate & Triclosan-handwash agents with broad antimicrobial effect. They also have “substantivity” or a residual action on washed tissues for extended periods of time. Chlorine-a powerful oxidizing agent that inactivates bacteria and most viruses by oxidizing free sulfhydryl groups. Chlorine is the active component of hypochlorite (bleach) and is used as a disinfectant. Pasteurization-the treatment of dairy foods (milk) for short intervals using HEAT to kill certain disease-causing microorganisms. The target of pasteurization is to destroy MYCOBACTERIUM TUBERCULOSIS.

Concentration & contact time are critical factors that determine the effectiveness of an antimicrobial agent against a particular microorganism. Any or all three of the major portions of microbial cells can be affected: cell membrane, cytoplasmic contents (especially enzymes), and nuclear material (DNA, RNA). Studies show an increased risk of developing an allergic reaction to latex protein (Type 1) or certain chemicals used in the latex manufacturing process (Type IV) in certain groups of people. Current information has NOT shown a cross-reaction between pollen allergies and watersoluble latex allergens. Individuals who are predisposed to readily developing Type I hypersensitivity reactions (i.e. people who are atopic), can become sensitized to latex allergens more readily than people with few or no allergies. • Latex Allergy Risk Factors: rubber industry workers, atopy, persons with multiple surgeries and banana allergies, and spina bifida. Antigens most responsible for an immediate Type I allergic reaction to natural rubber latex are PROTEINS. Only a few of the 250 proteins are found in sap of the rubber tree. Hevea brasiliensis-water-soluble macromolecules that can leach out of latex gloves when a person

perspires, or may be detected on surfaces of other products containing natural rubber latex (NRL). These proteins cause Type I, immediate, IgEmediated reactions to natural rubber latex. If a patient develops a Type I, immediate allergic reaction to latex during dental treatment, the next time you treat them wear VINYL or NITRILE gloves. • A wide variety of latex-alternative infection control items appeared in the market within the last 10 years. The most widely recognized are newer generations of vinyl or nitrile gloves that do not cross-react with latex allergens. Products designated “hypoallergenic” are no longer labeled latex alternatives since they contain latex with a chemical coating over the latex. • Recent studies show not all latex-allergenic people were able to use hypoallergenic gloves, since many Type I allergic individuals still developed allergic manifestations when using these gloves. Irritation Dermatitis-the most common form of an adverse epithelial (skin) reaction noted for healthcare professionals. Many published reports have cited data suggesting 20-30% of healthcare workers suffer occasional or chronic dermatitis on their hands.

The most common manifestation is irritation dermatitis (a non-specific immune reaction often caused by contact with a substance that physically or chemically damages the skin). The condition is aggravated by frequent handwashing, residual glove powder left on hands, and the harshness of repeated use of some antiseptic handwash agents. Healthcare workers who live in colder climates may also experience chapping during the winter months. Americans with Disabilities Act: both state and federal statutes define disability as having a physical or mental impairment that substantially limits one or more major life activities of the individual, a record of such impairment exists, and the patient is regarded as having such impairment. • Dentists cannot deny anyone care due to a disability and cannot dismiss employees due to a disability. • Dental offices must undergo structural changes to allow access for the disabled. • HIV patients are protected under the Americans with Disabilities Act.

DENTIN 1,001+ “HIGH-SPEED DRILL” NBDE II PREVIEW 1. Oral manifestations associated with TRISOMY 21 (DOWN SYNDROME) include bull-shaped molars (TAURODONTISM), missing teeth (HYPODONTIA), increased gingivitis and periodontal disease, and DECREASED CARIES. 2. Patient with DOWN SYNDROME (TRISOMY 21) would most likely present with the CLASS III malocclusion (protruded mandible). 3. The MOST common site for oral cancer is the LATERAL BORDER OF THE TONGUE. 4. The most likely cause of an OPPORTUNISTIC infection presenting as a WHITE CREAMY LESION on the BUCCAL MUCOSA or TONGUE that WIPES OFF is CANDIDA. 5. Stages of PERIAPICAL CEMENTAL DYSPLASIA when viewed radiographically in a middle-aged African American female are: RADIOLUCENT (Stage 1), MIXED (Stage 2), & RADIOPAQUE (Stage 3).

6. Inflammation and fissuring at the corners of the mouth (labial commissures) caused by CANDIDA ALBICANS, RIBOFLAVIN (B2) DEFICIENCY, or ILL-FITTING DENTURE (DECREASED VERTICAL DIMENSION) is called ANGULAR CHEILITIS (PERLECHE). 7. A MALIGNANT tumor of BONE-FORMING tissue is an OSTEOSARCOMA. 8. A BENIGN radiopaque BONY OVERGROWTH or protuberance of pre-existing bone found at the MIDLINE of the HARD PALATE and/or mandibular lingual surfaces is an EXOSTOSIS (TORI = OSTEOMA). 9. The panorex of a 30 year-old female reveals a painless, unilocular radiolucency with well-defined margins around the CROWN of an impacted permanent mandibular 3rd molar. This is most likely a DENTIGEROUS CYST (FOLLICULAR CYST). 10. During a routine extra-oral examination, a 10-year old child presents with a painless swelling at the anterior midline of the neck just below the hyoid bone. Your knowledge of pathology tells you this is the MOST COMMON CONGENITAL, BENIGN NECK CYST that requires a referral to an ENT physician for further evaluation. This lesion is a THYROGLOSSAL DUCT/TRACT CYST.

11. The most common primary lesion of carcinoma metastasizing to the MANDIBLE in females is the BREAST. 12. The MOST COMMON head and neck malformation in humans is CLEFT LIP & PALATE. 13. A 62-year old man who complains that “my dentures don’t fit” or “my hat is too tight” with radiographs that show COTTON WOOL RADIOPACITIES and ROOT HYPERCEMENTOSIS, most likely has PAGET’S DISEASE. 14. High basal metabolism associated with HYPERTHYROIDISM is due to EXCESSIVE THYROXIN (T4) PRODUCTION. 15. PARATHYROID ADEMOMA is the most common cause of HYPERPARATHYROIDISM. 16. A 62-year old male with no allergies presents for root planing. He had bilateral hip replacement surgery nearly two years ago without complication, has mitral valve prolapse with regurgitation, and has a cardiac pacemaker. The appropriate antibiotic prophylaxis is PRE-MEDICATION IS NOT REQUIRED. 17. A rare, but dangerous adverse effect of taking CLINDAMYCIN is PSEUDOMEMBRANOUS COLITIS (overgrowth of Clostridium difficile that produces a toxin

causing colitis). 18. Small, irregular red dots with bluish-white centers on the BUCCAL MUCOSA and SOFT PALATE in a patient diagnosed with the RNA MEASELS (RUBEOLA) VIRUS are called KOPLIK SPOTS. 19. NITROUS OXIDE (N2O) works by ELEVATING THE PAIN THRESHOLD. 20. NITROUS OXIDE is contraindicated in the 1st Trimester of Pregnancy, COPD, cystic fibrosis, sickle-cell anemia, upper respiratory infections, and patients with EMOTIONAL or BEHAVIORAL INSTABILITY. 21. ACETAMINOPHEN (TYLENOL) is an ANALGESIC AND ANTIPYRETIC, but is NOT an antiinflammatory (NSAID) and DOES NOT AFFECT PLATELET AGGREGATION (CLOTTING). 22. For a patient on anti-coagulants (Coumadin/Warfarin) or with Peptic Ulcer Disease, who has a headache and fever, the drug of choice is ACETAMINOPHEN (TYLENOL), NOT IBUPROFEN or ASPIRIN. 23. GINGIVECTOMY contraindications are HIGH CARIES INDEX, INFRA-BONY ABSCESS, & NARROW ZONE OF ATTACHED GINGIVA.

24. Operculum inflammation around a partially impacted MANDIBULAR 3rd MOLAR crown due to plaque accumulation and resulting in pain, erythema, edema, and foul taste is PERICORNITIS. 25. When evaluating an OSSEOUS DEFECT, the ONLY WAY TO DETERMINE the number of walls left surrounding the tooth is by periodontal EXPLORATORY SURGERY. 26. The MOST COMMON method of DISPERSION in oral hygiene research that represents the RANGE OF DATA is the STANDARD DEVIATION. 27. The greater the RANGE (STANDARD DEVIATION) the WIDER the distribution curve. 28. Conducting a TRIAL RUN of the planned study on a small sample of the population is the purpose of a PILOT STUDY. 29. Experimental treatment is withheld from CONTROL GROUPS that receive the PLACEBO. 30. The PRIMARY goal in any oral health preventive program is PLAQUE CONTROL. 31. The only index that is REVERSIBLE & IRREVERSIBLE because it measures both gingivitis and periodontitis is the PERIODONTAL DISEASE

INDEX (PDI). 32. The BEST dental index to detect EARLY signs of GINGIVITIS is the SULCULAR BLEEDING INDEX (SBI). 33. The only condition contraindicated with WATER FLUORIDATION is a patient with KIDNEY FAILURE on RENAL DIALYSIS since it requires the use of mineral-free water. 34. ENVIRONMENTAL PROTECTION AGENCY (EPA) monitors fluoride concentration in COMMUNITY DRINKING WATER, while the FDA establishes fluoride limits in BOTTLED WATER. 35. Pit and fissure SEALANTS are generally comprised of BIS-GMA RESIN. 36. The purpose of pouring casts of alginate impressions within 10 minutes of taking the impressions and wrapping the impressions in MOIST paper towels (rather than soaking them in a water bowl for 30 minutes), is to prevent distortion by expansion, or IMBIBITION. 37. The first step when placing a dental sealant is to ISOLATE TO OBTAIN A DRY FIELD. 38. The first-line oral hypoglycemic of choice to treat

DIABETES MELLITUS TYPE 2 (Non-Insulin Dependent) when exercise and diet have failed is METFORMIN (GLUCOPHAGE). 39. The most common cause of HYPOTHYROIDISM in the U.S., affecting more women then men is the autoimmune disorder HASHIMOTO’S DISEASE (THYROIDITIS). 40. A rare, but life-threatening sequela from suddenly discontinuing SYNTHROID (Levothyroxine) for HYPOTHYROIDISM is MYXEDEMA COMA.

YOU ARE PREPARED FOR SUCCESS WITH DENTIN™ COMING SOON! TEST YOUR KNOWLEDGE USING DENTIN’s “1001+ HIGH SPEED DRILLS FOR THE NBDE II” www.dentin.co